Está en la página 1de 274

Y

M
E
D
9
59
CA
95
50
98
A
:8
E
h
.P
IC
AD
AB
O
ER
CH
YD
,H
T
AM
H

v
M
AM
IG

ㄬu\섲H±…Õ Jxß JO‰§° H›=~Ÿ K͋¬¶ë =òMì¼O‰§°


KH

v MODELWISE
R

=¶ki ‹¬=°‹¬¼°

(„¬sH›Æ…Õ Z䛽ø=Qê J_È°Q®°`Ç°#ß "³¶_ȅ˜ž)

v ㄬu Concept 䛽 ‹¬O|OkOz ㇐H©“‹¹ >ÿ‹¬°“°

v Jxß ‹¬=°‹¬¼ä›½ ‘~Ÿ“H›\˜ž`Ë ª^Î#°.

v Q®`Ç „¬sH›Æ…Õ J_y# ㄬ‰×߁°


"³~¡Ä…˜ s[xOQ· 3-145

1. HË_OQ· _™HË_OQ· (Coding-Decoding) 3-27


2. ã‰õ}°° (Series) 28-52
3. a#ß „¬sH›Æ (Oddmanout) 53-61
4. JH›Æ~¡=¶ „¬sH›Æ (Alphabet Test) 62-68
5. ‡éeH› / ª^Î$‰×¼ „¬sH›Æ (Analogy) 69-89

Y
6. ~¡H›ë ‹¬O|O^¥° (Blood Relations) 90-97

M
7. k‰§`ÇàH› „¬sH›Æ (Direction Sense Test) 98-107
#O|~Ÿ, ~Œ¼OH÷OQ· Ê >ÿØ"£° ‹ÔïHÞ<£ž >ÿ‹¹“ (Number, Ranking & Time Sequence Test) 108-114

E
8.
9. ‹Ô\÷OQ· / À„Á‹²OQ· Jö~*˜"³°O\˜ž (Seating / Placing Arrangements) 115-120

D
9
59
10. „¬l…˜ >ÿ‹¹“ (Puzzle Test) 121-124
CA
95
11. …ìlH›…˜ "³<£ zã`Œ° (Logical Venn Diagrams) 125-128

50
12. =¶¼^ŠÎ"³°\÷H›…˜ ̋<£ž >ÿ‹¹“ (Mathematical Sense Test) 129-131
98
A
13. q°‹²žOQ· Hê¼ï~H›“~Ÿ (Missing Character) 132-145
:8
E
h

…ìlH›…˜ s[xOQ· 146-194


.P
IC

ㄬH›@#° - f~Œà<Œ° (Statements - Conclusions) 146-154


AD

1.
2. ㄬH›@#° - T‚¬ì#° (Statements - Assumptions) 155-164
AB
O

3. ㄬH›@#° =°i†Çò "Œ^Î#° (Statements and Arguments) 165-171


ER
CH

4. xtó`ÇO - Hê~¡}O (Assertion - Reason) 172-177


YD

5. _¨\ì ‹¬„¦²+²†Ç°hž (^Î`ŒëO‰× †³¶Q®¼`Ç) (Data Sufficiency) 178-183


,H

6. [_û"³°O\˜ (Situation Reaction Test) 184-187


T
AM

7. ãu‡^Î `Ç~¡ø"Œ^Î=ò (Syllogism) 188-194


H
M
AM
IG

<Œ<£-"³~¡Ä…˜ s[xOQ· 195-272


KH

1. ã‰õ}°° (Series) 195-200


R

2. J<Œr (Analogy) 201-209


3. a#ß"³°Ø# ^¥xx Q®°iëOKÇ°@ (Oddmanout) 210-215
4. J‹¬O„¬î~¡â zã`Œ#° „¬îiOKÇ°@ (Completion of Incomplete Figures) 216-224
5. ^¥y†Çò#ß zã`Œxß H›#°Qù#°@ (Finding out Hidden/ Embedded Figures) 225-231
6. J^ÎíO…Õ ㄬuaO|O (Mirror Image) 232-238
7. h\÷…Õ ㄬuaO|O (Water Image) 239-243
8. À„„¬~Ÿ ‡¦éeÛOQ· Ê H›\÷“OQ· (Paper folding & Cutting) 244-248
9. XöH ^Î~Œà° H›e¾# „¬\ì#° Q®°iëOKÇ°@ (Finding out Identical figures) 249-253
10. „¬@ =¶ãuH› (Figure Matrix) 254-260
11. n~¡É„¦¬°#O, ‹¬=°„¦¬°#O =°i†Çò ‡zH›° (Cuboid, Cube & Dice) 261-272
1 HË_OQ· _™HË_OQ·
=òMì¼O‰§°...
(Coding-Decoding)

G HË_£ : HË_£ J#Qê ~¡‚¬ì‹¬¼ ‹¬OöH`ÇO.


G HË_OQ· : ~¡‚¬ì‹¬¼ q+¬†Ç¶#° ‹¬O|Ok`Ç =¼ä›½ëä›½ `DŽ¬æ q°ye# "ŒiH÷ `³e†Ç°ä›½O_¨ LO_ÍO^ΰ䛽 P†Ç¶ q+¬†Ç¶#°

Y
‹¬OöH`Ç ~¡¶„¬O…ÕH÷ =¶ió „¬O„²ªë~¡°. nx<Í 'HË_OQ·Ñ JO\ì~¡°.
G _™HË_OQ· :

M
‹¬OöH`Ç ~¡¶„¬O…ÕH÷ (HË_OQ·) =¶~¡ó|_# q+¬†Ç°O ‹¬O|Ok`Ç =¼H÷ëx KÍ~¡Qê<Í =¼H÷ë P q+¬†Ç¶xß ª^¥~¡}
~¡¶„¬O…ÕH÷ =¶ió, ª~ŒO‰§xß J~¡ÖO K͋¬°ä›½O\ì_È°. nx<Í '_™HË_OQ·Ñ JO\ì~¡°.
G

E
HË_OQ·-_™HË_OQ· ㄬ‰×߁ ^¥Þ~Œ Jƒ’¼iÖ †³òH›ø J#°"Œ^Î *ìý<Œxß, =¶#‹²H› P…ÕKÇ<Œ ª=°~¡Ö¼O#° =°i†Çò T‚¬ð‰×H÷ëx

D
9
JOKÇ<Œ "ͪë~¡°.
G

59
D qƒìQ®O…Õ Hùxß JH›Æ~Œ ‹¬=ü‚¬ð#° …è^¥ „¬^¥#° "Œ\÷ ‹¬OöH`Ç (~¡‚¬ì‹¬¼) ~¡¶„¬O…Õ Wªë~¡°. g\÷ P^¥~¡OQê "Í~ùH›

CA
95
„¬^ÎO …è^¥ JH›Æ~¡ ‹¬=ü‚¬ìO †³òH›ø ‹¬OöH`Ç ~¡¶„¬O Qêh …è^¥ "Í~ùH› ‹¬OöH`Ç ~¡¶„¬O †³òH›ø J‹¬° ~¡¶‡xß Qêh H›#°Qù#=‹²

50
=ôO@°Ok.
G 98
A
HË_OQ· q^¥#O…Õ XH› „¬^ÎO…Õx JH›Æ~Œä›½ …è^¥ XH› ‹¬OY¼…Õx JOïHä›½ |^ΰ°Qê "Í~ùH› JH›Æ~Œ#° …è^¥ JOïH#° XH›
:8
„¬^Μu ㄬHê~¡O "Œ_ ~¡‚¬ì‹¬¼ ‹¬OöHuH› ƒì+¬#° `džǶ~¡°K͋² P „¬^ÎO…Õx ª~ŒO‰×=ò#° =ü_Ë =¼H÷ëH÷ `³e†Ç°ä›½O_¨ W^Îí~¡°
E
h

=¼ä›½ë° XH›i #°O_ =°~ùH›iH÷ „¬O„¬ô䛽O\ì~¡°.


.P

G
IC

"Íö~Þ~¡° ‹¬=°‹¬¼ä›½ "Íö~Þ~¡° HË_OQ· - _™HË_OQ· „¬^Μ`Ç°°<Œß~ò. XHùøH›ø „¬^Μu XHùøH›ø q^¥#O#° J#°‹¬i‹¬°ëOk. XH›
AD

‹¬=°‹¬¼…Õx ‹¬OöH`Ç q^¥#O#° "Í~ùH› ‹¬=°‹¬¼ä›½ J#Þ~òOKÇ_¨xH÷ g°…è^ΰ.


G
AB
O

POQ®Á JH›Æ~Œ°, „¬^¥°, JOïH°, ‹¬OY¼ P^¥~¡OQê HË_OQ· „¬^Μ`Ç°° =ôO\ì~ò.


G
ER
CH

XH› POQ®Á„¬^ÎO…Õx JH›Æ~Œ#° [OaÁOQ· K͆ǰ_ÈO ^¥Þ~ŒQêh, XH› "ŒH›¼O…Õx „¬^¥#° JOïH ^¥Þ~Œ Qêh, XH› ‹¬OY¼…Õx
JOïH#° POQ®Á JH›Æ~Œ ^¥Þ~Œ Qêh, XH› "ŒH›¼O…Õx „¬^¥#° "Í~ùH› „¬^¥ ^¥Þ~ŒQêh Wzó# ‹¬OöH`Œ P^¥~¡OQê HË_OQ·
YD

…è^¥ _™HË_OQ· K͆ǰ=‹² =ôO@°Ok.


,H

G D qƒìQ®O…Õx ㄬ‰×߁#° ‹¬°°=ôQê ªkOKÇ°@䛽 Jƒ’¼iÖH÷ P…ì权ÿ\ ÷H›…˜ P~¡Û~Ÿ g°^Î, i=~Ÿž P~¡Û~Ÿ g°^Î =°i†Çò "Œ\÷ ªÖ#
T
AM

‹¬OY¼ g°^Î „¬îië J=Qꂬì# =ôO_¨e.


H
M

; ‹¬=¼k‰×…Õ POQ®Á =~¡â=¶ #O|iOQ· (A to Z)


AM
IG

Z_È=° 1 2 3 4 5 6 7 8 9 10 11 12 13 14 15 16 17 18 19 20 21 22 23 24 25 26 䛽_
KH

#°O_ A B C D E F G H I J K L M N O P Q R S T U V W X Y Z #°O_
R

® 26 25 24 23 22 21 20 19 18 17 16 15 14 13 12 11 10 9 8 7 6 5 4 3 2 1 ¬
; J„¬‹¬=¼k‰×…Õ POQ®Á =~¡â=¶ #O|iOQ· (Z to A)

Z_È=° 1 2 3 4 5 6 7 8 9 10 11 12 13 14 15 16 17 18 19 20 21 22 23 24 25 26 䛽_

#°O_ Z Y X W V U T S R Q P O N M L K J I H G F E D C B A #°O_

ÿÿ® 26 25 24 23 22 21 20 19 18 17 16 15 14 13 12 11 10 9 8 7 6 5 4 3 2 1 ¬
; =¼uö~H› JH›Æ~¡=ò#° `³°‹¬°Hù#°@䛽 "Shortcut Trick".
" ; # : $ 9 % 8 & 7 ' 6 ( 5
"B;
#P:
$P9
%F8
7BMV&
'6FM
(SFB5

)
)PS4F

4 * *O3
3 +
+B2

2 ,
,FF1

1 -
-0X

0 . .B/
/
REASONING ABILITY (VERBAL) ○ ○ ○ ○ ○ ○ ○ ○ ○ ○
3
○ ○ ○ ○ ○ ○ ○ ○ ○ ○ ○ ○ ○ ○ ○ ○ ○ ○
Study Material
H ÿ XH› JH›~Æ O
¡ †³òH›ø ªÖ#O Z_È=° #°O_ 䛽_H÷ `³e‹²#„¬ô_È°, ( 3 0 6 / %
䛽_ #°O_ Z_È=°# ^¥x ªÖ#O H›#°Qù#°@ :      

+ XH› JH›Æ~¡O †³òH›ø ªÖ#O Z_È=° #°O_ 䛽_H÷ 'm' J~ò# ) 4 1 7 0 &
䛽_ #°O_ Z_È=°# ^¥x ªÖ#O « 27 – m. \ GROUND †³òH›ø HË_£ HSPVOE.
L^¥ : J †³òH›ø ªÖ#O 䛽_ #°O_ Z_È=°ä›½ ZO`Ç? 2. CODING #° DPEJOH Qê ~Œ‹²#@Á~ò# ALPHA-
ª^¥#: J †³òH›ø ªÖ#O Z_È=° #°O_ 䛽_H÷ 10. #° Uq^ÎOQê ~Œªë~¡°?
\
BET
J †³òH›ø ªÖ#O 䛽_ #°O_ Z_È=°ä›½
ª^Î#: $ 0 % * / (
« 27-10 « 17.

+ HË_OQ· - _™HË_OQ· „¬^Μ`Ç° ~¡Hꁰ :


1) JH›Æ~¡ HË_OQ· (Letter coding)

%

1

&

+

0

)
CODING…Õx ㄬu JH›Æ~¡O `Ç~¡°"Œ`Ç JH›Æ~¡OQê HË_£
2) #O|~Ÿ / ‹²O|…˜ HË_OQ· (Number / Symbol
K͆ǰ|_#k. J^Í x†Ç°=°O P^¥~¡OQê ALPHABET#°
coding)

Y
HË_£ K͆ǰQê.
3) ㄬuöHƄ¬} HË_OQ· (Substitution coding)
" - 1 ) " # & 5

M
4) ‹¬OY¼ =°i†Çò Q®°~¡°
ë #°O_ ‹¬=¶KŒ~¡O H›#°Qù#°@
       
(Deciphering Number & Symbol Codes for
# . 2 * # $ ' 6

E
messages)
\

D
‹¬=¶KŒ~¡ „¬^Î=ò HË_£x H›#°Qù#°@ (Deci- ALPHABET †³òH›ø HË_£ BMQIBCFU.

9
5)

59
phering message) (Word coding) 3. XH› ‹¬OöH`Ç ƒì+¬…Õ ICET #° HBDS Qê ~Œ‹²#@Á~ò#
CA
95
6) =¶ãuH› HË_OQ· (Matrix coding) J^Í ‹¬OöH`Ç ƒì+¬…Õ EAMCET#° U q^ÎOQê ~Œªë~¡°?

50
7) q‰õÁ+¬} HË_OQ· (Analysis coding)
ª^Î#: * $ & 5

98
A
TYPE - I : …ÿ@~Ÿ HË_OQ·
   
:8
) # % 4

+
E
h

…ÿ@~Ÿ HË_OQ· q^¥#O…Õ POQ®Á JH›Æ~Œ#°, "Í~ùH› POQ®Á ICET …Õx ㄬu JH›Æ~¡O, ^¥x =òO^ΰ JH›Æ~¡OQê HË_£
.P
IC

JH›Æ~Œ`Ë<Í HË_£ Kͪë~¡°. J…ì HË_£ K͋²# „¬^¥° U K͆ǰ|_#k. J^Í x†Ç°=°O P^¥~¡OQê EAMCET#°
AD

x†Ç ° =°O P^ ¥ ~¡ O Qê HË_£ KÍ † Ç ° |_¨Û † ³ ¶ Q® = °xOz HË_£ K͆ǰQê


AB
O

J^Íq^O
Î Qê =°~ùH› „¬^¥xH÷ HË_£#° H›#°Qù<Œe …è^¥ _™HË_£ & " . $ & 5
ER
CH

     
K͆ǰ|_# „¬^¥xH÷ HË_£#° H›#°Qù<Œe.

+ % ; - # % 4
YD

…ÿ@~Ÿ HË_OQ· „¬^Μu…Õx ~¡Hꁰ :


1) ‹¬=¶# `Í_¨ Q® „¬sH›Æ. \
EAMCET †³òH›ø HË_£ DZLBDS.
,H

4. XH› ‹¬OHö `Çƒì +¬…Õ KALAM #° NDODP Qê ~Œ‹²#@Á~ò#


T

2) P~˂¬ì}ãH›=° `Í_¨ Q® „¬sH›Æ.


AM

J^Í ‹¬OHö `Çƒì +¬…Õ MODI x U q^O


Î Qê ~Œªë=ò.
H

3) J=~˂¬ì}ãH›=° `Í_¨ Q® „¬sH›Æ.


M

4) JO`Ç~¡¾`Ç =¶~¡°æQ® „¬sH›Æ. ª^Î#: , " - " .


AM
IG

5) =¼uö~H› k‰§ãH›=° „¬sH›Æ.     


/ % 0 % 1
KH

6) ㄬ`ǼH›Æ JH›Æ~¡ HË_OQ· „¬sH›Æ.


R

J^Í x†Ç°=°O ㄬHê~¡O MODI#° HË_£ K͆ǰQê PRGL


MODEL - 1
‹¬=¶# `Í_¨ Q® „¬sH›Æ
=KÇ°ó#°.

. 0 % *
   
1. XH› HË_£ ƒì+¬…Õ BOWLER #° CPXMFS Qê ~Œ‹²#
J^Í HË_£ ƒì+¬…Õ GROUND #° Z…ì ~Œªë~¡°?
1 3 ( -
5. XH› ‹¬OöH`Ç ƒì+¬…Õ #° PUBLICATION
ª^Î#: # 0 8 - & 3
      NSZJGAYRGML Qê ~Œ†Ç°Qê J^Í ‹¬OöH`Ç ƒì+¬…Õ

$ 1 9 . ' 4 'PRINTING'#° Z…ì ~Œªë~¡°.

BOWLER …Õx ㄬu JH›Æ~¡O `Ç~¡°"Œ`Ç JH›Æ~¡OQê HË_£


ª^Î#: 1 6 # - * $ " 5 * 0 /
K͆ǰ|_#k. J^Í x†Ç°=°O P^¥~¡OQê GROUND#°           

HË_£ K͆ǰQê. / 4 ; + ( " : 3 ( . -


REASONING ABILITY (VERBAL) ○ ○ ○ ○ ○ ○ ○ ○ ○ ○
4
○ ○ ○ ○ ○ ○ ○ ○ ○ ○ ○ ○ ○ ○ ○ ○ ○ ○
Study Material
J^Íq^O
Î Qê PRINTING #° HË_£ K͆°Ç Qê NPGLRGLE J^Íq^ÎOQê INDIA #° HË_£ K͆ǰQê LSKRL =KÇ°ó#°.
=KÇ°ó#°.
* / % * "

1 3 * / 5 * / (     
        - 4 , 3 -

/ 1 ( - 3 ( - & 4. XH› HË_£ ƒì+¬…Õ GOA #° ISG Qê ~Œ†Ç°Qê J^Í HË_£


6. XH› HË_£ ƒì+¬…Õ CIVIL #° GMZMP Qê ~Œ‹²# J^Í ƒì+¬…Õ BEACH #° U q^ÎOQê ~Œªë~¡°.
HË_£ ƒì+¬…Õ PAPER #° U q^ÎOQê ~Œªë~¡°? ( 0 "
ª^Î#:
ª^Î#: $ * 7 * -   
     * 4 (
( . ; . 1 J^Íq^ÎOQê BEACH#° HË_£ K͆ǰQê DIGKR =KÇ°ó#°.
J^Íq^ÎOQê,

Y
# & " $ )
1 " 1 & 3     

M
    
% * ( , 3
5 & 5 * 7

E
PAPER À„„¬~Ÿ †³òH›ø HË_£ TETIV MODEL - 3

D
9
59
MODEL - 2 J=~˂¬ì} ãH›=° `Í_¨ Q® „¬sH›Æ
CA
95
P~˂¬ì} ãH›=° `Í_¨ Q® „¬sH›Æ 1. XH› HË_£ƒì+¬…Õ PARK #° OYOGQê ~Œ‹²# J^Í HË_£

50
ƒì+¬…Õ LOVE #° Z…ì ~Œªë~¡°?
98
A
1. XH› HË_£ ƒì+¬…Õ ACTION #° BEWMTT Qê ~Œ‹²#
ª^Î#: 1 " 3 ,
:8
J^Í HË_£ ƒì+¬…Õ INCOME #° Z…ì ~Œªë~¡°?
E

   
h

ª^Î#: " $ 5 * 0 / 0 : 0 (
.P
IC

     
LOVE #° KMSA =KÇ°ó#°.
AD

J^Íq^ÎOQê HË_£ K͆ǰQê


# & 8 . 5 5
- 0 7 &
AB
O

J^Í x†Ç°=°O ㄬHê~¡O INCOME #° HË_£ K͆ǰQê


   
ER

JPFSRK =KÇ°ó#°.
CH

, . 4 "
YD

* / $ 0 . &
     
2. XH› ‹¬OöH`Ç ƒì+¬…Õ READ #° PAUY Qê ~Œ†Ç°Qê J^Í
,H

+ 1 ' 4 3 ,
‹¬OöH`Ç ƒì+¬…Õ SAW #° U q^ÎOQê ~Œªë~¡°?
T
AM

2. XH› ‹¬OHö `Ç ƒì+¬…Õ CRICKET J#Qê DTLGPKA J~ò#


ª^Î#: 3 & " %
H

   
M

J^Í ‹¬OHö `Ç ƒì+¬…Õ HOCKEY #° Z…ì ~Œªë~°¡ ? 1 " 6 7


AM
IG

ª^Î#: $ 3 * $ , & 5 J^Íq^ÎOQê SAW#° HË_£ K͆ǰQê QWQ =KÇ°ó#°.


KH

      
4 " 8
% 5 - ( 1 , "
R

  

J^Í x†Ç ° =°O ㄬ H ê~¡ O HOCKEYx HË_£ KÍ † Ç ° Qê


2 8 2
IQFOJE =KÇ°ó#°.
3. XH› HË_£ ƒì+¬…Õ COAL #° BMXH Qê ~Œ†Ç°Qê J^Í
) 0 $ , & : HË_£ ƒì+¬…Õ FUEL #° Z…ì ~Œªë~¡°?
     
ª^Î#: $ 0 " -
* 2 ' 0 + &    

3. XH› HË_£ ƒì+¬…Õ STATE #° VYHCP Qê ~Œ†Ç°Qê J^Í # . 9 )


HË_£ ƒì+¬…Õ INDIA #° U q^ÎOQê ~Œªë~¡°? J^Íq^ÎOQê FUEL#° HË_£ K͆ǰQê ESBH =KÇ°ó#°.

ª^Î#: 4 5 " 5 & ' 6 & -


        

7 : ) $ 1 & 4 # )

REASONING ABILITY (VERBAL) ○ ○ ○ ○ ○ ○ ○ ○ ○ ○


5
○ ○ ○ ○ ○ ○ ○ ○ ○ ○ ○ ○ ○ ○ ○ ○ ○ ○
Study Material
4. XH› HË_£ ƒì+¬…Õ CAKE #° BXFXQê ~Œ†Ç°Qê J^Í
MODEL - 5
‹¬OöH`Ç ƒì+¬…Õ FOOD#° Z…ì ~Œªë~¡°?
ª^Î#: $ " , &
=¼uö~H› k‰§ãH›=° „¬sH›Æ
   
1. XH› HË_£ƒì +¬…Õ SATURN #° NRUTAS Qê ~Œ‹²#@Á~ò#
# 9 ' 9 J^Í HË_£ ƒì+¬…Õ SYSTEM#° Z…ì ~Œªë~¡°?
J^Íq^ÎOQê FOOD ° HË_£ K͆ǰQê ELJW =KÇ°ó#°.
ª^Î#: SATURN =¼uö~H› k‰× NRUTAS J^Íq^O Î Qê SYSTEM
' 0 0 %
   
=¼uö~H›k‰× METSYS.

& - + 8 2. XH› ‹¬OöH`Ç ƒì+¬…Õ BREAD #° DAERB Qê ~Œ†Ç°Qê


J^Í ‹¬OöH`ǃì+¬…Õ VEGETABLE #° Z…ì ~Œªë~¡°?
MODEL - 4 ª^Î#: BREAD #° =¼uö~H› k‰×…Õ ~Œ†Ç°Qê DAERB =KÇ°ó#°.
JO`Ç~¡¾`Ç =¶~¡°æ „¬sH›Æ J^Íq^ÎOQê VEGETABLE #° =¼uö~H›k‰×…Õ ~Œ†Ç°Qê

Y
ELBATEGEV =KÇ°ó#°.

M
1. XH› HË_£ ƒì+¬…Õ SQUARE #° AQUSERQê ~Œ†Ç°Qê 3. XH› ‹¬OöH`Ç ƒì+¬…Õ TIGER #° REGITQê ~Œ†Ç°Qê
J^Í HË_£ ƒì+¬…Õ CODING #° Z…ì ~Œªë~¡°?
J^Í ‹¬OöH`ǃì+¬…Õ LION #° Z…ì ~Œªë~¡°?

E
ª^Î#: 4 2 6 " 3 & ª^Î#: TIGER #° =¼uö~H› k‰×…Õ ~Œ†Ç°Qê REGIT =KÇ°ó#°.

D
9
59
J^Íq^ÎOQê LION #° =¼uö~H›k‰×…Õ ~Œ†Ç°Qê NOIL
" 2 6 4 & 3
CA
95
=KÇ°ó#°.

CODING x HË_£ K͆°Ç Qê IODCGN =KÇ°ó#°.

50
J^Íq^O
Î Qê

$ 0 % * / (
MODEL - 6
98
A
ㄬ`ǼH›Æ JH›Æ~¡ HË_OQ· „¬sH›Æ
:8
E

D `Ç~¡‚¬ð ㄬ‰×߁…Õ W=Þ|_# „¬^Î=ò…Õx XHËø


h

* 0 % $ ( /
Q®=°xH› :
.P
IC

2. XH› ‹¬OHö `Ç ƒì+¬…Õ ANDHRA #° NAHDAR Qê ~Œ†Ç°Qê JH›Æ~¡=ò "Í~ùH› JH›Æ~¡=ò`Ë HË_£ K͆ǰ|_È°`Ç°Ok.
AD

J^Í ‹¬OHö `Ç ƒì+¬…Õ MUMBAI x Z…ì ~Œªë~°¡ ? 1. XH›"ˆ


Í × 'TEACHER' #° 'LMKJNMP' Qê HË_£ K͋#²
AB
O

ª^Î#: " / % ) 3 " J^Í HË_£ ㄬHê~¡O 'HEART'x U=°x ã"Œªë~¡°?


ER
CH

ª^Î#: 5&"$)&3
YD

Î Qê MUMBAIx HË_£ K͆°Ç Qê UMBMIA =KÇ°ó#°.


J^Íq^O

-.,+/.1
,H

. 6 . # " *
T
AM

̄á^¥x #°O_ HEART


3. XH› ‹¬OöH`Ç ƒì+¬…Õ SECRET #° ECRSET Qê ~Œ†Ç°Qê
H

)&"35
M

J^Í ‹¬OöH`Ç ƒì+¬…Õ PRIVATE #° Z…ì ~Œªë~¡°?


AM
IG

a) IRPVATI b) TREATPI
/.,1-
KH

\ HEART †³òH›ø HË_£ ® NMKPL.


c) RATEVIP d) VARSTEP
R

ª^Î#: ECRSET #O^ΰ SECRET „¬^ÎO…Õx Jxß JH›Æ~Œ°


=ô<Œß~ò. J^Íq^ÎOQê P„¬Â#° c) RATEVIP #O^ΰ 2. XH› HË_£ ƒì+¬…Õ EARTHx QPMZS Qê ã"Œ‹²# J^Í
PRIVATE „¬^ÎO…Õ Q® Jxß JH›Æ~Œ° =ô<Œß~ò. ƒì+¬…Õ 'HEART' Z…ì HË_£ Kͪë=ò ?
4. XH› HË_£ ƒì+¬…Õ SPREAD #° ARPDES Qê ~Œ†Ç°Qê &"35)
J^Í HË_£ ƒì+¬…Õ MONKEY #° Z…ì ~Œªë~¡°?
ª^Î#:
ª^Î#: 4 1 3 & " % 21.;4
̄á^¥x #°O_

" 3 1 % & 4 )&"35


. 0 / , & :
421.;
& / 0 : , . \ HEART †³òH›ø HË_£ ® SQPMZ.
REASONING ABILITY (VERBAL) ○ ○ ○ ○ ○ ○ ○ ○ ○ ○
6
○ ○ ○ ○ ○ ○ ○ ○ ○ ○ ○ ○ ○ ○ ○ ○ ○ ○
Study Material
3. XH› HË_£ ƒì+¬…Õ 'PEN' x NZO Qê, 'BARK' x 4. XH› "Íˆ× 'PORTER'x 'MBNZQN' Qê HË_£ K͋²#,
'CTSL' Qê ã"Œ‹²# J^Í ƒì+¬…Õ 'PRANK' Z…ì HË_£ J^Í ƒì+¬…Õ 'REPORT'x Z…ì HË_£ Kͪë=ò ?
Kͪë=ò ? 1035&3
1&/ #"3, ª^Î#:
ª^Î#:
.#/;2/
/;0 $54- ̄á^¥x #°O_
̄á^¥x #°O_
3&1035
13"/,
/2.#/;
/450-
\ REPORT †³òH›ø HË_£ ® NQMBNZ.
\ PRANK †³òH›ø HË_£ ® NSTOL.

Y
123456789012345678901234567890121234567890
123456789012345678901234567890121234567890123456789012345678901212
12345678901234567890123456789012123456789012345678901234567890121234567890123456789012345678901212345678901234567890123
1234567890123456789012345678901212345678901234567890123456789012123456789012345678901234567890121234567890123
1234567890123456789012345678901212345678901234567890123456789012123456789012345678901234567890121234
123456789012345678901234567890121234567890
123456789012345678901234567890121234567890123456789012345678901212
12345678901234567890123456789012123456789012345678901234567890121234567890123456789012345678901212345678901234567890123456789012123
12345678901234567890123456789012123456789012345678901234567890121234567890123456789012345678901212345678901234567890123
1234567890123456789012345678901212345678901234567890123456789012123456789012345678901234567890121234567890123
1234567890123456789012345678901212345678901234567890123456789012123456789012345678901234567890121234
12345678901234567890123456789012123456789012345678901234567890121234567890123456789012345678901212345678901234567890123456789012123
123456789012345678901234567890121234567890123456789012345678901212
12345678901234567890123456789012123456789012345678901234567890121234567890123456789012345678901212345678901234567890123
1234567890123456789012345678901212345678901234567890123456789012123456789012345678901234567890121234567890123
1234567890123456789012345678901212345678901234567890123456789012123456789012345678901234567890121234
123456789012345678901234567890121234567890
12345678901234567890123456789012123456789012345678901234567890121234567890123456789012345678901212345678901234567890123456789012123

M
123456789012345678901234567890121234567890123456789012345678901212
12345678901234567890123456789012123456789012345678901234567890121234567890123456789012345678901212345678901234567890123456789012123
12345678901234567890123456789012123456789012345678901234567890121234567890123456789012345678901212345678901234567890123
1234567890123456789012345678901212345678901234567890123456789012123456789012345678901234567890121234567890123
1234567890123456789012345678901212345678901234567890123456789012123456789012345678901234567890121234
PRACTICE TEST on TYPE - I
12345678901234567890123456789012123456789012345678901234567890121234567890123456789012345678901212345678901234567890123456789012123
12345678901234567890123456789012123456789012345678901234567890121234567890123456789012345678901212345678901234567890123
1234567890123456789012345678901212345678901234567890123456789012123456789012345678901234567890121234567890123
1234567890123456789012345678901212345678901234567890123456789012123456789012345678901234567890121234
123456789012345678901234567890121234567890
123456789012345678901234567890121234567890123456789012345678901212
12345678901234567890123456789012123456789012345678901234567890121234567890123456789012345678901212345678901234567890123456789012123
12345678901234567890123456789012123456789012345678901234567890121234567890123456789012345678901212345678901234567890123
1234567890123456789012345678901212345678901234567890123456789012123456789012345678901234567890121234567890123
1234567890123456789012345678901212345678901234567890123456789012123456789012345678901234567890121234
12345678901234567890123456789012123456789012345678901234567890121234567890123456789012345678901212345678901234567890123456789012123
123456789012345678901234567890121234567890123456789012345678901212
12345678901234567890123456789012123456789012345678901234567890121234567890123456789012345678901212345678901234567890123
1234567890123456789012345678901212345678901234567890123456789012123456789012345678901234567890121234567890123
1234567890123456789012345678901212345678901234567890123456789012123456789012345678901234567890121234
12345678901234567890123456789012123456789012345678901234567890121234567890123456789012345678901212345678901234567890123456789012123

E
1. XH› ㄬ`ͼH› HË_£ ƒì+¬…Õ TEACHER #° VGCEJGT 8. XH› ‹¬OHö `Ç ƒì+¬…Õ EXCURTION #° CXEURTNOIQê

D
9
Jx ã"Œ‹²#, DULLARDx ƒì+¬…Õ U=°x ã"Œªë=ò? ~Œ‹²# J^Í ‹¬OHö `Ç ƒì+¬…Õ OUTBURSTS #° U q^O Î Qê

59
1) FWMNCTF 2) FWNNBTE ~Œªë~¡°?
CA
95
3) FWNNCSF 4) FWNNCTF 1) TUOBRSUTS 2) OUTRUBSTS

50
2. XH› „¬iƒì+¬…Õ TEARS#° UGDVXQê ~Œ‹²# SMILE#° 3) UTBTRUTSS 4) TUOBURSTS
98
9. XH› ‹¬OöH`Ç ƒì+¬…Õ CHILD #° IMNQJ Qê ~Œ‹²# J^Í
A
Z…ì ~Œª=ò?
‹¬OöH`Ç ƒì+¬…Õ BABE #° U q^ÎOQê ~Œªë~¡°?
:8
1) TLOPJ 2) TOLPJ
E

3) JPLOT 4) PLJTO 1) FGFK 2) HFHJ 3) FFGJ 4) HFGJ


h
.P

3. XH› ‹¬OöH`Ç ƒì+¬…Õ ADD #° WOOQê, SUM #° 10. XH› ‹¬ O ö H `Ç ƒ ì +¬ … Õ VACCINATIONS #°


IC
AD

QJM Qê =°i†Çò TOTAL #° KPKWX Qê ~Œ‹²# NICCAVSNOITAQê ~Œ‹²# J^Í ‹¬OHö `Ç ƒì+¬…Õ COM-
J^Í HË_£ ƒì+¬…Õ TOADS#° U q^ÎOQê ~Œªë~¡°? MUNICATIONS #° U q^ÎOQê ~Œªë~¡°?
AB
O

1) KPWOQ 2) KPOWQ 1) SNOITACINUMMOC


ER
CH

3) KPWOJ 4) KPWQO 2) INUMMOCSNOITAC


YD

3) MMOCUNICAITSNO
4. XH› ‹¬OöH`Ç ƒì+¬…Õ ELECTION #° GLGCVIQN Qê
4) SNOIMMOCUNICAT
,H

~Œ‹²# J^Í ‹¬OöH`Ç ƒì+¬…Õ VOTER#° U q^ÎOQê


11. XH› HË_£ ƒì+¬…Õ PLANNING #° UFFHSCASAQê
T
AM

~Œªë~¡°?
~Œ‹²# J^Í HË_£ ƒì+¬…Õ AUTHORITY #° U q^ÎOQê
H

1) VOXET 2) VQTGR
M

~Œªë~¡°?
3) XOVET 4) WPUFU
AM
IG

1) FBOYTNLTN 2) FOYBTLNND
5. XH› HË_£ ƒì+¬…Õ MEDAL #° XPOLWQê ~Œ‹²# J^Í
3) FYOTBNNLT 4) FBOYTLNTN
KH

HË_£ ƒì+¬…Õ CADGE #° U q^ÎOQê ~Œªë~¡°?


12. XH› ‹¬OöH`Ç ƒì+¬…Õ ACTOR #° EGXSVQê ~Œ‹²# J^Í
R

1) PNQRT 2) NLORP
‹¬OöH`Ç ƒì+¬…Õ STAGE #° U q^ÎOQê ~Œªë~¡°?
3) LJMPN 4) MLOQP
1) WYDJH 2) XYFLJ 3) WXEKI 4) VWFLJ
6. XH› HË_£ ƒ ì +¬ … Õ CORRESPONDENCE #°
13. XH› ‹¬OöH`Ç ƒì+¬…Õ QUESTION #° NXBVQLLQQê
NUTTRAXUPQRPNR Qê ~Œ‹²# J^Í HË_£ ƒì+¬…Õ ~Œ‹²# J^Í ‹¬OHö `Ç ƒì+¬…Õ REPLY #° U q^O
Î Qê ~Œªë~°¡ ?
SCOPE #° U q^ÎOQê ~Œªë~¡°? 1) YHMOV 2) OBMVI
1) AUXNR 2) ANUXR 3) OHMOV 4) VHSOB
3) RNUXA 4) XUPAR 14. XH› ‹¬OHö `Ç ƒì+¬…Õ EDUCATION #° DECUTATION
7. XH› ‹¬OöH`Ç ƒì+¬…Õ VACATE #° YEFEWI Qê ~Œ‹²# Qê ~Œ‹²# J^Í ‹¬OHö `Ç ƒì+¬…Õ COLLEGE #° U q^O Î Qê
J^Í ‹¬OöH`Ç ƒì+¬…Õ OCCUPY #° U q^ÎOQê ~Œªë~¡°? ~Œªë~¡°?
1) RGFYSC 2) SHGZTD 1) EOLCGLE 2) COELLEG
3) QEEXRB 4) SFGXTC 3) OCLLGEE 4) OCLGEEL
REASONING ABILITY (VERBAL) ○ ○ ○ ○ ○ ○ ○ ○ ○ ○
7
○ ○ ○ ○ ○ ○ ○ ○ ○ ○ ○ ○ ○ ○ ○ ○ ○ ○
Study Material
15. XH› ‹¬OHö `Ç ƒì+¬…Õ FAILURE #° OHRSDYN Qê ~Œ‹²# 18. XH› ‹¬OöH`Ç ƒì+¬…Õ OPFGBCST #° NEAR †³òH›ø
J^Í ‹¬OöH`Ç ƒì+¬…Õ SUCCESS #° U q^ÎOQê ~Œªë~¡°? HË_£Qê ~Œ†Ç°Qê J^Í ‹¬OöH`Ç ƒì+¬…Õ IJVWHI J#°#k
1) BBNNZBB 2) BLNNZBB ^Íx †³òH›ø HË_£ J=ô`Ç°Ok?
3) BLNNZZB 4) BBLJNZB 1) KEG 2) HUG
16. XH› HË_£ ƒì+¬…Õ APPROACH #° CHOAPRAP Qê 3) HAG 4) HUT
~Œ‹²# J^Í HË_£ ƒì+¬…Õ RESTRICT#° U q^ÎOQê
19. XH› HË_£ ƒì+¬…Õ FINGER #° DGLECP Qê ~Œ‹²#
~Œªë~¡°?
J^Í HË_£ ƒì+¬…Õ KIDNEY #° U q^ÎOQê ~Œªë~¡°?
1) TCIRSTRE 2) CTRISTRE
1) IGBLCW 2) IGCLBW
3) CTRISTER 4) ERTSIRTC
17. XH› ‹¬OöH`Ç ƒì+¬…Õ PENCIL #° LICNEPQê ~Œ‹²# 3) IBCGLE 4) IGBKCV
J^Í ‹¬OöH`Ç ƒì+¬…Õ PREPARATION #° U q^ÎOQê 20. XH› ‹¬OöH`Ç ƒì+¬…Õ UNITED#° SLGRCB Qê ~Œ‹²#
~Œªë~¡°? J^Í ‹¬OöH`Ç ƒì+¬…Õ DISOWN#° U q^ÎOQê ~Œªë~¡°?

Y
1) NOITARAPREP 2) NOITRAPAERP 1) BGQLUM 2) CGRLTK

M
3) RAPERPATION 4) NOITARAPERP 3) BGQMUL 4) CGRTLK
12345678901234567890123456789012
12345678901234567890123456789012123456789012345678901234567890121234567890123456789012345678901212345678901234567890123
12345678901234567890123456789012

q=~¡}ì`ÇàH› ["Œ|°°
12345678901234567890123456789012123456789012345678901234567890121234567890123456789012345678901212345678901234567890123
12345678901234567890123456789012 12345678901234567890123456789012

E
12345678901234567890123456789012123456789012345678901234567890121234567890123456789012345678901212345678901234567890123
12345678901234567890123456789012 12345678901234567890123456789012
12345678901234567890123456789012123456789012345678901234567890121234567890123456789012345678901212345678901234567890123
12345678901234567890123456789012 12345678901234567890123456789012
12345678901234567890123456789012123456789012345678901234567890121234567890123456789012345678901212345678901234567890123
12345678901234567890123456789012 12345678901234567890123456789012
12345678901234567890123456789012123456789012345678901234567890121234567890123456789012345678901212345678901234567890123
12345678901234567890123456789012 12345678901234567890123456789012
12345678901234567890123456789012123456789012345678901234567890121234567890123456789012345678901212345678901234567890123
12345678901234567890123456789012 12345678901234567890123456789012

D
9
59
1. (4) 5 & " $ ) & 3 7 0 5 & 3
CA
95
         
7 ( $ & + ( 5 9 0 7 & 5

50
\ VOTER †³òH›ø HË_£ ® XOVET.
98
A
J^Íq^ÎOQê,
. & % " -
:8
% 6 - - " 3 % 5. (2)
E

      
h

    
9 1 0 - 8
.P

' 8 / / $ 5 '
IC

\ DULLARD †³òH›ø HË_£ ® FWNNCTF.


AD

J^Íq^ÎOQê,
AB
O

2. (2) 5 & " 3 4 $ " % ( &


         
ER
CH

6 ( % 7 9 / - 0 3 1
YD

J^Íq^ÎOQê, \ CADGE †³òH›ø HË_£ ® NLORP.


,H

4 . * - & 6. (2) C O R R E S P O N D E N C E
T
AM


5

0

-

1

+
¯ ¯ ¯ ¯¯¯ ¯¯¯¯¯¯¯¯
H
M

N U T T R A X U P Q R P N R
\ SMILE †³òH›ø HË_£ ® TOLPJ.
AM

̄á "Œ\÷ #°O_


IG

3. (1) A D D S U M T O T A L S C O P E
KH

¯ ¯ ¯ ¯ ¯ ¯ ¯¯¯¯ ¯ ¯ ¯ ¯ ¯
R

W O O Q J M K P K W X A N U X R
̄á "Œ\÷ #°O_ \ SCOPE †³òH›ø HË_£ ANUXR. ®
T O A D S
7. (1) 7 " $ " 5 &
¯ ¯ ¯ ¯ ¯      
K P W O Q : & ' & 8 *
\ TOADS †³òH›ø HË_£ KPWOQ.® J^Íq^ÎOQê,
4. (3) & - & $ 5 * 0 / 0 $ $ 6 1 :
         
( - ( $ 7 * 2 / 3 ( ' : 4 $
J^Íq^ÎOQê, \ OCCUPY †³òH›ø HË_£ ® RGFYSC.
REASONING ABILITY (VERBAL) ○ ○ ○ ○ ○ ○ ○ ○ ○ ○
8
○ ○ ○ ○ ○ ○ ○ ○ ○ ○ ○ ○ ○ ○ ○ ○ ○ ○
Study Material
8. (4) & 9 $ 6 3 5 * 0 / 14. (3) & % 6 $ " 5 * 0 /

$ 9 & 6 3 5 / 0 * 0 & $ 6 5 " 0 * /


J^Íq^ÎOQê, J^Íq^ÎOQê, $ 0 - - & ( &
0 6 5 # 6 3 4 5 4
0 $ - - ( & &
5 6 0 # 6 3 4 5 4 \ COLLEGE †³òH›ø HË_£ ® OCLLGEE.
\ OUTBURSTS †³òH›ø HË_£ ® TUOBURSTS. 15. (4) ' " * - 6 3 &
9. (4) $ ) * - %       
     0 ) 3 4 % : /
* . / 2 + J^Íq^OÎ Qê, 4 6 $ $ & 4 4
J^Íq^ÎOQê,       
# # - + / ; #

Y
# " # &
    \ SUCCESS †³òH›ø HË_£ ® BBLJNZB.

M
) ' ( + 16. (2) 1 2 3 4 5 6 7 8
\ BABE †³òH›ø HË_£ ® HFGJ. A P P R O A C H

E
10. (2) VACCINATIONS ® VACCIN / ATIONS HË_£ 7 ® 8 5 6 3 4 1 2

D
9
(i=~Ÿž K͆ǰO_) C H O A Q R A P

59
NICCAV / SNOITA J^Íq^ÎOQê,
CA
95
\ HË_£ ® NICCAVSNOITA 1 2 3 4 5 6 7 8
COMMUNICATIONS ®

50
R E S T R I C T
COMMUNI / CATIONS
98 ®
A
HË_£ 7 8 5 6 3 4 1 2
(i=~Ÿž K͆ǰO_)
:8
C T R I S T R E
INUMMOC / SNOITAC
E

17. (4) „¬^ÎO HË_£


\ HË_£ ® INUMMOCSNOITAC
h
.P

j> Ÿ
IC

11. (2) 1 - " / / * / ( PENCIL LICNEP


AD

        J^Íq^ÎOQê,
AB
O

6 ' ' ) 4 $ 4 " j> Ÿ


J^Íq^ÎOQê, PREPRATION NOITARPERP
ER
CH

" 6 5 ) 0 3 * 5 : 18. (2) 0 1 ' ( # $ 4 5


YD

                
/ & " 3
,H

' 0 : # 5 - / / %
\ AUTHORITY †³òH›ø HË_£ ® FOYBTLNND.
T
AM

J^Íq^ÎOQê, * + 7 8 ) *
H

12. (3) " $ 5 0 3      


M

     ) 6 (
AM

\ HË_£ ® HUG
IG

& ( 9 4 7
KH

J^Íq^ÎOQê, 19. (1) ' * / ( & 3


R

4 5 " ( &      

     % ( - & $ 1
8 9 & , * J^Íq^ÎOQê, , * % / & :
\ STAGE †³òH›ø HË_£ ® WXEKI      

13. (3) 2 6 & 4 5 * 0 / * ( # - $ 8


       
20. (3) 6 / * 5 & %
/ 9 # 7 2 - - 2      

J^Íq^ÎOQê, 4 - ( 3 $ #
3 & 1 - : J^Íq^ÎOQê, % * 4 0 8 /
          

0 ) . 0 7 # ( 2 . 6 -
\ REPLY †³òH›ø HË_£ ® OHMOV \ DISOWN †³òH›ø HË_£ ® BGQMUL.
REASONING ABILITY (VERBAL) ○ ○ ○ ○ ○ ○ ○ ○ ○ ○
9
○ ○ ○ ○ ○ ○ ○ ○ ○ ○ ○ ○ ○ ○ ○ ○ ○ ○
Study Material
TYPE - II : #O|~Ÿ / ‹²O|…˜ HË_OQ· 5. XH› HË_£ ƒì+¬…Õ DARLING #° 65Qê HË_£ K͋²# J^Í
HË_£ƒì+¬…Õ FRIEND #° Uq^ÎOQê HË_£ Kͪë~¡°?
G D qƒìQ®O…Õ POQ®Á =~¡â=¶…Õx JH›Æ~Œä›½ ‹¬OY¼#°
ª^Î#: D A R L I N G
HË_£°Qê W=Þ|_ÈO [~¡°Q®°`Ç°Ok. ㄬ‰×ß…Õ W=Þ|_#
¯ ¯ ¯ ¯ ¯ ¯ ¯
„¬^ÎO…Õx ㄬu JH›Æ~ŒxH÷ XH› HË_£ ‹¬OY¼ =ôO_Í…ì …è^¥
(4) (1) (18) (12) (9) (14) (7)
U^Ë XH› ‹²Ö~¡ x†Ç°=°O J#°‹¬iOKͅì HË_£ Kͪë~¡°. P
x†Ç°=¶xß =°#O Q®°iëOz, P HË_£#° L„¬†³¶yOz
Þÿ4 + 1 + 18 + 12 + 9 + 14 + 7 = 65
J^Íq^ÎOQê F R I E N D
=°~ùH› „¬^¥xH÷ HË_£#° Q®°iëOKŒe.
¯ ¯ ¯ ¯ ¯ ¯
(6) (18) (9) (5) (14) (4)
MODEL - 1
1. XH› HË_£ ƒì+¬…Õ ZEBRA #° 2652181 Jx ã"Œ‹²#
Þÿ6 + 18 + 9 + 5 + 14 + 4 = 56

Y
COBRA #° U=°x ã"Œªë=ò? MODEL - 2

M
ª^Î#: POQ®Á =~¡â=¶ ㄬHê~¡=ò A = 1 ; B = 2; E = 5; ..... R
= 18; Z = 26 ã"Œ†Ç°Qê 1. XH› HË_£ ㄬHê~¡O TWENTY J#° „¬^= Î ò#° 74221372
® Jx ã"Œ‹²#, J^Í ƒì+¬…Õ NINE J#° „¬^Î=ò#° U=°x

E
ZEBRA 2652181 Hê=ô#
®ÿ ã"Œªë=ò?

D
J^Í ã„¬Hê~¡=ò COBRA †³òH›ø HË_£ 3152181.

9
®

59
2. XH› ‹¬OöH`Ç ƒì+¬…Õ DEAR #° 45118 Qê ~Œ†Ç°Qê J^Í ª^Î#: TWENTY †³òH›ø HË_£ 74221372 POQ®Á =~¡= â ¶…Õx
CA
95
‹¬OöH`Ç ƒì+¬…Õ SURE †³òH›ø HË_£ Uq°\÷? JH›~Æ Œ#° J„¬‹=
¬ ¼ k‰×…Õx ‹¬OY¼KÍ HË_£ K͆°Ç _È"°³ #
Ø k.
®
50
ª^Î#: D E A R J^Í ã„¬Hê~¡=ò NINE †³òH›ø HË_£ 13181322
¯ ¯ ¯ ¯ 98
A
J=ô`Ç°Ok.
®
:8
4 5 1 18 45118. 2. XH› ㄬ`ͼH› ƒì+¬…Õ 'PRABA' #° '27595' Qê =°i†Çò
E

J^Íq^ÎOQê, S U R E
h

'THILAK' #° 368451 Qê ã"Œ‹²# J^Í ƒì+¬…Õ


¯ ¯ ¯ ¯
.P
IC

BHARATI x U q^ÎOQê ã"Œªë~¡°.


® 1921185
AD

19 21 18 5
ª^Î#: P R A B A T H I L A K
AB
O

3. XH› ‹¬OöH`ǃì+¬…Õ HEAD #° 8514Qê HË_£ K͆ǰQê


¯ ¯ ¯ ¯ ¯ ¯ ¯ ¯ ¯ ¯ ¯
J^Í ‹¬OöH`ǃì+¬…Õ HIDE †³òH›ø HË_£ Uq°\÷?
ER
CH

2 7 5 9 5 3 6 8 4 5 1
ª^Î#: POQ®Á JH›Æ~¡=¶…Õx JH›Æ~¡ ªÖ<Œ ㄬHê~¡O
YD

̄á^¥x #°O_,
A – 1, D – 4, E – 5, H – 8 ÿÞ
,H

H E A D B H A R A T I
T

¯ ¯ ¯ ¯ ¯ ¯ ¯ ¯ ¯ ¯ ¯
AM
H

9 6 5 7 5 3 8
M

8 5
1 4
Þ \ BHARATI †³òH›ø HË_£ ®ÿ9657538.
AM

J^Íq^ÎOQê D – 4, E – 5, H – 8, I – 9
IG

H I D E 3. XH› ‹¬OHö `Ç ƒì+¬…Õ PAPER †³òH›ø HË_£ 12145 =°i†Çò


KH

¯ ¯ ¯ ¯ RAPE †³òH›ø HË_£ 5214 J~ò# J^Í HË_£ ƒì+¬…Õ


R

8 9 4 5 PEPPER †³òH›ø HË_£ Uq°\÷?


4. XH› HË_£ ƒì+¬…Õ LIKE #° 129115 Qê HË_£ K͋²# J^Í ª^Î#: P A P E R R A P E
‹¬OöH`ǃì+¬…Õ HATE †³òH›ø HË_£ Uq°\÷? ¯ ¯ ¯ ¯ ¯ ¯ ¯ ¯ ¯
ª^Î#: POQ®Á JH›Æ~¡=¶…Õx JH›Æ~¡ ªÖ<Œ ㄬHê~¡O 1 2 1 4 5 5 2 1 4
E – 5, I – 9, K – 11, L – 12 ÿÞ Wzó# ï~O_È° „¬^¥ HË_£ #°O_,
L I K E P – 1, A – 2, E – 4, R – 5 Jx `³e†ÇòKÇ°#ßk.
12 9 11 5 ® 129115 \ P E P P E R
J^Íq^ÎOQê A – 1, E – 5, H – 8, T = 20 Þ ¯ ¯ ¯ ¯ ¯ ¯
H A T E
1 4 1 1 4 5
8 1 20 5 ® 81205

REASONING ABILITY (VERBAL) ○ ○ ○ ○ ○ ○ ○ ○ ○


10
○ ○ ○ ○ ○ ○ ○ ○ ○ ○ ○ ○ ○ ○ ○ ○ ○ ○ ○
Study Material
4. XH› HË_£ ƒì+¬…Õ CHINA #° 38126 Qê#¶, NEPAL#° HË_£ ® (4+2)/(5+2)/(18+2) ® 6/7/20 ® 6720
25769 Qê#¶ ~Œ‹²# J^Í „¬iƒì+¬…Õ PEALEN #° J^Íq^ÎOQê
Z…ì HË_£ Kͪë~¡°? GREEN N E E R G
ª^Î#: C H I N A N E P A L ¯ ¯ ¯ ¯ ¯
¯ ¯ ¯ ¯ ¯ ¯ ¯ ¯ ¯ ¯ 14 5 5 18 7
3 8 1 2 6 2 5 7 6 9
\ HË_£ ®ÿ (14+2)/(5+2)/(5+2)/(18+2)/(7+2)
̄á# W=Þ|_# HË_£ #°O_ P – 7, E – 5, A – 6, L
– 9, N – 2 Jx `³e†ÇòKÇ°#ßk.
® 16/7/7/20/9 ® 1677209
4. XH› HË_£ ƒì+¬…Õ GREAT #° 51Qê `³e†Ç°*苲# J^Í
P E A L E N
¯ ¯ ¯ ¯ ¯ ¯ HË_£ ƒì+¬…Õ FOOL †³òH›ø HË_£ Uq°\÷?
7 5 6 9 5 2 ª^Î#: G R E A T
\ ¯ ¯ ¯ ¯ ¯

Y
P HË_£ ƒì+¬…Õ PEALEN †³òH›ø HË_£ 756952
J=ô`Ç°Ok. 7 18 5 1 20 ® 7 + 18 + 5 + 1 + 20 = 51

M
5. XH› „¬iƒì+¬…Õ 'PARK'x '5394ÑQê, 'SHIRT'x J^Íq^ÎOQê, F O O L
¯¯ ¯ ¯

E
'17698Ñ Qê =°i†Çò 'PANDIT'x '532068ÑQê, HË_£
² , J^Í HË_£ ƒì+¬…Õ 'NISHAR' x Z…ì HË_£ Kͪë=ò?
6 15 15 12 ® 6 + 15 + 15 + 12 = 48

D
9
K͋#

59
ª^Î#: P A R K S H I R T P A N D I T 5. XH› HË_£ ƒì+¬ DOUBLE J#° „¬^¥xß 118 Qê =°i†Çò
CA
95
¯ ¯ ¯ ¯ ¯ ¯ ¯¯¯ ¯ ¯ ¯ ¯ ¯ ¯ HUNDRED J#° „¬^¥xß 148Qê ~Œ‹²# J^Í HË_£ ƒì+¬…Õ

50
5 3 9 4 1 7 6 9 8 5 3 2 0 6 8 TRIPLE †³òH›ø HË_£ Uq°\÷?
98
A
̄á# "Œ\÷ #°O_ ª^Î#: % 0 6 # - &
:8
N I S H A R
¯ ¯ ¯ ¯ ¯ ¯
E

     


h
.P
IC

2 6 1 7 3 9
¨
\ ®
AD

NISHAR †³òH›ø HË_£ 261739.


) 6 / % 3 & %
AB
O

MODEL - 3
ER

      


CH

1. XH› HË_£ ƒì+¬…Õ TOM = 48; DICK = 27 J~ò#


YD

HARRY U=°=ô`Ç°Ok? ¨


,H

ª^Î#: POQ®Á =~¡â=¶ ㄬHê~¡=ò T = 20; O = 15; M = 13; D J^Íq^ÎOQê, 5 3 * 1 - &


T
AM

= 4; I = 9; C = 3; K = 11 Hê=ô# TOM = 20 + 15 + 13
H

     


M

= 48 =°i†Çò DICK = 4 + 9 + 3 + 11 = 27 J^Í


AM
IG

ㄬHê~¡=ò HARRY = 8 + 1 + 18 + 18 + 25 = 70 ¨


KH

J=ô`Ç°Ok. 6. XH› HË_£ ƒì+¬…Õ RISK – CAST = 16 =°i†Çò SAT –


R

2. XH› "Íˆ× E = 5, PEN = 35 J~ò# PAGE = ? EAT = 5 J~ò# J^Í HË_£ ƒì+¬…Õ PAME – EAP
ª^Î#: POQ®Á =â=¶…Õx JH›Æ~Œ J#°~¡¶„¬ ‹¬OY¼ ㄬHê~¡O †³òH›ø q°= ZO`Ç?
A = 1, B = 2, C = 3 ....... Z = 26 ª^Î#: R I S K – C A S T
Hê=ô# E = 5 ; PEN = P + E + N = 16 + 5 + 14 = 35 (18) (9) (19) (11) (3) (1) (19) (20)
PAGE = P + A + G + E Þÿÿ
(18 – 9 – 19 – 11) – (3 – 1 – 19 – 20)
= 16 + 1 + 7 + 5 = 29. = –21 – (–37) = –21 + 37 = 16
3. XH› "Íˆ× RED x '6720ÑQê HË_£ K͋²# J^Í ã„¬Hê~¡=ò J^Íq^ÎOQê,
'GREEN' x Z…ì HË_£ Kͪë~¡°? P A M E – E A P
ª^Î#: D E R (16) (1) (13) (5) (5) (1) (16)
¯ ¯ ¯ Þÿÿ
(16 – 1 – 13 – 5) – (5 – 1 – 16)
4 5 18 = (–3) – (–12) = –3 + 12 = 9

REASONING ABILITY (VERBAL) ○ ○ ○ ○ ○ ○ ○ ○ ○


11
○ ○ ○ ○ ○ ○ ○ ○ ○ ○ ○ ○ ○ ○ ○ ○ ○ ○ ○
Study Material
7. XH› ‹¬OHö `Çƒì +¬…Õ Z = 52 =°i†Çò MAN = 56 J~ò# 2. XH› ㄬ`¼Í H› HË_£ ƒì+¬…Õ 'BRDER'x '×Ó$#Ó' Qê ã"Œ‹²#
P ‹¬OöH`Ç ƒì+¬…Õ STORY q°= ZO`Ç? J^Í ƒì+¬…Õ 'BOARD' x '×H%Ó$' Qê ã"Œ‹²# J^Í
ª^Î#: POQ®Á JH›Æ~Œ ª^¥~¡} ªÖ# q°=ä›½ ï~\÷“O„¬ô K͋² ƒì+¬…Õ ABODE Uq^ÎOQê ã"Œªë~¡°?
W=Þ|_¨Û~ò. Z ª^¥~¡} ªÖ# q°= 26. Hêx WH›ø_
ª^Î#: B R DE R B O A R D
HË_£ ƒì+¬…Õ 2(26) « 52Qê W=Þ|_#k. Jxß
JH›Æ~Œä›½ W^Í x†Ç°=°O =i달°ëOk.
¯ ¯ ¯¯ ¯ ¯ ¯ ¯ ¯ ¯
M A N × Ó $ # Ó × H % Ó $
(26) (2) (28) Þ 26 + 2 + 28 = 56 ̄á "Œ\÷ #°O_ A B O D E
J^Íq^ÎOQê S T O R Y ¯ ¯ ¯¯ ¯
(38) (40) (30) (36) (50)
Þ 38 + 40 + 30 + 36 + 50 = 194.
% × H $ #
\ ABODE †³òH›ø HË_£ ® %×H$#

Y
8. XH› =¼H÷ë HOSPITAL J#° „¬^¥xß 1000Qê HË_£
K͋²# J^Í ‹¬OöH`Ç ƒì+¬…Õ AMBULANCE †³òH›ø 3. XH› HË_£ ƒì+¬…Õ 'RAT'x 'HD%'Qê, 'CAT'x '# %'Qê D

M
HË_£ ZO`Ç? =°i†Çò 'FAT'x '$D%'Qê HË_£ K͋²#, J^Í ƒì+¬…Õ
ª^Î#: H O S P I T A L 'CAR' †³òH›ø HË_£ „¬^ÎO U=°=ô`Ç°Ok.

E
¯ ¯ ¯ ¯ ¯ ¯ ¯ ¯ ª^Î#: R A T C A T F A T

D
9
¯ ¯ ¯ ¯ ¯ ¯ ¯ ¯ ¯

59
(8) (15) (19) (16) (9) (20) (1) (12)
Þÿ 8 + 15 + 19 + 16 + 9 + 20 + 1 + 12 = 100 H D % # D % $ D %
CA
95
Þ 100 × 10 = 1000

50
̄á# "Œ\÷ #°O_
J^Íq^ÎOQê A M B U L A N C E
98
A
C A R
¯ ¯ ¯ ¯ ¯ ¯ ¯ ¯ ¯ ¯ ¯ ¯
:8
(1) (13) (2) (21) (12) (1) (14) (3)(5)
# D H
E

Þÿ1 + 13 + 2 + 21 + 12 + 1 + 14 + 3 + 5 = 72
h

\ ® #DH.
.P
IC

CAR †³òH›ø HË_£


Þÿ72 × 10 = 720
AD

9. BAT + BALL = 50 =°i†Çò SIX + FOUR = 112 4. XH› HË_£ ƒì+¬…Õ 'RAID'x '%#H$'Qê, 'RIPE'x
AB
O

J~ò# BREAK + STOP q°= ZO`Ç? '%H@©'Qê HË_£ K͋²# J^Í HË_£ ƒì+¬…Õ 'DEAR'x
®ÿ
ER
CH

ª^Î#: B A T + B A L L 23 + 27 = 50 Z…ì HË_£ Kͪë=ò?


(2) (1) (20) (2) (1)(12)(12)
YD

ª^Î#: R A I D R I P E
S I X + F O U R ®ÿ
52 + 60 = 112
¯ ¯ ¯ ¯ ¯ ¯ ¯ ¯
,H

(19)(9)(24) (6)(15)(21)(18)
H H
T

% # $ % @ ©
AM

J^Íq^ÎOQê B R E A K + S T O P
H

̄á# "Œ\÷ #°O_


M

(2) (18) (5)(1)(11) (19) (20)(15)(16)

Þÿ37 + 70 = 107 D E A R
AM
IG

¯ ¯ ¯ ¯
KH

MODEL - 4 $ © # %
\ ® $©#%.
R

DEAR †³òH›ø HË_£


1. XH› xií+¬“ HË_£ ƒì+¬…Õ 'STRING'x '%=*- $Ó' Jh,
5. XH› HË_£ ƒì+¬…Õ 'P'x '#'Qê, 'A'x '%'Qê, 'C'x 'f'Qê
=°i†Çò PRAISE x '?*@ – % ×' Qê HË_£ K͋²# J^Í
Jx 'E'x '@'Qê HË_£ K͋#
² J^Í HË_£ ƒì+¬…Õ 'PEACE'x
ƒì+¬…Õ 'GRAPES' x U=°x ã"Œªë~¡°?
Z…ì HË_£ Kͪë=ò?
ª^Î#: S T R I N G P R A I S E
¯ ¯ ¯¯ ¯ ¯ ¯ ¯ ¯ ¯ ¯ ¯ ª^Î#: HË_£ ƒì+¬ ㄬHê~¡O
% = * – $ Ó ? * @ – % þ ®ÿ
P #, A %, C ,E @ ®ÿ ®ÿf ®ÿ
̄á "Œ\÷ #°O_ G R A P E S \
P E A C E
¯ ¯ ¯¯ ¯ ¯ ¯ ¯ ¯ ¯ ¯
Ó * @ ? þ % # @ % f @
\ GRAPES †³òH›ø HË_£ ® Ó*@?×% \ PEACE †³òH›ø HË_£ ® #@%f@.
REASONING ABILITY (VERBAL) ○ ○ ○ ○ ○ ○ ○ ○ ○
12
○ ○ ○ ○ ○ ○ ○ ○ ○ ○ ○ ○ ○ ○ ○ ○ ○ ○ ○
Study Material
MODEL - 5 ̄á "Œ\÷ #°O_ G O L D
¯ ¯ ¯¯
1. XH› HË_£ ƒì+¬…Õ 'TREAD'x '7%#94'Qê =°i†Çò 5 % @ 3
'PREY'x '$%#8'Qê HË_£ K͋²#, J^Í HË_£ ƒì+¬…Õ \ GOLD †³òH›ø HË_£ ® 5%@3.
'ARTERY'x U q^ÎOQê HË_£ Kͪë=ò?
4. XH› HË_£ ƒ ì +¬ … Õ 'FIRE' x '#%@$' Qê =°i†Ç ò
ª^Î#: T R E A D P R E Y
¯ ¯ ¯ ¯ ¯ ¯ ¯ ¯ ¯ 'DEAL'x '©$H 'Qê HË_£ K͋²# J^Í HË_£ ƒì+¬…Õ­
7 % # 9 4 $ % # 8
'FAIL'x Z…ì HË_£ Kͪë=ò?
ª^Î#: F I R E D E A L
̄á "Œ\÷ #°O_ A R T E R Y
¯ ¯ ¯¯ ¯ ¯ ¯ ¯ ¯ ¯ ¯ ¯ ¯ ¯
9 % 7 # % 8 # % @ $ © $ H ­
\ ARTERY †³òH›ø HË_£ 9%7#%8. ® ̄á# "Œ\÷ #°O_

Y
F A I L
2. XH› HË_£ ƒì+¬…Õ 'GONE'x '5@©9'Qê =°i†Çò
¯ ¯ ¯ ¯

M
'SEAL'x '69%H'Qê HË_£ K͋#² J^Í ƒì+¬…Õ 'LOGS'x # H % ­
Z…ì HË_£ Kͪë=ò?
\ 'FAIL' †³òH›ø HË_£ ® #H%­.

E
ª^Î#: G O N E S E A L
¯ ¯ ¯ ¯ ¯ ¯ ¯ ¯

D
9
5. XH› HË_£ ƒì+¬…Õ 'SAFER'x '5@3#2'Qê =°i†Çò

59
5 @ © 9 6 9 % H 'RIDE'x '2©%#'Qê HË_£ K͋²# J^Í HË_£ ƒì+¬…Õ
CA
95
̄á "Œ\÷ #°O_ L O G S 'FEDS'x Z…ì HË_£ Kͪë=ò?
¯ ¯ ¯¯
50
ª^Î#: S A F E R R I D E
H ¯ ¯ ¯ ¯ ¯ 98 ¯ ¯ ¯ ¯
A
@ 5 6
\ LOGS †³òH›ø HË_£ ® H@56
:8
5 @ 3 # 2 2 © % #
E
h

3. XH› HË_£ ƒì+¬…Õ 'GONE'x '5%2#'Qê =°i†Ç ò ̄á# "Œ\÷ #°O_


.P
IC

'MEDAL'x '4#3$@'Qê HË_£ K͋²# J^Í HË_£ ƒì+¬…Õ F E D S


AD

'GOLD'x Z…ì HË_£ Kͪë=ò. ¯ ¯ ¯ ¯


AB
O

ª^Î#: G O N E M E D A L 3 # % 5
¯ ¯ ¯ ¯ ¯ ¯ ¯ ¯ ¯ \ ® 3#%5.
ER

'FEDS' †³òH›ø HË_£


CH

5 % 2 # 4 # 3 $ @
YD

TYPE - III : ㄬuöHƄ¬} HË_OQ·


,H
T
AM

G D `Ç~‚¡ ð¬ ㄉ¬ ß× …Õ Hùxß „¬^¥ä›½ |^ΰ° "Í~ùH› „¬^¥#° 3. XH› „¬ i ƒ ì +¬ … Õ =$`Ç ë O #° n~¡ É K Ç ` Ç ° ~¡ 㠋¬ O Qê#°,
H

ㄬuöHƄ²OKÇ@O [~¡°Q®°`Ç°Ok.
M

n~¡ É K Ç ` Ç ° ~¡ 㠋¬ O #° n~¡ É = $`Ç ë O Qê#°, n~¡ É = $`Ç ë O #°


AM
IG

1. XH›"Íˆ× '䛽H±Ñx '|@Á~ŸÑ Jh, '|@Á~ŸÑx '"Í°<Í[~ŸÑ Jh, KÇ`Ç°~¡ã‹¬OQê#°, KÇ`Ç°~¡ã‹¬O#° ~ŒO|‹¹Qê#° „²°=|_#
"Í°<Í[~Ÿx '\©K~ Ç ÑŸ Jh, '\©K~ Ç ÑŸ x H›~
Á øŸ Jh =°i†Çò 'H›~
Á øŸ Ñx
KH

P „¬iƒì+¬ ㄬHê~¡O ãQ®°_È°Û U PHê~¡O…Õ =ôO_È°#°?


'ㄲxž‡…˜Ñ Jx Jx# `Ç~¡Q®u Q®k…Õ ‡~”¡=ò#° ƒÕkOKÇ°
R

ª^Î#: Q®°_È°Û n~¡É=$`ŒëHê~¡O…Õ =ôO_È°#°.


"Œ~¡° Z=~¡°?
ª^Î#: '\©K~Ç ÑŸ ‡~”O¡ x ƒÕkªë_°È . Hêx ^Î`ŒëO‰×O ㄬHê~¡O '\©K~Ç ÑŸ Hêx Wzó# „¬iƒì+¬…Õ n~¡= É $`ÇOë , KÇ`°Ç ~¡ã‹¬OQê =ôO_È°#°.
J#Qê H›Á~Ÿø Hê=ô# 'H›Á~ŸøÑ ‡~”¡=ò#° ƒÕkªë_È°. \
ãQ®°_È°Û †³òH›ø PHê~¡O 'KÇ`Ç°~¡ã‹¬OÑ.
2. XH› „¬iƒì+¬…Õ PHê‰×O#° ‹¬=òã^ÎOQê#°, ‹¬=òã^ÎO#° 4. XH› HË_£ ƒì+¬…Õ 5 J#Qê 10  10 J#Qê 15  15
h~¡°Qê#°, h~¡°#° QêeQê#°, Qêex "Í°„¦°¬ OQê#°, "Í°„¦°¬ O#° J#Qê 20  20 J#Qê 25  25 J#Qê 29  29 J#Qê
#kQê#° „²°=|_Ok. J~ò# ^¥‚¬ìO "͋²# =°#=ò 31  31 J#Qê 38. J~ò# ‰×Ã^Μ =~¡¾ ‹¬OY¼ Uk?
ã`ŒöQq Uq? ª^Î#: Wzó# ‹¬OY¼° 5, 10, 15, 20, 25, 39, 31, 38…Õ
ª^Î#: ^¥‚¬ìO "͋# ² „¬ô_È° h~¡° ã`ŒQ®°`ŒO. Hêx Wzó# „¬iƒì+¬…Õ
‰×Ã^Μ =~¡¾‹¬OY¼ 25.
h~¡°#° QêeQê „²°‹¬°ë<Œß~¡°. Hê=ô# ^¥‚¬ìO "͋²#„¬ô_È°
Qêex ã`ŒQêe. Hêx HË_£ ㄬHê~¡=ò 25 J#Qê 29 Hê=ô# ‰×Ã^Μ=~¡¾ ‹¬OY¼
\ ‹¬=¶^¥#O 'QêeÑ. 29 J=ô`Ç°Ok.

REASONING ABILITY (VERBAL) ○ ○ ○ ○ ○ ○ ○ ○ ○


13
○ ○ ○ ○ ○ ○ ○ ○ ○ ○ ○ ○ ○ ○ ○ ○ ○ ○ ○
Study Material
5. XH› "Íˆ× '=~¡Â=òÑ#° 'h~¡°Ñ Jx 'h~¡°Ñ#° '~Ë_È°ÛÑ Jx, 6. XH› ƒì+¬…Õ H›#°ß#° KÍ`Ç°x, KÍ`Ç°#° <Ë~¡° Jx,
'~Ë_È°ÛÑ#° '"Í°„¦¬°=òÑ Jx, '"Í°„¦¬°=òÑ#° 'PHê‰×=òÑ Jx, <Ë~¡°#° K³=ô°, K³=ô#° =ò䛽ø, =ò䛽ø#° <Œ°H› Jx
'PHê‰×=òÑ#° '‹¬=òã^Î=òÑ Jx '‹¬=òã^Î=òÑ#° 'ƒì@Ñ Jx ƒìqÀ‹ë, XH› =¼H÷ë ^Íx`Ë qO\ì_È°?
JO\ì=ò. J~ò# q=¶#=ò ZH›ø_È ZQ®°~¡°#°?
ª^Î#: ª^¥~¡}OQê =¼ä›½ë° K³=ô`Ë qO\ì~¡°. WH›ø_È HË_£
ª^Î#: ª^¥~¡}OQê q=¶#=ò PHê‰×O…Õ ZQ®°~¡°`Ç°Ok. HË_£
ㄬHê~¡=ò PHê‰×=ò#° ‹¬=òã^Î=ò Jx JO\ì=ò. Hê=ô# ƒì+¬…Õ K³=ô#° =ò䛽ø JO\ì=ò. Hê=ô# HË_£ ƒì+¬…Õ
HË_£ ƒì+¬…Õ q=¶#=ò ZQ®°~¡° ㄬ^͉×=ò ‹¬=òã^Î=ò. =ò䛽ø`Ë qO\ì_È°.
123456789012345678901234567890121234567890
123456789012345678901234567890121234567890123456789012345678901212
12345678901234567890123456789012123456789012345678901234567890121234567890123456789012345678901212345678901234567890123
1234567890123456789012345678901212345678901234567890123456789012123456789012345678901234567890121234567890123
1234567890123456789012345678901212345678901234567890123456789012123456789012345678901234567890121234
123456789012345678901234567890121234567890
123456789012345678901234567890121234567890123456789012345678901212
12345678901234567890123456789012123456789012345678901234567890121234567890123456789012345678901212345678901234567890123456789012123
12345678901234567890123456789012123456789012345678901234567890121234567890123456789012345678901212345678901234567890123
1234567890123456789012345678901212345678901234567890123456789012123456789012345678901234567890121234567890123
1234567890123456789012345678901212345678901234567890123456789012123456789012345678901234567890121234
123456789012345678901234567890121234567890123456789012345678901212
12345678901234567890123456789012123456789012345678901234567890121234567890123456789012345678901212345678901234567890123456789012123
12345678901234567890123456789012123456789012345678901234567890121234567890123456789012345678901212345678901234567890123
1234567890123456789012345678901212345678901234567890123456789012123456789012345678901234567890121234567890123
1234567890123456789012345678901212345678901234567890123456789012123456789012345678901234567890121234
123456789012345678901234567890121234567890
12345678901234567890123456789012123456789012345678901234567890121234567890123456789012345678901212345678901234567890123456789012123
12345678901234567890123456789012123456789012345678901234567890121234567890123456789012345678901212345678901234567890123
1234567890123456789012345678901212345678901234567890123456789012123456789012345678901234567890121234567890123
1234567890123456789012345678901212345678901234567890123456789012123456789012345678901234567890121234
123456789012345678901234567890121234567890123456789012345678901212
PRACTICE TEST on TYPE - III
12345678901234567890123456789012123456789012345678901234567890121234567890123456789012345678901212345678901234567890123456789012123
12345678901234567890123456789012123456789012345678901234567890121234567890123456789012345678901212345678901234567890123
1234567890123456789012345678901212345678901234567890123456789012123456789012345678901234567890121234567890123
1234567890123456789012345678901212345678901234567890123456789012123456789012345678901234567890121234
123456789012345678901234567890121234567890
12345678901234567890123456789012123456789012345678901234567890121234567890123456789012345678901212345678901234567890123456789012123
123456789012345678901234567890121234567890123456789012345678901212
12345678901234567890123456789012123456789012345678901234567890121234567890123456789012345678901212345678901234567890123
1234567890123456789012345678901212345678901234567890123456789012123456789012345678901234567890121234567890123
1234567890123456789012345678901212345678901234567890123456789012123456789012345678901234567890121234
12345678901234567890123456789012123456789012345678901234567890121234567890123456789012345678901212345678901234567890123456789012123
123456789012345678901234567890121234567890123456789012345678901212
12345678901234567890123456789012123456789012345678901234567890121234567890123456789012345678901212345678901234567890123
1234567890123456789012345678901212345678901234567890123456789012123456789012345678901234567890121234567890123
1234567890123456789012345678901212345678901234567890123456789012123456789012345678901234567890121234
12345678901234567890123456789012123456789012345678901234567890121234567890123456789012345678901212345678901234567890123456789012123

1. ~Œ|O^ΰ#° ^ùOQ®Qê, ^ùOQ®#° ƒÕ@°Qê, ƒÕ@°#° ‡éb‹¬°Qê, 6. XH› „¬iƒì+¬…Õ „¬ä½Æ› #° ~ŒA°Qê, ~ŒA#° „¬ô=ôށ°Qê,
‡éb‹¬°#° „¬ô‹¬ëH›OQê, „¬ô‹¬ëH›O#° `Ç°‡H©Qê, `Ç°‡H©x „¬ô=ôށ#° „¦¬°<Œ°Qê, „¦¬°<Œ#° >è|°…˜žQê, >è|°…˜ž#°

Y
̄#°ßQê, ̄#°ß#° …ìsQê „²°=|_Í „¬iƒì+¬…Õ #k…Õ =°#°+¬µ¼°Qê =°i†Çò =°#°+¬µ¼#° „¬ä›Æ½°Qê „²ez#,

M
ㄬ†Ç¶}÷OKÇ°@䛽 L„¬†³¶Q®„¬_Í ª^Î#O Uk? Q®°…ìc#° U=°x „²°ªë~¡°?
1) ƒÕ@° 2) ‡éb‹¬° 3) „¬ô‹¬ëH›O 4) `Ç°‡H© 1) „¦¬°<Œ° 2) >è|°…˜ž 3) „¬ä›Æ½° 4) „¬ô=ôށ°

E
2. XH› „¬iƒì+¬…Õ WO[h~Ÿ#° Hêï~æO@~ŸQê, Hêï~æO@~Ÿ#° 7. XH› „¬iƒì+¬…Õ Z~¡°„¬ô#° QêeQê, Qêex #°„¬ôQê, #°„¬ô#°

D
9
̄~òO@~ŸQê, ̄~òO@~Ÿ#° ^ÎsûQê, ^Îsûx _¨H›“~ŸQê, PHê‰×OQê, PHꉧxß hOQê, hO#° ^Î¶oQê =°i†Çò

59
_¨H›“~Ÿ#° ~¡KÇ~ò`ÇQê, ~¡KÇ~ò`Ç#° H›OªeQê, H›Oªex ^Î¶ox `³°„¬ôQê „²ez#, „¬ä›Æ½° ZH›ø_È ZQ®°~¡°`Œ~ò?
CA
95
^ùOQ®Qê „²ez#, |@“° 䛽>è“ =¼H÷ë Z=~¡°? 1) PHê‰×O 2) Qêe 3) hO 4) ^Î¶o

50
1) ^Îsû 2) H›Oªe 3) ~¡KÇ~ò`Ç 4) _¨H›“~¡° 8. J}°=ô#° [O`Ç ° =ôQê, [O`Ç ° =ô#° „¬ ~ ¡ = ¶}°=ôQê,
98
A
3. K³~ä¡ ½› #° P„²…˜Qê, P„²…˜#° K³Hø› Qê, K³Hø› #° J~¡\ H÷ ê†Ç°Qê, „¬~¡=¶}°=ô#° Z¢Hê“<£Qê, Z¢Hê“<£#° ‹²O‚¬ìOQê,
:8
J~¡\H÷ ê†Ç°#° =¶q°_Hê†Ç°Qê, =¶q°_Hê†Ç°#° ̋…˜‡é¦ <£Qê, ‹²O‚¬ìO#° #¶¼ã\ì<£Qê, #¶¼ã\ì<£#° „¬ôeQê, „¬ôex
E

ã‡é\ì<£Qê, ã‡é\ì<£#° ‚Ôìe†Ç°OQê „²°=|_Í „¬iƒì+¬…Õ


h

̋…˜‡¦é<£#° Hê¼e䛽¼…è@~ŸQê, Hê¼e䛽¼…è@~Ÿ#° Hêy`ÇOQê


.P

J_Èq…Õ x=tOKÇ° ã䛀~¡=°$Q®O.


IC

„²  °=|_Í „¬ i ƒ ì +¬ … Õ „¦ ¬ i ßKÇ ~ Ÿ `Ç † Ç ¶ ~¡ ° KÍ † Ç ò @ä› ½


AD

L„¬†³¶yOKÇ° „¬^¥~¡ÖO. 1) „¬ôe =°i†Çò ‹²O‚¬ìO


2) ‚Ôìe†Ç°O =°i†Çò [O`Ç°=ô
AB
O

1) K³H›ø 2) Hêy`ÇO 3) P„²…˜ 4) J~¡\ ÷Hê†Ç°


3) #¶¼ã\ì<£ =°i†Çò ã‡é\ì<£
4. XH› „¬iƒì+¬…Õ ƒ’¶q°x ‹¬~¡‹¬°žQê#¶, ‹¬~¡‹¬°ž#° ~Œ~ò
ER
CH

4) [O`Ç°=ô =°i†Çò Z¢Hê“<£


Qê#¶, ~Œ~òx |~¡°=ôQê#¶, |~¡°=ô#° À‹“_†Ç°OQê#¶,
YD

9. XH› „¬iƒì+¬…Õ h\÷x ~Œ~òQê, ~Œ~òx #¶<³Qê, #¶<³#°


À‹“_† °Ç O#° ‹¬=òã^ÎOQê#¶, ‹¬=òã^¥xß "Œ#Qê#°, "Œ##°
QêeQê, Qêex K³H›øQê, K³H›ø#° "Œ†Çò=ôQê, "Œ†Çò=ô#°
,H

̄¦~
á Q
Ÿ ê#¶ „²ez#, ãH÷Hï \˜ =¶¼KŸ P_È° ‹¬ Ö O P „¬iƒì+¬…Õ
T

ã^Î=OQê „²ez#, PH÷ž[<£ XH›


AM

1) À‹“_†Ç°O 2) ‹¬=òã^ÎO 3) ‹¬~¡‹¬°ž 4) "Œ#


1) "Œ†Çò=ô 2) ã^Î=O 3) ~Œ~ò 4) K³H›ø
H

5. „¬‹¬°„¬ô#° Hê~¡°Qê, Hê~¡°#° =òiH÷Qê, =òiH÷x ‰×Ãハ’`ÇQê,


M

10. XH› „¬iƒì+¬…Õ ~Œyx |OQê~¡OQê, |OQê~¡O#° `ÇQ~ ® O


¡ Qê,
AM

‰×Ãハ’`Ç#° #°„¬ôQê, #°„¬ô#° `³°„¬ôQê, `³°„¬ô#°


IG

`ÇQ®~¡O#° "³O_Qê, "³O_x ‹Ô“°Qê, ‹Ô“°#° K³H›øQê,


K³@°“Qê, K³@°“#° ^ΰ=òàQê, ^ΰ=òà#° pH›\ ÷Qê „²°=|_Í
KH

K³H›ø#° h~¡°Qê „²ez# q^ΰ¼`Ÿ fQ®° `džǶ~¡°K͆Çò@䛽


„¬iƒì+¬…Õ „¬ä›Æ½° ^Íx̄á "Œ`Œ~ò? L„¬†³¶yOKÇ° „¬^¥~¡ÖO À„~¡°
R

1) ^ΰ=òà 2) K³@°“ 3) Hê~¡° 4) „¬‹¬°„¬ô 1) |OQê~¡O 2) ~Œy 3) ‹Ô“° 4) "³O_


12345678901234567890123456789012
12345678901234567890123456789012123456789012345678901234567890121234567890123456789012345678901212345678901234567890123
12345678901234567890123456789012

q=~¡}ì`ÇàH› ["Œ|°°
12345678901234567890123456789012123456789012345678901234567890121234567890123456789012345678901212345678901234567890123
12345678901234567890123456789012 12345678901234567890123456789012
12345678901234567890123456789012
12345678901234567890123456789012123456789012345678901234567890121234567890123456789012345678901212345678901234567890123
12345678901234567890123456789012
12345678901234567890123456789012
12345678901234567890123456789012123456789012345678901234567890121234567890123456789012345678901212345678901234567890123
12345678901234567890123456789012 12345678901234567890123456789012
12345678901234567890123456789012123456789012345678901234567890121234567890123456789012345678901212345678901234567890123
12345678901234567890123456789012
12345678901234567890123456789012
12345678901234567890123456789012123456789012345678901234567890121234567890123456789012345678901212345678901234567890123
12345678901234567890123456789012
12345678901234567890123456789012
12345678901234567890123456789012123456789012345678901234567890121234567890123456789012345678901212345678901234567890123
12345678901234567890123456789012

1. (2) ㄬ‰ß× ã„¬Hê~¡O ƒÕ@° J#Qê '‡éb‹¹Ñ. Hê=ô# ‹¬=¶^¥#O 4. (2) ㄬ‰×ß ã„¬Hê~¡O À‹“_†Ç°O J#Qê ‹¬=òã^ÎO. Hê=ô#
'‡éb‹¹Ñ. ‹¬=¶^¥#O '‹¬=òã^ÎOÑ.
2. (4) ㄬ‰×ß ã„¬Hê~¡O ^Îsû J#Qê _¨H›“~Ÿ. Hê=ô# ‹¬=¶^¥#O 5. (1) ㄬ‰×ß ã„¬Hê~¡O K³@°“#° ^ΰ=òàQê „²°ªë~¡°. Hê=ô#
'_¨H›“~ŸÑ. ‹¬=¶^¥#O '^ΰ=òàÑ.
3. (4) ㄬ‰×ß ã„¬Hê~¡O K³H›ø J#Qê J~¡\ ÷ Hê†Ç°. Hê=ô# 6. (1) 7. (3)
‹¬=¶^¥#O 'J~¡\ ÷Hê†Ç°Ñ. 8. (3) ‹²O‚¬ìO Ê „¬ôe   #¶¼ã\ì<£ Ê ã‡é\ì<£.
9. (2) 10. (1)

REASONING ABILITY (VERBAL) ○ ○ ○ ○ ○ ○ ○ ○ ○


14
○ ○ ○ ○ ○ ○ ○ ○ ○ ○ ○ ○ ○ ○ ○ ○ ○ ○ ○
Study Material
TYPE - IV : ‹¬OY¼ =°i†Çò Q®°~¡°ë 5. 673 J„¬Qê abscond from duty ; 351 J#Qê

#°O_ ‹¬=¶KŒ~¡O H›#°Qù#°@ On government duty ; 164 J#Qê Obtained


from government J~ò`Í 7#° ‹¬¶zOKÍ Code
MODEL - 1 Uk ?

1. XH› xiœ+¬“ HË_£ ƒì+¬…Õ '253Ñ J#Qê "books are ª^Î#: mBCTDPOEGSPNEVUZ

old" Jx J~¡Ö=ò '546Ñ J#Qê 'man is old' Jx mPOHPWFSONFOUEVUZ
J~¡Ö=ò =°i†Çò '378Ñ J#Qê 'buy good books'
Jx J~¡Ö=ò, J~ò# P HË_£…Õ 'are' †³òH›ø HË_£ mPCUBJOFEGSPNHPWFSONFOU
U=°=ô`Ç°Ok? ̄á "Œ\÷ #°O_
ª^Î#: #PPLTBSFPMEm 7 †³òH›ø HË_£ „¬^ÎO ® abscond.

Y
6. 246 J#Qê Manage your work ; 803 J#Qê
Man is m
PME  They bring it ; 631 J#Qê They work quickly

M
#VZHPPECPPLTm J~ò`Í 3#° ‹¬¶zOKÍ Code Uk ?
m.BOBHFZPVSXPSL

E
\ are Þ '2'. ª^Î#: 

D
m5IFZCSJOHJU

9
2. 37 J#Qê which class, 583 J#Qê Caste and

59
class J~ò`Í Caste#° ‹¬¶zOKÍ Code Uk?
CA m5IFZXPSLRVJDLMZ

95
ª^Î#:   m XIJDI DMBTT

50
̄á "Œ\÷ #°O_
m 98 ® They.
A
  DBTUF BOE DMBTT 3 #° ‹¬¶zOKÍ HË_£
:8
\ ̄á "Œ\÷ #°O_
MODEL - 2
E

®
h

Caste †³òH›ø HË_£ 5 (…è^¥) 8.


.P
IC

3. XH› HË_£ …ìOöQÞ*˜…Õ 123 J#Qê 'how are you' 1. XH› xií+¬“ HË_£ ƒì+¬…Õ 'pen pencil' x '$ ' Qê ¸
AD

Jh 423 J#Qê 'how are they' Jh, 356 J#Qê 'Eraser sharper' x '@#' Qê =°i†Çò 'pencil
AB
O

'how is she' Jx HË_£ K͆ǰ|_#k. J~ò`Í D eraser'x '$@'Qê ã"ŒÌ‹#°. J^Í HË_£ ƒì+¬…Õ 'pen'
ER
CH

ãH÷Ok "Œx…Õ Uk '3Ñ †³òH›ø HË_£ „¬^Î=°=ô`Ç°Ok. †³òH›ø HË_£ ?


® IPX BSF you
YD

ª^Î#: 1  
ª^Î#: m w
QFOQFODJM 
,H

4   ® IPX BSF they


T

&BSTFSTIBSQFSm!
AM

 5 6 ® IPX is she
H

1FODJM&SBTFSm!
M

\ ̄á "Œ\÷ #°O_


AM
IG

3 †³òH›ø HË_£ „¬^ÎO ®ÿ'how'.


Hê=ô# pen Þ '¸'.
KH

4. 123 J#Qê Admit him tommoroow ; 345


2. XH› HË_£ ƒì+¬…Õ 'go for morning walk' J#Qê
R

J#Qê tommorow evening come ; 256 J#Qê


come and admit ; 675 J#Qê go and come '$H?#' Jx, 'good for health' J#Qê '£?@' Jx
J~ò`Í tommorrow #° ‹¬¶zOKÍ Code Uk ? =°i†Çò 'good to walk fast' J#Qê '+­@#' Jx
J~¡ÖO. J~ò# 'health' x ‹¬¶zOKÍ HË_£ Uk?
ª^Î#: "ENJUIJNUPNNPSSPXm
ª^Î#: HPGPSNPSOJOHXBML +  m
5PNNPSSPX&WFOJOH$PNFm
m
HPPEGPSIFBMUI b !
$PNFBOEBENJUm
HPPEUPXBMLGBTUm !l
go and come ®6 7 5
̄á "Œ\÷ #°O_
̄á "Œ\÷ #°O_
Tommorrow †³òH›ø HË_£ ® 3. health †³òH›ø HË_£ ® £.

REASONING ABILITY (VERBAL) ○ ○ ○ ○ ○ ○ ○ ○ ○


15
○ ○ ○ ○ ○ ○ ○ ○ ○ ○ ○ ○ ○ ○ ○ ○ ○ ○ ○
Study Material
3. XH› HË_£ ƒì+¬…Õ 'Spark and fire' x '$H?' Qê
ten sti SQT ® every tuesday NPSOJOH
=°i†Çò 'Spark is cause'x '$#@' Qê, 'fire is
effect'x '£#©' Qê HË_£ K͋²# P ƒì+¬…Õ 'cause' TLJ ptr qim ® OJDF market place
†³òH›ø HË_£ ZO`Ç? ㄬHø› # HË_£ "ŒH›¼=ò #°O_ Sunday J#° „¬^=
Î ò#䛽
ª^Î#: m
4QBSLBOEGJSF + HË_£ tri.

4QBSLJTDBVTFm !
4. XH› HË_£ ƒì+¬…Õ 'Pin ton sudjo' J#Qê 'Bring
me a pen' Jx, 'Sud jo ton don' J#Qê 'Bring
m
GJSFJT&GGFDU  bª
me a pencil' Jx HË_£ K͆ǰ|_#k. J~ò`Í D
̄á "Œ\÷ #°O_ ãH÷Ok "Œx…Õ Uk 'Pen' H÷ ‹¬ï~á# HË_£ Uk?
Cause †³òH›ø HË_£ ® @. ª^Î#: 1JOUPOTVEKP #SJOHNFBQFO m
TYPE - V : ‹¬=¶KŒ~¡ „¬^Î=ò HË_£x m

Y
4VEKPEPO #SJOHNFBQFODJM
H›#°Qù#°@

M
̄á "Œ\÷ #°O_
1. XH› HË_£ ƒì+¬…Õ 'nec po tam' J#Qê 'boys are
\ÿpen †³òH›ø HË_£ ® pin.

E
studying' Jx, 'me tam sam' J#Qê 'grapes 5. XH› HË_£ ƒì+¬…Õ 'Sin tan jin' J#Qê 'a red rose'

D
9
are sour' Jx 'ism po me' J#Qê 'boys and Jh  'tan chin ke' J#Qê 'big red box' Jh,

59
CA
'kon sin he' J#Qê 'a lotus flower' Jx HË_£

95
grapes' Jx HË_£ K͆ǰ|_#k. D ãH÷Ok "Œx…Õ
'sour' x ‹¬¶zOKÍ HË_£ Uk? K͆ǰ_È"³°Ø#k. J~ò`Í P ƒì+¬…Õ 'Rose'#° ‹¬¶zOKÍ

50
m HË_£ Uk?
98
A
ª^Î#: OFDQPUBN CPZTBSFTUVEZJOH
®ÿ B SFE rose
:8
NFUBNTBNmHSBQFTBSFTPVS ª^Î#: 4JO UBO jin
E

5BO chin ke ®ÿ big SFE box


h

m
.P
IC

kon TJO he ®ÿ B lotus flower


JTNQPNF CPZTBOEHSBQFT
AD

̄á "Œ\÷ #°O_


AB
O

\ÿsour †³òH›ø HË_£ ® sam. ̄á "Œ\÷ #°O_


\ÿrose †³òH›ø HË_£ ® jin.
ER
CH

2. XH› ㄬ`¼Í H› HË_£ ƒì+¬…Õ "sim min tin" J#Qê 'Wild


TYPE - VI : =¶ãuH› HË_OQ·
YD

Animal Dangerous" Jx  'Min ken Pit" J#Qê


"One pet animal" Jx, "Kim Pin Pit" J#Qê
,H
T

"Pet and will" Jx J~¡=œ ò J~ò# ken J#° „¬^=Î ò L^¥‚¬ì~¡} : ãH÷O^Î W=Þ|_# „¬\ ÷“H› P^¥~¡OQê ㄬ‰×߁䛽
AM

†³òH›ø HË_£ „¬^Î=ò Uk?


H

["Œ|°° Q®°iëOKÇO_. „¬\ ÷“H›…Õ Hùxß JOïH°, Hùxß


M

ª^Î#: Sim .JO ®ÿ wild BOJNBM dangerous


tin
AM

JH›Æ~Œ° W=Þ_ÈO [iyOk. „¬\ ÷“H›…Õx U^Íx JH›Æ~Œxß


IG

NJO ken QJU ® one QFU BOJNBM HË_£ K͆°Ç =‹²#„¬C_È° "³ò^Î@Qê P JH›~Æ O¡ H›ey =ô#ß
KH

J_È°Û =~¡°‹¬…Õx JOïH#° f‹¬°Hùx, `Ç~ŒÞ`Ç J^Í JH›Æ~¡O


R

kim pin QJU ® QFU and will H›ey L#ß x°=ô =~¡°‹¬…Õx JOïH#° „¬H›ø<Í [`ÇK͋²
ㄬH›ø# HË_£ "ŒH›¼=ò #°O_ ken J#° „¬^Î=ò#䛽 HË_£Qê KÇ^ΰ==…ÿ#°.
HË_£ one.
         
3. XH› xiœ+¬“ ƒì+¬…Õ "Ski rps tri" J#Qê "nice
 " # $ % &  1 - 0 5 /
sunday morning" Jx, "ten sti rps" J#Qê
 & " # $ %  5 1 / - 0
"every tuesday morning" =°i†Çò "ski ptr  % & " # $  1 / 5 0 -
qim" J#Qê "nice market place" Jx J~¡œ=ò.  $ % & " #  0 / 5 1 -
"sunday" J#° „¬^Î=ò ^Íx HË_£ JQ®°#°?  # $ % & "  - 0 1 / 5

®ÿ NPSOJOH =¶ã\÷H±ž - I =¶ã\÷H±ž - II


ª^Î#: 4LJ USJ OJDF sunday

REASONING ABILITY (VERBAL) ○ ○ ○ ○ ○ ○ ○ ○ ○


16
○ ○ ○ ○ ○ ○ ○ ○ ○ ○ ○ ○ ○ ○ ○ ○ ○ ○ ○
Study Material
1. ̄á# W=Þ|_# ï~O_È° =¶ã\÷H±ž P^¥~¡OQê BOLD J#° a) 75,
11, 89, 86 b) 97, 68, 44, 79
„¬^ÎO †³òH›ø HË_£ D ãH÷Ok "Œx…Õ (P)
(A) (L) (M) (P) (A) (L) (L)
a) 34, 57, 95, 42 b) 12, 69, 68, 31 c) 55,
56, 67, 98
c) 23, 96, 89, 20 d) ̄á=x߆Çü (P)
(L) (N) (N)
®
ª^Î#: B #° ‹¬¶zOKÇ° HË_£° ÿ01, 12, 23, 34, 40 Hê=ô# a, b, c…Õ Un Plan #° ‹¬¶zOKÇ^ΰ.
3. ̄á# W=Þ|_# ï~O_È° =¶ã\÷H±ž P^¥~¡OQê CLEAN
O #° ‹¬¶zOKÇ° HË_£° ® 57, 69, 78, 85, 96
J#° „¬^ÎO †³òH›ø HË_£ D ãH÷Ok "Œx…Õ
L #° ‹¬¶zOKÇ° HË_£° ®56, 68, 79, 89, 95
a) 13, 79, 11, 98, 41 b) 11, 24, 33, 89, 76
D #° ‹¬¶zOKÇ° HË_£° ® 03, 14, 20, 31, 42.
c) 30, 10, 67, 89, 86 d) 41, 89, 04, 44, 59
a) 34, 57, 95, 42 b) 12, 69, 68, 31
(B) (O) (L) (D) (B) (O) (L) (D)
ª^Î#: C #° ‹¬¶zOKÇ° HË_£° 02, 13, 24, 30, 41 ®
L #° ‹¬¶zOKÇ° HË_£° 56, 68, 79, 89, 95 ®

Y
c) 23, 96, 89, 20
E #° ‹¬¶zOKÇ° HË_£° 04, 10, 21, 32, 43 ®
(B) (O) (L) (D)
®

M
A #° ‹¬¶zOKÇ° HË_£° 00, 11, 22, 33, 44
Hê=ô# a, b, c° =ü_ȶ Bold J#° „¬^¥<Íß N #° ‹¬¶zOKÇ° HË_£° 59, 67, 76, 86, 98 ®

E
‹¬¶zªë~ò. a) 13, 79, 11, 98, 41
2. ̄á# W=Þ|_# ï~O_È° =¶ã\÷H±ž P^¥~¡OQê PLAN J#°

D
9
(C) (L) (A) (N) (C)

59
„¬^ÎO †³òH›ø HË_£ D ãH÷Ok "Œx…Õ b) 11, 24, 33, 89, 76
CA
95
a) 75, 11, 89, 96 b) 97, 68, 44, 79 (A) (C) (A) (L) (N)

50
c) 30, 10, 67, 89, 86
c) 55, 56, 67, 98 d) UnHê^ΰ
(C) (E) (N) (L) (N)
98
A
ª^Î#: P #° ‹¬¶zOKÇ° HË_£° ® 55, 66, 75, 88, 97 d) 41, 89, 04, 44, 59
:8
L #° ‹¬¶zOKÇ° HË_£° ® 56, 68, 79, 89, 95 (C) (L) (E) (A) (N)
E
h

®
.P

A #° ‹¬¶zOKÇ° HË_£° 00, 11, 22, 33, 44 ̄á# KǶ„¬|_# HË_£…Õ


IC

N #° ‹¬¶zOKÇ° HË_£° ® 59, 67, 76, 86, 98 d) 41, 89, 04, 44, 59
AD

C L E A N
AB
O

=¶ã`Ç"Í° CLEAN #° ‹¬¶z‹¬°ëOk.


ER
CH

123456789012345678901234567890121234567890
123456789012345678901234567890121234567890123456789012345678901212
123456789012345678901234567890121234567890123456789012345678901212
12345678901234567890123456789012123456789012345678901234567890121234567890123456789012345678901212345678901234567890123
1234567890123456789012345678901212345678901234567890123456789012123456789012345678901234567890121234567890123
1234567890123456789012345678901212345678901234567890123456789012123456789012345678901234567890121234
123456789012345678901234567890121234567890
12345678901234567890123456789012123456789012345678901234567890121234567890123456789012345678901212345678901234567890123456789012123
12345678901234567890123456789012123456789012345678901234567890121234567890123456789012345678901212345678901234567890123
1234567890123456789012345678901212345678901234567890123456789012123456789012345678901234567890121234567890123
1234567890123456789012345678901212345678901234567890123456789012123456789012345678901234567890121234
123456789012345678901234567890121234567890123456789012345678901212
12345678901234567890123456789012123456789012345678901234567890121234567890123456789012345678901212345678901234567890123456789012123
YD

12345678901234567890123456789012123456789012345678901234567890121234567890123456789012345678901212345678901234567890123
1234567890123456789012345678901212345678901234567890123456789012123456789012345678901234567890121234567890123
1234567890123456789012345678901212345678901234567890123456789012123456789012345678901234567890121234
123456789012345678901234567890121234567890
12345678901234567890123456789012123456789012345678901234567890121234567890123456789012345678901212345678901234567890123456789012123
123456789012345678901234567890121234567890123456789012345678901212
PRACTICE TEST on TYPE - VI
12345678901234567890123456789012123456789012345678901234567890121234567890123456789012345678901212345678901234567890123
1234567890123456789012345678901212345678901234567890123456789012123456789012345678901234567890121234567890123
1234567890123456789012345678901212345678901234567890123456789012123456789012345678901234567890121234
12345678901234567890123456789012123456789012345678901234567890121234567890123456789012345678901212345678901234567890123456789012123
123456789012345678901234567890121234567890123456789012345678901212
12345678901234567890123456789012123456789012345678901234567890121234567890123456789012345678901212345678901234567890123
1234567890123456789012345678901212345678901234567890123456789012123456789012345678901234567890121234567890123
1234567890123456789012345678901212345678901234567890123456789012123456789012345678901234567890121234
123456789012345678901234567890121234567890
12345678901234567890123456789012123456789012345678901234567890121234567890123456789012345678901212345678901234567890123456789012123
12345678901234567890123456789012123456789012345678901234567890121234567890123456789012345678901212345678901234567890123456789012123
12345678901234567890123456789012123456789012345678901234567890121234567890123456789012345678901212345678901234567890123
1234567890123456789012345678901212345678901234567890123456789012123456789012345678901234567890121234567890123
1234567890123456789012345678901212345678901234567890123456789012123456789012345678901234567890121234
123456789012345678901234567890121234567890123456789012345678901212
12345678901234567890123456789012123456789012345678901234567890121234567890123456789012345678901212345678901234567890123456789012123
1234567890123456789012345678901212345678901234567890123456789012123456789012345678901234567890121234567890123
1234567890123456789012345678901212345678901234567890123456789012123456789012345678901234567890121234
,H

H (5 - 8) XH› HË_£ ƒì+¬…Õ 'hope to see you' #° 're so


T

1. DEAF = 32 J~ò# LEAF = ?


AM

ÿ
H

(Central Bank of India - 2008) na di' Qê#° 'Please come to see the party' #°
M

1) 24 2) 16 3) 48 4) 32 'fi ge na di ke zo' Qê#°, 'hope to come' #° 'di


AM
IG

2. XH› ‹¬OöH`ǃì+¬…Õ ADVENTURE 䛽 BFYISZBZN so ge' Qê#° =°i†Çò 'see you the party' #° 're fi
KH

Qê HË_£K͋²# J^Í ‹¬OöH`ǃì+¬…Õ COUNTRY †³òH›ø zo na' Qê#¶ HË_£ K͋²#, D ãH÷Ok (5-8) ㄬ‰×߁䛽
R

HË_£ Uq°\÷? ‹¬=¶^¥<Œ° W=ÞO_.


1) DQXRYXF 2) DXQRXYF (RBI Grade B officer - 2011)
3) EXQRYFX 4) DQXFYXF 5. P HË_£ ƒì+¬…Õ Please †³òH›ø HË_£
3. XH› ‹¬OöH`Ç ƒì+¬…Õ GONE #° 5@9© Qê =°i†Çò 1) fi 2) ke 3) di 4) na
SEAL#° 69%* Qê HË_£K͋²# J^Í ‹¬OöH`Ç ƒì+¬…Õ 6. P HË_£ ƒì+¬…Õ 'So di re' J#Qê
LOGS †³òH›ø HË_£ Uq°\÷? (Ind. Bank Cl. 2010) 1) the hope of 2) you hope to
1) %5*6 2) 5*@6 3) ©56* 4) *@56
3) hope you please 4) you see hope
4. XH› HË_£ ƒì+¬…Õ MARATHA #° NCUEYNH Qê ~Œ‹²#
7. P HË_£ ƒì+¬…Õ 'Please see you' †³òH›ø HË_£ ~¡¶„¬O
J^Í HË_£ ƒì+¬…Õ ZAMBIA#° U q^ÎOQê ~Œªë~¡°?
1) so re na 2) na di ke
1) ABPGNF 2) ACPFNG
3) re na ke 4) zo re ne
3) ACPGND 4) ACPFNH

REASONING ABILITY (VERBAL) ○ ○ ○ ○ ○ ○ ○ ○ ○


17
○ ○ ○ ○ ○ ○ ○ ○ ○ ○ ○ ○ ○ ○ ○ ○ ○ ○ ○
Study Material
8. P HË_£ ƒì+¬…Õ 'Party' †³òH›ø HË_£ 19. XH› HË_£ ƒì+¬…Õ TRANSMISSION #° RTANM
1) re …è^¥ fi 2) na …è^¥ zo SISISONQê ~Œ‹²# PHË_£ ƒì+¬…Õ COMMUNICA-
3) ke …è^¥ fi 4) zo …è^¥ fi TIONS #° U q^ÎOQê ~Œªë~¡°?
9. Water #° food Qê, food #° tree Qê, tree #° 1) OCMNMUICTIAONS
skyQê, sky #° well Qê, well #° pond Qê HË_£K‹Í #² , 2) OCNMUNICTANSOI
P HË_£ ƒì+¬…Õ „¬O_È°Á g\÷H÷ Hꪐë~ò? 3) OCMMNUICTAIOSN
1) tree 2) sky 4) OCMMNUICTAOINS
3) well 4) pond 20. XH› HË_£ ƒì+¬…Õ WORK#° 0918Qê ROUND#°
10. XH› HË_£ ƒì+¬…Õ PRAMOD#° ARPDOMQê HË_£K‹ Í #
²
19354Qê KIND#° 8654Qê BRING #° 71652Qê
J^Í HË_£ ƒì+¬…Õ KESHAV#° U q^ÎOQê HË_£ Kͪë~¡°?
BROOD #° 71994Qê~Œ‹²# '2Ñ JOïH#° ‹¬¶zOKÇ°
1) SEKVAH 2) SEKAVH
JH›Æ~¡O

Y
3) SKEVAH 4) SKEWAH
1) R 2) G 3) N 4) B
11. BAD#° 7Qê, HIS#° 9Qê HË_£K͋²# P HË_£ ƒì+¬…Õ

M
21. 'man box fox' J#Qê 'green and blue', 'sic vic
LOW †³òH›ø HË_£ ‹¬OY¼
bic' J#Qê 'Red or White' ; 'box kri bic' J#Qê

E
1) 5 2) 6 3) 8 4) 16
12. XH› HË_£ ƒì+¬…Õ BARK#° 0735Qê DIRT#° 4927Qê 'Yellow or Blue' J~ò`Í 'Blue' #° ‹¬¶zOKÍ Code

D
9
59
WAGE#° 4086Qê =°i†Çò RISK#° 5718Qê ~Œ‹²# Uk ?

CA
95
P HË_£ ƒì+¬…Õ R#° ‹¬¶zOKÇ° JOïH. 1) fox 2) bic 3) box 4) kri
22. 'kog li nfg' J#Qê 'Balu acts good' ; 'kung

50
1) 8 2) 1 3) 5 4) 7
13. RESULT#° 798206Qê HË_£ K͋²# LET †³òH›ø HË_£ cung norg' J#Qê 'he is stout' ; 'norg li bri'
98
A
J#Qê 'Balu seems stout' ; 'ghi Zig norg' J#Qê
:8
1) 806 2) 076 3) 096 4) 078
E

14. XH› ‹¬OöH`Ç ƒì+¬…Õ LIBERATE #° 56403170Qê HË_£ 'Lean and stout' J~ò`Í stout #° ‹¬¶zOKÍ Code
h
.P

K͋²#, P ‹¬OöH`ǃì+¬…Õ TRIBAL †³òH›ø HË_£


IC

Uk ?
AD

1) 716435 2) 736415 1) cung 2) li 3) Zig 4) norg


3) 736405 4) 736105
AB
O

23. 'Rum Zim bram' J#Qê 'Laxman teaches well';


15. w #° a Qê, s #° r Qê =°i†Çò r #° w Qê HË_£ K͋²# 'whi med bee' J#Qê 'Ravi is drinking' ; 'Zim
ER
CH

answer#° D q^ÎOQê ~Œ†Ç°=KÇ°ó#°. pin med' J#Qê 'Laxman and Ravi' J~ò`Í
YD

1) answer 2) anraew Laxman #° ‹¬¶zOKÍ Code Uk ?


,H

3) anarew 4) anrawe 1) Med 2) Rum 3) Zim 4) Bee


T
AM

16. XH› ‹¬OöH`ǃì+¬…Õ '357Ñ J#Qê 'get me toy' Jx†Çò 24. 'Chin shan chou' J#Qê 'Lavanya looks me' ;
H

'843Ñ J#Qê 'bring good toy' Jx†Çò '746Ñ J#Qê


M

'Chou min tin win' J#Qê 'Gopal saw lavanya


AM

'bring me water' Jx†Çò J~¡ÖO J~ò# P ‹¬OöH`Ç


IG

yesterday' ; 'Rid sal min' J#Qê 'Raman called


ƒì+¬…Õ 'good' #° ‹¬¶zOKÇ° ‹¬OY¼
KH

Gopal' J~ò`Í Gopal #° ‹¬¶zOKÍ Code Uk ?


1) 5 2) 7 3) 8 4) 4
R

1) tin 2) Chou 3) win 4) min


17. XH› ‹¬OöH`Ç ƒì+¬…Õ „¬‹¬°„¬ô#° Z~¡°„¬ôQê, `³°„¬ô#°
25. 'fnm lrt odp' J#Qê 'I am seeing' ; 'ptm nro
P䛽„¬KóÇ Qê, Z~¡°„¬ô#° <ŒiO[~¡OQ®°Qê, hO#° `³°„¬ôQê
=°i†Çò P䛽„¬KóÇ #° hOQê „¬iQ®}O ÷ z# P HË_£ ƒì+¬…Õ dlf' J#Qê 'He has come' ; 'lrt pac nro' J#Qê
PHê‰×O †³òH›ø ~¡OQ®°? 'He is seeing' J~ò`Í He #° ‹¬¶zOKÍ Code Uk?
1) `³°„¬ô 2) Z~¡°„¬ô 1) Ptm 2) lst 3) nro 4) det
3) „¬‹¬°„¬ô 4) hO 26. 'Srt dih krm' J#Qê 'It was decided' ; 'Tdk
18. PAPER #° QBQFS Qê HË_£K͋²# GLASS †³òH›ø rin tmh' J#Qê 'I had seen' ; 'dih rin zfg' J#Qê
HË_£ 'I was appointed' J~ò`Í appointed #° ‹¬¶zOKÍ
1) MHTBT 2) MBHTT Code Uk ?
3) BHMTT 4) HMBTT 1) Krm 2) rin 3) dih 4) Zfg

REASONING ABILITY (VERBAL) ○ ○ ○ ○ ○ ○ ○ ○ ○


18
○ ○ ○ ○ ○ ○ ○ ○ ○ ○ ○ ○ ○ ○ ○ ○ ○ ○ ○
Study Material
12345678901234567890123456789012
12345678901234567890123456789012123456789012345678901234567890121234567890123456789012345678901212345678901234567890123
12345678901234567890123456789012

q=~¡}ì`ÇàH› ["Œ|°°
12345678901234567890123456789012123456789012345678901234567890121234567890123456789012345678901212345678901234567890123
12345678901234567890123456789012 12345678901234567890123456789012
12345678901234567890123456789012123456789012345678901234567890121234567890123456789012345678901212345678901234567890123
12345678901234567890123456789012 12345678901234567890123456789012
12345678901234567890123456789012
12345678901234567890123456789012123456789012345678901234567890121234567890123456789012345678901212345678901234567890123
12345678901234567890123456789012
12345678901234567890123456789012123456789012345678901234567890121234567890123456789012345678901212345678901234567890123
12345678901234567890123456789012 12345678901234567890123456789012
12345678901234567890123456789012123456789012345678901234567890121234567890123456789012345678901212345678901234567890123
12345678901234567890123456789012 12345678901234567890123456789012
12345678901234567890123456789012123456789012345678901234567890121234567890123456789012345678901212345678901234567890123
12345678901234567890123456789012 12345678901234567890123456789012

1. (3) POQ®Á=~¡¾=ò…Õx JH›Æ~Œ †³òH›ø ‹¬OY¼ ㄬHê~¡O 7. (3) please see you †³òH›ø HË_£ ke na re ®
D = 4, E= 5, A + 1, F= 6 re na ke ®
DEAF = (4 + 5 + 1 + 6) × 2 = 16 × 2 = 32 8. (4) party †³òH›ø HË_£ zo (…è^¥) fi ®
J^Íq^ÎOQê L = 12, E = 5, A= 1, f=6 9. (2) „¬O_È°Á 'tree' H÷ Hꪐë~ò. Hêx HË_£ ㄬHê~¡O
®ÿ
LEAF (12 + 5 + 1 + 6) × 2 = 24 × 2 = 48 'trees' J#Qê 'sky'
2. (1) " % 7 & / 5 6 3 &
\
„¬O_È°Á 'sky' H÷ Hꪐë~ò
10. (3) 1 3 " . 0 %
        

# ' : * 4 ; # ; /
" 3 1 % 0 .

Y
J^Íq^ÎOQê
J^Íq^ÎOQê , & 4 ) " 7
$ 0 6 / 5 3 :

M
      

E
% 2 9 3 : 9 ' 4 & , 7 " )
\ J^Íq^ÎOQê ® DQXRYXF \ KESHAV †³òH›ø HË_£ ® SEKVAH

D
9
59
11. (1) B A D HIS
3. (4) ( 0 / & 4 & " -
CA
95
2 + 1 + 4 = 07 8 + 9 + 19 = 36
=0+7=7 Þ 3+6 9Þ

50
 !  ©    + LOW
98
A
̄á "Œ\÷ #°O_ Þÿ50 Þÿ5 + 0 Þÿ5
12 + 15 + 23
:8
- 0 ( 4 12. (4) # "3 , % * 3 5 8"( & 3 * 4 ,
E
h
.P
IC

+ !          


AD

\ ® H@56
LOGS †³òH›ø HË_£ BARK, DIRT =°i†Çò RISK =ü_\ ÷…Õ R LOk.
AB
O

4. (2) . " 3 " 5 ) " =°i†Çò 7 䛀_¨ =ü_\ ÷…Õ LOk.


Hê=ô# R †³òH›ø HË_£ 7.
ER
CH

      
13. (3) 3 &46-5
/ $ 6 & : / )
YD
,H

J^Íq^ÎOQê ; " . # * "  


T
AM

      ̄á ^¥x #°O_ - & 5


H
M

" $ 1 ' / (   
AM

5. (2)
IG

14. (2) - * # & 3 " 5 &


KH

IPQF UP TFF ZPV SF TP OB EJ


      
R

GJ HF OB EJ LF [P
QMFBTF DPNF UP TFF UIFQBSUZ ̄á ^¥x #°O_ 5 3 * # " -
IPQF UP DPNF EJ TP HF

TFF ZPV GJ[P OB  


UIFQBSUZ SF
15. (2) HË_£ ƒì+¬ ㄬHê~¡O w ® a ; s ® r; r ®w
̄á "ŒH꼁 #°O_ B O TXF S
Please †³òH›ø HË_£ = ke
®
the party – fi zo to – di ® B O S B FX
®
see – na you – re ® 16. (3)    HFUNFUPZ
®
hope – so come – ge ®    CSJOHHPPEUPZ
®
please – ke
6. (2) so di re ®
hope to you
   CSJOHNFXBUFS
ÿÿÿÞ
you hope to ̄á "ŒH꼁 #°O_ 'good' #° ‹¬¶zOKÇ° JOïH ® 8

REASONING ABILITY (VERBAL) ○ ○ ○ ○ ○ ○ ○ ○ ○


19
○ ○ ○ ○ ○ ○ ○ ○ ○ ○ ○ ○ ○ ○ ○ ○ ○ ○ ○
Study Material
17. (1) PHê‰×O †³òH›ø ~¡OQ®° 'hOÑ 22. (4) kog -J nfg ® #BMV acts good
Hêx HË_£ ƒì+¬ ㄬHê~¡O hO J#Qê `³°„¬ô
Hê=ô# PHê‰×O †³òH›ø ~¡OQ®° '`³°„¬ôÑ
Kung Cung OPSH ® he is TUPVU

18. (4) 1 " 1 & 3 OPSH -J bri ® #BMV seems TUPVU

     ghi zig OPSH ® Lean and TUPVU


2 # 2 ' 4 ̄á "Œ\÷#°O_
J^Íq^ÎOQê stout †³òH›ø HË_£ ® norg.
( - " 4 4 23. (3) Rum [JN bram ® -BYNBO teaches well
whi NFE bee ®ÿ 3BWJ is drinking
    
) . # 5 5 [JN pin NFE ®ÿ -BYNBO and 3BWJ
\ ̄á "ŒH꼁 #°O_

Y
19. (3) 5 3" / 4 .  4 4 * 0 /
Laxman †³òH›ø HË_£ ® zim.

M
3 5" / . 4 * 4 * 4 0 / 24. (4) Chin shan DIPV ® -BWZOB looks me

E
J^Íq^ÎOQê DIPV NJO tin win ® (PQBM saw

D
9
$ 0. . 6 / * $ " 5 * 0 / 4

59
-BWZOB yesterday
CA
95
0 $. . / 6 * $ 5 " * 0 4 / Rid sal NJO ® Raman called (PQBM

50
\ ̄á "ŒH꼁 #°O_
803 , 3 06 / % , * / % 98
A
20. (2) Gopal †³òH›ø HË_£ ® min.
:8

     ® I am TFFJOH


E

25. (3) fnm odp


h

MSU
.P

def ÿ®ÿ )F has come


IC

#3 * /( # 300 % ptm OSP


AD

pac OSP ®ÿ )F is TFFJOH


AB
O

MSU
    
\ '2Ñ #° ‹¬¶zOKÇ° JH›Æ~¡O ® 'G' \ ̄á "ŒH꼁 #°O_
ER
CH

He †³òH›ø HË_£ ® nro.


man CPY fox ÿ® green and CMVF
YD

21. (3)
26. (4) Srt EJI krm ® It XBT decided
sic vic CJD ®ÿ Red PS white
,H

Tdk SJO tmh ®ÿ had seen


T
AM

CPY kri CJD ® yellow PS #MVF EJI SJO zfg ® XBT appointed
H

*
M

̄á "Œ\÷#°O_ \ ̄á "ŒH꼁 #°O_


AM
IG

Blue †³òH›ø HË_£ ® box. appointed †³òH›ø HË_£ ® zfg.


KH

TYPE - VII : q‰õÁ+¬} HË_OQ·


R

L^¥ : ãH÷O^Î W=Þ|_# ï~O_È° Hê"£°…Õ XH› ^¥x…Õ POQ®Á $PMVNO * $PMVNO  **
JH›Æ~¡=¶ ̄^Îí JH›Æ~¡=¶…Õ, ï~O_È= Hê"£°…Õ z#ß  406/% B
BCJ
JH›~Æ =
¡ ¶…Õ W=Þ|_#k. J~ò`Í Hê"£°-1…Õx „¬^¥xH÷ . "%%3&44 C
DKNW
 $369 D
JLNPQ
Z^ΰ~¡°Qê L#ß „¬^Î"Í° HË_£ Hê"Œež# x†Ç°=°O …è^ΰ.
4. /&5 E
JKLUW
J~ò`Í U „¬^¥xß U „¬^ÎO HË_£ JQ®°<Ë H›#°Qùx, qq^Î  $30/: F
KLHPUW
JH›Æ~Œä›½ 䛀_¨ HË_£ H›x̄\÷“ ãH÷O^Î W=Þ|_# ㄬ‰×߁䛽  $308%: G
CMPPQQW
["Œ|°° ~Œ†Ç°O_. 1. 'A'JH›Æ~ŒxH÷ HË_£ Uk ?
1) b 2) l 3) v 4) UnHê^ΰ
REASONING ABILITY (VERBAL) ○ ○ ○ ○ ○ ○ ○ ○ ○
20
○ ○ ○ ○ ○ ○ ○ ○ ○ ○ ○ ○ ○ ○ ○ ○ ○ ○ ○
Study Material
2. 'C' JH›Æ~ŒxH÷ HË_£ Uk ? g\÷…Õ Hê=°<£Qê L#ß JH›Æ~Œä›½ HË_£ H›#°Qù<Í q^¥#O
1) j 2) k 3) l 4) UnHê^ΰ ^¥Þ~Œ Jxß JH›Æ~Œä›½ HË_£ H›#°Qù#=KÇ°ó.
3. 'D ' JH›Æ~ŒxH÷ HË_£ Uk ?
NET =°i†Çò ADDRESS J#° „¬^¥…Õ E J#°
1) k 2) l 3) m 4) UnHê^ΰ
Letter Hê=°<£Qê LOk. =°i†Çò "Œ\÷ HË_£° abi
4. 'N' JH›Æ~ŒxH÷ HË_£ Uk ?
1) a 2) e 3) q 4) UnHê^ΰ =°i†Çò blooppv …Õ Hê=°<£Qê LO_Í UïHáH› JH›Æ~¡O 'b'
5. 'O' JH›Æ~ŒxH÷ HË_£ Uk ? J<Ík 'e' H÷ HË_£Qê LOk. D q^ÎOQê ㄬu JH›Æ~Œxß
1) i 2) j 3) k 4) UnHê^ΰ H›#°Qù#=KÇ°ó#°.
ª^Î# : W@°=O\÷ ㄬ‰×߁° ªkOKÇ°@䛽 Qê#° =òO^ΰQê U
IÌ`¤
 J C B W K N D L U P H Q *
„¬^¥eß U „¬^ÎO HË_£Qê ~Œ†Ç°|_O^Ë H›#°Qù<Œe.
KIœÇ¢P / & 5 3 $ 6 9 0 : % 8 4 "
„¬^ÎO…Õx JH›Æ~Œ ‹¬OY¼ WO^ΰ䛽 L„¬†³¶Q®„¬_È°`Ç°Ok.
NET Þ abi H›#°H› 1= ㄬ‰×ß䛽 A Þ I

Y
CRUX Þ cjmv 2= ㄬ‰×ß䛽 C Þ j
Þ

M
CRONY ijktv
3= ㄬ‰×ß䛽 D Þ o
SOUND Þ ikmop
4= ㄬ‰×ß䛽 N Þ i
Þ

E
5= ㄬ‰×ß䛽 O Þ k ° ‹¬=¶^¥#=ò°.
CRPWDY jkgotv
Þ

D
9
ADDRESS blooppv

59
123456789012345678901234567890121234567890123456789012345678901212
12345678901234567890123456789012123456789012345678901234567890121234567890123456789012345678901212345678901234567890123
1234567890123456789012345678901212345678901234567890123456789012123456789012345678901234567890121234567890123
1234567890123456789012345678901212345678901234567890123456789012123456789012345678901234567890121234
123456789012345678901234567890121234567890123456789012345678901212
12345678901234567890123456789012123456789012345678901234567890121234567890123456789012345678901212345678901234567890123456789012123
1234567890123456789012345678901212345678901234567890123456789012123456789012345678901234567890

CA
12345678901234567890123456789012123456789012345678901234567890121234567890123456789012345678901212345678901234567890123
1234567890123456789012345678901212345678901234567890123456789012123456789012345678901234567890121234567890123
1234567890123456789012345678901212345678901234567890123456789012123456789012345678901234567890121234
123456789012345678901234567890121234567890123456789012345678901212

95
12345678901234567890123456789012123456789012345678901234567890121234567890123456789012345678901212345678901234567890123456789012123
1234567890123456789012345678901212345678901234567890123456789012123456789012345678901234567890
12345678901234567890123456789012123456789012345678901234567890121234567890123456789012345678901212345678901234567890123
1234567890123456789012345678901212345678901234567890123456789012123456789012345678901234567890121234567890123
1234567890123456789012345678901212345678901234567890123456789012123456789012345678901234567890121234
12345678901234567890123456789012123456789012345678901234567890121234567890123456789012345678901212345678901234567890123456789012123
1234567890123456789012345678901212345678901234567890123456789012123456789012345678901234567890
TEST YOURSELF - 2
123456789012345678901234567890121234567890123456789012345678901212
12345678901234567890123456789012123456789012345678901234567890121234567890123456789012345678901212345678901234567890123
1234567890123456789012345678901212345678901234567890123456789012123456789012345678901234567890121234567890123
1234567890123456789012345678901212345678901234567890123456789012123456789012345678901234567890121234
12345678901234567890123456789012123456789012345678901234567890121234567890123456789012345678901212345678901234567890123456789012123
1234567890123456789012345678901212345678901234567890123456789012123456789012345678901234567890
12345678901234567890123456789012123456789012345678901234567890121234567890123456789012345678901212345678901234567890123
1234567890123456789012345678901212345678901234567890123456789012123456789012345678901234567890121234567890123
1234567890123456789012345678901212345678901234567890123456789012123456789012345678901234567890121234
123456789012345678901234567890121234567890123456789012345678901212
12345678901234567890123456789012123456789012345678901234567890121234567890123456789012345678901212345678901234567890123456789012123
1234567890123456789012345678901212345678901234567890123456789012123456789012345678901234567890
12345678901234567890123456789012123456789012345678901234567890121234567890123456789012345678901212345678901234567890123
1234567890123456789012345678901212345678901234567890123456789012123456789012345678901234567890121234567890123
1234567890123456789012345678901212345678901234567890123456789012123456789012345678901234567890121234
12345678901234567890123456789012123456789012345678901234567890121234567890123456789012345678901212345678901234567890123456789012123
1234567890123456789012345678901212345678901234567890123456789012123456789012345678901234567890
123456789012345678901234567890121234567890123456789012345678901212
12345678901234567890123456789012123456789012345678901234567890121234567890123456789012345678901212345678901234567890123456789012123
1234567890123456789012345678901212345678901234567890123456789012123456789012345678901234567890
12345678901234567890123456789012123456789012345678901234567890121234567890123456789012345678901212345678901234567890123
1234567890123456789012345678901212345678901234567890123456789012123456789012345678901234567890121234567890123
1234567890123456789012345678901212345678901234567890123456789012123456789012345678901234567890121234

50
98
A
1. '…ÿá\˜Ñx '=¶ißOQ·Ñ Jx, '=¶ißOQ·Ñx '_¨~ŸøÑ Jx, '_¨~ŸøÑx 6. COLLEGE †³òH›ø HË_£ AMJJCEC J~ò#KË
:8
'<³á\˜Ñ Jx, '<³á\˜Ñx '‹¬<£Ì+á<£Ñ Jx '‹¬<£Ì+á<£Ñx '_ȋ¹øÑ Jx UNIVERSITY †³òH›ø HË_£ (Group-IV - 2012)
E
h

1) JOVWFSTJUZ 2) SLGTCPQGRW
.P

JO>è, =°#O Z„¬C_È° xãkªëO?


IC

(Communication S.I. - 2012) 3) JOVWFSTJUZ 4) SLGTCPQRWG


AD

1) _ȋ¹ø 2) <³á\˜ 7. ZIP = 198 =°i†Çò ZAP = 246 J~ò# VIP =


AB
O

(S.I. - 2012)
3) ‹¬<£Ì+á<£ 4) _¨~Ÿø
ER
CH

1) 248 2) 342 3) 212 4) 222


2. TRUTH x SUQSTVSUGI Qê HË_£ KÍÀ‹ë FALSE †³òH›ø
YD

8. XH› ‹¬OöH`Ç ƒì+¬…Õ 'Kew Xas Huma deko' J#Qê


HË_£ (Communication S.I. - 2012)
,H

'She is eating apples' Jx J~¡OÖ . 'Kew tepo qua'


T

1) GHBCMNTUFG 2) EGZBKMTRDF J#Qê 'She sells toys' Jx J~¡ÖO =°i†Çò 'Sul lim
AM
H

3) DEZYTKQRCD 4) EGZBKMRTDF deko' J#Qê 'I like apples' Jx J~¡OÖ J~ò# 'She'
M

3. DRIVER = 12; PEDESTRIAN = 20, ACCIDENT =°i†Çò 'apples'#° ‹¬¶zOKÇ° HË_£° =~¡°‹¬Qê
AM
IG

= 16 J~ò# CAR = ? (Communication S.I. - '12) (S.I. - 2012)


KH

1) 8 2) 10 3) 6 4) 12 1) xas =°i†Çò kew 2) kew =°i†Çò deko


R

4. PRABA #° 27595Qê =°i†Ç ò THILAK #° 3) kew =°i†Çò kas 4) xas =°i†Çò deko
368451Qê HË_£K͋²# BHARATI †³òH›ø HË_£ Uq°\÷? 9. SYSTEM x SYSMET Qê NEARER x AENRERQê
~Œ‹²#@Á~ò# FRACTION x Z…ì ~Œ†Ç°=KÇ°ó?
(Dy. Jailors - 2012)
(S.I. - 2012)
1) 9657538 2) 9658537
1) FRATIOCN 2) FARCTION
3) 9675853 4) 9676865
3) CARFNOIT 4) CARFIONT
5. Table †³òH›ø HË_£ WDEOH J~ò#KË, BOARD 10. ZEBRA x 52102362 Qê ~ŒÀ‹ë COBRA #° Uq^O Î Qê
†³òH›ø HË_£ (Group-IV - 2012) ~Œ†Ç°=KÇ°ó#°? (S.I. - 2012)
1) EDRUG 2) DRUGE 1) 6302362 2) 6023362
3) ERDUG 4) EGURD 3) 6320362 4) 6230362

REASONING ABILITY (VERBAL) ○ ○ ○ ○ ○ ○ ○ ○ ○


21
○ ○ ○ ○ ○ ○ ○ ○ ○ ○ ○ ○ ○ ○ ○ ○ ○ ○ ○
Study Material
11. XH› HË_£ ƒì+¬…Õ 123 JO>è 'bright little boy' 16. XH› HË_£ ƒì+¬…Õ REFRIGERATOR x
Jx†Çò, 145 JO>è 'tall big boy' =°i†Çò 637 ROTAREGIRFER Qê ~ŒÀ‹ë NOITINUMMA †³òH›ø
JO>è 'beautiful little flower' Jx†Çò J~¡O
Ö J~ò# _™HË_£ ~¡¶„¬O Uk? (Communication S.I. - 2012)
P ƒì+¬…Õ 'bright' #° ‹¬¶zOKÇ° JOïH 1) AMMONITION 2) AMMUNITION
(Dy. Jailors - 2012) 3) COMMUNICATION 4) AMMOMIUTNI
17. REASON x '6Ñ Qê HË_£ KÍÀ‹ë, BELIEVED x '8ÑQê
1) 4 2) 2 3) 1 4) 3
HË_£ KÍÀ‹ë 'GOVERNMENT' †³òH›ø HË_£ ZO`Ç?
12. FAT#° 6 + 1 + 20 KÍ ‹¬¶zOz#KË LEAN #° nx
(Dy. Jailors - 2012)
^¥Þ~Œ ‹¬¶zOKÇ=KÇ°ó#°. (Group-IV - 2012)
1) 12
2) 8 3) 9 4) 10
1) 12 + 5 + 1 = 13 2) 11 + 5 + 1 = 14
18. Dust #° air Jx, air #° fire Jx, fire #° water
3) 12 + 5 + 1 + 14 4) 13 + 5 + 1 + 13
Jx, water x colour Jx, colour x rain Jx,

Y
13. XH› HË_£ ƒì+¬…Õ TWINKLY x SVHOJKX Qê ~ŒÀ‹ë rain x dust Jx „²ez#, K̈́¬° x=tOKÍ ã„¬^͉×O

M
FILTERS †³òH›ø HË_£ ~¡¶„¬O Uk? (S.I. - 2011)
(Communication S.I. - 2012)

E
1) Rain 2) Colour
1) EHKSDQR 2) EGKUDQR

D
9
3) Water 4) dust

59
3) EKUHQDR 4) EHKUDQR 19. DELHI x CCIDD Qê HË_£ KÍÀ‹ë BOMBAY †³òH›ø
CA
95
14. MATHEMATICS = 12345123678 J~ò`Í HË_£ (Constables - 2012)

50
MAHATMA U=°=ô`Ç°Ok? (Constables - 2012) 1) AMJXVS 2) AJMTVS
98
A
1) 1243221 2) 1242131 3) MJXVSA 4) WAJVXM
:8
20. BOX #° CDPQYZ KÍ ‹¬¶zÀ‹ë HERO #° ‹¬¶zOKÍ
E

3) 1242512 4) 1242321
h

„¬^ÎO…Õx z=i ï~O_È° JH›Æ~Œ° Uq? (S.I. - 2011)


.P

15. A = 26, SUN = 27 J~ò# CAR = ?


IC
AD

(Communication S.I. - 2012) 1) M =°i†Çò N 2) N =°i†Çò M


3) P =°i†Çò Q 4) M =°i†Çò N
AB
O

1) 47 2) 53 3) 57 4) 43
ER
CH

123456789012345678901
12345678901234567890123456789012123456789012345678901234567890121234567890123456789012345678901212345678901234567890123
1234567890123456789012

q=~¡}ì`ÇàH› ["Œ|°°
12345678901234567890123456789012123456789012345678901234567890121234567890123456789012345678901212345678901234567890123
123456789012345678901
123456789012345678901 1234567890123456789012
1234567890123456789012
12345678901234567890123456789012123456789012345678901234567890121234567890123456789012345678901212345678901234567890123
TEST YOURSELF - 2 :
YD

123456789012345678901
12345678901234567890123456789012123456789012345678901234567890121234567890123456789012345678901212345678901234567890123
123456789012345678901 1234567890123456789012
123456789012345678901 1234567890123456789012
12345678901234567890123456789012123456789012345678901234567890121234567890123456789012345678901212345678901234567890123
1234567890123456789012
12345678901234567890123456789012123456789012345678901234567890121234567890123456789012345678901212345678901234567890123
123456789012345678901 1234567890123456789012
12345678901234567890123456789012123456789012345678901234567890121234567890123456789012345678901212345678901234567890123
,H

1. (3) =°#O '<³á\˜Ñ xãkªëO. Hêx ‹¬OöH`Ç ƒì+¬…Õ <³á\˜#°


T

PEDESTRIAN „¬^ÎO…Õx JH›Æ~Œ ‹¬OY¼ þ 2


AM

‹¬<£Ì+á<£ Jx „²°‹¬°ë<Œß~¡°.
H

« 10 þ 2 « 20
M

\ Hê=‹²# ‹¬=¶^¥#O ‹¬<£Ì+á<£.


AM

ACCIDENT „¬^ÎO…Õx JH›Æ~Œ ‹¬OY¼ þ 2


IG

2. (4) T R U T H „¬^ÎO…Õx Jxß JH›Æ~Œ =òO^ΰ =°i†Çò « 8 þ 2 « 16


KH

"³#°H› JH›~Æ Œ#° H›°„¬Qê SUQSTVSUGI U~¡æ_#k.


R

J^Íq^ÎOQê CAR „¬^ÎO…Õx JH›Æ~Œ ‹¬OY¼ þ 2


5 3 6 5 ) « 3 þ 2 « 6.
46 24 57 46 ()
4. (1) 1 3 " # " 5 ) * - ",
J^Íq^ÎOQê FALSE „¬^ÎO…Õx Jxß JH›Æ~Œ =òO^ΰ           
=°i†Çò "³#°H› „¬^¥#° H›°„¬Qê,
# ) " 3 " 5 *
' " - 4 &       
&( ;# ,. 35 %'
\ Hê=‹²# „¬^ÎO « EGZBKMRTDF 5. (3) 5 " # - & # 0 "3 %
         
3. (3) DRIVER „¬^ÎO…Õx JH›Æ~Œ ‹¬OY¼ þ2 « 6 þ 2 8% &0 ) & 3 %6 (
« 12

REASONING ABILITY (VERBAL) ○ ○ ○ ○ ○ ○ ○ ○ ○


22
○ ○ ○ ○ ○ ○ ○ ○ ○ ○ ○ ○ ○ ○ ○ ○ ○ ○ ○
Study Material
6. (2) $ 0 - - & ( & $ 0 # 3 "
       





" . + + $ & $ ¨

¨

¨

¨


6 / * 7 & 3 4 * 5 :      
         

4 - ( 5 $ 1 2 ( 3 8 11. (2)    CSJHIU MJUUMF CPZ


7. (4) ZIP…Õx JH›Æ~Œ i=~Ÿž HË_£ "³ò`ÇëO
« 1 { 18 { 11 « 30    UBMM CJH CPZ
(30 { 3) þ 6 « 33 þ 6 « 198    CFBVUJGVM MJUUMF GMPXFS
J^Íq^ÎOQê ZAP…Õx JH›Æ~Œ i=~Ÿž HË_£ "³ò`ÇëO
̄á# KǶ„¬|_# ‹¬OöH`Œ #°O_ ‹¬æ+¬“OQê bright
1 { 26 { 11 « 38 Þÿ(38 { 3) þ 6

Y
#° ‹¬¶zOKÇ° ‹¬OY¼ 2.
« 41 þ 6 « 246
® 6 + 1 + 20.

M
J^Íq^ÎOQê VIP…Õx JH›Æ~Œ i=~Ÿž HË_£ "³ò`ÇëO 12. (3) ' " 5 J^Íq^ÎOQê




5 { 18 { 11 « (34 { 3) þ 6 « 37 þ 6 « 222.

E
8. (2)
- & " / ®ÿ12 + 5 + 1 + 14

D





9
59
,FX YBT )VNB EFLP 4IFJTFBUJOHBQQMFT 13. (4) 5 8 * / , - :
CA
95
      

,FX UFQP RVB 4IFTFMMTUPZT 4 7 ) 0 + , 9

50
J^Íq^ÎOQê ' * - 5 & 3 4
4VM MJN EFLP
98
A
*MJLFBQQMFT
      
:8
̄á# KǶ„¬|_# ‹¬OöH`Œ #°O_ kew - she #° & ) , 6 % 2 3
E
h

deko - apples #° ‹¬¶zOKÇ° HË_£°.


14. (3) . " 5 ) & . " 5
.P

* $ 4
IC

9. (3) 4:4
AD

5&.
          
AB
O

4:4 .&5 4:4.&5


. " ) " 5 . "
ER
CH

(=¼uö~H›k‰×) (=¼uö~H›k‰×)       
YD

/&" 3&3 15. (3) A 䛽 i=~Ÿž HË_£ - 26.


,H

"&/ 3&3 "&/3&3 A B C D E F G H I J K L M


T
AM

(=¼uö~H›k‰×) (=¼uö~H›k‰×) Z Y X W V U T S R Q P O N
H
M

J^Íq^ÎOQê SUN 䛽 i=~Ÿž HË_£ HFM


AM
IG

H F M ®
8 + 6 + 13 = 27. J^Í q^ÎOQê
KH

'3"$ 5*0/ (8) (6) (13)


R

$"3' /0*5 $"3'/0*5 CAT䛽 i=~ŸžHË_£ XZG ®


(=¼uö~H›k‰×) (=¼uö~H›k‰×) X Z G 24 + 26 + 7 = 57 ÿÿ®
(24) (26) (7)
10. (1) ; & # 3 "





16. (2) Wzó# „¬^¥xß =¼uö~H› k‰×…Õ ~Œª~¡°. Hê=ô#
¨ ¨ ¨ ¨
    NOITINUMMA †³òH›ø _™HË_£ ~¡¶„¬O -

      AMMUNITION J=ô`Ç°Ok.


17. (4) REASON „¬^ÎO…Õx JH›Æ~Œ ‹¬OY¼ -6.
ㄬu JH›Æ~¡O †³òH›ø ªÖ# q°=䛽 ï~\÷“O„¬ô HË_£
K͆°Ç |_#k. Hêx B †³òH›ø q°= =¶ã`ÇO '2ÑQê<Í BELIEVED „¬^ÎO…Õx JH›Æ~Œ ‹¬OY¼ -8.

HË_£ K͆ǰ|_#k. GOVERNMENT „¬^ÎO…Õx JH›Æ~Œ ‹¬OY¼ -10.

REASONING ABILITY (VERBAL) ○ ○ ○ ○ ○ ○ ○ ○ ○


23
○ ○ ○ ○ ○ ○ ○ ○ ○ ○ ○ ○ ○ ○ ○ ○ ○ ○ ○
Study Material
18. (2) K̈́¬° x=tOKÍ ã„¬^͉×O h~¡°. Hêx ‹¬OöH`Ç ƒì+¬…Õ 20. (3) B `Ç~¡°"Œ`Ç ï~O_È° JH›Æ~Œ° C, D
h~¡° (water)#° colour Qê HË_£ Kͪ~¡°. O `Ç~¡°"Œ`Ç ï~O_È° JH›Æ~Œ° P, Q
\ Hê=‹²# ‹¬=¶^¥#O Colour . X `Ç~¡°"Œ`Ç ï~O_È° JH›Æ~Œ° Y, Z
19. (1) % & - ) *
    
BOX ® CDPQYZ.
$ $ * % % J^Íq^OÎ Qê HERO #° ~Œ†Ç°Qê IJFGSTPQ =KÇ°ó#°.
J^Íq^ÎOQê # 0 . # " : \ z=i ï~O_È° JH›Æ~Œ° P, Q.
     

" . + 9 7 4
123456789012345678901234567890121234567890123456789012345678901212
12345678901234567890123456789012123456789012345678901234567890121234567890123456789012345678901212345678901234567890123
1234567890123456789012345678901212345678901234567890123456789012123456789012345678901234567890121234567890123
1234567890123456789012345678901212345678901234567890123456789012123456789012345678901234567890121234
12345678901234567890123456789012123456789012345678901234567890121234567890123456789012345678901212345678901234567890123456789012123
1234567890123456789012345678901212345678901234567890123456789012123456789012345678901234567890
123456789012345678901234567890121234567890123456789012345678901212
12345678901234567890123456789012123456789012345678901234567890121234567890123456789012345678901212345678901234567890123
1234567890123456789012345678901212345678901234567890123456789012123456789012345678901234567890121234567890123
1234567890123456789012345678901212345678901234567890123456789012123456789012345678901234567890121234
12345678901234567890123456789012123456789012345678901234567890121234567890123456789012345678901212345678901234567890123456789012123
1234567890123456789012345678901212345678901234567890123456789012123456789012345678901234567890
12345678901234567890123456789012123456789012345678901234567890121234567890123456789012345678901212345678901234567890123456789012123
1234567890123456789012345678901212345678901234567890123456789012123456789012345678901234567890
12345678901234567890123456789012123456789012345678901234567890121234567890123456789012345678901212345678901234567890123
1234567890123456789012345678901212345678901234567890123456789012123456789012345678901234567890121234567890123
1234567890123456789012345678901212345678901234567890123456789012123456789012345678901234567890121234
123456789012345678901234567890121234567890123456789012345678901212
PREVIOUS BITS OTHER EXAMS
12345678901234567890123456789012123456789012345678901234567890121234567890123456789012345678901212345678901234567890123456789012123
1234567890123456789012345678901212345678901234567890123456789012123456789012345678901234567890
12345678901234567890123456789012123456789012345678901234567890121234567890123456789012345678901212345678901234567890123
1234567890123456789012345678901212345678901234567890123456789012123456789012345678901234567890121234567890123
1234567890123456789012345678901212345678901234567890123456789012123456789012345678901234567890121234
123456789012345678901234567890121234567890123456789012345678901212

Y
12345678901234567890123456789012123456789012345678901234567890121234567890123456789012345678901212345678901234567890123456789012123
1234567890123456789012345678901212345678901234567890123456789012123456789012345678901234567890
12345678901234567890123456789012123456789012345678901234567890121234567890123456789012345678901212345678901234567890123
1234567890123456789012345678901212345678901234567890123456789012123456789012345678901234567890121234567890123
1234567890123456789012345678901212345678901234567890123456789012123456789012345678901234567890121234
12345678901234567890123456789012123456789012345678901234567890121234567890123456789012345678901212345678901234567890123456789012123
1234567890123456789012345678901212345678901234567890123456789012123456789012345678901234567890
123456789012345678901234567890121234567890123456789012345678901212
12345678901234567890123456789012123456789012345678901234567890121234567890123456789012345678901212345678901234567890123
1234567890123456789012345678901212345678901234567890123456789012123456789012345678901234567890121234567890123
1234567890123456789012345678901212345678901234567890123456789012123456789012345678901234567890121234
12345678901234567890123456789012123456789012345678901234567890121234567890123456789012345678901212345678901234567890123456789012123
1234567890123456789012345678901212345678901234567890123456789012123456789012345678901234567890
12345678901234567890123456789012123456789012345678901234567890121234567890123456789012345678901212345678901234567890123
1234567890123456789012345678901212345678901234567890123456789012123456789012345678901234567890121234567890123
1234567890123456789012345678901212345678901234567890123456789012123456789012345678901234567890121234
12345678901234567890123456789012123456789012345678901234567890121234567890123456789012345678901212345678901234567890123456789012123
1234567890123456789012345678901212345678901234567890123456789012123456789012345678901234567890

M
1. XH› HË_£ ƒ ì +¬ … Õ 'NUMBER' J#° „¬ ^ Î = ò#° 7. 'ãw<£Ñ #° 'ï~_£Ñ Jx, 'ï~_£Ñ #° '|¶ÁÑ Jx, '|¶ÁÑ x '"³á\˜Ñ
'156897' Qê#°, 'BARREN' J<Í „¬ ^ Î = ò#°

E
Jx, '"³á\˜Ñ x 'Z…ÕÁÑ Jx, 'Z…ÕÁÑ x '"³á…ÿ\˜Ñ Jx „²eÀ‹ë,
“847791”, J~ò`Í “RUBBER” U q^ÎOQê HË_£ Q®_Û †³òH›ø ~¡OQ®°

D
(A.P. S.I. Mains 2017)

9
59
Kͪë~¡° ? 1. |¶Á 2. Z…ÕÁ 3. "³á…ÿ\˜ 4. ï~_£
CA
95
(SSC CHSL (10+2) Tier-I (CBE) Exam.2017) 8. TEACHER =°i†Çò HIGHLY „¬^¥#° =~¡°‹¬Qê
XWPBRWM =°i†Çò RSNRDZ Qê ã"ŒÀ‹ë 'CHAR-

50
1) 759597 2) 758897 3) 795957 4) 795579
2. XH› HË_£ ƒì+¬…Õ, "PAPPER" J#Qê "@#@@#!" ITY' „¬^¥xß U q^ÎOQê ã"Œ†Ç°=KÇ°ó?
98
A
=°i†Çò "AIM" J#Qê "Ù?*". "PAMPER" J#°
:8
(AP.SI Prelims-2016)
E

„¬^Î=ò#° U q^ÎOQê ã"Œ†Ç°=KÇ°ó ? 1) BRPMSXZ 2) BPRNBSZ


h
.P

3) MBRPDSZ 4) BRPXSMZ
IC

(SSC CHSL Tier-1 Exam.2017)


AD

1) @Ù*@#! 2) @*Ù@#! 9. 'FURNITURE' #° HË_£ ƒì+¬…Õ ERUTINRUF, Qê


3) @Ù*#@! 4) @Ù*@!# ã"ŒÀ‹ë, 'FRAGRANCE' 䛽 HË_£ ? (AP.SI Prelims-'16)
AB
O

3. XH›"Íˆ× 'MOHAN' #° 'KMFYL'Qê HË_£ K͋²#, 1) FCNAGRFA 2) FARGRACEN


ER
CH

'COUNT' #° U q^ÎOQê HË_£ K͆ǰ=KÇ°ó ? 3) ECNARGARF 4) ARFGNCRE


YD

(SSC Multi-Tasking Staff Exam 2017) 10. 'DIAMOND' 䛽 HË_£ AFXJLKA J~ò`Í, 'JLKHBV'
,H

1) AMSLR 2) MSLAR 3) SAMLR 4) MASRL J<Í ^¥xß HË_£Qê Q®k? (AP.SI Prelims-2016)
T
AM

4. XH› xiœ+¬“"³°Ø# HË_£ ƒì+¬…Õ 'COMPUTERONE' #° 1) MONKER 2) KEYMON


H

'PMOCTUENOR' Jx ã"Œªë~°¡ . J^Í HË_£…Õ ã"Œ‹²#„¬C_È° 3) MONITER 4) MONKEY


M
AM

'ADVANTAGES' J<Ík (A.P. S.I. Mains 2017) 11. D ãH÷Ok HË_£, _™ HË_£#° „¬ijeOKÇO_. HË_£° ̄^Îí
IG

1. ADVANSEGAS 2. ADVTANSEAG (Hꄲ@…˜) JH›Æ~Œ…Õ ~Œ†Ç°|_#q.


KH

3. AVDANTAGES 4. AVDATNSEGA _™HË_£/H© : b a e s p r h i g t


R

5. XH› xiœ+¬“"³°Ø# HË_£ ƒì+¬…Õ 'TOMATO' #° '264126' HË_£ : L X P Z J Y Q M N B


Jx 'NET' #° '552' Jx ~Œªë ~ ¡ ° . J^Í HË_£ … Õ QPMNQB †³òH›ø _™ HË_£ ? (AP.SI Prelims-2016)

~Œ‹²#„¬C_È° 'TRACTOR' J<Ík (A.P. S.I.Mains '17) 1) Cought 2) Matery 3) Fights 4) Height
1. 2813269 2. 2913269 12. XH› HË_£ ƒ ì +¬ … Õ 'Triangle' J<Í „¬ ^ Î = ò#°
3. 2193269 4. 2913296 'SSHBMHKE' Qê ~ŒÀ‹ë, P HË_£ ƒì+¬…Õ 'EXAMPLE'

6. XH› ㄬ`ͼH›"³°Ø# ƒì+¬…Õ 'HEART' #° '@8531' Jx J<Í „¬^Î=ò#° W\ìÁ ~Œªë~¡°? (AP.Const.Prelims-'16)
'FEAST' #° '@8541' Jx ~ŒÀ‹ë, P HË_£ ƒì+¬…Õ 'FAR- 1) DYBNOMD 2) DYZNOMD
THEST' x W\ìÁ ~ŒªëO. (A.P. S.I. Mains 2017) 3) DYZNAMD 4) DYZNAMF
1. @8543#18 2. #5314@81 13. 'Ski rps tri' J<Í „¬^Î=ò 'Nice sunday morn-

3. #531@841 4. #531@481 ing' #°, "the sti rps" J<Ík "Ever y Thursday

REASONING ABILITY (VERBAL) ○ ○ ○ ○ ○ ○ ○ ○ ○


24
○ ○ ○ ○ ○ ○ ○ ○ ○ ○ ○ ○ ○ ○ ○ ○ ○ ○ ○
Study Material
Morning" #°, "Ski ptr qlm" J<Ík "NICE MAR- 22. XH› "Íˆ× #° 'VLCQFKMZ' Qê ã"ŒÀ‹ë,
'UMBRELLA'

KET PLACE" #° ‹¬¶zÀ‹ë "Sunday" #° ‹¬¶zOKÍ 'JUSTICE' J<Í „¬^¥xß W…ì ‹¬¶zªë~¡°.

„¬^ÎO (AP.Const. Prelims-2016) (TS.Const.Mains-2016)

1) Ski 2) rps 3) tri 4) qlm 1) KSSTJBF 2) KTTSICF


14. XH› HË_£ ƒì+¬…Õ "ACE" J<Í „¬^¥xß "AIY" Qê HË_£ 3) KTTSJBF 4) KTSSTAF
KÍÀ‹ë J^Í HË_£ ƒì+¬…Õ "BED" J<Ík U q^ÎOQê HË_£ 23. XH›"Íˆ× TBDIJO #° SACHIN 䛽 HË_£Qê ‹¬¶zÀ‹ë,
J=ô`Ç°Ok? (AP.Const. Prelims-2016) IZVOEBJ #° U „¬^¥xH÷ HË_£Qê ‹¬¶zOKÇ=KÇ°ó.

1) BYE 2) BYD 3) PYD 4) DYP (TS.Const.Mains-2016)


15. WOwÁ+µ¬ JH›~
Æ =
¡ ¶…Õx JH›~Æ Œ#° 1 #°O_ 26 ‹¬OY¼`Ë 1) SAMURAI 2) MARUTHI
=~¡°‹¬Qê J#°‹¬O^¥xOKŒ~¡°? 3) HYUNDAI 4) CRICKET
W J<Ík ㄬ^¥# ‹¬OY¼ Hêx ‹¬i‹¬OY¼#°, 24. XH›"Íˆ× TEN, GANG =°i†Çò MEAT J<Í „¬^¥#°

Y
$ J<Ík ㄬ^¥# ‹¬OY¼ Hêx ƒè‹²‹¬OY¼#°, =~¡°‹¬Qê 439, 6596 =°i†Çò 1354Qê ‹¬¶zÀ‹ë, MAN-

M
# J<Ík ㄬ^¥# ‹¬OY¼#° ‹¬¶zÀ‹ë,
AGEMENT J<Í „¬^¥xß Z…ì ‹¬¶zªë~¡°?

'BEAUTIFUL' J<Í „¬^Î=ò †³òH›ø HË_£ ~¡¶„¬=ò


(TS.Const.Mains-2016)

E
(AP.Const.Mains-2016)
1) 1595363199 2) 1595313694
1) ##$$W$W$W 2) #$#$WW$W#W

D
9
3) 1595361934 4) 1595631394

59
3) W#W$W$#W$ 4) ##$$$W$W$
CA
95
25. XH›"Íˆ× Z~¡°„¬ô#° „¬‹¬°„¬ô„¬KÇóQê#°, „¬‹¬°„¬ô „¬KÇó#°
16. XH› HË_£ ƒì+¬…Õ DIRECTOR #° WAIRXGBI #°
P䛽„¬KÇóQê#°, P䛽„¬KÇó#° hOQê#°, hO#° Q®°…ìa

50
‹¬¶zOz#, P ƒì+¬…Õ NEURO 䛽 HË_£
~¡OQ®°Qê#° J#°ä›½O>è, ª^¥~¡}OQê P䛽° D ~¡OQ®°…Õ
98
A
(AP.Const.Mains-2016)
LO\ì~ò.
:8
(TS.Const.Mains-2016)
1) MRHIB 2) AVCBK
E

1) P䛽„¬KÇó 2) Q®°…ìa~¡OQ®°
h

3) ORUNE 4) ENRBC
.P

3) „¬‹¬°„¬ô„¬KÇó 4) hO
IC

17. XH› HË_£ ƒ ì +¬ … Õ 'DONKEY' x 'FUPLIZ' `Ë


AD

26. XHê<ùH› xií+¬“"³°Ø# ~¡‚¬ì‹¬¼ ƒì+¬…Õ EMPHASIS J<Í


‹¬¶zOz#, P ƒì+¬…Õ "ELEPHANT" 䛽 HË_£
=¶@#° NDIOBRJR Qê ã"ŒÀ‹,ë J^Í ƒì+¬…Õ CREATURE
AB
O

(AP.Const.Mains-2016)
J<Í =¶@#° U…ì ã"Œªë~¡°.
ER
CH

(Group-I-2004)
1) FMFQIBOT 2) IMIQJEPV
3) GMFQBMSU 4) TNAHPELE 1) SBBDUTSD 2) QBBDTUSD
YD

18. XH› HË_£ ƒì+¬…Õ DOG = 29, PLANE = 53 J~ò# P 3) DSDBSTSP 4) SBBDUTSD
,H

ƒì+¬…Õ CAT = ..... XH› ~¡H›"³°Ø# ‹¬OöH`ÇO…Õ CATTLE JO>è DZUSMD,


T

27.
(AP.Const.Mains-2016)
AM

1) 31 2) 28 3) 27 4) 30 J~ò`Í STRAIN ‹¬OöH`Ç „¬^ÎO (J.L.-2003)


H
M

19. XH› HË_£ ƒì+¬…Õ 'PAPER' x QCSIW Jx ~ŒÀ‹ë J^Í 1) TSSYJM 2) TUSZJM
AM
IG

HË_£…Õ EXAMPLE Jx ~ŒÀ‹ q^¥#O 3) RSSBHM 4) TSSZJM


KH

(TS.S.I.Mains-2016) 28. D ãH÷Ok ‹¬=¶^¥<Œ…Õ Uk ㄬ‰§ß~¡ÖH› Q®°~¡°ë ªÖ#O…Õ


R

1) FZDUQRL 2) FZQDURL LO_È°#°. (J.L.-2004)

3) FZDQURL 4) FZQUDRL MILD : NKOH : : GATE : ?

20. 'AIRTEL' x 'CKTVGN' Qê HË_£ K͆ǰ|_#KË, 1) HDVQ 2) HCWI 3) IBUF 4) HDUR


'RELIANCE' H÷ HË_£ „¬^ÎO (TS.S.I.Mains-2016) 29. XH› ~¡‚¬ì‹¬¼ ƒì+¬…Õ HOTEL J<Í „¬^¥xß 300Qê HË_£
1) TGKNCPEG 2) TGNKCPEG K͆ǰ|_Ok. J~ò`Í BORE J<Í „¬^¥xß Z…ì HË_£
3) TGNKPCEG 4) TGNKCPGE Kͪë~¡°. (Group-I-2004)
21. CAR, TIN, BOAT #° =~¡°‹¬Qê 364, 182, 7561 1) 160 2) 340 3) 280 4) 420
Qê HË_£ KÍÀ‹ë 'CONTRACTOR' H÷ HË_£ 30. CAT q°= 24 =°i†Çò DOG q°= 26 J~ò#@Á~ò`Í
(TS.S.I.Mains-2016)
PIG q°= (Group-I 2002 & J.L.- 2003)
1) 3521643154 2) 3521463154
1) 37 2) 25 3) 29 4) 32
3) 3524163154 4) 3521634154

REASONING ABILITY (VERBAL) ○ ○ ○ ○ ○ ○ ○ ○ ○


25
○ ○ ○ ○ ○ ○ ○ ○ ○ ○ ○ ○ ○ ○ ○ ○ ○ ○ ○
Study Material
12345678901234567890123456789012123456789012345678901234567890121234567890123456789012345678901212345678901234567890123
12345678901234567890123 123456789012345678901234

q=~¡}ì`ÇàH› ["Œ|°°
12345678901234567890123456789012123456789012345678901234567890121234567890123456789012345678901212345678901234567890123
12345678901234567890123 123456789012345678901234
12345678901234567890123456789012123456789012345678901234567890121234567890123456789012345678901212345678901234567890123
12345678901234567890123 123456789012345678901234
12345678901234567890123456789012123456789012345678901234567890121234567890123456789012345678901212345678901234567890123
12345678901234567890123 123456789012345678901234
12345678901234567890123
12345678901234567890123 PREVIOUS BITS :
12345678901234567890123456789012123456789012345678901234567890121234567890123456789012345678901212345678901234567890123
123456789012345678901234
12345678901234567890123456789012123456789012345678901234567890121234567890123456789012345678901212345678901234567890123
123456789012345678901234
12345678901234567890123456789012123456789012345678901234567890121234567890123456789012345678901212345678901234567890123
12345678901234567890123 123456789012345678901234

1. (2) N U M B E R ̄á# Wzó# POQ®Á JH›Æ~Œä›½ JOïH° _³áï~H±“Qê HË_£


¯ ¯ ¯ ¯ÿÿÿ ¯ ¯ K͆ǰ|_# Wzó# ㄬ‰×߅Õx 'TRACTOR' J#°
1 5 6 8 9 7 „¬^Î=ò…Õx „¬^Î=òä›½
B A R R E N T R A C T O R
¯ ¯ ¯ ¯ÿÿÿ ¯ ¯ ¯ ¯ ¯ ¯ ¯ ¯ ¯
8 4 7 7 9 1 2 1 2 6
̄á# HË_£ K͋²# q^ÎOQê ̄á# HË_£ KÍÀ‹ q^¥#=ò…Õ 'R' 䛽 =°i†Çò 'C' #䛽
R U B B E R HË_£ JH›Æ~Œ° W=Þ_ÈO …è^ΰ. Wzó# ‹¬=¶^¥#=ò
¯ ¯ ¯ ¯ÿÿÿ ¯ ¯ ^¥Þ~Œ KǶÀ‹ë XH› '2Ñ = P„¬Â<£…Õ =¶ã`Ç"Í° 'R' 䛽
7 5 8 8 9 7 HË_£ 9Qê (Hê=°<£ Q ê) =°i†Ç ò C H÷ HË_£ 3

Y
2. (1) P E P P E R K͆ǰ|_#k. Hê=ô# ‹¬=¶^¥#=ò "2913269".

M
¯ ¯ ¯ ¯ÿÿÿ ¯ ¯
6. (3) H E A R T F E A S T
@ # @ @ # ! ¯ ¯ ¯ ¯ ¯ ¯¯ ¯ ¯¯

E
A I M @ 8 5 3 1 # 8 5 4 1

D
9
¯ ¯ ¯ Hê=ô#

59
Ù ? * F A R T H E S T
CA
95
̄á# HË_£ K͋²# q^ÎOQê ¯ ¯ ¯ ¯ ¯ ¯ ¯ ¯
# 5 3 1 @ 8 4 1

50
P E M P E R
¯ ¯ ¯ ¯ÿÿÿ ¯ ¯ ãw<£ ® ~ï _£ Qê#¶
98
A
7. (4)
@ Ù * @ # ! ~ï _£ ® |¶Á Qê#¶
:8

.0)"/,.':- |¶Á ® "³á\˜ Qê#¶


E
h

3. (1)

á ˜x ® Z…ÕÁ Qê#¶
.P

"³\
IC



Z…ÕÁx ® "³…
AD


á \
ÿ ˜ Qê#¶ HË_£ K͋#²
ª^¥~¡}OQê Q®_ۍ ãw<£ H›~Ÿ…Õ LO_È°#°. ãw<£ H›~Ÿ

AB
O



†³òH›ø HË_£ ï~_£.


ER


CH

Hê=ô# ‹¬=¶^¥#=ò ''ï~_£ÑÑ.


̄á# HË_£ K͋²# q^ÎOQê
YD

8. (1)
,H

$06/5".4-3 T E A C H E R H I G H L Y
¯ ¯ ¯ ¯ ¯ ¯ ¯ ¯ ¯ ¯ ¯ ¯¯
T
AM



X W P B R W M R S N R D Z
H



ÿ̄á"Œ\÷ #°O_
M


AM


IG

C H A R I T Y
¯ ¯ ¯ ¯ ¯ ¯ ¯

KH

4. (4) $0.16530/& B R P M S X Z
R

3FWFSTF 3FWFSTF 3FWFSTF \ CHARITY †³òH›ø HË_£ BRPMSXZ ®


9. (3) FURNITURE …Õx JH› Æ ~ Œ#° ãu„² æ ã"Œ†Ç ° Qê
1.0$56&/03 U~¡æ_È°#k ERUTINRUF J^Íq^ÎOQê
̄á# HË_£ K͋²# q^ÎOQê \ 'FRAGRANCE' †³òH›ø HË_£ ECNARGARF ®
"%7"/5"(&4 10. (4) % * " . 0 / %
      
3FWFSTF 3FWFSTF 3FWFSTF
" ' 9 + - , "
"7%"5/4&(" J^Íq^ÎOQê, + - , ) # 7
     
5. (2) T O M A T O N E T
¯ ¯ ¯ ¯ ¯ ¯ ¯ ¯ ¯ . 0 / , & :

2 6 4 1 2 6 5 5 2 \ Hê=‹²# „¬^ÎO ® MONKEY


REASONING ABILITY (VERBAL) ○ ○ ○ ○ ○ ○ ○ ○ ○
26
○ ○ ○ ○ ○ ○ ○ ○ ○ ○ ○ ○ ○ ○ ○ ○ ○ ○ ○
Study Material
11. (4) Wzó# ‹¬=¶KŒ~¡O ㄬHê~¡O 21. (2)
Þ Þ
Q h;P e;M i;N g;B Þ Þ Þt C A R T I N B O A T
\
Hê=‹²# _™ HË_£ height Þÿ ¯ ¯ ¯ ¯ ¯ ¯ ¯ ¯ ¯ ¯
12. (2) 5 3 * " / ( - & 3 6 4 1 8 2 7 5 6 1
        ÿ̄á"Œ\÷ #°O_
4 4 ) # . ) , ' C O N T R A C T O R
J^Íq^ÎOQê, & 9 " . 1 - & ¯ ¯ ¯ ¯ ¯ ¯ ¯ ¯ ¯ ¯
      
3 5 2 1 4 6 3 1 5 4
% : ; / 0 . % 22. (3) 6 . # 3 & - - "
\ EXAMPLE †³òH›ø HË_£ÿÞ DYZNOMD        

13. (3) 4LJ SQT USJ  /JDF 4VOEBZ NPSOJOH 7 - $ 2 ' , . ;


J^Íq^ÎOQê
the sti SQT ÞÿEvery Tuesday NPSOJOH + 6 4 5 * $ &

Y
      
4LJ ptr qlm Þ /JDF market place , 5 5 4 + # '

M
\ Sunday x ‹¬¶zOKÇ° „¬^ÎO Þÿtri 23. (3) 5 # % * + 0
14. (1)

E
     

15. (1) POQ®Á =~¡â=¶ JH›Æ~Œ †³òH›ø J#°~¡¶„¬ ‹¬OY¼ 4 " $ ) * /

D
9
P^¥~¡OQê B = 2, E = 5, A = 1, T = 20, U = 21, J^Íq^ÎOQê

59
* ; 7 0 & # +

I = 9, F = 6, L = 12
CA       

95
) : 6 / % " *
W=Þ|_# x†Ç°=¶ ㄬHê~¡O HË_OQ· K͆ǰQê D

50
24. (4) T E N G A N G M E A T
ãH÷Ok q^ÎOQê =KÇ°ó#°. ¯ ¯ ¯ ¯ ¯ ¯ ¯ ¯ ¯ ¯ ¯
98
A
B E A U T I F U L 4 3 9 6 5 9 6 1 3 5 4
¯ ¯ ¯ ¯ ¯ ¯ ¯¯ ¯
:8
ÿ̄á"Œ\÷ #°O_
E

# # $ $ H $ H $ H
h
.P

M A N A G E M E N T
IC

16. (1) DIRECTOR J#°„¬^ÎO…Õ I, E, O ° "Íö~ JH›Æ~Œ`Ë ¯ ¯ ¯ ¯ ¯ ¯ ¯ ¯ ¯ ¯


AD

HË_£ K͆ǰ|_¨Û~ò. q°ye# JH›Æ~Œ° "Œ\÷ =¼uö~H› 1 5 9 5 6 3 1 3 9 4


JH›Æ~Œ`Ë HË_£ K͆ǰ|_¨Û~ò. ^¥x ㄬHê~¡O KǶ‹²#
AB
O

25. (4) P䛽° P䛽„¬KÇó ~¡OQ®°…Õ LO\ì~ò. Hêx HË_£


NEURO †³òH›ø HË_£ MRHIB.
ER

ㄬHê~¡O P䛽„¬KÇó J#Qê hO.


CH

17. (2)
26. E M P H A S I S, C R E A T U R E
YD

18. (3) DOG …Õx JH›Æ~Œ ‹¬OY¼ « 3


(1)

| | | | | | | |
®
,H

D + O + G 4 + 15 + 7 = 26 + 3 = 29 N D I O B R J R, S B B D U T S D
T

PLANE …Õx JH›Æ~Œ ‹¬OY¼ « 5


AM

P + L + A + N + E 16 + 12 + 1 + 14 + 15 ® +1 –1 +1 –1 +1 –1 +1–1 +1 –1 +1 –1 +1 –1 +1 –1
H

27. C A T T L E, S T R A I N
M

(4)
= 48 + 5 = 53 +1 –1 +1 –1 +1 –1 +1 –1 +1 –1 +1 –1
® ®
AM
IG

J^Íq^ÎOQê C + A + T 3 + 1 + 20 24 +3= 27 D Z U S M D T S S Z J M
19. (3) 1 " 1 & 3
KH

28. (2) M I L D, G A T E
     +1 +2 +3 +4 +1 +2 +3 +4
R

2 $ 4 * 8 N K O H, H C W I
J^Íq^ÎOQê & 9 " . 1 - & 29. (1) 1 2 3 4 5 6 7 8 9 10 1 12 13 14 15 16 17 18 19
       20 21 22 23 24 25 26
' ; % 2 6 3 - a b c d e f g h i j k l m n o p q r s t u v w x yz
\ Example †³òH›ø HË_£ ®ÿFZDQURL HOTEL = 8 + 15 + 20 + 5 + 12
´
= 60 5 (5 letters) = 300
20. (2) " * 3 5 & - BORE = 2 + 15 + 18 + 5 = 40 4 (4 letters) ´
     
= 160
$ ,5 7 ( / 30. (4) 1 2 3 4 5 6 7 8 9 10 1 12 13 14 15 16 17 18 19
J^Íq^ÎOQê 3 & - * " / $ & 20 21 22 23 24 25 26
       
a b c d e f g h i j k l m n o p q r s t u v w x yz
5 ( / , $ 1 & ( CAT = 3 + 1 + 20 = 24, DOG = 4 + 15 + 7 = 26,
\ RELIANCE †³òH›ø HË_£ ®ÿTGNKCPEG PIG = 16 + 9 + 7 = 32
REASONING ABILITY (VERBAL) ○ ○ ○ ○ ○ ○ ○ ○ ○
27
○ ○ ○ ○ ○ ○ ○ ○ ○ ○ ○ ○ ○ ○ ○ ○ ○ ○ ○
Study Material
2 ã‰õ}°°
ã‰õ}°°
(Series)

=òMì¼O‰§°...
+ XH› =~¡°‹¬…Õ =ôOKÇ|_# ‹¬OY¼° …è^¥ JH›Æ~Œ° …è^¥ „¬^¥° XöH ^Î~Œàxß H›ey=ôO>è J\÷“ =~¡°‹¬ãH›=¶xß JO\ì~¡°.
+
'ã‰õ}÷Ñ

1) ‹¬OMì¼ ã‰õ}÷, 2) P…ì权ÿ\˜ (JH›Æ~¡) ã‰õ}÷.


+
ã‰õ}°…Õx ~¡Hꁰ :

s[xOQ·…Õ Wk XH› =òY¼"³°Ø# qƒìQ®O. ‡é\© „¬sH›Æ…Õ JkH› ㄬ‰×߁° D qƒìQ®O #°O_Í J_ÈQ®|_È`Œ~ò.
+ D qƒìQ®O #°O_ J_öQ ㄬ‰×߁䛽 ‹¬°°=ôQê ‹¬=¶^¥#O ~Œ|\쓁O>è Jƒ’¼iÖH÷ ‹¬‚¬ì[ ‹¬OY¼ä›½ ‹¬O|OkOz# =~Œ¾°

Y
=°i†Çò „¦¬°#=ò° H›h‹¬O 1 #°O_ '25Ñ =~¡ä›½ Yzó`ÇOQê =zó †ÇòO_¨e.
+

M
D qƒìQ®O #°O_ J_ÈQ®|_Í ã„¬‰×߁° U q^Î"³°Ø# x†Ç°=¶xß H›ey=ôO\솳¶ Q®°iëOKÇ°@ :
Z) Wzó# ‹¬OY¼ ãH›=°O…Õ Z_È=° #°O_ 䛽_"³á„¬ô䛽 ‹¬OY¼ q°= Hùk퇐\÷Qê ̄~¡°Q®°`Ƕ (…è^¥) `ÇQ®°¾`Ƕ =ôO>è Jq

E
䛀_H›ä›½ (…è^¥) f‹²"Í`Ç䛽 ‹¬O|OkOz# ㄬ‰×߁° J~ò =ôO\ì~ò.

D
9
59
a) Wzó# ‹¬OY¼ ãH›=°O…Õ Z_È=° #°O_ 䛽_"„á³ ô¬ 䛽 ‹¬OY¼ q°= ƒìQê ̄~¡°Q®°`Ƕ =ôO>è Jq Q®°}Hê~ŒxH÷ ‹¬O|OkOz#
CA
95
ㄬ‰×߁° J~ò =ôO\ì~ò.
‹²) Wzó# ‹¬OY¼ ãH›=°O…Õ Z_È=° #°O_ 䛽_"³á„¬ô䛽 ‹¬OY¼ q°= ƒìQê `ÇQ®°¾`Ƕ =ôO>è Jq ƒìQꂬð~ŒxH÷ ‹¬O|OkOz#

50
98
A
ㄬ‰×߁° J~ò =ôO\ì~ò.
:8
_) ㄬ‰×߅Õx ‹¬OY¼ ̄~¡°Q®°^΁ …è^¥ `ÇQ®°¾^΁ ãH›=¶xß |\÷“ =òO^ΰ ‹¬OY¼ †³òH›ø =~¡¾O (…è^¥) „¦¬°#=òä›½ ‹¬O|OkOz#
E

ㄬ‰×߅è"³¶#x Q®=°xOKŒe.
h
.P

+
IC

D qƒìQ®O #°O_ J_ÈQ®|_Í ã„¬‰×߁…Õ ‹¬OY¼° …è^¥ JH›Æ~Œ° ãH›=¶xß W=Þ_ÈO [~¡°Q®°`Ç°Ok. P ãH›=°O…Õ XH› ‹¬OY¼ …è^¥
AD

JH›Æ~¡O `DŽ¬CQê …è^¥ ㄬ‰§ß~¡ÖH›O (?) Q®°~¡°ë`Ë W=Þ_ÈO [~¡°Q®°`Ç°Ok. D ㄬ‰§ß~¡ÖH›O (?) Q®°~¡°ë =ô#ßKË@ =ôO_È=‹²# ‹¬OY¼
AB
O

…è^¥ JH›Æ~Œxß Wzó# ãH›=°O P^¥~¡OQê =°#O Q®°iëOKŒe.


ER
CH

TYPE - I : ‹¬OMì¼ã‰õ}°°
YD

+
,H

‹¬OMì¼ ã‰õ}°ä›½ ‹¬O|OkOz# ㄬ‰×߁#° ªkOKÍ =òO^ΰ Hùxß =°i†Çò


T

‹¬‚¬ì[ ‹¬OY¼ =~Œ¾° „¦¬°<Œ q°=°


AM

`³°‹¬°ä›½O^¥=ò. (1 - 25)
H
M

(x)
2
(x ) (x )
3 (x) (x )
2 3
(x )
AM

‹¬OY¼ =~¡¾=ò „¦¬°#=ò ‹¬OY¼ =~¡¾=ò „¦¬°#=ò


IG

1 1 169 2197
KH

1 13

4 8 14 196 2744
R

3 9 27 15 225 3375
4 16 64 16 256 4096
5 25 125 17 289 4913
36 216 18 324 5832
361 6859
6

49 343 19

400 8000
7

64 512
20
8
441 9261
81 729
21
9
22 484 10648
10 100 1000 23 529 12167
11 121 1331 24 576 13824
12 144 1728 25 625 15625
REASONING ABILITY (VERBAL) ○ ○ ○ ○ ○ ○ ○ ○ ○
28
○ ○ ○ ○ ○ ○ ○ ○ ○ ○ ○ ○ ○ ○ ○ ○ ○ ○ ○
Study Material
G (1-10) =~¡ä›½ =~¡¾=ò, „¦¬°#=ò° =°#䛽 `³‹¬°#°. Hêx (10-30) =~¡ä›½ Q® ‹¬OY¼ †³òH›ø =~¡¾=ò, „¦¬°#=ò°

H›#°Qù#°@. 'Vedic Maths' Techniques L„¬†³¶yÀ‹ë ‹¬°ƒ’`Ç~¡OQê H›#°Qù#=KÇ°ó.

; ï~O_ÈOïH ‹¬OY¼ (xy) =~¡¾=ò H›#°Qù#°@#°.


YZ
 Y   YZ  Z


m 
   s  s       m 
  

m 
   s  s       m 
  

m 
   s  s       m 
  

m 
   s  s       m          
  

2
m (‹¬OY¼ `Ë JO`Ç=°=ô`Ç°Ok) ^¥xHˋ¬O

Y
(15) '5'

m  m   m  

M
Short cut 1 ×(1+1)


   s  s  
m
    m     

E

 256

    m  
   m

D
9


   s  s  


59
289

    m      m    m CA
95


   s  s  

    (2+1)

50

m  s s  
 
m m m     m 98
A

    
    
  
:8

m ss  
m 

E


 
h

m  s s  m 
.P
IC

  

  
AD




m ss  
m 

 
AB
O




m   s  s   m    m  
  m 
 
ER
CH

m   s  s   m    m  
 

YD

  

= 576
,H

2
m (‹¬OY¼ `Ë JO`Ç=°=ô`Ç°Ok) Hê=ô#
T

(25) '5'
AM


  s  
   s    
H



m s  s   m    m  
 

AM

  
IG


 = 676
KH




m 
 s  s   m    m  
 
 m    m

m  
 
R

729




m 
 s  s   m    m

m m  
   784




m 
 s  s   m    m  
 
 m    m  
  m 





m 
 s  s   m    m 


REASONING ABILITY (VERBAL) ○ ○ ○ ○ ○ ○ ○ ○ ○


29
○ ○ ○ ○ ○ ○ ○ ○ ○ ○ ○ ○ ○ ○ ○ ○ ○ ○ ○
Study Material
TEST Yourself on MODEL - 1 5. (3) ^Î`Çëã‰õ}÷ ‡\÷OKÇ° ãH›=°=ò.
̄~¡°Q®°^΁ / `Ç~¡°Q®°^΁ ã‰õ}÷ 













1. 2, 6, 10, 14, 18, 22, ?


    
1) 16 2) 26 3) 36 4) 24
2. 5, 12, 19, 26, 33, ?
\ ㄬ‰×ß Q®°~¡°ë ªÖ#=ò…Õ =KÇ°ó ‹¬OY¼ '12Ñ .
1) 35 2) 39 3) 40 4) 42 6. (3) ^Î`Çëã‰õ}÷ ‡\÷OKÇ° ãH›=°=ò.
3. 52, 48, 44, 40, 36, ?
     
1) 38 2) 42 3) 44 4) 32     
4. 250, 220, 190, 160, 130, ?

1) 150 2) 140 3) 100 4) UnHê^ΰ


\ ㄬ‰×ß Q®°~¡°ë ªÖ#=ò…Õ =KÇ°ó ‹¬OY¼ '34Ñ .
7. (4) ^Î`Çëã‰õ}÷ ãH›=°=ò

Y
5. 2, 6, ?, 20, 30, 42, 56

1) 8 2) 19 3) 12 4) 14 29 31 37 41 43

M
47, 53, 59

6. 2, 10, 18, 26, , 42 \ W=hß =~¡°‹¬ ㄬ^¥# ‹¬OY¼…è


1) 32 2) 36 3) 34 4) 37

E
8. (2) W=hß =~¡°‹¬ ‹¬O†ÇòH›ë ‹¬OY¼…è.
\ Hê=‹²# ‹¬OY¼° =~¡°‹¬Qê

D
9
7. 29, 31, 37, 41, 43, , , 59 21, 22

59
1) 45, 47 2) 47, 49 3) 45, 49 4) 47, 53 9. (1) ^Î`Çëã‰õ}÷ ãH›=°=ò
CA
95
8. 12, 14, 15, 16, 18, 20, , , 24, 25, 26,
   

50

27, 28    

98
A
1) 22, 23 2) 21, 22 3) 21, 21 4) 22, 22 \ ㄬ‰×ß Q®°~¡°ë ªÖ#=ò…Õ =KÇ°ó ‹¬OY¼ .
:8
'48Ñ

10. (3) ^Î`Çëã‰õ}÷ ãH›=°=ò


E

9. 96, 80, 64, , 32.


h

1) 48 2) 42 3) 54 4) 58
.P
IC

     
AD

10. 61, 72, 83, 94, , ,


    
1) 106, 117 2) 104, 115 \ ㄬ‰ß× Q®°~¡°ë ªÖ#=ò…Õ =KÇ°ó ‹¬OY¼°
AB
O

.
3) 105, 116 4) 105, 126
'105, 116Ñ
ER
CH

MODEL - 1 : Solutions
YD

MODEL - 2
,H

1. (2) ^Î`Çëã‰õ}÷ ‡\÷OKÇ° ãH›=°=ò.


T
AM

1. XH› ‹¬OY¼#° q_z =°~ùH› ‹¬OY¼#° „¬iQ®}#…ÕxH÷

       D „¬^Μu…Õ W=Þ|_# ã‰õ}÷…Õ ï~O_È°


H

f‹¬°Hù#° ã‰õ}÷ :
     
M

\ ã‰õ}÷…Õ `Ç~¡°"Œ`Ç ‹¬OY¼ . (…è^¥) JO`ÇH›<Œß Z䛽ø= qƒìQꁰ =ôO\ì~ò. XHùøH›ø


AM
IG

'26Ñ

2. (3) ^Î`Çëã‰õ}÷ ‡\÷OKÇ° ãH›=°=ò. qƒìQêxH÷ XHùøH›ø ^Î~¡àO =ôO@°Ok. Jxß qƒìQê#°
KH

ä›Æ½}âOQê „¬ijeOz ‹¬=¶^¥#O ~Œ|\ì“e.


R

     
     L^¥ : 1. 24, 70, 30, 78, 36, 86, Y , Z .
\ ã‰õ}÷…Õ `Ç~¡°"Œ`Ç ‹¬OY¼ '40Ñ .   
3. (4) ^Î`Çëã‰õ}÷ ‡\÷OKÇ° ãH›=°=ò. ª^Î#:       Y Z
     
       
\ ã‰õ}÷…Õ `Ç~¡°"Œ`Ç ‹¬OY¼ '32Ñ . ̄á# W=Þ|_# ã‰õ}° #°O_,
4. (3) ^Î`Çëã‰õ}÷ ‡\÷OKÇ° ãH›=°=ò. 36 + 6 = x Þ x = 42

      86 + 8 = y ÿÞ y = 94

    


\ =°i†Çò
\
x = 42 y = 94.

ㄬ‰×ß Q®°~¡°ë ªÖ#=ò…Õ =KÇ°ó ‹¬OY¼ '100Ñ .

REASONING ABILITY (VERBAL) ○ ○ ○ ○ ○ ○ ○ ○ ○


30
○ ○ ○ ○ ○ ○ ○ ○ ○ ○ ○ ○ ○ ○ ○ ○ ○ ○ ○
Study Material
L^¥ : 2. 20, 63, 100, 27, 73, 113, 34, 83, 3 8 4 6 ® 6 4 8 3 (=¼uö~Y k‰×)
126, Y ,Z,[ . 5 1 6 2 ® 2 6 1 5 (=¼uö~Y k‰×)
ª^Î#: J^Íq^ÎOQê
   9 1 0 8 ®
(=¼uö~Y k‰×)
8 0 1 9

L^¥ : 4. 6157, 1675, 4136, 1463, 7815, .


         Y Z [
ª^Î#:  
   
   
̄á# qƒìQê#° „¬ijeOz#,
34 + 7 = x Þ x = 41
J^Íq^ÎOQê  

ÿÞ

Y
83 + 10 = y y = 93

ÿÞ TEST Yourself on MODEL - 2

M
126 + 13 = z z = 139

\ x = 41 ; y = 93 =°i†Çò z = 139.
1. 3, 4, 5, 7, 9, 10, 11, 14,

E
2. ã‰õ}…
÷ Õx ‹¬OY¼#° „¬îiëQê ãu„²æ ~Œ†Çò@ : D „¬^uœÎ …Õ, 1) 12 2) 20 3) 15 4) 16

D
Wzó# ã‰õ}÷…Õx ‹¬OY¼#° „¬îiëQê ãu„²æ ~Œ†Ç°@O

9
59
2. 8, 15, 13, 20, 18, 25, ,

[~¡°Q®°`Ç°Ok. P ã‰õ}÷…Õ ㄬ‰§ß~¡ÖH›O =ô#ßKË@ U ‹¬OY¼


CA
1) 24, 31 2) 22, 29

95
=‹¬°ëO^Ë Q®°iëOKŒe. 3) 28, 35 4) 23, 30

50
79, 97, 71, 17, 59, 95, 68, .
98
A
L^¥ : 1.
3. 9612, 6921, 1896, 8169, 2345,

68#° ãu„²æ ~Œ†Ç°Qê 86 =‹¬°ëOk.


:8
ª^Î# :
1) 3254 2) 4235 3) 5432 4) 5324
684, 486, 781, 187, 197, 791.
E
h

L^¥ : 2.
.P

4. 3842, 2843, 1849, 9841, , 6784

765, .
IC

1) 6874 2) 4786
AD

765#° ãu„²æ ~Œ†Ç°Qê 567 =‹¬°ëOk.


3) 4876 4) 4687
ª^Î# :
AB
O

3. ã‰õ}…
÷ Õx ‹¬OY¼ ªÖ<Œ° =¶ió ã"Œ†Çò@ : D „¬^uœÎ …Õ,
ER

5. 849, 948, 637, , 138, 831

XHùøH›ø ã‰õ}÷…Õ XHùøH›ø ~¡H›OQê ‹¬OY¼…Õx JOïH


CH

1) 376 2) 673 3) 736 4) 763


ªÖ<Œ° `Œ~¡°=¶~¡° K͋² ~Œªë~¡°. U „¬^Μu…Õ JOïH
YD

ªÖ<Œ° =¶~Œó~Ë Q®=°xOz P „¬^uœÎ ㄬHê~¡O ‹¬=¶^¥<Œxß MODEL - 2 : Solutions


,H
T

~Œ|\ì“e.
AM

   
156, 561, 892, 928, 176, .
H
M

L^¥ : 1.

1. (3) Y
AM
IG

ª^Î#:    

KH


   
Þ
R

x = 10 + 5 = 15

J^Íq^ÎOQê       

L^¥ : 2. 3456, 6354, 1234, 4132, 6798, . 2. (4) YZ


ª^Î#:        
̄á# KǶ„¬|_# ãH›=°O #°O_ x = 18 + 5 = 23 ;

     y = 25 + 5 = 30.

J^Íq^ÎOQê      3. (1)      


L^¥ : 3. 3846, 6483, 5162, 2615, 9108, . J^Íq^ÎOQê  
ª^Î#: =¼uö~Y k‰×…Õ ~Œ†Ç°_ÈO [iyOk.

REASONING ABILITY (VERBAL) ○ ○ ○ ○ ○ ○ ○ ○ ○


31
○ ○ ○ ○ ○ ○ ○ ○ ○ ○ ○ ○ ○ ○ ○ ○ ○ ○ ○
Study Material
4. (2)   TEST Yourself on MODEL - 3
1. 121, 16, 132, 36, 252,
 
1) 81 2) 64 3) 49 4) 121
5. (3) 849#° =¼uö~H› k‰×…Õ ~Œ†Ç°Qê 948
2. 31, 64, 42, 216, 34,

138#° =¼uö~H› k‰×…Õ ~Œ†Ç°Qê 831 J^Íq^ÎOQê


1) 256 2) 343 3) 512 4) 125
637#° =¼uö~H› k‰×…Õ ~Œ†Ç°Qê . 736

3. 61, 49, 39, 144, 86,

MODEL - 3 1) 196 2) 361 3) 225 4) 81


4. 17, 476, 22, 895, , 747
1. ã‰õ}÷…Õx ‹¬OY¼…Õx JOïH#° 䛀_ P 䛀_H› †³òH›ø

D „¬^uœÎ …Õ ã‰õ}…÷ Õ 1) 16 2) 19 3) 20 4) 18

Y
„¦°
¬ <Œxß `Ç~°
¡ "Œu ‹¬OY¼Qê ã"Œ†Çò@ :

W=Þ|_# ‹¬OY¼…Õx JOïH#° =~¡°‹¬Qê 䛀_, P 䛀_H›

M
5. 676, 19, 489, 21, 635,

†³òH›ø „¦¬°<Œxß `Ç~¡°"Œu ‹¬OY¼Qê ~Œªë~¡°. 1) 16 2) 18 3) 14 4) 21

E
61, 343, 45, 729, 121, 64, 233, .
MODEL - 3 : Solutions
L^¥ :

61 ® (6 { 1)3 « 73 « 343

D
9
59
ª^Î#:

45 ® (4 { 5) « 93 « 729 1. (1) (1 { 2 { 1)2 « 42 « 16  


CA
95
121 ® (1 { 2 { 1)3 « 43 « 64 (1 { 3 { 2)2 « 62 « 36.

50
J^Íq^ÎOQê 233 ® (2 { 3 { 3)3 « 83 « 512. J^Íq^ÎOQê (2 { 5 { 2)2 « 92 « .
98
A
81

2. (2) (3 { 1)3 « 43 « 64   (4 { 2)3 « 63 « 216.


:8
2. ã‰õ}÷…Õx ‹¬OY¼…Õx JOïH#° 䛀_ P 䛀_H› †³òH›ø

J^Íq^ÎOQê (3 { 4)3 « 73 « .
E
h

D „¬^Μu…Õ ã‰õ}÷…Õ
343
.P

3. (1) (6 { 1)2 « 72 « 49   (3 { 9)2 « 122 « 144.


=~Œ¾xß `Ç~¡°"Œu ‹¬OY¼Qê ã"Œ†Çò@ :
IC

W=Þ|_# ‹¬OY¼…Õx JOïH#° =~¡°‹¬Qê 䛀_, P 䛀_H›


AD

J^Íq^ÎOQê (8 { 6)2 « 142 « . 196

†³òH›ø =~Œ¾xß `Ç~¡°"Œu ‹¬OY¼Qê ~Œªë~¡°. 4. (4) 17 « 4 { 7 { 6   22 « 8 { 9 { 5  


AB
O

213, 36, 412, 49, 611, 64, 131, . J^Íq^ÎOQê « 7 { 4 { 7 «


ER

L^¥ :
CH

213 ® (2 { 1 { 3)2 « 62 « 36
18

5. (3) 6 { 7 { 6 « 19   4 { 8 { 9 « 21
YD

ª^Î#:

412 ® (4 { 1 { 2)2 « 72 « 49 J^Íq^ÎOQê « 6 { 3 { 5 «


,H

14

611 ® (6 { 1 { 1)2 « 82 « 64
T
AM

\ 131 ® (1 { 3 { 1)2 « 52 « 25 MODEL - 4


H
M

\ Hê=‹²# ‹¬OY¼ « 25. D „¬^Μu…Õ ã‰õ}÷…Õx


AM
IG

1. ‹¬OY¼ †³òH›ø ƒìœ ã‰õ}÷ :

‹¬OY¼#° Q®°}÷OKÇ°@ Hêh …èH› Q®°}÷OKÇQê =KÇ°ó "Œ\÷


KH

3. Wzó# ã‰õ}÷…Õx ‹¬OY¼#° 䛀_ `Ç~¡°"Œ`Ç ‹¬OY¼Qê

D „¬^Μu…Õ ã‰õ}÷…Õ W=Þ|_# ‹¬OY¼…Õx „¦¬e`ÇO#° =~¡¾=ò°Qê Qêh …èH› „¦¬°#=ò°Qê Qêh
R

~Œ†Çò@ :

JOïH#° =~¡°‹¬Qê 䛀_ `Ç~¡°"Œ`Ç ‹¬OY¼Qê ~Œªë~¡°. ã"Œ†Ç°|_È@O [~¡°Q®°`Ç°Ok.


88, 16, 68, 14, 76, 13, 89, 17, 78, . L^¥ : 1. 48, 32, 63, 18, 29, 18, 34, 12
̄á# W=Þ|_# ã‰õ}÷…Õ, 4 þ 8 « 32
L^¥ :

88 ® 8 { 8 « 16
6 þ 3 « 18
ª^Î#:

68 ® 6 { 8 « 14 2 þ 9 « 18
76 ® 7 { 6 « 13 3 þ 4 « 12
89 ® 8 { 9 « 17 L^¥ : 2. 234, 24, 163, 18, 362, 36
\ 78 ® 7 { 8 « 15 ̄á# W=Þ|_# ã‰õ}÷…Õ, 2 þ 3 þ 4 « 24
\ Hê=‹²# ‹¬OY¼ « 15. 1 þ 6 þ 3 « 18
3 þ 6 þ 2 « 36

REASONING ABILITY (VERBAL) ○ ○ ○ ○ ○ ○ ○ ○ ○


32
○ ○ ○ ○ ○ ○ ○ ○ ○ ○ ○ ○ ○ ○ ○ ○ ○ ○ ○
Study Material
L^¥ : 3. 32, 216, 52, 1000, 24, 512 
     
̄á# W=Þ|_# ã‰õ}÷…Õ, 
3 þ 2 « 6 Þ 63 « 216 
     
5 þ 2 «10 Þ 103 « 1000 
2 þ 4 « 8 Þÿ83 « 512 3. Q®°}Hê~¡O „¬~¡OQê ªÖ<Œ "ŒiQê =KÇ°ó ã‰õ}÷ :

L^¥ : 4. 63, 324, 56, 900, 72, 196 1) 11, 24, 39, 416, 525, 636, 749.
̄á# W=Þ|_# ã‰õ}÷…Õ, ̄á# W=Þ|_# ã‰õ}÷…Õ 1 þ 1 « 1 Þ 11
6 þ 3 « 18 Þ 182 « 324
2 þ 2 « 4 Þ 24
5 þ 6 « 30 Þ 302 « 900
7 þ 2 « 14 Þÿ142 « 196 3 þ 3 « 9 Þ 39
2. ‹¬OY¼ †³òH›ø ƒìQꂬð~¡ ã‰õ}÷ : D „¬^Μu…Õ ‹¬OY¼#° 4 þ 4 « 16 Þ 416
5 þ 5 « 25 Þ 525

Y
=~¡ ° ‹¬ Q ê U^Ë XH› ‹¬ O Y¼ #°O_ ㇐~¡ O a O z
ƒìQꂬìiOKÇ_ÈO [~¡°Q®°`Ç°Ok. 6 þ 6 « 36 Þ 636

M
G Wzó# ã‰õ}÷…Õx ‹¬OY¼…Õx JOïH#° XH›^¥x`Ë 7 þ 7 « 49 Þ 749

E
=°~ùH›\ ÷ ƒìyOKÇQê =KÇ°ó „¦¬e`ÇO#°, …è^¥ P „¦¬e`ÇO 2) 11, 28, 327, 464, 5125

D
†³ ò H› ø =~¡ ¾ = ò …è ^ ¥ „¦ ¬ ° #=ò#° ã‰õ } ÷ „¬ ^ ¥…Õ

9
̄á# W=Þ|_# ã‰õ}÷…Õ

59
W=Þ|_È°#°.
13 « 1 Þÿ11   23 « 8 Þÿ28
CA
95
480, 240, 120, 60, 30, 15
33 « 27 Þÿ327   43 « 64 Þÿ464
L^¥ : 1.

50
̄á# W=Þ|_# ã‰õ}÷…Õ "³ò^Î\ ÷ ‹¬OY¼ #°O_
98 53 « 125 Þÿ5125
A
ㄬu ‹¬OY¼#° '2ÑKÍ ƒìQ®OKÇQê `Ç~°¡ "Œ`Ç ‹¬OY¼
:8
=‹¬°ëOk. 3) 48, 510, 612, 714, 816, 918
E

̄á# W=Þ|_# ã‰õ}÷…Õ 4 þ 2 « 8 Þ 48


h

720, 120, 24, 6, 2


.P

L^¥ : 2.
IC

̄á# W=Þ|_# ã‰õ}÷…Õx ‹¬OY¼#° =~¡°‹¬Qê 5 þ 2 « 10 Þ 510


AD

6, 5, 4, 3`Ë ƒìQꂬðiOKÇ_ÈO [iy#k. 6 þ 2 « 12 Þ 612


AB
O

720 Ó 6 « 120 7 þ 2 « 14 Þ 714


ER

120 Ó 5 « 24
CH

24 Ó 4 « 6 8 þ 2 « 16 Þ 816
YD

6Ó3«2 9 þ 2 « 18 Þ 918
,H

63, 2, 84, 2, 91, 9, 81, 8 4) 14, 916, 2536, 4964, 81100


T
AM

L^¥ : 3.

̄á# KǶ„¬|_# ã‰õ}÷…Õ ̄á# W=Þ|_# ã‰õ}÷…Õ


H
M

    12 « 1, 22 « 4 Þÿ14
AM

; ; ;  


IG

    32 « 9, 42 « 16 Þÿ916
KH

L^¥ : 4. 42, 4, 61, 36, 93, 9 52 « 25, 62 « 36 Þÿ2536


R

̄á# KǶ„¬|_# ã‰õ}÷…Õ


72 « 49, 82 « 64 Þÿ4964


  
  92 « 81, 102 « 100 Þÿ81100
 5) 18, 2764, 125216, 343512, 7291000.
     
 ̄á# W=Þ|_# ã‰õ}÷…Õ

      13 « 1, 23 « 8 Þÿ18
 33 « 27, 43 « 64 Þÿ2764
93, 27, 61, 216, 42, 8
53 « 125, 63 « 216 Þÿ125216
L^¥ : 5.

̄á# KǶ„¬|_# ã‰õ}÷…Õ


73 « 343, 83 « 512 Þÿ343512


    

93 « 729, 103 « 1000 Þÿ7291000
REASONING ABILITY (VERBAL) ○ ○ ○ ○ ○ ○ ○ ○ ○
33
○ ○ ○ ○ ○ ○ ○ ○ ○ ○ ○ ○ ○ ○ ○ ○ ○ ○ ○
Study Material
ï~O_È= qƒìQ®O…Õx ‹¬OY¼° "³ò^Î\ ÷ ‹¬OY¼ #°O_
TEST Yourself on MODEL - 4
{1, {3, {5 ƒè‹²‹O¬ Y¼#° 䛀_ÈQê U~¡æ_<Œ~ò.
1. 23, 827, 41, 641, 31, G 48, 60, 47, 58, 44, 54, 39, 48

1) 6427 2) 271 3) 278 4) 9271   


2. 800, 400, 200, 100,

1) 50 2) 25 3) 25 4) 10
  
3. 61, 216, 14, , 31, 27

1) 48 2) 125 3) 64 4) 81 ̄á# ã‰õ}x÷ Q®=°xOz# P ã‰õ}…÷ Õ ~ï O_È° qƒìQꁰ<Œß~ò.


4. 613, 18, 862, 96, 765,
"³ò^Î\ ÷ qƒìQ®O :    
1) 144 2) 240 3) 180 4) 210   

ï~O_È= qƒìQ®O :    

Y
5. 39, 27, 68, 48, 97,

1) 64 2) 63 3) 56 4) 84

M
  

"³ò^Î\ ÷ qƒìQ®O…Õx ‹¬OY¼° "³ò^Î\ ÷ ‹¬OY¼ #°O_


MODEL - 4 : Solutions

E
1, 3, 5 =~¡°‹¬ ƒè‹‹² O¬ Y¼#° f‹²"†
Í °Ç Qê U~¡æ_<Œ~ò.
1. (2) 23 ® 2333 ® 827 ï~O_È= qƒìQ®O…Õx ‹¬OY¼° "³ò^Î\ ÷ ‹¬OY¼ #°O_

D
9
41 ® 4313 ® 641 J^Íq^ÎOQê

59
2, 4, 6 =~¡°‹¬ ‹¬i ‹¬OY¼#° f‹²"† Í °Ç Qê U~¡æ_<Œ~ò.
31 ® 3313 ® . G CA
95
271 35, 37, 40, 32, 50, 22, 65, 7.

2. (1) 800 Ó 2 « 400   400 Ó 2 « 200  

50
  

98
A
200 Ó 2 « 100

\ 100 Ó 2 «
:8
50
E

3. (3) 61 ®ÿ(6 þ 1)3 « 63 « 216     


h
.P

14 ®ÿ(1 þ 4)3 « 43 « ̄á# ã‰õ}x÷ Q®=°xOz# P ã‰õ}…÷ Õ ~ï O_È° qƒìQꁰ<Œß~ò.


IC

64
AD

31 ®ÿ(3 þ 1)3 « 33 « 27 "³ò^Î\ ÷ qƒìQ®O :    


AB
O

4. (4) 6 þ 1 þ 3 « 18, 8 þ 6 þ 2 « 96, 7 þ 6 þ 5   


ER

«
CH

210 ï~O_È= qƒìQ®O :    


5. (2) 3 þ 9 « 27, 6 þ 8 « 48, 9 þ 7 «
YD

63
  
,H

MODEL - 5 "³ò^Î\ ÷ qƒìQ®O…Õx ‹¬OY¼° "³ò^Î\ ÷ ‹¬OY¼ #°O_


T

ãH›=°OQê 5, 10, 15° H›°„¬Qê U~¡æ_<Œ~ò.


AM
H

1. ã‰õ } ÷ … Õx ï ~ O_È ° qƒ ì Qê ‹¬ O Y¼° =ô#ß ‹¬ O Y¼#°


ï~O_È= qƒìQ®O…Õx ‹¬OY¼° "³ò^Î\ ÷ ‹¬OY¼ #°O_
M
AM

䛀_È°@ …è^¥ f‹²"͆Çò@ :


ãH›=°OQê 5, 10, 15° f‹²"͆ǰQê U~¡æ_<Œ~ò.
IG

G 11, 7, 13, 8, 17, 11, 23, 16


200, 205, 215, 230, 250, 275 ‹¬OY¼° "³ò^Î\ ÷
KH

2.
  
#°O_ {5, {10, {15, {20, {25, {30...#° ãH›=°OQê
R

 H›°„¬ô`Ƕ =ôO_È°@ =# U~¡æ_# ã‰õ}÷.


   
̄#á ã‰õ}x÷ Q®=°xOz#, P ã‰õ}…÷ Õ ~ï O_È° qƒìQꁰ<Œß~ò.
     
"³ò^Î\ ÷ qƒìQ®O :    
3. 240, 230, 215, 195, 170, 140 ‹¬OY¼° "³ò^Î\ ÷
   #°O_ ãH›=°OQê 10, 15, 20, 25, 30 f‹²"†
Í °Ç Qê U~¡æ_#
ï~O_È= qƒìQ®O :     ã‰õ}÷.
   
"³ò^Î\ ÷ qƒìQ®O…Õx ‹¬OY¼° "³ò^Î\ ÷ ‹¬OY¼ #°O_
{2, {4, {6 ‹¬i‹¬OY¼#° 䛀_ÈQê U~¡æ_<Œ~ò.
    

REASONING ABILITY (VERBAL) ○ ○ ○ ○ ○ ○ ○ ○ ○


34
○ ○ ○ ○ ○ ○ ○ ○ ○ ○ ○ ○ ○ ○ ○ ○ ○ ○ ○
Study Material
TEST Yourself on MODEL - 5 MODEL - 6
1. 6, 21, 5, 20, 4, 19, , 1. Wzó# ã‰õ}÷ x3 + 1 …è^¥ x3 – 1 …è^¥ x3 + x …è^¥

1) 2, 17 2) 3, 18 3) 3, 17 4) 2, 19 x3 – x ~¡¶„¬O…Õx ã‰õ}°° :

2. 1, 2, 3, 6, 9, 12, 11, 16, 21, G x3 + 1 2, 9, 28, 65, 126, 217, 344


ã‰õ}÷ :

1) 16 2) 23 3) 29 4) 30 ̄á# W=Þ|_# ‹¬OY¼° 3 ã‰õ}÷ ~¡¶„¬O…Õ =ô<Œß~ò. x +1

3. 16, 21, 12, 17, , 13 { 1, 23 { 1, 33 { 1, 43 { 1, 53 { 1,


1) 8, 13 2) 10, 15 3) 8, 17 4) 6, 13 ¯ ¯ÿÿÿÿÿÿÿÿÿÿÿÿÿ¯ ÿÿÿÿÿÿÿÿÿ¯ ÿÿÿÿÿÿÿÿÿ¯
4. 3, 8, 13, 28, 23, 48, ,
2 9 28 65 126
1) 43, 58 2) 43, 68 3) 53, 78 4) 33, 68 6 { 1, 7 { 1
3 3
5. 193, 192, 199, 186, 205, 180, ,
ÿ¯ ÿ¯

Y
1) 209, 176 2) 213, 176 217 344

M
3) 211, 174 4) 208, 184
G x3 – 1 0, 26, 124, 342, 728, 1330
ã‰õ}÷ :

MODEL - 5 : Solutions

E
̄#á W=Þ|_# ã‰õ}…÷ Õx ‹¬OY¼° "³ò^Î\ ÷ ƒè‹² ‹¬‚ì¬ [
‹¬OY¼ 3 ã‰õ}÷ ~¡¶‡xß U~¡æ~¡°‹¬°ë<Œß~ò.

D
9
x – 1

59
  

13 - 1, 33 - 1, 53 - 1, 73 - 1, 93 - 1,
1. (2)
CA
95
YZ
¯ ÿ¯ÿÿÿÿÿÿÿÿÿÿÿ¯ ÿÿÿÿÿÿÿ¯ ÿÿÿÿÿÿÿ¯

50
   0 26 124 342 728
98
A
̄á# KǶ„¬|_# ãH›=°O #°O_, 11 - 1 3
:8
x = 4 – 1 = 3 =°i†Çò y = 19 – 1 = 18 ÿ¯
E
h

  1330
.P
IC

2. (1) G 10, 68, 222, 520, 1010


AD

Y x3 + x ã‰õ}÷ :

̄á# W=Þ|_# ã‰õ}÷…Õx ‹¬OY¼° "³ò^Î\ ÷ ‹¬i‹¬‚¬ì[


AB
O

    
‹¬OY¼ 3 ã‰õ}÷ ~¡¶„¬O…Õ U~¡æ~¡°‹¬°ë<Œß~ò.
ER

x + x
̄á# KǶ„¬|_# ãH›=°O #°O_
CH

16
23 { 2, 43 { 4, 63 { 6, 83 { 8, 103 { 10,
x = 11 + 5 =
YD

¯ ¯ÿÿÿÿÿÿÿÿÿÿÿÿ¯ ÿÿÿÿÿÿÿÿ¯ ÿÿÿÿÿÿÿÿÿÿ¯


 

3. (1)
,H

YZ 6810 222 520 1010


T
AM

  G x3 – x 6, 24, 120, 336, 1320, 2184


H

ã‰õ}÷ :

̄á# KǶ„¬|_# ãH›=°O #°O_,


M

̄á# W=Þ|_# ã‰õ}÷…Õx ‹¬OY¼° "³ò^Î\ ÷ ㄬ^¥#


AM

8 =°i†Çò
IG

x = 12 – 4 = y = 17 – 4 = 13 ‹¬OY¼ 3 ã‰õ}÷ ~¡¶„¬O…Õ U~¡æ~¡°‹¬°ë<Œß~ò.


x – x
KH

   23 - 2, 33 - 3, 53 - 5, 73 - 7, 113 - 11,
¯ ¯ÿÿÿÿÿÿÿÿÿÿÿ¯ ÿÿÿÿÿÿÿ¯ ÿÿÿÿÿÿÿÿÿ¯
R

4. (4) YZ
6 24 120 336 1320
  
̄á# KǶ„¬|_# ãH›=°O #°O_, 133 - 13
x = 23 + 10 = 33 =°i†Çò y = 48 + 20 = 68
ÿÿ¯
 
2184

2. „¦° D ~¡H"› °³ #Ø ã‰õ}°…Õ, ã‰õ}…÷ Õx ‹¬OY¼°
¬ #=ò ã‰õ}÷ :

5. (3) YZ =~¡°‹¬ ‹¬OY¼ „¦¬°#=ò…ì, …èH› =~¡°‹¬ „¦¬°# ‹¬OY¼


   „¦¬°#=ò…ì =~¡°‹¬ ‹¬i‹¬OY¼ „¦¬°#=ò…ì …èH› =~¡°‹¬ ƒè‹²
̄á# KǶ„¬|_# ãH›=°O #°O_, ‹¬OY¼ „¦¬°#=ò…ì …èH› =~¡°‹¬ ㄬ^¥# ‹¬OY¼ „¦¬°#=ò…ì
x = 205 + 6 = 211 =°i†Çò y = 180 – 6 = 174 Jx KǶ‹¬°Hù#=…ÿ#°.
REASONING ABILITY (VERBAL) ○ ○ ○ ○ ○ ○ ○ ○ ○
35
○ ○ ○ ○ ○ ○ ○ ○ ○ ○ ○ ○ ○ ○ ○ ○ ○ ○ ○
Study Material
L^¥ : 1. 1, 8, 27, 64, 125, 216 MODEL - 6 : Solutions
̄á# W=Þ|_# ã‰õ}÷…Õx ‹¬OY¼°
13, 23, 33, 43, 53, 63 ° =~¡°‹¬ ‹¬‚¬ì[
1. (2) 2, 4, 6, 8, 10 3 ~¡¶„¬O. x + x

‹¬OY¼ „¦¬°#=ò°.
23{2, 43{4, 63{6, 83{8, 103{10
L^¥ : 2. 8, 64, 216, 512, 1000, 1728 ¯ ¯ ¯ ¯ ¯
̄á# W=Þ|_# ã‰õ}÷…Õx ‹¬OY¼° 23, 43, 10 68 222 1010 520

63, 83, 103, 123° =~¡ ° ‹¬ ‹¬ i ‹¬ ‚ ¬ ì [


2. (3) 1, 2, 3, 4, 5, 6, 7 3 ~¡¶„¬O. x + 1

1 {1, 2 {1, 3 {1, 4 {1, 5 {1, 63{1, 73{1


3 3 3 3 3

¯ ¯ ¯ ¯ ¯ ¯ ¯
‹¬OY¼ „¦¬°#=ò°.

1, 27, 125, 343, 729, 1331.


2 9 28 126 217 344
L^¥ : 3.

̄á# W=Þ|_# ã‰õ}÷…Õx ‹¬OY¼° 13, 33,


65

3. (1) 1, 3, 5, 7, 9, 11 2 ~¡¶„¬O.

Y
53, 73, 93, 113° =~¡°‹¬
x – 1
ƒè‹² ‹¬‚ì
¬ [ ‹¬OY¼

13-1, 33-1, 53-1, 73-1, 93-1, 113- 1

M
¯ ¯ ¯ ¯ ¯ ¯
„¦¬°#=ò°.

L^¥ : 4. 8, 27, 125, 343, 1331, 2197 0 26 124 728 1330

E
342

̄á# W=Þ|_# ã‰õ}÷…Õx ‹¬OY¼° 4. (4) "³ò^Î\ ÷ 6 ㄬ^¥# ‹¬OY¼° 2, 3, 5, 7, 11, 13

D
9
23, 33, 53, 73, 113, 133°

59
=~¡°‹¬ ㄬ^¥#
†³òH›ø 3 ~¡¶„¬O. x – x

CA
95
23-2, 33-3, 53-5, 73-7, 113-11, 133 -13
‹¬OY¼ „¦¬°#=ò°.

64, 216, 512, 729, 1000, 1728 ¯ ¯ ¯ ¯ ¯ ¯


50
L^¥ : 5.

̄á# W=Þ|_# ã‰õ}÷…Õx ‹¬OY¼° 98


A
6 24 120 336 1320 2184
:8
43, 63, 83, 93, 103, 123 ° =~¡°‹¬ 5. (3) 53, 103, 153, 203, 253
E

ÿÿ¯ ¯ ¯ ¯ ¯
h
.P

‹¬O†ÇòH›ë ‹¬OY¼ „¦¬°#=ò°.


IC

27, 216, 729, 1728, 3375, 5832 125 1000 8000 15625
AD

L^¥ : 6. 3375

̄á# W=Þ|_# ã‰õ}÷…Õx ‹¬OY¼° (5 †³òH›ø Q®°}÷*쁰 „¦¬°#=ò°)


AB
O

33, 63, 93, 123, 153, 183°


ER

3 †³òH›ø

MODEL - 7
CH

=~¡°‹¬ Q®°}÷*ì „¦¬°#=ò°.


YD

L^¥ : 7. 64, 512, 1728, 4096, 8000 G x3 + x2 2, 12, 36, 80, 150, 252
ã‰õ}÷ :
,H

̄á# W=Þ|_# ã‰õ}÷…Õx ‹¬OY¼° ̄á# W=Þ|_# ã‰õ}÷…Õx ‹¬OY¼° 1, 2, 3, 4, 5, 6


T
AM

43, 83, 123, 163, 203° =~¡°‹ ¬ 4 †³òH›ø


JOïH 3 2 ã‰õ}÷ ~¡¶„¬O…Õ =ô<Œß~ò.
x + x
H

13 { 12, 23 { 22, 33 { 32, 43 { 42,53 { 52,


M

¯ ¯ ¯ ¯ ¯
=~¡°‹¬ Q®°}÷*ì „¦¬°#=ò°.
AM
IG

TEST Yourself on MODEL - 6 2 12 36 80 150


KH

6 { 62
3
R

1. 10, 68, 222, , 1010 ¯


1) 482 2) 520 3) 518 4) 516 252
2. 2, 9, 28, , 126, 217, 344
G x3 – x2 0, 4, 18, 48, 100, 180
ã‰õ}÷ :

1) 66 2) 72 3) 65 4) 73 ̄á# W=Þ|_# ã‰õ}÷…Õx ‹¬OY¼° 1, 2, 3, 4, 5, 6


3. 0, 26, 124, , 728, 1330
JOïH 2 2 ã‰õ}÷ ~¡¶„¬O…Õ =ô<Œß~ò.
x – x

1) 342 2) 255 3) 295 4) 313 13 - 12, 23 - 22, 33 - 33, 43 - 42, 53 - 52,


¯ ¯ ¯ ¯ ¯
4. 6, 24, 120, 336, , 2184
0 4 18 48 100
1) 1296 2) 2048 3) 1396 4) 1320 6 - 62
3
5. 125, 1000, , 8000, 15625
¯
1) 3125 2) 2175 3) 3375 4) 3250 180
REASONING ABILITY (VERBAL) ○ ○ ○ ○ ○ ○ ○ ○ ○
36
○ ○ ○ ○ ○ ○ ○ ○ ○ ○ ○ ○ ○ ○ ○ ○ ○ ○ ○
Study Material
G x3 + x2 + 1 3, 13, 37, 81, 151, 253
ã‰õ}÷ :
MODEL - 7 : Solutions
̄á# W=Þ|_# ã‰õ}÷…Õx ‹¬OY¼° 1, 2, 3, 4, 5, 6
JOïH 3 2 ã‰õ}÷ ~¡¶„¬O…Õ L<Œß~ò.
x + x + 1
1. (3) 1, 2, 3, 4, 5, 6 3 2 ~¡¶„¬O. x – x

13 { 12 { 1, 23 { 22 { 1, 33 { 33 { 1, 13 - 12, 23 - 22, 33 - 32, 43 - 42, 53 - 52,


¯ ¯ ¯ ¯ ¯
¯ ¯ ¯ 0 4 18 48 100

3 13 37 63 - 62
4 { 42 { 1,
3
5 { 52 { 1,
3
6 { 62 { 1
3
¯
¯ ¯ ¯ 180
81 151 253 2. (4) 1, 2, 3, 4, 5, 6, 7 3 2 ~¡¶„¬O. x + x

G x3 + x2 + x 3, 39, 155, 399, 819 13 { 12, 23 { 22, 33 { 32, 43 { 42,53 { 52,


¯ ¯ ¯ ¯ ¯
ã‰õ}÷ :

Y
̄á# W=Þ|_# ‹¬OY¼° 1, 3, 5, 7, 9 JOïH
ã‰õ}÷ ~¡¶„¬O…Õ =ô<Œß~ò. 2 12 36 150

M
x
3 2
+ x + x
80

1 { 1 { 1, 33 { 32 { 3, 53 { 52 { 5,
3 2 6 { 62
3

E
¯ ¯ ¯ 252

D
9
3 39 155

59
3. (1) 1, 3, 5, 7, 9 3 2 ~¡¶„¬O. x + x + x
7 { 72 { 7,
3
9 { 92 { 9
3
CA
95
13 { 12 { 1, 33 { 32 { 3, 53 { 52 { 5,
¯ ¯ ¯ ¯ ¯

50
399 819 3 39 155
98
A
G 7 { 72 { 7,
3
9 { 92 { 9
3
:8
x4 1, 16, 81, 256, 625, 1296
¯ ¯
ã‰õ}÷ :
E

̄á# W=Þ|_# ã‰õ}÷…Õx ‹¬OY¼° 1, 2, 3, 4, 5, 6


h
.P

819
IC

JOïH 4 ã‰õ}÷ ~¡¶‡xß U~¡æ~¡°‹¬°ë<Œß~ò.


x
399
AD

4. (4) 1, 2, 3, 4, 5, 6 ~¡¶„¬O. 4
14, 24, 34, 44, 54, 64
x

14, 24, 34, 44, 54, 64


AB
O

¯ ¯ ¯ ¯ ¯ ¯ ¯ ¯ ¯ ¯ ¯ ¯
ER
CH

1 16 81 256 625 1296 1 16 81 256 1296 625


YD

5. (2) 1, 2, 3, 4, 5, 6 3 2 ~¡¶„¬O. x + x – 1
TEST Yourself on MODEL - 7
,H

1 { 1 { 1, 2 { 2 - 1, 33 { 32 - 1,
3 2 3 2

¯ ¯ ¯
T
AM

1 11 35
H

‹¬¶KÇ# : (1-5) D ãH÷Ok "Œ\ ÷…Õ '?Ñ Q®°~¡°ë ªÖ#=ò…Õ ~Œ=‹²#


M

4 { 42 - 1,
3
5 { 52 - 1,
3
6 { 62 - 1
3
AM

‹¬OY¼#° H›#°Qù#°=ò?
IG

¯ ¯ ¯
KH

1. 0, 4, 18, 48, , 180

149 215
1) 141 2) 145 3) 100 4) 85
79
R

2. 2, 12, 36, , 150, 252 MODEL - 8


1) 95 2) 82 3) 79 4) 80 H ÿ =~¡¾=ò ã‰õ}÷ : D ~¡H›"³°Ø# ã‰õ}°…Õ, ã‰õ}÷…Õx ‹¬OY¼°
3. 3, 39, 155, , 819
=~¡°‹¬ ‹¬OY¼ =~¡=¾ ò…ì, …èH› =~¡°‹¬ ‹¬i‹¬OY¼ =~¡=¾ ò…ì
1) 399 2) 396 3) 440 4) 441 …èH› =~¡°‹¬ ƒè‹‹² O¬ Y¼ =~¡=¾ ò…ì …èH› =~¡°‹¬ ㄬ^¥ # ‹¬OY¼
4. 1, 16, 81, 256, , 1296
=~¡¾=ò…ì Jx KǶ‹¬°Hù#=…ÿ#°.
1, 4, 9, 16, 25, 36, 49
1) 524 2) 576 3) 484 4) 625
L^¥ : 1.

̄á# W=Þ|_# ã‰õ}÷…Õx ‹¬OY¼°


5. 1, 11, 35, , 149, 215
12, 22, 32, 42, 52, 62, 72° =~¡°‹¬ ‹¬‚¬ì[

1) 25 2) 79 3) 75 4) 50 ‹¬OY¼ =~¡¾=ò°.

REASONING ABILITY (VERBAL) ○ ○ ○ ○ ○ ○ ○ ○ ○


37
○ ○ ○ ○ ○ ○ ○ ○ ○ ○ ○ ○ ○ ○ ○ ○ ○ ○ ○
Study Material
L^¥ : 2. 4, 16, 36, 64, 100 G x2 – x 0, 2, 6, 12, 20, 30, 42, 56
ã‰õ}÷ :

̄á# W=Þ|_# ã‰õ}÷…Õx ‹¬OY¼° =~¡°‹¬ ‹¬i


̄á# W=Þ|_# ‹¬OY¼° 2 ã‰õ}÷ ~¡¶„¬O…Õ =ô<Œß~ò. x –x

22, 42, 62, 82, 102 12 - 1, 22 - 2, 32 - 3, 42 - 4, 52 - 5,


‹¬OY¼ =~¡¾=ò°
¯ ÿ¯ÿÿÿÿÿÿÿÿÿÿÿÿ¯ ÿÿÿÿÿÿÿ¯ ÿÿÿÿÿÿÿ¯
L^¥ : 3. 1, 9, 25, 49, 81, 121 0 2 6 12 20
̄á# W=Þ|_# ã‰õ}÷…Õx ‹¬OY¼° =~¡°‹¬ ƒè‹²
6 - 6, 7 - 7, 8 - 8
2 2 2

‹¬OY¼ =~¡=
¾ ò° 12, 32, 52, 72, 92, 112 ÿ¯ ¯ ÿÿÿÿÿÿÿÿ¯
L^¥ : 4. 4, 9, 25, 49, 121, 169, 289 30 42 56
̄á# W=Þ|_# ã‰õ}÷…Õx ‹¬OY¼° =~¡°‹¬
TEST Yourself on MODEL - 8
ㄬ^¥# ‹¬OY¼ =~¡¾=ò°

22, 32, 52, 72, 112, 132, 172. ‹¬¶KÇ# : (1-5) D ãH÷Ok "Œ\ ÷…Õ '?Ñ Q®°~¡°ë ªÖ#=ò…Õ =KÇ°ó

Y
‹¬OY¼#° H›#°Qù#°=ò?

16, 36, 64, 81, 100, 144, 196

M
L^¥ : 5.
1. 3, 6, 11, 18, , 38, 51

̄á# W=Þ|_# ã‰õ}÷…Õx ‹¬OY¼° =~¡°‹¬


1) 23 2) 27 3) 28 4) 25

E
‹¬O†ÇòH›ë ‹¬OY¼ =~¡¾=ò°
2. 132, 156, 182, , 240, 272

42, 62, 82, 92, 102, 122, 142.

D
1) 198 2) 210 3) 206 4) 205

9
59
v x2 + 1 x2 – 1 x2 + x
CA
95
Wzó# ã‰õ}÷ …è^¥ …è^¥ …è^¥ 3. 3, 15, 35, 63, 99, , 195, 255

x2 – x 1) 111 2) 120 3) 168 4) 143

50
JQ®°@ :

G x2 + 1 2, 5, 10, 17, 26, 37, 50 4. 2, 10, 26, 50, 82, , 170, 226

98
A
ã‰õ}÷ :

̄á# W=Þ|_# ‹¬OY¼° 2 ã‰õ}÷ ~¡¶„¬O…Õ =ô<Œß~ò. 1) 122 2) 121 3) 101 4) 119
:8
x +1
E

12 { 1, 22 { 1, 32 { 1, 42 { 1, 52 { 1,
5. 9, 36, 81, 144, , 324
h

¯ ¯ÿÿÿÿÿÿÿÿÿÿÿÿÿ¯ ÿÿÿÿÿÿÿÿÿ¯ ÿÿÿÿÿÿÿÿÿ¯ 1) 200 2) 196 3) 225 4) 176


.P
IC
AD

2 5 10 17 26 MODEL - 8 : Solutions
6 { 1, 7 { 1
AB
O

2 2

ÿ¯ ¯ 1. (2) 12 { 2« 3   22 { 2 « 6   32 { 2 « 11 
ER
CH

37 50 42 { 2 « 18   52 { 2 «   62 { 2 « 38   27
YD

G 2
x –1 0, 3, 8, 15, 24, 35, 48, 63, 80 72 { 2 « 51
,H

ã‰õ}÷ :

̄á# W=Þ|_# ‹¬OY¼° 2 ã‰õ}÷ ~¡¶„¬O…Õ =ô<Œß~ò. 1 #°O_ 7 =~¡ä›½ Q® ‹¬‚¬ì[ ‹¬OY¼ 2 ~¡¶„¬O. x + 2
T

x –1
AM

2. (2) 112 { 11 « 121 { 11 « 132 


12 - 1, 22 - 1, 32 - 1, 42 - 1, 52 - 1,
H

¯ ¯ÿÿÿÿÿÿÿÿÿÿÿ¯ ÿÿÿÿÿÿÿÿ¯ ÿÿÿÿÿÿÿÿ¯ 122 { 12 « 156 


M
AM

0 3 8 15 24 132 { 13 « 182   142 { 14 «  


IG

210

15 { 15 « 240   16 { 16 « 272.
2 2
KH

6 - 1, 7 - 1, 8 - 1, 9 - 1
2 2 2 2

ÿ¯ ÿ¯ ÿÿÿÿÿÿÿÿÿ¯ ÿ¯ 3. (4) 22 - 1 « 3   42 - 1 « 15   62 - 1 « 35  
R

35 48 63 80 82 - 1 « 63   102 - 1 « 99   122 - 1 « 143

G 2
x +x 2, 6, 12, 20, 30, 42, 56
ã‰õ}÷ :
  142 - 1 « 195   162 - 1 « 255
=~¡°‹¬ ‹¬‚¬ì[ ‹¬i‹¬OY¼ †³òH›ø 2 ~¡¶„¬O.
̄á# W=Þ|_# ‹¬OY¼° 2 ã‰õ}÷ ~¡¶„¬O…Õ =ô<Œß~ò.
x – 1

4. (1) 12 { 1 « 2   32 { 1 « 10   52 { 1 « 26  
x +x

12 { 1, 22 { 2, 32 { 3, 42 { 4, 52 { 5,
72 { 1 « 50   92 { 1 « 82   112 { 1 «
¯ ÿ¯ÿÿÿÿÿÿÿÿÿÿÿÿ¯ ÿÿÿÿÿÿÿÿÿ¯ ÿÿÿÿÿÿÿÿÿ¯   132 { 1 « 170   152 { 1 « 226
122

2 6 12 20 30
=~¡°‹¬ ƒè‹²‹¬‚¬ì[ ‹¬OY¼ †³òH›ø 2 ~¡¶„¬O.
62 { 6, 72 { 7
x + 1

ÿ¯ ¯ 5. (3) 32 « 9   62 « 36   92 « 81  122 « 144  


42 56 152 «   182 « 324
225

3 †³òH›ø =~¡°‹¬ Q®°}÷*ì =~Œ¾°.


REASONING ABILITY (VERBAL) ○ ○ ○ ○ ○ ○ ○ ○ ○
38
○ ○ ○ ○ ○ ○ ○ ○ ○ ○ ○ ○ ○ ○ ○ ○ ○ ○ ○
Study Material
+
123456789012345678901234567890121234567890123456789012345678901212
12345678901234567890123456789012123456789012345678901234567890121234567890123456789012345678901212345678901234567890123
1234567890123456789012345678901212345678901234567890123456789012123456789012345678901234567890121234567890123
1234567890123456789012345678901212345678901234567890123456789012123456789012345678901234567890121234
12345678901234567890123456789012123456789012345678901234567890121234567890123456789012345678901212345678901234567890123456789012123
12345678901234567890123456789012123456789012345678901234567890121234567890123456789012345678901212345678901234567890123
1234567890123456789012345678901212345678901234567890123456789012123456789012345678901234567890121234567890123
1234567890123456789012345678901212345678901234567890123456789012123456789012345678901234567890121234
123456789012345678901234567890121234567890123456789012345678901212
12345678901234567890123456789012123456789012345678901234567890121234567890123456789012345678901212345678901234567890123456789012123
12345678901234567890123456789012123456789012345678901234567890121234567890123456789012345678901212345678901234567890123
1234567890123456789012345678901212345678901234567890123456789012123456789012345678901234567890121234567890123
1234567890123456789012345678901212345678901234567890123456789012123456789012345678901234567890121234
12345678901234567890123456789012123456789012345678901234567890121234567890123456789012345678901212345678901234567890123456789012123
123456789012345678901234567890121234567890123456789012345678901212
TEST YOURSELF on all Models
12345678901234567890123456789012123456789012345678901234567890121234567890123456789012345678901212345678901234567890123
1234567890123456789012345678901212345678901234567890123456789012123456789012345678901234567890121234567890123
1234567890123456789012345678901212345678901234567890123456789012123456789012345678901234567890121234
12345678901234567890123456789012123456789012345678901234567890121234567890123456789012345678901212345678901234567890123456789012123
123456789012345678901234567890121234567890123456789012345678901212
12345678901234567890123456789012123456789012345678901234567890121234567890123456789012345678901212345678901234567890123
1234567890123456789012345678901212345678901234567890123456789012123456789012345678901234567890121234567890123
1234567890123456789012345678901212345678901234567890123456789012123456789012345678901234567890121234
12345678901234567890123456789012123456789012345678901234567890121234567890123456789012345678901212345678901234567890123456789012123
12345678901234567890123456789012123456789012345678901234567890121234567890123456789012345678901212345678901234567890123
1234567890123456789012345678901212345678901234567890123456789012123456789012345678901234567890121234567890123
1234567890123456789012345678901212345678901234567890123456789012123456789012345678901234567890121234
12345678901234567890123456789012123456789012345678901234567890121234567890123456789012345678901212345678901234567890123456789012123
123456789012345678901234567890121234567890123456789012345678901212
12345678901234567890123456789012123456789012345678901234567890121234567890123456789012345678901212345678901234567890123456789012123
12345678901234567890123456789012123456789012345678901234567890121234567890123456789012345678901212345678901234567890123
1234567890123456789012345678901212345678901234567890123456789012123456789012345678901234567890121234567890123
1234567890123456789012345678901212345678901234567890123456789012123456789012345678901234567890121234

<Ë\˜ : (1-20) : D ãH÷Ok ã‰õ}°…Õ '?Ñ Q®°~¡°ë ªÖ#O…Õ =KÇ°ó 10. 48, 91, 54, 92, 63, 95, 78, 100, x, y ã‰õ}÷…Õ x

‹¬OY¼#° H›#°Qù#°=ò? =°i†Çò y q°=° =~¡°‹¬Qê

1. 651, 615, 837, , 367, 376, 793, 739 1) 96, 105 2) 88, 103
1) 783 2) 873 3) 387 4) 738 3) 96, 113 4) 98, 107
11. 64, 512, 1728, , 8000
2. 13, 26, 26, 36, 39, 46, , 56

1) 52 2) 48 3) 66 4) 42 1) 4096 2) 1846 3) 1996 4) 2048


12. 30, 20, 12, 6, 2,
3. 4, 6, 8, 8, 12, 16, 16, 24, , 32, 48

1) 36 2) 28 3) 32 4) 16 1) 1 2) 2 3) 0 4) 3

Y
13. 0, 6, 20, 42, , 110
4. 340, 330, 315, , 270, 240

1) 295 2) 285 3) 290 4) 300 1) 64 2) 81 3) 80 4) 72

M
14. 0, 8, 24, 48, 80,

1) 120 2) 101 3) 96 4) 110


5. 43, 48, 58, 73, , 118

E
1) 95 2) 87 3) 91 4) 93

D
15. 5, 10, 26, 50, , 170, 290, 362

9
1) 120 2) 122 3) 145 4) 136

59
6. 16, 15, 26, 20, 36, 25, , 30, 56,

1) 44, 30 2) 46, 35
CA
95
16. 63, 4, 84, 4, 93, 9, 62,

3) 48, 40 4) 45, 40 1) 4 2) 9 3) 8 4) 27

50
98
A
7. 9, 11, 7, 9, 5, 7, 3,
17. 618, 721, 824, , 1030, 1133, 1236

1) 1 2) 7 3) 5 4) 3 1) 927 2) 924 3) 930 4) 936


:8
E
h

8. 100, 100, 95, 105, 85, 115, , 130, 50,


18. 49, 36, 56, 30, 68, , 48, 32

1) 75, 145 2) 70, 150


.P

1) 48 2) 54 3) 56 4) 42
IC

3) 65, 145 4) 75, 140


AD

19. 43, 1627, 38, 9512, 64, , 36, 9216

1) 8464 2) 6436 3) 3664 4) 36216


AB
O

9. 105, 84, 103, 87, 99, 92, p, q ã‰õ}÷…Õ p =°i†Çò


ER
CH

q q°=° =~¡°‹¬Qê 20. 13, 127, 25, 8125, 63, , 71, 3431

1) 93, 99 2) 91, 97 1) 27216 2) 51227


YD

3) 92, 100 4) 90, 103 3) 57627 4) 21627


,H

12345678901234567890123456789012
12345678901234567890123456789012123456789012345678901234567890121234567890123456789012345678901212345678901234567890123
12345678901234567890123456789012
T

q=~¡}ì`ÇàH› ["Œ|°°
12345678901234567890123456789012123456789012345678901234567890121234567890123456789012345678901212345678901234567890123
12345678901234567890123456789012 12345678901234567890123456789012
AM

12345678901234567890123456789012
12345678901234567890123456789012123456789012345678901234567890121234567890123456789012345678901212345678901234567890123
12345678901234567890123456789012 12345678901234567890123456789012
12345678901234567890123456789012123456789012345678901234567890121234567890123456789012345678901212345678901234567890123
12345678901234567890123456789012
12345678901234567890123456789012
12345678901234567890123456789012
12345678901234567890123456789012123456789012345678901234567890121234567890123456789012345678901212345678901234567890123
H

12345678901234567890123456789012
12345678901234567890123456789012123456789012345678901234567890121234567890123456789012345678901212345678901234567890123
12345678901234567890123456789012 12345678901234567890123456789012
12345678901234567890123456789012
12345678901234567890123456789012123456789012345678901234567890121234567890123456789012345678901212345678901234567890123
M
AM

̄á# KǶ„¬|_# ‹¬O|O^ÎO #°O_,


IG

1. (2)     


Þ
KH

16 + 16 = x x = 32
    
R

   4. (1) Y
Y
2. (1)
    
 
̄á# KǶ„¬|_# ãH›=°O #°O_,
  
315 – 20 = x Þ x = 295

̄á# KǶ„¬|_# ‹¬O|O^ÎO #°O_,


39 + 13 = x Þ x = 52
5. (4) Y
       

3. (3)
Y
 
̄á# KǶ„¬|_# ãH›=°O #°O_,
73 + 20 = x Þ x = 93

    

REASONING ABILITY (VERBAL) ○ ○ ○ ○ ○ ○ ○ ○ ○


39
○ ○ ○ ○ ○ ○ ○ ○ ○ ○ ○ ○ ○ ○ ○ ○ ○ ○ ○
Study Material
    14. (1) 1, 3, 5, 7, 9, 11 2 ~¡¶„¬O. x – 1

6. (2)  Y  Z 12-1, 32-1, 52-1, 72-1, 92-1, 112-1
   
¯ ¯ ¯ ¯ ¯ ¯
0 8 24 48 80 120

̄á# KǶ„¬|_# ãH›=°O #°O_, 15. (2) 2, 3, 5, 7, 11, 13, 17, 19 ㄬ^¥<ŒOHê 2
=°i†Çò
x +1
36 + 10 = x 30 + 5 = y

Þ x = 46=°i†Çò y = 35.
~¡¶„¬O.
   22{1, 32{1, 52{1, 72{1, 112{1,
7. (3) Y
¯ ¯ ¯ ¯ ¯
5 10 26 50 122

  
13 {1 17 {1 192{1
2 2
̄á# KǶ„¬|_# ãH›=°O #°O_, Þ ¯ ¯ ¯

Y
7 – 2 = x x = 5

   
170 290 362

M
8. (2)  Y  Z 16. (2) 6 3 ®ÿ6 Ó 3 « 2 ® 22 « 4

E
    8 4 ®ÿ8 Ó 4 « 2 ® 22 « 4

D
9 3 ®ÿ9 Ó 3 « 3 ® 32 « 9

9
̄á# KǶ„¬|_# ãH›=°O #°O_,

59
=°i†Çò 6 2 ®ÿ6 Ó 2 « 3 ® 32 «
CA
95
85 – 15 = x 130 + 20 = y

Þ
9

=°i†Çò
17. (1) 6 þ 3 « 18 Þÿ618
x = 70 y = 150.

50
7 þ 3 « 21 Þÿ721
  

98
A
9. (1) QR 8 þ 3 « 24 Þÿ824
:8
E

9 þ 3 « 27 Þÿ
h

  
.P

927
IC

̄á# KǶ„¬|_# ãH›=°O #°O_, 10 þ 3 « 30 Þÿ1030


AD

=°i†Çò
99 – 6 = p 92 + 7 = q 11 þ 3 « 33 Þÿ1133
AB
O

Þ p = 93=°i†Çò q = 99 12 þ 3 « 36 Þÿ1236
ER
CH

    18. (1) 49 Þÿ4 þ 9 « 36


YD

10. (4) YZ 56 Þÿ5 þ 6 « 30


,H

"³ò^Î\ ÷ JOïH þ ï~O_È= JOïH


T

   
AM

̄á# KǶ„¬|_# ãH›=°O #°O_, 68 Þÿ6 þ 8 «


H

48
M

78 { 20 =°i†Çò 100 { 7 « 48 Þÿ4 þ 8 « 32


AM

= x y

Þ
IG

=°i†Çò
19. (3) 43 Þÿ4233 Þÿ1627
x = 98 y = 107
KH

11. (1) 4 3
83
123 163 203 38 Þÿ3283 Þÿ9512
R

ÿ¯ ¯ ¯ ¯ ¯
64 512 1728 8000 4096
("³ò^Î\ ÷ JOïH)2 (ï~O_È= JOïH)3
12. (3) 6, 5, 4, 3, 2, 1 2 ~¡¶„¬O x – x 64 Þÿ6243 Þÿ 3664

62-6, 52-5, 42-4, 32-3, 22-2, 12-1 36 Þÿ3263 Þÿ9216


¯ ¯ ¯ ¯ ¯ ¯ 20. (4) 13 Þ 1333 Þÿ127
30 20 12 6 2 0
25 Þÿ2353 Þÿ8125
13. (4) 1, 3, 5, 7, 9, 11 2 ~¡¶„¬O x – x ("³ò^Î\ ÷ JOïH)3 (ï~O_È= JOïH)3
12-1, 32-3, 52-5, 72-7, 92-9, 112-11 63 Þÿ6333 Þÿ
¯ ÿ¯ ¯ ¯ ¯ ¯
21627

71 Þÿ7313 Þÿ3431
0 6 20 42 72 110
REASONING ABILITY (VERBAL) ○ ○ ○ ○ ○ ○ ○ ○ ○
40
○ ○ ○ ○ ○ ○ ○ ○ ○ ○ ○ ○ ○ ○ ○ ○ ○ ○ ○
Study Material
123456789012345678901234567890121234567890
123456789012345678901234567890121234567890
123456789012345678901234567890121234567890123456789012345678901212
12345678901234567890123456789012123456789012345678901234567890121234567890123456789012345678901212345678901234567890123
1234567890123456789012345678901212345678901234567890123456789012123456789012345678901234567890121234567890123
1234567890123456789012345678901212345678901234567890123456789012123456789012345678901234567890121234
12345678901234567890123456789012123456789012345678901234567890121234567890123456789012345678901212345678901234567890123456789012123
12345678901234567890123456789012123456789012345678901234567890121234567890123456789012345678901212345678901234567890123
1234567890123456789012345678901212345678901234567890123456789012123456789012345678901234567890121234567890123
1234567890123456789012345678901212345678901234567890123456789012123456789012345678901234567890121234
123456789012345678901234567890121234567890
123456789012345678901234567890121234567890123456789012345678901212
12345678901234567890123456789012123456789012345678901234567890121234567890123456789012345678901212345678901234567890123456789012123
12345678901234567890123456789012123456789012345678901234567890121234567890123456789012345678901212345678901234567890123
1234567890123456789012345678901212345678901234567890123456789012123456789012345678901234567890121234567890123
1234567890123456789012345678901212345678901234567890123456789012123456789012345678901234567890121234
12345678901234567890123456789012123456789012345678901234567890121234567890123456789012345678901212345678901234567890123456789012123
123456789012345678901234567890121234567890123456789012345678901212
12345678901234567890123456789012123456789012345678901234567890121234567890123456789012345678901212345678901234567890123
1234567890123456789012345678901212345678901234567890123456789012123456789012345678901234567890121234567890123
1234567890123456789012345678901212345678901234567890123456789012123456789012345678901234567890121234
PRACTICE TEST on TYPE - I
123456789012345678901234567890121234567890
12345678901234567890123456789012123456789012345678901234567890121234567890123456789012345678901212345678901234567890123456789012123
123456789012345678901234567890121234567890123456789012345678901212
12345678901234567890123456789012123456789012345678901234567890121234567890123456789012345678901212345678901234567890123
1234567890123456789012345678901212345678901234567890123456789012123456789012345678901234567890121234567890123
1234567890123456789012345678901212345678901234567890123456789012123456789012345678901234567890121234
12345678901234567890123456789012123456789012345678901234567890121234567890123456789012345678901212345678901234567890123456789012123
12345678901234567890123456789012123456789012345678901234567890121234567890123456789012345678901212345678901234567890123
1234567890123456789012345678901212345678901234567890123456789012123456789012345678901234567890121234567890123
1234567890123456789012345678901212345678901234567890123456789012123456789012345678901234567890121234
12345678901234567890123456789012123456789012345678901234567890121234567890123456789012345678901212345678901234567890123456789012123
123456789012345678901234567890121234567890123456789012345678901212
12345678901234567890123456789012123456789012345678901234567890121234567890123456789012345678901212345678901234567890123456789012123
12345678901234567890123456789012123456789012345678901234567890121234567890123456789012345678901212345678901234567890123
1234567890123456789012345678901212345678901234567890123456789012123456789012345678901234567890121234567890123
1234567890123456789012345678901212345678901234567890123456789012123456789012345678901234567890121234

PRACTICE TEST - 1
1. 22, 16, 34, 49, 46, ........ 12. 165, 195, 255, 285, 345, .........

1) 56 2) 81 3) 100 4) 121 1) 375 2) 435 3) 535 4) 635


2. 6, 15, 35, 77, 143, .......... 13. 4, 11, 31, 90, 266, .........

1) 144 2) 121 3) 169 4) 221 1) 361 2) 429 3) 793 4) 525


3. 6, 27, 128, 629, .......... 14. 400, 361, 324, 289, 256, ......

1) 4163 2) 4789 3) 3130 4) 3579 1) 220 2) 240 3) 225 4) 264


4. 15.

Y
16, 33, 65, 131, 261, ......... 1, 1, 3, 9, 6, 36, 10, 100, ........, 225

1) 523 2) 521 3) 613 4) 721 1) 20 2) 34 3) 15 4) 16

M
5. 350, 520, 738, ........ 16. 224, 223, 221, 218, 214, 209, ..........

1010 989 1000 990 203 204 205 208

E
1) 2) 3) 4) 1) 2) 3) 4)

6. 17.

D
7, 26, 63, 124, 215, 342, ......... 7, 14, 28, ..........., .........., 224

9
481 511 391 421 56, 72 56, 112

59
1) 2) 3) 4) 1) 2)

CA
95
7. 84, 64, 63, 27, 92, .......... 3) 112, 124 4) 76, 89
256 343 289 525

50
1) 2) 3) 4) 18. 2, 3, 5, 9, 17, 33, 65, ...........

131 98 140 129 127


A
8. 23, 6, 34, 12, 36, 18, 45, 20, 64, ....... 1) 2) 3) 4)
:8
1) 20 2) 28 3) 24 4) 30 19. 43, 7, 54, 9, 65, 11, .........., 13
E

69 88 95 76
h

9. 1) 2) 3) 4)
.P

36, 28, 22, 18, 16, ..........


IC

1) 15 2) 13 3) 11 4) 16 20.
AD

130, 222, 350, 520, ..........

10. 9, 4, 9, 5, 11, ........... 17 1) 652 2) 757 3) 738 4) 694


AB
O

1) 7 2) 8 3) 6 4) 23 21. 0, 7, 26, 63, 124, 215, .........


ER
CH

11. 1, 4, 15, 80, 567, ......... 1) 342 2) 442 3) 542 4) 642


YD

1) 6248 2) 6723 3) 6327 4) 6273


,H

12345678901234567890123456789012
12345678901234567890123456789012123456789012345678901234567890121234567890123456789012345678901212345678901234567890123
12345678901234567890123456789012

q=~¡}ì`ÇàH› ["Œ|°°
12345678901234567890123456789012123456789012345678901234567890121234567890123456789012345678901212345678901234567890123
12345678901234567890123456789012
12345678901234567890123456789012 12345678901234567890123456789012
12345678901234567890123456789012123456789012345678901234567890121234567890123456789012345678901212345678901234567890123
12345678901234567890123456789012
T

12345678901234567890123456789012 12345678901234567890123456789012
12345678901234567890123456789012123456789012345678901234567890121234567890123456789012345678901212345678901234567890123
AM

12345678901234567890123456789012
12345678901234567890123456789012123456789012345678901234567890121234567890123456789012345678901212345678901234567890123
12345678901234567890123456789012 12345678901234567890123456789012
12345678901234567890123456789012123456789012345678901234567890121234567890123456789012345678901212345678901234567890123
12345678901234567890123456789012 12345678901234567890123456789012
12345678901234567890123456789012
12345678901234567890123456789012123456789012345678901234567890121234567890123456789012345678901212345678901234567890123
H
M

ã‰õ}…÷ Õx "³ò^Î\ ÷ ‹¬OY¼…Õx JOïH#° H›°„¬Qê, =zó# 6. (2) ‹¬OY¼° : (23-1), (33-1), (43-1), (53-1)
AM

(3)
IG

1.

^¥x †³òH›ø =~¡=¾ ò ã‰õ}…÷ Õx `Ç~ŒÞ`Ç ‹¬OY¼ J=ô`Ç°Ok. (63-1), (73-1) ....... Hê|\÷“ `Ç~¡°"Œ`Ç ‹¬OY¼
KH

2. (4) 2 #°Oz =~¡°‹¬Qê =KÇ°ó ㄬ^¥# ‹¬OY¼#°, P `Ç~ŒÞ`Ç « (83-1) « 511.


R

=KÇ°ó ㄬ^¥# ‹¬OY¼KÍ Q®°}÷OKŒe. J#Qê 2þ3«6, 7. (2) Wzó# ã‰õ } ÷ … Õx "³ ò ^Î \ ÷ ‹¬ O Y¼…Õx JOï H #°
3þ5«15, 5þ7«35, 7þ11«77, 11þ13«143 XH›^¥x…Õ #°Oz XH›\ ÷ f‹²"† Í °Ç Qê =KÇ°ó ‹¬OY¼ †³òH›ø
J…ìöQ 13þ17«221 J=ô`Ç°Ok. „¦¬°#=ò#° ^¥x `Ç~ŒÞ`Ç ‹¬OY¼Qê ã"Œ†Ç°|_#k.
3. (3) ã‰õ}÷…Õx ‹¬OY¼° =~¡°‹¬Qê 51{1, 52{2, 53{3, 95 75 74 38 :3 454
54{4 x†Ç°=°O#° J#°‹¬i‹¬°ë<Œß~ò. 9.5¬54 7.4¬44 :.3¬84
4. (1) ㄬu ‹¬OY¼ ^¥x =òO^ΰ#ß ‹¬OY¼ä›½ ï~\÷“O„¬ô K͋² 8. (3) "³ò^Î\ ÷ ‹¬OY¼…Õ JOïH#° Q®°}÷OKÇQê ï~O_È= ‹¬OY¼
^¥xH÷ 1 H›„¬_ÈO (…è^¥) f‹²"͆ǰ_ÈO [~¡Qêe. Hê|\÷“ =‹¬°ëOk. J…ìöQ q°ye#q 䛀_¨
`Ç~¡°"Œ`Ç ‹¬OY¼ (2þ261{1) « 523. 23 « 2þ3 « 6 34 « 3þ4 « 12
5. (1) Wzó# ã‰õ}÷ =~¡°‹¬Qê 73{7, 83{8, 93{9 J#° 36 « 3þ6 « 18 45 « 4þ5 « 20
x†Ç°=°O#° J#°‹¬i‹¬°ëOk. 64 « 6þ4 « 24 "³ò^Î\ ÷ JOïH þ ï~O_È= JOïH
REASONING ABILITY (VERBAL) ○ ○ ○ ○ ○ ○ ○ ○ ○
41
○ ○ ○ ○ ○ ○ ○ ○ ○ ○ ○ ○ ○ ○ ○ ○ ○ ○ ○
Study Material
9. (4) 47!!!!!!39!!!!!!33!!!!!29!!!!!!27!!!!!27 1{2«3 Þÿ32 « 9
.9!!!!!!!.7!!!!!!.5!!!!!.3!!!!!!!1 3{3«6 Þÿ62 « 36
6{4«10 Þÿ102 « 100
   

10. (2) :!!!!!!!5!!!!!!!!:!!!!!!!6!!!!!!!22!!!!!9!!!!!!28 10{5«15 Þÿ152 « 225


.6!!!!! 3|2!!!!!.5!!!! 3|2!!!.4!!!! 3|2 16. (1) =~¡°‹¬Qê -1, -2, -3, -4, -5, -6 K͆ǰQê
209 - 6 « 203
11. (1) Wzó# ã‰õ}÷ ㄬHê~¡O ㄬu ‹¬OY¼#° =~¡°‹¬ ㄬ^¥#
-1 -2 -3 -4 -5 -6
‹¬OY¼`Ë Q®°}÷Oz =zó# ‹¬OY¼#° J^Í ã„¬^¥ # ‹¬OY¼ä›½
H›„¬|_Ok. 224 223 221 218 214 209 203
11þ2{2 44þ3{3 1515þ5{5 8080þ7{7 567567þ11{11 17. (2) WH›ø_È 7þ2«14, 14þ2«28, .... Hê=ô# D ã‰õ}÷
G.P. …Õ LOk.
6248
Hê=ô# a = 7, r = 2

Y
12. (2) Wzó# ã‰õ}÷…Õ 15 J#° ‹¬OY¼#° 11 #°O_ =~¡°‹¬Qê 4= „¬^ÎO « arn–1 = 7(2)4–1 = 56

M
=KÇ°ó ㄬ^¥# ‹¬OY¼`Ë Q®°}÷OKÇ|_#k. 15þ11 « 5= „¬^ÎO « arn–1 = 7(2)5–1 = 112
\ 4, 5= „¬^¥° « 56, 112 (…è^¥)

E
165, 15þ13 « 195, 15þ17 « 255, 15þ19 «
285, 15þ23 « 345. Hê=ô# `Ç~ŒÞ`Ç =KÇ°ó ‹¬OY¼ 7þ2«14, 14þ2«28, Hê=ô# J^Í q ^ Î O Qê

D
9
59
15þ29 « 435 J=ô`Ç°Ok. 28þ2«56, 56þ2«112 J=ô`Œ~ò.
CA
95
13. (3) Wzó# ã‰õ}÷ 18. (3) 2þ2-1 « 3 9þ2-1 « 17

50
3þ2-1 « 5 17þ2-1 « 33
5!!!!!!!22!!!!!!!!42!!!!!!!:1!!!!!!!377!!!!!8:4
5þ2-1 « 9 98 33þ2-1 « 65
A
4.2 4.3 4.4 4.5 4.6
:8
65þ2-1 « 129
14. (3) J=~˂¬ì} ãH›=°O…Õ =~¡¾=ò°
E

19. (4) Wzó# ã‰õ}÷…Õx "³ò^Î\ ÷ ‹¬OY¼…Õx JOïH †³òH›ø


h

202, 192, 182, 172, 162, 152


.P

=~Œ¾  =°^ Î ¼ ƒ è ^ Î O ï ~ O_È = ‹¬ O Y¼ J=ô`Ç ° Ok.


IC
AD

15. (3) 42-32«7, 52-42«9, 62-52«11, 72-62«13. (…è^¥)


36, 10, 100, ....
AB
O

1, 1, 3, 9, 6, 225 "³ò^Î\ ÷ ã‰õ}÷…Õx JOïH° "³ò`ÇëO « ï~O_È= ã‰õ}÷.


Wzó# ã‰õ}÷ 53{5, 63{6, 73{7, 83{8, 93{9
ER

20. (3)
CH

WH›ø_È "³ò^Î\ ÷ ã‰õ}÷…Õ ‹¬OY¼ä›½ =~¡°‹¬Qê {2, {3, q^ÎOQê LOk.


YD

{4, {5° H›°„¬Qê =zó# ‹¬OY¼ †³òH›ø =~¡¾=ò#° 21. (1) D ã‰õ}÷ 13-1, 23-1, 33-1, 43-1, 53-1, 63-1,
,H

ï~O_È= ã‰õ}÷…Õ `Ç~ŒÞ`Ç ‹¬OY¼Qê ã"Œ†Ç°@O [iyOk. 73-1 ~¡¶„¬O…Õ L#ßk.


T
AM
H

TYPE - II
M
AM
IG

G D qƒìQ®=ò#䛽 ‹¬O|OkOz# ㄬ‰×߁…Õ XH› JH›Æ~¡ã‰õ}÷ W=Þ|_È°`Ç°Ok. W=Þ|_# P ã‰õ}÷…Õx ãH›=°=ò#°  =~¡°‹¬Qê
KH

*ìãQ®`ÇëQê „¬ijeOz  J=Qꂬì# K͋¬°Hùx P ã‰õ}÷…Õ W=Þ|_# ãH›=°„¬^Μu (x†Ç°=°=ò)#° J#°‹¬iOz `Ç~¡°"Œ`Ç ~Œ=ež#
R

JH›Æ~¡=ò#° (…è^¥) „¬^Î=ò#° H›#°Qù#=‹² =ôO@°Ok.


G D qƒìQ®=ò#䛽 ‹¬O|OkOz# ㄬ‰×߁° =ü_È° ~¡HꁰQê =ôO\ì~ò.
i) JH›Æ~¡ ã‰õ}÷…Õ ~òzó# ㄬ‰§ß~¡ÖH›O (?)…Õ =KÍó JH›Æ~¡=ò (…è^¥) „¬^Î=ò#° H›#°Qù#°=ò.
ii) JH›Æ~¡ã‰õ}÷…Õ Mìm…Õ =KÇ°ó JH›Æ~¡ q°=#° H›#°Qù#°@.
iii) ãH›=¶#°Q®`Ç ã‰õ}÷

A B C D E F G H I J K L M N O P Q R S T U V W X Y Z
®
(A to Z)

1 2 3 4 5 6 7 8 9 10 11 12 13 14 15 16 17 18 19 20 21 22 23 24 25 26

¬
(Z to A) Z Y X W V U T S R Q P O N M L K J I H G F E D C B A

REASONING ABILITY (VERBAL) ○ ○ ○ ○ ○ ○ ○ ○ ○


42
○ ○ ○ ○ ○ ○ ○ ○ ○ ○ ○ ○ ○ ○ ○ ○ ○ ○ ○
Study Material
G JH›Æ~¡ã‰õ}÷…Õx ~¡Hꁰ :
a) JH›Æ~¡=¶…Õ ̄~¡°Q®°^΁ ã‰õ}÷ (Increasing series)
b) JH›Æ~¡=ò……Õ `Ç~¡°Q®°^΁ ã‰õ}÷ (Decreasing series)
c) q°ã‰×=° JH›Æ~¡ã‰õ}÷ (Mixed letter series)
d) XH›\ ÷ q_z =°~ùH›\ ÷ „¬iQ®}#…ÕxH÷ f‹¬°Hù<Í ã‰õ}÷ (Alternate series)
e) „¬ô<Œ~¡=$`Çë=ò J†Í°¼ JH›Æ~¡ã‰õ}÷ (Repeating letter series)
a) JH›Æ~¡=¶…Õ ̄~¡°Q®°^΁ ã‰õ}÷ (Increasing series) :
L^¥ : A, B, D, G, K, ..........
Sol : " }}m

# }}m 
% }}m 
( }}m

, }}m

1
\ ã‰õ}÷…Õ `Ç~¡°"Œ`Ç JH›Æ~¡=ò = P.

Y
b) JH›Æ~¡=¶…Õ `Ç~¡°Q®°^΁ ã‰õ}÷ (Decreasing series) :

M
L^¥ : Z, Y, V, Q, J, .............
}}m : }}m 7 }}m 2 }}m + }}m "

E
Sol : ;     

\ ã‰õ}÷…Õ `Ç~¡°"Œ`Ç JH›Æ~¡=ò = A.

D
9
ÿ

59
q°ã‰×=° JH›Æ~¡ã‰õ}÷ (Mixed letter series) :
CA
95
c)

L^¥ : BJ, CL, DN, EP, .............

50
+2 +2 +2
98
A
:8
Sol : BJ, CL, DN, EP FR
E
h

+1 +1 +1
.P
IC

Wzó# ã‰õ}÷…Õ ㄬu "³ò^Î\ ÷ JH›Æ~¡=ò `Ç~¡°"Œ`Ç JH›Æ~¡=ò#䛽 {1 ̄~¡°Q®°`Ç°Ok. J^Íq^Î=òQê ㄬu ï~O_È= JH›Æ~¡=ò
AD

^¥x `Ç~¡°"Œ`Ç JH›Æ~¡=ò#䛽 {2 ̄~¡°Q®°`Ç°Ok.


AB
O

Hê=ô# E+1= F
ER
CH

P+2= R
YD

d) XH›\ ÷ q_z =°~ùH›\ ÷ „¬iQ®}#…ÕxH÷ f‹¬°Hù<Í ã‰õ}÷ (Alternate series) :


,H

L^¥ : Z, F, Y, G, X, H, W, I, .............
T
AM

+1 +1 +1
H
M

Z, F, Y, G, X, H, W, I, V
AM

Sol :
IG
KH

–1 –1 –1
Wzó# ã‰õ}÷…Õ 1 ® 3 ® 5 ® 7 JH›Æ~Œ° `Ç~¡°"Œ`Ç JH›Æ~ŒxH÷ -1Qê =ôOk. J^Í q^Î=òQê 2 ®4®6
R

® 8 JH›Æ~Œ° `Ç~¡°"Œ`Ç JH›Æ~ŒxH÷ {1 Qê =ôOk.


Hê=ô# W – 1 = V
I+1= J
e) „¬ô<Œ~¡=$`ÇëO J†Í°¼ JH›Æ~¡ ã‰õ}÷ (Repeating letter series) :

L^¥ : bca – bb – acb – cacb – cac


1) ccbb 2) cbcb 3) bcbc 4) abab
Sol : Wzó# ã‰õ}÷…Õ Mìm`Ë H›°„¬ôHùx "³ò`ÇëO 19 JH›Æ~¡=ò° H›=ô. "Œ\÷ options …Õx …ÿ@~Ÿ`Ë W<£‹¬~Ÿ“ K͋¬¶ë
̋\˜Qê q_ÈQù@°“KǶ ̋\˜ „¬ô#~Œ=$`ÇëO J†Í°¼q^ÎOQê KǶ_È=…ÿ#°.
Option (1) ㄬHê~¡=ò bcacb/bcacb/bcacb/bcac. Hê=ô# „¬ô#~Œ=$`ÇëO J†Í°¼ ̋\˜ bcacb

REASONING ABILITY (VERBAL) ○ ○ ○ ○ ○ ○ ○ ○ ○


43
○ ○ ○ ○ ○ ○ ○ ○ ○ ○ ○ ○ ○ ○ ○ ○ ○ ○ ○
Study Material
PRACTICE TEST - 1
‹¬¶KÇ# : (1-20) D ãH÷Ok ã‰õ}°…Õ Mìm ªÖ#O…Õ =KÇ°ó 16. C, ......, K, O, S

JH›Æ~Œxß H›#°HËøO_? 1) D 2) E 3) F 4) G
1. ..... , M, P, T, Y 17. ....., K, O, S, W

1) I 2) H 3) K 4) L 1) H 2) G 3) I 4) J
2. Z, X, U, Q, ..... 18. Z, V, R, .....

1) L 2) K 3) J 4) I 1) O 2) M 3) N 4) T
3. H, J, M, Q, .....

Y
19. ....., G, K, O, S

1) Y 2) X 3) W 4) V 1) C 2) D 3) B 4) A

M
4. C, E, H, L, .....
20. A, E, ..... , M, Q

E
1) N 2) O 3) P 4) Q
1) G 2) H 3) J 4) I

D
9
5. A, C, ..... , J, O

59
21. K, N, Q, ..... , W
1) B 2) F 3) D 4) E
CA
95
1) S 2) T 3) U 4) Z

50
6. D, J, P, V, .....
22. G, ..... , M, P, S

98
A
1) Y 2) Z 3) A 4) B
1) J 2) I 3) K 4) H
:8
7. ....., S, Y, E, K
E
h

23. Z, W, T, Q, .....
1) M 2) N 3) O 4) P
.P
IC

1) P 2) O 3) N 4) M
AD

8. Z, T, N, H, .....
24. E, H, K, N, .....
1) D 2) A 3) B 4) C
AB
O

1) O 2) P 3) Q 4) R
ER

9. E, K, Q, ..... , C
CH

1) V 2) W 3) X 4) Y 25. ....., D, G, J, M
YD

1) A 2) B 3) C 4) E
,H

10. A, ..... , M, S, Y
T

1) D 2) E 3) J 4) G
AM

26. ....., L, N, P, R
H

1) G 2) I 3) J 4) K
M

11. G, L, Q, ....., A
AM

1) T 2) U 3) V 4) W
IG

27. R, ....., V, X, Z
KH

12. I, ....., S, X 1) S 2) U 3) T 4) W
R

1) K 2) L 3) M 4) N 28. G, I, K, M, .....

13. Z, U, P, .....
1) O 2) N 3) P 4) Q
1) M 2) L 3) K 4) J 29. Z, X, ....., T, R
14. B, G, L, Q, .....
1) Y 2) V 3) W 4) U
1) S 2) T 3) V 4) W
30. M, O, Q, S, .....
15. A, ......, K, P, U
1) N 2) P 3) R 4) U
1) C 2) D 3) E 4) F

REASONING ABILITY (VERBAL) ○ ○ ○ ○ ○ ○ ○ ○ ○


44
○ ○ ○ ○ ○ ○ ○ ○ ○ ○ ○ ○ ○ ○ ○ ○ ○ ○ ○
Study Material
12345678901234567890123456789012
12345678901234567890123456789012123456789012345678901234567890121234567890123456789012345678901212345678901234567890123
12345678901234567890123456789012

q=~¡}ì`ÇàH› ["Œ|°°
12345678901234567890123456789012123456789012345678901234567890121234567890123456789012345678901212345678901234567890123
12345678901234567890123456789012 12345678901234567890123456789012
12345678901234567890123456789012123456789012345678901234567890121234567890123456789012345678901212345678901234567890123
12345678901234567890123456789012 12345678901234567890123456789012
12345678901234567890123456789012123456789012345678901234567890121234567890123456789012345678901212345678901234567890123
12345678901234567890123456789012 12345678901234567890123456789012
12345678901234567890123456789012123456789012345678901234567890121234567890123456789012345678901212345678901234567890123
12345678901234567890123456789012 12345678901234567890123456789012
12345678901234567890123456789012123456789012345678901234567890121234567890123456789012345678901212345678901234567890123
12345678901234567890123456789012 12345678901234567890123456789012
12345678901234567890123456789012123456789012345678901234567890121234567890123456789012345678901212345678901234567890123
12345678901234567890123456789012 12345678901234567890123456789012

1. (3) K, L, M, NO, P, QRS, T, UVWX, Y Þÿ K, M, P, T, Y 15. (4) A, BCDE, F, GHIJ, K, LMNO, P, QRST, U
+4 +4 +4 +4
+1 +2 +3 +4

2. (1) Z, Y, X, WV, U, TSR, Q, PONM, L ÞÿZ, X, U, Q, L ÿÿÿÿÿÿÿÿÿÿÞÿA, F, K, P, U

–1 –2 –3 –4
16. (4) C, DEF, G, HIJ, K, LMN, O, PQR, S
+3 +3 +3 +3
3. (4) H, I, J, KL, M, NOP, Q, RSTU, V ÞÿH, J, M, Q, V ÞÿC, G, K, O, S
+1 +2 +3 +4 17. (2) G, HIJ, K, LMN, O, PQR, S, TUV, W
4. (4) C, D, E, FG, H, IJK, L, MNOP, Q ÞÿC, E, H, L, Q
+3 +3 +3 +3
ÞÿG, K, O, S, W

Y
+1 +2 +3 +4 18. ZYXW VUTS RQPON

M
(3)

5. (2) A, B, C, DE, F, GHI, J, KLMN, O ÞÿA, C, F, J, O


+3 +3 +3

E
+1 +2 +3 +4 19. (1) C, DEF, G, HIJ, K, LMN, O, PQR, S
+3 +3 +3 +3

D
9
6. (4) D, EFGHI, J, KLMNO, P, QRSTU, V, WXYZA, B
Þÿ , G, K, O, S

59
C

CA A, BCD, E, FGH, I, JKL, M, NOP, Q ÞÿA, E, M, Q

95
+5 +5 +5 +5
20.
ÞÿD, J, P, V, (4) I,

50
B
+3 +3 +3 +3
7. (1) M, NOPQR, S, TUVWX, Y, ZABCD, E, FGHIJ, K
98 ÞÿK, N, Q,
A
21. (2) K, LM,
+2
N, OP,
+2
Q, RS,
+2
T, UV,
+2
W T, W
:8
+5 +5 +5 +5

Þÿ
E
h

M, S, Y, E, K
ÞÿG,
.P

22. (1) G, HI, J, KL, M, NO, P, QR, S J, M, P, S


IC

8. (3) Z, YXWVU, T, SRQPO, N, MLKJI, H, GFEDC, B


AD

+2 +2 +2 +2
–5 –5 –5 –5
ÞÿZ, W, T, Q, N
AB
O

23. (3) Z, YX, W, VU, T, SR, Q, PO, N


ÞÿZ, T, N, H, B
ER
CH

–2 –2 –2 –2

9. E, FGHIJ, K, LMNOP, Q, RSTUV, W, XYZAB, C 24. E, FG, H, IJ, K, LM, N, OP, Q ÞÿE, H, K, N,
YD

(2) (3) Q
+5 +5 +5 +5
,H

+3 +3 +3 +3

ÞÿE, K, Q, ÞÿA, D, G, J, M
T

W, C 25. A, BC, D, EF, G, HI, J, KL, M


AM

(1)
H

10. (4) A, BCDEF, G, HIJKL, M, NOPQR, S, TUVWX, Y


M

+5 +5 +5 +5
ÞÿJ, L, N, P, R
AM

26. J, K, L, M, N, O, P, Q, R
IG

(3)

ÞÿA,
KH

G, M, S, Y
27. R, S, T, U, V, W, X, Y, Z ÞÿR, T, V, X, Z
R

(3)
11. (3) G, HIJK, L, MNOP, Q, RSTU, V, WXYZ, A
+4 +4 +4 +4

ÞÿG, L, Q, V, A
28. (1) G, H, I, J, K, L, M, N, O ÞÿG, I, K, M, O
12. (4) I, JKLM, N, OPQR, S, TUVW, X ÞÿI, N, S, X
29. (2) Z, Y, X, W, V, U, T, S, R ÞÿZ, X, V, T, R
+4 +4 +4

13. (3) Z, YXWV, U, TSRQ, P, ONML, K ÞÿZ, U, P, K


30. (4) M, N, O, P, Q, R, S, T, U,
+4 +4 +4

14. (3) B, CDEF, G, HIJK, L, MNOP, Q, RSTU, V ÿÞÿM, O, Q, S, U


+4 +4 +4 +4

ÞÿB, G, L, Q, V

REASONING ABILITY (VERBAL) ○ ○ ○ ○ ○ ○ ○ ○ ○


45
○ ○ ○ ○ ○ ○ ○ ○ ○ ○ ○ ○ ○ ○ ○ ○ ○ ○ ○
Study Material
123456789012345678901234567890121234567890123456789012345678901212
12345678901234567890123456789012123456789012345678901234567890121234567890123456789012345678901212345678901234567890123456789012123
1234567890123456789012345678901212345678901234567890123456789012123456789012345678901234567890
12345678901234567890123456789012123456789012345678901234567890121234567890123456789012345678901212345678901234567890123
1234567890123456789012345678901212345678901234567890123456789012123456789012345678901234567890121234567890123
1234567890123456789012345678901212345678901234567890123456789012123456789012345678901234567890121234
123456789012345678901234567890121234567890123456789012345678901212
12345678901234567890123456789012123456789012345678901234567890121234567890123456789012345678901212345678901234567890123456789012123
1234567890123456789012345678901212345678901234567890123456789012123456789012345678901234567890
12345678901234567890123456789012123456789012345678901234567890121234567890123456789012345678901212345678901234567890123
1234567890123456789012345678901212345678901234567890123456789012123456789012345678901234567890121234567890123
1234567890123456789012345678901212345678901234567890123456789012123456789012345678901234567890121234
12345678901234567890123456789012123456789012345678901234567890121234567890123456789012345678901212345678901234567890123456789012123
1234567890123456789012345678901212345678901234567890123456789012123456789012345678901234567890
123456789012345678901234567890121234567890123456789012345678901212
12345678901234567890123456789012123456789012345678901234567890121234567890123456789012345678901212345678901234567890123
1234567890123456789012345678901212345678901234567890123456789012123456789012345678901234567890121234567890123
1234567890123456789012345678901212345678901234567890123456789012123456789012345678901234567890121234
PREVIOUS BITS ON TYPE - I
12345678901234567890123456789012123456789012345678901234567890121234567890123456789012345678901212345678901234567890123456789012123
1234567890123456789012345678901212345678901234567890123456789012123456789012345678901234567890
123456789012345678901234567890121234567890123456789012345678901212
12345678901234567890123456789012123456789012345678901234567890121234567890123456789012345678901212345678901234567890123
1234567890123456789012345678901212345678901234567890123456789012123456789012345678901234567890121234567890123
1234567890123456789012345678901212345678901234567890123456789012123456789012345678901234567890121234
12345678901234567890123456789012123456789012345678901234567890121234567890123456789012345678901212345678901234567890123456789012123
1234567890123456789012345678901212345678901234567890123456789012123456789012345678901234567890
12345678901234567890123456789012123456789012345678901234567890121234567890123456789012345678901212345678901234567890123
1234567890123456789012345678901212345678901234567890123456789012123456789012345678901234567890121234567890123
1234567890123456789012345678901212345678901234567890123456789012123456789012345678901234567890121234
12345678901234567890123456789012123456789012345678901234567890121234567890123456789012345678901212345678901234567890123456789012123
1234567890123456789012345678901212345678901234567890123456789012123456789012345678901234567890
123456789012345678901234567890121234567890123456789012345678901212
12345678901234567890123456789012123456789012345678901234567890121234567890123456789012345678901212345678901234567890123456789012123
1234567890123456789012345678901212345678901234567890123456789012123456789012345678901234567890
12345678901234567890123456789012123456789012345678901234567890121234567890123456789012345678901212345678901234567890123
1234567890123456789012345678901212345678901234567890123456789012123456789012345678901234567890121234567890123
1234567890123456789012345678901212345678901234567890123456789012123456789012345678901234567890121234
12345678901234567890123456789012123456789012345678901234567890121234567890123456789012345678901212345678901234567890123456789012123
1234567890123456789012345678901212345678901234567890123456789012123456789012345678901234567890

1. W=Þ|_# ‹¬OY¼ ã‰õ}÷…Õ `Ç~¡°"Œ`Ç ‹¬OY¼ …è^ΰ. ‹¬i 12. 5, 7, 18, 60, 248 ......
J~ò# ‹¬=¶^¥<Œxß Q®°iëOKÇO_. (Assistant Director of Economics Exam -2012)

ã‰õ}÷ « 4, 6, 12, 14, 28, 30 ........... 1) 1250 2) 1280 3) 1180 4) 1140


(Multi Tasking Staff Exam - 2014) 13. 6, 15, 47, 97, 293, ........
1) 32 2) 64 3) 62 4) 60 (Assistant Director of Engineers Exam -2012)

2. 272, 136, 68, 34, ? (Group - D, 2014) 1) 486 2) 846 3) 589 4) 524
1) 6 2) 17 3) 12 4) 16 14. 2, 15, 41, 80, .......
(Assistant Director of Economics Exam -2012)
‹¬ ¶ KÇ # ° : (3-4) =~¡ ä › ½ Q®  J#°ãH› = ¶° XH› „¬ ^ Î œ u
1) 132 2) 121 3) 111 4) 120

Y
##°‹¬iªë~ò. Jk Q®°iëOz, ‹¬i†³Ø°# ‹¬=¶^¥<Œxß
15. ㄬ‰§ß~¡ÖH›O L#ßKË@ LO_È=‹²# ‹¬OY¼

M
Z#°ßHùx Mìm#° „¬îiOKŒe. (VRO - 2014)
2, 6, 14, 30, 62, ?, 254
3. 1, 3, 6, 10, 15, -, 28, 36

E
(Assistant Statistical Officer Exam -2012)
1) 18 2) 21 3) 23 4) 23

D
1) 124 2) 125 3) 126 4) 127

9
59
4. 1, 4, 10, 28, -, 244
16. ㄬ‰§ß~¡ÖH›O L#ßKË@ LO_È=‹²# ‹¬OY¼
CA
95
1) 62 2) 72 3) 82 4) 192 15, 18, 22, 27, ?

50
‹¬¶KÇ#° : (5-10) ㄬ‰§ß~¡ÖH›=ò (?) Q® ªÖ#=ò#° ‹¬i J~ò# (Assistant Statistical Officer Exam -2012)

98
A
‹¬=¶^¥#=ò`Ë „¬îiO„¬ô=ò. (H›=ü¼xöH+¬<£ S.I - 2012) 1) 38 2) 33 3) 35 4) 39
:8
5. 325, 259, 204, 160, 127, 105, ? 
E
h

17. 4, 6, 9, 13 ,? (S.I - 2012)


1) 94 2) 96 3) 98 4) 100 
.P
IC

6. 1, 2, 6, 7, 21, 22, 66, 67, ?


AD

  
1) 17 2) 22 3) 20 4) 19
1) 70 2) 90 3) 201 4) 134   
AB
O

      
ER


CH

7. 18. (S.I - 2012)


      
YD

       
1) 2) 3)  4)  1)2) 3) 4)
,H

     
T
AM

8. ãH÷Ok ~òzó# ã‰õ}…


÷ Õ ã„¬‰§ß~¡HÖ O
› Q® ªÖ#O…Õ LO_Í ‹¬OY¼ 19. 2, 5, 9, 19, 37, ? (S.I - 2012)
H

Uk ? 1, 5, 14, 30, 55, 91, ? (Dy. Jailor - 2012) 1) 75 2) 74 3) 72 4) 76


M
AM

1) 130 2) 150 3) 140 4) 160 20. 3, 12, 48, 192,


IG

(Group-IV - 2012)

1) 846 2) 729 3) 768 4) 676


KH

9. ãH÷Ok JOïH =~¡°‹¬…Õ ㄬ‰§ß~¡HÖ O


› L#ß KË@ LO_È=‹²#
21. 1, 3, 6, 10, 15,
R

‹¬OY¼ ? 6, 4, 6, 12, 28, ? (Group-IV - 2012)

(Medical Officers - 2012) 1) 19 2) 21 3) 23 4) 17


1) 75 2) 70 3) 85 4) 80 22. 1, 5, 14, 30, 55, 91, (Dy. Jailors - 2012)

10. 1, 4, 9, 16, 25....... 1) 132 2) 142 3) 140 4) 121


(Group - 1, Priliminary Exam - 2012) 
23. 50,  , 25, 20, (Group IV - 2012)
1) 30 2) 32 3) 34 4) 36 
11. 4, 10, 48, 138, ..... 1)   2) 15 3) 16 4)  
 
(Group - 1, Priliminary Exam - 2012)
24. 3, 5, 7, 9, 11 ...... ã‰õ}÷…Õ 16= „¬^Î=ò
1) 276 2) 286
(Dy. Jailors - 2012)
3) 280 4) ̄áq "ÍgHê=ô 1) 33 2) 31 3) 35 4) 29
REASONING ABILITY (VERBAL) ○ ○ ○ ○ ○ ○ ○ ○ ○
46
○ ○ ○ ○ ○ ○ ○ ○ ○ ○ ○ ○ ○ ○ ○ ○ ○ ○ ○
Study Material
25. 2, 3, 5, 7, 11, 13, (S.I. - 2011) 32. D ãH÷Ok ã‰õ}÷…Õ U ‹¬OY¼#° w`Çwz# ªÖ#=ò…Õ
1) 17 2) 15 3) 19 4) 14 L„¬†³¶yOKÇ=KÇ°ó#° ? 2, 5, 10, 17, ...., 37
26. 1, 4, 27, 256, 3125, (S.I. - 2011) (‡éb‹¹ HêxÀ‹“|°…˜ - 2009)
1) 46656 2) 15625 1) 27 2) 26 3) 24 4) UnHê^ΰ
3) 1726 4) 45656 33. 1, 2, 6, 24, 120, (S.I. - 2007, 2006)

27. 1, 2, 6, 15, 31, 56, (S.I. - 2011)


1) 480 2) 720 3) 600 4) 640
34. 0, 1, 1, 2, 3, 5, 8, 13, 21, 34, (S.I. - 2007)
1) 84 2) 96 3) 92 4) 82
1) 55 2) 45 3) 65 4) 72
28. 1, 4, 15, , 325 (Gr -I - 2010)
35. 1, 3, 7, 15, 31, 63, 127, (S.I. - 2007)
1) 54 2) 62 3) 63 4) 64
1) 245 2) 255 3) 235 4) 345

Y
29. 0, 2, 10, 42, (Gr -I - 2010)
36. 285, 253, 221, 189, (Non Gazzetted- 2005)

M
1) 180 2) 155 3) 170 4) 165 1) 137 2) 157 3) 147 4) 155
30. 1, 6, 2, 12, , 24, 7 (Gr -I - 2010)

E
37. 7, 10, 19, , 127 (Gr -II - 2005)
1) 8 2) 6 3) 18 4) 4 1) 48 2) 56 3) 46 4) 36

D
9
59
31. 3, 9, 21, 45, (Gr -I - 2010) 38. 41, 43, 39, 55, , 67 (Gr -I - 2005)

CA
95
1) 83 2) 95 3) 103 4) 93 1) 37 2) 57 3) 49 4) 67

50
12345678901234567890123456789012123456789012345678901234567890121234567890123456789012345678901212345678901234567890123
123456789012345678901 1234567890123456789012

q=~¡}ì`ÇàH› ["Œ|°° 98
A
12345678901234567890123456789012123456789012345678901234567890121234567890123456789012345678901212345678901234567890123
123456789012345678901 1234567890123456789012
123456789012345678901 1234567890123456789012
12345678901234567890123456789012123456789012345678901234567890121234567890123456789012345678901212345678901234567890123
123456789012345678901 1234567890123456789012
12345678901234567890123456789012123456789012345678901234567890121234567890123456789012345678901212345678901234567890123
123456789012345678901 PREVIOUS BITS -1 : 1234567890123456789012
12345678901234567890123456789012123456789012345678901234567890121234567890123456789012345678901212345678901234567890123
:8
123456789012345678901
123456789012345678901 1234567890123456789012
12345678901234567890123456789012123456789012345678901234567890121234567890123456789012345678901212345678901234567890123
1234567890123456789012
12345678901234567890123456789012123456789012345678901234567890121234567890123456789012345678901212345678901234567890123
E
h

^Î`Çëã‰õ}÷ ãH›=°=°=ò 5. (1) ^Î`Çëã‰õ}÷ ãH›=°=ò


.P

1. (4)
IC
AD

¨ ¨ 
    ¨        
   
AB
O

= ^Î`Çëã‰õ}÷…Õ `Ç~¡°"Œ~ò ‹¬OY¼ '60'      


ER
CH

^Î`Çëã‰õ}÷ ãH›=°=ò = Mìm ªÖ#O…Õ =KÇ°ó ‹¬OY¼ '94'


YD

2. (2)

6. (3) ^Î`Çëã‰õ}÷ ãH›=°=ò


,H

    
T

  
AM

     
w w w w  ¨  ¨  ¨  ¨
H
M

= ^Î`Çëã‰õ}÷…Õ `Ç~¡°"Œ~ò ‹¬OY¼ '17' = ã‰õ}÷…Õ `Ç~¡°"Œ~ò ‹¬OY¼ '201'


AM
IG

3. (2) 7. (3) ã‰õ}÷ ãH›=°=ò


KH

"Œ ãH›=°=ò :  }}m 


 }}m

 }}m

 }}m


R

      

       ‚¬ì~Œ ãH›=°=ò :  }}m  }}m  }}m  }}}m 
   

= Mìm =KÇ°ó‹¬OY¼ m 21 = '?Ñ Q®°~¡°ë ªÖ#O…Õ =KÇ°ó ‹¬OY¼ 



4. (3) ã‰õ}÷…Õ 8. (3) ^Î`Çëã‰õ}÷ ãH›=°=ò
(1×3) – 2 = 1 (10×3) – 2 = 28  
    
(2×3) – 2 = 4 (28×3) – 2 = 
     
(4×3) – 2 = 10 (82×3) – 2 = 244
= Mìm ªÖ#O…Õ =KÇ°ó ‹¬OY¼ '82'     
= ã‰õ}÷…Õ `Ç~¡°"Œ~ò ‹¬OY¼ '140'

REASONING ABILITY (VERBAL) ○ ○ ○ ○ ○ ○ ○ ○ ○


47
○ ○ ○ ○ ○ ○ ○ ○ ○ ○ ○ ○ ○ ○ ○ ○ ○ ○ ○
Study Material
9. (1) 18. (2) a<Œß…Õ
10. (4) 12 22 32 42 52  "Œ ãH›=°=ò :  }}m

 }}}

m  }}}m



= ã‰õ}÷…Õ `Ç~¡°"Œ~ò ‹¬OY¼ '36' ‚¬ì~Œ ãH›=°=ò :  }}}



m  }}}

m  }}}m 


11. (4)

12. (1) ^Î`Çëã‰õ}÷ ãH›=°=ò = ? Q®°~¡°ë ªÖ#O…Õ =KÇ°ó ‹¬OY¼ 
(5×1) + 2 = 7 19. (1) ^Î`Çëã‰õ}÷ ãH›=°=ò
(7×2) + 4 = 18
(18×3) + 6 = 60      
(60×4) + 8 = 248
¨  ¨ ¨  ¨ ¨ 
(248×5) + 10 = 
= ? Q®°~¡°ë ªÖ#O…Õ =KÇ°ó '75 '

Y
= ã‰õ}÷…Õ `Ç~¡°"Œ~ò ‹¬OY¼ '1250' ^Î`Çëã‰õ}÷ ãH›=°=ò

M
20. (3)
13. (3) ^Î`Çëã‰õ}÷ ãH›=°=ò


E
(6×2) + 3 = 15    
(15×3) + 2 = 47 ¨ ¨ ¨ ¨

D
9
59
(47×2) + 3 = 97
= ã‰õ}÷…Õ `Ç~¡°"Œ`Ç ‹¬OY¼ '768'
CA
95
(97×3) + 2 = 293
21. (2) ^Î`Çëã‰õ}÷ ãH›=°=ò

50
(293×2) + 3 = 
= ã‰õ}÷…Õ `Ç~¡°"Œ~ò ‹¬OY¼ '589' 98

A
    
:8
    
14. (1) ^Î`Çëã‰õ}÷ ãH›=°=ò
E

= ã‰õ}÷…Õ `Ç~¡°"Œ~ò ‹¬OY¼ '21'


h
.P


IC

    22. (3) ^Î`Çëã‰õ}÷ ãH›=°=ò


AD

   
AB
O


     
ER
CH

  
     
YD

15. (3) ^Î`Çëã‰õ}÷ ãH›=°=ò


,H

2×2+2 = 6 ; 6×2+2 = 14    

T
AM

14×2+2 = 30 ; 30×2+2 = 62
= ã‰õ}÷…Õ `Ç~¡°"Œ~ò ‹¬OY¼ '140'
H

62×2+2 =  126×2+2 = 254


M

23. (4) ^Î`Çëã‰õ}÷ ãH›=°=ò


AM

= ? Q®°~¡°ë ªÖ#O…Õ =KÇ°ó '126'


IG

 
    
KH

16. (2) ^Î`Çëã‰õ}÷ ãH›=°=ò  


R

    
    
   
    
= ? Q®°~¡°ë ªÖ#O…Õ =KÇ°ó '33 '
17. (3) ^Î`Çëã‰õ}÷ ãH›=°=ò = ã‰õ}÷…Õ `Ç~¡°"Œ~ò ‹¬OY¼  
  
  24. (1) ^Î`Çëã‰õ}÷ ãH›=°=ò
  
3, 5, 7, 9, 1 ....... JOH›ã‰õ}÷
s  s  s  a = 3, d = 5 – 3 = 2
s 
16= „¬^ÎO « a+15d = 3+15(2) = 3+30 = 33
 
= '?Ñ Q®°~¡°ë ªÖ#O…Õ =KÇ°ó ‹¬OY¼ 
 

= Hê=‹²# „¬^ÎO « 33
REASONING ABILITY (VERBAL) ○ ○ ○ ○ ○ ○ ○ ○ ○
48
○ ○ ○ ○ ○ ○ ○ ○ ○ ○ ○ ○ ○ ○ ○ ○ ○ ○ ○
Study Material
25. (1) 2, 3, 5, 7, 11, 13 ㄬ^¥# =~¡°‹¬ ‹¬OY¼° 32. (2) ^Î`Çëã‰õ}÷ ãH›=°=ò
13 `Ç~ŒÞu ㄬ^¥# ‹¬OY¼ '17'      
26. (1) ^Î`Çëã‰õ}÷ ãH›=°=ò
    
      = ã‰õ}÷…Õ `Ç~¡°"Œ~ò ‹¬OY¼ '26'
   
33. (2) ^Î`Çëã‰õ}÷ ãH›=°=ò
 
  
= ã‰õ}÷…Õ `Ç~¡°"Œ~ò ‹¬OY¼ 46656   

27. (3) ^Î`Çëã‰õ}÷ ãH›=°=ò ¨ ¨ ¨ ¨ ¨

       = ã‰õ}÷…Õ `Ç~¡°"Œ~ò ‹¬OY¼ '720'

Y
34. (1) ^Î`Çëã‰õ}÷ ãH›=°=ò

M
     
0 + 1 =1; 1 + 1 = 2 ; 1 + 2 = 3 ; 2 + 3 = 5;
= ã‰õ}÷…Õ `Ç~¡°"Œ~ò ‹¬OY¼ '92' 3 + 5 = 8;

E
28. (4) ^Î`Çëã‰õ}÷ ãH›=°=ò 5 + 8 =13; 8 + 13 = 21; 13 + 21 = 34

D
9
59
 
= ã‰õ}÷…Õ `Ç~¡°"Œ~ò ‹¬OY¼ 34 + 21 = 55
  
CA
95

= ã‰õ}÷…Õ `Ç~¡°"Œ`Ç ‹¬OY¼ '94'

50
¨  ¨  ¨  ¨  35. (2) ^Î`Çëã‰õ}÷ ãH›=°=ò
98
A
= ã‰õ}÷…Õ Mìm ªÖ#O…Õ =KÇ°ó ‹¬OY¼ '64'
:8
      

E

29. (3) ^Î`Çëã‰õ}÷ ãH›=°=ò 


h

     
.P
IC

 
AD

   ¨ ¨ ¨ ¨ ¨ ¨
AB
O

¨  ¨  ¨  ¨  = ã‰õ}÷…Õ `Ç~¡°"Œ~ò ‹¬OY¼ '255'


ER
CH

'?Ñ Q®°~¡°ë ªÖ#O…Õ =KÇ°ó ‹¬OY¼ '170' 36. (2) ^Î`Çëã‰õ}÷ ãH›=°=ò
YD

30. (4) ^Î`Çëã‰õ}÷ ãH›=°=ò     


,H
T

 
AM

   


H

 
   
= ã‰õ}÷…Õ `Ç~¡°"Œ~ò ‹¬OY¼ '157'
M


  
AM
IG

37. (3) ^Î`Çëã‰õ}÷ ãH›=°=ò


= ã‰õ}÷…Õ `DŽ²æ‡é~ò# ‹¬OY¼ '4'
KH


R

31. (4) ^Î`Çëã‰õ}÷ ãH›=°=ò    

       


¨  ¨  ¨  ¨ 
= ã‰õ}÷…Õ '?Ñ Q®°~¡°ë ªÖ#O…Õ =KÇ°ó‹¬OY¼ '+46'
'?Ñ Q®°~¡°ë ªÖ#O…Õ =KÇ°ó ‹¬OY¼ '93' 38. (1)   





 

 

REASONING ABILITY (VERBAL) ○ ○ ○ ○ ○ ○ ○ ○ ○


49
○ ○ ○ ○ ○ ○ ○ ○ ○ ○ ○ ○ ○ ○ ○ ○ ○ ○ ○
Study Material
123456789012345678901234567890121234567890123456789012345678901212
12345678901234567890123456789012123456789012345678901234567890121234567890123456789012345678901212345678901234567890123
1234567890123456789012345678901212345678901234567890123456789012123456789012345678901234567890121234567890123
1234567890123456789012345678901212345678901234567890123456789012123456789012345678901234567890121234
12345678901234567890123456789012123456789012345678901234567890121234567890123456789012345678901212345678901234567890123456789012123
1234567890123456789012345678901212345678901234567890123456789012123456789012345678901234567890
12345678901234567890123456789012123456789012345678901234567890121234567890123456789012345678901212345678901234567890123
1234567890123456789012345678901212345678901234567890123456789012123456789012345678901234567890121234567890123
1234567890123456789012345678901212345678901234567890123456789012123456789012345678901234567890121234
123456789012345678901234567890121234567890123456789012345678901212
12345678901234567890123456789012123456789012345678901234567890121234567890123456789012345678901212345678901234567890123456789012123
1234567890123456789012345678901212345678901234567890123456789012123456789012345678901234567890
12345678901234567890123456789012123456789012345678901234567890121234567890123456789012345678901212345678901234567890123
1234567890123456789012345678901212345678901234567890123456789012123456789012345678901234567890121234567890123
1234567890123456789012345678901212345678901234567890123456789012123456789012345678901234567890121234
12345678901234567890123456789012123456789012345678901234567890121234567890123456789012345678901212345678901234567890123456789012123
1234567890123456789012345678901212345678901234567890123456789012123456789012345678901234567890
123456789012345678901234567890121234567890123456789012345678901212
PREVIOUS BITS ON TYPE - II
12345678901234567890123456789012123456789012345678901234567890121234567890123456789012345678901212345678901234567890123
1234567890123456789012345678901212345678901234567890123456789012123456789012345678901234567890121234567890123
1234567890123456789012345678901212345678901234567890123456789012123456789012345678901234567890121234
12345678901234567890123456789012123456789012345678901234567890121234567890123456789012345678901212345678901234567890123456789012123
1234567890123456789012345678901212345678901234567890123456789012123456789012345678901234567890
123456789012345678901234567890121234567890123456789012345678901212
12345678901234567890123456789012123456789012345678901234567890121234567890123456789012345678901212345678901234567890123
1234567890123456789012345678901212345678901234567890123456789012123456789012345678901234567890121234567890123
1234567890123456789012345678901212345678901234567890123456789012123456789012345678901234567890121234
12345678901234567890123456789012123456789012345678901234567890121234567890123456789012345678901212345678901234567890123456789012123
1234567890123456789012345678901212345678901234567890123456789012123456789012345678901234567890
12345678901234567890123456789012123456789012345678901234567890121234567890123456789012345678901212345678901234567890123
1234567890123456789012345678901212345678901234567890123456789012123456789012345678901234567890121234567890123
1234567890123456789012345678901212345678901234567890123456789012123456789012345678901234567890121234
12345678901234567890123456789012123456789012345678901234567890121234567890123456789012345678901212345678901234567890123456789012123
1234567890123456789012345678901212345678901234567890123456789012123456789012345678901234567890
123456789012345678901234567890121234567890123456789012345678901212
12345678901234567890123456789012123456789012345678901234567890121234567890123456789012345678901212345678901234567890123456789012123
1234567890123456789012345678901212345678901234567890123456789012123456789012345678901234567890
12345678901234567890123456789012123456789012345678901234567890121234567890123456789012345678901212345678901234567890123
1234567890123456789012345678901212345678901234567890123456789012123456789012345678901234567890121234567890123
1234567890123456789012345678901212345678901234567890123456789012123456789012345678901234567890121234

1. ãH÷O^Î W=Þ|_# J#°ãH›=°=ò…Õ ㄬ‰×ß Q®°~¡°ë (?) #° `Çy# 11. YBXC, WDVE, UFTG .... J#° ãH›=°O…Õ …Õ„²Oz#
SzóH›=ò`Ë ƒ’së K͆Çò=ò? JH›Æ~¡ ‹¬=ò^¥†Ç°O. (AP Const. Mains -2016)

7, 26, ?, 342, 1330 (A.P. S.I. Mains 2017) 1) SJRH 2) SIHR


1. 122 2. 123 3. 124 4. 125 3) SHIR 4) SHRI
2. ãH÷O^Î W=Þ|_# J#°ãH›=°=ò…Õ ㄬ‰×ß Q®°~¡°ë (?)#° `Çy# 12. ãH÷O^Î Wzó# JH›Æ~¡ J=°iH›…Õ `Ç~ŒÞ`Ç =KÍó „¬^ÎO YEB,

SzóH›=ò`Ë ƒ’së K͆Çò=ò. (A.P. S.I. Mains 2017) WFD, UHG, SKI ...... (TS. S.I. Mains -2016)

AYBZC, DWEXF, GUHVI, JSKTL, ? 1) QGL 2) QOL 3) TOL 4) QNL


1. NQORP 2. MQNRO 13. JH›Æ~¡ ‹¬OY¼ J#°ãH›=°O ZG13, – , VM17, TP19 …Õ
3. MRNQO 4. OQNRM Mìm…Õ LO_È=‹²# ‹¬i†³Ø°# JH›Æ~¡ ‹¬OY¼

Y
3. ãH÷O^Î W=Þ|_# JH›Æ~¡ - ‹¬OMì¼#°ãH›=°=ò…Õ ㄬ‰×ß Q®°~¡°ë (TS. S.I. Mains -2016)

M
(?)#° `Çy# SzóH›=ò`Ë ƒ’së K͆Çò=ò. 1) XJ15 2) YI17 3) YI15 4) RS15
223BW319CS ? 713GM 1111KK 14. JH›Æ~¡ J=°iH› J#°ãH›=°O AD, FC, HK, MJ, .....…Õ

E
(A.P. S.I. Mains 2017)
`Ç~ŒÞu „¬^ÎO (TS. S.I. Mains -2016)

D
1) NQ 2) OQ 3) MP 4) OR

9
1. 513 EM 2. 420DT 3. 621FU 4. 517EQ

59
15. BE, CG, DI, - -, FM
4. ãH÷Ok =~¡°‹¬…Õ `Ç~¡°"Œu ^¥xx Q®°iëOKÇO_?
CA (VRO - 2014)

95
CB, HGF, MLKJ, ? (AP Panchyat Sec. Screening -2016) 1) EI 2) EJ 3) EK 4) EL

50
1) QPONM 2) SRQPO 16. Z, Y, X, U, T, S, P, O, N, K, ?, ?
98
A
3) RQPON 4) PONML (H›=ü¼xöH+¬<£ S.I - 2012)
:8
5. Wzó# JH›~Æ Œ=¶…Õ, JH›~ Æ Œ ãH›=°O…Õ, XH›KË@ ãH›=°O 1) H, G 2) H, I 3) I, H 4) J, I
E
h

17. ? a ? b ? abaa ? bab ? abba


.P

`DŽ²æOk. JH›Æ~¡ ãH›=¶xß ‹¬iK͆Çò@䛽 Hê=‹²# ^¥xx


IC

Wzó# ‹¬=¶^¥<Œ…Õ Q®°iëOKÇO_. JH›Æ~¡=¶ (H›=ü¼xöH+¬<£ S.I - 2012)


AD

1) aaabb 2) ababb
AB
O

"A A B B C A A B B C A B B C A A B B C A A B B C "

(AP Panchyat Sec. Screening -2016)


3) babab 4) babba
ER
CH

1) A 2) B 3) C 4) AA 18. ãH÷Ok ã‰õ}÷…Õ ㄬ‰§ß~¡ÖH›O Q® ªÖ#O…Õ LO_¨ež# „¬^¥xß


YD

H›#°Hùøx ã‰õ}÷x „¬îië K͆ǰO_.


‹¬¶KÇ#° : (6-8) =~¡ä›½ Q® POQ®Á JH›Æ~¡ J#°ãH›=¶°
,H

UPI, ?, ODP, MBQ, IAW (Dy. Jailor - 2012)


W=Þ|_#q. ㄬu J#°ãH›=°O…Õ#° …Õ„²Oz# JH›~Æ =¡ ò#°
1) SHJ 2) SIJ 3) RHJ 4) THK
T
AM

‹¬¶zOKÍ Mìm#° „¬îiOKÇ_¨xH÷ Mìm ãH›=°O…Õ <Œ°Q®°


19. ãH÷O^Î ~ò=Þ|_# ‹¬=¶^¥<Œ #°O_ ‹¬~
ï ~
á ò# ‹¬=¶^¥<Œxß
H

SzóH›=ò° W=Þ|_<Œ~ò. g\÷…Õ ‹¬i†³Ø°# SzóHêxß


M

Z#°ßHùx ã‰õ}÷x „¬îië K͆ǰO_.


AM

ZO„²H› K͆ǰO_.
IG

(AP Const. Prelims -2016)


a - c ac b c - b a c a - - b (Dy. Jailor - 2012)
6.
KH

– ac – /aacc/c – aa/cccc/ – aaa


1) baba 2) cacb 3) babc 4) abac
1) ccaa 2) aacc 3) caca 4) acac
R

20. DKM, FJP, HIS, JHV, (S.I. - 2012)


7. abc – /adcb/ – cda/ba – c/cdab/c – ad
1) LGY 2) HGY 3) LGX 4) IGY
1) dbdb 2) dcba 3) dabc 4) abab
21. AYBZC, DWEXF, GUHVI, JSKTL, (S.I. - '12)
8. abcb – acab – cbc – ab – c

1) caba 2) baca 3) abac 4) acab 1) NQMOR 2) QMONR


9. MN, LO, KP ....... `Ç~¡°"Œ`Ç =KÍó JH›Æ~Œ [`Ç 3) MQORN 4) MQNRO
(AP Const. Mains -2016)
22. PMT, OOS, NQR, MSQ,

1) JQ 2) AZ 3) YB 4) QJ (Communication S.I. - 2012)

10. BDFH, BEHK, BFJN.... J#° ãH›=°O…Õ …Õ„²Oz# 1) LVQ 2) LWP 3) LUP 4) LVP
JH›Æ~¡ ‹¬=ò^¥†Ç°O. (AP Const. Mains -2016) 23. stt tt tts (S.I. - 2012)

1) BLQG 2) BGQL 3) BGLQ 4) BLGQ 1) stst 2) sstt 3) tsst 4) tsts

REASONING ABILITY (VERBAL) ○ ○ ○ ○ ○ ○ ○ ○ ○


50
○ ○ ○ ○ ○ ○ ○ ○ ○ ○ ○ ○ ○ ○ ○ ○ ○ ○ ○
Study Material
24. ABZY, CDXW, EFVU, (Group IV - 2012) 28. AD 17 39 CF

1) GHTS 2) GHST 3) HGTS 4) FGST BP 258 108 HJ (S.I. - 2011)

25. ACF, HJM, OQT, (Communication S.I. - '12)


GN LM

1) VZA 2) VYZ 3) VXA 4) VXY 1) 203, 181 2) 179, 239


26. +–×++––××+++–––
3) 218, 210 4) 91, 209
29. a
2b2, a3b3a3b3, a4b4a4b4a4b4, (S.I. - 2008)
(S.I. - 2011)

1) –××× 2) ×××× 3) ××++ 4) ×××+ 1) a5b5a5b5a5b5 2) a5b5a5b5a5b5a5b5a5b5


3) a5b5a5b5a5b5a5b5 4) a6b6a6b6a6b6a6b6
27. a13d, e57h, ????, m1315p (S.I. - 2011)
30. E, ........ , P, T, W, Y, Z. (2006-Z‹¹.S.)
1) i911l 2) i811m 3) i711p 4) i912m
1) J 2) L 3) K 4) N
12345678901234567890123456789012123456789012345678901234567890121234567890123456789012345678901212345678901234567890123
1234567890123456789012 12345678901234567890123

q=~¡}ì`ÇàH› ["Œ|°°
12345678901234567890123456789012123456789012345678901234567890121234567890123456789012345678901212345678901234567890123
1234567890123456789012 12345678901234567890123
12345678901234567890123456789012123456789012345678901234567890121234567890123456789012345678901212345678901234567890123
1234567890123456789012 12345678901234567890123

1234567890123456789012
PREVIOUS BITS - 2 :
12345678901234567890123456789012123456789012345678901234567890121234567890123456789012345678901212345678901234567890123
1234567890123456789012 12345678901234567890123
12345678901234567890123456789012123456789012345678901234567890121234567890123456789012345678901212345678901234567890123
1234567890123456789012 12345678901234567890123
12345678901234567890123456789012123456789012345678901234567890121234567890123456789012345678901212345678901234567890123
1234567890123456789012 12345678901234567890123
12345678901234567890123456789012123456789012345678901234567890121234567890123456789012345678901212345678901234567890123
12345678901234567890123

Y
1. (3) 23 = 8 – 1 = 7 10. (3) ^Î`Çëã‰õ}÷ ãH›=°=ò

M
33 = 27 – 1 = 26
# }}m # }}m # }}m #

E
53 = 125 – 1 = 
73 = 343 – 1 = 342 % }}m

& }}m

' }}m

(

D
9
' }}m & }}m ' }}m

59
  
113 = 1331 – 1 = 1330 -
CA ) }}m & }}m ' }}m

95
2. (2)   
2

50
    

":#;$ %8&9' (6)7* +4,5- .2/30 = …Õ„²Oz# JH›Æ~¡ ‹¬=ò^¥†Ç°O « BGLQ


98
A
11. (4) ^Î`Çëã‰õ}÷ ãH›=°=ò : }}m 8 }}m 6 }}m 4
:8
             
.2/30 # }}m % }}m ' }}m
E

  
)
h

9 }}m 7 }}m 5 }}m 3


.P

  
IC

 

$ }}m & }}m ( }}m


AD

3. (4) #8$4&2(.,,   
*
= …Õ„²Oz# JH›Æ~¡ ‹¬=ò^¥†Ç°O « BGLQ
AB
O

   
 
^Î`Çëã‰õ}÷ ãH›=°=ò
ER
CH

12. (2)
4. (3)
YD

$#)('.-,+3210/ : }}m

8 }}m

6 }}m

4 }}m

2
          
,H

= ã‰õ}÷…Õ `Ç~¡°"Œ`Ç =KÇ°ó#k 3210/ & }}m ' }}m ) }}m , }}m 0
   
T
AM

5. (1) ã‰õ}÷ L#ß ãH›=°=ò # }}m



% }}m

( }}m

* }}m

-
H
M

AABBC / AABBC / " BBC / AABBC / AABBC


= ã‰õ}÷ `Ç~ŒÞ`Ç „¬^ÎO « QOL
AM
IG

= Hê=ô# =$`ÇOë KÇ°\÷#“ 'A' ㄬHø› # WOHùH› 'A' ~Œ"Œe. ^Î`Çëã‰õ}÷ ãH›=°=ò
KH

13. (1)
6. (1) ^Î`Çëã‰õ}÷ ãH›=°=ò cacc / aacc / caac / ccaa / aa
}}m 9 }}m 7 }}m 5
R

  
7. (1) ^Î`Çëã‰õ}÷ ãH›=°=ò ;
a b c d /a d c b /b c d a /b a b c /c d a b / ( }}m + }}m . }}m 1
  

c ba d  }}m  }}m  }}m 


  

8. (1) ^Î`Çëã‰õ}÷ ãH›=°=ò


abcbcacabacbcbabac
= Hê=‹²# JH›Æ~¡ ‹¬OY¼ « X515
9. (1) ^Î`Çëã‰õ}÷ ãH›=°=ò 14. (4) ^Î`Çëã‰õ}÷ ãH›=°=ò

. }}m
- }}m 
, }}m 
+ " }}m

' }}m

) }}m

. }}m

0
/ }}m 0 }}m 1 }}m 2
   % }}m $ }}m , }}m + }}m 3
   

= ã‰õ}÷…Õ `Ç~¡°"Œ~ò JH›Æ~¡ [`Ç - JQ = ã‰õ}÷…Õ `Ç~ŒÞu „¬^ÎO 'OR'


REASONING ABILITY (VERBAL) ○ ○ ○ ○ ○ ○ ○ ○ ○
51
○ ○ ○ ○ ○ ○ ○ ○ ○ ○ ○ ○ ○ ○ ○ ○ ○ ○ ○
Study Material
15 (3) ^Î`Çëã‰õ}÷ ãH›=°=ò
. ZY, XW, VU 䛽 =òO^ΰ J^Í ãH›=°O…Õ TS =‹¬°ëOk.

#  $  %  &  ' \ Hê=‹²# „¬^ÎO ( ) 5 4


&  (  *  ,  . 25. (3) ABCDEF HIJKLM OPQRST
ㄬu ‹¬=ü‚¬ìO…Õ#° =òO^ΰ 1 `Ç~¡°"Œ`Ç 2 JH›Æ~Œ°
= Mìm…Õ =KÇ°ó „¬^ÎO 'EK' =^΁|_¨Û~ò. J^Íq^"
Î °³ #
Ø ^~Î à¡ O Q®k Wzó# P„¬Â#°
16 (4) ^Î`Çëã‰õ}÷ ãH›=°=ò
.
#°O_ VXA =¶ã`Ç"Í°.
; : 9 6 5 4
 
    

V W X Y Z A (…è^¥) "$' )+. 02*


1 0 / , + *
     
 
\

Y
17. (4) ^Î`Çëã‰õ}÷ ãH›=°=ò Hê=ô# ‹¬=¶^¥#O VXA.
26. (4) |. ||.. |||... !!|

M
b a a b b a / ba a b b a / b a a bba
18. (1) ^Î`Çëã‰õ}÷ ãH›=°=ò
\ Hê=‹²# ƒìQ®O þþþ{

E
6 }}m 
4 }}}m 0 }}m . }}m
  
* 27. (1) BCDE FGHI

D
9
59
1 }}m 
) }}}m % }}m # }}m
  
"  
CA
95
}}m 
}}}m }}m }}m  

50
* + 1 2 8
JKLM NOPQ

= Hê=‹²# JH›Æ~¡ ‹¬=ü‚¬ìO =SHJ 98


A
 
:8
19. (3) ^Î`Çëã‰õ}÷ ãH›=°=ò „¬ ^ ¥ J=°iH› U~¡ æ _ # q^ ¥ #Ōá # KÇ ¶ „¬ _ È O
E
h

a b c a c/ b c a b a/c a b c b
[iy#k.
.P

=
IC

Hê=‹²# JH›Æ~Œ° = babc \ Hê=‹²# „¬^ÎO « i911l.


AD

28. (1) A D ® 1 + 42 = 1 + 16 = 17
AB
O

     
20. (1) %,.'+1)*4+)7-(: (1) (4)
ER
CH

     
C F ® 3 + 62 = 3 + 36 = 39
YD

(3) (6)

21. (4) ":#;$ %8&9' ®


,H

B P 2 + 162 = 2 + 256 = 258


T
AM

(6)7* +4,5- (2) (16)

®
H

<Œ°Q®° „¬^¥…Õ A, B, C, D, E, F, G, H, I, J, K H J 8 + 102 = 8 + 100 = 108.


M
AM

(8) (10)
L° ̄á# KǶ„²# ãH›=°O…Õ =ô<Œß~ò.
IG

J…ìöQ S, T, U, V, W, X, Y, Z° 䛀_¨ ̄á# KǶ„²# G N ® 7 + 196 = 203


KH

ãH›=°O…Õ =ô<Œß~ò. J^Í ãH›=°O J#°‹¬iOKÇQê (7) (14)


R

\ Hê=‹²# „¬^ÎO « . 2 / 3 0 29. (3) BC 

BC BC 
     

22. (3) 1.5004/23.42-61 


         BC BC BC

BC BC BC BC 


23. (3)
}}m , }}m P }}m T }}m
UTUUTUUTUUTU
30. (3) E    
'tst' i„Ô\˜ J†Í°¼ „¬^ÎO. Hê=ô# q°Ì‹á ž# JH›Æ~Œ°
W }}m Y }}m Z.
UTTU
 
24. (1) ABZY, CDXW, EFVU,
\ Mìm ªÖ#O…Õ =KÇ°ó JH›Æ~¡O « K.
AB, CD, EF `Ç~¡°"Œ`Ç GH =‹¬°ëOk.

REASONING ABILITY (VERBAL) ○ ○ ○ ○ ○ ○ ○ ○ ○


52
○ ○ ○ ○ ○ ○ ○ ○ ○ ○ ○ ○ ○ ○ ○ ○ ○ ○ ○
Study Material
3 a#ß „¬sH›Æ (Oddmanout)

=òMì¼O‰§°...
1. [#~¡…˜<Œ…ÿ_£ûH÷ ‹¬O|OkOz# ㄬ‰×߁°, Q®}÷`Ç „¬iãH÷†Ç°ä›½ ‹¬O|OkOz# ㄬ‰×߁° D qƒìQ®O…Õ ㄬtßOKÇ|_È`Œ~ò.
2. ‹¬i‹¬OY¼°, ƒè‹² ‹¬OY¼°, =~Œ¾°, „¦°¬ <Œ°, ㄬ^¥ # ‹¬OY¼°, ‹¬O†ÇòH›ë ‹¬OY¼°, ‰×Ã^Μ ‹¬OY¼° ............ g\÷H÷ ‹¬O|OkOz#
"ŒxÌ„á „¬îië J=Qꂬì# H›ey=ôO_¨e.
3. XH› ‹¬=ü‚¬ìO…Õx „¬^¥° …è^¥ ‹¬OY¼…Õ q°Q®`Œ"Œ\÷ H›Æ}ìä›½ ‹¬O|O^ÎO …è䛽O_¨ =ô#ß „¬^ÎO …è^¥ ‹¬OY¼#° 'a#ß
„¬^ÎOÑ …è^¥ 'a#ß ‹¬OY¼Ñ JO^ΰ~¡°.

Y
4. D qƒìQ®O…Õ <Œ°Q®° „¬^¥° …è^¥ ‹¬OY¼° W=Þ|_È`Œ~ò. "Œ\÷…Õ =ü_È° XöH ~¡H›"³°Ø# H›Æ}=ò#° H›ey=ôO\ì~ò.

M
<Œ¾=k q°Q®`Œ "Œ\÷`Ë ‡éeó`Í a#ß"³°Ø# H›Æ}O H›ey =ôO@°Ok. D q^Î"³°Ø# a#ß"³°Ø# H›Æ}O H›ey# „¬^ÎO …è^¥ ‹¬OY¼#°
=°#O Q®°iëOKŒe.

E
5. ~Œ¢‘“° Ð ~Œ[^¥#°°, ^͉§° Ð ~Œ[^¥#°°, [O`Ç°=ô°, q°ã‰×=¶°, …Õ‚¬ð° ......... =O\÷ [#~¡…˜ <Œ…ÿ_Hû£ ÷ ‹¬O|OkOz#

D
9
59
"Œ\÷̄á 䛀_¨ „¬îië J=Qꂬì# =ôO_¨e.

CA
95
MODEL - 1 1) 1) NOP 2) RTU 3) JKL 4) EFG

50
…Õx =ü_È° JH›Æ~Œ° =~¡°‹¬
UH›„¬^Î a#ß „¬sH›Æ
ª^Î# : (2) NOP, JKL, EFG

98
A
JH›~Æ Œ°. Hêx RTU P H›}
Æ ìxß H›ey†ÇòO_ȅ^
è °Î .
G \ ‹¬=¶^¥#O (2).
:8
D qƒìQ®O…Õ <Œ°Q®° "Íö~Þ~¡° „¬^¥° Wªë~¡°. "Œ\÷…Õ
E
h

=ü_È° „¬^¥° XöH H›Æ}O H›ey=ôO\ì~ò. J#Qê P 2) 1) RUX 2) CFI 3) BDG 4) FIL
.P
IC

=ü_È° „¬^¥ä›½ ª~¡¶„¬¼`Ç =ôO@°Ok. q°ye# „¬^ÎO ª^Î# : (3) 3 69 $ '*
AD

a#ßOQê =ôO@°Ok. J#Qê q°ye# „¬^¥`Ë ª~¡¶„¬¼`Ç    


AB
O

…è䛽O_¨ LO@°Ok. P a#ß „¬^¥xß =°#O Q®°iëOKŒe. # %(


 ' *-
ER
CH

1) : 1) H›„¬æ 2) `Œƒè° 3) „Ô`Ç 4) K̈́¬    


: (4) H›„¬æ, `Œƒè°, „Ô`ǁ° h\÷…Õ#¶, <́g°^Î rqOKÇ
YD

ª^Î#
Q® Lƒ’†Ç°KÇ~¡ r=ô°. Hêx K̈́¬ h\÷…Õ =¶ã`Ç"Í° MODEL - 3
,H

rqOKÇQ®^ΰ. Hê=ô# Wk a#ß"³°Ø#k. ‹¬OY¼ a#ß „¬sH›Æ


T
AM

: 1) `ÇeÁ 2) `ÇOã_ 3) ªé^Îi


2)
G
H

W=Þ|_# <Œ°Q®° ‹¬OY¼…Õ =ü_È° ‹¬OY¼° XöH


M

4) ªé^Î~¡°_È° 5) À‹ß‚²ì`Ç°_È°
AM

H›}
Æ ìxß H›ey LO\ì~ò. <Œ¾= ‹¬OY¼ P ª~¡¶„¬¼`Ç#°
IG

ª^Î# : (5) À‹ß‚² ì `Ç ° _È ° `Ç „ ¬ æ q°ye# "Œ~¡ O ^Î ~ ¡ ¶ ~¡ H › ë


H›ey =ôO_È^ΰ. D a#ß ‹¬OY¼#° =°#O Q®°iëOKŒe.
KH

‹¬O|Onä›½°. Hê=ô# À‹ß‚²ì`Ç°_È° a#ß"³°Ø#k.


1) 1) 34 – 43 2) 62 – 71
R

3) : 1) ‹Ô‹¬O 2) ‡^Î~¡‹¬O 3) ~Œy


3) 55 – 62 4) 83 – 92 (Gr. - '04)
4) W#°=ò 5) \÷<£
ª^Î# : (3) 34 + 9 = 43; 62 + 9 = 71
ª^Î# : (2) ‡^Î~‹ ¡ O¬ `Ç„æ¬ q°ye# =hß „¦°¬ #…Õ‚¬ð° ‡^Î~‹¡ O¬
55 + 7 = 62; 83 + 9 = 92
=¶ã`ÇO ã^Î=…Õ‚¬ìO. Hê=ô# Wk a#ß"³°# Ø k. 2) 1) 602 2) 431 3) 530 4) 813
MODEL - 2 ª^Î# : (4) 602 ® 6 + 0 + 2 = 8

®
JH›Æ~Œ a#ß „¬sH›Æ 431

®
4 + 3 + 1 = 8

+
530 5 + 3 + 0 = 8

D qƒìQ®O…Õx ㄬ‰×߁x߆Çò POQ®Á JH›Æ~¡=¶ä›½ 813 ® 8 + 1 + 3 = 12

‹¬O|OkOz =ôO_È°#°. W=Þ|_# ㄬ‰×߅Õx <Œ°Q®° 3) 1) 18 2) 12 3) 30 4) 20


2 2 2 + 4 = 20
P„¬Â<£…Õ XH›\ ÷ `DŽ¬æ q°ye#=hß XöH H›Æ}ìxß ª^Î# : (1) 3 + 3 = 12; 5 + 5 = 30; 4

H›ey†ÇòO_È°#°. a#ß"³°Ø# ^¥xx =°#O Q®°iëOKŒe. Hêx 18 J…ìO\÷ ª~¡¶„¬¼`Ç#° H›e¾†ÇòO_ȅè^ΰ.

REASONING ABILITY (VERBAL) ○ ○ ○ ○ ○ ○ ○ ○ ○


53
○ ○ ○ ○ ○ ○ ○ ○ ○ ○ ○ ○ ○ ○ ○ ○ ○ ○ ○
Study Material
4) 1) 84 2) 91 3) 119 4) 104 (4) 2) 1) JuM – jUm 2) iLo – ILo
ª^Î# : (4) 84 = 7 × 12 3) pSa – psA 4) ZeX – zEx
91 = 7 × 13 ª^Î# : (2) ㄬu ãQ®¶„¬ô…Õ "³ò^Î\ ÷ ‹¬=ü‚¬ìO…Õx ̄^Îí `Ç~¡Q®u
119 = 7 × 17 JH›Æ~Œ#° z#ß `Ç~¡Q®u…ÕxH÷, z#ß `Ç~¡Q®u
JH›Æ~Œ#° ̄^Îí `Ç~¡Q®u…ÕxH÷ `Œ~¡°=¶~¡° Kͪ~¡°.
MODEL - 4 Hêx (2) …Õx JH›Æ~¡ ‹¬=ü‚¬ð° P H›Æ}ìxß
„¬^Î ‹¬=ü‚¬ð a#ß „¬sH›Æ ‡\÷OKDžè^ΰ. L #° L Qê<Í ~Œª~¡°. Hê=ô# Wk
a#ß"³°Ø#k.
G „¬^Î ‹¬=ü‚¬ð a#ß „¬sH›Æ…Õ <Œ°Q®° [O@„¬^¥° 3) 1) RUBY – BURY 2) SPOT – TOPS
=ôO\ì~ò. "Œ\÷…Õ ㄬu XH›ø [O@ „¬^¥ =°^Î¼ U^Ë 3) MEAL – LAME 4) BARK – KERB
XH› ‹¬O|O^ÎO LO@°Ok. Hêx XH› [O@„¬^ÎO…Õx ª^Î# : (4) 1, 2, 3 …Õx "³ò^Î\ ÷ JH›Æ~¡ ‹¬=ü‚¬ìO…Õx
ï~O_È° „¬^¥ =°^Î¼ Z@°=O\÷ ‹¬O|O^ÎO …è䛽O_¨ JH›Æ~Œ`Ë J~¡Ö=O`Ç"³°Ø# „¬^ÎO ï~O_È= JH›Æ~¡

Y
=ôO\ì~ò. J@°=O\÷ a#ß [O@ „¬^¥xß =°#O ‹¬ = ü‚¬ ì OQê U~¡ æ _ # k. Hêx 4 …Õx
Q®°iëOKŒe.

M
JH›Æ~¡‹¬=ü‚¬ð° P H›Æ}ìxß ‡\÷OKDžè^ΰ.
1) 1) …ì†Ç°~Ÿ Ð H›Á~òO\˜ 2) "Œ¼‡i Ð Mì`Œ^¥~¡°_È° 4) 1) ABC – CBA 2) JKL – KLJ

E
3) "³á^ΰ¼_È° Р̋“`Ǫéø„¹ 3) XYZ – ZYX 4) MNO – ONM
4) |‹¹¢_³á=~Ÿ Рㄬ†Ç¶}÷䛽° ㄬu ‹¬=ü‚¬ìO…Õ (1, 3, 4…Õ) "³ò^Î\ ÷ JH›Æ~¡

D
9
ª^Î# : (2)

59
ª^Î# : (3) 3 …Õ `DŽ¬æ q°ye# [O@„¬^¥xßO\÷…Õ ï~O_È° ‹¬=ü‚¬ìO…Õx JH›Æ~Œ#° =¼uö~H›k‰×…Õ ~Œ†Ç°Qê
CA
95
„¬^¥¶ =¼ä›½ë#° Q®°iOz `³°æKÇ°#ßq. Hêx 3 ï~O_È= JH›Æ~¡ ‹¬=ü‚¬ìO U~¡æ_#k. Hêx JKL –
…Õ ̋“`Ǫéø„¹ (=‹¬°ë=ô) Q®°iOz `³°æ`Ç°#ßk. KLJ P H›Æ}ìxß H›e¾†ÇòO_ȅè^ΰ.

50
1) H›`³ë~¡ Ð =GO 2) KŒä›½ Ð ãƒÿ_£
98
A
2)
3) ~¡O„¬O Ð K³H›ø 4) ‹¬°uë Ð "Ͱ䛽 MODEL - 6
:8
‹¬OMì¼ ‹¬=ü‚¬ð a#ß „¬sH›Æ
E

ª^Î# : (4) 1, 2, 3 …Õ "³ò^Î\ ÷ „¬^ÎO…Õx =‹¬°ë=ô ï~O_È=


h
.P

„¬^ÎO…Õx ^¥xx H›uëiOKÇ_¨xH÷ L„¬†³¶yªë~¡°.


G
IC

W=Þ|_# ‹¬OY¼ ‹¬=ü‚¬ð…Õ =ü_È° ‹¬=ü‚¬ðä›½


AD

Hêx ‹¬°uë`Ë "Ͱ䛽#° H›uëiOKÇ~¡°.


XöH ~¡H›"³°Ø# H›Æ}O =ôO@°Ok. <Œ¾= ‹¬=ü‚¬ìO P
3) 1) „¬Q®° Ð ~Œãu 2) ‹¬æ+¬“O Ð J‹¬æ+¬“O
AB
O

H›}Æ ìxß H›e=


¾ ôO_È^°Î . D a#ß"³°#
Ø ‹¬=ü‚¬ìO#° =°#O
3) KÍ~¡° Ð =KÇ°ó 4) `³e"³á#"Œ_È° Ð "³ãi"Œ_È°
Q®°iëOKŒe.
ER
CH

ª^Î# : (3) 1, 2, 4 ° =¼uö~H› J~ŒÖ° H›ey# [O@„¬^¥°.


1) 1) 123, 36 2) 234, 99
YD

Hêx 3 …Õx „¬^¥~¡° XöH J~ŒÖxß WKÇ°ó [O@ 3) 143, 90 4) 131, 29


„¬^¥°. ®ÿ 3
,H

ª^Î# : (3) 123 1 + 2


3 + 3
3 = 1 + 8 + 27 = 36

® 3
T

1) PO„²†Ç°~Ÿ : q^ΰ¼`Ç°ë 2) "Œ\˜ : ª=°~¡Ö¼O 3 3


AM

4) 234 2 + 3 + 4 = 8 + 27 + 64 = 99

3) "˅˜“ : ‡Ú>ÿx†ǰ…˜ 4) #¶¼@<£ : P"͉×O ® 3 3 3


H

143 1 + 4 + 3 = 1 + 64 + 27 = 92

® 3
M

3 3
ª^Î# : (4) #¶¼@<£ J#°#k |…ìxH÷ ㄬ=¶}O 29
AM

131 1 + 3 + 1 = 1 + 27 + 1 =
IG

q°ye#=hß ‹¬i†³Ø°#"Í. Hê=ô# a#ß [O@„¬^ÎO (4). 2) 1) 15, 35 2) 16, 32 3) 40, 80 4) 14, 28
KH

ª^Î# : (1) …Õx ‹¬OY¼ ‹¬=ü‚¬ð…Õ ~ï O_È= ‹¬OY¼


2, 3, 4

MODEL - 5 "³ò^Î\ ÷ ‹¬OY¼ †³òH›ø ï~\÷“O„¬ô. Hêx 15, 35


R

JH›Æ~¡ ‹¬=ü‚¬ð a#ß „¬sH›Æ ‹¬OY¼° P ^Î~Œàxß H›e¾†ÇòO_ȅè^ΰ.


2) 16 × 2 = 32 3) 40 × 2 = 80
G Wzó# <Œ°Q®° JH›Æ~¡ ‹¬=ü‚¬ð…Õ a#ß H›Æ}O 4) 14 × 2 = 28
H›e¾†Çò#ß ^¥xx Q®°iëOKŒe. Hêx 15 × 2 ¹ 35.
1) 1) DXD – XDX 2) KUK – UKU 3) 1) 20, 441 2) 10, 141
3) FHF – EHE 4) RSR – SRS 3) 30, 941 4) 40, 1621
ª^Î# : (3) ㄬu ãQ®¶„¬ô…Õ "³ò^Î\ ÷ ‹¬=ü‚¬ìO…Õ XH›ªi =zó# ª^Î# : (4) 1) 202 + 41 = 400 + 41 = 441
JH›Æ~¡O ï~O_È= ‹¬=ü‚¬ìO…Õ ï~O_È° ª~¡°Á =°i†Çò 2) 102 + 41 = 100 + 41 = 141
"³ò^Î\ ÷ ‹¬=ü‚¬ìO…Õ ï~O_È° ª~¡°Á =zó# JH›Æ~¡O 3) 302 + 41 = 900 + 41 = 941
ï~O_È= ‹¬=ü‚¬ìO…Õ XH›ªi =KŒó~ò. Hêx (3) 4) 402 + 41 = 1600 + 41 = 1641
…Õx JH›~Æ ¡ ‹¬=ü‚¬ð° WO^ΰ䛽 a#ßOQê =ô<Œß~ò. Hêx 4 a#ßOQê =ô#ßk.

REASONING ABILITY (VERBAL) ○ ○ ○ ○ ○ ○ ○ ○ ○


54
○ ○ ○ ○ ○ ○ ○ ○ ○ ○ ○ ○ ○ ○ ○ ○ ○ ○ ○
Study Material
123456789012345678901234567890121234567890
123456789012345678901234567890121234567890
123456789012345678901234567890121234567890123456789012345678901212
12345678901234567890123456789012123456789012345678901234567890121234567890123456789012345678901212345678901234567890123
1234567890123456789012345678901212345678901234567890123456789012123456789012345678901234567890121234567890123
1234567890123456789012345678901212345678901234567890123456789012123456789012345678901234567890121234
12345678901234567890123456789012123456789012345678901234567890121234567890123456789012345678901212345678901234567890123456789012123
12345678901234567890123456789012123456789012345678901234567890121234567890123456789012345678901212345678901234567890123
1234567890123456789012345678901212345678901234567890123456789012123456789012345678901234567890121234567890123
1234567890123456789012345678901212345678901234567890123456789012123456789012345678901234567890121234
123456789012345678901234567890121234567890
123456789012345678901234567890121234567890123456789012345678901212
PRACTICE TEST on UH›„¬^Î a#ß „¬sH›Æ
12345678901234567890123456789012123456789012345678901234567890121234567890123456789012345678901212345678901234567890123456789012123
12345678901234567890123456789012123456789012345678901234567890121234567890123456789012345678901212345678901234567890123
1234567890123456789012345678901212345678901234567890123456789012123456789012345678901234567890121234567890123
1234567890123456789012345678901212345678901234567890123456789012123456789012345678901234567890121234
12345678901234567890123456789012123456789012345678901234567890121234567890123456789012345678901212345678901234567890123456789012123
123456789012345678901234567890121234567890123456789012345678901212
12345678901234567890123456789012123456789012345678901234567890121234567890123456789012345678901212345678901234567890123
1234567890123456789012345678901212345678901234567890123456789012123456789012345678901234567890121234567890123
1234567890123456789012345678901212345678901234567890123456789012123456789012345678901234567890121234
123456789012345678901234567890121234567890
12345678901234567890123456789012123456789012345678901234567890121234567890123456789012345678901212345678901234567890123456789012123
123456789012345678901234567890121234567890123456789012345678901212
12345678901234567890123456789012123456789012345678901234567890121234567890123456789012345678901212345678901234567890123
1234567890123456789012345678901212345678901234567890123456789012123456789012345678901234567890121234567890123
1234567890123456789012345678901212345678901234567890123456789012123456789012345678901234567890121234
12345678901234567890123456789012123456789012345678901234567890121234567890123456789012345678901212345678901234567890123456789012123
12345678901234567890123456789012123456789012345678901234567890121234567890123456789012345678901212345678901234567890123
1234567890123456789012345678901212345678901234567890123456789012123456789012345678901234567890121234567890123
1234567890123456789012345678901212345678901234567890123456789012123456789012345678901234567890121234
12345678901234567890123456789012123456789012345678901234567890121234567890123456789012345678901212345678901234567890123456789012123
123456789012345678901234567890121234567890123456789012345678901212
12345678901234567890123456789012123456789012345678901234567890121234567890123456789012345678901212345678901234567890123456789012123
12345678901234567890123456789012123456789012345678901234567890121234567890123456789012345678901212345678901234567890123
1234567890123456789012345678901212345678901234567890123456789012123456789012345678901234567890121234567890123
1234567890123456789012345678901212345678901234567890123456789012123456789012345678901234567890121234
12345678901234567890123456789012123456789012345678901234567890121234567890123456789012345678901212345678901234567890123456789012123

1. 1) KÇ`Ç°~¡ã‹¬O 2) n~¡É KÇ`Ç°~¡ã‹¬O 12. 1) ‡¦Á\˜ 2) J‡~Ÿ“"³°O\˜


3) =$`ÇëO 4) ãuHË}O (Gr.I. - 1991) 3) Qê¼ö~*˜ 4) HêÞ~¡“~Ÿ (Gr.I. - 1995)

2. 1) ƒì^Î 2) q‹¬°Q®° 3) qKŒ~¡=ò 4) ^ΰ—Y=ò 13. 1) JÁ=ò 2) L~¡ÁQ®_ÈÛ

3) @"³¶@ 4) Hê¼e„¦¬Á=~Ÿ
3. 1) PH›e 2) ^΄²æH› 3) J‹¬@ 4) ã‰×=°

1) J_Èq 2) #k 3) „¬~¡Þ`ÇO 4) ãQ®‚¬ð°


1) =°#°+¬µ° 2) ̄^Î큰
4. 14.

1) <³„¬î“ ¼<£ 2) ‰×Ãã䛽_È°


3) =ò‹¬e"Œˆ×ä 4) „²Á°
15.

3) KÇOã^ΰ_È° 4) ‰×x (Gazetted - 2006)


5. 1) ̋H›#° 2) Q®O@
16. 1) `Ƕ~¡°æ 2) PöQ߆ǰO 3) L`Çë~¡O 4) ^ÎH÷Æ}O
3) Hê=ò 4) ‹¬O=`Ǟ~¡=ò
17. 1) =¶ió 2) "Í° 3) „¦²ã|=i 4) [#=i
1) P=ô 2) "Í°H› 3) „²eÁ 4) QùŽÿ]

Y
6.
18. 1) Q®°O_³ 2) T„²iu`ǰ끰
7. 1) <́ 2) ㄬ^͉×=ò

M
3) =üã`DŽ²O_¨° 4) H›ˆ×ÃÁ
3) KË@° 4) ‹¬ÖO
19. 1) „¬^Î=ò 2) "ŒH›¼=ò 3) JH›Æ~¡=ò 4) À„l

E
8. 1) _¨H›“~¡° 2) …ì†Ç°~¡°
20. 1) QêOk 2) <³ã‚¬ú 3) „¬>腘 4) l<Œß
3) L‡^¥¼†Çò_È° 4) #@°_È°

D
9
21. 1) ‡~”¡‰§ 2) ^Í"Œ†Ç°=ò

59
9. 1) `ÇOã_ 2) q°ã`Ç°_È°
3) "³°Ø^¥#=ò 4) W°Á

CA
95
3) `ÇeÁ 4) "Í°#`Çë
22. 1) `Œƒè° 2) H›„¬æ 3) "³ò‹¬e 4) K̈́¬

50
1) ‚²ì`ÇO 2) ‹¬‚¬ð (Gr.I. - 1995)
1) K³q 2) =ò䛽ø 3) <Œ°H› 4) H›#°ß
10.
23.

98
A
3) ‹¬¶KÇ# 4) P^͉×O
24. 1) #k 2) U~¡° 3) K³Ž°=ô 4) Hꁰ=
:8
11. 1) ãH©.‰×.400 2) ãH©.‰×.700 25. 1) ƒ’~¡`Ç<Œ@¼O 2) ‹¬°~¡a
E
h

3) ãH©.‰×.1100 4) ãH©.‰×.1900 3) H›^ŠÎH›o 4) X_‹²ž


.P
IC

12345678901234567890123456789012
12345678901234567890123456789012123456789012345678901234567890121234567890123456789012345678901212345678901234567890123
12345678901234567890123456789012
12345678901234567890123456789012123456789012345678901234567890121234567890123456789012345678901212345678901234567890123
12345678901234567890123456789012 12345678901234567890123456789012
12345678901234567890123456789012
q=~¡}ì`ÇàH› ["Œ|°°
12345678901234567890123456789012123456789012345678901234567890121234567890123456789012345678901212345678901234567890123
12345678901234567890123456789012
AD

12345678901234567890123456789012
12345678901234567890123456789012123456789012345678901234567890121234567890123456789012345678901212345678901234567890123
12345678901234567890123456789012 12345678901234567890123456789012
12345678901234567890123456789012 12345678901234567890123456789012
12345678901234567890123456789012123456789012345678901234567890121234567890123456789012345678901212345678901234567890123
12345678901234567890123456789012
12345678901234567890123456789012123456789012345678901234567890121234567890123456789012345678901212345678901234567890123
12345678901234567890123456789012 12345678901234567890123456789012
12345678901234567890123456789012123456789012345678901234567890121234567890123456789012345678901212345678901234567890123
AB
O

=$`ÇëO `DŽ¬æ q°ye#q ƒ’°*ì ‹¬OY¼ ^¥Þ~Œ …ÿH÷øOKÇ Hê¼e„¦¬Á=~Ÿ '„¬î=ôÑ *ìuH÷ K³Ok#q.
ER
CH

1. (3) 13. (4)

Q®°¾`Œ=ò. ãQ®‚¬ð° `DŽ¬æ q°ye#q ƒ’¶q° g°^Î LO_È°#°.


YD

14. (4)

2. (2) ƒì^Î, qKŒ~¡=ò, ^ΰ—Y=ò Wq XH›^¥x"³O@ XH›\ ÷ 15. (3) KÇOã^ΰ_È° `DŽ¬æ q°ye#q ‹¬¶~¡¼ä›½@°Oƒì…Õx
,H

LO\ì~ò. Hê=ô# 'q‹¬°Q®°Ñ a#ß"³°Ø#k. ãQ®‚¬ð°. KÇOã^ΰ_È° =¶ã`Ç=ò L„¬ãQ®‚¬ìO.


T
AM

3. (4) PH›eH÷ ƒÕ[#O, ^΄²æH›ä›½ h~¡°, J‹¬@䛽 xã^Î Wq 16. (2) PöQ߆ǰO J<Ík =ü. q°ye#q k䛽ø°.
H
M

‰×s~ŒxH÷ J`ǼO`Ç P=‰×¼H›"°³ # Ø q. ã‰×=°ä›½ „¦e


¬ `ÇO. Wk 17. (3) „¦²ã|=i `DŽ¬æ q°ye# <³ä›½ 31 ~ËA° LO_È°#°.
AM
IG

=°#‹¬°ä›½ J=‹¬~¡"³°Ø#k. Hê=ô# 'ã‰×=°Ñ a#ß"³°Ø#k. 18. (1) Q®°O_³ XH›ø>è. q°ye#q [`ǁ°Qê LO\ì~ò.
KH

4. (1) ̄^Î큰, =ò‹¬e"Œˆ×ä, „²Á° JO^Î~¡¶ =°#°+¬µ¼…è. 19. (4) À„l…Õ "ŒH›¼=ò, "ŒH›¼=ò „¬^¥`Ë, „¬^¥° JH›~Æ Œ`Ë
̋H›#°, Q®O@, ‹¬O=`Ǟ~¡=ò J<Íq Hê…ìxH÷ ㄬ=¶}쁰. U~¡æ_È°`Œ~ò.
R

5. (3)

6. (3) „²eÁ XH›ø>è =¶Oª‚¬ði. q°ye#q ‰§M사ð~¡°°. 20. (4) l<Œß ‡H÷ªÖ<£ "³ò^Î\ ÷ ㄬ^¥x. q°ye# "Œ~¡° ƒì~¡`Ç
7. (1) <́ XH›ø>è 'ƒ’¶q°Ñ Jx q°ye#q ZH›ø_³á<Œ Hê=KÇ°ó. ^͉§xH÷ K³Ok#"Œ~¡°.
8. (4) #@°_È° U ‡ã`Ç<³á<Œ ‡é+²ªë_È°. 21. (3) ‡~”¡‰§, ^Í"Œ†Ç°=ò =°i†Çò W°Á H›@“_¨°.
9. (2) q°ã`Ç°_È° ~¡H›ë ‹¬O|Onä›½_È° HêH›‡é=KÇ°ó. 22. (4) KÍ „ ¬ , h\ ÷ … Õ =¶ã`Ç " Í ° rq‹¬ ° ë O k. q°ye#q
10. (4) P^͉×=ò `DŽ¬æ q°ye#q `Ç=° ªÖ~ò "Œi #°O_ Lƒ’†Ç°KÇ~Œ°.
‡ÚO^ΰ`Œ~¡°. P^͉×O J<Ík ̄á JkHêi #°O_ ‡ÚO^Í 23. (3) Jx߆Çò WOãk†Ç¶…è. <Œ°H› XH›ø>è LO@°Ok.
‹¬=¶KŒ~¡O. q°ye#q [`ǁ°.
11. (1) ãH©.‰×. 400 b„¬ô ‹¬O=`Ǟ~¡=ò. 24. (3) K³Ž°=ô…Õ h~¡° xH›_ÈQê LO@°Ok. q°ye# "Œ\÷…Õ
12. (3) Qê¼ö~*˜ `DŽ¬æ q°ye#q qq^Î ~¡Hê WO_ÈÁ x~Œà}ìä›½ h~¡° ㄬ=‚²ì‹¬°ëOk.
Q® À„~¡°Á. 25. (2) ‹¬°~¡a J<Ík <Œ@H›O. q°ye#q #$`Œ¼°.

REASONING ABILITY (VERBAL) ○ ○ ○ ○ ○ ○ ○ ○ ○


55
○ ○ ○ ○ ○ ○ ○ ○ ○ ○ ○ ○ ○ ○ ○ ○ ○ ○ ○
Study Material
123456789012345678901234567890121234567890
123456789012345678901234567890121234567890123456789012345678901212
12345678901234567890123456789012123456789012345678901234567890121234567890123456789012345678901212345678901234567890123456789012123
12345678901234567890123456789012123456789012345678901234567890121234567890123456789012345678901212345678901234567890123
1234567890123456789012345678901212345678901234567890123456789012123456789012345678901234567890121234567890123
1234567890123456789012345678901212345678901234567890123456789012123456789012345678901234567890121234
123456789012345678901234567890121234567890
123456789012345678901234567890121234567890123456789012345678901212
12345678901234567890123456789012123456789012345678901234567890121234567890123456789012345678901212345678901234567890123456789012123
12345678901234567890123456789012123456789012345678901234567890121234567890123456789012345678901212345678901234567890123
1234567890123456789012345678901212345678901234567890123456789012123456789012345678901234567890121234567890123
1234567890123456789012345678901212345678901234567890123456789012123456789012345678901234567890121234
PRACTICE TEST on JH›Æ~Œ a#ß „¬sH›Æ
12345678901234567890123456789012123456789012345678901234567890121234567890123456789012345678901212345678901234567890123456789012123
123456789012345678901234567890121234567890123456789012345678901212
12345678901234567890123456789012123456789012345678901234567890121234567890123456789012345678901212345678901234567890123
1234567890123456789012345678901212345678901234567890123456789012123456789012345678901234567890121234567890123
1234567890123456789012345678901212345678901234567890123456789012123456789012345678901234567890121234
123456789012345678901234567890121234567890
12345678901234567890123456789012123456789012345678901234567890121234567890123456789012345678901212345678901234567890123456789012123
123456789012345678901234567890121234567890123456789012345678901212
12345678901234567890123456789012123456789012345678901234567890121234567890123456789012345678901212345678901234567890123
1234567890123456789012345678901212345678901234567890123456789012123456789012345678901234567890121234567890123
1234567890123456789012345678901212345678901234567890123456789012123456789012345678901234567890121234
12345678901234567890123456789012123456789012345678901234567890121234567890123456789012345678901212345678901234567890123456789012123
12345678901234567890123456789012123456789012345678901234567890121234567890123456789012345678901212345678901234567890123
1234567890123456789012345678901212345678901234567890123456789012123456789012345678901234567890121234567890123
1234567890123456789012345678901212345678901234567890123456789012123456789012345678901234567890121234
123456789012345678901234567890121234567890
12345678901234567890123456789012123456789012345678901234567890121234567890123456789012345678901212345678901234567890123456789012123
123456789012345678901234567890121234567890123456789012345678901212
12345678901234567890123456789012123456789012345678901234567890121234567890123456789012345678901212345678901234567890123456789012123
12345678901234567890123456789012123456789012345678901234567890121234567890123456789012345678901212345678901234567890123
1234567890123456789012345678901212345678901234567890123456789012123456789012345678901234567890121234567890123
1234567890123456789012345678901212345678901234567890123456789012123456789012345678901234567890121234
123456789012345678901234567890121234567890123456789012345678901212
12345678901234567890123456789012123456789012345678901234567890121234567890123456789012345678901212345678901234567890123456789012123
12345678901234567890123456789012123456789012345678901234567890121234567890123456789012345678901212345678901234567890123
1234567890123456789012345678901212345678901234567890123456789012123456789012345678901234567890121234567890123
1234567890123456789012345678901212345678901234567890123456789012123456789012345678901234567890121234

ãH÷Ok "Œx…Õ a#ß"³°Ø# ^¥xx H›#°Qù#°=ò. 11. 1) FJQP 2) CGON 3) HLTS 4 LPXW

1. 1) WM 2) CC 3) HH 4) EE (Gr.I - 2002)

2. 1) PRT 2) QOM 3) CEG 4) UWY 12. 1) DEB 2) MNL 3) IJG 4) XYV

(S.I - 2007) 13. 1) AAb 2) Lmn 3) Rst 4) Xyz

3. 1) S 2) E 3) H 4) L 14. 1) ZYX 2) RQP 3) KJI 4) CDE

4. 1) BAT 2) CAT 3) RAT 4) SET 15. 1) B 2) C 3) E 4) K

5. 1) B3E 2) C5G 3) E9K 4) G11M 16. 1) DGTM 2) QTWZ 3) FHKN 4) XADG

Y
6. 1) ABCD 2) BCAD 3) CDAB 4) DABC (Gr.I - 2003)

M
7. 1) OT 2) LP 3) NR 4) MQ 17. 1) MPS 2) RTV 3) GJM 4) ADG

8. 1) ACF 2) TVY 3) MOR 4) JLN 18. 1) ABD 2) JKM 3) UVX 4) STU

E
9. 1) JMKL 2) GIHJ 3) EGFH 4) MONP 19. 1) C 2) D 3) E 4) F

D
9
10. 20.

59
1) LMON 2) ABDC 3) PQRS 4) STVU 1) H 2) I 3) M 4) X

CA
95
12345678901234567890123456789012
12345678901234567890123456789012123456789012345678901234567890121234567890123456789012345678901212345678901234567890123
12345678901234567890123456789012

q=~¡}ì`ÇàH› ["Œ|°°
12345678901234567890123456789012123456789012345678901234567890121234567890123456789012345678901212345678901234567890123
12345678901234567890123456789012
12345678901234567890123456789012 12345678901234567890123456789012
12345678901234567890123456789012123456789012345678901234567890121234567890123456789012345678901212345678901234567890123
12345678901234567890123456789012
12345678901234567890123456789012
12345678901234567890123456789012123456789012345678901234567890121234567890123456789012345678901212345678901234567890123
12345678901234567890123456789012 12345678901234567890123456789012
12345678901234567890123456789012
12345678901234567890123456789012123456789012345678901234567890121234567890123456789012345678901212345678901234567890123

50
12345678901234567890123456789012
12345678901234567890123456789012123456789012345678901234567890121234567890123456789012345678901212345678901234567890123
12345678901234567890123456789012 12345678901234567890123456789012
12345678901234567890123456789012
12345678901234567890123456789012123456789012345678901234567890121234567890123456789012345678901212345678901234567890123

98
A
:8
1. (1) `ǁÁãH÷O^ΰ°Qê =¶ió# JH›Æ~¡O =¶~¡^ΰ. XH›ø M 10. (3) PQSR Qê …è^ΰ.
E

=¶ã`Ç"Í° Qê =¶~¡°#°.
h

W    
.P

'+21 $(0/ )-54 -198


}}m 3 }}m 5
11. (1)
IC

2. (2) 1  
       
AD

2 }}m 0 }}m .
 
12. (2) MNL. (MNK Qê …è^ΰ)
AB
O

$ }}m & }}m (


 
13. (1) "³ò^Î\ ÷ ï~O_ÈH›Æ~Œ° ̄^Îí ~¡H›O䛽 K³Ok L<Œß~ò.
ER

6 }}m 8 }}m :
CH

 
14. (4) CDE XH›ø>è Z_È=° #°Oz 䛽_H÷ f‹¬°HË|_ÈÛq.
YD

\ 'QOM' a#ß"³°Ø#k. 15. (1) B XH›ø>è ‹¬i JH›Æ~¡O. q°ye#q ƒè‹² JH›Æ~Œ°.
,H

(1) XH› ø >è =O„¬ ô ° uiy LOk. q°ye#q


T

3. S    
%(+. 258; '),/ 9"%(
AM

(3)
ö~MìYO_¨`Ë U~¡æ_ L<Œß~ò.
16.
H

       
M

4. (4) Jx߆Çò AT `Ë =òQ®°‹¬°ë<Œß~ò. XH›ø SET `DŽ¬æ.


AM

/0 23 6 )*
IG

17. (2) M P S, R 4 T V, G J
5. (3) 2(B), 3, 5(E) ; 3(C), 5, 7(G) ; 5(E), 7, 11(K) ;

,-
KH

7(G), 11, 13(M).


M, A #$ D &' G.
R

6. (2) BCDA Qê …è^ΰ. 18. (4) ABCD, JKLM, UVWX, STU. ["Œ|° : 4
7. (1) - . / 0 19. (4) C, D, E ° `ǁãÁ H÷O^ΰ°Qê =¶ió<Œ PHê~¡O…Õ =¶~¡°æ
   
LO_È^ΰ. ["Œ|°. F.

1 2 3 5
20. (3) H, I =°i†Çò X ° ‡~¡ÅÞ q…Õ=°=ò =°i†Çò
\ 'OS' Qê …è^ΰ. `ǁÁãH÷O^ΰ° =¶ió<Œ PHê~¡O…Õ =¶~¡°æ~Œ^ΰ. M #°
8. (4) JLO Qê …è^ΰ.
`ǁÁãH÷O^ΰ° K͋²# PHê~¡O =¶~¡°`Ç°Ok.
9. (1) JLKM Qê …è^ΰ.

REASONING ABILITY (VERBAL) ○ ○ ○ ○ ○ ○ ○ ○ ○


56
○ ○ ○ ○ ○ ○ ○ ○ ○ ○ ○ ○ ○ ○ ○ ○ ○ ○ ○
Study Material
123456789012345678901234567890121234567890
123456789012345678901234567890121234567890123456789012345678901212
12345678901234567890123456789012123456789012345678901234567890121234567890123456789012345678901212345678901234567890123456789012123
12345678901234567890123456789012123456789012345678901234567890121234567890123456789012345678901212345678901234567890123
1234567890123456789012345678901212345678901234567890123456789012123456789012345678901234567890121234567890123
1234567890123456789012345678901212345678901234567890123456789012123456789012345678901234567890121234
123456789012345678901234567890121234567890
123456789012345678901234567890121234567890123456789012345678901212
12345678901234567890123456789012123456789012345678901234567890121234567890123456789012345678901212345678901234567890123456789012123
12345678901234567890123456789012123456789012345678901234567890121234567890123456789012345678901212345678901234567890123
1234567890123456789012345678901212345678901234567890123456789012123456789012345678901234567890121234567890123
1234567890123456789012345678901212345678901234567890123456789012123456789012345678901234567890121234
PRACTICE TEST on ‹¬OY¼ a#ß „¬sH›Æ
12345678901234567890123456789012123456789012345678901234567890121234567890123456789012345678901212345678901234567890123456789012123
123456789012345678901234567890121234567890123456789012345678901212
12345678901234567890123456789012123456789012345678901234567890121234567890123456789012345678901212345678901234567890123
1234567890123456789012345678901212345678901234567890123456789012123456789012345678901234567890121234567890123
1234567890123456789012345678901212345678901234567890123456789012123456789012345678901234567890121234
123456789012345678901234567890121234567890
12345678901234567890123456789012123456789012345678901234567890121234567890123456789012345678901212345678901234567890123456789012123
123456789012345678901234567890121234567890123456789012345678901212
12345678901234567890123456789012123456789012345678901234567890121234567890123456789012345678901212345678901234567890123
1234567890123456789012345678901212345678901234567890123456789012123456789012345678901234567890121234567890123
1234567890123456789012345678901212345678901234567890123456789012123456789012345678901234567890121234
12345678901234567890123456789012123456789012345678901234567890121234567890123456789012345678901212345678901234567890123456789012123
12345678901234567890123456789012123456789012345678901234567890121234567890123456789012345678901212345678901234567890123
1234567890123456789012345678901212345678901234567890123456789012123456789012345678901234567890121234567890123
1234567890123456789012345678901212345678901234567890123456789012123456789012345678901234567890121234
123456789012345678901234567890121234567890
12345678901234567890123456789012123456789012345678901234567890121234567890123456789012345678901212345678901234567890123456789012123
123456789012345678901234567890121234567890123456789012345678901212
12345678901234567890123456789012123456789012345678901234567890121234567890123456789012345678901212345678901234567890123456789012123
12345678901234567890123456789012123456789012345678901234567890121234567890123456789012345678901212345678901234567890123
1234567890123456789012345678901212345678901234567890123456789012123456789012345678901234567890121234567890123
1234567890123456789012345678901212345678901234567890123456789012123456789012345678901234567890121234
123456789012345678901234567890121234567890123456789012345678901212
12345678901234567890123456789012123456789012345678901234567890121234567890123456789012345678901212345678901234567890123456789012123
12345678901234567890123456789012123456789012345678901234567890121234567890123456789012345678901212345678901234567890123
1234567890123456789012345678901212345678901234567890123456789012123456789012345678901234567890121234567890123
1234567890123456789012345678901212345678901234567890123456789012123456789012345678901234567890121234

1. 1) 567 2) 234 3) 123 4) 458 12. 1) 13 2) 23 3) 33 4) 43

2. 1) 93 2) 75 3) 86 4) 42 13. 1) 524 2) 621 3) 972 4) 828

3. 1) 34-43 2) 62-71 3) 55-62 4) 83-92 14. 1) 1 2) 27 3) 64 4) 81

(Non-Gazetted- 2005) (J.L - 2004)

4. 1) 1, 3, 5, 7 2) 2, 4, 6, 8 15. 1) 6 2) 28 3) 198 4) 496

3) 11, 13, 14, 17 4) 34, 36, 38, 40 16. 1) 2, 3 2) 3, 5 3) 5, 7 4) 17, 19

5. 1) 6, 8, 9, 12 2) 6, 12, 8, 9 17. 1) 2 2) 5 3) 7 4) 11

3) 12, 9, 8, 6 4) 9, 12, 6, 8 18. 1) 13 2) 21 3) 31 4) 40

Y
6. 1)  2)  3)  4)  19. 1) 41 2) 57 3) 16 4) 91

    20. 1) 49 2) 64 3) 1 4) 125

M
7. 1) 2) 3) 4)
    21. 1) 56 2) 57 3) 58 4) 59

E
8. 1) 111 2) 11 3) 1111 4) 111111 22. 1) 9, 12, 15 2) 15, 20, 30

9. 1) 9 2) 25 3) 36 4) 48 3) 3, 4, 5 4) 6, 8, 10

D
9
59
(J.L. - 2004) 23. 1) 1 2) 8 3) 27 4) 36

CA
95
10. 1) 57 2) 67 3) 77 4) 87 24. 1) 2 2) 3 3) 4 4) 5

50
11. 1) 7 2) 17 3) 27 4) 37 25. 1) 25 2) 225 3) 500 4) 361

98
A
12345678901234567890123456789012
12345678901234567890123456789012123456789012345678901234567890121234567890123456789012345678901212345678901234567890123
12345678901234567890123456789012

q=~¡}ì`ÇàH› ["Œ|°°
12345678901234567890123456789012123456789012345678901234567890121234567890123456789012345678901212345678901234567890123
12345678901234567890123456789012
12345678901234567890123456789012 12345678901234567890123456789012
12345678901234567890123456789012 12345678901234567890123456789012
12345678901234567890123456789012123456789012345678901234567890121234567890123456789012345678901212345678901234567890123
12345678901234567890123456789012
12345678901234567890123456789012123456789012345678901234567890121234567890123456789012345678901212345678901234567890123
:8
12345678901234567890123456789012
12345678901234567890123456789012123456789012345678901234567890121234567890123456789012345678901212345678901234567890123
12345678901234567890123456789012 12345678901234567890123456789012
12345678901234567890123456789012 12345678901234567890123456789012
12345678901234567890123456789012123456789012345678901234567890121234567890123456789012345678901212345678901234567890123
12345678901234567890123456789012
12345678901234567890123456789012123456789012345678901234567890121234567890123456789012345678901212345678901234567890123
E
h

° J<Íq „¦¬°#=ò° Hêx 81 J<Ík


.P

(4) 458…Õ `DŽ¬æ q°ye#q =~¡°‹¬ ‹¬OY¼°.


IC

1. 1, 27, 64

9-3«6. q°ye# "Œ\÷H÷ 2 =‹¬°ëOk. Yzó`Ç"³°Ø# „¦¬°#=ò Hê^ΰ.


AD

2. (1)

6, 28, 496 ° „¬i„¬î~¡â ‹¬OY¼°. XH› ‹¬OY¼ †³òH›ø


AB
O

3. (3) 3 + 4 = 7 6 + 2 = 8 5 + 5 = 10 8 + 3 = 11 15. (3)

Hê~¡}ìOH›=ò "³ò`Çë=ò P ‹¬OY¼ä›½ ï~\÷“Ōá# P


ER
CH

4 + 3 = 7 7 + 1 = 8 6 + 2 = 8 9 + 2 = 11

11, 13, 14, 17° ã‰õ_…Õ …è=ô. ‹¬OY¼#° „¬i„¬î~¡â ‹¬OY¼ …èH› ‰×Ã^Μ ‹¬OY¼ JO^ΰ~¡°.
YD

4. (3)

5. (2) 6, 12, 8, 9° J#°‡`ÇO…Õ …è=ô. '6Ñ Hê~¡}ìOH›=ò° : 1, 2, 3, 6. g\÷ "³ò`ÇëO 12.
,H

15䛽 Yzó`Ç =~¡¾=üO …è^ΰ. 16. (1) Jx߆Çò ㄬ^¥ # ‹¬OY¼ [`Dž.è J~ò`Í 2, 3 =¶ã`Ç=ò
T

6. (4)
AM

 =~¡°‹¬ ㄬ^¥ # ‹¬OY¼ [`Ç. q°ye#q H›= ㄬ^¥ #


H

7. (3)

XH› JO`ÇHêx P=ië`Ç ^Ή§O‰×=ò. ‹¬OY¼°.
M
AM

111, 11KÍ x‰õÅ+¬OQê ƒìyOKÇ|_È^ΰ. Jx߆Çò ㄬ^¥# ‹¬OY¼…è. J~ò`Í 2 XH›ø>è ‹¬iㄬ^¥#
IG

8. (1) 17. (1)


2
‹¬OY¼.
KH

9. (4) 9 = 3

2
1{3«4, 2{1«3, 3{1«4, 4{0«4. ["Œ|° 2.
R

25 = 5 18. (2)
2
36 = 6 19. (3) '16Ñ ‹¬i‹¬OY¼. q°ye#q ƒè‹² ‹¬OY¼° (…è^¥) (1)
9, 25, 36° J<Íq Yzó`Ç"³°Ø# =~Œ¾°. 48 J<Ík q°ye#q ‹¬O†ÇòH›ë ‹¬OY¼°.
=~¡¾=ò Hê^ΰ. 20. (4) 49, 64, 1 ° Yzó`Ç =~¡¾ ‹¬OY¼°. 125 =~¡¾‹¬OY¼
10. (2) 67 XH›ø>è ㄬ^¥ # ‹¬OY¼. q°ye#q ‹¬O†ÇòH›ë ‹¬OY¼°. Hê^ΰ.
11. (3) 27 `DŽ¬æ q°ye#q ㄬ^¥# ‹¬OY¼°. 21. (4) '59Ñ ã„¬^¥#‹¬OY¼. q°ye#q ‹¬O†ÇòH›ë ‹¬OY¼°.
12. (3) 33 `DŽ¬æ q°ye#q ㄬ^¥# ‹¬OY¼°. 22. (2) 15, 20, 30° O|HË} ãuƒ’°[ Hù`ǁ° Hê=ô.
13. (1) 524, 9 KÍ x‰õÅ+¬OQê ƒìyOKÇ|_È^ΰ. q°ye#q 9KÍ
23. (4) 1, 8, 27° Yzó`Ç „¦¬°# ‹¬OY¼°. 36 XH› =~¡¾
x‰õÅ+¬OQê ƒìyOKÇ|_È°#°.
3
‹¬OY¼.
14. (4) 1 = 1

3
24. (3) 2, 3, 5° ㄬ^¥# ‹¬OY¼°. 4 XH› ‹¬O†ÇòH›ë ‹¬OY¼.
3 = 27

4
3
= 64
25. (3) 500 Yzó`Ç =~¡¾ ‹¬OY¼ Hê^ΰ.

REASONING ABILITY (VERBAL) ○ ○ ○ ○ ○ ○ ○ ○ ○


57
○ ○ ○ ○ ○ ○ ○ ○ ○ ○ ○ ○ ○ ○ ○ ○ ○ ○ ○
Study Material
123456789012345678901234567890121234567890123456789012345678901212
12345678901234567890123456789012123456789012345678901234567890121234567890123456789012345678901212345678901234567890123
1234567890123456789012345678901212345678901234567890123456789012123456789012345678901234567890121234567890123
1234567890123456789012345678901212345678901234567890123456789012123456789012345678901234567890121234
12345678901234567890123456789012123456789012345678901234567890121234567890123456789012345678901212345678901234567890123456789012123
123456789012345678901234567890121234567890123456789012345678901212345678901234567890123
12345678901234567890123456789012123456789012345678901234567890121234567890123456789012345678901212345678901234567890123
1234567890123456789012345678901212345678901234567890123456789012123456789012345678901234567890121234567890123
1234567890123456789012345678901212345678901234567890123456789012123456789012345678901234567890121234
123456789012345678901234567890121234567890123456789012345678901212
12345678901234567890123456789012123456789012345678901234567890121234567890123456789012345678901212345678901234567890123456789012123
123456789012345678901234567890121234567890123456789012345678901212345678901234567890123
12345678901234567890123456789012123456789012345678901234567890121234567890123456789012345678901212345678901234567890123
1234567890123456789012345678901212345678901234567890123456789012123456789012345678901234567890121234567890123
1234567890123456789012345678901212345678901234567890123456789012123456789012345678901234567890121234
12345678901234567890123456789012123456789012345678901234567890121234567890123456789012345678901212345678901234567890123456789012123
123456789012345678901234567890121234567890123456789012345678901212345678901234567890123
123456789012345678901234567890121234567890123456789012345678901212
PREVIOUS BITS
12345678901234567890123456789012123456789012345678901234567890121234567890123456789012345678901212345678901234567890123
1234567890123456789012345678901212345678901234567890123456789012123456789012345678901234567890121234567890123
1234567890123456789012345678901212345678901234567890123456789012123456789012345678901234567890121234
12345678901234567890123456789012123456789012345678901234567890121234567890123456789012345678901212345678901234567890123456789012123
123456789012345678901234567890121234567890123456789012345678901212345678901234567890123
123456789012345678901234567890121234567890123456789012345678901212
12345678901234567890123456789012123456789012345678901234567890121234567890123456789012345678901212345678901234567890123
1234567890123456789012345678901212345678901234567890123456789012123456789012345678901234567890121234567890123
1234567890123456789012345678901212345678901234567890123456789012123456789012345678901234567890121234
12345678901234567890123456789012123456789012345678901234567890121234567890123456789012345678901212345678901234567890123456789012123
123456789012345678901234567890121234567890123456789012345678901212345678901234567890123
12345678901234567890123456789012123456789012345678901234567890121234567890123456789012345678901212345678901234567890123
1234567890123456789012345678901212345678901234567890123456789012123456789012345678901234567890121234567890123
1234567890123456789012345678901212345678901234567890123456789012123456789012345678901234567890121234
12345678901234567890123456789012123456789012345678901234567890121234567890123456789012345678901212345678901234567890123456789012123
123456789012345678901234567890121234567890123456789012345678901212345678901234567890123
123456789012345678901234567890121234567890123456789012345678901212
12345678901234567890123456789012123456789012345678901234567890121234567890123456789012345678901212345678901234567890123456789012123
123456789012345678901234567890121234567890123456789012345678901212345678901234567890123
12345678901234567890123456789012123456789012345678901234567890121234567890123456789012345678901212345678901234567890123
1234567890123456789012345678901212345678901234567890123456789012123456789012345678901234567890121234567890123
1234567890123456789012345678901212345678901234567890123456789012123456789012345678901234567890121234

1. ãH÷Ok "Œ\÷…Õ q°Q®`Œ "Œ\÷`Ë ‡éeÀ‹ë a#ß"³°Ø#k Uk? ‹¬¶KÇ#° : (20 #°O_ 25 =~¡ä›½) ãH÷O^Î ~òzó# 7 ㄬ‰×߁…Õ
(AP. Panchyat Secretar y Screening -2016) <ŒQ®° „¬^¥° W=Þ|_#q. JO^ΰ…Õ =ü_È° „¬^¥° ª^Î$‰×¼OQê
1) KÇ`Ç°~¡ã‹¬=ò 2) ‹¬=¶O`Ç~¡ KÇ`°Ç ~¡°Ä[=ò L<Œß~ò. <Œ¾=„¬^ÎO =¶ã`ÇO a#ßOQê LOk. a#ßOQê L#ß
3) =$`Çë=ò 4) „¬~Œ=†Ç°=ò „¬^ÎO H›#°Qù#O_. (H›=ü¼xöH+¬<£ S.I. - 2012)
20. 1) „¦¬ô\˜ƒì…˜ 2) "Œbƒì…˜ 3) ãH÷ïH\˜ 4) K³‹¹
2. 338, 158, 639, 543, 449 …Õ ‹¬i‡éxk
21. 1) ‹¬[û 2) P"Œ° 3) =i 4) QË^Î°=°
(TS. SI. Mains -2016)
22. 1) q‰×ÞO 2) ‹¬¶~¡°¼_È° 3) KÇOã^ΰ_È° 4) ãQ®‚¬ð°
1) 449 2) 639 3) 543 4) 158
23. 1) #H›ø 2) 䛽H›ø 3) "Í°H› 4) „²eÁ
3. 17, 19, 23, 27, 29 …Õ XH›\ ÷ `Ç„æ¬ q°ye# <Œe¾O\÷…Õ
24. 1) ̄á# 2) ãH÷O^Î 3) z#ß 4) kQ®°=

Y
qƒk
è ‹¬°O
ë k. g\÷…Õ Pqƒk
è OKÍk. (TS. SI. Mains-2016)
25. D ãH÷Ok "Œ\÷…Õ Uk `Í_¨Qê L#ßk ? (ARO - 2012)

M
1) 17 2) 19 3) 27 4) 29 1) UPKEA 2) OMIDB

(4-9) : =~¡ä›½ Q® ㄬ‰×߁…Õ ‹¬i‡é‹² ‹¬OY¼x Q®°iëOKÇ°=ò. 3) 4)

E
VTOJE USNID

(TS. SI. Prelims -2016)


ãH÷O^Î ~ò=Þ|_# ㄬu ㄬ‰ß× …Õ
‹¬¶KÇ#° : (26 #°O_ 30 =~¡ä½› )

D
9
1) 282 2) 364 3) 4168 4) 6248

59
4. <Œ°Q®° „¬^Î=ò° ~ò=Þ|_¨Û~ò. JO^ΰ…Õ =ü_È° „¬^Î=ò°
1) 1285 2) 2213 3) 3074 4) 4172
CA
95
5. XöH ‹¬O|O^ÎO H›ey L<Œß~ò. <Œ¾= „¬^ÎO =¶ã`Ç=ò a#ßOQê
1) 1339 2) 2665 3) 3991 4) 4223 LOk. a#ßOQê L#ß „¬^¥xß H›#°Qù#O_.

50
6.

1) 11 2) 246 3) 327 4) 464


98
A
7. (Asst. Statistical Officer - 2012)

1) 2)
:8
8. 1) 5 2) 12 3) 25 4) 36 26. Japan Sri Lanka

3) 4)
E

1) 81 2) 441 3) 529 4) 613 Indonesia India


h

9.
.P

27. 1) 2) 3) 4)
IC

Club Hotel Hostel Inn


(10-12) : D ãH÷O^Î W=Þ|_# <Œ°Q®° „¬^¥…Õ a#ß"³°Ø#
1) 2)
AD

28. Apple Guava


^¥xx H›#°Qù#O_? (AP. SI. Prelims -2016)
3) Watermelon 4) Jackfruit
AB
O

10. 1) _™~Ÿ 2) rãƒì 3) ̂áì<Œ 4) l~Œ„¦² 29. 1) Nose 2) Eyes 3) Legs 4) Hands
ER
CH

11. 1) 13 2) 23 3) 37 4) 71 30. 1) Garlic 2) Chilli 3) Ginger 4) Potato


YD

12. 1) _¨eæ<£ 2) rãƒì 3) †Ç°<³‹¹ 4) ãHùHù_³á…˜ 31. H÷Ok "Œ\÷…Õ a#ß"³°Ø# ^¥xx Q®°iëOKÇO_.
,H

(Asst. Statistical Officer - 2012)


(13-16) D ãH÷ O ^Î W=Þ|_ # "Œ\ ÷ … Õ a # ß"³ ° Ø # ^¥xx
1) 2) 3) 3)
T
AM

IDRW LKOP GFTV JCQK


H›#°Qù#O_. (Deputy Jailors-2016)
H

32. ãH÷Ok "Œx…Õ XH› =¶@ q°Q®`Œ "Œ\÷H÷ a#ßOQê LOk.


1) _³a\˜ 2) _‡l\˜ 3) __ÈH›Æ<£ 4) q`Ÿã_¨
M

13.
P =¶@#° H›#°Qù#O_.
AM

1) ~¡¶a 2) ‰§Ì„¦á~Ÿ 3) ãQê<³á\˜ 4) \Շ*˜


IG

14.
(Health & Municipal Engineering - 2012)
1) NJLQ 2) TPPC 3) BKDF 4) OLTF
KH

15.
1) BFD 2) NRP 3) HLG 4) QUS

1) 3214 2) 2332 3) 3205 4) 2015


R

16. 33. ãH÷Ok "Œx…Õ XH› =¶@ q°Q®`Œ "Œ\÷H÷ a#ßOQê LOk. P
17. ãH÷Ok ㄬ‰×ß…Õ ‹¬i‡éx^¥xß ZO„²H› K͆ǰO_. =¶@#° H›#°Qù#O_. (Health & Municipal Engin - '12)
(Village Revenue Officer - 2014) 1) GTSH 2) BYXC

1) 14 2) 12 3) 13 4) 16 3) ETUF 4) LONM

18. ãH÷Ok ㄬ‰×߁…Õ ‹¬i‡éx^¥xß ZO„²H› K͆ǰO_. 34. D ãH÷Ok "Œ\÷…Õ J‹¬O|^Μ"³°Ø# ^¥xx H›#°Qù#°=ò.

(Village Revenue Officer - 2014) (‡éb‹¹ HêxÀ‹“|°…˜ - 2012)

1) 25 2) 49 3) 64 4) 81 1) 324 2) 841 3) 529 4) 756


35. ãH÷Ok <Œ°yO\÷…Õ =ü_È° U^Ë XH› ~¡H›=òQê XöH
19. H÷Ok =¶@…Õ ‹¬ï~á#k Hêxk
q^=
Î òQê LO_ XH› =~¡=
¾ ò#° U~¡æ~¡K°Ç #°. P =~¡=
¾ ò#䛽
(Lecturers in Govt. Polytechnic College - 2013)
K³O^Îxk Uk ? (S.I.-2006, 2008)
1) PEAR 2) TORE 3) REAP 4) TEAR
1) 25 2) 29 3) 63 4) 49
REASONING ABILITY (VERBAL) ○ ○ ○ ○ ○ ○ ○ ○ ○
58
○ ○ ○ ○ ○ ○ ○ ○ ○ ○ ○ ○ ○ ○ ○ ○ ○ ○ ○
Study Material
xö~‰
í H
× =
› ò° (36-45) : H÷O^Î W=Þ|_# "Œx…Õ Jª^¥~¡}=òQê 54. ãH÷Ok JH›Æ~Œ ‹¬=ü‚¬ð…Õ a#ß"³°Ø#k..
(ADTP, IFS -2012)
L#ß ^¥xx H›#°Qù#°=ò? (Communication S.I-2011)

36. 1) K³q 2) QùO`Ç° 1) IDRW 2) LKOP 3) GFTV 4) LCOX

55. ãH÷Ok ‹¬OMì¼ ‹¬=ü‚¬ð…Õ a#ß"³°Ø#k?(ATT, ADLIS-'12)


3) <Œ°H› 4) =ò䛽ø
1) 54 2) 108 3) 135 4) 96
37. 1) ABCD 2) DCBA
56. ãH÷Ok "Œx…Õ a#ß"³°#
Ø ^¥xx Q®°iëOKÇO_?(ASO-2012)
3) FGHI 4) KLMN
1) MKG 2) TDF 3) CRJ 4) KPO

38. 1) öH~¡ˆ× 2) <ŒQê…ìO_£ 57. ãH÷Ok =¶@…ÕÁ XH›\ ÷ ‹¬i‡éxk Hê^ΰ. ‹¬i‡éx P =¶@#°
3) ‹²O^£ 4) ‚¬ì~Œ¼<Œ Q®°iëOKÇO_? (ACF-2012)

39. 1) 19 2) 7 3) 9 4) 13 1) BFD 2) NBP 3) HLG 4) QUS

40. 1) KÇOã^ΰ_È° 2) ‰×Ãã䛽_È° 58. ãH÷Ok JH›Æ~¡ ‹¬=ü‚¬ð…ÕÁ a#ß"³°ØOk Uk? (PL-2012)

3) Q®°~¡°_È° 4) |°^Î°_È° 1) 2) 3) 4)

Y
MPS GJM DGK KNQ

59. D ãH÷Ok ㄬ‰×ß…Õ ‹¬i‡éq^¥xß ZO„²H› K͆ǰO_?


1) Qù°‹¬° 2) qQ·

M
41.
(VRA-2014)
3) „¬`ÇH›=ò 4) LOQ®~¡=ò
1) 53 2) 63 3) 73 4) 83

E
42. 1) „¬~¡Þ`Ç=ò 2) ‹¬=òã^Î=ò 60. D ãH÷Ok "Œx…Õ a#ß"³°Ø#k Uk? (VRA- 2014)

D
3) J_Èq 4) #H›Æã`Ç=ò

9
1) 14 2) 16 3) 25 4) 81

59
43. 1) q=¶#=ò 2) |‹¹ 61.
CA
D ãH÷Ok "Œ#…Õ a#ß"³°Ø#^Í^Ë H›#°Qù#O_?

95
3) ~¡"Œ}ì 4) ï~ၰ (SSC-20 14)(CGL-Tier-I)

50
44. 1) ‹¬Oz 2) \Õ„Ô 3) ƒìbó 4) |°@“ 1) 12-144 2) 13-156
98
A
45. 1) <͇…˜ 2) NOH› 3) 15-180 4) 16-176
:8
62. D ãH÷Ok "Œx…Õ a#ß"³°Ø#^Í^Ë ä›#°HËøO_?
3) WO_†Ç¶ 4) JO_È=¶<£
E
h

(SSC-20 14)(CGL-Tier-I)
.P

xö~œ‰×H›=ò° (46-50) : a#ß"³°Ø# ^¥xx H›#°HËøO_.


IC

1) 101 2) 212 3) 326 4) 111


AD

(Communication S.I-2011)
63. D ãH÷Ok "Œx…Õ Uk a#ß"³°Ø#k? (SSC-CGL-14)
1) \ìHê 2) ~¡¶|°…˜
AB
O

46.
1) 96 2) 16 3) 80 4) 36
3) +²eÁOQ· 4) \ì<£
ER
CH

64. 441, 484, 529, 566, 625 …Õ XH› ‹¬OY¼ a#ß"³°Ø#k
47. 1) ‹¬~¡‹¬°ž 2) „¬~¡Þ`Ç=ò P ‹¬OY¼Uk ?
YD

(Gr-1, 03)

3) ‹¬=òã^Î=ò 4) "Í_h\÷ =ü@ 1) 484 2) 529 3) 625 4) 566


,H

48. 1) _„¦Ôëï~á<£ 2) =°…èi†Ç¶ 65. D ãH÷Ok "Œ\÷…Õ J‹¬O|^Μ"³°Ø# ^¥xx H›#°Qù#°=ò?


T
AM

3) @¶¼=°~Ÿ 4) H›~Œ (P.C, 12)


H

1) 324 2) 841 3) 529 4) 756


M

49. 1) K³~¡ä›½ 2) q°…ÿÁ\˜


AM

66. a#ß"³°Ø# ^¥xx Q®°iëOKÇ°=ò.7,9,13,19


IG

(P.C, 06)
3) QË^Î°=° 4) *ç#ß
1) 19 2) 7 3) 9 4) 13
KH

50. 1) KÇió…˜ 2) =¶~ùøx 67. a#ß"³°Ø#^Í^Ë `³„¬O_. 15,21,63,81


R

3) ‚²ì@Á~Ÿ 4) J…ÿQêûO_È~Ÿ 1) 15 2) 21 3) 81 4) 63
51. ãH÷Ok ㄬ‰×ß…Õ qH›Æ}"³°Ø# ^¥xß Q®°iëOKÇ°=ò? 68. a#ß"³°Ø# ‹¬OY¼ U^Ë H›#°Qù#O_. 9, 25, 36, 48
(Group-II-2012) 1) 9 2) 25 3) 36 4) 48
1) 289 2) 361 3) 529 4) 441 69. D ãH÷Ok "Œ\÷…Õ a#ß"³°Ø#^Í^Ë H›#°HËøO_.

52. ãH÷Ok "Œx…Õ XH› =¶@ q°Q®`Œ "Œ\÷H÷ a#ßOQê LOk.P 1, 27, 81, 64 (J.L. 2004)

=¶@#° H›#°Qù#O_? (Ass.Engg.& PH, ME-2012)


1) 1 2) 27 3) 81 4) 64
70. 1) _³`Ÿ : _‹Ô*˜ 2) q°…˜ø : |@~Ÿ
1) BFD 2) NRP 3) HLG 4) QUS
3) ãöQ„¹ : "³á<£ 4) ã䛀_£ P~ò…˜ : Uªæ¼…˜“
53. ãH÷Ok "Œx…Õ XH› =¶@ q°Q®`Œ "Œ\÷H÷ a#ßOQê LOk.
(A.P. S.I. Mains 2017)

P =¶@#° H›#°Qù#O_? (Ass.Engg.& PH, ME-2012)


71. 1) 3214 2) 2332 3) 3205 4) 2015

1) GTSH 2) BYXC 3) ETUF 4) LONM (A.P. S.I. Mains 2017)

REASONING ABILITY (VERBAL) ○ ○ ○ ○ ○ ○ ○ ○ ○


59
○ ○ ○ ○ ○ ○ ○ ○ ○ ○ ○ ○ ○ ○ ○ ○ ○ ○ ○
Study Material
12345678901234567890123456789012123456789012345678901234567890121234567890123456789012345678901212345678901234567890123
1234567890123456789012 123456789012345678901234

q=~¡}ì`ÇàH› ["Œ|°°
12345678901234567890123456789012123456789012345678901234567890121234567890123456789012345678901212345678901234567890123
1234567890123456789012 123456789012345678901234
12345678901234567890123456789012123456789012345678901234567890121234567890123456789012345678901212345678901234567890123
1234567890123456789012 123456789012345678901234

1234567890123456789012
PREVIOUS BITS :
12345678901234567890123456789012123456789012345678901234567890121234567890123456789012345678901212345678901234567890123
1234567890123456789012 123456789012345678901234
12345678901234567890123456789012123456789012345678901234567890121234567890123456789012345678901212345678901234567890123
1234567890123456789012 123456789012345678901234
12345678901234567890123456789012123456789012345678901234567890121234567890123456789012345678901212345678901234567890123
1234567890123456789012 123456789012345678901234
12345678901234567890123456789012123456789012345678901234567890121234567890123456789012345678901212345678901234567890123
123456789012345678901234

1. (4) 1, 2, 3° ö H Æ ã `Ç q °u…Õ =KÇ ° ó PHê~Œ°, Hêx 9. (4) 613 `DŽ¬æ q°ye#=x߆Çü =~¡¾ ‹¬OY¼…è
2 2 2
„¬ ~ Œ=†Ç ° =ò J#°#k Conic section H÷ 81 - 9   441 « 21   529 « 23

‹¬|OkOz#k. 10. (3) '̂áì<ŒÑ `DŽ¬æ q°ye#=xß '‰§Hꂬð~¡°…èÑ.

2. (1) Wzó# ‹¬OY¼…Õ 11. (2) 1 { 3 « 4   2 { 3 « 5   3 { 7 « 10  

3 + 3 + 8 = 14 7 { 1 « 8

1 + 5 + 8 = 14 ‹¬i‹¬OY¼…è '5Ñ ƒè‹²‹¬OY¼.

6 + 3 + 9 = 18
12. (4) Ju ã䛀~¡"³°Ø# [O`Ç°=ô.

5 + 4 + 3 = 12
13. (2) (2)= ^¥x…Õ q°ye# JxßO\÷…Õ ƒì¼OH± JHÒO@°

#°O_ _È|°Ä° f‹²"͆ǰ|_È`Œ~ò.


Hêx, 4 { 4 { 9 « 17 (ƒè‹² ‹¬OY¼)

Y
Hê=ô# '449Ñ a#ß"³°Ø#k. 14. (3) Ju q°"³á# ~Œ~ò. (W=Þ|_# ~Œˆ×¤…Õ)

M
15. (4)
3. (3) W=Þ|_# ‹¬OY¼…Õ '27Ñ XH›ø>è ㄬ^¥#‹¬OY¼ Hê^ΰ.
16. (4)

E
W=Þ|_# ‹¬OY¼…Õ
q°ye#=xß ƒè‹² ㄬ^¥# ‹¬OY¼…è.
Wzó# ‹¬OY¼…Õ 2, 3, 4 ä›½ =iëOKÇ°#k
3214 ® 3 { 2 { 1 « 4 « 10

D
®

9
4. (1)

59
2332 2 { 3 { 3 { 2 « 10
‹¬OY¼…Õx z=i JOïH…Õx ‹¬Q®O þ "³ò^Î\ ÷ JOïH «
CA ®

95
3205 3 { 2 { 0 { 5 « 10
=°^Î¼‹¬Ö JOïH
® 9

50
2015 2 { 0 { 1 { 5 «
  
s     s     s   
98
A
17. (3) XH› '13Ñ `DŽ¬æ q°ye#q Jxß ‹¬i ‹¬OY¼…è.
  
:8
Hêx Wk '282Ñ ä›½ =iëOKÇ^ΰ. 18. (3) XH›ø '64Ñ `DŽ¬æ q°ye#=xß    
E
h

 ƒè‹² ‹¬OY¼ =~Œ¾…è


.P

sx
IC

   

AD

5. (2) Wzó# ‹¬OY¼…Õ 1, 3, 4ä›½ =iëOKÇ°#k. 19. (2) 'TORE' …Õ `DŽ¬æ q°ye# "Œ@xßO\÷…Õ E, A, R
AB
O

JOïH "³ò`ÇëO « „¬^ΰªÖ#O…ÕH÷ JOïH䛽 ï~\÷“O„¬ô J#° JH›Æ~Œ° =‹¬°ë<Œß~ò. Hê=ô# W^Í a#ß"³°Ø#k.
1 { 2 { 8 { 5 « 16 Þ 8 þ 2 « 16
ER
CH

20. (4) XH›ø 'K³‹Ñ¹ `Ç„æ¬ q°ye#=xß outdoor games. Hê=ô#


3 { 0 { 7 { 4 « 14 Þ 7 þ 2 « 14
YD

W^Í a#ß"³°Ø#k.
4 { 1 { 7 { 2 « 14 Þ 7 þ 2 « 14
,H

21. (2) 'P"Œ°Ñ a#ß"³°#


Ø q. Jk XH›ø>è kÞ^΁c[O q°ye#q
\ÿ2213 a#ß"³°Ø#k.
T
AM

UH›^΁c*쁰.
4= ‹¬OY¼…Õ `DŽ¬æ q°ye# JxßO\÷…Õ
H

6. (4)
22. (1)
M

q‰×ÞO…Õx ƒìQꁰ ‹¬¶~¡°¼_È°, KÇOã^ΰ_È°, ãQ®‚¬ð°.


‹¬OY¼…Õx "³ò^Î\ ÷ ï~O_ÈOïH°, z=i ï~O_ÈOïH° 13
AM
IG

Hê=ô# Wk a#ß"³°Ø#k.
KÍ`Ç ƒìyOKÇ|_È°`Œ~ò.
KH

23. (3) XH›ø "Í°H› `DŽ¬æ q°ye#=xß =¶Oª‚¬ð~¡°…è.


       
R

n n n n n n 24. (3) XH› z#ß `DŽ¬æ q°ye=x߆Çü ㄬ^͉§xß ‹¬¶zªë~ò.


¨ ¨ ¨ ¨ ¨ ¨
25. (1) XH›ø U P K E A …Õ Vowel `Ë "³ò^΅ÿá, vowel `Ë
7. (2) XH›ø ï~O_È= ‹¬OY¼…Õ `DŽ¬æ q°ye# JxßO\÷…Õ JO`Ç=°=ô`Ç°Ok. Hê=ô# Wk a#ß"³°Ø#k.
‹¬OY¼…Õ ("³ò^Î\ ÷ JOïH)3 J#°#k ï~O_È= JOïHQê 26. (4) W=xß n=ô°. WO_†Ç¶ nބ¬YO_È=ò.
W=Þ|_Ok.
27. (1) 'Club' menbers LO\ì~¡°.
        28. (3) XH›ø 'Watermelon' …Õ 'water' content Z䛽ø=.
n n n
   29. (1) ‰×s~¡O…Õ Ì„á ƒìQ®O…Õ 'eyes'
8. (3) ï~O_È° =~¡°‹¬ ㄬ^¥# ‹¬OY¼ "³ò`ÇëO#° ‹¬OY¼Qê
=°^Î¼…Õ 'hands'
W=Þ_È"³°Ø#k.
z=~¡# 'legs' LO\ì~ò.
2 { 3 « 5   5 { 7 «12   11 { 13 « 24  
17 { 19 « 36.
\ Nose a#ß"³°Ø#k.

REASONING ABILITY (VERBAL) ○ ○ ○ ○ ○ ○ ○ ○ ○


60
○ ○ ○ ○ ○ ○ ○ ○ ○ ○ ○ ○ ○ ○ ○ ○ ○ ○ ○
Study Material
30. (2) Chilli XH›ø>è "³òH›ø Ì„á ƒìQ®O…Õ =‹¬°ëOk. 52. (3) q°ye#q {4, -2#° J#°‹¬iOKÇ°KÇ°#ßq.

31. (3) =¼uö~H› JH›Æ~Œ° …è=ô (3)…Õ 53. (3) (=¼uö~H› JH›Æ~Œ° q°ye#q).

(Z)…Õ *ZZX 3  % YZZ


YZZ ZZX 8
G T S H B Y C X
(a)…Õ - YZZ
ZZX 0  , YZZ
ZZX 1
(‹²)…Õ ( YZZ
ZZX 5  ' YZZ
ZZX 7
E V U F L O M N
ZZX 2  $ YZZ
(_)…Õ + YZZ ZZX 9
'‹²Ñ a#ß"³°Ø#k. 54. (3)

32. (3) # }}m



' }}m

% 55. (3) '135Ñ `DŽ¬æ q°ye#=xß ‹¬i‹¬OY¼°

/ }}m

3 }}m

1 56. (2) 57. (3) 58. (3)

) }}m - }}m (
 
59. (2) 63 `DŽ¬æ q°ye#=xß ã„¬^¥# ‹¬OY¼°
2 }}m 6 }}m 4 \ 'HLG'

Y
 
a#ß"³°Ø#k. 60. (1) 4 = 16; 52 = 25; 9 2 = 81
2

M
33. (3) XH›ø ^¥x…Õ<Í =¼uö~H› JH›Æ~Œ° …è=ô.
'14Ñ `DŽ¬æ q°ye#=xß U^Ë ‹¬OY¼ =~Œ¾…è
34. (4) '756Ñ `DŽ¬æ q°ye#=xß „¬i„¬î~¡â =~¡¾ ‹¬OY¼…è.
144 156 180 176

E
18
2 2 2
« 324   29 « 841   23 « 529. 61. (4) = 12; = 13; = 15; = 16
12 12 12 11

D
9
59
35. (2) Wk ㄬƒ’= ‹¬OY¼ q°ye#q ‹¬O†ÇòH›ë ‹¬OY¼°. 62. (1) 101 ㄬ^¥# ‹¬OY¼, q°ye#q ‹¬O†ÇòH›ë ‹¬OY¼°.

CA
95
36. (2) q°ye#q *ìý<ÍOãk†Ç¶°. 63. (3) 96 16 80 36

50
+1 +1 +1
37. (2) A ⎯⎯→ B ⎯⎯→ C ⎯⎯→ D 9{6«15  1{6«7  8{0«8  3{6«9

98
A
−1 −1 −1 2 2 2 2
D ⎯⎯→ C ⎯⎯→ B ⎯⎯→ A 64. (4) 441«21 , 484«22 , 529«23 , 625«25
:8
+1 +1 +1
F ⎯⎯ → G ⎯⎯ → H ⎯⎯ →I 441, 484, 529, 625° Yzó`Ç=~¡¾‹¬OY¼°, 566
E
h

+1 +1 +1 Hê^ΰ H›#°H› a#ß"³°Ø#k.


K ⎯⎯→ L ⎯⎯→ M ⎯⎯→ N
.P
IC

2 2 2
65. (4) 324«18 , 841«29 , 529«23 Hê=ô# 324, 841,
AD

38. (3) q°ye#=xß ~Œ¢‘“°.


529° Yzó`Ç =~¡¾‹¬OY¼° 756 Yzó`Ç =~¡¾‹¬OY¼
AB
O

39. (3) 9 `DŽ¬æ q°ye#=xß ã„¬^¥# ‹¬OY¼…è.


Hê^ΰ H›#°H› a#ß"³°Ø#k.
ER

40. (1) 'KÇOã^ΰ_È°Ñ `DŽ¬æ q°ye#=xß 'ãQ®‚¬ð…èÑ


CH

66. (3) 7,13,19° ㄬ^¥ # ‹¬OY¼°, 9 ‹¬O†ÇòH›ë ‹¬OY¼ Hê=ô#


YD

41. (2) qQ· XH›ø>è a#ß"³°Ø#k (Pƒ’~¡} Hê^ΰ) 9 a#ß"³°Ø#k.


67. (1)
,H

42. (4) #H›Æã`Ç=ò `DŽ¬æ q°ye#=xß 'ƒ’¶q°Ñ g°^Î<Í LO\ì~ò.


2 2 2
T

68. (4) 9«3 , 25«5 , 36«6


AM

43. (3) q=¶#O, |‹¹, ï~ၰ J#°#k ~¡"Œ}ì =¶~Œ¾°.


9,25,36° Yzó`Ç =~¡¾‹¬OY¼°, 48Hê^ΰ H›#°H› 48
H

44. (2)
M

a#ß"³°Ø#k.
AM
IG

45. (4) JO_È=¶<£ XH›ø>è n=ô° q°ye#=xß ^͉§…è. 69. (3) 1,27,64° Yzó`Ç „¦¬°<Œ° Hêx 81 Yzó`Ç =~¡¾=ò
KH

46. (4) 47. (4) 48. (3) 49. (3) Hê=ô# a#ß"³°Ø#k.
R

50. (1) 70. (1)


71. (4) 2015 ‹¬OY¼…Õx JOïH "³ò`ÇëO 10 ~Œ=@O…è^ΰ.
51. (4) 441 `DŽ¬æ q°ye#=xß ã„¬^¥# ‹¬OY¼ =~Œ¾…è
ㄬ‰×ß…Õ Wzó# Jxß ‹¬OY¼…Õx JOïH "³ò`ÇëO 10
            =‹¬°ëOk.

REASONING ABILITY (VERBAL) ○ ○ ○ ○ ○ ○ ○ ○ ○


61
○ ○ ○ ○ ○ ○ ○ ○ ○ ○ ○ ○ ○ ○ ○ ○ ○ ○ ○
Study Material
4 JH›Æ~¡=¶ „¬sH›Æ (Alphabet Test)
=òMì¼O‰§°...
1. D qƒìQ®O…Õx ㄬ‰ß× ° POQ®Á =~¡=
â ¶ g°^Î P^¥~¡„_
¬  =ªë~ò. J…ìöQ JH›~Æ =
¡ ò ‹¬OY¼ Hù~¡ä½› Chapter-1…Õ ‡ÚO^ΰ„¬ió#
>è|°…˜ž KǶ_ÈO_.
2. nx†Ç°O^ΰ =òY¼OQê JH›Æ~¡ã‰õ}÷…Õ „¬ô#~Œ=$`Ç=°†Í°¼ JH›Æ~Œ°, ï~O_È° JH›Æ~Œ =°^Î¼Q® JH›Æ~Œ°, XH› JH›Æ~ŒxH÷ 䛽_/Z_È=°
"³á„¬ô# Q® JH›Æ~Œ° H›#°Qù#°@, „¬^¥#° POQ®Á x„¦¬°O@°=ô ㄬHê~¡O J=°~¡°ó@ …ìO\÷ ㄬ‰×߁° =ªë~ò.

Y
=¶ki ㄬ‰×߁°

M
E
„¬^ÎO…Õx JH›Æ~Œ POQ®Á =~¡â=¶…Õx JH›Æ~Œ°
MODEL - 1

D
. /01

9
% 0/ "5&

59
1. P…ì权ÿ\˜…Õ Z_È=° #°O_ 22= JH›Æ~ŒxH÷, 䛽_ #°O_
CA %0/ " 5&

95
21= JH›Æ~ŒxH÷ =°^Î¼‹¬ÖOQê LO_È° JH›Æ~¡O Uk?
" %$# "

50
1) N 2) P 3) L 4) UnHê^ΰ 2. 'OSTENTATION' J#° „¬^O
Î …Õx U^Íx ~
ï O_È° JH›~
Æ Œ
98
A
ª^Î#: (1) POQ®Á =~¡â=¶ =°^Î¼ `Í_¨ P…ì权ÿ\˜ž…Õ J^Í JH›Æ~Œ =°^Î¼ `Í_¨`Ë
®
:8
Z_È=° ABCDE ' GHIJKLM / OPQRS ‹¬=¶#OQê ƒè^ÎO H›ey# JH›Æ~¡ [O@° Zxß H›=ô?
E

¬
h

TU 7 YWXYZ 䛽_ (W~¡°"³á„¬ô KǶ‹²#KË)


.P
IC

JH›Æ~¡=¶…Õ Z_È=° #°O_ 22= JH›Æ~¡O « V 1) 2 2) 3 3) 4 4) UnHê^ΰ


AD

䛽_ #°O_ 21= JH›Æ~¡O « F ª^Î#: (2) „¬^ÎO…Õx JH›Æ~Œ =~¡â=¶…Õx JH›Æ~Œ°
AB
O

\ =°^Î¼ JH›Æ~¡O « N ãH›=°=ò ãH›=°=ò


ER

0 45&/ * "5*0/ 0 1234 5


CH

2. P…ì权ÿ\˜…Õx "³ò^Î\ ÷ ‹¬Qêxß =¼uö~H› k‰×…Õ ~Œ‹²#,


0 45 &/5"5*0/ 123 45
YD

䛽_ #°O_ 18= JH›~ Æ ŒxH÷ 䛽_…Õ 6= JH›~


Æ O
¡ U=°Q®°#°?
045&/5"5* 0/ 0/.-3
,H

1) O 2) P 3) M 4) N
\ Hê=‹²# JH›Æ~¡ [O@° '3Ñ H›=ô.
T

ª^Î#: (1) POQ®Á=~¡â=¶ "³ò^Î\ ÷ ‹¬Qêxß u„²æ ã"Œ†Ç°Qê


AM

®
Z_È=° MLKJIHGFEDCBAN 0 PQRSTU
H
M

(OT, ST, ON)

¬
7 YWXYZ 䛽_
AM

MODEL - 3
IG

䛽_# 18 JH›Æ~ŒxH÷ 䛽_…Õ 6= JH›Æ~¡O


KH

1. 'DISTURBANCE' J#° „¬^ÎO…Õx "³ò^Î\ ÷ JH›Æ~Œxß


« 䛽_ #°O_ (18-6)= JH›Æ~¡O
R

z=i JH›Æ~¡O`Ë#¶  ï~O_È= JH›Æ~Œxß 10= JH›Æ~¡O`Ë#¶


« 䛽_ #°O_ 12= JH›Æ~¡O
.... „¬~‹
¡ æ¬ ~¡ =¶iæ_ `Ç~ŒÞ`Ç =KÇ°ó =°^¼Î JH›~
Æ O
¡ `Ç~°¡ "Œ`Ç
«O JH›Æ~¡O ?
MODEL - 2 1) U 2) N 3) S 4) D
ª^Î#: (1) ^Î`Ç넬^ÎO DISTURBANCE ®
1. 'DONATE' J#° „¬^ÎO…Õx U^Íx ï~O_È° JH›Æ~Œ =°^Î¼ ㄬ‰ß× …Õ K³„æ² # JH›~Æ Œ ªÖ<Œ =¶~¡°æ K͋#² U~¡æ_È°
^ζ~¡O P…ì权ÿ\˜…Õx ï~O_ÈH›Æ~Œ =°^Î¼ ƒè^¥xß ‹¬=°OQê ãHù`Çë „¬^Î=ò ECNAB 3 UTSID ®
Q® JH›Æ~¡ [O@° W~¡°"³á„¬ô…ì KǶ‹²# Zxß Q®=ô? Ì„á „¬^ÎO…Õ =°^Î¼‹¬Ö JH›Æ~¡O « R
1) 1 2) 3 3) 4 4) UnHê^ΰ ^¥x `Ç~ŒÞ`Ç JH›Æ~¡O « U
ª^Î#: (4) 'DONATE' J#° „¬^ÎO…Õ AD, NO J#° ï~O_È° 2. 'CREDENTIALS' J#° „¬^ÎO…Õx "³ò^Î\ ÷ =°i†Çò
[O@° H›=ô. ï~O_È=, 3= Ê 4=, =°i†Çò 5= Ê 6= JH›Æ~Œ#°

REASONING ABILITY (VERBAL) ○ ○ ○ ○ ○ ○ ○ ○ ○


62
○ ○ ○ ○ ○ ○ ○ ○ ○ ○ ○ ○ ○ ○ ○ ○ ○ ○ ○
Study Material
„¬~¡‹¬æ~¡O =¶iæ_ K͋²<Œ Z_È=° #°O_ 7= JH›Æ~ŒxH÷ 2. 'NUMKIPP' J#° JH›Æ~¡=ò#° J~¡Ö=O`Ç"³°Ø# POQ®Á
䛽_…Õ 3= JH›Æ~¡O Uk? „¬^ÎOQê =¶ió ã"Œ‹²# P „¬^ÎO…Õ z=i JH›Æ~¡O Uk?
1) N 2) H 3) I 4) UnHê^ΰ 1) 2) 3) 4) UnHê^ΰ
ª^Î#: (4) ^Î`Ç넬^ÎO ® CREDENTIALS
ª^Î#: (1)
N M

^Î`Çë „¬^Î=ò…Õx JH›Æ~Œ#° ‹¬i†Ç°Q®° ãH›=°O…Õ


K

Ì„á „¬^ÎO…Õx JH›Æ~Œ#° ㄬ‰×ß…Õ K³„²æ# q^ÎOQê


J=°ió# U~¡æ_È° „¬^ÎO
=¶~¡°æ° K͋²# U~¡æ_È°
ãHù`Çë „¬^Î=ò ® RCDENEITL " S
® PUMPKIN (Q®°=°à_Hê†Ç°)

Ì„á „¬^ÎO…Õx Z_È=° #°O_ 7= JH›Æ~¡O « I


\ Ì„á „¬^Î…Õ z=i JH›Æ~¡O « 'N'.

'I' H÷ 䛽_…Õ 3= JH›Æ~¡O « A


MODEL - 6
MODEL - 4 1. 'WONDERFUL' J#° „¬^ÎO…Õx Jxß JH›Æ~Œ#°

Y
1. 'STAINLESS' J#° „¬^ÎO…Õx JH›Æ~Œ#° =¶~¡ó䛽O_¨ P…ì权ÿ\˜ P~¡Û~Ÿ…Õ J=°~¡óQê `Ç# ªÖ<Œxß =¶~¡°óHËx

M
L#ßk L#ß@°Á qƒ’lOz# Zxß J~¡Ö=O`Ç"³°Ø# „¬^¥°
JH›Æ~Œ° Zxß ?
U~¡æ_È°#°.
1) 2) 3) 4) UnHê^ΰ

E
1) 1 2) 2 3) 3 4) UnHê^ΰ 1 2 4

ª^Î#: (2) STAIN, LESS J#° ï~O_È° „¬^¥° U~¡æ_È°`Œ~ò. ª^Î#: (1) ^Î`Çë „¬^Î ãH›=°=ò ®

D
9
WONDERF U L

POQ®Á =~¡â=¶ ㄬHê~¡O ®

59
2. PERSONALITY J#° „¬^Î=ò…Õ JH›Æ~Œ#° "Œ\÷ DEFLNOR U W

CA
95
ãH›=°O =¶~¡ó䛽O_¨ L„¬†³¶yOz J~¡Ö=ò H›ey# Zxß JH›Æ~Œ ãH›=°=ò
‹¬Þ`ÇOã`Ç „¬^Î=ò°Qê ã"Œ†Ç°=KÇ°ó ?
\ XöH XH› JH›~Æ O¡
50
'U' ªÖ<Œxß =¶~¡°óHË=_ÈO …è^°Î .
1) 2 2) 1 3) 3 4) UnHê^ΰ
98
A
ª^Î#: (1) PERSONAL, SON J#°#q ‹¬Þ`ÇOã`Ç „¬^¥°. 2. 'RATIONALISATION' J#° „¬ ^ Î O …Õx Jxß
:8
JH›Æ~Œ#° POQ®Á JH›Æ~¡ ãH›=°O…Õ J=°~¡óQê Z_È=° #°O_
E

MODEL - 5
h
.P

5= JH›Æ~¡"Í°=°=ô`Ç°Ok ?
IC

1. OVERRCE J#° JH›Æ~¡=ò#° J~¡Ö=O`Ç"³°Ø# POQ®Á


AD

1) N 2) I 3) A 4) O
„¬^ÎOQê =¶ió ã"Œ‹²# P „¬^ÎO…Õ Z_È=° #°O_ 5=
AB
O

ª^Î#: (2) RATIONALISATION JH›Æ~Œ#° POQ®Á =~¡â=¶


JH›Æ~¡O Uk?
ㄬHê~¡O J=°~¡ó=KÇ°ó.
ER
CH

1) E 2) V 3) R 4) O
YD

ª^Î#: (2) ^Î`Çë „¬^Î=ò…Õx JH›Æ~Œ#° ‹¬i†Ç°Q®° ãH›=°O…Õ AAAI * ITNNOORSTT

J=°ió# U~¡æ_È° „¬^ÎO RECOVER ̄á^¥x…Õ Z_È=° #°O_ 5= JH›Æ~¡O 'I'.


,H

\Ì„á „¬^ÎO…Õ Z_È=° #°O_ 5= JH›Æ~¡O « V.


T
AM

123456789012345678901234567890121234567890123456789012345678901212
12345678901234567890123456789012123456789012345678901234567890121234567890123456789012345678901212345678901234567890123456789012123
1234567890123456789012345678901212345678901234567890123456789012123456789012345678901234567890
12345678901234567890123456789012123456789012345678901234567890121234567890123456789012345678901212345678901234567890123
1234567890123456789012345678901212345678901234567890123456789012123456789012345678901234567890121234567890123
1234567890123456789012345678901212345678901234567890123456789012123456789012345678901234567890121234
123456789012345678901234567890121234567890123456789012345678901212
12345678901234567890123456789012123456789012345678901234567890121234567890123456789012345678901212345678901234567890123456789012123
1234567890123456789012345678901212345678901234567890123456789012123456789012345678901234567890
H

12345678901234567890123456789012123456789012345678901234567890121234567890123456789012345678901212345678901234567890123
1234567890123456789012345678901212345678901234567890123456789012123456789012345678901234567890121234567890123
1234567890123456789012345678901212345678901234567890123456789012123456789012345678901234567890121234
12345678901234567890123456789012123456789012345678901234567890121234567890123456789012345678901212345678901234567890123456789012123
1234567890123456789012345678901212345678901234567890123456789012123456789012345678901234567890
123456789012345678901234567890121234567890123456789012345678901212
12345678901234567890123456789012123456789012345678901234567890121234567890123456789012345678901212345678901234567890123
1234567890123456789012345678901212345678901234567890123456789012123456789012345678901234567890121234567890123
1234567890123456789012345678901212345678901234567890123456789012123456789012345678901234567890121234
PRACTICE TEST
12345678901234567890123456789012123456789012345678901234567890121234567890123456789012345678901212345678901234567890123456789012123
1234567890123456789012345678901212345678901234567890123456789012123456789012345678901234567890
M

12345678901234567890123456789012123456789012345678901234567890121234567890123456789012345678901212345678901234567890123
1234567890123456789012345678901212345678901234567890123456789012123456789012345678901234567890121234567890123
1234567890123456789012345678901212345678901234567890123456789012123456789012345678901234567890121234
123456789012345678901234567890121234567890123456789012345678901212
12345678901234567890123456789012123456789012345678901234567890121234567890123456789012345678901212345678901234567890123456789012123
1234567890123456789012345678901212345678901234567890123456789012123456789012345678901234567890
12345678901234567890123456789012123456789012345678901234567890121234567890123456789012345678901212345678901234567890123
1234567890123456789012345678901212345678901234567890123456789012123456789012345678901234567890121234567890123
1234567890123456789012345678901212345678901234567890123456789012123456789012345678901234567890121234
12345678901234567890123456789012123456789012345678901234567890121234567890123456789012345678901212345678901234567890123456789012123
1234567890123456789012345678901212345678901234567890123456789012123456789012345678901234567890
123456789012345678901234567890121234567890123456789012345678901212
12345678901234567890123456789012123456789012345678901234567890121234567890123456789012345678901212345678901234567890123
1234567890123456789012345678901212345678901234567890123456789012123456789012345678901234567890121234567890123
1234567890123456789012345678901212345678901234567890123456789012123456789012345678901234567890121234
12345678901234567890123456789012123456789012345678901234567890121234567890123456789012345678901212345678901234567890123456789012123
1234567890123456789012345678901212345678901234567890123456789012123456789012345678901234567890
AM

12345678901234567890123456789012123456789012345678901234567890121234567890123456789012345678901212345678901234567890123456789012123
1234567890123456789012345678901212345678901234567890123456789012123456789012345678901234567890
IG

POQ®Á JH›Æ~¡=¶…Õ JKÇ°ó#° `ùyOKÇQê q°ye# COUNTRY „¬^O


Î …Õ JH›~
Æ Œ`Ë U~¡æ_Í U „¬^O
Î _H› #s…Õ
KH

1. 5.

JH›Æ~Œ…Õ 12= JH›Æ~¡O䛽 =°i†Çò 16= JH›Æ~¡O䛽 Q® "³ò^Î@Qê =‹¬°ëOk?


R

=°^Î¼ JH›Æ~¡"Í°k? 1) CNOTRUY 2) CNOTRYU

1) N 2) Q 3) R 4) S 3) CNORTYU 4) CNORTUY

2. 14, 15, 20 JH›Æ~Œ`Ë U~¡æ_Í J~¡Ö=O`Ç"³°Ø# „¬^Î"Í°k? 6. POQ®Á JH›Æ~¡=¶ #°Oz ƒè‹² JH›Æ~Œ#° `ùyOz#>ÿåÁ`Í
1) NOT 2) ONT q°ye# "Œ\÷…Õ =°^Î¼ JH›Æ~¡"Í°k?
3) TNO 4) TON 1) L 2) M 3) N 4) O

3. Z =°i†Çò R  =°^Î¼ U =°i†Çò S #° `ùyOz# 7. POQ®Á JH›Æ~¡=¶…Õ ƒè‹² JH›Æ~Œ° f‹¬°Hù#ß>ÿåÁ`Í =°^Î¼
„²=°à@ =KÍó =°^Î¼ JH›Æ~¡"Í°k? JH›Æ~¡O Uk?
1) V 2) W 3) U 4) X 1) K 2) L 3) M 4) N

4. nekpremjtnekekepkek …Õ k `Ç~ŒÞ`Ç =KÍó 'e' ° 8. SECOND J<Í „¬^ÎO…Õ =zó# U JH›Æ~¡O ECONOM-

Zxß? ICS …Õ …è^ΰ.


1) 3 2) 5 3) 6 4) 2 1) E 2) C 3) D 4) O

REASONING ABILITY (VERBAL) ○ ○ ○ ○ ○ ○ ○ ○ ○


63
○ ○ ○ ○ ○ ○ ○ ○ ○ ○ ○ ○ ○ ○ ○ ○ ○ ○ ○
Study Material
9. APPROPRIATE J<Í „¬^ÎO…Õ Z䛽ø=ª~¡°Á =zó# 17. POQ®Á JH›Æ~¡=¶…Õ Z_È=° #°Oz XH›\ ÷, 䛽_ #°Oz
JH›Æ~¡"Í°k? XH›\ ÷ Hù\÷“"͋¬¶ë =À‹ë z=~¡Qê q°y…è JH›Æ~Œ° Zxß?
1) A 2) P 3) R 4) E 1) 0 2) 1 3) 2 4) K³„¬æ…è=ò
10. ATTENDENCE J<Í „¬^ÎO…Õ XH›øªi =¶ã`Ç"Í° Zxß 18. POQ®Á JH›~
Æ =
¡ ¶…Õ 䛽_ #°O_ 9= JH›~
Æ O
¡ #°Oz Z_È=°ä›½
JH›Æ~Œ° =KŒó~ò? 4= JH›Æ~¡"Í°k?
1) 0 2) 1 3) 2 4) 3 1) M 2) N 3) R 4) V

11. D #°Oz 12= JH›Æ~¡"Í°k? 19. POQ®Á JH›Æ~¡=¶…Õ 䛽_ =°^Î¼ JH›Æ~¡O #°Oz Z_È=°
1) L 2) N 3) O 4) P "³á„¬ô䛽 5= JH›Æ~¡"Í°k?
12. D ãH÷Ok "Œ\÷…Õ U JH›Æ~Œxß `ǁÁãH÷O^ΰ° K͋²# 1) I 2) J 3) R 4) S

PHê~¡O…Õ =¶~¡°æ LO_È^ΰ. 20. POQ®Á JH›Æ~¡=¶…Õ Z_È=° =°^Î¼ JH›Æ~¡O #°Oz Z_È=°
1) D 2) J 3) P 4) Y "³á„¬ô䛽 4= JH›Æ~¡"Í°k?
13. ãH÷Ok "Œ\÷…Õ ‡~¡ÅÞ q…Õ=°=ò K³Ok# =¶ö~ JH›~
Æ "
¡ °Í k? 1) L 2) J 3) K 4) I

Y
1) I 2) M 3) O 4) S 21. pgkembcdiyt. WO^ΰ…Õ JKÇ°ó =°^Î¼ JH›Æ~Œ…ÿxß?

M
14. "³ò^Î\ ÷ 5 JH›Æ~Œ°, z=i 16 JH›Æ~Œ° `ùyOz# 1) 6 2) 4 3) 5 4) 9

„²=°à@ q°ye# JH›Æ~Œ…Õ =°^Î¼ JH›Æ~¡"Í°k? 22. ãH÷Ok "Œ\÷…Õ JKÇ°ó…èx „¬^Î"Í°k?

E
1) G 2) H 3) I 4) J 1) SKY 2) NOT 3) BUY 4) END

Z_È=° "³á„¬ô# "³ò^Î\ ÷ 9 JH›Æ~Œ°, 䛽_"³á„¬ô# z=i 8

D
WO^ΰ…Õ JKÇ°ó…ÿxß?

9
15. 23. AZBYCXDWEVFUGT.

59
JH›Æ~Œ° `ùyOz# „²=°à@ q°ye# JH›Æ~¡=¶…Õ =°^Î¼ 1) 0 2) 2 3) 3 4) 5

CA
95
JH›Æ~¡"Í°k? 24. aeikmgopt …Õ Zxß JKÇ°ó°<Œß~ò?

50
1) L 2) M 3) N 4) O 1) 2 2) 3 3) 4 4) 5

Z_È=° "³„
á ô¬ # "³ò^Î\ ÷ 10 JH›~Æ Œ#°, 䛽_"„á³ ô¬ # z=i 6 WO^ΰ…Õ 䛽 J@°W@°Qê
98
A
16. 25. pgmbttmgbkpmgm g m

JH›~
Æ Œ#° `ùyOz# JH›~ Æ =
¡ ¶…Õ Zxß JH›~ Æ Œ°O\ì~ò. Zxߪ~¡°Á =zóOk?
:8
E

1) 26 2) 16 3) 20 4) 10 1) 0 2) 1 3) 3 4) 2
h

1234567890123456789012
12345678901234567890123456789012123456789012345678901234567890121234567890123456789012345678901212345678901234567890123
.P

1234567890123456789012 12345678901234567890123
12345678901234567890123456789012123456789012345678901234567890121234567890123456789012345678901212345678901234567890123
12345678901234567890123
q=~¡}ì`ÇàH› ["Œ|°°
IC

1234567890123456789012
12345678901234567890123456789012123456789012345678901234567890121234567890123456789012345678901212345678901234567890123
1234567890123456789012 12345678901234567890123
12345678901234567890123
12345678901234567890123456789012123456789012345678901234567890121234567890123456789012345678901212345678901234567890123
1234567890123456789012 PRACTICE TEST : 12345678901234567890123
12345678901234567890123456789012123456789012345678901234567890121234567890123456789012345678901212345678901234567890123
AD

1234567890123456789012
12345678901234567890123456789012123456789012345678901234567890121234567890123456789012345678901212345678901234567890123
1234567890123456789012 12345678901234567890123
12345678901234567890123
12345678901234567890123456789012123456789012345678901234567890121234567890123456789012345678901212345678901234567890123
AB
O

1. (3) B, C, D, F, G, H, J, K, L, M, N, P, Q, R, S, T. 14. (2) F, G, H, I, J =°^Î¼ JH›Æ~¡O H.


ER
CH

Hê=‹²# JH›Æ~¡O R. 15. (3) J, K, L, M, N, =°^Î¼ JH›Æ~¡O N.


O, P, Q, R,
YD

2. (1) 14= JH›Æ~¡O N, 15= JH›Æ~¡O 'O', 20= JH›Æ~¡O T, 16. (4) K, L, M, N, O, P, Q, R, S, T. 10 JH›Æ~Œ°O\ì~ò.

g\÷`Ë U~¡æ_Í „¬^ÎO NOT. Hù\÷“"͋¬¶ë ‡é`Í XH›ø JH›Æ~¡O 䛀_È q°Q®^ΰ.
,H

17. (1)
R, T, V, W, X, Y, Z. =°^Î¼ JH›Æ~¡O W.
T

3. (2) (2) Z #°O_ 9= JH›Æ~¡O R. nxH÷ Z_È=°"³á„¬ô 4= JH›Æ~¡O


AM

18.

(1) "³ò`Ç=
ë ò 'e' ° 6. K `Ç~ŒÞ`Ç =KÍó 'e' ° =¶ã`Ç=ò 3.
H

4.
N.
M

5. (4) "³ò^Î\Qê =KÍó „¬^ÎO : CNORTUY, z=~¡Qê =KÍó 19. 䛽_ =°^¼Î JH›~Æ O¡ N I J K L M N . N #°Oz Z_È=°"³„
(1) á ô¬
AM
IG

„¬^ÎO : YUTRONC. 5= JH›Æ~¡O I.


‹¬i JH›Æ~Œ° : B, D, F, H, J, L, N, P, R, T, V, X, Z.
KH

(3)
20. (4) M, N ° =°^Î¼ JH›Æ~Œ°. Z_È=° =°^Î¼ JH›Æ~¡O M.
6.

Hê=‹²# JH›Æ~¡O N.
R

I J K L M . M #°Oz Z_È=°"³á„¬ô 4= JH›Æ~¡O I.


7. (3) ƒè‹² JH›Æ~Œ° : A, C, E, G, I, K, M, O, Q, S, U, W, 21. (2) e =°i†Çò i  =°^Î¼ 4 JH›Æ~Œ° H›=ô.
Y. Hê=‹²# JH›Æ~¡O M.
22. (1) A, E, I, O =°i†Çò U J<Íq 5 JKÇ°ó°. SKY „¬^O Î …Õ
8. (3) 'D' JH›Æ~¡O ECONOMICS J<Í „¬^ÎO…Õ …è^ΰ.
JKÇ°ó° …è=ô.
9. (2) P XH›ø>è J`ǼkH›OQê 3 ª~¡°Á =zóOk.
23. (3) A, E =°i†Çò U JKÇ°ó° H›=ô.
10. (4) A, D =°i†Çò C ° =¶ã`Ç"Í° XH›øªi =KŒó~ò.
24. (3) a, e, i =°i†Çò o JKÇ°ó° H›=ô.
11. (3) 'D' 4= JH›Æ~¡O. 12= JH›Æ~¡O L. D #°O_ 4= JH›Æ~¡O
25. (2) "³ò`ÇëOQê 'g' JH›Æ~¡O 3 ª~¡°Á, g 䛽 J@° (䛽_"³á„¬ô)Qê
« L #°O_ 4= JH›Æ~¡O « O
m 2 ª~¡°Á, g 䛽 W@° (Z_È=°"³á„¬ô)Qê m 2 ª~¡°Á
12. (1) JH›Æ~Œxß `ǁÁãH÷O^ΰ K͋²##° PHê~¡=°O^ΰ =¶~¡°æ
<ùO^Îx JH›Æ~Œ° : C, D, E, H, I, K, O, X. =zóOk. g 䛽 J@°Qêx …èH› W@°Qêx m 3 ª~¡°Á
13. (4) ‡~¡ÅÞ q…Õ=°=ò<ùOk##° PHê~¡=°O^ΰ =¶~¡°æ =zóOk. S`Í g 䛽 J@° =°i†Çò W@° m XH›øªö~
<ùO^Îx JH›Æ~Œ° : A, H, I, M, O, T, U, V, W, X. =zóOk.

REASONING ABILITY (VERBAL) ○ ○ ○ ○ ○ ○ ○ ○ ○


64
○ ○ ○ ○ ○ ○ ○ ○ ○ ○ ○ ○ ○ ○ ○ ○ ○ ○ ○
Study Material
123456789012345678901234567890121234567890123456789012345678901212
12345678901234567890123456789012123456789012345678901234567890121234567890123456789012345678901212345678901234567890123
1234567890123456789012345678901212345678901234567890123456789012123456789012345678901234567890121234567890123
1234567890123456789012345678901212345678901234567890123456789012123456789012345678901234567890121234
12345678901234567890123456789012123456789012345678901234567890121234567890123456789012345678901212345678901234567890123456789012123
123456789012345678901234567890121234567890123456789012345678901212345678901234567890123
12345678901234567890123456789012123456789012345678901234567890121234567890123456789012345678901212345678901234567890123
1234567890123456789012345678901212345678901234567890123456789012123456789012345678901234567890121234567890123
1234567890123456789012345678901212345678901234567890123456789012123456789012345678901234567890121234
123456789012345678901234567890121234567890123456789012345678901212
12345678901234567890123456789012123456789012345678901234567890121234567890123456789012345678901212345678901234567890123456789012123
123456789012345678901234567890121234567890123456789012345678901212345678901234567890123
12345678901234567890123456789012123456789012345678901234567890121234567890123456789012345678901212345678901234567890123
1234567890123456789012345678901212345678901234567890123456789012123456789012345678901234567890121234567890123
1234567890123456789012345678901212345678901234567890123456789012123456789012345678901234567890121234
12345678901234567890123456789012123456789012345678901234567890121234567890123456789012345678901212345678901234567890123456789012123
123456789012345678901234567890121234567890123456789012345678901212345678901234567890123
123456789012345678901234567890121234567890123456789012345678901212
PREVIOUS BITS
12345678901234567890123456789012123456789012345678901234567890121234567890123456789012345678901212345678901234567890123
1234567890123456789012345678901212345678901234567890123456789012123456789012345678901234567890121234567890123
1234567890123456789012345678901212345678901234567890123456789012123456789012345678901234567890121234
12345678901234567890123456789012123456789012345678901234567890121234567890123456789012345678901212345678901234567890123456789012123
123456789012345678901234567890121234567890123456789012345678901212345678901234567890123
123456789012345678901234567890121234567890123456789012345678901212
12345678901234567890123456789012123456789012345678901234567890121234567890123456789012345678901212345678901234567890123
1234567890123456789012345678901212345678901234567890123456789012123456789012345678901234567890121234567890123
1234567890123456789012345678901212345678901234567890123456789012123456789012345678901234567890121234
12345678901234567890123456789012123456789012345678901234567890121234567890123456789012345678901212345678901234567890123456789012123
123456789012345678901234567890121234567890123456789012345678901212345678901234567890123
12345678901234567890123456789012123456789012345678901234567890121234567890123456789012345678901212345678901234567890123
1234567890123456789012345678901212345678901234567890123456789012123456789012345678901234567890121234567890123
1234567890123456789012345678901212345678901234567890123456789012123456789012345678901234567890121234
12345678901234567890123456789012123456789012345678901234567890121234567890123456789012345678901212345678901234567890123456789012123
123456789012345678901234567890121234567890123456789012345678901212345678901234567890123
123456789012345678901234567890121234567890123456789012345678901212
12345678901234567890123456789012123456789012345678901234567890121234567890123456789012345678901212345678901234567890123456789012123
123456789012345678901234567890121234567890123456789012345678901212345678901234567890123
12345678901234567890123456789012123456789012345678901234567890121234567890123456789012345678901212345678901234567890123
1234567890123456789012345678901212345678901234567890123456789012123456789012345678901234567890121234567890123
1234567890123456789012345678901212345678901234567890123456789012123456789012345678901234567890121234

1. WOwÁ+¬µ JH›Æ~¡=¶…Õx "³ò^Î\ ÷, ï~O_È= JH›Æ~Œ°  13. JH›Æ~¡ ãH›=°O…Õ I H÷ Z_È=° ㄬH›ø# L#ß <Œ¾= JH›Æ~Œx,
=ü_È=, <Œ¾= JH›Æ~Œ°  S^Î=, P~¡= JH›Æ~Œ°, 䛽_ ㄬH›ø# 16= ªÖ#O…Õ L#ß JH›Æ~¡O Uk?
Pq^O Î Qê U~¡æ_# JH›~
Æ =
¡ ¶…Õ Z_È=° #°O_ 14= JH›~
Æ O
¡ (_„¬î¼\© *ÿá~Ÿž - 2012)
=°i†Çò 䛽_ #°O_ 6= JH›Æ~¡O Uk? (S.I.-2012)
1) U 2) S 3) T 4) V
1) H,U 2) L,V 3) M,V 4) K,U
14. POQ® Á JH› Æ ~ ¡ = ò…Õ Z_È = ° #°O_ 10= JH› Æ ~ ŒxH÷
2. ãH÷O^Î ~òzó# „¬^Î=ò#° Jq x„¦¬°O@°=ô…Õ U q^ÎOQê
J~ò`Í =ªë†³¶, P =~¡°‹¬…Õ P=°~¡°ó=ò? (S.I.-2012) Z_È=°"³á„¬ô# L#ß 7= JH›Æ~¡O Uk? (S.I.-2011)

1) Rumbustious 2) Rumanian 1) A 2) C 3) B 4) E
3) Rumour 4) Ruminate 15. CREDENTIALS „¬ ^ Î O …Õ XH› \ ÷ =°i†Ç ò P~¡ °
JH›~Æ = ¡ ò° `Œ~¡°=¶~¡†¶ Ç ¼~ò. J^Íq^OÎ Qê ~
ï O_È° =°i†Çò

Y
5) Rumple

1) 2,1,4,3,5 2) 2,4,3,1,5
7 JH›Æ~Œ° J^Í ãH›=°=ò…Õ q°ye#q 䛀_¨ J~ò`Í

M
3) 2,1,3,4,5 4) 2,4,1,3,5
䛽_ #°O_ 8= JH›Æ~¡O Uk? (S.I.-2011)
ㄬ‰×߁°(3-5):ãH÷O^Πㄬ‰×߁…Õ <Œ°Q®° „¬^¥° W=Þ|_¨Û~ò.

E
"Œ\÷x _H›Æ#s ãH›=°O…Õ J=°ió#„¬ô_È° Z_È=° #°O_ 1) A 2) T 3) D 4) C
16. RACTOR J<Í „¬^ÎO…Õx JH›Æ~Œ° ‹¬ï~á# ãH›=°O……Õ

D
=ü_È= „¬^ÎO H›#°Qù#O_. (H›=ü¼xöH+¬<£ S.I.-2012)

9
59
3. 1) Spine 2) Spinal ~Œ‹²# XH› Hê†Ç°Q®¶~¡ À„~¡° =‹¬°ëOk. J~ò`Í ^¥x z=i
CA
95
3) Spinner 4) Spinet JH›Æ~¡O Uk? (S.I.-2011)

4.

50
1) Cruise 2) Crumb
1) A 2) O 3) C 4) R 5) T
98
A
3) Crupper 4) Crude
17. ‘CORRESPONDENCE’ „¬^ÎO…Õx "³ò^Î\ ÷, ï~O_È°
5.
:8
1) Intense 2) Intellect
JH›~ Æ Œ° `Œ~¡°=¶~¡° K͆°Ç |_<Œ~ò. J^Íq^O Î Qê =ü_È°,
E

3) Intend 4) Intelligent
h

ㄬ‰×߁°(6-12) :ãH÷O^Î Wzó# „¬^¥#° _H›Æ#s ãH›=°O…Õ J=°ió <Œ°Q®°, S^ΰ =°i†Çò P~¡° etc.., =¶~¡°æ K͆°Ç Qê U~¡æ_#
.P
IC

Jq U =~¡°‹¬…Õ =ªë†³¶ ‹¬ï~á~ò# ãH›=¶xß ‹¬¶zOKÇO_ „¬^O Î …Õ 䛽_ #°O_ =KÍó 10= JH›~Æ O
¡ Uk? (S.I.-2011)
AD

(H›=ü¼xöH+¬<£ S.I.-2012) 1) O 2) P 3) R 4) N 5) S
AB
O

6. 1. Repoint 2. Reptile 18. ‘BEHIND’ J#° „¬ ^ Î O #°O_ JH› Æ ~ ¡ ãH› = °=ò#°


ER
CH

3. Repent 4. Repute
=¶~¡ ó ä› ½ O_¨ XH› JH› Æ ~ ¡ = ò#° XH› ø ªi =¶ã`Ç " Í °
YD

1) 3,1,2,4 2) 3,2,1,4 3) 1,4,3,2 4) 4,1,3,2


L„¬†³¶yOz Zxß ‹¬Þ`ÇOã`Ç"³°Ø# „¬^¥#° ~¡¶‡ÚOkOKÇ
7. 1. Select 2. Seldom
=KÇ°ó#°
,H

(H›=ü¼xöH+¬<£ S.I.-2011)
3. Send 4. Seller
T

1) 3 2) 4 3) 2 4) 1
AM

1) 1,2,4,3 2) 2,1,3,4 3) 2,1,4,3 4) 2,4,1,3

19. =¶i L#ß JH›Æ~¡=ò#° XH› J~¡Ö=O`Ç"³°Ø# „¬^Î=òQê


H

8. 1.Page 2. Pagen
M

3. Palisade 4. Pageant
䛀~¡°ó=ò. P q^Î=òQê uiy 䛀ió# „¬^Î=ò…Õ ''z=i
AM
IG

1) 1,4,2,3 2) 2,4,1,3 3) 1,4,3,2 4) 2,1,4,3


JH›Æ~¡=òÑÑ Qê nxx ‹¬OöHuOKÇ=KÇ°ó#°? 'LOPIEC'
KH

9. 1. Awadesh 2. Avadhesh
(H›=ü¼xöH+¬<£ S.I.-2011)
R

3. Avdesh 4. Awdhesh

1) 2,3,1,4 2) 1,3,2,4 3) 2,3,4,1 4) 1,2,3,4 1) 2) P


L 3) O 4) E
10. 1. Mother 2. Monitor 20. ‘ESDO’ J#° JH›Æ~¡=ò…Õ XHùøH›ø JH›Æ~¡=ò#° XH›øªi
3. Matter 4. Master
=¶ã`Ç " Í ° L„¬ † ³ ¶ yOz, Zxß POQ® Á „¬ ^ Î = ò#°
1) 4,3,2,1 2) 4,3,1,2 3) 4,1,3,2 4) 4,1,2,3

11. 1. Dialogue 2. Diabolic


~¡¶‡ÚOkOKÇ=KÇ°ó? (H›=ü¼xöH+¬<£ S.I.-2011)

3. Diagonal 4. Dialect
1) XH›\ ÷ 2) Ug°…è^ΰ 3) ï~O_È° 4) =ü_È°
1) 2,1,3,4 2) 2,3,1,4 3) 2,1,4,3 4) 2,3,4,1 21. ‘CARPENTER’ J#° „¬^Î=ò…Õx JH›Æ~¡=ò`Ë U XH›
12. 1. Intrinsic 2. Intrude 3. Intricate
„¬^Î=ò `džǶ~¡° K͆Çò@䛽 g°…èxq?
4.Introvert 5. Intrigue 6. Introduce
(H›=ü¼xöH+¬<£ S.I.-2011)
(_³á„¬î¼\© *ÿá~Ÿž-2012)
1) CARPET 2) NECTAR
1) 3,1,5,4,6,2 2) 3,5,1,6,4,2

3) 5,1,3,2,4,6 4) 3,5,6,4,1,2
3) PAINTER 4) REPENT

REASONING ABILITY (VERBAL) ○ ○ ○ ○ ○ ○ ○ ○ ○


65
○ ○ ○ ○ ○ ○ ○ ○ ○ ○ ○ ○ ○ ○ ○ ○ ○ ○ ○
Study Material
22. L,M,N,O,P,Q,R,S, =°i†Çò T JH›Æ~¡=ò ãH›=°=ò䛽 33. EPTS =¶@ #°O_ J~¡Ö=O`Ç"³°Ø# Zxß WOwÁ+¹ =¶@#°
|^ΰ°Qê `ùq°àk „¬î~ŒâOHꁰ 1 #°O_ 9 =~¡ä½› ãH›=°=ò (P`Ë ã‡~¡OaOz) ~¡¶‡ÚOkOKÇ=KÇ°ó. EPTS…Õx
…è佛 O_¨ LOKÇ=…ÿ#°. P H÷ 4 x~¡~â òOKÇ=…ÿ#°, P =°i†Çò JH›Æ~Œ#° XH›ø ^¥xx XHùø =¶@…Õ XH›ªi "Œ_¨e?
T  =°^Î¼ ƒè^Î=ò 5, N =°i†Çò T  =°^Î¼ ƒè^Î=ò 3 (Wardens Grade-I & II - 2012)

‘N’ 䛽 x~¡â~òOz# „¬î~ŒâOH›=ò Uk? (S.I.-2011) 1)XH›\ ÷ 2) ï~O_È° 3) =ü_È° 4) <Œ°Q®°
1) 4 2) 5 3) 6 4) 7 34. ãH÷Ok =¶@#° _H›Æ#s „¬^Μu…Õ =¶ió#KË =KÍó
‹¬¶KÇ# (23-26) : q^Íj „¬^= Î ò ‘HIPPNOWADIASM’ †³òH›ø =°^Î¼=¶@ Mobile, Merit, Minute, Market,
"³ò^Î\ ÷, <Œ°Q®° JH›Æ~Œ° ãu„²æ ã"Œ†Ç°|_#q. `Ç~¡°"Œ`Ç Master (Wardens Grade-I & II - 2012)

S^ΰ JH›Æ~¡=ò° ãH›=°=ò `DŽ¬æ䛽O_¨ ã"Œ†Ç°|_#q 1) Minute 2)


3) Master 4) Mobile Merit

=°i†Çò q°ye# JH›Æ~¡=ò° =°m¤ ãu„²æ ã"Œ†Ç°|_`Í 35. ‘ORGANISATION’ =¶@ #°O_ H÷Ok "Œ\÷…Õ U
(S.I.-2008) =¶@#° ~¡¶‡ÚOkOKDžè=ò?
23. ‘W’H÷ Z_È=° "³á„¬ô Zxß JKÇ°ó° LO_È°#°? (Wardens Grade-I & II - 2012)

Y
1) 2 2) 3 3) 1 4) 4 1) GAIN 2) STATION

M
24. ï~O_È° ㄬH›øㄬH›ø#°#ß JKÇ°óä›½ =òO^ΰ#ß JH›Æ~¡O Uk? 3) 4) GRANT
NATION

1) N 2) S 3) W 4) D 36. ‘CONTEMPTIBLE’ =¶@…Õx JH›~


Æ Œ`Ë U~¡æ_Í =¶@

E
25. 2 JKÇ°ó =°^Î¼ U JH›Æ~¡=ò LO_È°#°? (Medical officers(UNANI) - 2013)

1) P 2) H 3) W 4) N 1) EMPTY 2) TEMPLE

D
9
59
26. ‹¬iQê¾ =°^Î¼…Õ U JH›Æ~¡=ò LO_È°#°? 3) TEMPTED 4) LEAP
CA
95
1) H 2) P 3) J 4) W 37. D ãH÷O^Î Hùxß JH›Æ~Œ° =°i†Çò "Œ\÷H÷ Hùxß JOïH°

50
27. D ãH÷Ok „¬^¥#° _H›Æ#s ãH›=°=ò…Õ J=°ió# U öH\ì~òOKÇ|_¨Û~ò. J~ò`Í P JH›Æ~Œ#° ‹¬i†³Ø°#
ãH›=°=ò…Õ =KÇ°ó#°? ãH›=°O…Õ J=°ió# XH› J~¡Ö=O`Ç"³°Ø# „¬^Î=ò =KÇ°ó#°.
98
A
(S.I.-2008)

P J~¡ œ = ò †³ ò H› ø ãH› = °=ò#° ãH÷ O k W=Þ|_ #


:8
A. Select B. Seldom
E

C. Send D. Selfish E.Seller ㄬ`Œ¼=¶ß†Ç¶ #°O_ H›#°Qù#O_? (SSC-2014)


h
.P

1) B,A,E,C,D 2) B,A,D,E,C
IC

N A E X I M O T N A I

3) B,E,D,A,C 4) A,B,D,E,C
AD

9 2 3 4 5 6 7 8 9 2 5

28. ãH÷Ok POQ®Á=¶@#° _H›Æ#s =~¡°‹¬…Õ ̄_`Í PYiQê 1) 3,6,2,4,2,9,5,7,5,9,8


AB
O

=KÍó=¶@ (Aso / Sr.Entomologist - 2012) 2) 3,4,2,2,6,5,9,7,8,9,5


ER
CH

1) Expound 2) Exposure 3) 3,4,2,6,5,9,2,8,5,7,9


YD

3) Expulsion 4) Expose 4) 3,4,6,2,9,5,8,5,2,9,7


29. ‘MASQUERADE’ =¶@…Õx JH›~ Æ Œ`Ë H÷Ok =¶@…ÕÁ 38. POQ®Á x„¦¬°O@°=ô ㄬHê~¡O D ãH÷O^Î W=Þ|_# „¬^¥°
,H

^Íxx ~¡¶‡ÚOkOKDžèO. U =~¡°‹¬…Õ H›x„²ªë~ò?


T

(A.C.F-2012)
AM

1) SQUARE 2) MOSQUE 1) Caricature 2) Cardinal


H
M

3) MADE 4) MARE 3) Carnivore 4) Cartoon


AM
IG

30. H÷Ok =¶@eß _H›Æ#s ㄬHê~¡O J=°ió`Í, =°^Î¼…Õ LO_Í 5) Category

=¶@ Uk?
KH

(A.C.F-2012) 1) 4,5,1,3,2 2) 1,2,3,4,5

1) Gunny 2) Gunnery
R

3) 2,1,3,5,4 4) 2,1,3,4,5

3) Gunmetal 4) Gunpowder 39. D ãH÷O^Î W=Þ|_# „¬^¥…Õ Uk


31. H÷Ok =¶@eß _H›Æ#s ㄬHê~¡O J=°ió`Í, PYiQê LO_Í ‘THERMODYNAMICS’ J#° „¬^ÎO…Õx JH›Æ~Œ

=¶@ Uk? (A.C.F-2012) #°O_ U~¡æ_È#k? (SSC-2014)

1) DREARY 2) DREAM 1) MOTHER 2) MODERN


3) DREAD 4) DREDGE 3) DYNAMO 4) MATHEMATICS
32. NRAE =¶@ #°O_ J~¡Ö=O`Ç"³°Ø#xß WOwÁ+¬µ =¶@#° 40. ãH÷O^Î W=Þ|_# „¬^ÎO…Õx JH›Æ~Œ#° XH› J~¡Ö=O`Ç"³°Ø#
('E' `Ë ã‡~¡OaOz) ~¡¶‡ÚOkOKÇ=KÇ°ó. NRAE …Õx „¬^ÎO =KÍó q^ÎOQê uiy J=°ió`Í, ^Î`Çë „¬^ÎO…Õx ㄬu
JH›~Æ Œ#° XHùøH›ø ^¥xx XH›ø =¶@…Õ XH›øªi "Œ_¨e? JH›Æ~ŒxH÷ ~ò=Þ|_# JOïH †³òH›ø #¶`Ç# ãH›=°O.
(Wardens Grade-I & II - 2012) H N R C A B

1) XH›ø=¶@ 2) ï~O_È° =¶@° 1 2 3 4 5 6 (A.P. Const. Prelims-2016)

3) =ü_È° =¶@° 4) ̄áq UgHê=ô 1) 653412 2) 356412 3) 415623 4) 635241

REASONING ABILITY (VERBAL) ○ ○ ○ ○ ○ ○ ○ ○ ○


66
○ ○ ○ ○ ○ ○ ○ ○ ○ ○ ○ ○ ○ ○ ○ ○ ○ ○ ○
Study Material
12345678901234567890123456789012123456789012345678901234567890121234567890123456789012345678901212345678901234567890123
1234567890123456789012 123456789012345678901234

q=~¡}ì`ÇàH› ["Œ|°°
12345678901234567890123456789012123456789012345678901234567890121234567890123456789012345678901212345678901234567890123
1234567890123456789012 123456789012345678901234
12345678901234567890123456789012123456789012345678901234567890121234567890123456789012345678901212345678901234567890123
1234567890123456789012 123456789012345678901234
12345678901234567890123456789012123456789012345678901234567890121234567890123456789012345678901212345678901234567890123
1234567890123456789012 123456789012345678901234
1234567890123456789012
1234567890123456789012 PREVIOUS BITS :
12345678901234567890123456789012123456789012345678901234567890121234567890123456789012345678901212345678901234567890123
123456789012345678901234
12345678901234567890123456789012123456789012345678901234567890121234567890123456789012345678901212345678901234567890123
123456789012345678901234
12345678901234567890123456789012123456789012345678901234567890121234567890123456789012345678901212345678901234567890123
1234567890123456789012 123456789012345678901234

1. Sol : (3) ㄬ‰×ß…Õ K³„²æ# q^ÎOQê P…ì权ÿ\˜<Í =¶ió`Í 18. Sol : (3) ‹¬Þ`ÇOã`Ç „¬^¥° → BE, HIND
ãH÷Ok q^ÎOQê LO@°Ok. 19. Sol : (4) J~¡Ö=O`Ç"³°Ø# „¬^Î=ò → POLIC E
BADCFEHGJILKNMPORQTSVUXWZY 20. Sol : (3)

̄á =~¡°‹¬…Õ Z_È=° #°O_ 14= JH›Æ~¡O « M POQ®Á„¬^¥° → DOES, DOSE

䛽_ #°O_ 6 = JH›Æ~¡O « V 21. Sol : (3)

2. Sol : (1) „¬^¥#° _H›Æ#s P~¡Û~Ÿ…Õ J=°~¡óQê ãH÷Ok CARPET → $"31& / 5 &3
q^ÎOQê LO_È°#°.
NECTAR → $"3 1 &/5&3
Rumanian; Rumbustious; Ruminate; Rumour;

Y
Rumple.
REPENT → 7" 31&/5& 3

M
3. Sol : (4) 4. Sol : (2) 5. Sol : (3) PAINTER …Õx JH›Æ~Œ…Õx 'I' J#°#k Wzó# „¬^ÎO…Õ
6. Sol : (1) 7. Sol : (3) 8. Sol : (1) …è^ΰ. Hê=ô# Wk ^Î`Çë „¬^ÎO #°O_ U~¡æ_È^ΰ.

E
9. Sol : (1) 10. Sol : (1) 11. Sol : (4) 22. Sol : (3)

D
9
12.

59
Sol : (2)
- . / 0 1 2 3 4 5
13. P…ì权ÿ\˜ž…Õx H÷ 䛽_ ㄬH›ø# 16-4«12=
CA
95
Sol : (1) ‘I’
        
JH›Æ~¡O « “U”

50
(23-26) :
∴Hê=‹²# JH›Æ~¡O J=ô`Ç°Ok.
98
A
“U”
23. Sol : (1)
14.
:8
Sol : (2)
Wzó# „¬^Î=ò → HIPPNOWADIASM
E
h

ABCDEFGHIJKLMNOPQRSTUVWXYZ
Wzó# ^Î`ŒëO‰×O ㄬHê~¡O =¶ió ã"Œ†Ç°Qê
.P
IC

Z_È=° #°O_ 10= JH›Æ~ŒxH÷ Z_È=° "³á„¬ô L#ß 7= JH›Æ~¡O


AD

PPIHNOWADMSAI
« Z_È=° #°O_ (10-7)= JH›Æ~¡O
Z_È=° "³á„¬ô# J#° 2 JKÇ°ó° H›=ô.
AB
O

W I, O
« Z_È=° #°O_ 3= JH›Æ~¡O « C
24.
ER

Sol : (2)
CH

15. Sol : (1) Wzó# „¬^ÎO → CREDENTIALS


ㄬ‰×ß…Õ K³„²æ# q^ÎOQê =¶~¡°æ° D ãH÷Ok q^ÎOQê ï~O_È° ㄬH›ø ㄬH›ø# Q® JKÇ°ó°AI H÷ =òO^ΰ#ß JH›Æ~¡=ò
YD

K͆Ƕe. ‘S’.
,H

25.
T

Sol : (4)
AM

ï~O_È° JKÇ°ó =°^Î¼ JH›Æ~¡=ò → W


H
M

C R E D E N T I A L S

26. Sol : (4)


AM
IG

27. Sol : (2)


KH

Wzó# „¬^¥#° _H›Æ#s ãH›=°=ò…Õ J=°~¡óQê


R

ãHù`ÇëQê =KÇ°ó„¬^ÎO → N T I " L C R E D E S


B.Seldom


䛽_ #°O_ '8Ñ = JH›Æ~¡O 'A'.
A. Select
16. Sol : (5) Wzó# „¬^ÎO → RACTOR

JH›Æ~Œ#° ‹¬i†³Ø°# ãH›=°O…Õ J=°ió# =KÇ°ó „¬^ÎO D. Selfish

C A R R O 5 . ↓
17. Sol : (5) Wzó# „¬^ÎO → CORRESPONDENCE E. Seller

ㄬ=ß…Õ K³„²æ q^ÎOQê =¶~¡°æ° K͋²# U~¡æ_È° „¬^ÎO ↓


C. Send
O C R R 4 E O P D N N E E C

∴ ‹¬i†Ç°Q®° ãH›=°=ò → B, A, D, E, C
䛽_ #°O_ 10= JH›Æ~¡O → S

REASONING ABILITY (VERBAL) ○ ○ ○ ○ ○ ○ ○ ○ ○


67
○ ○ ○ ○ ○ ○ ○ ○ ○ ○ ○ ○ ○ ○ ○ ○ ○ ○ ○
Study Material
28. Sol : (3)
35. Sol : (2) GAIN → 03 ("/* 4"5*0/
Wzó# „¬^¥#° _H›Æ#s ãH›=°O…Õ J=°~¡óQê
→ 03( "/ *4" 5*0/
¯
4. Expose N AT I O N

G R A NT → 0 3("/ *4" 5 *0/


¯
2. Exposure

STATION J#° „¬^ÎO…Õ ‘T’ ï~O_È°ª~¡°Á =zóOk. Wzó#

¯
1. Expound
„¬^ÎO…Õ ‘T’ ï~O_È° ª~¡°Á …è^ΰ. Hê=ô# W^Í W=Þ|_#
3. Expulsion „¬^ÎO…Õx JH›Æ~Œ #°O_ U~¡æ_#k.
∴PYiQê =KÇ°ó=¶@ → Expulsion (3) 36. Sol : (2)
29. Sol : (2)
→ $0/ 5&.1 5*# -&
SQUARE → ." 426&3" %& TEMPLE

→ ."426&3 "%&
37. Sol : (3)

Y
MADE

J~¡Ö=O`Ç=°Q®° „¬^Î=ò
→ ." 426 &3 "%&

M
MARE

Wzó# „¬^ÎO…Õ ‘O' …è^ΰ. Hê=ô# ‘MOSQUE’ J#°#k E X A M I N A T I O N

E
Wzó# „¬^ÎO…Õx JH›Æ~Œ`Ë ~¡¶‡ÚOkOKDžè=ò. 3 4 2 6 5 9 2 8 5 7 9

30.

D
38.

9
Sol : (1,2) Sol : (4)

59
Wzó# „¬^¥#° _H›Æ#s ãH›=°O…Õ J=°~¡óQê Wzó# „¬^¥#° POQ®Á x„¦¬°O@°=ô ㄬHê~¡O J=°ió#
CA
95
¯
3. Gunmetal
2) Cardinal

50

¯ 98
2. Gunnery
A
:8
1) Caricature

¯
1. Gunny
E


h
.P
IC

4. Gunpowder 3) Carnivore
AD

31. Sol : (4)


Wzó# „¬^¥#° _H›Æ#s ãH›=°O…Õ J=°~¡óQê ↓
AB
O

4) Cartoon

¯
3. Dread
ER
CH


YD

¯
2. Dream
5) Cateogry

\ Hê=‹²# ãH›=°=ò - 2, 1, 3, 4, 5.
,H

¯
1. Dreary
T
AM

39. Sol : (4)


H

4. Dredge
M

∴ PYiQê =KÇ°ó=¶@ → DREDGE MOTHER ⇒ 5)&3.0 %:/".*$4


AM
IG

32.
⇒ 5) &3.0% : / ".*$4
Sol : (1)
MODERN
KH

E`Ë ã‡~¡Oƒ’=°†Í°¼ „¬^ÎO → EARN


⇒ 5)&3. 0%:/". *$4
R

XöH XH›ø „¬^ÎO U~¡æ_È°#°. DYNAMO

33. Sol : (2) Wzó# „¬^ÎO…Õ ï~O_È° T ° ï~O_È° A ° …è=ô. Hê=ô#


34. Sol : (2)
Wzó# „¬^ÎO…Õx JH›Æ~Œ #°O_ U~¡æ_È°#k.
Wzó# „¬^¥#° _H›Æ#s ãH›=°O…Õ J=°ió# =KÇ°ó ãH›=°=ò
‘MATHEMATICS’

¯
Market

40. Sol : (4)

¯
Master
^Î`Çë JH›Æ~Œ#° ‹¬i†Ç°Q®° ãH›=°O…Õ J=°ió# U~¡æ_È° „¬^ÎO

¯
Merit B R A N C H

¯ ¯ ¯ ¯ ¯ ¯
¯
Minute
6 3 5 2 4 1

Mobile ∴ =°^Î¼Qê =KÇ°ó=¶@ → Merit

REASONING ABILITY (VERBAL) ○ ○ ○ ○ ○ ○ ○ ○ ○


68
○ ○ ○ ○ ○ ○ ○ ○ ○ ○ ○ ○ ○ ○ ○ ○ ○ ○ ○
Study Material
5 ‡éeH› / ª^Î$‰×¼ „¬sH›Æ (Analogy)

=òMì¼O‰§°...
G Z<Œr J#Qê ‡éeH› / ª^Î$‰×¼=ò.
G D qƒìQ®=ò #°O_ XH› xií+"
“¬ °³ #Ø ‹¬O|O^OÎ H›ey L#ß „¬^¥ [`ǁ°, JH›~Æ Œ° (…è^¥) ‹¬OY¼ g°^Πㄬ‰ß×  J_È°Q®°`Œ~¡°.
G J~ò`Í W=Þ|_# ㄬ`Œ¼=¶ß†Ç¶ #°O_ U^³á`Í "³ò^Î\ ÷ ï~O_È° „¬^¥°, JH›Æ~Œ° (…è^¥) ‹¬OY¼ =°^Î¼ Q® ‹¬O|O^¥xH÷
‹¬iQê¾ ‹¬i‡é`Ç°O^Ë ^¥xx Z#°ßHË"Œe.

Y
G W=Þ|_# „¬^¥ JH›Æ~Œ° (…è^¥) ‹¬OY¼ [`ǁ° colon (…è^¥) x+¬æuë Q®°~¡° (:)`Ë ‹¬¶zOKÇ|_È`Œ~ò. A : B JO>è A

M
J#°#k B `Ë XH› xiœ+¬“"³°Ø# ‹¬O|O^ÎO H›ey L#ßk Jx J~¡ÖO.
G _È|°…˜ Hê<£ (::) J#°#k W=Þ|_# [`ǁ =°^Î¼ ''XöH q^Î=òQêÑÑ J#° ‹¬O|O^Î=ò#° `³°æ`Ç°Ok.

E
G ‡éeH› „¬sH›Æ° =ü_È° ~¡Hꁰ. Jq

D
9
1) „¬^¥ ‡éeH› (Word Analogy)

59
2) ‹¬OMì¼ ‡éeH› (Number Analogy)
CA
95
3) JH›Æ~Œ ‡éeH› „¬sH›Æ (Alphabetical Analogy)

50
+ J<Œrx ‹¬°ƒ’OQê J~¡ÖOK͋¬°Hù#°@䛽 Hùxß L^¥‚¬ì~¡}° :
98
A
+ D qƒìQ®O…Õ XH› xiœ+¬» ‹¬O|O^¥xß H›ey# ï~O_È° „¬^¥° W=Þ|_È`Œ~ò =°i†Çò "Í~ùH› „¬^ÎO W=Þ|_È°`Ç°Ok. "³ò^Î\ ÷
:8
[O@ „¬^¥ ‹¬O|O^Î P^¥~¡OQê Wzó# =°~ùH› „¬^ÎO#䛽 P„¬Â#° #°O_ Uk ‹¬i‡ée =ôO@°O^Ë ^¥xx ZOKÇ°HË"Œe.
E
h

+ D qƒìQ®O…Õ XöH ~¡H›"³°Ø# H›Æ}쁰 H›ey# =ü_È° „¬^¥° W=Þ|_È`Œ~ò. Wzó# <Œ°Q®° P„¬Â#° #°O_ J^Í ~¡H›"³°Ø#
.P
IC

^Î~¡àO H›ey# „¬^¥xß Q®°iëOKŒe.


AD

+ D qƒìQ®O…Õ Hùxß [O@ „¬^¥° …è^¥ JO`ÇH›<Œß Z䛽ø= „¬^¥° W=Þ|_È`Œ~ò. P „¬^¥ =°^Î¼Q® ‹¬O|O^¥xß
AB
O

J#°‹¬iOz Wzó# P„¬Â#°…Õ J…ìO\÷ ‹¬O|O^Î"Í° H›ey# ‹¬=¶^¥<Œxß Q®°iëOKŒe.


ER
CH

1. qq^Î ^͉§° Ð ~Œ[^¥#°° Ð H›ï~hž#° `³°„¬ô


YD

^͉×O ~Œ[^¥x H›ï~hž


‹¬=¶KŒ~¡O :
,H

W@b ~Ë"£° W@b †Çü~Ë


T

^͉×O ~Œ[^¥x H›ï~hž


AM

†Çò.†Ç°‹¹.Z "Œ+²OQ·@<£ _¨~Ÿ


H

WO_†Ç¶ #¶¼_bÁ ~¡¶‡~ò HùOa†Ç¶ ƒçQË@ ̄ªé


M

„¦¬¶<Œ JãHê ‹²_Í


AM

"³°H÷žHË "³°H÷žHË ‹²\ ÷ "³°H÷žHË Ì„ªé


IG

̄~¡¶ e=¶ #¶¼ ªé…˜ zb ‰§O\©†Ç¶QË ze†Ç°<£ ̄ªé


KH

q†Ç°`ŒßO ‚¬ð<ˆŸ° _ùOQ· =°…è+²†Ç¶ H҅ìO„¬î~Ÿ iOy¾\˜


R

[‡<£ \ÕHù¼ †³°<£ "³ò~ŒHË ~¡ƒì\˜ k~¡Ý"£°


䛀¼ƒì ‚¬ì"Œ# ̄ªé ªÏnJö~a†Ç¶ i†Ç¶^£ i†Ç¶…˜
L~¡°öQÞ =¶O>ÿq_†³¶ ̄ªé W~Œ<£ >ÿã̂ì~ò<£ i†Ç¶…˜
„¦²e„²<£ž =°h ̄ªé ãƒÿl…˜ ãƒÿle†Ç¶ i†Ç¶…˜
Jï~ûO\÷<Œ |¶¼#‹¹ Z~ò~Ÿž ̄ªé H›OƒÕ_†Ç¶ <ŒŌ¦<£ i†Ç¶…˜
㇦<£ž ‡i‹¹ †Çü~Ë <³^Î~ŒÁO_£ž J=°‹¬“~Ÿ _¨¼O †Çü~Ë
P¢‹Ô“†Ç¶ q†Ç°<Œß †Çü~Ë Q®†Ç¶<Œ *ìiû\º<£ Q®†Ç¶<Œ _¨~Ÿ
`³á"Œ<£ =¶ †³°OQ· *º †Çü~Ë DïHÞ_¨~Ÿ H÷Þ\Õ †Çò.Z‹¹._¨~Ÿ
[~¡àh ƒÿiÁ<£ †Çü~Ë L`Çë~¡Hùi†Ç¶ ㇐OQ·†Ç¶OQ· =<£
ƒÿeû†Ç°O ã|̋ž…˜ †Çü~Ë ^ÎH}
ƛ Hùi†Ç¶ ‹²†³¶…˜ =<£
̋æ~ò<£ =¶¼ã_™H± †Çü~Ë ‡é~¡°óQ®…˜ e‹¬Ä~Ÿ †Çü~Ë

REASONING ABILITY (VERBAL) ○ ○ ○ ○ ○ ○ ○ ○ ○


69
○ ○ ○ ○ ○ ○ ○ ○ ○ ○ ○ ○ ○ ○ ○ ○ ○ ○ ○
Study Material
^͉×O ~Œ[^¥x H›ï~hž öH~¡ˆ× ãu"ÍOã_ÈO
Dl„¹“ ïHá~Ë Dl„²“†Ç¶<£ ‡ÏO_£ Xiªž ƒ’°=<͉×Þ~Ÿ
^ÎH›Æ} ‹¬¶_¨<£ Eƒì ^ÎH›Æ}‹¬¶_¨<£ ‡ÏO_£ ‚²ì=¶KDž˜ã„¬^͉˜ ‹²=¶Á
@iø JOHê~Œ @iø b~Œ J~¡°}ìKDž˜ã„¬^͉˜ D@#Q®~Ÿ
_³<Œà~Ÿø Hù̄<£ ‚¬ðQ®<£ _³x‹¹ ãHù<³ L`Çë~¡ã„¬^͉˜ HËß
*ìOa†Ç¶ °‡Hê HêÞKÇ =°^Î¼ã„¬^͉˜ ƒÕ‡…˜
Wã*ì~ò…˜ *ÿ~¡¶‹¬…ÿO ̄ïH…˜ c‚¬ð~Ÿ ‡\ìß
Jbûi†Ç¶ Pbû~Ÿž Jbûi†Ç¶ n<Œ~Ÿ Q®°[~Œ`Ÿ QêOn#Q®~Ÿ
ea†Ç¶ ã\÷‡ée ea†Ç¶ n<Œ~Ÿ QË"Œ ‡<Œl
#g°a†Ç¶ q<˜H± #g°a†Ç¶ _¨~Ÿ ‹²H÷øO Qê¼OQ·@<£
JOQË…ì °"ŒO_¨ s_£ [‹¹“_£ HêÞO*ì ãu„¬ô~¡ JQ®~¡ë
†Çò.ïH O_È<£ ̋“iÁO_£ ‡ÏO_£ ~Œ[ªÖ<£ *ÿᄬî~Ÿ

Y
ãw‹¹ U^Š³<£ž ã_¨KŸ=¶ „¬tó=° ƒÿOQꅘ H›H›`Œë

M
=°_ÈQꋬø~Ÿ JO@<Œ<Œi"Ë Pi†Ç¶i JªžO _‹¹„¬î~Ÿ
<³ári†Ç¶ J|°*ì <³á~Œ =°}÷„¬î~Ÿ WO‡…˜

E
ªé=¶e†Ç¶ "³°Q®k+¬µ +²eÁOQ· <ŒQê…ìO_£ Hù‚²ì=¶
ïH<Œ¼ <³á<Ëa +²eÁOQ·

D
‚¬ì~Œ¼<Œ KÇO_™„¦¬°~Ÿ

9
59
"³<³A…ì Hê~¡H›‹¹ ƒçe=~Ÿ „¬O*샘 KÇO_™„¦¬°~Ÿ
CA
95
Hê"³¶~¡¶<£ †Ç¶=îO_Í ‹².Z„¦¹.Z ㄦ¬OH± "Í°„¦¬¶†Ç° +²…ìÁOQ·
HêOQË ã|[ûq…ÿÁ ‹².Z„¦¹.Z ㄦ¬OH±

50
‹²Þ@û~ŒÁO_£ ƒÿ~Ÿß ‹²Þ…˜ ㇦OH± 3. Hö Oã^‡
Î e`Ç ‡
㠐O`Œ° : ƒì+¬° Ð =¶\ìÁ_Í ‡ã O`Œä›½
98
A
~¡¶"ŒO_¨ H÷Qêe ~¡°"ŒO_¨ ㇦OH± ‹¬O|OkOz# [O@ „¬^¥° :
:8
ªÞl…ìO_£ |ƒì<Í e…ìOyx
E

㇐O`ÇO ƒì+¬
h

ƒÕ\˜ž"Œ<Œ Qêƒç~Ë<£ „¬î…ì


.P
IC

^Š¥†Ÿ°…ìO_£ ƒì¼OHêH± ƒì\˜ L`Çë~¡ã„¬^͉˜ ‚²ìOn


AD

|¶\ì<£ kŠO„¬î Q®°…˜ã@"£° POã^Îㄬ^͉˜ Ê `³OQê}ì `³°Q®°


AB
O

=°bœ=ô° =¶…ÿ =°eíq†Ç¶<£ ~¡¶‡~ò `Çq°ˆ×<Œ_È° `Çq°ˆ×O


NOH› HˁOƒÕ ãjOH› ~¡¶„¦²
ER
CH

‡H÷ªë<£ WªÁ=¶ƒì^£ ‡H÷ªë<£ ~¡¶„¦² QË"Œ HùOH›}©


YD

<͇…˜ Mì@àO_È° <͇e ~¡¶„¦² ‹²H÷øO <͇e


,H

|OQêÁ^͉˜ _¨Hê \ìHê ~Œ[ªÖ<£ ‚²ìOn


T

~¡‘¼ =¶ªéø ~¡¶|°…˜


AM

„¬O*샘 „¬O*ìa
K³á<Œ clOQ· †Çò"Œ<£
H

=°^Î¼ã„¬^͉˜ ‚²ìOn
M

W~ŒH± ƒìQêí^£ n<Œ~Ÿ


AM

X_ªž X_†Ç¶
IG

["³°ØHê H÷OQ·‹¬“<£ ["³°ØHê _¨~Ÿ


<ŒQê…ìO_£ WOwÁ+¹
KH

‹²OQ®„¬î~Ÿ ‹²OQ®„¬î~Ÿ ‹²\ ÷ ‹²OQ®„¬î~Ÿ _¨~Ÿ


P¢À‹“e†Ç¶ Hê<£ƒÿã~Œ P¢À‹“e†Ç¶<£ _¨~Ÿ q°*Õ~¡O q°*Õ
R

lOƒìƒèÞ ‚¬ì~Œö~ _¨~Ÿ (¢+¬“En) „¬tó=° ƒÿOQꅘ ƒÿOQêe


Hï <Œ_¨ |\ì“"Œ ïH<³_†Ç¶<£ _¨~Ÿ JªžO ƒÕ_Ë
2. ~Œ¢‘“° Ð ~Œ[^¥#°° =°‚¬ð~ŒR =°~Œi”
öH~¡ˆ× =°ˆ×†Ç¶O
~ŒRO ~Œ[^¥x
H›~Œß@H› H›#ß_ÈO
POã^Îㄬ^͉˜ (#=¼) J=°~Œ=u *ì~ŒšO_£ ‹¬O`Œe
`³OQê} ̂áì^Î~Œƒì^£
‚²ì=¶KDž˜ ㄬ^͉˜ _Ëãw
`Çq°ˆ×<Œ_È° K³<³ßå
H›~Œâ@H› ƒÿOQ®°ˆ×´~¡° =°}÷„¬î~Ÿ 䛽H÷
=°‚¬ð~ŒR =òO|~ò c‚¬ð~Ÿ "³°ØkŠe

REASONING ABILITY (VERBAL) ○ ○ ○ ○ ○ ○ ○ ○ ○


70
○ ○ ○ ○ ○ ○ ○ ○ ○ ○ ○ ○ ○ ○ ○ ○ ○ ○ ○
Study Material
4. ^͉§° Ð *ìf†Ç° z‚¬ð߁䛽 ‹¬O|OkOz# [O@ 6. „¬iH›~O¡ =°i†Çò Hù`ǁ䛽 ‹¬O|OkOz# [O@
„¬^¥°: „¬^¥°:
^͉×O z‚¬ìßO „¬iH›~¡O Hù=¶#O
WO_†Ç¶ =ü_È° `ǁ ‹²O‚¬ìO Pg°à@~Ÿ H›ï~O@°
_³<Œà~Ÿø ‹¬=òã^΄¬ô X_È°Û ã‹²æOQ· ã`Œ‹¬° |~¡°=ô
W~Œ<£ Q®°…ìa =¶<Ëg°@~Ÿ "Œ†Çò=ô „Ô_È#O
~¡‘¼ QË^°Î =° =~¡â Z°Q®° |O\÷ ã`Œ‹¬° ã^Î=¼~Œt
P¢À‹“e†Ç¶ H›OQê~¡¶ "˅˜“ g°@~Ÿ q^ΰ¼`Ÿ ‡Ú>ÿx†ǰ…˜ ("˅è“l)
[~¡àx Hê~Ÿß„¦¬Á=~Ÿ ƒì~Ëg°@~Ÿ (ƒì~¡q°u) „Ô_È#O
‡H÷ªë<£ ãH÷̋O\˜ (KÇOã^Î=OH›) F"£°g°@~Ÿ q^ΰ¼`Ÿ x~Ë^ÎO
̋æ~ò<£ =°i†Çò ‹²i†Ç¶ ãQ®^Îí Pb“g°@~¡° ƒ’¶ L„¬i`ǁO #°O_

Y
㇦<£ž ebÁ q=¶#O Z`Ç°ë Hù°KÇ°@
W@b `³ÁebÁ

M
^ŠÎ~Œàg°@~¡° L‘éâãQ®`Ç
[‡<£ KÍ=°Ou„¬î=ô ̋æ¢H˓g°@~Ÿ HêOu ^Î~Œà°

E
J"³°iHê |OQê~¡°H›_™Û ïH…Õsg°@~¡° L+¬âO

D
qªéøg°@~Ÿ ‹²ßQ®Ö`Ç

9
5. ~Œ‰×Á°Ðㄬ=¶}ìä›½ ‹¬O|OkOz# [O@ „¬^¥°:

59
~Œt ㄬ=¶}O ‹¬¶ø $öQl =°O^ÎO, "Œ¼‹¬O
CA
95
J†Ç°ªøO`Ç ã„¬"͉׼j`Ç Ì‚ìãh / g°@~Ÿ À‹ø° ‡Ú_È=ô

50
J†Ç°ªøO`Ç öHÆã`ÇO J†Ç°~ŸÌ‹“_£ ‹Ôæ_Ëg°@~¡° "Œ‚¬ì# "ÍQ®O
98
A
J†Ç°ªøO`Ç Ja"Œ‚¬ìO "³|~Ÿ Jx"³¶g°@~¡° Qêe"ÍQ®O
:8
‡Ú_È=ô g°@~¡°Á Q®_†Ç¶~¡O HêO
E

ï~~ò<£ öQ*˜ =~¡Â‡`ÇO


h

ª=°~¡Ö¼O "Œ\˜
.P
IC

ã^Î=¼~Œt H÷.ãQê ̂áìãQËg°@~Ÿ P¢~¡Ö`Ç


AD

ªOã^Î`Ç H÷.ãQê/g°3 ‹²ªéàãQꄦ¬ô ƒ’¶H›O„¬ fã=`Ç


`ǁ`Ç#¼`Ç #¶¼@<£ / g°@~¡°
AB
O

‹²æQËà=¶<Ëg°@~¡° ~¡H›ë‡é@°
HêO ̋H›#°° ̋áãHËg°@~Ÿ Qêe…Õx Pqi aQ®°=ô
ER
CH

q^ΰ¼`Ç°ë PO„²†Ç°~¡°Á F_Ëg°@~Ÿ "Œ‚¬ì#O ㄬ†Ç¶}÷Oz# ^ζ~¡O


YD

L‘éâãQ®`Ç _ãw
q^ΰ¼`Ÿ x~Ë^ÎO F"£° (ohm) ƒÕ…Õg°@~Ÿ q^ΰ¼^ΆǰªøO`Ç qH÷~¡}쁰
,H

‡Ú>ÿx†ǰ…˜ F…˜“ ‡Ú>ÿx†³¶g°@~Ÿ "˅è“l


T
AM

‰×H÷ë B…˜ ̄á~Ëg°@~Ÿ JkH› L‘éâãQ®`ǁ°


H

|O #¶¼@<£ "³°ØãHËg°@~¡° `Ç䛽ø= ^ζ~Œ°


M

‡Ï#—„¬ô#¼O ̂ì~Ÿ“û …ìH˓g°@~Ÿ ‡†³òH›ø qt+¬» ªOã^Î`Ç


AM
IG

L‘éâãQ®`Ç _ãw°
KH

„Ô_È#O ‡‹¬ø…˜ 7. =¼ä›½ë° Ð „¬xKÍÀ‹ ㄬ^͉§#° `³e†Ç°*è†Çò [O@


R

„¬x *º…˜ „¬^¥°:


„¦¬°#„¬i=¶}O b@~¡°Á (g°3)
"³á‰§¼O g°2. =¼H÷ë „¬xK͆Çò ㄬ^͉×O
HêOu fã=`Ç Hê¼O_…˜ L‡^Š¥¼†Çò_È° ‡~”¡‰§
H›O_ÈH›“q\© "³¶ (mho) #@°_È° ~¡OQ®‹¬ÖO
P"͉×O 䛽¶Oƒ˜° ã‡Ú̄¦‹¬~Ÿ q‰×Þq^¥¼†Ç°O
"ÍQ®O g°/̋H›#° zã`ÇHê~¡°_È° Qês
`ÇÞ~¡}O g°/̋H›#°2 _¨H›“~Ÿ ‚¬ð‹²æ@…˜
L„¬i`ǁ ªOã^Î`Ç H÷.ãQê/g°2.
ï~á`Ç° ‡ÚO
q^ΰ¼`Ÿ ªOã^Î`Ç PO„²†Ç°~Ÿ/g°@~Ÿ
J†Ç°ªøO`Ç öHÆã`Ç |O PO„²†Ç°~Ÿ/g°@~Ÿ "³°HêxH± Qê¼ö~r
ãÀ„~¡H›`ÇÞO ̂ìãh †Çò^Μg~¡°_È° ~¡}öHÆã`ÇO

REASONING ABILITY (VERBAL) ○ ○ ○ ○ ○ ○ ○ ○ ○


71
○ ○ ○ ○ ○ ○ ○ ○ ○ ○ ○ ○ ○ ○ ○ ○ ○ ○ ○
Study Material
=¼H÷ë „¬xK͆Çò ㄬ^͉×O 11. „¬x"Œ_È° Ð L`ÇæuëH÷ ‹¬O|OkOz# [O@„¬^¥° :
"Œ¼‡i ^ΰHê}O „¬x"Œ_È° L`Çæuë
"³~ò@~Ÿ ï~ª“ï~O\˜ =O@"Œ_È° P‚¬ð~¡O
=O@"Œ_È° =O@Q®k ï~á`Ç° „¬O@
̄á\˜ HêH±„\ ² ˜
H›Oªe Pƒ’~¡}쁰
„¬x"Œ_È° W°Á
H›q öQ†Ç°O
¢_³á=~Ÿ Hê¼a<£
H›Á~Ÿø P„¦Ô‹¬° ~¡KÇ~ò`Ç ~¡KÇ#° („¬ô‹¬ëH›O)
JŌá~Ÿ „²KŸ x~Œà`Ç Kǁ#zã`ÇO
…ì†Ç°~Ÿ HË~¡°“ ^Îsû"Œ_È° |@“° 䛽@°“@
‰§G"Í`Çë ㄬ†³¶Q®‰§ H›ª~ò =¶O‹¬O

Y
L‡^¥¼†Çò_È° q^μ
8. =°`Œ° Ð „¬qã`Ç ãQ®O^¥° Ð ^³á= öHÆã`Œ°ä›½ [_™û <Œ¼†Ç°O

M
‹¬O|OkOz# [O@ „¬^¥° : Piø>ÿH±“ x~Œà}O
=°`Ç=ò „¬qã`Ç ãQ®O^ÎO ^³á= öHÆã`Ç=ò

E
KÇ~¡àHê~¡°_È° K³„¬C°, |¶@°Á
‚²ìO^ζ ƒ’Q®=n¾`Ç Q®°_ =ã_ÈOy „¦¬ißKÇ~Ÿ

D
9
59
=ò‹²ÁO Y°~Œ<£ =°‹Ô^ΰ Z_@~Ÿ #¶¼‹¹À„„¬~Ÿ
CA
95
ãH÷‹²“†Ç°<£ ƒÿáa…˜ KÇió `Œ„Ô„¬x"Œ_È° QË_ȁ° …è^¥ W°Á
‹²H± Q®°~¡°ãQ®O^Š£ ªÌ‚샘 Q®°~¡°^¥Þ~Œ

50
‡ià [O_¨ J=ªÖ Jyß^Í"Œ†Ç°O 12. r=ô° Ð x"Œ‹¬O#䛽 ‹¬O|OkOz# [O@„¬^¥° :
98
A
†Çü^ΰ° `Ë~Œ ‹²#QêQ· rq x"Œ‹¬O
:8
‹²O‚¬ìO Q®$‚¬ì
E

9. J"Œ@Á䛽 ‹¬O|OkOz# ‡éeH›° :


h

„¬ä›Æ½° „¬O[~¡O (Q®¶_È°)


.P
IC

=¶#=ô° ‰§M사ði, =¶Oª‚¬ði =¶#=ô_È° W°Á


AD

„¬ôe, ‹²O‚¬ìO =¶Oª‚¬ð~¡°° K̈́¬ ZöHÞi†Ç°O


AB
O

P=ô, "Í°H› ‰§M사ð~¡°° Hˈ×ä ‡¦"£°


ER
CH

HËu ‰§M사ði Q®°ã~¡O Q®°ã~¡„¬ô‰§


䛽ä›ø =¶Oª‚¬ði, ‰§M사ði „¬‰×Ã=ô° ‡H›
YD

„²eÁ =¶Oª‚¬ði, ‰§M사ði `Í<³\ ©Q® `Í<³`Ç@°“ (J̄ás)


,H

XO>ÿ, U#°Q®° ‰§M사ð~¡°° Z°H› ƒçi†Ç°


T
AM

lOH› ‰§M사ði p=° p=°„¬ô@“


H

10. „¬x"Œ_È° =°i†Çò =°x=ò@Á#° H›e„²=ôOKÇ° „¬^¥° : ‡=ò ‡=ò„¬ô@“


M
AM

䛽H›ø ƒÕ#°
IG

„¬x"Œ_È° „¬x=ò@°“
KH

‡éb‹¬° …ìs”
13. k‰§ ‹¬O|O^Î ‡éeH›° :
R

L`Çë~¡O Ð ^ÎH÷Æ}O
ï~á`Ç° <ŒQ®e
`Ƕ~¡°æ Ð „¬_È=°~¡
~¡KÇ~ò`Ç H›=ò
D‰§#¼O Ð <³á~¡°u=ü
_¨H›“~¡° ̋“`Ǫéø„¹
PöQ߆ǰO Ð "Œ†Çò=¼k‰×
`Ë@=¶e ‡~¡
H›>ÿ“° Hù@°“"Œ_È° Qù_ÈÛe 14. XöH ~¡H›"³°Ø# H›Æ}ìä›½ ‹¬O|OkOz# ‡éeH›° :
=ã_ÈOy ~¡O„¬O ‹¬O`Ë+¬O Ð P#O^ÎO
̋áx䛽_È° `Ç°‡H© x[O Ð "Œ‹¬ë=O, †Ç°^¥~¡ÖO
Hêï~æO@~Ÿ ~¡O„¬O Hù# Ð z=~¡, JOKÇ°
^Îsû ‹¬¶k Q®\ ÷“k Ð ^Î$_È"³°Ø#k, |e+¬»"³°Ø#k
~¡KÇ~ò`Ç Ì„#°ß (H›=ò) q"ÍH›O Ð `³e"³á#
teæ Le Jq"ÍH÷ Ð |°kí‚Ôì#°_È°, `³eq…èx"Œ_È°
REASONING ABILITY (VERBAL) ○ ○ ○ ○ ○ ○ ○ ○ ○
72
○ ○ ○ ○ ○ ○ ○ ○ ○ ○ ○ ○ ○ ○ ○ ○ ○ ○ ○
Study Material
15. ãH©_ȁ° Ð P_Í ‹¬Ö…ìä›½ ‹¬O|OkOz# ‡éeH›° : 16. =òY¼"³°Ø# P†Ç¶° Ð Jq =ô#ß ã„¬^͉§° :
"Œbƒì…˜ Ð HË~¡°“ "³OH›>è‰×Þ~¡ªÞq° P†Ç°O Ð u~¡°=°, POã^Îㄬ^͉˜
ƒì¼_àO@<£ Ð HË~¡°“ ‹¬¶~¡¼ ^Í"Œ†Ç°O Ð HË}ì~Ÿø
ãH÷Hï \˜ Ð „²KŸ ("³°Ø^¥#O) [Q®<Œß^Š¥†Ç°O Ð „¬îi
„¦¬ô\˜ƒì…˜ Ð "³°Ø^¥#O Y[~¡‚¬ìÙ P†Ç°O Ð =°^Î¼ã„¬^͉˜
+¬\ ÷…˜ Ð HË~¡°“
J|¶ P†Ç°O Ð ~Œ[ªÖ<£
>ÿxߋ¹ Ð HË~¡°“
‚¬ðH© Ð "³°Ø^¥#O #@~Œ[ P†Ç°O Ð z^ÎO|~¡O
…ìOQ·[O„¹ Ð "³°Ø^¥#O ªé=°<Œ^Š£ P†Ç°O Ð Q®°[~Œ`Ÿ
Q˅˜æ Ð "³°Ø^¥#O eOQ®~Œ*˜ P†Ç°O Ð ƒ’°=<͉×Þ~Ÿ

„¬^¥ J<Œl

Y
TYPE - I :

M
MODEL - 1 ㄬ`ǼH›Æ ‡éeH›
ª^Î#: (1) †Çü~Ë Ð ã‡¦<£ž H›ï~hž
_¨~Ÿ Ð †Çò.Z‹¹.Z H›ï~hž

E
1. ^ŠÎ~Œàg°@~¡° : : : Q®_†Ç¶~¡=ò : ‹¬=°†Ç°O
1) L‘éâãQ®`Ç 2) L+¬âO 3) ‰×H÷ë 4) UnHê^ΰ \ìHê Ð |OQêÁ^͉˜ H›ï~hž

D
9
59
ª^Î#: (1) Q®_†Ç¶~¡O, ‹¬=°†Ç¶xß Z…ì KǶ„²‹¬°ëO^Ë, ^Î~Œà ∴ ‹¬=¶^¥#O '㇦<£žÑ : †Çò.Z‹¹.Z : |OQêÁ^͉˜

CA
95
g°@~¡° J^Í q^ÎOQê L‘éâãQ®`Ç#° `³e†Ç°*苬°ëOk. 2. JOQê~¡ 䛽_È° : ‰×x : ƒ’¶q° : :
1) ^Î$= #H›Æã`ÇO : ‹¬¶~¡°¼_È° : KÇOã^ΰ_È°

50
∴ ‹¬=¶^¥#O : L‘éâãQ®`Ç.
2) PHê‰×O : "Í°„¦¬¶° : =~¡ÂO 98
A
2. „¬ô‹¬ëH›O : À„r° : : „¬ô=ôÞ :
:8
1) HêO_ÈO 2) P䛽° 3) ö~䛽° 4) UnHê^ΰ 3) g°#O : "Í°+¬O : =$+¬ƒ’O
E

ª^Î#: (3) „¬ô‹¬ëH›O À„r#° Z…ì H›ey=ôO@°O^Ë, „¬ô=ôÞ 4) "Í~¡°‰×#Q® : c\˜~¡¶\˜ : K³~¡ä›½
h
.P

ö~䛽#° J^Íq^ÎOQê H›ey LO@°Ok. ª^Î#: (3) g°#O : "Í°+¬O : =$+¬ƒ’O.


IC
AD

∴ ‹¬=¶^¥#O : ö~䛽° ㄬ‰×ß…Õ Wzó#=hß ãQ®‚¬ð°


[O@ „¬^¥ ‡éeH›
AB
O

MODEL - 2 g°#O, "Í°+¬O, =$+¬ƒ’O° Jhß ~Œ‰×Á°.


q°ye# P„¬  #°…Õ Un W…ì Xö H ^ Î ~ Œàxß H› e y
ER
CH

1. P‹¬°„¬ãu : ~ËQ®°° : : H›ˆì‰§ :


†ÇòO_ȅè^ΰ.
YD

1) q^¥¼~¡°Ö° 2) =°#°+¬µ° 3) [O`Ç°=ô° 4) UnHê^ΰ


ª^Î#: (1) P‹¬°„¬ãu…Õ ~ËQ®°° =ôO\ì~¡°. MODEL - 4 ª~¡¶„¬¼ „¬^¥ J<Œl
,H
T

H›ˆì‰§…Õ q^¥¼~¡°Ö° =ôO\ì~¡°.


AM

1. ‹Ô“iOQ· : ãƒè䛽° : H›ÁKŸ :


2. : Y°~Œ<£ : : ãH÷‹²“†Ç°<£ : ƒÿáa…˜
H

1) ‚¬ð~¡<£ 2) ̋áH÷° 3) ‹¬¶ø@~Ÿ 4) UnHê^ΰ


M

1) ãH÷‹²“†Ç°<£ 2) ‚²ìO^ΰ=ô° 3) =ò‹²ÁO 4) UnHê^ΰ


AM

ª^Î#: (1) ‹Ô“iOQ·, ãƒè䛽°, H›ÁKŸ° Hê~¡°…Õx ƒìQꁰ. ‚¬ð~¡<£


IG

ª^Î#: (3) ãH÷‹²“†Ç°<£ =°`Ç ãQ®O^ŠÎO ƒÿáa…˜. =ò‹²ÁO =°`Ç ä›€_¨ Hê~¡°…Õx ƒìQ®"Í°.
KH

ãQ®O^ŠÎO Y°~Œ<£. ‹¬=¶^¥#O '=ò‹²ÁOÑ. ∴ ‹¬=¶^¥#O '‚¬ð~¡<£Ñ


R

3. _¨H›“~¡° : ̋“`Ǫéø„¹ : : ï~á`Ç° : 2. ㇦<£ž : K³á<Œ : [~¡àh :


1) <ŒQ®e 2) Qù_ÈÛe 3) H›uë 4) UnHê^ΰ
1) ̂áì^Î~Œƒì^£ 2) K³á<Œ 3) J"³°iHê 4) UnHê^ΰ
ª^Î#: (1) _¨H› “ ~ ¡ ° `Ç # =$uë „ ¬ ~ ¡ O Qê ̋“ ` Ç ª éø„¬ ô #° Z…ì ª^Î#: (3) ㇦<£ž, K³á<Œ, [~¡àh° ^͉§ À„~¡°Á. J"³°iHê 䛀_¨
L„¬†³¶yªë_Ë, ï~á`Ç° `Ç# =$uë „¬~¡OQê <ŒQ®ex J…ìöQ XH› ^͉×"Í°.
L„¬†³¶yªë_È°. Hê=ô# ‹¬=¶^¥#O '<ŒQ®eÑ. ∴ ‹¬=¶^¥#O 'J"³°iHêÑ
MODEL - 3 |‚¬•ˆ× „¬^¥ ‡éeH› 3. H÷~ˋ²<£ : _™l…˜ : ̄ã\Ձ° :
1) …Õ‚¬ð° 2) ƒçQ®°¾ 3) „¬O@° 4) UnHê^ΰ
1. †Çü~Ë : _¨~Ÿ : \ìHê : : ? : ?
1) ㇦<£ž - †Çò.Z‹¹.Z : |OQêÁ^͉˜
ª^Î#: (2) H÷~ˋ²<£, _™l…˜, ̄ã\Յ˜° WO^Î<Œ°. ƒçQ®°¾ä›½O_¨
2) \ìHê - †Çò.Z‹¹.Z H›ï~hž
WO^Î#"Í°
∴ ‹¬=¶^¥#O 'ƒçQ®°¾Ñ
3) _¨~Ÿ - |OQêÁ^͉˜ H›ï~hž 4) UnHê^ΰ
REASONING ABILITY (VERBAL) ○ ○ ○ ○ ○ ○ ○ ○ ○
73
○ ○ ○ ○ ○ ○ ○ ○ ○ ○ ○ ○ ○ ○ ○ ○ ○ ○ ○
Study Material
123456789012345678901234567890121234567890
123456789012345678901234567890121234567890
123456789012345678901234567890121234567890123456789012345678901212
12345678901234567890123456789012123456789012345678901234567890121234567890123456789012345678901212345678901234567890123
1234567890123456789012345678901212345678901234567890123456789012123456789012345678901234567890121234567890123
1234567890123456789012345678901212345678901234567890123456789012123456789012345678901234567890121234
12345678901234567890123456789012123456789012345678901234567890121234567890123456789012345678901212345678901234567890123456789012123
12345678901234567890123456789012123456789012345678901234567890121234567890123456789012345678901212345678901234567890123
1234567890123456789012345678901212345678901234567890123456789012123456789012345678901234567890121234567890123
1234567890123456789012345678901212345678901234567890123456789012123456789012345678901234567890121234
123456789012345678901234567890121234567890
123456789012345678901234567890121234567890123456789012345678901212
„¬^¥ J<Œl : TYPE - I : PRACTICE TEST
12345678901234567890123456789012123456789012345678901234567890121234567890123456789012345678901212345678901234567890123456789012123
12345678901234567890123456789012123456789012345678901234567890121234567890123456789012345678901212345678901234567890123
1234567890123456789012345678901212345678901234567890123456789012123456789012345678901234567890121234567890123
1234567890123456789012345678901212345678901234567890123456789012123456789012345678901234567890121234
12345678901234567890123456789012123456789012345678901234567890121234567890123456789012345678901212345678901234567890123456789012123
123456789012345678901234567890121234567890123456789012345678901212
12345678901234567890123456789012123456789012345678901234567890121234567890123456789012345678901212345678901234567890123
1234567890123456789012345678901212345678901234567890123456789012123456789012345678901234567890121234567890123
1234567890123456789012345678901212345678901234567890123456789012123456789012345678901234567890121234
123456789012345678901234567890121234567890
12345678901234567890123456789012123456789012345678901234567890121234567890123456789012345678901212345678901234567890123456789012123
123456789012345678901234567890121234567890123456789012345678901212
12345678901234567890123456789012123456789012345678901234567890121234567890123456789012345678901212345678901234567890123
1234567890123456789012345678901212345678901234567890123456789012123456789012345678901234567890121234567890123
1234567890123456789012345678901212345678901234567890123456789012123456789012345678901234567890121234
12345678901234567890123456789012123456789012345678901234567890121234567890123456789012345678901212345678901234567890123456789012123
12345678901234567890123456789012123456789012345678901234567890121234567890123456789012345678901212345678901234567890123
1234567890123456789012345678901212345678901234567890123456789012123456789012345678901234567890121234567890123
1234567890123456789012345678901212345678901234567890123456789012123456789012345678901234567890121234
12345678901234567890123456789012123456789012345678901234567890121234567890123456789012345678901212345678901234567890123456789012123
123456789012345678901234567890121234567890123456789012345678901212
12345678901234567890123456789012123456789012345678901234567890121234567890123456789012345678901212345678901234567890123456789012123
12345678901234567890123456789012123456789012345678901234567890121234567890123456789012345678901212345678901234567890123
1234567890123456789012345678901212345678901234567890123456789012123456789012345678901234567890121234567890123
1234567890123456789012345678901212345678901234567890123456789012123456789012345678901234567890121234

1. ï~lÁOQ· : H›~Œ>è : ƒìH÷žOQ· : ..... 14. „¬ô‹¬ëH›=ò : ~¡KÇ~ò`Ç : : qãQ®‚¬ì=ò : .....


1) ‹²Þq°àOQ· 2) ‡é…Õ 3) H›|_™Û 4) E_Ë 1) zã`ÇHê~¡°_È° 2) teæ
2. …ÕH±‹¬ƒ’ : ~Œ[¼‹¬ƒ’ : : ‰§‹¬#‹¬ƒ’ : ..... 3) 䛽=°ài 4) H›q
1) ‡~¡Á"³°O@° 2) q^¥#‹¬ƒ’ 15. ZQ®°~¡°@ : „¬H÷Æ : : ㇐䛽@ : .....
3) ̂áìHË~¡°“ 4) ‹ÔæH›~¡° 1) 䛽H›ø 2) H›„¬æ 3) K̈́¬ 4) ‡=ò
3. HêO : Q®_†Ç¶~¡O : `Ík : ...... 16. HËß : L`Çë~¡ã„¬^͉˜ : : ƒÕ‡…˜ : ..... (R.R.B.-2004)
1) H꼅ÿO_È~Ÿ 2) „¬ô‹¬ëH›=ò 1) =°^Î¼ã„¬^͉˜ 2) =òOƒì~ò
3) Q®_†Ç¶~¡=ò 4) UnHê^ΰ 3) POã^Îㄬ^͉˜ 4) UnHê^ΰ
4. 䛽H›ø : ö~c‹¹ : : ^Ë=° : ..... 17. ^ΰ—Y=ò : ..... : : ‹¬O`Ë+¬=ò : „¬ô@°“H›

Y
1) H©@Hꁰ 2) >ÿ؇¦~ò_£ 1) ƒì^Î 2) qKŒ~¡=ò 3) ^³|Ä 4) KŒ=ô@

M
3) =°…èi†Ç¶ 4) UnHê^ΰ 18. ^ŠÎ~Œàg°@~¡° : L‘éâãQ®`Ç : : ƒì~Ëg°@~¡° : .....
5. ^ΰ‹¬°ë° : ^¥~¡=ò : : W°Á : ..... 1) „Ô_È#=ò 2) |=ò

E
1) h~¡° 2) W@°H› 3) Qêe 4) P‚¬ð~¡=ò 3) Xuë_ 4) ƒì~¡=ò

D
9
6. <Œ¼†Ç°"Œk : ..... : : <Œ¼†Ç°=üië : f~¡°æ 19. kQ®°= ‹¬ƒ’ : ‰§‹¬# ‹¬ƒ’ : : ..... : =òY¼=°Oãu

59
1) ㄬ‹¬OQ®=ò 2) ƒÕ^Î# 1) ‹ÔæH›~¡° 2) ~ŒR„¬u
CA
95
3) "Œ^Î# 4) qx„²OKÇ_ÈO 3) ㄬ^¥#=°Oãu 4) Q®=~¡ß~¡°

50
7. ‹¬O=`Ǟ~¡=ò : <³ : : ..... : ~ËA 20. ƒ’¶q° : ‹¬¶~¡°¼_È° : : KÇOã^ΰ_È° : .....
98
A
1) „¬H›Æ=ò 2) <³ 3) Q®O@ 4) "Œ~¡=ò 1) q‰×Þ=ò 2) #H›Æã`Ç=ò
:8
8. ã‰×=° : „¦¬e`ÇO : : 䛀b : ..... 3) ‹¬¶~¡°¼_È° 4) ƒ’¶q°
E
h

1) r`Ç=ò 2) X„¬æO^Î=ò 21. ‡é\© : ïQ°„¬ô : : „¬sH›Æ : .....


.P
IC

3) H›+¬“=ò 4) ‹¬°Y=ò 1) J„¬[†Ç°O 2) Lfë~¡â`Ç


AD

9. ..... : ƒÕ[#=ò : : ^΄²æH› : h~¡° 3) ^括°ä›½‡é=@O 4) „¬~Œ[†Ç°O


AB
O

1) W+¬“`Ç 2) PH›e 3) H˄¬=ò 4) P"͉×=ò 22. „¬_È= : h~¡° : : ..... : Qêe


ER
CH

10. H˄¬=ò : ..... : : ‰§O`Ç=ò : q°ã`Ç°_È° 1) ^Î¶=°‰×H›@O 2) L„¬ãQ®‚¬ìO


1) ƒO^Î°=ô 2) À‹ß‚²ì`Ç°_È° 3) q=¶#=ò 4) Hê~¡°
YD

3) ‰×ã`Ç°=ô 4) ~¡H›Æ [#=i : _‰×O|~¡° : : Pk"Œ~¡=ò : .....


,H

23.

`ÇeÁ : "Í°#=¶=° : : ..... : "Í°#`Çë 1) ªé=°"Œ~¡=ò 2) ‰×x"Œ~¡=ò


T

11.
AM

1) J°Á_È° 2) J#ß 3) `Ç=òà_È° 4) `ÇOã_ 3) 12 <³° 4) 7 ~ËA°


H
M

12. J=¶"Œ‹¬¼ : ‹¬¶~¡°¼_È° : : ‡Ï~¡âq° : ..... 24. W#°=ò : ~Œy : : lOH± : ..... (R.R.B.-2005)
AM
IG

1) KÇOã^ΰ_È° 2) ƒ’¶q° 1) ‹²~Œq°H± 2) Hê~¡Ä<£ 3) W`Çë_ 4) ƒçQ®°¾


KH

3) ªÏ~¡=°O_ȁO 4) ãQ®‚¬ì}O 25. ƒì~¡`Ç^͉×=ò : ~ŒR„¬u : : POã^Îㄬ^͉˜ : .....


R

13. H›=ò : „¬ô‹¬ëH›=ò : : ..... : „¬H› 1) =òY¼=°Oãu 2) <Œ¼†Ç°=üië


1) ̄xž° 2) ̋øKŸ 3) |„¬=ò 4) #°„¬ô 3) H›…ÿH›“~¡° 4) Q®=~¡ß~¡°
12345678901234567890123456789012
12345678901234567890123456789012123456789012345678901234567890121234567890123456789012345678901212345678901234567890123
12345678901234567890123456789012

q=~¡}ì`ÇàH› ["Œ|°°
12345678901234567890123456789012123456789012345678901234567890121234567890123456789012345678901212345678901234567890123
12345678901234567890123456789012
12345678901234567890123456789012 12345678901234567890123456789012
12345678901234567890123456789012123456789012345678901234567890121234567890123456789012345678901212345678901234567890123
12345678901234567890123456789012
12345678901234567890123456789012
12345678901234567890123456789012123456789012345678901234567890121234567890123456789012345678901212345678901234567890123
12345678901234567890123456789012
12345678901234567890123456789012
12345678901234567890123456789012123456789012345678901234567890121234567890123456789012345678901212345678901234567890123
12345678901234567890123456789012 12345678901234567890123456789012
12345678901234567890123456789012123456789012345678901234567890121234567890123456789012345678901212345678901234567890123
12345678901234567890123456789012
12345678901234567890123456789012
12345678901234567890123456789012123456789012345678901234567890121234567890123456789012345678901212345678901234567890123
12345678901234567890123456789012

1. (4) Wzó# =ü_È° „¬^¥…Õx P@° =¶~¡Â…˜ P~Ÿ“ žH÷ 3. (1) ‹¬=°†Ç°=ò#° Q®_†Ç¶~¡O…Õ KǶªë=ò. J^Íq^ÎOQê
K³Ok#q. (W^Îíi =¼ä›½ëH÷) `ŒsY°#° H꼅ÿO_È~Ÿ…Õ KǶªë=ò.
Hê|\÷“ ["Œ|° J^Í HË=䛽 K³Ok#k E_Ë. 4. (3) 䛽H›ø Hê@° =Á =KÍó "Œ¼k ö~c‹¹, J^Íq^Î=òQê
2. (2) ^͉§xH÷ …ÕH±‹¬ƒ’, ~Œ[¼‹¬ƒ’° kQ®°=, ZQ®°= ‹¬ƒ’°. ^Ë=° Hê@° =Á =KÍó "Œ¼k =°…èi†Ç¶.
J…ìöQ ~Œ¢‘“xH÷ ‰§‹¬#‹¬ƒ’ =°i†Çò q^¥#‹¬ƒ’°. 5. (2) ^ΰ‹¬°ë° ^¥~¡O`Ë, W°Á W@°H›`Ë `džǶ~¡=ô`Œ~ò.
6. (3) <Œ¼†Ç°"Œk "Œkªë_È°. <Œ¼†Ç°=üië f~¡°æxªë_È°.
REASONING ABILITY (VERBAL) ○ ○ ○ ○ ○ ○ ○ ○ ○
74
○ ○ ○ ○ ○ ○ ○ ○ ○ ○ ○ ○ ○ ○ ○ ○ ○ ○ ○
Study Material
7. (4) ‹¬O=`Ǟ~¡=ò#䛽 12 <³° Z…ìQË "Œ~ŒxH÷ 7 17. (4) „¬ô@°“H› ‹¬O`ˑxßÀ‹ë, KŒ=ô ^ΰ—Mìxߋ¬°ëOk.
~ËA° J…ìQ®. 18. (1) ^Š Î ~ Œàg°@~¡ ° `Ë L‘éâ ã Q® ` Ç # °, ƒì~Ëg°@~¡ ° `Ë
8. (1) ã‰×=° =Á „¦¬e`Ç=ò, 䛀b=# r`Ç=ò =‹¬°ëOk. „Ô_È#=ò#° Hù°ªë~¡°.
9. (2) PH›e`Ë L#ß"ŒiH÷ ƒÕ[#=ò, ^΄²æH›`Ë L#ß"ŒiH÷ 19. (3) ^͉§xH÷ kQ®°=‹¬ƒ’, ~Œ¢‘“xH÷ ‰§‹¬# ‹¬ƒ’ Z…ìQË
h~¡° ã„Ôu ‡ã`Ç=ò. ^͉§xH÷ ㄬ^¥#=°Oãu, ~Œ¢‘“xH÷ =òY¼=°Oãu.
10. (3) H˄¬=ò ‰×ã`Ç°=ô#° ̄Oz`Í, ‰§O`Ç=ò q°ã`Ç°#° 20. (4) ƒ ’ ¶ q°, ‹¬ ¶ ~¡ ° ¼x KÇ ° @¶“ u~¡ ° Q® ° `Ç ° Ok. J…ìö Q
̄OKǰ`ǰOk.
KÇOã^ΰ_È°, ƒ’¶q° KÇ°@¶“ u~¡°Q®°`Ç°Ok.
11. (4) `ÇeÁ ªé^Î~¡°_È°#° "Í°#=¶=° Jx, `ÇOã_ ªé^Îix
21. (2) ‡é\©…Õ ïQ=_ÈO =òY¼O. J…ìöQ „¬sH›Æ…Õ
"Í°#`Çë Jx „²°ªë~¡°.
Lfë~¡°â_È=@O.
12. (1) J=¶"Œ‹¬¼ ‹¬¶~¡¼ãQ®‚ì¬ }O ~ËA#, ‡Ï~¡qâ ° KÇOã^ÎãQ®‚ì¬ }O
~ËA# =ªë~ò. 22. (3) h\÷…Õ „¬_È=, Qêe…Õ q=¶#=ò ㄬ†Ç¶}÷ªë~ò.
"³ò^Î\ ÷<³ [#=i, z=i<³ _‰×O|~¡°. J…ìöQ

Y
13. (3) „¬ô‹¬ëH›O…Õ H›O`Ë ~Œªë~¡°. „¬H›g°^Î |„¬O`Ë 23. (2)
~Œªë~¡°. Pk"Œ~¡=ò "³ò^Î\ ÷^³á`Í, ‰×x"Œ~¡=ò z=ik.

M
14. (2) „¬ô‹¬Hë êxß ~¡K~Ç ò`Ç ~¡zªë_°È . qãQ®‚ð¬ xß `dž¶Ç ~¡°KÍÀ‹k 24. (3) W#°=ò, ~Œy, lOH±° J<Íq …Õ‚¬ì=ò° =°i†Çò

E
teæ. W`Çë_ 䛀_¨ XH› …Õ‚¬ì=ò.
„¬H÷Æ ZQ®°~¡°`Ç°Ok. ‡=ò ㇐䛽`Ç°Ok. ^͉§xH÷ "³ò^Î\ ÷ ‡Ï~¡°_È° ~ŒR„¬u Z…ìQË ~Œ¢‘“xH÷

D
9
15. (4) 25. (4)

59
16. (1) HËß J<Ík L`Ç~ë ã¡ „¬^‰Í ˜ †³òH›ø ~Œ[^¥x. J^Íq^=Î òQê ㄬ^ŠÎ=° ‡Ï~¡°_È° Q®=~¡ß~¡°.
CA
95
ƒÕ‡…˜ J<Ík =°^¼Î ㄬ^‰Í ˜ †³òH›ø ~Œ[^¥x.

50
123456789012345678901234567890121234567890
123456789012345678901234567890121234567890123456789012345678901212
12345678901234567890123456789012123456789012345678901234567890121234567890123456789012345678901212345678901234567890123
1234567890123456789012345678901212345678901234567890123456789012123456789012345678901234567890121234567890123
1234567890123456789012345678901212345678901234567890123456789012123456789012345678901234567890121234
123456789012345678901234567890121234567890
123456789012345678901234567890121234567890123456789012345678901212
12345678901234567890123456789012123456789012345678901234567890121234567890123456789012345678901212345678901234567890123456789012123
12345678901234567890123456789012123456789012345678901234567890121234567890123456789012345678901212345678901234567890123
1234567890123456789012345678901212345678901234567890123456789012123456789012345678901234567890121234567890123
1234567890123456789012345678901212345678901234567890123456789012123456789012345678901234567890121234
12345678901234567890123456789012123456789012345678901234567890121234567890123456789012345678901212345678901234567890123456789012123
123456789012345678901234567890121234567890123456789012345678901212
98
A
12345678901234567890123456789012123456789012345678901234567890121234567890123456789012345678901212345678901234567890123
1234567890123456789012345678901212345678901234567890123456789012123456789012345678901234567890121234567890123
1234567890123456789012345678901212345678901234567890123456789012123456789012345678901234567890121234
123456789012345678901234567890121234567890
12345678901234567890123456789012123456789012345678901234567890121234567890123456789012345678901212345678901234567890123456789012123
PREVIOUS BITS on TYPE - I
12345678901234567890123456789012123456789012345678901234567890121234567890123456789012345678901212345678901234567890123
1234567890123456789012345678901212345678901234567890123456789012123456789012345678901234567890121234567890123
1234567890123456789012345678901212345678901234567890123456789012123456789012345678901234567890121234
123456789012345678901234567890121234567890123456789012345678901212
12345678901234567890123456789012123456789012345678901234567890121234567890123456789012345678901212345678901234567890123456789012123
12345678901234567890123456789012123456789012345678901234567890121234567890123456789012345678901212345678901234567890123
1234567890123456789012345678901212345678901234567890123456789012123456789012345678901234567890121234567890123
1234567890123456789012345678901212345678901234567890123456789012123456789012345678901234567890121234
123456789012345678901234567890121234567890
12345678901234567890123456789012123456789012345678901234567890121234567890123456789012345678901212345678901234567890123456789012123
123456789012345678901234567890121234567890123456789012345678901212
12345678901234567890123456789012123456789012345678901234567890121234567890123456789012345678901212345678901234567890123456789012123
12345678901234567890123456789012123456789012345678901234567890121234567890123456789012345678901212345678901234567890123
1234567890123456789012345678901212345678901234567890123456789012123456789012345678901234567890121234567890123
1234567890123456789012345678901212345678901234567890123456789012123456789012345678901234567890121234
:8
12345678901234567890123456789012123456789012345678901234567890121234567890123456789012345678901212345678901234567890123456789012123
12345678901234567890123456789012123456789012345678901234567890121234567890123456789012345678901212345678901234567890123
1234567890123456789012345678901212345678901234567890123456789012123456789012345678901234567890121234567890123
1234567890123456789012345678901212345678901234567890123456789012123456789012345678901234567890121234
E
h

1. ö~_†³¶ : ã‰Õ`Ç : : ‹²x=¶ : 7. HêiۆǶl : Q®°O_³ : : A"Œl :


.P
IC

(Communication S.I - 2012) (Communication S.I - 2012)


AD

1) x~Œà`Ç 2) #@°_È° 3) ãÀ„H›Æ䛽_È° 4) ^Î~¡Å䛽_È° 1) tbOã^¥° 2) ‹¬¶H›;r=ô°


AB
O

2. ‰×H÷ë : *º° : : „¦¬°#„¬i=¶}O : 3) [O`Ç°=ô° 4) "³òH›ø°


ER

8. ƒì~¡q°u : „Ô_È#O : : ^Î~Œàg°@~¡° :


CH

(Communication S.I - 2012)

1) ª=°~¡Ö¼O 2) b@~¡°
YD

(Dy. Jailors - 2012)

3) H÷…ÕãQê=ò 4) „¦¬°#=ò 1) L+¬âO 2) "Œ`Œ=~¡}O


,H

3. KÇOã^ΰ_È° : L„¬ãQ®‚¬ìO : : ƒ’¶q° : 3) L‘éâãQ®`Ç 4) ‰×H÷ë


T
AM

9. ‹¬°ãHË*˜ : KÇïHø~¡ : : …ìH˓*˜ : (Dy. Jailors - 2012)


H

(Communication S.I - 2012)


M

1) ãQ®‚¬ìO 2) Lø 3) `ËH›KÇ°H›ø 4) ‹¬¶~¡°¼_È° 1) ‹¬°#ßO 2) HËH± 3) `Í<³ 4) ‡°


AM
IG

4. ‹²~Œ : H›=ò : : ~¡H›ë=ò : 10. X_È°Û : #k : : f~¡O : (Dy. Jailors - 2012)

1) =~¡^Î 2) ‹¬=òã^ÎO
KH

(Communication S.I - 2012)


3) ̋†Í°~¡° 4) "ŒQ®°
R

1) ^¥#=ò 2) Z=òH›°
3) T„²iu`ǰ끰 4) <Œˆ×=ò 11. Qêe : "Œ`Œ=~¡}O : : h~¡° :
(Dy. Jailors - 2012)
5 ãH÷q°‹¬O‚¬ð~¡H›=ò : „¬O@ : : †Ç¶O\÷̋„²“H± :
(Communication S.I - 2012)
1) […ì=~¡}O 2) ‹¬=òã^ÎO
3) =~¡ÂO 4) r"Œ=~¡}O
1) ƒì¼O_Í_£ 2) WO[H›Æ<£
12. `³°„¬ô : ‰§Ou : : Z~¡°„¬ô :
3) ~¡H›ë㪐=O 4) Qê†Ç°=ò
(Hostel welfare officers - 2011)
6. ^ÎH÷Æ}O : D‰§#¼O : : „¬_È=°~¡ :
1) Q®°…ìc° 2) ‚²ìO‹¬ 3) J‹¬‚¬ì¼O 4) ~¡H›ëO
(Communication S.I - 2012)
13. ̂션à : ~¡|Ä~Ÿ :: ïH̄“<£ :? (F.R.O-2012)
1) "Œ†Çò=¼O 2) PöQ߆ǰO
1) <Œq䛽_È° 2) F_È "³#°H› ƒìQ®O
3) D‰§#¼O 4) <³á~¡°u 3) ª“~ŸƒÕ~Ÿß 4) †Ç°Oã`ÇO

REASONING ABILITY (VERBAL) ○ ○ ○ ○ ○ ○ ○ ○ ○


75
○ ○ ○ ○ ○ ○ ○ ○ ○ ○ ○ ○ ○ ○ ○ ○ ○ ○ ○
Study Material
14. ö~c‹¹ : 䛽H›ø : : : ^Ë=° (Dy. Jailors - 2012) 28. `Ë ‹¬O|Ok`"
GOAT, MILK Ç °³ #Ø k J^Íq^OÎ Qê 'FAC-
1) H›~Œ 2) „¬H›Æ"Œ`ÇO TORY' nx`Ë ‹¬O|Ok`" Ç °³ #Ø k. (H›=ü¼xöH+¬<£ S.I-2011)
3) =°…èi†Ç¶ 4) H›Æ†Ç° 1) L`Çæuë 2) ‡ÚQ®
15. WO_†Ç¶ : P‹²†Ç¶ : : WOQêÁO_£ : (S.I - 2011) 3) ㉧q°ä›½_È° 4) UgHê=ô
1) O_È<£ 2) †Çü~¡„¹ 29. MULE, PACK `Ë ‹¬O|Ok`Ç"³°Ø#k, J^Íq^ÎOQê 'OK'
3) WOwÁ+¹ 4) P¢À‹“e†Ç¶ ^Íx`Ë ‹¬O|Ok`Ç "³°Ø#k.. (H›=ü¼xöH+¬<£ S.I-2011)
16. ƒì¼_£q°O@<£ : P@‹¬ÖO : : (S.I - 2008) 1) ‚¬ìi 2) "³¶H± 3) ‚¬ð‹¹ 4) ‚¬ú„¦¹
1) ãH÷ïH\˜ : ƒì…˜ 2) ‚¬ðH© : KÍuH›ã~¡ 30. POET J#°#k VERSE`Ë ‹¬O|Ok`Ç"³°Ø#k, J^Í
3) À‹ø\÷OQ· : =°OKǰㄬ^͉×O 4) „¦¬ô\˜ƒì…˜ : Q˅˜ q^ÎOQê WOMEN nx`Ë ‹¬O|Ok`Ç"³°Ø#k
17. ^Î#¼"Œ^¥° : H›$`Ç[ý`Ç : : (S.I - 2008) ( H›=ü¼xöH+¬<£ S.I-2011)
1) ‰Õ^Î# : `DŽ¬C 2) =°~Œ¼^Î : ‹¬ƒ’¼`Ç 1) ƒ’~¡ë 2) „¬^μH›q`Ç

Y
3) J‹¬=°àu : H˄²OKÇ° 4) =O^Î#O : „¬`ŒH› 3) JO^Î=ò 4) ‹¬Ow`Ç ƒì}©
31. 'STATUE' J#°#k SHAPE `Ë ‹¬O|Ok`Ç"³°Ø#k.

M
18. 䛽só : =ã_ÈOy : : (S.I - 2008)

1) =O`³# : WO[h~¡° 2) ö~_†³¶ : ö~_†Ç¶l‹¬°“ J^Íq^ÎOQê SONG ^Íx`Ë ‹¬O|Ok`Ç"³°Ø#k

E
3) B+¬^ÎO : "³á^ΰ¼_È° 4) „¬zóH› : `Ë@=¶e ( H›=ü¼xöH+¬<£ S.I-2011)
1) „¬^Î=ò° 2) „¬^μH›q`Ç

D
9
19. ^ÎO`Ç=ò : ^ÎO`Ç "³á^ΰ¼_È° : : (Dy. Jailors - 2012)

59
1) Hêˆ×ä Ð „¬~˄¬Hêi 3) JO^Î=ò 4) ‹¬Ow`Ç ƒì}©
CA
95
2) KǶ„¬ô : ãÀ„H›Æ䛽_È° 32. 'P@"£°Ñ J#°#k '=¶e䛀¼…˜Ñ `Ë ‹¬O|Ok`Ç"³°Ø#k.

50
3) K³=ô° : <Íã`Ç x„¬ô}°_È° J^Íq^Î=òQê '̋…˜Ñ J#°#k nx`Ë ‹¬O|Ok`Ç"³°Ø#k.
98
A
4) H›ˆ×ä : <Íã`Ç x„¬ô}°_È° ( H›=ü¼xöH+¬<£ S.I-2011)
:8
20. ã_Èy¾‹¹“ : ‡¦~¡à‹² : : 1) „¬~¡Þ`Ç=ò 2) rq
E

(Dy. Jailors - 2012)


h

1) Hêï~æO@~Ÿ : K³H›ø 2) „¦²l+²†Ç°<£ : À„Ì+O\˜ 3) „¬^¥~¡œ=ò 4) UgHê=ô


.P
IC

3) K³„¦¹ : ï~ª“ï~O\˜ 4) 䛽=°ài : Pƒ’~¡}쁰 33. PIPE J#°#k WATER, WIRE J#°#k
AD

21. H꼅ÿO_È~Ÿ : `Ín : : (S.I - 2008)


(H›=ü¼xöH+¬<£ S.I-2011)
AB
O

1) ‹¬=°†Ç°O : Q®O@ 2) _H›Â#s : „¬^ÎO 1) ZãH÷“‹²\ © 2) H›ï~O\˜


ER

3) F>è“r 4) PO„²†Ç°~Ÿ
CH

3) ~¡"Œ}ì : |‹¬°ž 4) „¬@“}O : „²<£HË_£


34. 'i̋„¬Âx‹¹“Ñ, 'P„¦Ô‹¬°Ñ H÷ ‹¬O|OkOz#„¬ô_È°, J^Íq^ÎOQê
YD

22. H÷…ÕãQê"£° : H÷ÞO\션 : : ̄᪐ : (S.I - 2008)

1) ã^Î=¼O 2) ~¡¶‡~ò '‚¬ú̋“<£Ñ nx`Ë ‹¬O|Ok`Ç"³°Ø LO_È°#°.


,H

H›=ü¼xöH+¬<£ S.I-2011)
T

3) ‹¬O„¬^Î 4) <Œ}ÿO (
AM

1) ‹¬=ü‚¬ì=ò 2) Z~ò~Ÿ‡é~Ÿ“
H

23. ‡ÚQê䛽 : <Œ_È°° : : =°^Î¼=ò : (S.I - 2008)


M

1) Hê…è†Ç°O 2) x‘ 3) t~¡‹¬°ž 4) ª~Œ~ò 3) Hê~Ÿ 4) Z~ò~ŸãHꄦ¹“


AM
IG

35. 'H˅˜Ñ J#°#k 'À‚ì\˜Ñ 䛽 ‹¬O|OkOKÇ|_#k J^Íq^ÎOQê


24. J^ÎíO : ㄬuaO|O : : h~¡° : (S.I - 2008)
KH

'"ÍH±žÑ nx`Ë ‹¬O|Ok`Ç"³°Ø#k. (H›=ü¼xöH+¬<£ S.I-2011)


1) qöHƄ¬}O 2) =¼uH›~¡}O
R

1) `Í<³ 2) DQ® 3) HêOu 4) x„¬C


3) =ãH©ƒ’=#O 4) x=°[û#O
36. 'Hê~ŸÛƒÕ~¡°ÛÑ,'|‡ä›€¼Ñ`Ë ‹¬O|Ok`Ç"³°Ø#k. J^Í q^ÎOQê
25. "³¶\ì~ŸÌ‹áH÷…˜ : ƒì¼\s : : rq`ÇO : (S.I - 2008)
'QêÁ‹¹Ñ J#°#k nxH÷ ‹¬O|OkOz#k.
1) ƒ’¶q° 2) KÇOã^ΰ_È°
(H›=ü¼xöH+¬<£ S.I-2011)
3) #H›Æã`ÇO 4) ‹¬¶~¡°¼_È°
1) ‡~¡^Î~¡ÅH›`Ç 2) ㄬHê‰×=O`Ç"³°Ø#
26. Q®}÷`ÇO : ‹¬OY¼° : : KÇiã`Ç : (S.I - 2008)
3) QêÁ‹Ô 4) ^Î$_È=ò
1) ㄬ[° 2) †Çò^Îí=ò°
37. 'HOPE' J#°#k 'EXHILARATION' 䛽 K³Ok#@°Á
3) `Ín° 4) ‹¬O„¦¬°@#°
'DISAPPOINTMENT' J#°#k nxH÷ K³Ok#k..
27. ‡Ú_ : <Œ#ƒÿ\ ÷“# : : (S.I - 2007)
( H›=ü¼xöH+¬<£ S.I-2011)
1) Z_¨i : |†Ç¶‹²‹¹ 2) Qêe : P¢~¡Ö`Ç
1) ENVY 2) MELANCHOLY
3) h~¡° : h\÷Pqi 4) `Ç_…èx : ‹¬O`Ç$„¬ë ã^¥=}O
3) LIFE 4) GRAVE
REASONING ABILITY (VERBAL) ○ ○ ○ ○ ○ ○ ○ ○ ○
76
○ ○ ○ ○ ○ ○ ○ ○ ○ ○ ○ ○ ○ ○ ○ ○ ○ ○ ○
Study Material
38. 'BREEZE' J#°#k 'CYCLONE' 䛽 K³Ok#@°Á, 1) Water 2) Tip 3) Cap 4) Pat
'DRIZZLING' J#°#k nxH÷ K³Ok#k? 50. 'Cork' J<Ík 'Bottle' H÷ J~ò`Í.... J<Ík Box H÷
H›=ü¼xöH+¬<£ S.I-2011)
( J=ô`Ç°Ok. (AEE-2012)
1) TEMPTATION 2) FLOOD 1) Lock 2) Side 3) Colour 4) Lid
3) DOWNPOUR 4) DRENCH 51. Uㄲ…˜ : E…ÿá ::̋̄“O|~¡°:?
39. ......J#°#k NOH›ä›½ J~ò`Í _bÁ J#°#k 1) JH˓|~¡° 2) #=O|~Ÿ
H›=ü¼xöH+¬<£ S.I-2011)
( 3) _Ì‹O|~Ÿ 4) PQ®‹¬°“
1) HùOƒÕ, WO_†Ç¶ 2) Mì_¨àO_È°, ~Œ[^¥x 52. 'Drama' J<Í =¶@H÷ 'stage' J<Í =¶@…Õ ‹¬O|O^ÎO
3) WO_†Ç¶, ‡i‹¹ 4) nބ¬=ò, <ñHêã‰×†Ç°=ò J^Í q^OÎ Qê 'Tennis' J<Í =¶@䛽 ‹¬O|O^OÎ L#ß=¶@
40. Q®k : <ñH› (S.I.-2008) (A.E.in Muncipalities-2012)

1) HêH±„²\˜ : q=¶#=ò 2) KÇãH›=ò : 䛽só 1) Game 2) Court

Y
3) Q®k : ƒ’=Ou 3) Net 4) Racket
4) =O@Q®k : ª=¶#°Q®k

M
53. D ãH÷Ok Mìmx ‹¬O|Ok`Ç „¬^Î=ò`Ë „¬îiO„¬ô=ò?
41. _ȋ¬“~Ÿ : ‹¬°^Îí=òH›ø (S.I.-2008) =$H›Æ‰§G=ò : "³òH›ø° :: H©@H› ‰§G=ò:?

E
1) #Á|Á : ‹¬°^Îí~Œ~ò 2) ~¡|Ä~¡° : ã"Œ`Ç (Muncipal Juniors A/cs in A.P.-2012)
3) Q®°_ÈÛ : Qêe 4) ªæOl : h~¡°

D
1) ‡=ò° 2) ãH÷=ò°

9
59
42. `Ç°‡H÷ Q®°O_È°:`Ç°‡H÷ LOKÇ° „¬_È= (S.I.-2008) 3) „¬ä›Æ½° 4) „¬ô~¡°Q®°°
CA
95
1) „¬_È= : ‹¬~¡‹¬°ž 2) „¬zóH› : `Ë@ 54. Rich : Poor :: Enemy :? (ACF-2012)

50
3) ï~ၰ : „¬\쓁° 4) ‡m : H›=ò 1) "Œ~Ÿ 2) "³„¬<£ 3) ã̄¦O_£ 4) ‚¬ìº\˜
98
A
43. ~¡OQ®°° "͆Çò@ : H›ˆìHê~¡°Q®° (S.I.-2008) 55. 'Kathak' =¶@ä› ½ ” 'Uttarpradesh' =¶@…Õ
:8
1) ‡@ : Qê†Ç°ä›½_È° 2) ~¡KÇ~ò`Ç : #= ‹¬O|O^OÎ LOk. J…ìöQ 'oddisy' =¶@ä›½ U =¶@`Ë
E
h

3) „¬^μ=ò : H›q 4) ‹¬Ow`ÇAý_È° : ‹¬Ow`Ç=ò ‹¬O|O^ÎO LO@°Ok.


.P

(ACF-2012)
IC

44. ƒì^Î : ^³á#¼O (S.I.-2008) 1) =°‚¬ð~ŒR 2) JªžO


AD

1) "Œ¼k : À„^ÎiH›O 3) Xiªž 4) Q®°[~Œ`Ÿ


AB
O

2) x~Œ‰× : UHêO`Ç=ò 56. „¬ô=ôÞ : „¬O_È° :: „¬O_È° :? (F.R.O-2012)


ER

3) J*ìý#=ò : Q®O^Î~¡Qˈ×O
CH

1) P䛽 2) 䛀~¡Qê†Ç°°
4) =ü_È#=°àH›O : ï~á`Ç°° 3) q`Çë#O 4) "³òH›ø
YD

45. fã="Œ^Î = ò q°`Ç " Œ^Î = ò#ä› ½ ‹¬ O |Ok O z#^³ á ` Í 57. 'Earthquake' H÷ 'Earth' `ˋ¬O|O^O Î J…ìöQ 'Thun-
,H

q„¬Á==ò nxH÷ ‹¬O|OkOz#k dering' H÷ ^Íx`Ë ‹¬O|O^ÎO (A.E.Inspectors-'12)


T

(S.I.-2008)
AM

1) Hêy`Ç=ò 2) ‹¬`Ǽ=ò 1) Earth 2) Sea


H
M

3) ^ڐë"ÍA° 4) JOwHê~¡=ò 3) Fair 4) Sky


AM
IG

46. ~ù>ÿ“, ~ù>ÿ“ `džǶsä›½ ‹¬O|OkOz#k. J^Í q^ÎOQê 58. 'Author H÷ 'writing'`Ë ‹¬O|O^ÎO J…ìöQ 'Thief' H÷
KH

W@°H› ~Œ~ò nxH÷ ‹¬O|OkOz#k (S.I.-2008) ^Íx`Ë ‹¬O|O^ÎO? (A.E.Inspectors-2012)


R

1) @OH›‰§ 2) W\÷H÷ J„¬O 1) Night 2) Fear


3) Hùeq° |\÷“ 4) Q®x 3) Steal 4) Wander
47. "³°ÁKǶ„¬ô :H›#°ß :: „²O_È° :? (Group-I-2012) 50. 'Mosque' H÷ 'Islam' `Ë ‹¬O|O^ÎO J…ìöQ church
1) <Œ°H› 2) K³~ò¼ H÷ ^Íx`Ë ‹¬O|O^ÎO? (A.E.Inspectors-2012)

3) |@“° 4) QùO`Ç° 1) Hinduism 2) Sikkism


48. "³á^ΰ¼_È° : ~ËQ®x~ŒÖ~¡} :: <Œ¼†Ç°=üië? (DL-2012) 3) Christianity 4) Budlism
1) <Œ¼†Ç°ªÖ#=ò 2) f~¡°æ 60. 'Needle' H÷ 'Thread'`Ë ‹¬O|O^ÎO J…ìöQ 'pen'H÷
3) tH›Æ 4) <Œ¼†Ç°"Œk ^Íx`Ë ‹¬O|O^ÎO? (A.E.Inspectors-2012)

49. Nap H÷ Pan J~ò`Í Tap ......H÷ (AEE-2012) 1) Word 2) Ink 3) Paper 4) Cap

REASONING ABILITY (VERBAL) ○ ○ ○ ○ ○ ○ ○ ○ ○


77
○ ○ ○ ○ ○ ○ ○ ○ ○ ○ ○ ○ ○ ○ ○ ○ ○ ○ ○
Study Material
12345678901234567890123456789012
12345678901234567890123456789012123456789012345678901234567890121234567890123456789012345678901212345678901234567890123
12345678901234567890123456789012

q=~¡}ì`ÇàH› ["Œ|°°
12345678901234567890123456789012123456789012345678901234567890121234567890123456789012345678901212345678901234567890123
12345678901234567890123456789012 12345678901234567890123456789012
12345678901234567890123456789012123456789012345678901234567890121234567890123456789012345678901212345678901234567890123
12345678901234567890123456789012 12345678901234567890123456789012
12345678901234567890123456789012
12345678901234567890123456789012123456789012345678901234567890121234567890123456789012345678901212345678901234567890123
12345678901234567890123456789012
12345678901234567890123456789012123456789012345678901234567890121234567890123456789012345678901212345678901234567890123
12345678901234567890123456789012 12345678901234567890123456789012
12345678901234567890123456789012123456789012345678901234567890121234567890123456789012345678901212345678901234567890123
12345678901234567890123456789012 12345678901234567890123456789012
12345678901234567890123456789012123456789012345678901234567890121234567890123456789012345678901212345678901234567890123
12345678901234567890123456789012 12345678901234567890123456789012

1. (3) ö~_†³¶ q<Í"Œ_È° ã‰Õ`Ç 20. (3) =°O^ΰ° J=òà"Œ_È° ‡¦~¡à‹²…Õ „¬xKͪë_È°. J…ìöQ
‹²x=¶ KǶÀ‹"Œ_È° ãÀ„H›Æ䛽_È° K³„¦¹ (=O@…Õ <Í~¡æi) ï~ª“ï~O\˜…Õ „¬xKͪë_È°.
2. (2) ‰×H÷ëH÷ ㄬ=¶}쁰 *º°. 21. (2) `ÍnHˋ¬O H꼅ÿO_È~Ÿ "³`ǰ䛽`ŒO J…ìöQ „¬^¥
„¦¬°# „¬i=¶}ìxß b@~¡Á…Õ Hù°ªë~¡° J~¡ÖOHˋ¬O _H›Â#s "³`ǰ䛽`ŒO.
3. (1) KÇOã^ΰ_È° XH› L„¬ãQ®‚¬ìO, ƒ’¶q° XH› ãQ®‚¬ìO 
4. (4) ‹²~Œ H›=ò…Õ =ôO@°Ok 22. (2) H÷…ÕãQê"£° «  × H÷ÞO\션 . J…ìöQ
~¡H›ëO ~¡H›ë <Œˆì…Õ LO@°Ok 
5. (4) „¬O@䛽 ãH÷q°‹¬O‚¬ð~¡H›O Z…ìQË, Qê†Ç¶xH÷ ̄᪐ «  × ~¡¶‡~ò.
†Ç¶O\©Ì‹„²“H± J…ìQ®. 23. (1) ‡ÚQê䛽 =# ‰×s~¡O…Õx <Œ_È°° ^³|ÄuO\ì~ò.

Y
6. (2) ^Î H ÷ Æ } O #°O_ J„¬ ‹ ¬ = ¼k‰× … Õ 135 HË}O…Õ J…ìöQ =°^Î¼‡#O =# ‰×s~¡O…Õx Hê…è†Ç°O

M
D‰§#¼O =ôO@°Ok. ^³|ÄuO@°Ok.
J…ìöQ „¬_È=°~¡ #°O_ 135 J„¬‹¬=¼ k‰×…Õ PöQ߆ǰO 24. (3) J^ÎíO ㄬuaOƒìxß WKÇ°ó ^Î~Œàxß H›ey†Çò#ß>èÁ, h~¡°

E
=ôO@°Ok. =ãH©ƒ’=<Œxß H›°Q®*èÀ‹ ^Î~Œàxß H›ey†ÇòO@°Ok.

D
9
7. (3) Q®°O_³ Q®°iOz J^μ†Ç°#O KÍÀ‹ ‰§¢ªëxß HêiۆǶr

59
25. (4) "³¶\ì~Ÿ ̋áH÷…˜ä›½ ƒì¼@s U q^ÎOQê J=‹¬~¡"³¶
JO\ì~¡°.
CA
95
rq`ŒxH÷ ‹¬¶~¡°¼_È° J^Í q^ÎOQê J=‹¬~¡O.
[O`Ç°=ô Q®°iOz J^Î¼†Ç°#O KÍÀ‹ ‰§¢ªëxß A"Œl 26. (4) ‹¬OY¼ä›½ ‹¬O|OkOz# ‰§GO Q®}÷`ÇO.

50
JO\ì~¡°. Q®`ÇO…Õ [iy# ‹¬O„¦¬°#ä›½ ‹¬O|OkOz# ‰§GO
98
A
8. (3) ƒì~¡q°u`Ë "Œ`Œ=~¡} „Ô_ÈO#° Hù°ªë~¡°. KÇiã`Ç.
:8
^ŠÎ~Œàg°@~¡°`Ë L‘éâãQ®`Ç#° Hù°ªë~¡°.
E

27. (3) ‡Ú_ Ð <Œ#ƒÿ\ ÷“# =¼uö~H›„¬^¥°. J…ìöQ


h
.P

9. (4) KÇïHø~¡…Õ ‹¬°ãHË*˜ =ôO@°Ok. `Ç_…èx Ð ‹¬O`Ç$„¬ë ã^¥=}쁰 䛀_¨ =¼uö~H›„¬^¥…è.


IC

‡…Õ …ìH˓*˜ =ôO@°Ok.


AD

28. (1) Goat #°Oz Milk Produce J=ô`Ç°Ok. J>è Fac-


10. (2) #kH÷ X_È°Û Z…ìQË, ‹¬=òã^¥xH÷ f~¡O J…ìQ®
AB
O

tory #°O_ Product (L`Çæuëï) =‹¬°ëOk.


11. (1) "Œ`Œ=~¡ } O…Õ Qêe =ôO@°Ok. J…ìö Q
ER

29. (2)
CH

[…ì=~¡}O…Õ h~¡° =ôO@°Ok.


30. (3)
YD

12. (2) `³°„¬ô ~¡OQ®° ‰§OuH÷ ‹¬OöH`ÇO


31. (4) Statue H÷ Hê=‹²#k XH› xiœ+¬“"³°Ø# Shape. J>èÁ
,H

Z~¡°„¬ô ~¡OQ®° ‚²ìO‹¬ä›½ ‹¬OöH`ÇO


T

Song H÷ Hê=‹²#k ‹¬Ow`ǃì}©.


AM

13. (1)
32. (2) '=¶e䛀¼…˜Ñ …Õ 'P@"£°žÑ LO_È°#°. J>èÁ 'rqÑ …Õ
H

14. (3) ö~c‹¹ "Œ¼k 䛽H›ø Hê@°=# =‹¬°ëOk.


M

̋…˜ž LO_È°#°.
AM

=°…èi†Ç¶ "Œ¼k ^Ë=°Hê@° =# =‹¬°ëOk.


IG

15. (2) WO_†Ç¶, P‹²†Ç¶ YO_ÈO…Õx ^͉×O. 33. (1) PIPE …Õ WATER ㄬ=°‚²ì‹¬°ëOk.J>èÁ WIRE …Õ
KH

WOQêÁO_£, †Çü~¡„¹ YO_ÈO…Õx ^͉×O. ZãH÷“‹²\ © ㄬ=‚²ì‹¬°ëOk.


R

16. (3) P@‹¬ÖO…Õ ƒì¼_àO@<£ P@ P_È°`Œ~¡°. =°OKÇ°=ô#ß 34. (2)


ㄬ^͉×O…Õ<Í À‹ø\÷OQ· P_È°`Œ~¡°. 35. (4)
17. (3) =°#䛽 Z=ï~á<Œ ‹¬‚¬ð†Ç°O K͋²#„¬ô_È° H›$`Ç[ý`Ç`Ë 36. (1) ‡~¡^Î~¡ÅH›`Ç.
^Î#¼"Œ^¥° Z…ì `³e†Ç°*誐ë"³¶, J…ìöQ H˄¬O 37. (2)
=zó#„¬ô_È° J‹¬=°àux `³e†Ç°*誐ëO.
38. (3)
18. (1) 䛽sóx =ã_ÈOy `džǶ~¡°Kͪë_È°.
=O`³##° WO[h~¡° xiઐë_È°. 39. (1) NOH› ^Í‰× ~Œ[^¥x 'HùOƒÕÑ J>èÁ ƒì~¡`Ç^͉×
19. (4) ^ÎO`Ç=ò#䛽 ‹¬O|OkOz# ‹¬=°‹¬¼#° ^ÎO`Ç"³á^ΰ¼_È° ~Œ[^¥x '_bÁÑ.
„¬i+¬øiªë_°È . J…ìöQ H›ˆ¤× 䛽 ‹¬O|OkOz# ‹¬=°‹¬¼#° 40. (3) <ñH›…Õ Hê¼a<£ Q®k LO_È°#@°Á, ƒ’=Ou…Õ Q®k
<Íã`Ç x„¬ô}°_È° „¬i+¬øiªë_È°. LO@°Ok.

REASONING ABILITY (VERBAL) ○ ○ ○ ○ ○ ○ ○ ○ ○


78
○ ○ ○ ○ ○ ○ ○ ○ ○ ○ ○ ○ ○ ○ ○ ○ ○ ○ ○
Study Material
41. (4) ‹¬ ° ^Î í = òH› ø (KŒH± „ Ô ‹ ¹ ) `Ë ~Œ‹² # ^¥xx _È ‹ ¬ “ ~ Ÿ ` Ë 50. (4) Bottle ̄á# =ü`Çx 'cork' JO\ì~¡°. J…ìöQ 'Box'
`ùyOKÇ Q ®  O. J^Í q ^ Î O Qê h\ ÷ x ªæOl`Ë Ì„á ^¥xx 'Lid' JO\ì~¡°.
`ùyOKÇQ®O. 51. (2)
42. (4) 43. (1) 44. (3) 52. (2)
45. (3) fã="Œ^Î=ò, q°`Ç"Œ^Î=ò° ‹¬~¡‹¬æ~¡O =¼uö~Hê~ŒÖxß 53. (4)
WKÇ°ó#°.
54. (3) Rich (^ÎxH›)䛽 =¼uö~H›=ò Poor (c^Î)
46. (2)
J…ìö Q Enemy (‰× ã `Ç ° =ô)H÷ =¼uö ~ H› = ò ã̄¦ O _£
47. (3) H›#°ß`Ë '"³°ÁKǶ„¬ôÑ KǶ_È=KÇ°ó. J>èÁ '|@“xßÑ KÍu`Ë
(À‹ß‚²ì`Ç°_È°)
'„²O_È=KÇ°óÑ.
55. (3) 'Uttar Pradesh' #O^ΰ „¬ô\÷“# "ŒH›¼O Kathak
48. (2) "³ á ^ Î ° ¼_È ° '~ËQ® x ~Œœ ~ ¡ } Ñ KÍ † Ç ò #°. J^Í q ^ Î O Qê
(H›^¥H›e) J…ìöQ 'Oddisy' J#°#k Xiªž…Õ „¬ô\÷“#
<Œ¼†Ç°=üië 'f~¡°æÑ K͆Çò#°.

Y
<Œ@¼O.
49. (4)

M
56. (3)
N a p → P a n
57. (4) 58. (3)

E
J^Íq^ÎOQê
59. (3) 60.

D
(2)

9
N a p → P a n

59
CA
95
TYPE - II : #O|~Ÿ J<Œl
50
98
A
:8
+ #O|~Ÿ J<Œl…Õ ㄬ‰×ß…Õ W=Þ|_# "³ò^Î\ ÷ ï~O_È° ‹¬OY¼ XH› xiÖ+¬»"³°Ø# ‹¬O|O^¥xß H›ey=ôO\ì~ò. J^Í q^Î"³°Ø#
E
h

‹¬O|O^ÎO H›ey=ô#ß ‹¬OY¼#° Wzó# P„¬Â<£ #°O_ ãQ®‚²ìOKŒe.


.P
IC
AD

MODEL - 1 (‹²Oy…˜ #O|~Ÿ J<Œr)


3. 31 : 33 : : 57 :
AB
O

ª^Î#: 31, 33 ° =~¡°‹¬ ƒè‹² ‹¬OY¼°


1. JOïH =°^Î¼ `Í_¨ : ㄬ‰×߅Õx "³ò^Î@ JOïH [`Ç
ER
CH

∴ 57 `Ç~¡°"Œ`Ç =KÇ°ó ƒè‹² ‹¬OY¼ 59


=°^¼Î ƒ^è OÎ `Ç~°¡ "Œ`Ç JOïH [`Ç =°^¼Î ƒè^OΏ ‹¬=¶#OQê
YD

=ôO_È°@. MODEL - 3
,H
T

L^¥ : 15 : 40 : : 75 : + JOïH "³ò`ÇOë 䛀_ `Ç~°¡ "Œ`Ç ‹¬OY¼Qê ~Œ†Çò@, JOïH


AM
H

ª^Î#: 40 Ð 15 « 25 |œO#° `Ç~¡°"Œ`Ç ‹¬OY¼Qê ~Œ†Çò@, ªÖ<Œ "ŒsQê


M

J^Í q^ÎOQê, Hê=‹²# ‹¬OY¼ ? 100 Ð 75 « 25.


AM

=KÇ°ó ‹¬OY¼ J<Œr, =¼uö~H› k‰×…Õ ~Œ†Çò@ :


IG
KH

MODEL - 2 1. 3746 : 6473 : : 1604 :


R

ª^Î#: 3746 #° =¼uö~H›k‰×…Õ ~Œ†Ç°Qê 6473


G =~¡°‹¬ ‹¬i‹¬OY¼° …è^¥ =~¡°‹¬ ƒè‹² ‹¬OY¼° …è^¥
J^Í q^ÎOQê 1604#° =¼uö~H› k‰×…Õ ~Œ†Ç°Qê 4061
=~¡°‹¬ ㄬ^¥# ‹¬OY¼° …è^¥ =~¡°‹¬ ‹¬O†ÇòH›ë ‹¬OY¼
2. 34 : 25 : : 13 :
[`ǁ䛽 ‹¬O|OkOz# J<Œl :
1. 29 : 31 : : 59 : ª^Î#: 34 → 32 + 42 = 9 + 16 = 25
ª^Î#: 29, 31 ° =~¡°‹¬ ㄬ^¥# ‹¬OY¼° J^Í q^ÎOQê 13 → 12 + 32 = 1 + 9 = 10
∴ 59 `Ç~¡°"Œ`Ç =KÇ°ó ㄬ^¥# ‹¬OY¼ « 61 3. 5 : 6 : 30 : : 7 : 5 :
2. 18 : 20 : : 9 : ª^Î#: 5 : 6 : 30 → 5 × 6 = 30
ª^Î#: 18, 20 ° =~¡°‹¬ ‹¬O†ÇòH›ë ‹¬OY¼°. 7 : 5 : ? → 7 × 5 = 35
∴ 9 `Ç~¡°"Œ`Ç =KÇ°ó ‹¬O†ÇòH›ë ‹¬OY¼ « 10 ∴ Hê=‹²# ‹¬OY¼ « 35MO

REASONING ABILITY (VERBAL) ○ ○ ○ ○ ○ ○ ○ ○ ○


79
○ ○ ○ ○ ○ ○ ○ ○ ○ ○ ○ ○ ○ ○ ○ ○ ○ ○ ○
Study Material
MODEL - 4 MODEL - 7
G =~¡°‹¬ ‹¬‚¬ì[ ‹¬OY¼ †³òH›ø =~¡¾=ò J<Œl : G x3 + x2 ~¡¶„¬O =°i†Çò x3 – x2 ~¡¶„¬O…Õx ‹¬OY¼
1. 169 : 289 : : 361 : J<Œl :
ª^Î#: 132 « 169  172 « 289  192 « 361 13, 17, 19 1. 2 : 12 : : 36 :
° =~¡°‹¬ ㄬ^¥ # ‹¬OY¼°. `Ç~°¡ "Œ`Ç ã„¬^¥ # ‹¬OY¼ 23. ª^Î#: 1 + 1 = 1 + 1 = 2 ; 2 + 2 = 8 + 4 = 12
3 2 3 2

∴ Hê=‹²# ‹¬OY¼ « 232 « 529. 33 + 32 = 27 + 9 = 36; 43 + 42 = 64 + 16 = 80


2. 169 : 196 : : 225 : 2. 180 : 100 : : 48 :
ª^Î#: 13 « 169  14 « 196  15 « 225  16 « 256 ª^Î#: 63 – 62 = 216 – 36 = 180
2 2 2 2

∴ Hê=‹²# ‹¬OY¼ « 256. 53 – 52 = 125 – 25 = 100


G =~¡°‹¬ ‹¬‚¬ì[ ‹¬OY¼ †³òH›ø „¦¬°#=ò J<Œl : 43 – 42 = 64 – 16 = 48
3. 1 : 8 : : 27 : ∴ 33 – 32 = 27 – 9 = 18

Y
ª^Î#: 13 « 1  23 « 8  33 « 27 MODEL - 8

M
(_È|°…˜ #O|~Ÿ J<Œr)
∴ Hê=‹²# ‹¬OY¼ « 43 « 64.
1. 25 : 625 : : ?

E
MODEL - 5 1) 15:215 2) 10:110 3) 12:144 4) UnHê^ΰ

D
9
ª^Î#: 25 → 252 « 625 J^Íq^ÎOQê

59
G x2 + 1 ~¡¶„¬O, x2 – 1 ~¡¶„¬O, x2 + x ~¡¶„¬O =°i†Çò
12 → 122 « 144
CA
95
x2 – x ~¡¶„¬O…Õx ‹¬OY¼ J<Œl 2. 3 : 243 : : ?
1. x2 – x ~¡¶„¬O 20 : 30 : : 42 :

50
1) 5 : 125 2) 5 : 3125 3) 5 : 25 4) UnHê^ΰ
ª^Î#: 52 Ð 5 « 25 Ð 5 « 20 98
A
ª^Î#: 3 → 35 « 243 J^Íq^ÎOQê
:8
62 Ð 6 « 36 Ð 6 « 30 5 → 55 « 3125
E

72 Ð 7 « 49 Ð 7 « 42
h

MODEL - 9
.P

∴ Hê=‹²# ‹¬OY¼ « 82 Ð 8 « 64 Ð 8 « 56
IC
AD

2. 110 : 132 : : 156 : + D qƒìQ®O…Õ Hùxß ‹¬OY¼ ‹¬=ü‚¬ìO XH› x†Ç°=°O


P^¥~¡OQê W=Þ|_È°`Ç°Ok. D ‹¬OY¼ ‹¬=ü‚¬ìO U
AB
O

ª^Î#: 112 – 11 = 121 – 11 = 110


x†Ç°=¶xß PKÇiOKÇ°KÇ°#ß"Ë J^Í x†Ç°=¶xß J#°‹¬i‹¬°#
ë ß
122 – 12 = 144 – 12 = 132
ER
CH

"Í~ùH› ‹¬=ü‚¬ðxß Wzó# P„¬Â#° #°O_ Q®°iëOKŒe.


132 – 13 = 169 – 13 = 156
YD

1. (348, 925, 476)


142 – 14 = 196 – 14 = 182
1) (177, 682, 737) 2) (854, 637, 924)
,H

3. x2 + x ~¡¶„¬O 42 : 56 : : 12 :
T

3) (385, 487, 581) 4) (188, 961, 169)


AM

ª^Î#: 62 { 6 « 36 { 6 « 42 ª^Î#: 348 → 3 + 4 + 8 = 15


H

72 { 7 « 49 { 7 « 56
M

925 → 9 + 2 + 5 = 16 =~¡°‹¬ ‹¬OY¼°


AM

32 { 3 « 9 { 3 « 12
IG

476 → 4 + 7 + 6 = 17
∴ Hê=‹²# ‹¬OY¼ « 42 { 4 « 16 { 4 « 20
KH

J^Í q^OÎ Qê Q® ‹¬OY¼ ‹¬=ü‚¬ìO (177, 682, 737)


177 → 1 + 7 + 7 = 15
R

MODEL - 6 682 → 6 + 8 + 2 = 16 =~¡°‹¬ ‹¬OY¼°


G x3 + 1 ~¡¶„¬O, x3 – 1 ~¡¶„¬O, x3 + x2 ~¡¶„¬O, x3 – x2 737 → 7 + 3 + 7 = 17
~¡¶„¬O…Õx ‹¬OY¼ J<Œel : ∴ ‹¬=¶^¥#O (177, 682, 737)
1. 3
x + 1 ~¡¶„¬O 2. (96, 48, 24)
2 : 9 : : 126 : 1) (84, 42, 12) 2) (64, 32, 14)
ª^Î#: 13 23
+ 1 = 1 + 1 = 2; + 1 = 8 + 1 = 9 3) (104, 52, 26) 4) (48, 12, 6)
53 + 1 = 125 + 1 = 126; 63 + 1 = 216 + = 217 ª^Î#: (96, 48, 24)
2. 9 : 28 : : 65 : 96 …Õ ‹¬Q®O 48 =°i†Çò 48…Õ ‹¬Q®O 24. J^Í q^ÎOQê
=ô#ß ‹¬=ü‚¬ìO 3) (104, 52, 26) 104…Õ ‹¬Q®O 52
ª^Î#: 23 + 1 = 8 + 1 = 9 ; 33+ 1 = 27 + 1 = 28
=°i†Çò 52…Õ ‹¬Q®O 26.
43+ 1 = 64 + 1 = 65 ; 53 + 1 =125 + 1 = 126 ∴ ‹¬=¶^¥#O (104, 52, 26)

REASONING ABILITY (VERBAL) ○ ○ ○ ○ ○ ○ ○ ○ ○


80
○ ○ ○ ○ ○ ○ ○ ○ ○ ○ ○ ○ ○ ○ ○ ○ ○ ○ ○
Study Material
123456789012345678901234567890121234567890
123456789012345678901234567890121234567890123456789012345678901212
123456789012345678901234567890121234567890
123456789012345678901234567890121234567890123456789012345678901212
12345678901234567890123456789012123456789012345678901234567890121234567890123456789012345678901212345678901234567890123456789012123
12345678901234567890123456789012123456789012345678901234567890121234567890123456789012345678901212345678901234567890123
1234567890123456789012345678901212345678901234567890123456789012123456789012345678901234567890121234567890123
1234567890123456789012345678901212345678901234567890123456789012123456789012345678901234567890121234
12345678901234567890123456789012123456789012345678901234567890121234567890123456789012345678901212345678901234567890123456789012123

#O|~Ÿ J<Œl : TYPE - II : PRACTICE TEST


12345678901234567890123456789012123456789012345678901234567890121234567890123456789012345678901212345678901234567890123
1234567890123456789012345678901212345678901234567890123456789012123456789012345678901234567890121234567890123
1234567890123456789012345678901212345678901234567890123456789012123456789012345678901234567890121234
123456789012345678901234567890121234567890
123456789012345678901234567890121234567890123456789012345678901212
12345678901234567890123456789012123456789012345678901234567890121234567890123456789012345678901212345678901234567890123456789012123
12345678901234567890123456789012123456789012345678901234567890121234567890123456789012345678901212345678901234567890123
1234567890123456789012345678901212345678901234567890123456789012123456789012345678901234567890121234567890123
1234567890123456789012345678901212345678901234567890123456789012123456789012345678901234567890121234
12345678901234567890123456789012123456789012345678901234567890121234567890123456789012345678901212345678901234567890123456789012123
123456789012345678901234567890121234567890123456789012345678901212
12345678901234567890123456789012123456789012345678901234567890121234567890123456789012345678901212345678901234567890123
1234567890123456789012345678901212345678901234567890123456789012123456789012345678901234567890121234567890123
1234567890123456789012345678901212345678901234567890123456789012123456789012345678901234567890121234
123456789012345678901234567890121234567890
12345678901234567890123456789012123456789012345678901234567890121234567890123456789012345678901212345678901234567890123456789012123
123456789012345678901234567890121234567890123456789012345678901212
12345678901234567890123456789012123456789012345678901234567890121234567890123456789012345678901212345678901234567890123
1234567890123456789012345678901212345678901234567890123456789012123456789012345678901234567890121234567890123
1234567890123456789012345678901212345678901234567890123456789012123456789012345678901234567890121234
12345678901234567890123456789012123456789012345678901234567890121234567890123456789012345678901212345678901234567890123456789012123
12345678901234567890123456789012123456789012345678901234567890121234567890123456789012345678901212345678901234567890123
1234567890123456789012345678901212345678901234567890123456789012123456789012345678901234567890121234567890123
1234567890123456789012345678901212345678901234567890123456789012123456789012345678901234567890121234
12345678901234567890123456789012123456789012345678901234567890121234567890123456789012345678901212345678901234567890123456789012123
123456789012345678901234567890121234567890123456789012345678901212
12345678901234567890123456789012123456789012345678901234567890121234567890123456789012345678901212345678901234567890123456789012123
1234567890123456789012345678901212345678901234567890123456789012123456789012345678901234567890121234567890123
1234567890123456789012345678901212345678901234567890123456789012123456789012345678901234567890121234

  16. W=Þ|_# [`Ç : (6, 15, 28)


1.  : 64 : :

:
1) (46, 56, 66) 2) (50, 59, 71)
1) 100 2) 81 3) 64 4) 49 3) (60, 67, 72) 4) (60, 69, 82)
2. 15 : 50 : : 25 : 17. W=Þ|_# [`Ç : (3, 18, 36)
1) 60 2) 90 3) 80 4) 110 1) (2, 10, 16) 2) (4, 24, 48)
3. 5 : 155 : : 10 : 3) (6, 42, 48) 4) (12, 72, 96)
1) 1100 2) 210 3) 1110 4) 510 18. W=Þ|_# [`Ç : (2, 14, 16)
4. 3 : 13 : : 37 :
1) (2, 7, 8) 2) (2, 9, 16)
1) 81 2) 49 3) 50 4) 65 3) (3, 21, 24) 4) (4, 16, 18)
5. 19.

Y
123 : 36 : : 367 :
(62, 49, 36)
1) 216 2) 256 3) 324 4) 49 1) (64, 84, 106) 2) (48, 62, 42)

M
6. 18 : 48 : : 100 :
3) (76, 53, 30) 4) (96, 36,82)
1) 150 2) 169 3) 144 4) 180 20.

E
(112, 56, 14)
7. 13 : 127 : : 26 :
1) (144, 72, 18) 2) (256, 64, 16)

D
9
1) 8216 2) 464 3) 3416 4) 8512

59
3) (196, 98, 24) 4) (120, 60, 30)
8. 36 : 50 : : 64 :
CA
95
21. (11, 71, 12)
1) 80 2) 82 3) 81 4) 83
1) (12, 65, 13) 2) (13, 83, 14)

50
9. 3 : 10 : : 5 :
3) (16, 78, 12) 4) (11, 96, 13)
98
A
1) 125 2) 126 3) 24 4) 26
22.
:8
(546, 120, 0)
10. 46 : 20 : : 92 :
1) (643, 72, 14) 2) (146, 74, 12)
E
h

1) 45 2) 46 3) 43 4) 40
.P

3) (841, 32, 8) 4) (638, 48, 6)


IC

11. 7 : 7497 : 6 :
23.
AD

(761, 845, 657)


1) 6646 2) 6816 3) 8646 4) 6366
1) (845, 763, 641) 2) (546, 729, 767)
12.
AB
O

368 : 469 : : 486 :


3) (961, 543, 345) 4) (446, 719, 666)
1) 587 2) 580 3) 387 4) 527
ER
CH

13. 2 : 8 : : 3 :
24. (623, 491, 292)
YD

1) 9 2) 27 3) 4 4) 16 1) (813, 262, 614) 2) (264, 384, 354)


3) (761, 165, 791) 4) (144, 169, 441)
,H

14. 13 : 2197 : : 9 :
T

25.
AM

(256, 64, 16)


1) 729 2) 81 3) 60 4) 17
1) (64, 32, 8) 2) (128, 64, 32)
H

15. 2 : 8 : 16 : : 5 : 12 :
M

1) 48 2) 72 3) 60 4) 17 3) (160, 40, 32) 4) (144, 36, 9)


AM
IG

12345678901234567890123456789012
12345678901234567890123456789012123456789012345678901234567890121234567890123456789012345678901212345678901234567890123
12345678901234567890123456789012

q=~¡}ì`ÇàH› ["Œ|°°
12345678901234567890123456789012123456789012345678901234567890121234567890123456789012345678901212345678901234567890123
12345678901234567890123456789012 12345678901234567890123456789012
KH

12345678901234567890123456789012123456789012345678901234567890121234567890123456789012345678901212345678901234567890123
12345678901234567890123456789012
12345678901234567890123456789012 12345678901234567890123456789012
12345678901234567890123456789012123456789012345678901234567890121234567890123456789012345678901212345678901234567890123
12345678901234567890123456789012
12345678901234567890123456789012 12345678901234567890123456789012
12345678901234567890123456789012123456789012345678901234567890121234567890123456789012345678901212345678901234567890123
12345678901234567890123456789012 12345678901234567890123456789012
12345678901234567890123456789012123456789012345678901234567890121234567890123456789012345678901212345678901234567890123
R

12345678901234567890123456789012 12345678901234567890123456789012
12345678901234567890123456789012123456789012345678901234567890121234567890123456789012345678901212345678901234567890123

 4. (1) 13 { 12 { 1 « 1
1. (2) : 64 (1 { 7)2 « 82 « 64
 23 { 22 { 1 « 13
   33 { 32 { 1 « 37 J^Í q^ÎOQê
J^Í q^ÎOQê →  + 
=  = 
 43 { 42 { 1 « 81
2. (2) 15 þ 4 « 60 → 60 Ð 10 « 50 J^Í q^ÎOQê 5. (2) 123 : 36 → (1 { 2 { 3)2«62«36 J^Í q^ÎOQê
25 þ 4 « 100 → 100 Ð 10 « 90 367 → (3 { 6 { 7)2 « (16)2 « 256
3. (3) 53 { 52 { 5 « 125 { 25 { 5 « 155 J^Í 6. (4) 33 Ð 32 « 27 Ð 9 « 18
q^ÎOQê 43 Ð 42 « 64 Ð 16 « 48
103 { 102 { 10 « 1000 { 100 { 10 « 53 Ð 52 « 125 Ð 25 « 100
1110. J^Í q^ÎOQê, 63 Ð 62 « 216 Ð 36 « 180
REASONING ABILITY (VERBAL) ○ ○ ○ ○ ○ ○ ○ ○ ○
81
○ ○ ○ ○ ○ ○ ○ ○ ○ ○ ○ ○ ○ ○ ○ ○ ○ ○ ○
Study Material
7. (1) 13 : 127 → 1333 → 127. J^Í q^ÎOQê 21. (2) (11, 71, 12) ⇒ (12 þ 5) { 11 « 60 { 11
26 → 23 63 → 8216 « 71
8. (2) 36 : 50 64 : z=i ‹¬OY¼ þ 5 { "³ò^Î\ ÷ ‹¬OY¼ « =°^Î¼…Õ
↓ ↓ ↓ ↓
‹¬OY¼
62 72 { 1 82 92 { 1 « 81 { 1
J^Í ^Î~Œàxß ‡\÷‹¬°ë#ß ‹¬=ü‚¬ìO (13, 83, 14)
« 82
2 (14 þ 5) { 13 « 70 { 13 « 83
9. (4) 3 : 10 ⇒ 3 { 1 « 9 { 1 « 10. J^Í q^ÎOQê
52 { 1 « 25 { 1 « 26. 22. (1) (546, 120, 0)
 546 → 5 þ 4 þ 6 « 120 1 þ 2 þ 0 « 0
10. (3) =  ⇒  −  = 
 W^Í q^Î"³°Ø# ^Î~Œàxß H›e¾†Çò#ß ‹¬=ü‚¬ìO (643,

J^Í q^ÎOQê =  ⇒  −  =  72, 14)

Y

11. (4) 7 → 72 « 49 643 → 6 þ 4 þ 3 « 72 → 7 þ 2 « 14

M
49H÷ W~¡°"³á„¬ô…ì 7 LOKÇQê 7497 U~¡æ_È°`Ç°Ok. 23. (4) (761, 845, 657)
761 → 7 { 6 { 1 « 14

E
J^Í q^ÎOQê 6H÷ K͆ǰQê 6366 U~¡æ_È°`Ç°Ok.
12. (1) 368 { 101 « 469 J^Íq^ÎOQê 845 → 8 { 4 { 5 « 17

D
9
59
486 { 101 « 587 657 → 6 { 5 { 7 « 18
CA
95
13. (2) 23 « 8. J^Í q^ÎOQê 33 « 27 W^Í q^ÎOQê JOïH "³ò`ÇëO Q® ‹¬=ü‚¬ìO (446,
14. (1) 133 « 2197. J^Í q^ÎOQê 93 « 729

50
719, 666)
15. (3) 2 þ 8 « 16 J^Í q^ÎOQê 5 þ 12 « 60 98
A
446 → 4 { 4 { 6 « 14
:8
16. (4) 15 → 6 + 9 ; 28→ 15 + 13
719 → 7 { 1 { 9 « 17
E

J^Íq^ÎOQê
h
.P

666 → 6 { 6 { 6 « 18
IC

69 → 60 + 9 ; 82 → 69 + 13
AD

17. (2) 18 → 3 × 6 ; 36 → 18 × 2 24. (1) (623, 491, 292)


AB
O

J^Íq^ÎOQê 623 → 6 þ 2 þ 3 « 36
24 → 4 × 6 ; 48 → 24 × 2
ER
CH

491 → 4 þ 9 þ 1 « 36
18. (3) 14 → 2 × 7 ; 16 → 2 × 8
YD

292 → 2 þ 9 þ 2 « 36
J^Íq^ÎOQê
W…ìO\÷ ^Î~Œàxß H›ey†Çò#ß ‹¬OY¼ ‹¬=ü‚¬ìO
,H

21 → 3 × 7 ; 24 → 3 × 8
T

(813, 262, 614)


AM

19. (3) (62, 49, 36)


H

813 → 8 þ 1 þ 3 « 24
M

 +  
= = 
AM

626 → 2 þ 6 þ 2 « 24
IG

 
z=i ‹¬OY¼ ‹¬Q®@° « =°^Î¼ ‹¬OY¼
KH

614 → 6 þ 1 þ 4 « 24
W^Í ^Î~Œàxß (76, 53, 30) ‡\÷‹¬°ëOk.
R

 
25. (4) (256, 64, 16) → =  = 
 +    
= = 
  J^Í q^Î"³°Ø# ^Î~Œàxß H›ey†Çò#ß ‹¬=ü‚¬ìO
 
20. (1) (112, 56, 14) ⇒ « 56 Ê « 14 (144, 36, 9)
 
J^Í ^Î~Œàxß (144, 72, 18) ‡\÷‹¬°ëOk.  
=  =
   
    
 

REASONING ABILITY (VERBAL) ○ ○ ○ ○ ○ ○ ○ ○ ○


82
○ ○ ○ ○ ○ ○ ○ ○ ○ ○ ○ ○ ○ ○ ○ ○ ○ ○ ○
Study Material
123456789012345678901234567890121234567890
123456789012345678901234567890121234567890123456789012345678901212
12345678901234567890123456789012123456789012345678901234567890121234567890123456789012345678901212345678901234567890123456789012123
12345678901234567890123456789012123456789012345678901234567890121234567890123456789012345678901212345678901234567890123
1234567890123456789012345678901212345678901234567890123456789012123456789012345678901234567890121234567890123
1234567890123456789012345678901212345678901234567890123456789012123456789012345678901234567890121234
123456789012345678901234567890121234567890
123456789012345678901234567890121234567890123456789012345678901212
12345678901234567890123456789012123456789012345678901234567890121234567890123456789012345678901212345678901234567890123456789012123
12345678901234567890123456789012123456789012345678901234567890121234567890123456789012345678901212345678901234567890123
1234567890123456789012345678901212345678901234567890123456789012123456789012345678901234567890121234567890123
1234567890123456789012345678901212345678901234567890123456789012123456789012345678901234567890121234
12345678901234567890123456789012123456789012345678901234567890121234567890123456789012345678901212345678901234567890123456789012123
123456789012345678901234567890121234567890123456789012345678901212
12345678901234567890123456789012123456789012345678901234567890121234567890123456789012345678901212345678901234567890123
1234567890123456789012345678901212345678901234567890123456789012123456789012345678901234567890121234567890123
1234567890123456789012345678901212345678901234567890123456789012123456789012345678901234567890121234
123456789012345678901234567890121234567890
12345678901234567890123456789012123456789012345678901234567890121234567890123456789012345678901212345678901234567890123456789012123
PREVIOUS BITS on TYPE - II
123456789012345678901234567890121234567890123456789012345678901212
12345678901234567890123456789012123456789012345678901234567890121234567890123456789012345678901212345678901234567890123
1234567890123456789012345678901212345678901234567890123456789012123456789012345678901234567890121234567890123
1234567890123456789012345678901212345678901234567890123456789012123456789012345678901234567890121234
12345678901234567890123456789012123456789012345678901234567890121234567890123456789012345678901212345678901234567890123456789012123
12345678901234567890123456789012123456789012345678901234567890121234567890123456789012345678901212345678901234567890123
1234567890123456789012345678901212345678901234567890123456789012123456789012345678901234567890121234567890123
1234567890123456789012345678901212345678901234567890123456789012123456789012345678901234567890121234
12345678901234567890123456789012123456789012345678901234567890121234567890123456789012345678901212345678901234567890123456789012123
123456789012345678901234567890121234567890123456789012345678901212
12345678901234567890123456789012123456789012345678901234567890121234567890123456789012345678901212345678901234567890123456789012123
12345678901234567890123456789012123456789012345678901234567890121234567890123456789012345678901212345678901234567890123
1234567890123456789012345678901212345678901234567890123456789012123456789012345678901234567890121234567890123
1234567890123456789012345678901212345678901234567890123456789012123456789012345678901234567890121234
12345678901234567890123456789012123456789012345678901234567890121234567890123456789012345678901212345678901234567890123456789012123

1. 2 : 9 : 64 : : : : (7-10) : D ãH÷Ok =~¡°‹¬…Õx Mìm#° „¬îiO„¬ô=ò?


[Asst. Motor Veh. Inspectors - 2013] (Drug Inspectors-2012)

1) 6 : 25 : 49 2) 5 : 25 : 125 7. 5:12:19:: ___: ___: ____


3) 3 : 16 : 216 4) 4 : 25 : 216 1) 10:17:24 2) 7:14:28
2. 25 : 625 : : 35 :
3) 4:10:17 4) 9:18:45
[Polytechnic lecturers - '11]
8. 5:24::10: ........
1) 1225 2) 1525 3) 1725 4) 1425
1) 44 2) 99 3) 56 4) 57
3.

Y
8 : 20 : : 14 : [Constables - 2012]

1) 30 2) 25 3) 35 4) 28 9. 42:115 :: 53 : .....

M
4. 18 : 342 : : 21 : [S.I. - 2011]
1) 126 2) 124 3) 150 4) 114
10.

E
1) 441 2) 462 3) 420 4) 425 182: ___ :: 210 : 380

D
1) 352 2) 272 3) 240 4) 156

9
(5-7) : D ãH÷Ok =~¡°‹¬…Õx Mìm#° „¬îiO„¬ô=ò?

59
11. 1:2:9 :: 2:4:?
CA
95
(Polytechnic Lectures-2012)
(Warders Grade - I&II - 2012)

5. 4:32::10:? 1) 20 2) 36 3) 24 4) 20

50
12. 1:8::?:125
98
A
1) 200 2) 600 3) 700 4) 800 (Polytechnic Lectures-2012)
:8
6. 3:10:28::..... : ..... : ...... 1) 10 2) 16 3) 25 4) 15
E
h

1) 7:50:344 2) 5:16:31 13. 25:81:: 36:?


.P

(PL-2012)
IC

3) 6:36:216 4) 5:80:390 1) 103 2) 65 3) 93 4) 121


AD
AB
O

12345678901234567890123456789012
12345678901234567890123456789012123456789012345678901234567890121234567890123456789012345678901212345678901234567890123
12345678901234567890123456789012

q=~¡}ì`ÇàH› ["Œ|°°
12345678901234567890123456789012123456789012345678901234567890121234567890123456789012345678901212345678901234567890123
12345678901234567890123456789012
12345678901234567890123456789012 12345678901234567890123456789012
12345678901234567890123456789012123456789012345678901234567890121234567890123456789012345678901212345678901234567890123
12345678901234567890123456789012
12345678901234567890123456789012 12345678901234567890123456789012
12345678901234567890123456789012123456789012345678901234567890121234567890123456789012345678901212345678901234567890123
ER
CH

12345678901234567890123456789012
12345678901234567890123456789012123456789012345678901234567890121234567890123456789012345678901212345678901234567890123
12345678901234567890123456789012 12345678901234567890123456789012
12345678901234567890123456789012123456789012345678901234567890121234567890123456789012345678901212345678901234567890123
12345678901234567890123456789012 12345678901234567890123456789012
12345678901234567890123456789012
12345678901234567890123456789012123456789012345678901234567890121234567890123456789012345678901212345678901234567890123
YD

1. (4) 21 = 2 ; 32 = 9 ; 43 = 64 6. (1) 3 × 10 – 2 = 28 then 7 × 50 – 6 = 344.


,H
T

J^Í ^Î~Œàxß H›e¾†Çò#ß ‹¬=ü‚¬ìO 7. 5 + 7 = 12 + 7 = 19 then 10 + 7 = 17 + 7 = 24


AM

4 : 25 : 216 (1)
H

41 = 4 ; 52 = 25 ; 63 = 216 8. 52 – 1 = 29 J…ìöQ 102 – 1 = 99


M

(2)
AM
IG

2. (1) 25 : 625 ⇒ 252 = 625 9. (3) (42 × 3) – 9 = 115 J…ìöQ (53 × 3) – 9 = 150
KH

J^Í q^ÎOQê 352 « 1225 10. (1) 182 + 170 = 352


R

 11. (1 + 3)2 = 9 then (2 + 4)2 = 36


3. (3) 8 : 20 → × 5 → 4 × 5 = 20 J^Í q^ÎOQê (2)

 12. (2) 12 ; 23 → 1:8
×  →  × = 
 J…ìöQ 42 ; 53 → 16 : 125
2
4. (2) 18 : 342 ⇒ 18 { 18 « 324 { 18 « 342 13. (4) 52 = 25 ; (5+4)2 = 92 = 81
2
J^Í q^ÎOQê, 21 { 21 « 441 { 21 « 462 =°i†Çò
5. (1)
62 = 36 ; (6+5)2 = 112 = 121

REASONING ABILITY (VERBAL) ○ ○ ○ ○ ○ ○ ○ ○ ○


83
○ ○ ○ ○ ○ ○ ○ ○ ○ ○ ○ ○ ○ ○ ○ ○ ○ ○ ○
Study Material
TYPE - III : P…ì权ÿ\˜ J<Œl
; ‹¬=¼k‰×…Õ POQ®Á =~¡â=¶ #O|iOQ· (A to Z)

Z_È=° 1 2 3 4 5 6 7 8 9 10 11 12 13 14 15 16 17 18 19 20 21 22 23 24 25 26 䛽_
#°O_ A B C D E F G H I J K L M N O P Q R S T U V W X Y Z #°O_
→ 26 25 24 23 22 21 20 19 18 17 16 15 14 13 12 11 10 9 8 7 6 5 4 3 2 1 ←

; J„¬‹¬=¼k‰×…Õ POQ®Á =~¡â=¶ #O|iOQ· (Z to A)

Z_È=° 1 2 3 4 5 6 7 8 9 10 11 12 13 14 15 16 17 18 19 20 21 22 23 24 25 26 䛽_
#°O_ Z Y X W V U T S R Q P O N M L K J I H G F E D C B A #°O_

Y
→ 26 25 24 23 22 21 20 19 18 17 16 15 14 13 12 11 10 9 8 7 6 5 4 3 2 1 ←
123456789012345678901234567890121234567890
123456789012345678901234567890121234567890

M
123456789012345678901234567890121234567890123456789012345678901212
1234567890123456789012345678901212345678901234567890123456789012123456789012345678901234567890121234567890123
1234567890123456789012345678901212345678901234567890123456789012123456789012345678901234567890121234
12345678901234567890123456789012123456789012345678901234567890121234567890123456789012345678901212345678901234567890123456789012123
12345678901234567890123456789012123456789012345678901234567890121234567890123456789012345678901212345678901234567890123
1234567890123456789012345678901212345678901234567890123456789012123456789012345678901234567890121234567890123
1234567890123456789012345678901212345678901234567890123456789012123456789012345678901234567890121234
123456789012345678901234567890121234567890
123456789012345678901234567890121234567890123456789012345678901212
P…ìæ ƒÿ\˜ J<Œl : TYPE - III : PRACTICE TEST
12345678901234567890123456789012123456789012345678901234567890121234567890123456789012345678901212345678901234567890123456789012123
12345678901234567890123456789012123456789012345678901234567890121234567890123456789012345678901212345678901234567890123
1234567890123456789012345678901212345678901234567890123456789012123456789012345678901234567890121234567890123
1234567890123456789012345678901212345678901234567890123456789012123456789012345678901234567890121234
12345678901234567890123456789012123456789012345678901234567890121234567890123456789012345678901212345678901234567890123456789012123
123456789012345678901234567890121234567890123456789012345678901212
12345678901234567890123456789012123456789012345678901234567890121234567890123456789012345678901212345678901234567890123
1234567890123456789012345678901212345678901234567890123456789012123456789012345678901234567890121234567890123
1234567890123456789012345678901212345678901234567890123456789012123456789012345678901234567890121234
123456789012345678901234567890121234567890
12345678901234567890123456789012123456789012345678901234567890121234567890123456789012345678901212345678901234567890123456789012123
12345678901234567890123456789012123456789012345678901234567890121234567890123456789012345678901212345678901234567890123
1234567890123456789012345678901212345678901234567890123456789012123456789012345678901234567890121234567890123
1234567890123456789012345678901212345678901234567890123456789012123456789012345678901234567890121234
123456789012345678901234567890121234567890123456789012345678901212
12345678901234567890123456789012123456789012345678901234567890121234567890123456789012345678901212345678901234567890123456789012123
12345678901234567890123456789012123456789012345678901234567890121234567890123456789012345678901212345678901234567890123
1234567890123456789012345678901212345678901234567890123456789012123456789012345678901234567890121234567890123
1234567890123456789012345678901212345678901234567890123456789012123456789012345678901234567890121234
123456789012345678901234567890121234567890
12345678901234567890123456789012123456789012345678901234567890121234567890123456789012345678901212345678901234567890123456789012123
123456789012345678901234567890121234567890123456789012345678901212

E
12345678901234567890123456789012123456789012345678901234567890121234567890123456789012345678901212345678901234567890123
1234567890123456789012345678901212345678901234567890123456789012123456789012345678901234567890121234567890123
1234567890123456789012345678901212345678901234567890123456789012123456789012345678901234567890121234
12345678901234567890123456789012123456789012345678901234567890121234567890123456789012345678901212345678901234567890123456789012123
123456789012345678901234567890121234567890123456789012345678901212
12345678901234567890123456789012123456789012345678901234567890121234567890123456789012345678901212345678901234567890123456789012123
12345678901234567890123456789012123456789012345678901234567890121234567890123456789012345678901212345678901234567890123
1234567890123456789012345678901212345678901234567890123456789012123456789012345678901234567890121234567890123
1234567890123456789012345678901212345678901234567890123456789012123456789012345678901234567890121234

D
9
1. 12. TM : 33 : : GP : ?

59
EFG : HIJ : : MNO :
1) 27 2) 23 3) 25 4) 31
CA
95
(Medical Officers – 2013)

1) RST 2) QRS 3) PQR 4) TUV 13. GERM : MERT : : STAR :

50
2. PIN : NIP : : TEN :
1) RTAS 2) RATS 3) RSTA 4) RAST
98
A
(Hostel Welfare Officers – 2010)
14. GKC : TPX : : JUM :
:8
1) NTE 2) NET 3) TNE 4) TIN 1) FNQ 2) QXN 3) QFN 4) QMN
E

15.
h

" # $ % METRE : JBQOB : : MILLI :


.P

3.     1) JFIIF 2) KNIFE 3) KNIFF 4) JEIIF


IC

,
AD

(Hostel Welfare Officers – 2010)


16. MQMR : KSKT : : SRKT :

1) TIQV 2) QITV 3) QUIT 4) QTIV


AB
O

& ' & ' & ' & '


1) 2) 3) 4) 17. FIAT : GHJKBCUV : LION :
ER

       
CH

4. 1) MNJKPOOP 2) MNJKPQOP
YD

VT : QO : MK : (Bank Cl – 2011)
3) MNJKPOOQ 4) NMJKPQOR
1) HF 2) IJ 3) HG 4) IK
18.
,H

PL : 16 : : XQ :
5. KILM : MILK : : EAKM :
T

1) 64 2) 45 3) 36 4) 49
AM

(Jr. Lecturers – 2011)


19.
H

MM : 27 : : NN :
M

1) KAME 2) MEAK 3) MAKE 4) KEAM 1) 26 2) 27 3) 25 4) 24


AM

6.
IG

SONG : GONT : : FELT : (S.I – 2011) 20. S : 361 : : P :


KH

1) GELT 2) TLEG 3) TELG 4) LEGT 1) 279 2) 324 3) 256 4) 196


7. 21.
R

PENCIL : NEPLIC : : MANGET : (S.I – 2011) BD : T : : EG :


1) MAGNET 2) GAMNET 1) X 2) U 3) Z 4) V
3) NETGAM 4) NAMTEG 22. DE : 20 : : HI :
8. DEAR : FGCT : : READ : (S.I – 2011) 1) 64 2) 56 3) 48 4) 72
1) TSFG 2) GTFC 3) TGEF 4) TGCF 23. ABCD : WXYZ : : EFGH : (S.I – 2008)

9. CFDG : LOMP : : HKIL : (S.I – 2011) 1) MNOP 2) UVWX 3) STUV 4) PQRS


1) QTRU 2) QRTU 3) PSQT 4) PQST 24. ZA : YB :: XC : (S.I – 2008)

10. TGIR : QJLO : : PKMN : (S.I – 2011) 1) WD 2) XD 3) WC 4) XA


1) NMPQ 2) MNPQ 3) MNPK 4) PKMN 24. ZA : YB :: XC : (S.I – 2008)

11. TAME : OVHZ : : LUDO : (S.I – 2011)


1) WD 2) XD 3) WC 4) XA
1) GQAM 2) GPYJ 3) QUIZ 4) ZIQU
25. AD : BE : : CF : (S.I – 2008)

1) EG 2) FG 3) DF 4) DG
REASONING ABILITY (VERBAL) ○ ○ ○ ○ ○ ○ ○ ○ ○
84
○ ○ ○ ○ ○ ○ ○ ○ ○ ○ ○ ○ ○ ○ ○ ○ ○ ○ ○
Study Material
12345678901234567890123456789012
12345678901234567890123456789012123456789012345678901234567890121234567890123456789012345678901212345678901234567890123
12345678901234567890123456789012

q=~¡}ì`ÇàH› ["Œ|°°
12345678901234567890123456789012123456789012345678901234567890121234567890123456789012345678901212345678901234567890123
12345678901234567890123456789012 12345678901234567890123456789012
12345678901234567890123456789012 12345678901234567890123456789012
12345678901234567890123456789012123456789012345678901234567890121234567890123456789012345678901212345678901234567890123
12345678901234567890123456789012
12345678901234567890123456789012123456789012345678901234567890121234567890123456789012345678901212345678901234567890123
12345678901234567890123456789012
12345678901234567890123456789012
12345678901234567890123456789012123456789012345678901234567890121234567890123456789012345678901212345678901234567890123
12345678901234567890123456789012
12345678901234567890123456789012
12345678901234567890123456789012123456789012345678901234567890121234567890123456789012345678901212345678901234567890123
12345678901234567890123456789012
12345678901234567890123456789012123456789012345678901234567890121234567890123456789012345678901212345678901234567890123
12345678901234567890123456789012 12345678901234567890123456789012

1. (3) EFG : HIJ (E, F, G, H, I, J ° =~¡°‹¬ JH›Æ~Œ°) 8. (4) D E A R

J^Í q^ÎOQê ./0 1 2 3 +2 +2 +2 +2 J^Í q^ÎOQê

(Q M, N, O, P, Q, R °=~¡°‹¬ JH›Æ~Œ°) F G C T
2. (2) PIN #° =¼uö~H› k‰×…Õ ~Œ†Ç°Qê NIP U~¡æ_È°#°. R E A D
J^Í q^ÎOQê +2 +2 +2 +2
TEN #° =¼uö~H›k‰×…Õ ~Œ†Ç°Qê NET U~¡æ_È°#°.
T G C F
" # $ % & '

Y
3. (4)
      ∴ Hê=‹²# JH›Æ~¡ ‹¬=ü‚¬ìO 5($'

M
9. (1) C F D G
7 5 . ,

E
4. (1) − ↓ − ↓ J^Í q^ÎOQê − ↓  − ↓  +9 +9 +9 +9 J^Í q^ÎOQê
) '

D
9
2 0

59
L O M P

CA
95
∴ Hê=‹²# JH›Æ~¡ ‹¬=ü‚¬ìO )' H K I L

50
, * - . J^Í q^ÎOQê & " , . +9 +9 +9 +9
5. (3)
98
A
:8
Q T R U
. * - ,
E

. " , , 2536
h

∴ Hê=‹²# JH›Æ~¡ ‹¬=ü‚¬ìO


.P

.",&
IC

∴ Hê=‹²# JH›Æ~¡ ‹¬=ü‚¬ìO 10. (3) T G I R


AD

4 0 / ( –3 +3 +3 –3 J^Í q^ÎOQê
AB
O

6. (3) (S `Ç~¡°"Œ`Ç JH›Æ~¡O)


ER
CH

Q J L O
( 0 / 5
YD

P K M N
J^Íq^ÎOQê
,H

–3 +3 +3 –3
T

' & , 5
AM
H

(F `Ç~¡°"Œ`Ç JH›Æ~¡O) M N P K
M

./1,
AM

5 & - ( ∴ Hê=‹²# JH›Æ~¡ ‹¬=ü‚¬ìO


IG
KH

∴ Hê=‹²# JH›Æ~¡ ‹¬=ü‚¬ìO 5&-( 11. (2) T A M E


R

7. (4) 1 & / $ * - –5 –5 –5 –5 J^Í q^ÎOQê

O V H Z
/ & 1 - * $ L U D O
. " / ( & 5
G P Y J
/ " . 5 & ( ∴ Hê=‹²# JH›Æ~¡ ‹¬=ü‚¬ìO (1:+
∴ Hê=‹²# JH›Æ~¡ ‹¬=ü‚¬ìO /".5&( 12. (2) TM : 33 → T(20) M(13) → 20 + 13 = 33
J^Í q^ÎOQê
GP → G(7) P(16) → 7 + 16 = 

REASONING ABILITY (VERBAL) ○ ○ ○ ○ ○ ○ ○ ○ ○


85
○ ○ ○ ○ ○ ○ ○ ○ ○ ○ ○ ○ ○ ○ ○ ○ ○ ○ ○
Study Material
13. (1) GERM : MERG 17. (2) F, I, A, T  `Ç~¡°"Œ`Ç =KÇ°ó ï~O_È JH›Æ~Œ° =~¡°‹¬Qê
"³ ò ^Î \ ÷ JH› Æ ~ ¡ ‹¬ = ü‚¬ ì O…Õx z=i JH› Æ ~ Œ° GH, JK, BC, UV → GHJKBCUV
`Œ~¡°=¶~¡° K͆ǰ|_¨Û~ò. L, I, O, N  `Ç~¡°"Œ`Ç =KÇ°ó ï~O_È° JH›Æ~Œ° =~¡°‹¬Qê
MN, JK, PQ, OP → MNJKPQOP
GERM :MERG
18. (4) PL : 16 → P(16) L(12) → (16 – 12)2 → 42 = 16
J^Í q^ÎOQê S T A R …Õ S, R  ªÖ<Œ° `Œ~¡°=¶~¡°
XQ → X(24) Q(17) → (24 – 17)2 → 72 = 
K͆ǰQê
19. (2) MM → 27
35"4 U~¡æ_È°`Ç°Ok.
M q°= ‹¬i†³Ø°# k‰×…Õ 13
14. (3)
M q°= =¼uö~H› k‰×…Õ 14
" # $ % & ' ( ) * + , - . 13 + 14 = 27

Y
; : 9 8 7 6 5 4 3 2 1 0 / J^Í q^ÎOQê NN → 14 + 13 = 27

M
"³ò^Î\ ÷ 13 JH›Æ~Œ° ‹¬i†³Ø°# k‰×…Õ =°i†Çò 20. (3) S : 361 → S(19) → 192 = 361
q°ye# 13 JH›Æ~Œ° =¼uö~H› k‰×…Õ ~Œ†Ç°Qê ̄á# P(16) → 162 = 

E
KǶ„¬|_# „¬\ ÷“H›…Õ =ô#ß@°Á JH›Æ~Œ° =ôO\ì~ò.
21. (4) BD → B(2) D(4) → 22 + 42 → 4 + 16 = 20 (T)

D
9
∴ ‹¬=¶^¥#O 4) a

59
J^Í q^ÎOQê
CA
95
G K C J U M
EG → E(5) G(7) → 52 + 72 → 25 + 49 → 74

50
J^Í q^OÎ Qê 
98
A
→  7


:8
T P X Q F N
22. (4) DE → D(4) E(5) → 4 × 5 = 20
E
h

∴ Hê=‹²# JH›Æ~¡ ‹¬=ü‚¬ìO 2'/ J^Í q^ÎOQê HI → H (8) I (9) → 8 × 9 = 72


.P
IC

15. (1) M E T R E 23. (3)


AD

" # $ % & ' ( ) * + , - .


AB
O

–3 –3 –3 –3 –3
; : 9 8 7 6 5 4 3 2 1 0 /
ER
CH

J B Q O B "³ò^Î\ ÷ 13 JH›Æ~Œ#° ‹¬i†³Ø°# k‰×…Õ, `Ç~¡°"Œ`Ç


YD

J^Í q^ÎOQê
13 JH›Æ~Œ#° =¼uö~H› k‰×…Õ ~Œ‹² „¬ijeOKÇQê
,H

M I L L I
" # $ %
T

& ' ( )
AM

–3 –3 –3 –3 –3 ↓ ↓ J^Í q^ÎOQê ↓ ↓ ↓ ↓
↓ ↓
H
M

;') :') ')9 ')


8) 7
') 6
') 5
') 4'
AM
IG

J F I I F
∴ Hê=‹²# JH›Æ~¡ ‹¬=ü‚¬ìO 4567
KH

∴ Hê=‹²# JH›Æ~¡ ‹¬=ü‚¬ìO +'**'


R

16. (4) M Q M R 24. (1) ; "


: #
–2 +2 –2 +2 J^Í q^OÎ Qê
9 $
K S K T 8 %
S R K T 25. (4) " %
–2 +2 –2 +2
# &
$ '
Q T I V % (
∴ Hê=‹²# JH›Æ~¡ ‹¬=ü‚¬ìO 25*7

REASONING ABILITY (VERBAL) ○ ○ ○ ○ ○ ○ ○ ○ ○


86
○ ○ ○ ○ ○ ○ ○ ○ ○ ○ ○ ○ ○ ○ ○ ○ ○ ○ ○
Study Material
123456789012345678901234567890121234567890
123456789012345678901234567890121234567890123456789012345678901212
12345678901234567890123456789012123456789012345678901234567890121234567890123456789012345678901212345678901234567890123456789012123
12345678901234567890123456789012123456789012345678901234567890121234567890123456789012345678901212345678901234567890123
1234567890123456789012345678901212345678901234567890123456789012123456789012345678901234567890121234567890123
1234567890123456789012345678901212345678901234567890123456789012123456789012345678901234567890121234
123456789012345678901234567890121234567890
123456789012345678901234567890121234567890123456789012345678901212
12345678901234567890123456789012123456789012345678901234567890121234567890123456789012345678901212345678901234567890123456789012123
12345678901234567890123456789012123456789012345678901234567890121234567890123456789012345678901212345678901234567890123
1234567890123456789012345678901212345678901234567890123456789012123456789012345678901234567890121234567890123
1234567890123456789012345678901212345678901234567890123456789012123456789012345678901234567890121234
12345678901234567890123456789012123456789012345678901234567890121234567890123456789012345678901212345678901234567890123456789012123
123456789012345678901234567890121234567890123456789012345678901212
12345678901234567890123456789012123456789012345678901234567890121234567890123456789012345678901212345678901234567890123
1234567890123456789012345678901212345678901234567890123456789012123456789012345678901234567890121234567890123
1234567890123456789012345678901212345678901234567890123456789012123456789012345678901234567890121234
123456789012345678901234567890121234567890
12345678901234567890123456789012123456789012345678901234567890121234567890123456789012345678901212345678901234567890123456789012123
PREVIOUS BITS on all Types (Remaining)
123456789012345678901234567890121234567890123456789012345678901212
12345678901234567890123456789012123456789012345678901234567890121234567890123456789012345678901212345678901234567890123
1234567890123456789012345678901212345678901234567890123456789012123456789012345678901234567890121234567890123
1234567890123456789012345678901212345678901234567890123456789012123456789012345678901234567890121234
12345678901234567890123456789012123456789012345678901234567890121234567890123456789012345678901212345678901234567890123456789012123
12345678901234567890123456789012123456789012345678901234567890121234567890123456789012345678901212345678901234567890123
1234567890123456789012345678901212345678901234567890123456789012123456789012345678901234567890121234567890123
1234567890123456789012345678901212345678901234567890123456789012123456789012345678901234567890121234
12345678901234567890123456789012123456789012345678901234567890121234567890123456789012345678901212345678901234567890123456789012123
123456789012345678901234567890121234567890123456789012345678901212
12345678901234567890123456789012123456789012345678901234567890121234567890123456789012345678901212345678901234567890123456789012123
12345678901234567890123456789012123456789012345678901234567890121234567890123456789012345678901212345678901234567890123
1234567890123456789012345678901212345678901234567890123456789012123456789012345678901234567890121234567890123
1234567890123456789012345678901212345678901234567890123456789012123456789012345678901234567890121234
12345678901234567890123456789012123456789012345678901234567890121234567890123456789012345678901212345678901234567890123456789012123

1. 7544 : 5322 :: 4673 : ... (Deputy Jailors - 2017) 13. 11 : 123 : : 23 : ....... (AP. SI. Prelims - 2016)

1) 2367 2) 2451 3) 2531 4) 4472 1) 531 2) 529 3) 527 4) 523


2. 'Bird', 'Flock' H÷ Z…ìQË 'Subject' ãH÷Ok^¥xH÷ 14. 97 : 89 :: 43 : ........ (AP. SI. Prelims - 2016)

J…ìQ®? (AP. Panchyat Secretary Screening -2016) 1) 31 2) 37 3) 40 4) 41


1) Object 2) Story 15.        (AP. SI. Prelims - 2016)

3) Populace 4) Matter 1) 8 2) 12 3) 24 4) 
3. =¶#= „¬i}ì=°‰§GO : =¶#=ô_È° : †Ç¶O`Œr : 16. 0 : .... :: ̋ež†Ç°‹¹ : ïHeÞ<£ (AP. SI. Prelims - 2016)
...... ? (TS. SI. Mains - 2016) 1) 32 2) 180 3) 212 4) 273
1) ㄬH›$u 2) K³@°Á 17. 7544 : 5322 :: 4673 : ... (Deputy Jailors - 2017)

Y
3) "Œ#~Œ° 4) „¬^¥¼° 1) 2367 2) 2451 3) 2531 4) 4472

M
4. =òO^ΰ *ìãQ®`Çë : ㄬ=¶^ÎO : : ...... : ~ËQꁰ 18. 7528 J<Í ‹¬OY¼ä›½ 5306 ‹¬O|OkOz# su…Õ 4673䛽
‹¬O|OkOz# ‹¬OY¼

E
(TS. SI. Mains - 2016) (TS. SI. Mains - 2016)

1) "³á^ΰ¼_È° 2) ~Ëy 3) ‰×Ãハ’`Ç 4) =°O^ΰ 1) 2367 2) 2451 3) 2531 4) 2641

D
9
5. KÇ`°Ç ~¡ã‹¬O : „¦°¬ #O :: "³‰á §¼O...(TS. SI. Mains - 2016) 19. 25 : 125 :: 49 : .....

59
(TS. SI. Mains - 2016)

CA
95
1) ƒ’°[O 2) qH›~¡â=ò 1) 206 2) 180 3) 216 4) 343
3) „¦¬°#„¬i=¶}O 4) KÇ°@°“Hù`Ç 20. 34 : 12 :: 59 : ......

50
(TS. SI. Mains - 2016)

6. ƒèï~…˜ : =†Ç°…˜ :: ...... : .... (AP. SI. Prelims - 2016) 1) 14 2) 47 3) 38 4) 45


98
A
1) ‘é"³…˜ : ‚¬ìÙ 2) H© : _Ë~Ÿ ‹¬¶KÇ# (21-28) : ãH÷O^Î L#ß ã„¬‰ß× ° ãH÷O^Î ~òzó# „¬@=ò…Õx
:8
E

3) "Œ¶¼"£° : "³¶<ËãQꄦ¹ 4) ‚¬ìh : q°…˜ø JH›Æ~ŒÌ„á P^¥~¡„¬_ L<Œß~ò. ㄬu ㄬ‰×߅Õ#° :: H÷


h
.P

7. „¦¬î¼†Ç°~Ÿ :: ...... :: HÒO@|°…˜ : J<£HÒO@|°…˜ Z_È=° ㄬH›ø ï~O_È° „¬^¥°, 䛽_ ㄬH›ø ï~O_È° „¬^¥°
IC
AD

(AP. SI. Prelims - 2016) LO_¨e. 䛽_ã„¬H›ø# L#ß „¬^¥ =°^Î¼ ‹¬O|O^ÎO, Z_È=°
1) W"Í°l#i 2) \Õ@…˜ 3) …ÿ‹¹ 4) ‹¬…˜ø ㄬH›ø# L#ß „¬^¥ =°^Î¼ ‹¬O|O^ÎO XH›…ìöQ LO@°Ok.
AB
O

(8-10) : =~¡ä›½ Q® ㄬ‰×߁䛽 xö~í‰×Hꁰ : ㄬu [`Ç =°^Î¼ XöH ãH÷Ok ~òzó# <Œ°Q®° „¬^¥…Õ ㄬ‰§ß~¡œH›O (?) Q®
ER
CH

~¡H›"³°Ø# ‹¬O|O^¥xß U~¡æ~¡KÍ „¬^ÎO („¬^¥`Ë)`Ë ªÖ#O…Õ ‹¬~¡~ò# ‹¬=¶^¥<Œxß Z#°ßHËO_.


YD

„¬îiOKÇ°=ò ? (A.P. S.I. Mains 2017) C (Dy.Jailors-2012)


A
,H

8. \ì_£‡é…˜ : ? : : Hê@~Ÿ „²Á~Ÿ : |@~Ÿ ̄¦åÁ


B
T
AM

1) ãHË 2) Q®¶‹¹ 3) „¦²+¹ 4) ㇦Q·


J K D
H

9. WO_ù…ÿ<£ž : c=~Ÿ :: ? : ?
M

O M
AM

1) ZeQê<£ž, „²HêH± 2) ̄¦~ˋ²\ ÷, …ì"£°ž P


IG

I E
H N F
3) ‡‹²e\÷, HÒ 4) *Õq†Ç¶e\÷, ̂áì<Œ
KH

10. A : qO@~Ÿ   B : =°…èi†Ç¶


R

G
1) A = H÷…˜Û, B = H÷Þ<³á<£ 2) A = H÷…˜Û, B = Z„²_³q°H± 21. AKJ : GNH :: EMD :?
3) A = H˅˜Û, B = =°‹Ôø\Õ 4) A = T…˜, B = „¦Ô#~Ÿ 1) CLB 2) AKB 3) CLD 4) EMF
11. =¶¼Q®r<£ : ªé“s : Pi“H›…˜ :: .... : .... : .... 22. HNP:PDA:: DLP:?
(A.P. S.I. Mains 2017) 1) CDP 2) PME 3) CLP 4) PJG
1) \© : q°…˜ø : +¬µQ®~Ÿ 23. AFHO : GBDM :: CHFM :?
2) >ÿeq[<£ : #¶¼‹¹ À„„¬~Ÿ : ZO@~Ÿ>ÿØ<£"³°O\˜ 1) GBLD 2) GBDM 3) GBO 4) GPLD
3) ƒÿ_£ : H÷ޅ˜Û : „²…ÕÁ 24. AKJO : IOHN :: CLBK :?
4) <Ë=…˜ : ã_¨=¶ : e@ö~KÇ~Ÿ 1) EMGH 2) EMDL 3) LDME 4) GNFM
12. Q®[O : JOQ®°ˆ×O :: HêÞ~Ÿ“ : ... (TS. SI. Mains - 2016) 25. AOE: ? :: GMA : NKM
1) Q꼁<£ 2) *º#°ž 3) ‡° 4) ã^Î=O 1) KMN 2) KLM 3) KLF 4) KLO

REASONING ABILITY (VERBAL) ○ ○ ○ ○ ○ ○ ○ ○ ○


87
○ ○ ○ ○ ○ ○ ○ ○ ○ ○ ○ ○ ○ ○ ○ ○ ○ ○ ○
Study Material
26. MHUSMD : LITTLE : NTUD :? 37. J_È°Q®° : „¦¬¶° « KÍ`Ç°° : .......
(Polytechnic Lectures-2012) 1) ã"͈×ä 2) ƒ’¶[=ò°
1) MOE 2) MUTE 3) MITE 4) MATE 3) `ù_È°Q®°° 4) KÍu Q®_†Ç¶~¡O
27. EFG : HIJ :: MNO :? (Sr.Entomologist-2012)
38. #k : P#H›@“ « ã\섦²H± : .......
1) PQR 2) RQP 3) QRP 4) QRJ 1) ‹²Q®ß…˜ 2) …è<£
28. GITA : HINDUISM :: QURAN:?
3) "Œ‚¬ì#=ò 4) H›^ÎeH›
(Sr.Entomologist-2012)
39. [†Ç°=ò: ‹¬O`Ë+¬=ò « J„¬[†Ç°=ò :.....
1) ISLAM 2) BHAGVAD GITA
1) F_‡é=ô@ 2) ƒ’OQ®‡@°
3) MOKSHA 4) ALLAH
3) ^ΰ—Y=ò 4) H˄¬=ò
‹¬¶KÇ#° (29-32) : =~¡ä›½ ï~O_È° „¬^¥°, XH› ‹¬O|O^ÎO`Ë
[`DŽ¬~¡KÇ|_#k. J^Í ‹¬O|O^ÎO H›ey L#ß „¬^¥ 40. ZQ®~¡°@ : „¬H÷Æ « ㇐䛽@:.....
[`Ç#° H›#°HËøO_. 1) „¬ô=ôÞ 2) #`Çë

Y
(S.I.-2011)

29. BODY : HAND :: 3) q=¶#=ò 4) <́

M
1) Pin : Nail 2) Chair : Table 41. „²_È°Q®° : "Œ# « ~Œãu :.....
3) Automobile:Wheel 4) Thorn : Flower 1) ª†Ç°OHêO 2) „¬Q®°

E
5) UnHê^ΰ 3) pH›\ ÷ 4) ‹¬O^¥¼Hê=ò

D
9
30. 42.

59
PISTOL : TRIGGER :: Cow : AMU = Dog : .....
1) Sword: scabbard 2) Gun:Holster
CA
95
1) BME 2) BNH 3) GOD 4) EPH
3) Motor:Switch 4) Rifle:Solder
xö~œ‰×H›=ò° (ㄬ‰×߁° 43-46 =~¡ä›½) : H÷O^Î À„~ùø#ß ã„¬‰×ß

50
5) None of these (UnHê^ΰ)
98
A
XH› ‹¬O|Ok`Ç „¬^Î=ò [`Ç <Œ°Q®° [`ǁ „¬^Î=ò`Ë
31. RADIUM : CURIE ::
:8
"³#°"³O@ H›ey LO_È°#°. W=Þ=‹²# „¬^Î=òä›½
1) Museum:Artefalt 2) Atom:Galileo
E

^¥^¥„¬ô ‹¬g°„¬=òQê ‹¬O|O^Î=ò#° "³e„¬~¡°KÇ° [`Ç#°


h

3) Telephone:bell 4) Drama:Shakespeare
.P
IC

5) None of these (UnHê^ΰ) ZxßH› K͆Çò=ò? (H›=ü¼xöH+¬<£ S.I-2011)


AD

32. OCEAN : SALINE :: 43. =ã_ÈOy : ~¡O„¬=ò :: .....


AB
O

1) Honey:Bee 2) Sugar:Sweet 1) a_ÈÛ : ƒç=°à 2) ~¡KÇ~ò`Ç : H›=ò


ER
CH

3) Rose:Red 4) Heaven : Paradise 3) >ÿ؄²‹¹“ : Hêy`Ç=ò 4) "Í°„¦¬°=ò : "Œ#


44. ‹¬=ü‚¬ì=ò : `Œˆ×=ò :: ..... : .......
YD

5) None of these (UnHê^ΰ)


P^͉×=ò° (ㄬ‰×߁° 33 #°O_ 42 =~¡ä›½) : W=Þ|_# =ü_È° 1) „¬îQ®°uë:„¬î=ô 2) [@°“:‡é\©
,H
T

„¬ ^ Î = ò…Õ, "³ ò ^Î \ ÷ ï ~ O_È ¶ U^Ë XH› q^ Î = ò 3) `Í<³Q®¶_È°:DQ® 4) „¬sH›Æ:‹¬ƒÿû䛽“


AM

‹¬O|OkOk`Ç"³°Ø L#ßq. J^Íq^Î=òQê ‹¬O|O^Î=ò`Ë, 45. …샏’=ò : #+¬“=ò :: ..... : ......


H
M

W=Þ|_# ZxßH› #°O_ ''«ÑÑ J#° ‹¬OöH`Ç=ò †³òH›ø 1) ~¡¶‡~ò:̄᪐ 2) ̄^Îí:a_ÈÛ


AM
IG

R.H.S. …Õ =ü_È= „¬^Î=ò`Ë ‹¬O|O^Î=ò Q® ^¥xx 3) [†Ç°=ò:J„¬[†Ç°=ò 4) "³ò`Çë=ò:ƒìQ®=ò


KH

ZO„²H› K͆Çò=ò. (H›=ü¼xöH+¬<£ S.I-2011) 46. "³òH›: K³@°“ :: ...... : .........


R

33. "͋¬q : =¶q°_ « j`ŒHê=ò: 1) Q®°ã~¡=ò:P_ÈQ®°ã~¡=ò 2) q^¥¼iÖ:J^¥¼„¬ä›½_È°


1) j`ǁ=ò 2) <ŒiO[ 3) "³òQ®¾:„¬ô+¬æ=ò 4) K³@°“:„¦¬sßKÇ~Ÿ
3) ~¡°`Ç°=ô 4) „¬ôKÇóHê†Ç°
47. ''„¬Q®°ÑÑ, ''~ŒãuÑÑH÷ ‹¬O|Ok`Ç=ò. J^Íq^ÎOQê ''`³°„¬ôÑÑ
34. ^ζ_È : P=ô « t‰×Ã=ô :
䛽 ‹¬O|Ok`Ç"³°Ø#k. (H›=ü¼xöH+¬<£ S.I-2011)
1) =°‚²ìˆ× 2) [O`Ç°=ô
1) „¬KÇó 2) Z~¡°„¬ô
3) ƒì¼, †Ç°¿=# =°^Î¼ ^Î‰× 4) a_ÈÛ
3) h=ò 4) #°„¬ô
35. „¬Q®° : ~Œã`Ç « †Çò^Μ=ò :
1) q‹²ï~_È° P†Çò^Î=ò 2) ̋áx䛽° 48. HEAD, CAP `Ë ‹¬O|O^"Î °³ #Ø k, J^Íq^OÎ Qê FINGER
3) `³eq 4) ‰§Ou nx`Ë ‹¬O|Ok`Ç"³°Ø#k? (H›=ü¼xöH+¬<£ S.I-2011)
36. ̄~òO\˜ :H›ˆìHê~¡°_È° « H›„¬ : ...... 1) QË~¡° 2) ̂ì…ÿà\˜
1) „¦¬ißKÇ~Ÿ 2) J_Èq 3) x„¬C 4) =ã_ÈOy 3) JOQ®°oã`Œ}=ò 4) UgHê=ô

REASONING ABILITY (VERBAL) ○ ○ ○ ○ ○ ○ ○ ○ ○


88
○ ○ ○ ○ ○ ○ ○ ○ ○ ○ ○ ○ ○ ○ ○ ○ ○ ○ ○
Study Material
12345678901234567890123456789012
12345678901234567890123456789012123456789012345678901234567890121234567890123456789012345678901212345678901234567890123
12345678901234567890123456789012

q=~¡}ì`ÇàH› ["Œ|°°
12345678901234567890123456789012123456789012345678901234567890121234567890123456789012345678901212345678901234567890123
12345678901234567890123456789012 12345678901234567890123456789012
12345678901234567890123456789012123456789012345678901234567890121234567890123456789012345678901212345678901234567890123
12345678901234567890123456789012 12345678901234567890123456789012
12345678901234567890123456789012
12345678901234567890123456789012123456789012345678901234567890121234567890123456789012345678901212345678901234567890123
12345678901234567890123456789012
12345678901234567890123456789012123456789012345678901234567890121234567890123456789012345678901212345678901234567890123
12345678901234567890123456789012 12345678901234567890123456789012
12345678901234567890123456789012123456789012345678901234567890121234567890123456789012345678901212345678901234567890123
12345678901234567890123456789012 12345678901234567890123456789012
12345678901234567890123456789012123456789012345678901234567890121234567890123456789012345678901212345678901234567890123
12345678901234567890123456789012 12345678901234567890123456789012

1. (2) 5322 + 2222 = 7544 J^Íq^ÎOQê N T U D


J^Íq^ÎOQê 2451 + 2222 = 4673. −1
↓ +1↓ −1↓ +1

2. (4) „¬ä›Æ½ (Birds) ‹¬=ü‚¬ðxß 'Flock' JO\ì=ò. J…ìöQ M U T E
Matter H÷ ‹¬O|OkOz Subject JO\ì=ò. 27. (1) E F G
3. (4) =¶#=ôx Q® ¶ ió J^ Î ¼ †Ç ° #O KÍ † Ç ò #k - =¶#= +3 +3 +3
„¬i}ì=°‰§GO J^Íq^ÎOQê, ↓ ↓ ↓
„¬^¥¼ Q®¶ió J^Î¼†Ç°#O K͆Çò#k - †Ç¶O`Œr. H I J
4. (3) ㄬ=¶^ÎO [~¡Q®ä›½O_¨ =òO^ΰ *ìãQ®`Çë J=‹¬~¡O J^Í q^ÎOQê M N O
J…ìöQ ~ËQꁰ ~Œä›½O_¨ ‰×Ãハ’`Ç J=‹¬~¡O. +3

+3

+3

5. (3) KÇ`Ç°~¡ãªxH÷ "³á‰§¼O LO@°Ok. J\ìÁöQ „¦¬°<ŒxH÷

Y
Q R
„¦¬°#„¬i=¶}O LO@°Ok. P
6. (3) ƒè~ï …˜ä½› =†Ç°…˜ Z…ìQË,J…ìöQ "Œ¶¼"£°ä›½ "³¶<ËãQꄦ.¹ 28. (1) ‚²ìO^ΰ=ô „¬qã`ÇãQ®O^ŠÎO → GITA (ƒ’Q®=n¾`Ç) J…ìöQ

M
7. (3) HÒO@|°…˜ =¼uö~H›O J<£HÒO@|°…˜ Y°~Œ<£ J#°#k - =ò‹²ÁO (ISLAM) „¬qã`Ç ãQ®O^ŠÎO
J…ìöQ „¦¬î¼†Ç°~Ÿä›½ …ÿ‹¹ 29. (4)

E
8. (4) Hê@~Ÿ „²Á~Ÿ. Z…ìïQá`Í |@“~ŸÌ„¦åÁ J=ô`Ç°O^Ë, 30. (3) „²‹…“¬ ˜#° L„¬†¶³ yOKÇ°@䛽 ã\÷Q~¾® Ÿ J=‹¬~O¡ . J^Íq^OÎ Qê

D
"³¶@~Ÿ#° L„¬†³¶yOKÇ°@䛽 Qê#° ‹²ÞKŸ J=‹¬~¡O.

9
J…ìöQ \ì_£‡é…˜, ㇦Q· J=ô`Ç°Ok.

59
c=~Ÿä›½ WO_ù…ÿ<£ž Z…ìQË „²HêH±ä›½ ZeQê<£ž J…ìQ® 31. (2) ö~_†Ç°O#° 䛀¼s H›x̄\÷“i. J^Íq^ÎOQê >ÿe‡¦é<£#°
9. (1)
CA
95
10. (1) A : qO@~Ÿ…Õ L„¬†³¶yOKÇ°#k - H÷ޅ˜Û
ãQ®‚¬ìOƒÿ…˜ H›x̄\ì“#°.
32. (2) =°‚¬ð‹¬=òã^ÎO =}©†Ç°`Ç H›e¾ LO_È°#°. J^Íq^ÎOQê

50
B : =°…èi†Ç¶#° x~ËkOKÇ_¨xH÷ L„¬†¶ ³ yOKÇ°#k H÷Þ<³<á £
+¬µQ®~Ÿ…Õ u†Ç°¼^Î#O LO@°Ok.
98
A
11. (1) =¶¼Q®r<£…Õ ªé“s, Pi“H›…˜ž L#ß@°ÁQê<Í \©…Õ q°…˜ø, +¬µQ®~Ÿ 33. (2) =¶q°_ Z䛽ø=Qê aOKÇ° season "͋q¬ . J^Íq^OÎ Qê
:8
LO\ì~ò. <ŒiO[ Z䛽ø=Qê aOKÇ° season j`ŒHê=ò.
E

12. (2) Q®*ìxß JOQ®°ˆì…Õ 䛀_¨ Hù°ªë~¡°.


h

34. (1)
.P

J…ìöQ HêÞ~Ÿ“#° '*º#°žÑ…Õ Hù°ªë~¡°.


IC

35. (4) „¬Q®°ä›½ =¼uö~H›=ò ~Œãu. J^Íq^ÎOQê †Çò^Μ=ò#䛽


AD

13. (1) 112 + 2 = 121 + 2 = 123 =¼uö~H›=ò '‰§OuÑ.


J…ìöQ 232 + 2 = 529 + 2 = 531 36. (4) H›ˆìHê~¡°_È° '̄~òO\˜Ñ "ͪë_°È . J^Íq^OÎ Qê =ã_ÈOy H›„¬`Ë
AB
O

14. (4) „¬xKͪë_È°.


ER
CH

15. (3)  s    s    J…ìöQ 24 × 3 = 72 37. (3)


YD

16. (4) 0°C = 273 k 38. (1) #k : P#H›@“ « ã\섦²H± : ‹²Q®ß…˜


∴ 0 : 273 : : ̋ež†Ç°‹¹ : ïHeÞ<£ 39. (3) [†Ç°=ò ‹¬O`Ë+¬=ò xKÇ°ó#°. J^Íq^OÎ Qê J„¬[†Ç°=ò ^ΰ—
,H

Y=ò xKÇ°ó#°.
T

17. (2) 5322 + 2222 = 7544


AM

J^Íq^ÎOQê 2451 + 2222 = 4673. 40. (2) „¬H÷Æ ZQ®°~¡°`Ç°Ok. J~ò`Í #`Çë ㇐䛽`Ç°Ok.
H

41. (3) "Œ# „¬_Í@„¬C_È° „²_È°Q®° „¬_È°ëOk. J…ìöQ ~Œãu „¬î@


M

18. (2) 7528 – 2222 = 5306


AM

J…ìöQ 4673 – 2222 = 2451 pH›\ ÷ „¬_È°ë`Ç°Ok.


IG

19. (4) 52 : 53 → 25 : 125 J…ìöQ 72 : 73 → 49 : 343 C O W D O G


KH

20. (4) 3 × 4 - 12 J…ìöQ 5 × 9 = 45 −2


↓ −2
↓ −2
↓ J^Í q^ÎOQê −2
↓ −2
↓ −2

R

21. (2) zã`ÇO…Õ AKJ =O\÷ ãuƒ°’ *ìHê~¡O, Z^ΰ~¡°Qê GNH ãuƒ°’ [O 42. (1)
A M U B M E
LOz J…ìöQ EMD ãuƒ°’ *ìxß Z^ΰ~¡°Qê AKB H H›^ΰ.
22. (4) HNP ãuƒ°’ [O䛽” PDA ãuƒ°’ [O ‡éó|_Ok. J^Íq^OÎ Qê, 43. (2) =ã_ÈOy "Œ_È° „¬iH›~¡=ò ~¡O„¬=ò. J…ìöQ ~¡KÇ~ò`Ç "Œ_È°
DLP ãuƒ°’ [O䛽” PJG ãuƒ°’ [O J=ô`Ç°Ok. „¬iH›~¡=ò ̄#°ß (H›=ò)
23. (3) CHFM =…ÿ GBJO LOk. 44. (1) `Œˆ×=òxß H›eÀ‹ë ‹¬=ü‚¬ìO. J>èÁ „¬îxß H›eÀ‹ë „¬îQ®°uë.
24. (2) 'AKJO' H÷ 'IOHN' Z…ìO\÷ ‹¬O|O^ÎO H›e¾ LO^Ë 45. (3) …샏=’ ò#䛽 =¼uö~H›=ò #+¬=“ ò. J^Íq^OÎ Qê [†Ç°=ò#䛽
J^Íq^ÎOQê 'CLBK' H÷ 'EMDL' ‹¬O|O^ÎO H›e¾ LOk. =¼uö~H›=ò J„¬[†Ç°=ò.
25. (1) 'GAM'H÷ 'NKM' `Ô Ë Z@°=O\÷ ‹¬O|O^OÎ H›^Ë 'AOE' 46. (3) "³òH› ("³òH›ø) K³@°“ J=ô`Ç°Ok. J…ìöQ "³òQ®¾ „¬ô+¬æ=ò
H÷ 'KMN' `Ë J@°=O\÷ ‹¬O|O^ÎO H›^ΰ. J=ô`Ç°Ok.
26. (2) M H U S M D 47. (4) „¬Q® °H÷ =¼uö~H›=ò ~Œãu. J>èÁ '`³°„¬ôÑH÷ =¼uö~H›=ò '#°„¬ôÑ
−1
↓ +1↓ −1↓ +1↓ −1↓ +1↓
48. (3) HEAD 䛽 ^iÎ OKÇ°#k CAP. J>èÁ FINGER 䛽 ^iÎ OKÇ°#k
L I T T L E RING (JOQ®°oã`Œ}=ò)

REASONING ABILITY (VERBAL) ○ ○ ○ ○ ○ ○ ○ ○ ○


89
○ ○ ○ ○ ○ ○ ○ ○ ○ ○ ○ ○ ○ ○ ○ ○ ○ ○ ○
Study Material
6 ~¡H›ë ‹¬O|O^¥°
(Blood Relations)

=òMì¼O‰§°...
G ‹¬O|O^¥…Õ ㄬ^¥#"³°Ø#k. ‹¬O|O^Î=ò JO>è =¼H÷ëH÷ =¼H÷ëH÷ …èH› =¼H÷ëH÷ =‹¬°ë=ô䛽 …èH› =‹¬°ë=ô䛽 =‹¬°ë=ô䛽
''~¡H›ë ‹¬O|O^¥°ÑÑ

…èH› q+¬†Ç°O, q+¬†Ç°O =°^Î¼ LO_Í J=Qꂬì#.


G D J^¥¼†Ç°O #°O_ J_öQ ㄬ‰×߁° <Œ°Q®° ~¡HꁰQê =ôO\ì~ò.
Jq : 1) <Í~¡°Qê ‹¬O|O^Î=ò 2) 䛽@°O| ‹¬ƒ’°¼° 3) =ü_È= =¼H÷ë 4) HË_£ ^¥Þ~Œ ‹¬¶zOKÍ ‹¬O|O^¥°.

Y
G =°#KÇ°@¶“ Q® ‹¬O|O^¥° `Ç~K¡ °Ç Qê =°#䛽 `³e‹²#q =°i†Çò =°#O qO@°#ß"Í J~ò#„¬æ\÷H© D kQ®°=# Hùxß ‹¬O|O^¥#°

M
‡ÚO^ΰ„¬~¡KÇ|_#q. g\÷ P^¥~¡OQê<Í Jƒ’¼~¡°Ö° ㄬ‰×߁䛽 ‹¬=¶^¥#O ~Œ|\ì“e.

E
‹¬O|OkH› =¼H÷ë ‹¬O|O^ÎO À„~¡° ‹¬O|OkH› =¼H÷ë ‹¬O|O^ÎO À„~¡°

D
9
59
1. <Œ `ÇeÁ …è^¥ `ÇOã_ †³òH›ø 16. 䛀`Ç°i †³òH›ø Hù_Ȱ䛽 …è^¥
CA
95
UïHáH› 䛀`Ç°~¡° - ªé^Îi Hù_Ȱ䛽 †³òH›ø Hù_Ȱ䛽 - =°#=_È°
2. `ÇeÁ …è^¥ `ÇOã_Qêi `ÇOã_ - `Œ`džǰ¼ 17. 䛀`Ç°i †³òH›ø 䛀`Ç°~¡° …è^¥

50
3. `ÇeÁ Qêi `ÇeÁ - J=°à=°à Hù_Ȱ䛽 †³òH›ø 䛀`Ç°~¡°
98 - =°#=~Œ°
A
4. `ÇOã_ Qêi `ÇeÁ - <Œ†Ç°#=°à
:8
18. JH›ø ƒ’~¡ë - ƒì=
E

5. `ÇeÁ Qêi JH›ø - ̄^Îí=°à 19. ̄^Îí<Œ#ß …è^¥ ƒìƒì~ò „²Á° - ^¥†Ç¶^ΰ°
h
.P

6. `ÇeÁ Qêi K³…ÿÁ° - „²xß 20. "Í°#`Çë „²Á° - ^¥†Ç¶^ΰ°


IC
AD

7. `ÇOã_Qêi JH›ø …è^¥ K³…ÿÁ° - "Í°#`Çë 21. <Œ `Œ`ÇQêi UïHáH› 䛽=¶~¡°_È° - <Œ#ß
8. `ÇOã_ Qêi `Ç=òà_È° - ƒìƒì~ò (…è^¥)
AB
O

22. <Œ `Œ`ÇQêi UïHáH› 䛽=¶ï~ë - J=°à


z<Œß#ß 23. JH›ø 䛀`Ç°~¡° - "Í°#HË_ȁ° (…è^¥)
ER
CH

9. `ÇOã_ Qêi J#ß - ̄^Îí<Œ#ß =~¡‹¬ä›½ 䛀`Ç°~¡°


YD

10. `ÇeÁ Qêi J#ß …è^¥ `Ç=òà_È° - "Í°#=¶=° 24. "Í°#`Çë ƒ’~¡ë - =¶=°
11. J#ß ƒì~¡¼ - =k#
,H

25. 䛽=¶~¡°x ƒì~¡¼ - HË_ȁ°


T

12. `Ç=òà_ ƒì~¡¼ - =°~¡^΁°


AM

26. 䛽=¶ï~ë ƒ’~¡ë - J°Á_È°


13. ƒì~¡¼ …è^¥ ƒ’~¡ë †³òH›ø `ÇOã_ - =¶=°
H

27. ̄^Îí<Œ#ß ƒì~¡¼ - ̄^Îí=°à


M

14. ƒ’~¡ë …è^¥ ƒì~¡¼ †³òH›ø ªé^Î~¡°_È° - =°ik


AM

28. "Í°#=¶=° ƒì~¡¼ - J`Çë


IG

15. JH›ø Hù_Ȱ䛽 - "Í°#°Á_È° (…è^¥)


29. ƒìƒì~ò ƒì~¡¼ - „²xß
KH

=~¡‹¬ä›½ Hù_Ȱ䛽
G
R

XH› `Ç~¡O #°O_ =°~ùH› `Ç~ŒxH÷ Q® ‹¬O|O^¥#° `³e†Ç°*è†Çò ãH›=°zã`ÇO :

"³ò^Î\ ÷ `Ç~¡O `Œ`džǰ¼, <Œ†Ç°#=°à, J=°à=°à

ï~O_È= `Ç~¡O `ÇeÁ, `ÇOã_, ̄^Îí<Œ#ß, ƒìƒì~ò, "Í°#=¶=°, =¶=°, "Í°#`Çë, J`Çë, ̄^Îí=°à, „²xß

<Í#° =ü_È= `Ç~¡O ªé^Îi, ƒ’~¡ë, ƒì~¡¼, ƒì=, ƒì==°ik, ªé^Î~¡°_È°, =°~¡^΁°, =k#

<Œ¾= `Ç~¡O 䛀`Ç°~¡°, Hù_Ȱ䛽, HË_ȁ°, J°Á_È°, "Í°#HË_ȁ°, "Í°#°Á_È°

S^Î= `Ç~¡O =°#=_È°, =°#=~Œ°

REASONING ABILITY (VERBAL) ○ ○ ○ ○ ○ ○ ○ ○ ○


90
○ ○ ○ ○ ○ ○ ○ ○ ○ ○ ○ ○ ○ ○ ○ ○ ○ ○ ○
Study Material
=¶ki ㄬ‰×߁°
MODEL - 1 1. 䛽@°O|O…Õx „¬ô~¡°+¬µ° Z=~¡°?

=°i†Çò B †³òH›ø `³e†Ç°^ΰ Hê=ô#


<Í~¡°Qê ‹¬O|O^ÎO ª^Î#: C

„¬ô~¡°+¬µx K³„¬æ…è=ò.
Gender

2. „²Á †³òH›ø `ÇeÁ^ÎOã_È° [O@ ãH÷Ok "Œx…Õ Uk?


1. XH› ‡¦é\ՅÕx =¼H÷ëx KǶ„²‹¬¶ë ‰Üá[ J<³#° ''P"³°

䛽=¶~¡°x `ÇOã_, <Œ `Çe†


Á ò
³ H›ø J°Á_°
È ÑÑ. J~ò# ‰Ü
á [,
ª^Î#: E =°i†Çò D J#° „²Á `ÇeÁ^ÎOã_È° [O@ A

‡¦é\ՅÕx =¼H÷ë U=°Q®°#°?


=°i†Çò F.
ª^Î#: ãH÷Ok KŒ~¡°“ #°O_ `ÇeÁ †³òH›ø J°Á_È° J#Qê ‰Üá[ 3. q"Œ‚²ì`Ç [O@ †³òH›ø `ÇeÁ^ÎOã_È°#° ‹¬¶zOKÇ° #k

ƒ’~¡ë …è^¥ ªé^Îi ƒ’~¡ë Jx J~¡œ=ò.


Uk?

q"Œ‚²ì`Ç [O@ AF †³òH›ø `ÇeÁ^ÎOã_È° [O@ .

Y
ª^Î#: BC

ŠÝâ‚\ Q#´± 圾>·¢±y!aÈPä`Ž 4. 䛽@°O|O…Õ ¢‹Ô끰 ZO`Ç =°Ok H›~¡°?

M
ª^Î#: D, E, F (B, C …Õ XH›~¡°) J#° #°Q®°~¡° ¢‹Ô끰 H›~¡°.
aÈfÂ!K‚±Â`ɱ «ê_Ïj!„“¢ì „“¢ì D E

E
5. =°i†Çò  =°^Î¼ ‹¬O|O^Î=ò D ãH÷Ok "Œx…Õ

Wzó# ["Œ|° ㄬHê~¡O J`Ç#° ªé^Îi ƒ’~¡ë J=ô`Œ_È°. Uk?

D
9
\ÿ ‡¦é\ՅÕx ¢‹ÔëH÷ ‰Üá[ ªé^Îi J=ô`Ç°Ok. =°i†Çò E° ªé^Îs=°}°°.

59
ª^Î#: D

CA
95
2. XH› =¼H÷ë, XH› ¢‹Ôë`Ë ''g° `ÇeÁ ƒ’~¡ë †³òH›ø ªé^Îi, <Œ
MODEL - 3 =ü_È= =¼H÷ë

50
"Í°#`ÇëÑÑ Jx K³„²æ#, P ¢‹Ôë, P =¼H÷ëH÷ U=°Q®°#°?

98
A
ª^Î#: zã`ÇO #°O_ W^Îíi "Í°#`Çë XH›ö~ J=ô`Ç°<Œß~¡°. 1. Jx…˜, ~˂²ì`Ÿ#° ''J`Çx †³òH›ø `ÇOã_ ƒì~¡¼ †³òH›ø
:8
m
XöH ‹¬‚¬ìÙ^Î~¡°x 䛽=¶~¡°xÑÑQê „¬iKdžǰO KÍ̋#°.

#α$aÈì
E

aÈf! „“ì¢!«ê_Ïj! !#α$aÈì


h

J~ò# ~˂²ì`Ÿ Jx…˜ä›½ U q^ÎOQê |O^Î°=ô ?


.P

(S.I. - 2008)
IC

>½Iøì £ŒÕì
AD

ª^Î#: Jx…˜ †³òH›ø `ÇOã_ †³òH›ø ƒì~¡¼ J#Qê Jx…˜ `ÇeÁ


Hê=ô# P ¢‹Ôë, P =¼H÷ëH÷ ªé^Îi J=ô`Ç°Ok.
AB
O

Jx J~¡ÖO. Jx…˜ `ÇeÁ †³òH›ø ‹¬‚¬ìÙ^Î~¡°x 䛽=¶~¡°_È°


J#Qê Jx…˜ †³òH›ø "Í°#=¶=°ä›½ „¬ôã`Ç°_È° Jx J~¡ÖO.
ER
CH

3. ''h=ô =¶<Œ#ß `Çe¾H÷ XöH XH› 䛀`Ç°~¡°ÑÑ. S`Í h䛽

<Í#° U=°=ô`Œ#°? Hê=ô# Jx…˜ä›½ ~˂²ì`Ÿ ^¥†Ç¶k J=ô`Œ_È°.


YD

ª^Î#: <Œ#ß `ÇeÁ ®ÿ<Œ†Ç°#=°à UïHáH› 䛀`Ç°~¡° ®ÿ"Í°#`Çë 2. s\ì, =°xn„¹`Ë W…ì J#ßk. ''<Œä›½ x#ß cKŸ…Õ
,H

P =¼H÷ë "Í°#°Á_È° J=ô`Œ_È°.


T

H›e‹²# ƒìeH›, <Œ À‹ß‚²ì`Ç°_ `ÇeÁ †³òH›ø ƒì=䛽 z#ß


AM

䛽@°O| ‹¬ƒ’°¼°
ä› € `Ç ° ~¡ ° ÑÑ. J~ò# P ƒìeH› , s\ì À‹ß‚² ì `Ç ° _ H ÷

MODEL - 2
H
M

U=°Q®°#°?

s\ì À‹ß‚²ì`Ç°_ `ÇeÁ ƒì= ® ̄^Îí<Œ#ß …è^¥ z<Œß#ß


AM
IG

‹¬¶KÇ# : (ㄬ‰×߁° 1 #°O_ 5) : D ãH÷O^Î W=Þ|_# ª^Î#:

†³òH›ø 䛀`Ç°~¡° ® ^¥†Ç¶k J=ô`Ç°Ok.


KH

‹¬=¶KŒ~¡=ò#° KÇkq =¼ä›½ë =°^Î¼ ‹¬O|O^¥° ~Œ|\ ÷“


R

ㄬ‰×߁䛽 ‹¬=¶^¥<Œ° ã"Œ†Çò=ò. 3. XH› =ò‹¬e =¼H÷ëx KǶ„²‹¬¶ë 䛽<Œ…˜ W…ì J<Œß_È°.

i) A, B, C, D, E =°i†Çò F° XH› 䛽@°O|O…Õx ''J`Çx 䛽=¶~¡°_È°, <Œ 䛽=¶~¡°_ ̄^Îí<Œ#ßÑÑ J~ò#

P~¡°Q®°~¡° =¼ä›½ë°. (q‰õÁ+¬})


P =ò‹¬e =¼H÷ë, 䛽<Œ…˜ä›½ U=°=ô`Œ_È°?

ii) XH› q"Œ‚²ì`Ç [O@ "Œi \PA PMENBO


ª^Î#:

`ÇeÁ ^ÎOã_È°#° =°i†Çò $ # 圾=†™


XH› ‹¬ O `Œ#O#° 圾>·¢±`ɱ 圾>·¢±`ɱ 圾>·¢±`ɱ
䛽@°O|O…Õ H›ey L#ßk. ' 
ͅ_Ïî=$à
iii) A, C †³òH›ø 䛽=¶~¡°_È°
"
„
í ¢
½
=°i†Ç ò E, A †³ ò H› ø 圾>·ðì Ì„á „¬@=ò #°O_ 䛽<Œ…˜ 䛽=¶~¡°_ ̄^Îí<Œ#ß
圾>·ðì Þ ä›½<Œ…˜ ªé^Î~¡°_È°, =ò‹¬e =¼H÷ë 䛽=¶~¡°_È°
䛽=¶ï~ë. aÈfÂ
iv) E †³òH›ø `ÇeÁ J~ò# F & 
% 
Þÿ䛽<Œ…˜ 䛀_¨ =ò‹¬e=¼H÷ë 䛽=¶~¡°_È° J=ô`Œ_È°
†³òH›ø 䛽=¶ï~ë D.
«ê_Ï t >±~±‚± Þÿ䛽<Œ…˜ †³òH›ø `ÇOã_
REASONING ABILITY (VERBAL) ○ ○ ○ ○ ○ ○ ○ ○ ○
91
○ ○ ○ ○ ○ ○ ○ ○ ○ ○ ○ ○ ○ ○ ○ ○ ○ ○ ○
Study Material
MODEL - 4 ‹¬¶KÇ# : (ㄬ‰×߁° 2 #°O_ 4) : D ãH÷Ok ‹¬=¶KŒ~¡O P^¥~¡OQê

HË_£ ^¥Þ~Œ ‹¬¶zOKÍ ‹¬O|O^ÎO ㄬ‰×߁䛽 ["Œ|°e=òà.

P × Q J#Qê P, Q †³òH›ø `ÇeÁ.


1. X + Y J#Qê X, Y †³òH›ø ªé^Î~¡°_È°. P – Q J#Qê P, Q †³òH›ø ªé^Î~¡°_È°.

X–Y J#Qê X, Y †³òH›ø `ÇeÁ. P + Q J#Qê P, Q †³òH›ø ªé^Îi.

X+Y J#Qê X, Y †³òH›ø ªé^Îi. J~ò# D ãH÷Ok P ¸ Q J#Qê P, Q †³òH›ø `ÇOã_ Jx J~¡ÖO.

"Œx…Õ S †³òH›ø JO䛽…˜ R J"ŒÞO>è Uk x[=ò ? 2. T †³òH›ø "Í°#=¶=° R #° ‹¬¶zOKÇ°#k ãH÷Ok "Œx…Õ

a) R + P + S b) S + Q – R Uk? a) T + M + R, b) R – M × T, c) R × M + T
c) R + Q – S d) R + Q + S ª^Î#: R–M×T Þÿ R, M †³òH›ø ªé^Î~¡°_È° =°i†Çò M, T

ª^Î#: { J#Qê ªé^Î~¡°_È° †³òH›ø `ÇeÁ Þÿ T †³òH›ø "Í°#=¶=° R.

- J#Qê `ÇeÁ 3. H †³òH›ø <Œ#ßQêi `ÇOã_ F #䛽 ‹¬=¶#"³°Ø#k Uk?

þ J#Qê ªé^Îi ¸ Þ †³òH›ø ªé^Î~¡°_È° =°i†Çò J, H

Y
ª^Î#: a) F–J H F, J

JO䛽…˜ J#Qê <Œ#ß …è^¥ J=°à †³òH›ø ªé^Î~¡°_È°. †³òH›ø `ÇOã_, ÞÿF, H †³òH›ø ̄^Îí<Œ#ß / z<Œ#ß

M
ㄬ‰×ß…Õ <Œ#ß䛽 Q® HË_£ „¬^ÎO W=ޅè^ΰ. Hê=ô# `ÇeÁ J=ô`Œ_È° (J‹¬`ǼO)
F¸J–H Þ F, J †³òH›ø `ÇOã_ =°i†Çò J, H †³òH›ø
†³òH›ø ªé^Î~¡°x JO䛽…˜Qê ƒìqOKŒe.

E
b)

`ÇeÁ, ªé^Î~¡°_È° †³òH›ø HË_£ Q®°~¡°ë° -, { Hê=ô# ªé^Î~¡°_È° ÞÿF, H 䛽 䛀_¨ `ÇOã_ (J‹¬`ǼO)

D
9
F¸J×H Þ F, J †³òH›ø `ÇOã_ =°i†Çò J, H †³òH›ø
a) =°i†Çò d) P„¬Â<£#° =ke"͆ǰ=KÇ°ó#°.

59
c)

`ÇeÁ ÞÿF, H †³òH›ø ƒ’~¡ë (J‹¬`ǼO)


({) (-)

CA
95
P„¬Â<£ b) S + Q – R Þ S ........... Q
F¸J¸H Þ F, J †³òH›ø `ÇOã_ =°i†Çò J, H †³òH›ø
ÿ
50
d)
|
`ÇOã_ ÞÿF, H †³òH›ø `Œ`Ç
98
A
(‹¬`ǼO)

R–M–K K, M
R
:8
J#Qê R †³òH›ø JO䛽…˜ 'S' J=ô`Ç°<Œß_È°. Hêx
4. …Õ 䛽 U=°=ô`Œ_È°?

Þÿ †³òH›ø ªé^Î~¡°_È° =°i†Çò M, K


E

ª^Î#: R – M – K R, M
‹¬=¶^¥#O S †³òH›ø JO䛽…˜ R J"ŒÞe.

†³òH›ø ªé^Î~¡°_È°
.P

32
IC

Hê=ô# ‹¬=¶^Î#O (c). ªé^Îi / ªé^Î~¡°_È°. (K Gender `³e†Ç°^ΰ)


AD

4
123456789012345678901234567890121234567890123456789012345678901212
AB
O

12345678901234567890123456789012123456789012345678901234567890121234567890123456789012345678901212345678901234567890123456789012123
1234567890123456789012345678901212345678901234567890123456789012123456789012345678901234567890
123456789012345678901234567890121234567890123456789012345678901212
12345678901234567890123456789012123456789012345678901234567890121234567890123456789012345678901212345678901234567890123
1234567890123456789012345678901212345678901234567890123456789012123456789012345678901234567890121234567890123
1234567890123456789012345678901212345678901234567890123456789012123456789012345678901234567890121234
12345678901234567890123456789012123456789012345678901234567890121234567890123456789012345678901212345678901234567890123456789012123
1234567890123456789012345678901212345678901234567890123456789012123456789012345678901234567890
12345678901234567890123456789012123456789012345678901234567890121234567890123456789012345678901212345678901234567890123
1234567890123456789012345678901212345678901234567890123456789012123456789012345678901234567890121234567890123
1234567890123456789012345678901212345678901234567890123456789012123456789012345678901234567890121234
123456789012345678901234567890121234567890123456789012345678901212
12345678901234567890123456789012123456789012345678901234567890121234567890123456789012345678901212345678901234567890123456789012123
1234567890123456789012345678901212345678901234567890123456789012123456789012345678901234567890
12345678901234567890123456789012123456789012345678901234567890121234567890123456789012345678901212345678901234567890123
1234567890123456789012345678901212345678901234567890123456789012123456789012345678901234567890121234567890123
1234567890123456789012345678901212345678901234567890123456789012123456789012345678901234567890121234
PRACTICE TEST
12345678901234567890123456789012123456789012345678901234567890121234567890123456789012345678901212345678901234567890123456789012123
1234567890123456789012345678901212345678901234567890123456789012123456789012345678901234567890
123456789012345678901234567890121234567890123456789012345678901212
12345678901234567890123456789012123456789012345678901234567890121234567890123456789012345678901212345678901234567890123
1234567890123456789012345678901212345678901234567890123456789012123456789012345678901234567890121234567890123
1234567890123456789012345678901212345678901234567890123456789012123456789012345678901234567890121234
12345678901234567890123456789012123456789012345678901234567890121234567890123456789012345678901212345678901234567890123456789012123
1234567890123456789012345678901212345678901234567890123456789012123456789012345678901234567890
12345678901234567890123456789012123456789012345678901234567890121234567890123456789012345678901212345678901234567890123
1234567890123456789012345678901212345678901234567890123456789012123456789012345678901234567890121234567890123
1234567890123456789012345678901212345678901234567890123456789012123456789012345678901234567890121234
123456789012345678901234567890121234567890123456789012345678901212
12345678901234567890123456789012123456789012345678901234567890121234567890123456789012345678901212345678901234567890123456789012123
1234567890123456789012345678901212345678901234567890123456789012123456789012345678901234567890
ER
CH

12345678901234567890123456789012123456789012345678901234567890121234567890123456789012345678901212345678901234567890123456789012123
1234567890123456789012345678901212345678901234567890123456789012123456789012345678901234567890
12345678901234567890123456789012123456789012345678901234567890121234567890123456789012345678901212345678901234567890123
1234567890123456789012345678901212345678901234567890123456789012123456789012345678901234567890121234567890123
1234567890123456789012345678901212345678901234567890123456789012123456789012345678901234567890121234
12345678901234567890123456789012123456789012345678901234567890121234567890123456789012345678901212345678901234567890123456789012123
1234567890123456789012345678901212345678901234567890123456789012123456789012345678901234567890

1. A, M 'D', M 6.
YD

° `ÇOã_, 䛽=¶ï~끰. †³òH›ø 䛽=¶~¡°_È°. ‹¬°|Ä~ŒA 䛽=¶ï~ë Hù_Ȱ䛽䛽 eOQ®~ŒA `ÇOã_ J~ò`Í,

A, 'D'
,H

H÷ U=°=ô`Œ_È°? eOQ®~ŒA, ‹¬°|Ä~ŒA䛽 U=°=ô`Œ_È°?

J|Ä Ì„^Îí`ÇOã_ `Œ`Ç =¶=° J°Á_È° =¶=° `ÇOã_ 䛽=¶~¡°_È°


T

1) 2) 3) 4) 1) 2) 3) 4)
AM

2. ‡¦é\ՅÕx =¼H÷ëx `Ç# 䛽=¶~¡°xH÷ KǶ„²‹¬¶ë XH› `ÇOã_ 7. XH› ¢‹Ô`


ë Ë XH› „¬ô~¡°+¬µ_È° W…ì JO@°<Œß_È°. ''h 䛽=¶~¡°_
H
M

W…ì JO@°<Œß_È°. ''g° J=°à ƒ~


’ ë¡ †³òH›ø K³…
Áÿ °ä›½ `ÇOã_.ÑÑ "Í°#=¶=°ä›½ UïHH
á › ƒì=#°ÑÑ. P ¢‹Ôë „¬ô~¡°+¬µ ‹¬O|O^"
Î °
Í q°?
AM

J#ß-K³…ÿÁ° ƒì~¡¼-ƒ’~¡ë
IG

䛽=¶~¡°xH÷, ‡¦é\ՅÕx =¼H÷ë U=°=ô`Œ_È°? 1) 2)

`Œ`Ç *è[†Ç°¼ `ÇOã_ =¶=° =¶=°-HË_ȁ° `ÇOã_-䛽=¶ï~ë


KH

1) 2) 3) 4) 3) 4)

3. 8.
R

"ÍkH›Ì„á# |‚¬•=°u#O^ΰHùO@°#ß ƒìeH›, `Ç# UïHáH› ㄬH›ø#°#ß q°ã`Ç°x`Ë ~Œ*è+¹ W…ì JO@°<Œß_È°. `³Áp~¡,

ªé^Î~°
¡ x "Í°#HË_ȁ° JO>è, P ƒìeH› D ¢‹ÔH
ë ÷ U=°=ô`Ç°Ok? #Á JOKÇ°, „¬î„¬î *ìïH@°“`Ë KÇO\ ÷a_ÈÛ<³`Ç°ë䛽#ß ¢‹Ôë

1) 䛽=¶ï~ë 2) HË_ȁ° 3) K³…ÿÁ° 4) =°~¡^΁° <Œ HË_Èe ƒ’~¡ë䛽 `ÇeÁ.ÑÑ ~Œ*è+¹ä›½ P ¢‹Ôë U=°=ô`Ç°Ok?

4. XH› „¬ô~¡°+¬µxß „¬iKdžǰO K͋¬¶ë XH› ¢‹Ôë W…ì JOk. <Œ 1) `ÇeÁ 2) ƒì~¡¼ 3) 䛽=¶ï~ë 4) HË_ȁ°
ƒ~
’ ë¡ `ÇeQ
Á êi ƒ~
’ ä
ë¡ ½
› W`Ç_°
È UïHH
á › ‹¬O`Œ#=òÑÑ. P „¬ô~¡°+¬µ_È°, 9. ㄬH›ø#°#ß =¼H÷ëx „¬iKdžǰO K͋¬¶ë ‹¬a`Ç W…ì JOk. ''=¶

P ¢‹ÔëH÷ U=°=ô`Œ_È°? `ÇOã_ 䛽=¶ï~ä


ë ½
› UïHH
á › ªé^Î~°
¡ _È°ÑÑ. ‹¬a`Ç P =¼H÷H
ë ÷ U=°=ô`Ç°Ok?

1) ªé^Î~¡°_È° 2) ƒì= 3) =¶=° 4) ƒ’~¡ë 1) ƒì~¡¼ 2) `ÇeÁ 3) HË_ȁ° 4) ªé^Îi


5. XH› ¢‹Ôë zã`Ç „¬\ìxß KǶ„²‹¬¶ë ‚¬ìi W…ì J<Œß_È°. ''D"³° 10. QË_Èg°k ‡¦é\ՅÕx XH› ƒìeH›#° KǶ„²‹¬¶ë ‹¬On„¹ W…ì

<Œ ªé^Îi `ÇeÁQêiH÷ P_Ȅ¬_ÈKÇ° 䛽=¶ï~ë.ÑÑ ‚¬ìiH÷ P ¢‹Ôë J<Œß_È°. ''D"³°ä›½ ªé^Îs° Hêx ªé^Î~¡°° Hêx …è~¡°.

U=°=ô`Ç°Ok? D"³° J=°à =¶ J=°à䛽 䛽=¶ï~ë.ÑÑ ‡¦é\ՅÕx J=¶à~ò

1) JH›ø …èH› K³…ÿÁ° 2) J`Çë …èH› „²xß ‹¬On„¹ä›½ U=°=ô`Ç°Ok?

3) =k# …èH› =°~¡^΁° 4) `ÇeÁ …èH› J`Çë 1) 䛽=¶ï~ë 2) `ÇeÁ 3) "Í°#HË_ȁ° 4) ƒì~¡¼
REASONING ABILITY (VERBAL) ○ ○ ○ ○ ○ ○ ○ ○ ○
92
○ ○ ○ ○ ○ ○ ○ ○ ○ ○ ○ ○ ○ ○ ○ ○ ○ ○ ○
Study Material
11. "Í°#`Çë †³òH›ø UïHH
á › ªé^Î~°
¡ _ XöH XH›ø ‹¬O`Œ#=ò Z=~¡°? 19. a,ÿb ° J<Œß K³…ÿÁ°. q,ÿa 䛽=¶~¡°_È°. f,ÿb 䛽=¶ï~ë.

1) `Ç=òà_È° 2) J#ß q,ÿfÿ  ‹¬O|O^Î=ò `³e†Ç°*è†Ç°O_?

3) ƒì= …èH› ƒì==°ik 4) <Í#° 1) J#ß-K³…ÿÁ° 2) ƒì=-=°~¡^΁°


12. "Í°#=¶=° UïHáH› ªé^Îi 䛽=¶~¡°_È° U=°=ô`Œ_È°? 3) =¶=°-HË_ȁ° 4) `ÇOã_-䛀`Ç°~¡°
1) ªé^Î~¡°_È° 2) ªé^Îi 20. 'B', A H÷ `ÇOã_. 'C', A †³òH›ø 䛽=¶~¡°_È°. 'P', C H÷

3) J°Á_È° 4) ƒì=
K³…ÿÁ°. 'B', P 䛽 U=°Q®°#°?

13. `ÇeÁ †³òH›ø `Ç=òàx "Í°#HË_ȁ° U=°=ô`Ç°Ok?


1) `Œ`Ç 2) =¶=°
1) J`Çë 2) ªé^Îi 3) =°~¡^΁° 4) „²xß 3) ̄^Îí`ÇOã_ 4) *è[†Ç°¼

14. K³…ÿÁe ƒ’~¡ë `ÇeÁx U=°x „²"Œe?


21. 'N' M. 'E', N ƒì~¡¼ H÷ ªé^Î~¡°_È°. E, S ° J<Œß K³…ÿÁ°Á.

1) J=°à 2) J`Çë 3) „²xß 4) ̄^Îí=°à 'P', S †³òH›ø 䛽=¶~¡°_È°. P, M #° U=°x „²"Œe?

15. M, N ° ^ÎO„¬`Ç°°. 'K' M. 'N'


䛽=¶~¡°_È° 䛽=¶~¡°_È°
1) J°Á_È° 2) ̄^Îí=°à 3) J`Çë 4) =¶=°
'P'. P 'D'. M, D

Y
†³òH›ø z<Œß#ß  ‹¬O|O^Î"Í°q°?
22. X, Z †³òH›ø ƒ’~¡ë. 'Y', X †³òH›ø JH›ø. 'Z', Y x U=°x

1) `ÇOã_-䛽=¶~¡°_È° 2) =k#-=°ik

M
„²"Œe?
3) J#ß-K³…ÿÁ°Á 4) J#ß^Î=òà°
J`Çë JH›ø =°~¡^΁° =k#
16. A, B 'C', A D, B
1) 2) 3) 4)

E
23. 'A', Z Z N. A
†³òH›ø `ÇOã_. †³òH›ø `ÇOã_. °

C, D †³òH›ø ªé^Î~¡°_È°. †³òH›ø "Í°#HË_ȁ°

N
ƒì~Œ¼ ƒ’~¡ë°.  ‹¬O|O^Î"Í°q°?

D
9
`ÇOã_-䛀`Ç°~¡° `Œ`Ç-=°#°=°~Œ°
=°i†Çò  ‹¬O|O^Î"Í°k J==KÇ°ó ?

59
1) 2)
`ÇOã_-䛀`Ç°~¡° `ÇeÁ-䛀`Ç°~¡°
3) J`Çë-J°Á_È° 4) =¶=°-HË_ȁ°
CA
1) 2)

95
17. 'X', Y Y, X 3) J#ß-K³…ÿÁ° 4) =k#-=°~¡^΁°

50
24. F K. 'A', F F A
†³òH›ø `ÇOã_. Hêx †³òH›ø 䛽=¶~¡°_È° Hê^ΰ.

'Y', X †³òH›ø 䛽=¶ï~ë H÷ ªé^Îi. ¢‹Ôë J~ò`Í #°

98
A
=°i 䛽 U=°Q®°#°?

HË_ȁ° J°Á_È° 䛀`Ç°~¡° `ÇeÁ K U=°x „²"Œe?


:8
1) 2) 3) 4)

18. M, N N A. A, M 1) J`Çë 2) =k# 3) „²xß 4) 䛀`Ç°~¡°


E

° ƒì~Œ¼-ƒ’~¡ë°. †³òH›ø `ÇeÁ 

25. 'B', M 'G', B M


h
.P

†³òH›ø 䛽=¶ï~ë. †³òH›ø `ÇeÁ. Z=~¡°?


‹¬O|O^Î=ò Uk?
IC

J`Çë-HË_ȁ° `ÇeÁ-䛀`Ç°~¡° 1) ¢‹Ôë 2) G †³òH›ø ƒ’~¡ë


AD

1) 2)

3) =¶=°-HË_ȁ° 4) `ÇOã_-䛽=¶~¡°_È°
3) 'G' H÷ `ÇeÁ 4) 'G' H÷ ƒì~¡¼
AB
O

1234567890123456789012
12345678901234567890123456789012123456789012345678901234567890121234567890123456789012345678901212345678901234567890123
12345678901234567890123

q=~¡}ì`ÇàH› ["Œ|°°
12345678901234567890123456789012123456789012345678901234567890121234567890123456789012345678901212345678901234567890123
1234567890123456789012 12345678901234567890123
ER
CH

1234567890123456789012 12345678901234567890123
12345678901234567890123456789012123456789012345678901234567890121234567890123456789012345678901212345678901234567890123
1234567890123456789012
1234567890123456789012 PRACTICE TEST :
12345678901234567890123456789012123456789012345678901234567890121234567890123456789012345678901212345678901234567890123
1234567890123456789012 12345678901234567890123
12345678901234567890123
12345678901234567890123456789012123456789012345678901234567890121234567890123456789012345678901212345678901234567890123
12345678901234567890123
12345678901234567890123456789012123456789012345678901234567890121234567890123456789012345678901212345678901234567890123
YD

1234567890123456789012 12345678901234567890123
12345678901234567890123456789012123456789012345678901234567890121234567890123456789012345678901212345678901234567890123

(3) ÿ
,H

1. A ® (M) ® D ¢‹Ôë
ÿ䛽=¶~¡°_È° (HË_ȁ°)
T

'D', A †³òH›ø 䛽=¶ï~ë Hù_Ȱ䛽. 䛽=¶ï~ë Hù_Ȱ䛽䛽 (2) ~Œ*è+¹ ( .)


AM

8. ®
H

`Ç#° `Œ`Ç J=ô`Œ_È°. HË_Èe ƒ’~¡ë ''䛽=¶~¡°_È°ÑÑ ä›½=¶~¡°x `ÇeÁ ''ƒì~¡¼ÑÑ.


M

(1) J=°à ƒ’~¡ë « `ÇOã_. W`Çx K³…ÿÁ° « "Í°#`Çë. D"³° 9. (4) `ÇOã_ ÿ‹¬a`Ç
AM

`ÇOã_ 䛽=¶ï~ë J#Qê


IG

2.
®

`ÇOã_ `Œ`džǰ¼. ÿÿÿÿ¯ `Ç#䛽 UïHáH› ªé^Î~¡°_È°.


KH

(1) UïHáH› ªé^Î~¡°x "Í°#HË_ȁ° JO>è `Ç# 䛽=¶ï~ë Jx ªé^Î~¡°_È°


R

3.

J~¡ÖO. 10. (3) ( ) ÿ‹¬On„¹ ® ‡¦é\ՅÕx ƒìeH›,


4. (4) ƒ~’ ë¡ †³òH›ø `Çe^
Á OÎ ã_È°°, J`Ç= ë ¶=°° J=ô`Œ~¡°. J`Ç,ë ÿÿÿÿ¯ ‹¬On„¹ ªé^Îi 䛽=¶ï~ë.
=¶=° UïHáH› ‹¬O`Œ#=ò P"³°ä›½ ƒ’~¡ë<Í J=ô`Œ_È°. ( ) ®ÿ( )

5. (3) ªé^Îi `ÇeÁ `Ç#䛽 䛀_È `Çe† Á °Í . `ÇeÁ P_Ȅ_¬ K


È °Ç ''"Í°#`ÇÑë Ñ ‡¦é\ՅÕx ƒìeH›
J=ô`Ç ° Ok. J`Ç ë ä› € `Ç ° ~¡ ° =k# …è H › =°~¡ ^ Î  ° 11. (4) "Í°#`Çë, UïHáH› ªé^Î~¡°_È° JO>è `ÇOã_. `ÇOã_H÷ UïHáH›
J=ô`Ç°Ok. ‹¬O`Œ#=ò JO>è <Í#°.
12. (1) "Í°#=¶=°ä›½ UïHáH› ªé^Îi JO>è `ÇeÁ.
6. (1) ‹¬°|Ä~ŒA 䛽=¶ï~ë ƒ’~¡ë « eOQ®~ŒA
eOQ®~ŒA, ‹¬°|Ä~ŒA䛽 J°Á_È=ô`Œ_È°. `ÇeÁ 䛽=¶~¡°_È° ªé^Î~¡°_È° J=ô`Œ_È°.
13. (2) `ÇeÁ `Ç=òà_È° ''"Í°#=¶=°ÑÑ J=ô`Œ_È°. J`Çx "Í°#HË_ȁ°
7. (2) 䛽=¶~¡°_ "Í°#=¶=° « ƒì~¡¼ `Ç=òà_È° (J#ß). W`Çx
ƒì= « J`Çx ªé^Îi ƒ’~¡ë. ''ªé^ÎiÑÑ J=ô`Ç°Ok.

REASONING ABILITY (VERBAL) ○ ○ ○ ○ ○ ○ ○ ○ ○


93
○ ○ ○ ○ ○ ○ ○ ○ ○ ○ ○ ○ ○ ○ ○ ○ ○ ○ ○
Study Material
14. (2) K³…ÿÁ° ƒ’~¡ë ''ƒì= (ƒì==°ik)ÑÑ J=ô`Œ_È°. 21. (3) N (M)

ƒì=䛽 J=°à ''J`ÇëÑÑ J=ô`Ç°Ok. ¯ 'N' J#ß, 'S' K³…ÿÁ°

15. (4) K ® M(N) ® Pÿ E K³…ÿÁ° 䛽=¶~¡°_È° J°Á_È°.

ÿÿÿÿ¯ ¯ 'P', M #° J`Çë Jx „²"Œe.

D
(S) ®ÿP

16. (2) C ® A ÿ ® ƒì~¡¼ : D, ƒ’~¡ë : B


B (D) 22. (4) (Y)

'C' =°#=°_È° 'B', B ƒì~¡¼ 'C' 䛽 =°#°=°~Œ° J=ô`Ç°Ok. ­ ƒ’~¡ë JH›ø ''=k#ÑÑ J=ô`Ç°Ok.

17. (3) X ÿ(Y)® `ÇOã_ : X, 䛽=¶ï~ë : Y X (Z)

18. (1) (A) ÿN(M) A : J`Çë, M : HË_ȁ°


23. (1) (Z)

19. ÿ(2) ÿa
®

® ( ÿq )
¯ 'A' 䛽=¶ï~ë 'N'.
ÿ¯ q,ÿf ° ƒì==°~¡^΁° …èH› =°ik-=k#. (3)
A ®ÿ(N)

b f

Y
24. (F) ®ÿ(K)

(4)
( ) ®ÿ(

ÿ
)
ÿ¯ A, F

M
° J#ß K³…ÿÁ°
° ªé^Îi°

'K', A
20. B ® A ® C C, P

¯ C, P ° A ‹¬O`Œ#=ò (2)
(A)

ÿÿÿ
#° „²#`ÇeÁ Jx „²"Œe.

E
25. M (B)
B =°#°=°~Œ° « P
®

M, G
(P)

D
(G)

9
'P', B x J|Ä …èH› *è[†Ç°¼ Jx „²"Œe.
® ƒ’~¡ë : ƒì~¡¼ :

59
123456789012345678901234567890121234567890123456789012345678901212
12345678901234567890123456789012123456789012345678901234567890121234567890123456789012345678901212345678901234567890123456789012123
123456789012345678901234567890121234567890123456789012345678901212345678901234567890123
12345678901234567890123456789012123456789012345678901234567890121234567890123456789012345678901212345678901234567890123
1234567890123456789012345678901212345678901234567890123456789012123456789012345678901234567890121234567890123
1234567890123456789012345678901212345678901234567890123456789012123456789012345678901234567890121234
123456789012345678901234567890121234567890123456789012345678901212
CA
95
12345678901234567890123456789012123456789012345678901234567890121234567890123456789012345678901212345678901234567890123456789012123
123456789012345678901234567890121234567890123456789012345678901212345678901234567890123
12345678901234567890123456789012123456789012345678901234567890121234567890123456789012345678901212345678901234567890123
1234567890123456789012345678901212345678901234567890123456789012123456789012345678901234567890121234567890123
1234567890123456789012345678901212345678901234567890123456789012123456789012345678901234567890121234
12345678901234567890123456789012123456789012345678901234567890121234567890123456789012345678901212345678901234567890123456789012123
123456789012345678901234567890121234567890123456789012345678901212345678901234567890123
123456789012345678901234567890121234567890123456789012345678901212
12345678901234567890123456789012123456789012345678901234567890121234567890123456789012345678901212345678901234567890123
1234567890123456789012345678901212345678901234567890123456789012123456789012345678901234567890121234567890123
1234567890123456789012345678901212345678901234567890123456789012123456789012345678901234567890121234
PREVIOUS BITS
12345678901234567890123456789012123456789012345678901234567890121234567890123456789012345678901212345678901234567890123456789012123
123456789012345678901234567890121234567890123456789012345678901212345678901234567890123
12345678901234567890123456789012123456789012345678901234567890121234567890123456789012345678901212345678901234567890123
1234567890123456789012345678901212345678901234567890123456789012123456789012345678901234567890121234567890123
1234567890123456789012345678901212345678901234567890123456789012123456789012345678901234567890121234
123456789012345678901234567890121234567890123456789012345678901212
12345678901234567890123456789012123456789012345678901234567890121234567890123456789012345678901212345678901234567890123456789012123
123456789012345678901234567890121234567890123456789012345678901212345678901234567890123
12345678901234567890123456789012123456789012345678901234567890121234567890123456789012345678901212345678901234567890123
1234567890123456789012345678901212345678901234567890123456789012123456789012345678901234567890121234567890123
1234567890123456789012345678901212345678901234567890123456789012123456789012345678901234567890121234
12345678901234567890123456789012123456789012345678901234567890121234567890123456789012345678901212345678901234567890123456789012123
123456789012345678901234567890121234567890123456789012345678901212345678901234567890123
123456789012345678901234567890121234567890123456789012345678901212
12345678901234567890123456789012123456789012345678901234567890121234567890123456789012345678901212345678901234567890123456789012123
123456789012345678901234567890121234567890123456789012345678901212345678901234567890123
12345678901234567890123456789012123456789012345678901234567890121234567890123456789012345678901212345678901234567890123
1234567890123456789012345678901212345678901234567890123456789012123456789012345678901234567890121234567890123
1234567890123456789012345678901212345678901234567890123456789012123456789012345678901234567890121234

50
12345678901234567890123456789012123456789012345678901234567890121234567890123456789012345678901212345678901234567890123456789012123
123456789012345678901234567890121234567890123456789012345678901212345678901234567890123
1234567890123456789012345678901212345678901234567890123456789012123456789012345678901234567890121234567890123
1234567890123456789012345678901212345678901234567890123456789012123456789012345678901234567890121234

98
A
1. KÇ~}
¡ #
Ÿ ° KǶ„²‹¶
¬ ,
ë ''W`Çx `ÇeQ
Á êi ªé^Î~°
¡ _È°, <Œ 䛽=¶~¡°_È° 5. A+B C×D Ó ‹¬=¶‹¬O…Õ 'D' A H÷ `Ë Q® ‹¬O|O^ÎO
:8
1) `ÇOã_ 2) JO䛽…˜ 3) PO\© 4) `Œ`Ç
E

J~¡°<
û £ †³òH›ø `ÇOã_ÑÑ Jx Hê[…˜ JO>è, Hê[…˜`Ë KÇ~}
¡ H
Ÿ ÷
h

A–B+C×D C 'A'
.P

Q® ‹¬O|O^O
Î (A.P.Const Prelims -2016) 6. J~ò`Í H÷ `Ë Q® ‹¬O|O^ÎO
IC

1) =k# (…è^¥) =°~¡^΁° 1) H›l<£ 2) `ÇOã_


AD

2) ªé^Î~¡°x (…è^¥) ªé^Îi 䛽=¶~¡°_È° 3) ªé^Î~¡°x 䛽=¶ï~ë 4) JO䛽…˜


AB
O

3) PO\© 7. XH› ƒì°x`Ë "³ˆ×Ãë#ß XH› ¢‹Ôëx =°~Ë ¢‹Ôë "Œi =°^Î¼
ER
CH

4) ªé^Î~¡°x (…è^¥) ªé^Îi 䛽=¶ï~ë |O^Î°`ÇÞO U=°x J_yOk. "³ò^Î\ ÷ ¢‹Ôë ''<Œ "Í°#=¶=°,
YD

2. XH› „¬ô~¡°+¬µ_x „¬iKdžǰO K͋¬¶ë ''D†Ç°# ƒì~¡¼ =¶<Œ#ß P ƒì°x "Í°#=¶=°ä›½ "Í°#=¶=° XH›~
ö , Jx ‹¬=¶^¥#O
,H

Qêi UïHáH› 䛽=¶ï~ëÑÑ Jx XH› ¢‹Ôë JO>è, P ¢‹Ôë`Ë P WzóOk. P ¢‹ÔëH÷, ƒì°xH÷ Q® |O^Î°`ÇÞO.
T
AM

„¬ô~¡°+¬µ_H÷ Q® ‹¬O|O^O


Î (A.P.Const Prelims -2016) (T.S. S.I. Mains-2016)

1) ªé^Î~¡°_È° 2) =¶=°Qê~¡° 1) J=°à=°à, =°#=°_È° 2) `ÇeÁ, Hù_Ȱ䛽


H
M

3) "Í°#=¶=° 4) ƒ’~¡ë 3) J`Çë, "Í°#°Á_È° 4) <Œ#=°à, =°#=°_È°


AM
IG

3. =°<Ë*˜`Ë =°À‚ì+¹ ''x#ß <Í#° =¶<Œ#=°à Qêi 䛽=¶ï~ë 8. G x KǶ„¬ô`Ƕ XH› =¼H÷ë D ''P†Ç°# †³òH›ø XöH XH›
KH

†³òH›ø UïHH
á › ªé^Î~°
¡ _x F_OKŒ#°ÑÑ Jx JO>è, =°À‚ì+¹KÍ ªé^Î~¡°_È° <Œ 䛀`Ç°~¡° `ÇOã_H÷ `ÇOã_ÑÑ Jx K³‡æ_È°.
R

F_O„¬|_# =¼H÷ë, J`Çx (A.P.Const. Prelims -2016) J„¬C_È° D G H÷ `Ë |O^Î°`ÇÞO. (T.S. S.I. Mains-2016)

1) Hù_Ȱ䛽 2) `ÇOã_ 3) ªé^Î~¡°_È° 4) =¶=°Qê~¡° 1) `Œ`Ç 2) `ÇOã_ 3) ƒì= 4) ƒìƒì~ò


4. A †³òH›ø ªé^Î~°
¡ x `ÇOã_ †³òH›ø ƒì~¡¼ä›½ 'P' `ÇeÁ J=ô`Ç°Ok. 9. P~¡°Q®°~¡° =¼ä›½
ë ° A, B, C, D, E, F ° Q® XH› 䛽@°O|

A P H÷ `Ë Q® ‹¬O|O^O
Î (A.P.Const. Mains -2016) ‹¬ƒ’°¼…ÕÁ ãH÷Ok ‹¬O|O^¥°<Œß~ò.

1) J=°àªé^Îi 2) H›l<£ 3) <Œ#ß=°à 4) J=°à=°à i) 䛽@°O|O…Õ P_È"Œi ‹¬OY¼ =°Q®"Œi ‹¬OY¼ä›½

(5-6) ãH÷Ok ‹¬=¶KŒ~¡=ò ‹¬‚¬ð†Ç°O`Ë D ãH÷O^Î W=Þ|_# ‹¬=¶#O

ㄬ‰×߁䛽 ["Œ|°° ã"Œ†Çò=ò. (A.P.Const. Mains -'16) ii) A, E F ° 䛽 䛽=¶~¡°°

P+Q J#Qê 'Q' †³òH›ø `ÇOã_ 'P'. iii) XH› ƒì°_È°, XH› ƒìeH› W^ÎíiH© `ÇeÁ 'D'
P–Q J#Qê 'Q' †³òH›ø ªé^Î~¡°_È° 'P'. iv) A B H÷ 䛽=¶~¡°_È°

P×Q J#Qê 'Q' †³òH›ø ªé^Îi 'P'. v) ㄬ‹¬°ë`ŒxH÷ 䛽@°O|O…Õ XH› "³á"Œ‚²ìH› [O@ LOk.

P Q Ó J#Qê 'Q' †³òH›ø `ÇeÁ 'P'. (T.S. S.I. Mains-2016)

REASONING ABILITY (VERBAL) ○ ○ ○ ○ ○ ○ ○ ○ ○


94
○ ○ ○ ○ ○ ○ ○ ○ ○ ○ ○ ○ ○ ○ ○ ○ ○ ○ ○
Study Material
1) A, B, C ° JO^Î~¡¶ P_È"Œö~ 18. F E H÷ `Ë Z…ìO\ ÷ ‹¬O|O^ÎO LOk?

2) D H÷ A ƒ’~¡ë 1) Hù_Ȱ䛽 2) JO䛽…˜ 3) ƒ’~¡ë 4) 䛀`Ç°~¡°


3) F H÷ D =°#°=°~Œ° 4) E, F ° D ‹¬O`Œ#O 19. B ‹¬Ì‚ì€^Î~¡°_È° A, C †³òH›ø ‹¬Ì‚ì€^Îi B,D `ÇOã_ C
10. XH› ‡¦Ú\Õx KǶ„¬ô`Ƕ H÷~}
¡ Ÿ D q^O
Î Qê K³‡æ_È°. ''J`Ç#° J~ò# A D H÷ ____ J=ô`Œ_È° (S.I.-2011)

=¶ ªé^Î~¡°_ `ÇOã_ †³òH›ø XöH 䛀`Ç°i Hù_Ȱ䛽.ÑÑ H÷~¡}Ÿ,


1) Brother 2) Sister

‡¦é\Õ…Õ L#ß =¼H÷ëH÷ U=°Q®°#°? (S.I.-2012)


3) Nephew 4) Data inadequate 5) UnHê^ΰ
1) "Í°#°Á_È° 2) "Í°#=¶=° 3) `ÇOã_ 4) ªé^Î~¡°_È° 20. n„¬H± XH› =¼H÷ëx KǶ„²‹¬¶ë, ''W`Çx XöH XH› ªé^Î~¡°_È°

`Ç# 䛽=¶ï~ë `ÇOã_ †³òH›ø `ÇOã_ÑÑ J~ò# P =¼H÷ëH÷ n„¬H±


11. g}, =ò~¡o †³òH›ø =k#/ =°~¡^΁°, =°i†Çò ƒ’"Œh

=°^Î¼ ‹¬O|O^Î=ò (S.I.-2011)


†³òH›ø HË_ȁ°, ~¡"Í°+¹, À‚ì=°O`Ÿ †³òH›ø `ÇOã_ =°i†Çò
1) Father 2) Grandfather
À‚ì=°O`Ÿ =ò~¡m †³òH›ø XöH XH› ªé^Î~°
¡ _È°. ƒ"
’ Œh =ò~¡mH÷
3) Uncle 4) Brother-in-law 5) UnHê^ΰ
U=°=ô`Ç°Ok? (S.I.-2012)
21. XH› =¼H÷ë, XH› ¢‹Ôë`Ë "Í~ùH› =¼H÷ëx KǶ„²‹¬¶ë ''J`Çx `ÇeÁ h
1) ƒì~¡¼ 2) JO\© 3) J`Çë 4) `ÇeÁ

Y
A3P A,P
`ÇOã_ †³òH›ø UïHáH› 䛽=¶ï~ëÑÑ J~ò`Í P ¢‹Ôë "Í~ùH› =¼H÷ë

M
12. JO>è †³òH›ø `ÇeÁ
(S.I.-2012)
A4P A,P
=°^Î¼ ‹¬O|O^Î=ò?

JO>è †³òH›ø ªé^Î~¡°_È°


1) Sister 2) Mother
A9P A,P

E
JO>è †³òH›ø ƒ’~¡ë
3) Wife 4) Daughter 5) UnHê^ΰ
A5P A,P A, B C, A B C

D
9
JO>è †³òH›ø 䛀`Ç°~¡°
22. †³òH›ø `ÇOã_ †³òH›ø 䛀`Ç°~¡° =°i†Çò

K,M

59
ãH÷Ok "Œ\ ÷…Õ Uk †³òH›ø J`Çë Jx KǶ„¬ô`Ç°Ok.
=°^¼
Î Q® ‹¬O|O^O
Î Uk? (Communication S.I.-2011)

CA
95
(S.I.-2012) 1) `ÇOã_ =°i†Çò 䛀`Ç°~¡° 2) ƒ’~¡ë =°i†Çò ƒì~¡¼
1) K5J9M3N 2) K3J9N4M 3) J#ß =°i†Çò K³…ÿÁ° 4) =¶=° =°i†Çò "Í°#°Á_È°

50
3) M9N5K3J 4) M9N3K4J A,B C D,C D
98
A
23. =°i†Çò †³òH›ø `Çe.
Á †³òH›ø ƒ~
’ ë¡ J~ò`Í H÷

A B,C C A
:8
13. †³òH›ø 䛽=¶~¡°_È° x q"Œ‚¬ìO K͋¬°ä›½<Œß_È°. U=°Q®°#°? (Communication S.I.-2011)

D,B E 1) `ÇeÁ 2) "Í°#`Çë 3) K³…ÿÁ° 4) J`ÇëQê~¡°


E
h

†³òH›ø ªé^Îi †³òH›ø ªé^Î~¡°_³á# x q"Œ‚¬ìO

'D', A
.P
IC

K͋¬°HùOk. H÷ U=°=ô`Ç°Ok? (S.I.-2012) 24. JOH÷`Ÿ |*ì~¡°…Õ XH› ¢‹Ô`


ë Ë #_È°‹¬°<
ë Œß_È°. P ¢‹Ôë JOH÷`#
Ÿ °
AD

1) HË_ȁ° 2) ªé^Îi `Ç# À‹ß‚²ì`Ç°#䛽 W@°Á „¬iKdžǰO KÍ̋#°. ''J`Çx `ÇeÁ

3) =k# …è^¥ =°~¡^΁° 4) H›l<£


AB
O

<Œ `ÇeÁH÷ XH›øQê <ùH›ø 䛀`Ç°~¡°ÑÑ P ¢‹Ôë`Ë JOH÷`Ÿä›½ Q®

M,N N,V M,P ‹¬O|O^ÎO? (S.I.-2008)


ER

14. †³òH›ø `ÇOã_ =°i†Çò †³òH›ø Hù_Ȱ䛽, H÷


CH

=°^¼
Î ‹¬O|O^O
Î `³e†Ç¶O>è ãH÷Ok "Œ\÷…Õ U ㄬ=KÇ<Œ°
1) ªé^Î~¡°_È° 2) Hù_Ȱ䛽 3) =¶=°†Ç°¼ 4) "Í°#°Á_°È
YD

25. Jx…˜ ~˂²ì`Ÿ#° J`Çx †³òH›ø `ÇOã_ ƒì~¡¼ †³òH›ø XöH


J=‹¬~¡O. (S.I.-2012)
,H

1) P,Q †³òH›ø ƒ’~¡ë ‹¬‚¬ìÙ^Î~¡°x 䛽=¶~¡°_Qê „¬iKdžǰO KÍ̋#°. ~˂²ì`Ÿ


T

2) N †³òH›ø 䛀`Ç°~¡° M H÷ =°#=~Œ°


(S.I.-2007)
AM

Jx…˜ä›½ U q^ÎOQê |O^Î°=ô?

1) ^¥†Ç¶k 2) Hù_Ȱ䛽”
H

3) N †³òH›ø 䛀`Ç°~¡° V †³òH›ø =°#=~Œ°


M

3) ̄á`ÇOã_ (…è^¥) „²#`ÇOã_ / "Í°#=¶=° 4) J°Á_È°


4) P,V †³òH›ø ªé^Î~¡°_È°
AM
IG

26. H÷~¡}Ÿ D q^ÎOQê J<Œß_È°. ''~¡„¦¬ò `ÇeÁ <Œ `ÇeÁ †³òH›ø


(15-18) ãH÷Ok L#ß ‹¬=¶KŒ~Œxß *ìãQ®`ÇëQê KÇkq ~ò=Þ|_#
KH

UïHáH› „¬ôãuH›ÑÑ. Jx JO>è H÷~¡}Ÿ`Ë ~¡„¦¬òH÷ Q® ‹¬O|O^Î

ㄬ‰×߁䛽 ‹¬=¶^¥#O ~Œ†Ç°O_ :


R

"Í°q°? (Group-I-2012)
A,B,C,D,E,F ° P~¡°Q®°~¡° =¼ä›½ë°,, C,F†³òH›ø ªé^Îi. 1) `Œ`Ç 2) `ÇOã_
B,E †³òH›ø ƒ’~¡ë䛽 Hù_Ȱ䛽. A †³òH›ø `ÇOã_ D =°i†Çò
3) "Í°#=¶=° 4) J#ß/ `Ç=òà_È°
'F' †³òH›ø `Œ`Ç 'D' P ãQ®¶„¬ô…Õ "³ò`ÇëO W^Îí~¡° `ÇOã_È°°,
27. A R H÷ ªé^Î~¡°_È°, T S H÷ ªé^Îi, S AH÷ 䛽=¶~¡°_È°

=òQ®°¾~¡° ªé^Î~¡°° =°i†Çò XH› `ÇeÁ L<Œß~¡°. J„¬C_È° S `Ë R 䛽 U q^Î"³°Ø# ‹¬O|O^ÎO LOk?

(Communication S.I.-2012) (Group-II-2012)

15. ãH÷Ok "Œx…Õ `ÇeÁ Z=~¡°? 1) "Í°#°Á_È° 2) 䛽=¶~¡°_È° 3) ªé^Î~¡°_È° 4) `ÇOã_


1) E 2) D 3) B 4) A 28. XH› ¢‹Ôëx „¬iKdžǰO K͆ÇòKǶ XH› „¬ô~¡°+¬µ_È° J<Œß_È°.

16. E †³òH›ø ƒ’~¡ë Z=~¡°? ''P"³° `ÇeÁ <Œ J`ÇQ


ë êi †³òH›ø UïHH
á › „¬ôãuH›ÑÑ. P „¬ô~¡°+¬µ_È°

1) C 2) A 3) B 4) F P ¢‹ÔëH÷ U=°=ô`Œ_È°? (DL-2012)

17. P ãQ®¶„¬ô…Õ ZO`Ç =°Ok „¬ô~¡°+¬µ° L<Œß~¡°? 1) =¶=°/„²#`ÇOã_ 2) `ÇOã_


1) 5 2) 3 3) 4 4) 2 3) ªé^Î~¡°_È° 4) 䛽=¶~¡°_È°
REASONING ABILITY (VERBAL) ○ ○ ○ ○ ○ ○ ○ ○ ○
95
○ ○ ○ ○ ○ ○ ○ ○ ○ ○ ○ ○ ○ ○ ○ ○ ○ ○ ○
Study Material
29. ''~Œ}÷ †³òH›ø `ÇeÁ =¶ J=°à †³òH›ø UïHáH› 䛀`Ç°~¡°ÑÑ Jx 34. ~¡q XH› ƒì°x `Ç# ã̄¦O_£H÷ KǶ„²‹¬¶ë W…ì J<Œß_È°.

H›=°…˜ K³„²æ#KË H›=°…˜ ~Œ}÷H÷ U=°=ô`Œ_È°? J`Ç_È° <Œ ƒì~¡¼ †³òH›ø =°~¡^Îe UïHáH› 䛽=¶~¡°_È° Hêx

(A.P.Mun Junior A/c Off-2012) `Ç# `ÇeÁ^ÎOã_È°ä›½ `Œ<ùH›ø_<Í ‹¬O`Œ#O J~ò# P

1) `Œ`džǰ¼ 2) `ÇOã_ 3) "Í°#=¶=° 4) ªé^Î~°¡ _È° ƒì°_È° ~¡q †³òH›ø 䛽=¶~¡°_H÷ U=°=ô`Œ_È°?(SSC-2013)

30. JO[e W…ì J<³#° ''J`Ç_È° <Œ K³eÁ †³òH›ø ªé^Î~¡°x 1) Nephew 2) Cousin
`ÇOã_ †³òH›ø UïHáH› 䛽=¶~¡°_È°ÑÑ J~ò# JO[eH÷ P =¼H÷ë 3) Brother 4) Uncle
U=°=ô`Œ_È°? (SSC.2014) H ‹¬¶KÇ#° : (ㄬ. 35-39) XH› 䛽@°O|=ò…Õx P~¡°Q®°~¡°

1) H›l<£ 2) ªé^Î~¡°_È° 3) `ÇOã_ 4) JO䛽…˜ ‹¬ƒ’°¼° A, B, C, D, E, F JO^Î~¡¶ H›e‹²

31. A †³òH›ø `ÇeÁ B †³òH›ø K³eÁ =°i†Çò C †³òH›ø 䛽=¶ï~ë ㄬ†Ç¶}÷‹¬°ë<Œß~¡°. C B B


†³òH›ø 䛽=¶~¡°_È° Hêx †³òH›ø

D †³ ò H› ø `Ç e Á B =°i†Ç ò K³ e Á E J~ò# C,E H÷ `ÇeÁC Hê^ΰ. A, C C ° ̄ˆÜá¤# [O@. †³òH›ø ªé^Î~¡°_È°

U=°=ô`Ç°Ok? (IB-2013) E, A †³òH›ø D. B


䛽=¶ï~ë F. †³òH›ø ªé^Î~¡°_È°

1) ªé^Îi 2) `ÇeÁ (A.P. S.I. Mains 2017)

Y
3) `ÇOã_ 4) `ÇeÁ (…èH›) `ÇOã_ 35. D E H÷ `Ë Q® ‹¬O|O^ÎO Uk ?

A*B B A, A@B B

M
32. J#Qê †³òH›ø `ÇOã_ J#Qê †³òH›ø 1. `ÇOã_ 2. ªé^Î~¡°_È°
A,A!B
`ÇeÁ J#Qê B †³òH›ø ƒì~¡¼ A J~ò# D ãH÷Ok 3. JO䛽…˜ 4. x~ŒœiOKDžè=ò

E
"Œx…Õ'U' †³òH›ø <Œ#=°à 'T'Jx `³…èæ ‹¬O|O^ÎO? 36. E †³òH›ø ƒì~¡¼ Z=~¡°?

1. 2. B

D
9
(PA/SA-2013) A

59
1) T@R*Y!U 2) T@S*U!Y 3. C 4. x~ŒœiOKDžè=ò
CA
95
3) T@R!S!U 4) ̄áUgHê=ô 37. ãH÷Ok"Œx…Õ ¢‹Ôë [`Ç Uk?

Sunil Kesav SIMRAN, 1. 2. A, D

50
33. J#° =¼H÷ë †³òH›ø 䛽=¶~¡°_È°. D, F

Kesav Maruti, 3. 4. x~ŒœiOKDžè=ò


98
A
†³òH›ø K³…
Áÿ ° =°i†Çò `Ç# 䛽=¶~¡°_È° B, D

SITA; Prem, Maruti A


:8
䛽=¶ï~ë †³òH›ø "Í°#=¶=°. 38. H÷ Q® „²Á…ÿO^Î~¡°?

Maruti 1. 2. 3. 4.
E

1 2 3 4
h

J~ò# ‹¬°h…˜ J#° =¼H÷ë H÷ U=°=ô`Œ_È°?

F
.P
IC

(SSC-2013) 39. †³òH›ø `ÇOã_ Z=~¡°?

1) Nephew 2) 1. 2. 3. 4.
AD

Cousin A B C D

3) Uncle 4)
AB
O

Brother

12345678901234567890123456789012123456789012345678901234567890121234567890123456789012345678901212345678901234567890123
1234567890123456789012 123456789012345678901234
12345678901234567890123456789012123456789012345678901234567890121234567890123456789012345678901212345678901234567890123
1234567890123456789012 123456789012345678901234

q=~¡}ì`ÇàH› ["Œ|°°
1234567890123456789012 123456789012345678901234
12345678901234567890123456789012123456789012345678901234567890121234567890123456789012345678901212345678901234567890123
ER
CH

1234567890123456789012 123456789012345678901234
12345678901234567890123456789012123456789012345678901234567890121234567890123456789012345678901212345678901234567890123
1234567890123456789012
1234567890123456789012
1234567890123456789012 PREVIOUS BITS : 123456789012345678901234
12345678901234567890123456789012123456789012345678901234567890121234567890123456789012345678901212345678901234567890123
123456789012345678901234
12345678901234567890123456789012123456789012345678901234567890121234567890123456789012345678901212345678901234567890123
123456789012345678901234
12345678901234567890123456789012123456789012345678901234567890121234567890123456789012345678901212345678901234567890123
YD

(3) KÇ~}¡ Ÿ `ÇeHÁ ÷ ªé^Î~°¡ _È°, Hê[…˜ 䛽=¶~¡°_³#á J~¡°<û £ `ÇOã_ ̄á^¥x #°O_
,H

1.

JO>è Hê[…˜H÷ ƒ’~¡ë, Hê=ô# KÇ~¡}ŸH÷ Hê[…˜ =¶=° ƒì~¡¼ 'D' †³òH›ø ªé^Îi 'C', 'C' `ÇeÁ 'B', 'B' †³òH›ø `ÇOã_
T
AM

'A' Hê=ô# DH÷ 'A' `Œ`Ç J=ô`Œ_È°.


JO>è J`Çë (PO\©) J=ô`Ç°Ok.
H
M

6. (3) ^Î`Çë ‹¬=¶‹¬=ò


(4) ¢‹Ôë †³òH›ø <Œ#Qêi UïHáH› 䛽=¶ï~ë J#Qê P ¢‹Ôë<Í.
AM

2.
IG

P ¢‹Ôë †³òH›ø ƒ’~¡ë P „¬ô~¡°+¬µ_È°. Hê=ô# P „¬ô~¡°+¬µ_È° A – B + C × D


KH

¢‹Ôë †³òH›ø ƒ’~¡ë A – B + C × D


R

ªé^Î~¡°_È° `ÇOã_ ªé^Îi


3. (2) =°À‚ì+¹ <Œ#=°à Qêi 䛽=¶ï~ë †³òH›ø ªé^Î~¡°_È° J#Qê
̄á^¥x #°O_
=°À‚ì+¹ †³òH›ø <Œ#Qê~¡°. Hê=ô# =°À‚ì+¹KÍ F_O„¬|_#
C (J=¶à~ò). 'C' H÷ `ÇOã_ 'B', 'B' H÷ ªé^Î~¡°_È° 'A'.
=¼H÷ë J`Çx `ÇOã_.
Hê=ô# 'C', 'A' 䛽 ªé^Î~¡°x 䛽=¶ï~ë J=ô`Ç°Ok.
4. (4) A †³òH›ø ªé^Î~¡°x `ÇOã_, 'A' H÷ `ÇOã_ J=ô`Œ_È°.
7. (2, 3)
`ÇOã_ ƒì~¡¼ `ÇeÁ. 'A' †³òH›ø `ÇeÁH÷ `ÇeÁ 'P' Hê=ô#. ^Î`Çë ‹¬=¶KŒ~¡O ㄬHê~¡O,
P, A 䛽 J=°à=°à J=ô`Ç°Ok.
¢‹Ôë "Í°#=¶=°, ƒì°x "Í°#=¶=° J#Qê ƒì°x =¶=°.
5. (4) ^Î`Çë ‹¬=¶‹¬=ò J`Çx "Í°#=¶=° Jx `³°‹¬°ëOk. J#Qê ƒì°x
A + B Ó C × D "Í°#=¶=° P ¢‹Ôë †³òH›ø ªé^Î~¡°_È°. Hê=ô# ‹¬=¶^¥#O
A + B Ó C × D `Çe,Á Hù_Ȱ䛽 J=ô`Ç°Ok. Hùxß ‹¬O^Î~ŒÄ…Õ J`Ç,ë "Í°#°Á_°È
`ÇOã_ `ÇeÁ ªé^Îi 䛀_¨ Hê=KÇ°ó.
REASONING ABILITY (VERBAL) ○ ○ ○ ○ ○ ○ ○ ○ ○
96
○ ○ ○ ○ ○ ○ ○ ○ ○ ○ ○ ○ ○ ○ ○ ○ ○ ○ ○
Study Material
8. (4) ^Î`Çë ‹¬=¶KŒ~¡O ㄬHê~¡O, ∴ ¢‹Ôë †³òH›ø 䛽=¶~¡°_È° → JOH÷`Ÿ
†³òH›ø 䛀`Ç°~¡° `ÇOã_ , 25. (1) ~˂²ì`Ÿ `ÇOã_ †³òH›ø ƒì~¡¼ → ~˂²ì`Ÿ `ÇeÁ
†³òH›ø `ÇOã_H÷ XöH XH› ªé^Î~¡°_È°. JO>è H÷ J#° =¼H÷ë ~˂²ì`Ÿ `ÇeÁ †³òH›ø ‹¬‚ì¬ Ù^Î~°¡ _È° → ~˂²ì`Ÿ "Í°#=¶=°
ƒìƒì~ò J=ô`Œ_È°. ~˂²ì`Ÿ "Í°#=¶=° 䛽=¶~¡°_È° → Jx…˜
9. (2) ^Î`ŒëO‰×O ㄬHê~¡O, ∴ ~˂²ì`Ÿ, Jx…˜ä›½ ^¥†Ç¶k J=ô`Œ_È°.
䛽@°O|O…Õ P_È"Œi ‹¬OY¼ « =°Q®"Œi ‹¬OY¼ 26. (3) H÷~¡}Ÿ `ÇeÁ †³òH›ø „¬ôãuH› → ~¡„¦¬ò `ÇeÁ
A, E ° F 䛽 䛽=¶~¡°° Hê=ô# H÷~¡}Ÿ, ~¡„¦¬òH÷ "Í°#=¶=° J=ô`Œ_È°.
® A, E ° W^Îí~¡° =°Q®"Œ~¡° 27. (2) S 䛽=¶~¡°_È° → A; R →’ A’ ªé^Î~¡°_È°
® A H÷ B 䛽 䛽=¶~¡°_È° ∴ S H÷R 䛽=¶~¡°_È° J=ô`Œ_È°.
XH› ƒì°_È° (B), ƒìeH› W^ÎíiH© `ÇeÁ 'D' 28. (2)
Hê=ô# D, A ° ƒì~¡¼ƒ’~¡ë° J=ô`Œ~¡°. „¬ô~¡°+¬µx J`ÇëQêi †³òH›ø UïHáH› „¬ôãuH› → „¬ô~¡°+¬µx ƒì~¡¼

Y
Hê=ô# D H÷ 'A' ƒ’~¡ë. ƒì~¡¼ „¬ôãuH› → „¬ô~¡°+¬µx „¬ôãuH›

M
10. (2) H÷~¡}Ÿ `ÇOã_ 䛀`Ç°~¡° → H÷~¡}Ÿ †³òH›ø ªé^Îi JO>è P ¢‹Ôë „¬ô~¡°+¬µx 䛀`Ç°~¡°.
ªé^Îi 䛽=¶~¡°_È° → "Í°#°Á_È°. ¢‹ÔëH÷ „¬ô~¡°+¬µ_È° `ÇOã_.

E
∴ H÷~¡}Ÿ ‡¦Ú\ՅÕx =¼H÷ëH÷ ''"Í°#=¶=°ÑÑ J=ô`Œ_È°. 29. (3) "Í°#=¶=° J=ô`Œ_È°.

D
30. (2) W=Þ|_# ^Î`ŒëO‰×O ㄬHê~¡O

9
11. (4) Wzó# ‹¬=¶KŒ~¡O ㄬHê~¡O

59
JO[e K³…ÿÁe ªé^Î~¡°_È° †³òH›ø `ÇOã_ → JO[e
=ò~¡m → ~¡"Í°+¹ 䛽=¶~¡°_È°
CA
95
~¡"Í°+¹ ƒì~¡¼ → ƒ’"Œh `ÇOã_ Hê=ô# P =¼H÷ë JO[e †³òH›ø ªé^Î~¡°_È°.

50
JO>è =ò~¡mH÷ ƒ’"Œh '`ÇeÁÑ J=ô`Ç°Ok. 31. (4) W=Þ|_# ^Î`ŒëO‰×O#° „¬ijeOz KǶÀ‹ë C †³òH›ø
98
A
(3) 䛽=¶ï~ë E J~ò# C †³òH›ø eOQ®=° Q®¶ió ㄬ‰×߅Õ
:8
12.

(1) †³òH›ø 䛽=¶~¡°° W=ޅè^ΰ. Hê=ô# C J#°=¼H÷ë E †³òH›ø `ÇeÁ (…è^¥)


E


h

13. A B,E
`ÇOã_ J=ô`Œ~¡°.
.P

Ñx q"Œ‚¬ìO K͋¬°Hù<³#°.
IC

'D 'E'
32. (2) W=Þ|_#…Õx ㄬHê~¡O "³o¤#
AD

Hê=ô# D,A H÷ HË_ȁ° J=ô`Ç°Ok.


T@SU!Y → S †³òH›ø `ÇeÁ T
(4)
AB
O

14.
→ U †³òH›ø `ÇOã_ S, → Y †³òH›ø ƒì~¡¼ U
ER
CH

(15-18) Wzó# ‹¬=¶KŒ~¡O ㄬHê~¡O


∴ T J#°#k U †³òH›ø `ÇOã_ †³Ø°# S †³òH›ø `ÇeÁ U
→ ¢‹Ôë, E → ¢‹Ôë, B → „¬ô~¡°+¬µ_È°
YD

C
†³òH›ø <Œ#=°à T.
D→ „¬ô~¡°+¬µ_È° B,F ° „¬ô~¡°+¬µ°
33. (2) W=Þ|_# ^Î`ŒëO‰×O ㄬHê~¡O „¬ijeOKÇQê Sunil
,H

C «ê_Ïj F aaÈ D 圾>·¢±‚± A „í¢½ E >±jl


T

B
†³òH›ø J`Çë Simran, Simran †³òH›ø 䛽=¶~¡°_È°
AM

(1) ̄á #°O_ 'E' J#°=¼H÷ë '`ÇeÁÑ Jx `³°‹¬°ëOk.


H

15. Maruti, Sunil †³òH›ø Cousin =¶~¡°u.


M

(2) E †³òH›ø ƒ’~¡ë 'A' 34. (2) W=Þ|_# ^Î`ŒëO‰×O ㄬHê~¡O „¬ijeOKÇQê ~¡q †³òH›ø
AM

16.
IG

(3) #°Q®°~¡° (F,D,A,B) ƒì= Qêi „²Á"Œ_È° - P ƒì°_È°. Hê=ô# ~¡q †³òH›ø
KH

17.

(1) F †³òH›ø `ÇeÁ 'E' 䛽=¶~¡°_È° P ƒì°xH÷ Cousin J=ô`Œ_È°.


R

18.

19. (3) Wzó# ‹¬=¶KŒ~¡O ‹¬i‡é^ΰ. Nephew ("Í°#°Á_È°/ (ㄬ. 35-39)

=~¡‹¬ä›½ Hù_Ȱ䛽) &


$
" 

20. (3) JO䛽…˜ J=ô`Œ_È°.


21. (2) Wzó# ‹¬=¶KŒ~¡O ㄬHê~¡O, P =¼H÷Hë ÷ ¢‹ëÔ '`ÇeÑÁ J=ô`Ç°Ok. #
% 
'

22. (3) B †³òH›ø `ÇOã_ A, A †³òH›ø 䛀`Ç°~¡° C


35. JO䛽…˜
∴ B =°i†ÇòE ‹¬O|O^ÎO J#ß =°i†Çò K³…ÿÁ°.
36. x~ŒœiOKDžè=ò
23. (4) A → B, C †³òH›ø `ÇeÁ
C †³òH›ø ƒ’~¡ë D'
37. A, D ¢‹Ôë [`Ç
∴ D H÷ A J`ÇëQê~¡° J=ô`Ç°Ok. 38. 'A' H÷ =òQ®°¾~¡° ‹¬O`Œ#O H›~¡°.
24. (2) ¢‹Ôë `ÇeÁH÷ XH›øQê<ùH›ø 䛀`Ç°~¡° → P ¢‹Ôë†Í° 39. 'F' †³òH›ø `ÇOã_ 'C'

REASONING ABILITY (VERBAL) ○ ○ ○ ○ ○ ○ ○ ○ ○


97
○ ○ ○ ○ ○ ○ ○ ○ ○ ○ ○ ○ ○ ○ ○ ○ ○ ○ ○
Study Material
k‰§`ÇàH› „¬sH›Æ
7 (Direction Sense Test)

=òMì¼O‰§°...
G k‰§`ÇàH› „¬sH›Æ…Õx ㄬ‰×߁° ª^¥~¡}OQê Jƒ’¼iÖ †³òH›ø „¬ij<Œ *ìý<Œxß, k‰§ x~Œœ~¡}ì *ìý<Œxß =°i†Çò "ÍQêxß
JOKÇ<Œ "͆Çò@䛽 L^ÍítO„¬|_#q.
G D qƒìQ®O…Õx ㄬ‰×߁#° ªkOKÇ°#„¬ô_È° Jƒ’¼iÖ "³ò^Î@ k‰× †³òH›ø ~¡„¦¹ „¬@O#° `DŽ¬æH› w‹¬°HË=…ÿ#°.
G D qƒìQ®O…Õx ㄬ‰×߁…Õ Jƒ’¼iÖH÷ XH› =¼H÷ë Q®°iOzQêh …è^¥ XH› =‹¬°ë=ô Q®°iOz, "Œ\÷ †³òH›ø k䛽ø #°O_ ~¡H›~¡HꁰQê

Y
#_z "³o¤# …èH› 䛽_, Z_È=° "³á„¬ô uiy<Œ PY~¡°ä›½ U k‰×…Õ =ô<Œß~Ë, ZO`Ç ^ζ~¡O…Õ =ô<Œß~Ë q‰õÁ+²OKÇ=°x

M
J_È°Q®°`Œ~¡°.
G D qƒìQ®O…Õx ㄬ‰×߁#° ªkOKÇ°@䛽 Jƒ’¼iÖ. ㄬ‰×ß…Õ W=Þ|_# =¼H÷ëx `Ç#䛽 `Œ#°Qê T‚²ìOKǰ䛽x, ㄬ‰×ß…Õ W=Þ|_#

E
䛽_, Z_È=° k䛽ø#° `Ç# 䛽_ Z_È=°°Qê ƒìqOz K͆Çò@ ^¥Þ~Œ "ÍQ®OQê ‹¬=¶^¥#O Q®°iëOKÇ=KÇ°ó.

D
9
G D qƒìQ®O…Õx ㄬ‰×߁#° k‰×#° P^¥~¡OQê K͋¬°ä›½x W=Þ_ÈO [~¡°Q®°`Ç°Ok.

59
G k‰§`ÇàH› „¬sH›Æ…Õ k䛽ø°, "Œ\÷ =°^Î¼ ^ζ~Œxß Q®°iëOKÇ_ÈO ̄á# ㄬ‰×߁° W=Þ|_È`Œ~ò.
CA
95
G D qƒìQ®O…Õx ㄬ‰×߁#° ªkOKÇ°@䛽 Jƒ’¼iÖH÷ k‰×°, =ü°. k‰× =°^Î¼ HË}ìÌ„á „¬îië J=Qꂬì# =ôO_¨e

50
=°i†Çò h_ȁ̄á 䛀_¨ J=Qꂬì# =ôO_¨e.
98
A
G O|HË} ãuƒ’°[O U~¡æ_#„¬ô_È° L„¬†³¶yOKŒež# ‹¬¶ã`ÇO H›~¡â=ò « 3 3.
:8
)„“±\>ó* |)„“±\>ó*
E

G =¼H÷ë ㄬ†Ç¶}÷Oz# =¶~¡¾O XH› O|HË} ãuƒ’°*ìxß U~¡æiÀ‹ë ̄á^Š¥Q®~¡‹¹ ‹²^¥œO`ÇO L„¬†³¶yOz ^ζ~¡O =°i†Çò k‰×
h
.P

H›#°Qù<Œe.
IC
AD

G D qƒìQ®O…Õx ㄬ‰×߁#° ªkOKÇ°@䛽 H›h‹¬O 1 #°O_ 30 =~¡ä›½ Q® ‹¬OY¼ =~Œ¾° `³e‹²=ôO>è =°Ozk.
G k䛽ø° : k䛽ø° (…è^¥) k‰×° <Œ°Q®°. Jq L`Çë~¡O, ^ÎH÷Æ}O, `Ƕ~¡°æ =°i†Çò „¬_È=°~¡.
AB
O

Ma铢P! /PSUI

ER
CH
YD
,H

90º 90º
…­`É>±¢! 8FTU
aÈ·¢±ç! &BTU

T
AM

90º 90º
H
M

_ÏIøÇ~P! 4PVUI

AM
IG

; Imp. POINTS :
KH

1) XH› =¼H÷ë U k‰×…Õ =ô#߄¬æ\÷H© 䛽_"³á„¬ô䛽 u~¡°Q®°@#° ‹¬=¼k‰×…Õ u~¡Q®@O JO\ì~¡°.


R

2) =¼H÷ë U k‰×…Õ =ô#߄¬æ\÷H© Z_È=°"³á„¬ô䛽 u~¡°Q®°@#° J„¬‹¬=¼k‰×…Õ u~¡Q®@O JO\ì~¡°.


3) XH› =¼H÷ë ‹¬¶~˼^Άǰ ‹¬=°†Ç¶# ^ÎH÷Æ} k䛽ø#° KǶ‹¬¶ë xOKÇ°O>è J`Çx h_È J`ÇxH÷ 䛽_"³á„¬ô =ôO@°Ok.
4) XH› =¼H÷ë ‹¬¶~Œ¼‹¬ë=°†Ç° ‹¬=°†Ç¶# ^ÎH÷Æ} k䛽ø#° KǶ‹¬¶ë xOKÇ°O>è J`Çx h_È J`ÇxH÷ Z_È=°"³á„¬ô =ôO@°Ok.
5) ‹¬¶~˼^Άǰ ‹¬=°†Ç¶# XH› =¼H÷ë ‹¬¶~¡°¼x "³á„¬ô uiy KǶÀ‹ë P =¼H÷ë KǶÀ‹ "³á„¬ô `Ƕ~¡°æ, g„¬ô "³á„¬ô „¬_È=°~¡, P =¼H÷ë
䛽_"³á„¬ô ^ÎH÷Æ}O =°i†Çò Z_È=°"³á„¬ô L`Çë~¡O k䛽ø° =ôO\ì~ò.
6) ‹¬¶~˼^Ά°Ç ‹¬=°†Ç¶# XH› =¼H÷ë ‹¬¶~¡°¼x "³„á ô¬ uiy KǶÀ‹ë P =¼H÷ë h_È J`Çx "³#°H›"„á³ ô¬ 䛽 J#Qê „¬_=È °~¡ k䛽ø䛽 =ôO@°Ok.
7) ‹¬¶~Œ¼‹¬ë=°†Ç° ‹¬=°†Ç¶# XH› =‹¬°ë=ô †³òH›ø h_È ZÁ„¬ô_ȶ `Ƕ~¡°æ k䛽øQê =ôO@°Ok.
8) XH› =¼H÷ë ‹¬¶~˼^Άǰ ‹¬=°†Ç¶# L`Çë~¡ k䛽ø䛽 KǶ‹¬¶ë xOKÇ°O>è J`Çx h_È J`ÇxH÷ Z_È=°"³á„¬ô =ôO@°Ok.
9) XH› =¼H÷ë ‹¬¶~Œ¼‹¬ë=°†Ç° ‹¬=°†Ç¶# L`Çë~¡k䛽ø䛽 KǶ‹¬¶ë xOKÇ°O>è J`Çx h_È J`ÇxH÷ 䛽_"³á„¬ô =ôO@°Ok.
10) =°^¥¼‚¬ìßO 12 Q®OII ‹¬=°†Ç°O…Õ ‹¬¶~¡¼H÷~¡}쁰 x@“x°=ôQê =ôO\ì~ò. J„¬C_È° h_È U~¡æ_È^ΰ.
REASONING ABILITY (VERBAL) ○ ○ ○ ○ ○ ○ ○ ○ ○
98
○ ○ ○ ○ ○ ○ ○ ○ ○ ○ ○ ○ ○ ○ ○ ○ ○ ○ ○
Study Material
G =ü° : =ü° <Œ°Q®°. Jq PöQ߆ǰO, <³á~¡°u, "Œ†Çò=¼O =°i†Çò D‰§#¼O.
; `Ƕ~¡°æ, L`Çë~¡O =°^Î¼Qê =ôO_È° =ü#° 'D‰§#¼O (N - E)Ñ JO\ì~¡°.
; `Ƕ~¡°æ, ^ÎH÷Æ}O =°^Î¼Qê =ôO_È° =ü#° 'PöQ߆ǰO (S - E)Ñ JO\ì~¡°.
; „¬_È=°~¡, L`Çë~¡O =°^Î¼Qê =ôO_È° =ü#° '"Œ†Çò=¼O (N - W)Ñ JO\ì~¡°.
; „¬_È=°~¡, ^ÎH÷Æ}O =°^Î¼Qê =ôO_È° =ü#° '<³á~¡°u (S - W)Ñ JO\ì~¡°.
Ma铢P! /

#‡Èó>½P! /8
EŠ¨$½P! /&

¡ ¡
…­`É>±¢! 8
¡
¡
¡
¡
aÈ·¢±ç! &

¡ ¡

Y
=´â¢±v! 48
Q÷Rà‡È±P! 4&

M
_ÏIøÇ~P! 4

E
G D P^¥¼†Ç°O #°O_ ï~O_È° ~¡Hê ã„¬‰×߁° J_È°Q®°`Œ~¡°.

D
9
Jq : 1) ï~O_È° ㄬ^͉×=ò =°^Î¼ ^ζ~¡O ä›#°Qù#°@. 2) k‰×#° H›#°Qù#°@.

59
CA
95
=¶ki ㄬ‰×߁°
50
98
A
:8
MODEL - 1 MODEL - 2
E
h
.P

o
1. „¬_È=°~¡ "³á„¬ô #_È°‹¬°ë#ß =¼H÷ë 225 ‹¬=¼k‰×…Õ uiy`Í 1. ~¡q `Ƕ~¡°æ "³á„¬ô#H÷ 4 H÷.g°. #_z, uiy Z_È=°
IC
AD

`Ç#ïH^ΰ~¡°Qê#°#ß k䛽ø Uk? "³á„¬ô#H÷ 3 H÷.g°. #_À‹ë |†Ç°°^Íi# ªÖ#O #°O_


AB
O

ª^Î#: ‹¬=¼k‰× JO>è Q®_†Ç¶~¡„¬ô =ò°Á k‰×. 䛽_"³á„¬ô䛽 ZO`Ç ^ζ~¡O…Õ LO\ì_È° ?
u~¡°Q®°@ ‹¬=¼k‰×. „¬_=È °~¡ #°O_ 180o Z@°"³„á ô¬ uiy<Œ
ER

/
CH

3
ª^Î#:
`Ƕ~¡°æ k‰× =‹¬°Oë k. `Ƕ~¡°æ #°O_ 45o 䛽_"„á³ ô¬ uiy`Í 4!Iø/h±/ 8 &
YD

PöQ߆ǰ=ü LO@°Ok. (180o+45o=225o). 1


5!Iø/h±/
2 4
,H

2. XH› =¼H÷ë „¬_È=°~¡ "³á„¬ôQê x°óx, ‹¬=¼k‰×…Õ 45°


T
AM

HË}=ò`Ë =°i†Çò `Ç~¡°"Œ`Ç 180° ° J^Í k‰×…Õ )¢r!}‡È±‚±_Îj$!«‘×$P*


H

uiy `Ç^ΰ„¬i 270°° J„¬‹¬=¼ k‰×…Õ uiïQ#°. H›~¡â=ò « (ƒ’°[O)2 { (ƒ’°[O)2


M
AM

ㄬ‹¬°ë`ÇO J`Ç_È° U k‰×…Õ x°ó<Œß_È°.


IG

13  12
 23
   
M
KH

=    H÷.g°.
#
R

2. ~¡„¦¬ò 20 H÷.g°. `Ƕ~¡°æ k䛽øQê #_K³#°. JH›ø_ #°O_


¡
" 21 H÷.g°. ^ÎH÷Æ} k‰×Qê #_K³#°. W„¬C_È° ~Œ„¦¬°"ÍOã^Î
…­ ¡
aÈ·
¡
`Ç# ㇐~¡Oƒ’ ªÖ#O KÍ~¡=…ÿ##ß Zxß H÷…Õg°@~¡Á
$ ^ζ~¡O #_È"Œe? =°i†Çò ㄬ‹¬°ë`ÇO J`Ç#° U k‰×…Õ
=ô<Œß_È° ? /

ª^Î#: „¬@O #°O_,
ª^Î#: XH› =¼H÷ë "³ò^Î\ ÷ ªÖ#=ò OA°
‹¬=¼k‰×…Õ 45° uiy# `Ç~¡°"Œ`Ç ªÖ#=ò OB° =°~¡ 0#  0"  "# 8
31!Iø/h±/ " &

0 h
øI/
‹¬=¼k‰×…Õ 180° ハ=’ °}O `Ç~°¡ "Œ`Ç ªÖ#O OC° z=~¡Qê     
!2
   3
J„¬‹¬=¼k‰×…Õ 270° ハ’=°}O `Ç~¡°"Œ`Ç ªÖ#O OD. #
⇒ OD <³á~¡°u k‰×#° ‹¬¶z‹¬°ëOk.   « 29 H÷.g°. 4

REASONING ABILITY (VERBAL) ○ ○ ○ ○ ○ ○ ○ ○ ○


99
○ ○ ○ ○ ○ ○ ○ ○ ○ ○ ○ ○ ○ ○ ○ ○ ○ ○ ○
Study Material
Z) ㇐~¡Oƒ’ ªÖ#O KÍ~¡°@䛽 ~¡„¦¬ò 29 H÷.g°. ^ζ~¡O MODEL - 5
#_È"Œe.
a) `Ƕ~¡°æ-^ÎH÷Æ}O =°^Î¼# PöQ߆ǰO =ôO@°Ok. 1. =~¡°}Ÿ, J~¡°}ŸH÷ 40 g°. ^ζ~¡O…Õ PöQ߆ǰO…Õ
Hê=ô# ~¡„¦¬ò ㄬ‹¬°ë`ÇO PöQ߆ǰ k‰×…Õ =ô<Œß_È°. L<Œß_È°. H›$+¬â, J~¡°}ŸH÷ 40 g°. ^ζ~¡O…Õ <³á|°°u…Õ
MODEL - 3 L<Œß_È°. J„¬C_È° H›$+¬â, =~¡°}ŸH÷ U k䛽ø…Õ L<Œß_È°?
ª^Î#: Wzó# ‹¬=¶KŒ~¡O P^¥~¡OQê XH› z`Ç°ë ƒç=°à wÀ‹ë
1. XH› =¼H÷ë J`Çx `ǁ ãH÷OkH÷, Hêˆ×ä ̄áH÷ ƒÿ\ ÷“ †³¶Qê
K͋¬°ë<Œß_È°. J`Çx =òYO „¬_È=°~¡H÷ Ja=òYOQê K¢±~  /
51h±/ 51h±/
LO>è, J`Çx †³òH›ø Z_È=°KÍ~ò U k‰×…Õ L#ßk? 8 &
Iœ%,­ã >¢±~  4
ª^Î#: D ㄬ H › ø „¬ @ O ^¥Þ~Œ / )M*
†³ ¶ Qꋬ # O "Í ‹ ² # =°x+² [`É>±!LÎó „¬@=ò#° „¬ijeOz# =~¡°}ŸH÷ H›$+¬â „¬_È=°~¡ k䛽ø…Õ
8 )…­* aȂ

Y
†³òH›ø Z_È=° KÍ~ò L`Çë~¡O 圾`Ž!LÎó L<Œß_È°.
4 )_Ï*

M
k‰×"³á„¬ôQê LO^Îx `³‹¬°ëOk. 2. P, Q, R, S° Hê¼~¡"£°ƒÕ~¡°Û P@ P_È°`Ç°<Œß_È°. P, Q
2. ~¡q L^Î~ò‹¬°ë#ß ‹¬¶~¡°¼xß ^¥¼x‹¬°ë<Œß_È°. J`Çx XH› [@°“ S L`Çë~¡O "³á„¬ô uiy 䛀~¡°ó<Œß_È°. P „¬_È=°~¡

E
䛽_KÍ~ò U"³á„¬ô L#ßk? "³„á ô¬ uiy 䛀~¡°óO>è, ^ÎH}
Æ÷ O䛽 uiy Z=~¡° 䛀~¡°ó<Œß~¡°?

D
9
ª^Î#: ㄬ‰×ß…Õ Wzó# ‹¬=¶KŒ~¡O P^¥~¡OQê XH› ƒç=°à

59
ª^Î#: / )M*
w‹²#KË
CA
95
[`É>±!LÎó
>½Iøì & )aÈ·* 3

50
圾`Ž!LÎó 2 1
98
A
Œ­·¢±½`ɱ
4 )_Ï* 4
:8
^ÎH÷Æ}O "³á„¬ô#䛽 uiy 䛀~ùó<Œß_È°.
E

'R'
h

MODEL - 4
.P

3. A, B, C, D J<Í #°Q®°~¡° J#ß^Î=òà° P@‹¬ÖO…Õ


IC
AD

1. D‰§<Œ¼xß `Ç ¶ ~¡ ° æQê#°, <³ á ~ ¡ ° ux „¬ _ È = °~¡ Q ê#°, x|_ L<Œß~¡°. "Œi…Õ A, C䛽 "Œ†Çò=¼ k‰×…Õ
PöQ߆Ƕxß ^ÎH÷Æ}OQê#° „²eó#„¬ô_È°, L`Çë~¡ k䛽ø#° L<Œß_È°. B, A䛽 <³~á °¡ u k‰×…Õ L<Œß_È°. D, A =°i†Çò
AB
O

U=°x „²"Œe ? C =ô#ß =¶~¡¾O…Õ<Í =ôO_ B 䛽 Yzó`ÇOQê L`Çë~¡


ER
CH

ª^Î#: M k‰×…Õ =ô<Œß_È°. J~ò`Í D䛽 C U k‰×…Õ L<Œß_È°?


YD

# E ª^Î#: Wzó# ‹¬=¶KŒ~¡O P^¥~¡OQê ƒç=°à w‹²#KË


,H

%
T

…­ aÈ·
AM

"
H

Q
M

=´â # $
AM
IG


„¬@O #°O_ D䛽 'C' PöQ߆ǰO k‰×…Õ L<Œß_È°.
KH

Ì„á „¬@O #°O_ L`Çë~Œxß D‰§#¼O JO\ì=ò.


4. XH› ƒìeH› `Ç# WO\÷ #°O_ |†Ç°°^Íi, "³ò^Î@Qê
R

2. XH› Q®_†Ç¶~¡O…Õ ‹¬=°†Ç°O 3.00 pm J~òOk.


30 g°II "Œ†Çò=¼ k‰×…Õ #_z# `Ç~¡°"Œ`Ç 30 g°II
nx…Õ xq°‘ =òˆ×ä D‰§#¼O "³á„¬ôQê LO>è, <³á~¡°u k‰×Qê #_K³#°. =°~¡ 30 g°II PöQ߆ǰO "³á„¬ô
Q®O@ =òˆ×ä U k‰× "³á„¬ôQê LO_È°#°? #_z z=~¡Qê `Ç# WO\÷"³á„¬ô uiïQ#°. ㄬ‹¬°ë`ÇO P
/& ƒìeH› U k‰×…Õ #_È°‹¬°ë#ßk?
   /
 
& ª^ Î # : ƒìeH› †³òH›ø H›^ÎeH›° #
  /8 4& A→B→C→D→A „¬@O
ª^Î#:  
8 4 #O^ΰ KǶ„¬|_#q.
  
48 $ "
z=~¡Qê P ƒìeH› DA k‰×…Õ
Ì„á „¬@O ^¥Þ~Œ Q®O@ =òˆ×ä 'PöQ߆ǰOÑ "³á„¬ôQê #_È°‹¬°ëOk.
LO_È°#°. J#Qê D‰§#¼O. %

REASONING ABILITY (VERBAL) ○ ○ ○ ○ ○ ○ ○ ○ ○


100
○ ○ ○ ○ ○ ○ ○ ○ ○ ○ ○ ○ ○ ○ ○ ○ ○ ○ ○
Study Material
123456789012345678901234567890121234567890123456789012345678901212
12345678901234567890123456789012123456789012345678901234567890121234567890123456789012345678901212345678901234567890123456789012123
1234567890123456789012345678901212345678901234567890123456789012123456789012345678901234567890
12345678901234567890123456789012123456789012345678901234567890121234567890123456789012345678901212345678901234567890123
1234567890123456789012345678901212345678901234567890123456789012123456789012345678901234567890121234567890123
1234567890123456789012345678901212345678901234567890123456789012123456789012345678901234567890121234
123456789012345678901234567890121234567890123456789012345678901212
12345678901234567890123456789012123456789012345678901234567890121234567890123456789012345678901212345678901234567890123456789012123
1234567890123456789012345678901212345678901234567890123456789012123456789012345678901234567890
12345678901234567890123456789012123456789012345678901234567890121234567890123456789012345678901212345678901234567890123
1234567890123456789012345678901212345678901234567890123456789012123456789012345678901234567890121234567890123
1234567890123456789012345678901212345678901234567890123456789012123456789012345678901234567890121234
12345678901234567890123456789012123456789012345678901234567890121234567890123456789012345678901212345678901234567890123456789012123
1234567890123456789012345678901212345678901234567890123456789012123456789012345678901234567890
123456789012345678901234567890121234567890123456789012345678901212
12345678901234567890123456789012123456789012345678901234567890121234567890123456789012345678901212345678901234567890123
1234567890123456789012345678901212345678901234567890123456789012123456789012345678901234567890121234567890123
1234567890123456789012345678901212345678901234567890123456789012123456789012345678901234567890121234
PRACTICE TEST
12345678901234567890123456789012123456789012345678901234567890121234567890123456789012345678901212345678901234567890123456789012123
1234567890123456789012345678901212345678901234567890123456789012123456789012345678901234567890
123456789012345678901234567890121234567890123456789012345678901212
12345678901234567890123456789012123456789012345678901234567890121234567890123456789012345678901212345678901234567890123
1234567890123456789012345678901212345678901234567890123456789012123456789012345678901234567890121234567890123
1234567890123456789012345678901212345678901234567890123456789012123456789012345678901234567890121234
12345678901234567890123456789012123456789012345678901234567890121234567890123456789012345678901212345678901234567890123456789012123
1234567890123456789012345678901212345678901234567890123456789012123456789012345678901234567890
12345678901234567890123456789012123456789012345678901234567890121234567890123456789012345678901212345678901234567890123
1234567890123456789012345678901212345678901234567890123456789012123456789012345678901234567890121234567890123
1234567890123456789012345678901212345678901234567890123456789012123456789012345678901234567890121234
12345678901234567890123456789012123456789012345678901234567890121234567890123456789012345678901212345678901234567890123456789012123
1234567890123456789012345678901212345678901234567890123456789012123456789012345678901234567890
123456789012345678901234567890121234567890123456789012345678901212
12345678901234567890123456789012123456789012345678901234567890121234567890123456789012345678901212345678901234567890123456789012123
1234567890123456789012345678901212345678901234567890123456789012123456789012345678901234567890
12345678901234567890123456789012123456789012345678901234567890121234567890123456789012345678901212345678901234567890123
1234567890123456789012345678901212345678901234567890123456789012123456789012345678901234567890121234567890123
1234567890123456789012345678901212345678901234567890123456789012123456789012345678901234567890121234
12345678901234567890123456789012123456789012345678901234567890121234567890123456789012345678901212345678901234567890123456789012123
1234567890123456789012345678901212345678901234567890123456789012123456789012345678901234567890

1. N#° ~¡qH÷ 䛽_"³á„¬ô 40 g°. ^ζ~¡O…Õ L<Œß_È°. ~ŒA 7. <Í#° "³ò^Î@ ^ÎH÷Æ} k䛽ø䛽 #_z, Z_È=°"³á„¬ô䛽 uiy
N#°ä›½ 60 g°. ^ζ~¡O…Õ ^ÎH÷Æ} k‰×…Õ L<Œß_È°. ª~ò =°~¡…ì Z_È=°"³á„¬ô䛽 uiy P `Ç~ŒÞ`Ç ä›½_"³á„¬ô䛽 uiy`Í
25 g°. ^ζ~¡O…Õ ~¡qH÷ „¬_È=°~¡ k䛽ø…Õ L<Œß_È°. ªQ®~Ÿ W„¬ô_È° U k‰×Qê ㄬ†Ç¶}O K͋¬°ë<Œß#°?
~ŒA#䛽 L`Ç~ë ¡ k䛽øQê 90g°. ^ζ~¡O…Õ L<Œß_È°. J~ò# 1) `Ƕ~¡°æ 2) „¬tó=°O 3) D‰§#¼O 4) L`Çë~¡O
N#°ä›½ Z_È=°"³á„¬ôQê „¬_È=°~¡ k‰×…Õ L#ß"Œi À„ö~q°\÷? 8. H÷‘é~Ÿ L`Çë~¡O "³á„¬ô 3½ H÷.g°. #_z Z_È=°"³á„¬ô 1½
1) ª~ò Ê ~¡q 2) ~ŒA H÷.g°. #_K³#°. J„¬ô_È° Z_È=°"³á„¬ô uiy =°~Ë 3½
3) N#° Ê ~¡q 4) N#° Ê ~ŒA H÷.g°. #_K³#°. J`Ç#° |†Ç°°^Íi# KË@ #°O_ ZO`Ç
2. 'X' J#° =¼H÷ë XH› ㄬ^͉×O #°O_ |†Ç°°^Íi 1 H÷.g°. ^ζ~¡O…Õ L<Œß_È°?

Y
#_z# `Ç~ŒÞ`Ç, Z_È=°"³á„¬ô#䛽 uiy ½ H÷.g°. #_K³#°. 1) 5 H÷.g°. 2) 6 H÷.g°.
P `Ç~ŒÞ`Ç =°m¤ `Ç# Z_È=°"³á„¬ô#䛽 uiy ㄬ†Ç¶}O 3) 1½ H÷.g°. 4) 4 H÷.g°.

M
`Ç ¶ ~¡ ° æ#ä› ½ Hù#ªy‹¬ ° ë < Œß_È ° . J~ò<Œ J`Ç # ° 9. H›$+¬â 'A' #°O_ |†Ç°°^Íi#, ^ÎH÷Æ} k‰×Qê 10 H÷.g°.

E
|†Ç°°^Íi# ªÖ#O U k‰×…Õ LOk? ㄬ†Ç¶}÷OK³#°. `Ç~ŒÞ`Ç `Ç# 䛽_"³á„¬ô#䛽 uiy 4 H÷.g°.
1) „¬_È=°~¡ 2) `Ƕ~¡°æ 3) ^ÎH÷Æ}O 4) L`Çë~¡O

D
9
#_KŒH›, =°m¤ `Ç# 䛽_"³á„¬ô䛽 uiy 10 H÷.g°. #_KŒH›

59
3. [†Í°Oã^Î L`Çë~¡O "³á„¬ô䛽 3 g°. #_z, „¬tó=°O "³á„¬ô䛽 `Ç# Z_È=°"³á„¬ôQê 10 H÷.g°. #_K³#°. J~ò<Œ J`Ç#°
CA
95
uiy 2 g°. #_K³#°. =°~¡…ì L`Çë~¡O "³á„¬ô#䛽 uiy 'A' #°O_ ZO`Ç ^ζ~¡O…Õ L<Œß_È°?

50
1 g°. #_z z=~¡Qê `Ƕ~¡°æ "³á„¬ô#䛽 uiy 5 g°. 1) 10 H÷.g°. 2) 16 H÷.g°.
98
A
#_ z # J`Ç # ° |†Ç ° °^Í i # ªÖ # O #°O_ ZO`Ç 3) 14 H÷.g°. 4) 20 H÷.g°.
:8
^ζ~¡O…Õ =ô<Œß_È°? 10. XH›"Íˆ× `Ƕ~¡°æk‰× / PöQ߆ǰ k‰×Qê ªÖ#ハ’O‰×O K³Ok`Í,
E
h

1) 8 g°. 2) 5 g°. 3) 10 g°. 4) 12 g°. „¬_È=°~¡ k‰× U k‰×Qê ªÖ#ハ’O‰×O K³O^ΰ`Ç°Ok?


.P
IC

4. 'A' J#° XH› ãQê=°O 'B' J#° ãQê=°O#䛽 L`Ç~ë O¡ Qê 20 1) D‰§#¼O 2) "Œ†Çò=¼O
AD

H÷.g°. ^ζ~¡O…Õ LOk. 'C' J#° =°~Ë ãQê=°O 'B' 3) `Ƕ~¡°æ 4) L`Çë~¡O
AB
O

ãQê=°O#䛽 18 H÷.g°. `Ƕ~¡°æ# LOk. ‹¬¶~¡¼ J#° =¼H÷ë 11. XH› "Œ‚¬ì#O `Ƕ~¡°æ k‰×Qê 9 H÷.g°. ㄬ†¶Ç }÷Oz# `Ç~ŒÞ`Ç
ER
CH

'C' J#° ãQê=°O #°O_ 'A' #䛽 |†Ç°°^Í~ï #°. J#Qê ^ÎH}Æ÷ k‰×Qê 6 H÷.g°. ㄬ†¶Ç }÷OK³#°. P `Ç~ŒÞ`Ç =°m¤
YD

J`Ç#° |†Ç°°^Íi# ªÖ#O #°O_ U k‰×…Õ L<Œß_È°? "Œ‚¬ì#O „¬_=È °~¡k‰×Qê 9 H÷.g°. ㄬ†¶Ç }÷OK³#°. J~ò<Œ
1) L`Çë~¡O 2) "Œ†Çò=¼O
,H

P "Œ‚¬ì#O |†Ç°°^Íi# ªÖ#O #°O_ U k‰×…Õ LOk?


T

3) ^ÎH÷Æ}O 4) PöQ߆ǰO
AM

1) |†Ç°°^Íi# ªÖ#O =^Îí<Í


H

5. XH› =¼H÷ë „¬_È=°~¡ k‰×Qê 4 H÷.g°. #_z, P `Ç~ŒÞ`Ç 2) „¬_È=°~¡ 3) `Ƕ~¡°æ 4) ^ÎH÷Æ}O
M

L`Çë~¡ k‰×Qê 6 H÷.g°. #_z uiy `Ƕ~¡°æ "³á„¬ô䛽 4


AM

12. Q® _  † Ç ¶ ~¡ O †³ ò H› ø ‹¬ = °†Ç ° =ò 7-15 Q® O @°Qê


IG

H÷.g°. #_z# J`Ç#° |†Ç°°^Íi# KË@ #°O_ ZO`Ç L#߄¬C_È° xq°‘ =ò°Á `Ƕ~¡°æ k‰×#° ‹¬¶zOKÇQê,
KH

^ζ~¡O…Õ L<Œß_È°? Q®O@ =ò°Á U k‰×…Õ LO@°Ok?


R

1) 6 H÷.g°. 2) 7 H÷.g°. 1) "Œ†Çò=¼O 2) ^ÎH÷Æ}O


3) 8 H÷.g°. 4) 3 H÷.g°. 3) <³á~¡°u 4) D‰§#¼O
6. ~¡q 4 H÷.g°. <Í~¡°Qê #_z, 䛽_"³á„¬ô䛽 uiy 3 H÷.g°.
13. 'A' J#°#k 'C' 䛽 PöQ߆ǰk‰×…Õ, 'B' J#°#k 'C' 䛽
#_z =°~¡…ì 3 H÷.g°. "³#°H›ä›½ #_K³#°. P `Ç~ŒÞ`Ç
`Ƕ~¡°æ# =°i†Çò 'A' 䛽 D‰§#¼k‰×Qê LOk. J~ò<Œ
J`Ç#° Z_È=° "³á„¬ô䛽 uiy 4 H÷.g°. #_z<Œ, ~¡q
'D' J#°#k 'A' ä› ½ L`Ç ë ~ ¡ O Qê =°i†Ç ò 'B' ä› ½
ZH›ø_È L<Œß_È°?
"Œ†Çò=¼OQê LOk. J~ò<Œ 'D' J#°#k 'C' 䛽 U
1) |†Ç°°^Íi# ªÖ#O #°O_ 6 H÷.g°. ^ζ~¡O…Õ L<Œß_È°.
k‰×…Õ LOk?
2) |†Ç°°^Íi# ªÖ#O #°O_ 7 H÷.g°. ^ζ~¡O…Õ L<Œß_È°.
1) PöQ߆ǰO 2) <³á~¡°u
3) |†Ç°°^Íi# KË@<Í L<Œß_È°.
3) "Œ†Çò=¼O 4) D‰§#¼O
4) U q+¬†Ç°O K³„¬æ…èO.
REASONING ABILITY (VERBAL) ○ ○ ○ ○ ○ ○ ○ ○ ○
101
○ ○ ○ ○ ○ ○ ○ ○ ○ ○ ○ ○ ○ ○ ○ ○ ○ ○ ○
Study Material
14. XH› =¼H÷ë |†Ç°°^Íi# KË@° #°O_ 20 g°. `Ƕ~¡°æ# ^ÎH}
Æ÷ k‰×Qê "³o¤<Œ, |†Ç°°^Íi# ªÖ#O #°Oz U k‰×…Õ
#_z, 䛽_"³á„¬ô䛽 uiy =°~¡…ì 10g°. #_z# `Ç~ŒÞ`Ç, H›^ΰ? =°i†Çò "³ò`ÇOë ZO`Ç ^ζ~¡O ㄬ†¶Ç }÷OzOk?
=°m¤ ä› ½ _ " ³ á „ ¬ ô ä› ½ uiy 9 g°. #_ z , =°m¤ `Ç # 1) 12 H÷.g°. "Œ†Çò=¼O 2) 8 H÷.g°. PöQ߆ǰO
Z_È=°"³á„¬ô䛽 uiy 5 g°. #_KŒH›, =°~¡…ì Z_È=°"³á„¬ô䛽 3) 14 H÷.g°. <³á~¡°u 4) 4 H÷.g°. D‰§#¼O
uiy 12 g°. #_z, z~¡=Qê - =°m¤ Z_È=°"³á„¬ô䛽 18. XH› =¼H÷ë `Ƕ~¡°æ"³á„¬ô 3 H÷.g°. #_z L`Çë~¡O "³á„¬ô uiy
uiy 6 g°. #_K³#°. "³ò`ÇëO g°^Î J`Ç#° |†Ç°°^Íö~ 4 H÷.g°. #_z, 䛽_"³á„¬ô 3 H÷.g°. #_z, =°~¡…ì
ªÖ#O #°O_ U k‰×…Õ L<Œß_È°? Z_È=°"³á„¬ô uiy 4 H÷.g°. #_z<Œ, |†Ç°°^Íi# KË@
1) L`Çë~¡O 2) ^ÎH÷Æ}O 3) PöQ߆ǰO 4) <³á~¡°u #°O_ ZO`Ç ^ζ~¡O…Õ L<Œß_È°? U k‰×…Õ L<Œß_È°?
15. "Œ‹¬° 12 g°. „¬_È=°~¡ä›½, JH›ø_ #°O_ 9 g°. L`Çë~¡ 1) 10 H÷.g°. D‰§#¼O 2) 9 H÷.g°. L`Çë~¡O
k‰×Qê "³o¤<Œ, J`Ç#° k‰×, ^ζ~¡O#° |†Ç°°^Íi# 3) 7 H÷.g°. PöQ߆ǰO 4) 8 H÷.g°. "Œ†Çò=¼O
ªÖ#O #°O_ H›#°Qù#°=ò? 19. XH› =¼H÷ë ^ÎH÷Æ}O KǶ‹¬¶ë x°óO>è, J`Çx Z_È=° KÍ~ò

Y
1) 15 g°. "Œ†Çò=¼O 2) 16 g°. <³á~¡°u U k=#° ‹¬¶z‹¬°ëOk?
3) 18 g°. PöQ߆ǰO 4) 19 g°. D‰§#¼O 1) „¬_È=°~¡ 2) L`Çë~¡O

M
16. XH› =¼H÷ë 5 H÷.g°. `Ƕ~¡°æ#䛽 ㄬ†Ç¶}÷Oz 䛽_"³á„¬ô䛽 3) `Ƕ~¡°æ 4) ^ÎH÷Æ}O

E
uiy 3 H÷.g°. ㄬ†Ç¶}÷Oz# J`Ç#° |†Ç°°^Íi# KË@ 20. XH› KË@ <Œ°Q®°~Ë_ÈÁ 䛀_Èe LOk. h=ô Q®°_H÷ "³ˆÃ× ¤@䛽
#°O_ U k‰×…Õ L<Œß_È°? ^ÎH÷Æ}O #°O_ =zó<Œ, h䛽 䛽_"³á„¬ô䛽 Q® ^¥i

D
9
59
1) <³á~¡°u 2) PöQ߆ǰO ‚¬ìÙ@…˜ä›½ "³ˆ×Ã`Ç°Ok. h=ô <Í~¡°Qê "³o`Í ‹¬¶ø°ä›½
CA
95
3) D‰§#¼O 4) "Œ†Çò=¼O KÍ~¡`Œ~¡°. J~ò# Q®°_ =ô#ß k‰×

50
17. ~Œ^Î 6 H÷.g°. L`Çë~¡OQê "³o¤ JH›ø_ #°O_ Z_È=°"³á„¬ô 1) „¬tó=°O 2) `Ƕ~¡°æ
98
A
uiy =°~Ë 4 H÷.g°. "³o¤#k. JH›ø_ #°Oz 2 H÷.g°. 3) ^ÎH÷Æ}O 4) L`Çë~¡O
:8
1234567890123456789012
12345678901234567890123456789012123456789012345678901234567890121234567890123456789012345678901212345678901234567890123
12345678901234567890123
E

q=~¡}ì`ÇàH› ["Œ|°°
12345678901234567890123456789012123456789012345678901234567890121234567890123456789012345678901212345678901234567890123
1234567890123456789012
1234567890123456789012 12345678901234567890123
12345678901234567890123
12345678901234567890123456789012123456789012345678901234567890121234567890123456789012345678901212345678901234567890123
h

1234567890123456789012 12345678901234567890123
12345678901234567890123456789012123456789012345678901234567890121234567890123456789012345678901212345678901234567890123
1234567890123456789012 PRACTICE TEST :
12345678901234567890123456789012123456789012345678901234567890121234567890123456789012345678901212345678901234567890123
.P

1234567890123456789012 12345678901234567890123
12345678901234567890123
12345678901234567890123456789012123456789012345678901234567890121234567890123456789012345678901212345678901234567890123
IC

1234567890123456789012 12345678901234567890123
12345678901234567890123456789012123456789012345678901234567890121234567890123456789012345678901212345678901234567890123
AD

1. (1) «‘R¯  3. (2)


AB
O

6
3!h±/ 4!h±/
ER
CH

41!h±/
% {>j«‘×$P
&
YD

36!h±/ 51!h±/ 2!h±/ 2!h±/


O$±
Ma铢P
,H

:1!h±/ $
MaÈì¢P 3!h±/ # 4!h±/
T
AM

…­`É>±¢ aÈ·¢±ç
H

…­`É>±¢ aÈ·¢±ç 71!h±/


M

" \‡Î±Pä_Ï!aúf«‘×$P _ÏIøÇ~P


AM

_ÏIøÇ~P
IG

B
„¬@O #°O_ ‹¬æ+¬“OQê, [†Í°Oã^Î䛽 |†Ç°°^Íi# ªÖ#O
KH

„¬@O #°O_ ‹¬æ+¬“OQê N#°ä›½ „¬_È=°~¡ k‰×…Õ ~¡q #°O_ Q® ^ζ~¡O,
R

=°i†Çò ª~ò =ô<Œß~¡°. "&  "' '&  "# #'


'&
2. (2) „¬@O #°O_ ‹¬æ+¬“OQê, P =¼H÷ë |†Ç°°^Íi# ªÖ<ŒxH÷       
„¬_È=°~¡ k‰×…Õ =ô<Œß_Èx `³°‹¬°ëOk.   =5 g°.
2!Iø/h±/ "
}‡È±‚±_Îj$!«‘×$P 4. (2) )#‡Èó>½P* MaÈì¢P
›!Iø/h±/ >½Iøì
3
aÈ·¢±ç#´…
â õ­ !䅭‡È·~P 1! >½Iøì
…­`É>±¢ aÈ·¢±ç /Iøh
±/
J#Qê P =¼H÷ë |†Ç°°^Íi# ªÖ#O `Ƕ~¡°æ"³á„¬ô
=ô#ß^Îx ‹¬æ+¬“OQê `³°‹¬°ëOk. # $
_ÏIøÇ~P 29!Iø/h±/

REASONING ABILITY (VERBAL) ○ ○ ○ ○ ○ ○ ○ ○ ○


102
○ ○ ○ ○ ○ ○ ○ ○ ○ ○ ○ ○ ○ ○ ○ ○ ○ ○ ○
Study Material
„¬@O #°O_ ‹¬æ+¬“OQê P =¼H÷ë "Œ†Çò=¼ k‰×…Õ „¬_È=°~¡Qê, „¬_È=°~¡-"Œ†Çò=¼OQê, "Œ†Çò=¼O-
ㄬ†Ç¶}÷‹¬°ë<Œß_Èx J~¡ÖO J=ôKÇ°#ßk. L`Çë~¡OQê, L`Çë~¡O-D‰§#¼OQê, D‰§#¼O-`Ƕ~¡°æQê
5. (1) „¬@O #°O_ ‹¬æ+¬“OQê 5!Iø/h±/ {>j!«‘×$P ªÖ#ハO’ ‰×O K³O^ΰ`Œ~ò.
P =¼H÷ë |†Ç°°^Íi# ±/ ±/ Hê=ô# „¬_È=°~¡ "Œ†Çò=¼OQê =¶~¡°`Ç°Ok.
/
ø h /
ø h
ªÖ#O #°O_ 6 H÷.g°. !I I MaÈì P¢
7 7!
^ζ~¡O…Õ =ô<Œß_È°. #‡Èó>½P EŠ¨$½P
aúf«‘×$P
5!Iø/h±/
5!Iø/h±/
6. (3) ¢r /±
h …­`É>±¢ aÈ·¢±ç
I!ø/
4

Y
4 H÷.g°. =òO^ΰ䛽, 䛽_"³á„¬ô䛽 3 H÷.g°. #_z uiy =ấ¢±v Q÷Rà‡È±P
_ÏIø~
Ç P

M
=¼uö ~ H› k‰× … Õ 3 H÷ . g°. "³ # °H› ä › ½ =°i†Ç ò Šp?o…_^LÄ{Øa
Z_È=°"³á„¬ô䛽 4 H÷.g°. #_z#, `ùe ªÖ<ŒxöH KÍ~¡°#°.

E
Hê=ô# ~¡q |†Ç°°^Íi# KË@<Í =ô<Œß_È°. #‡Èó>½P

D
9
…­`É>±¢ MaÈì P¢

59
7. (1) Ma铢P
CA
95
…­`É>±¢ aÈ·¢±ç

50
_
ÏI aÈ·¢±ç =ấ¢±v EŠ¨$½P
_ÏIøÇ~P øÇ~ ­õ 圾`Ž!#ấ…­õ
98
A
P! ấ#…
ấ…# ±!
:8
õ­ `[É> aÈ·¢±ç
E

_ÏIøÇ~P
h
.P

Q÷Rà‡È±P
IC

[`É>±!#´â…­õ
AD

„¬@O #°O_ ‹¬æ+¬O“ Qê <Í#° `Ƕ~¡°æ "³„á ô¬ 䛽 ㄬ†¶Ç }÷‹°¬ <ë Œß_È°. A§–_ÆLi·1Ö2¿f§02Ê–JdŒÔ@¨LÄ{ØaŠp¶la
AB
O

8. (3) 2›!Iø/h±/ 11. (4) „¬@O #°O_ ‹¬æ+¬“OQê, P "Œ‚¬ì#O |†Ç°°^Íi#


Ma铢P
ER

ªÖ#O #°O_ ^ÎH÷Æ} k‰×…Õ =ô#ßk.


CH

±/ ±/
YD

…­`É>±¢ aÈ·¢±ç /Iøh /øIh )aúf!«‘×$P* aÈ·¢±ç


! !
› ›
,H

_ÏIøÇ~P 4 4
:!Iø/h±/ 7
Iø/! IÇ_
Ï~
ø
T

2›!Iø/h±/
AM

h !l
/± ŠØ
H

aúf«‘×$P {>j!«‘×$P :!Iø/h±/


M

…­`É>±¢!lŠØ
AM
IG

„¬@O #°O_ ‹¬æ+¬“OQê, H÷‘é~Ÿ |†Ç°°^Íi# ªÖ#O


Œ­>±‡È±P
KH

#°O_ 1½ H÷.g°. ^ζ~¡O…Õ =ô<Œß_È°. 12. (3) „¬@O #°O_ ‹¬æ+¬“OQê, 
R

aÈ·¢±ç
21!Iø/h±/ 5!Iø/h±/
" )Iœ%,­ã!aúf«‘×$P* Q®O@ =ò°Á <³á~¡°u yr±’‘‚!
9. (3) R¯PA‚ >ó‚±Â
)Iœ%,­ã!{>j«‘×$P*
k‰×#° ‹¬¶z‹¬°ëOk. =ấ¢±v >ó‚±Â
21!Iø/h±/ 21!Iø/h±/
)_ÏIøÇ~P*
5!Iø/h±/ 13. (4) Wzó# ㄬ‰×߅Õx ‹¬=¶KŒ~¡O#°
%
„¬@O #°O_ ‹¬æ+¬“OQê, H›$+¬â |†Ç°°^Íi# ªÖ#O „¬@~¡¶„¬O…Õ KǶ„¬Qê ㄬH›ø#
#°O_ 4 { 10 « 14 H÷.g°. ^ζ~¡O…Õ =ô<Œß_È°. KǶ„¬|_# „¬@O U~¡æ_#k.
$ #
10. (2) `Ƕ~¡°æ-PöQ߆ǰOQê ªÖ#ハO’ ‰×O K³Ok#„¬ô_È° =~¡°‹¬Qê „¬@O #°O_ ‹¬æ+¬“OQê, D J#°#k
PöQ߆ǰO-^ÎH÷Æ}OQê, ^ÎH÷Æ}O-<³á~¡°uQê, <³á~¡°u- C 䛽 D‰§#¼k‰×…Õ =ô#ßk.
"

REASONING ABILITY (VERBAL) ○ ○ ○ ○ ○ ○ ○ ○ ○


103
○ ○ ○ ○ ○ ○ ○ ○ ○ ○ ○ ○ ○ ○ ○ ○ ○ ○ ○
Study Material
14. (3) 31!h±/ 18. (1) "$            H÷.g°.
$&      H÷.g°.
)aÈ·¢±ç*
21!h±/
&
:!h±/ /±
h
I!ø/
)Q÷Rà‡È±P*
6!h±/ 7!h±/ )_ÏIøÇ~P*
Ma铢P %
5
23!h±/ EŠ¨$½P ±/
$ 4!Iø/h±/
15. (1) 0#    øI/h
)#‡Èó>½P*
Ma铢P
aÈ·¢±ç !
# 5
   " #
:!h±/ 4!Iø/h±/
  " 23!h±/ aÈ·¢±ç
« 15 g°. …­`>
É ±¢ 0 „¬@O #°O_ ‹¬æ+¬“OQê, J`Ç#° |†Ç°°^Íi# ªÖ#O
∴ "Œ‹¬° |†Ç°°^Íi# #°O_ AC + CE = 5 + 5 = 10 H÷.g°. ^ζ~¡O…Õ =ô<Œß_È°,

Y
ªÖ#O #°O_ 15 g°. _ÏIøÇ~P =°i†Çò J`Ç#° ㄬ‹°¬ `ë OÇ D‰§#¼ k‰×…Õ =ô<Œß_È°.

M
^ζ~¡O…Õ "Œ†Çò=¼k‰×…Õ =ô<Œß_È°. 19. (3)
16. (2) „¬@O #°O_ ‹¬æ+¬“OQê,

E
6!Iø/h±/ aÈ·¢±ç
P =¼H÷ë |†Ç°°^Íi# KË@° aÈ·¢±ç

D
9
>½Iø!ì [`É>±#´…â õ­

59
4!Iø/h±/ _ÏI~
Çø P#´…â õ­ !>óZP
#°O_ PöQ߆ǰk‰×…Õ =ô<Œß_È°.
CA
95
)Q÷Rà‡È±P* _ÏIøÇ~P
17. (1) /±

50
5!Iø/h±/
/høI MaÈ
ì P
¢ „¬@O #°O_ ‹¬æ+¬“OQê, ^ÎH÷Æ}O "³á„¬ô KǶ‹¬°ë#ß =¼H÷ë
98
A
! )#‡Èó>½P*
3 )_Ï!䅭Œ­±ìaÈ!
†³òH›ø Z_È=° KÍ~ò `Ƕ~¡°æ k‰×#° ‹¬¶z‹¬°ëOk.
:8
E

!!!!!!!«‘×$P* 7!Iø/h±/
20. (1) „¬@O #°O_ ‹¬æ+¬“OQê,
h

)MaÈì¢P*!ˆ‘•¢Š¨‚
.P

Q®°_ „¬_È=°~¡ („¬tó=°O)


IC
AD

)_Ï!}‡È±‚±_Îj$!«‘×$P*
_ÏIøÇ~P
k‰×Qê =ô#ß^Îx R¯±`Ž! ƒ­íÚA†™!

„¬@O #°O_ ‹¬æ+¬“OQê, ~Œ^Î "³ò`ÇëO (6 { 4 { 2) `³e†ÇòKÇ°#ßk.


AB
O

)…­`É>±¢* )aÈ·¢±ç*

« 12 H÷.g°. ^ζ~¡O #_z#k =°i†Çò P"³° ㄬ‹¬°ë`ÇO


ER
CH

"Œ†Çò=¼k‰×…Õ H›^ΰ.
i>õ!)_ÏIøÇ~P*
YD

123456789012345678901234567890121234567890123456789012345678901212
1234567890123456789012345678901212345678901234567890123456789012123456789012345678901234567890121234567890123
1234567890123456789012345678901212345678901234567890123456789012123456789012345678901234567890121234
123456789012345678901234567890121234567890123456789012345678901212
12345678901234567890123456789012123456789012345678901234567890121234567890123456789012345678901212345678901234567890123456789012123
123456789012345678901234567890121234567890123456789012345678901212345678901234567890123
1234567890123456789012345678901212345678901234567890123456789012123456789012345678901234567890121234567890123
1234567890123456789012345678901212345678901234567890123456789012123456789012345678901234567890121234
12345678901234567890123456789012123456789012345678901234567890121234567890123456789012345678901212345678901234567890123
,H

123456789012345678901234567890121234567890123456789012345678901212
12345678901234567890123456789012123456789012345678901234567890121234567890123456789012345678901212345678901234567890123456789012123
123456789012345678901234567890121234567890123456789012345678901212345678901234567890123
1234567890123456789012345678901212345678901234567890123456789012123456789012345678901234567890121234567890123
1234567890123456789012345678901212345678901234567890123456789012123456789012345678901234567890121234
12345678901234567890123456789012123456789012345678901234567890121234567890123456789012345678901212345678901234567890123
12345678901234567890123456789012123456789012345678901234567890121234567890123456789012345678901212345678901234567890123456789012123
123456789012345678901234567890121234567890123456789012345678901212345678901234567890123
PREVIOUS BITS
1234567890123456789012345678901212345678901234567890123456789012123456789012345678901234567890121234567890123
1234567890123456789012345678901212345678901234567890123456789012123456789012345678901234567890121234
12345678901234567890123456789012123456789012345678901234567890121234567890123456789012345678901212345678901234567890123
123456789012345678901234567890121234567890123456789012345678901212
12345678901234567890123456789012123456789012345678901234567890121234567890123456789012345678901212345678901234567890123456789012123
123456789012345678901234567890121234567890123456789012345678901212345678901234567890123
12345678901234567890123456789012123456789012345678901234567890121234567890123456789012345678901212345678901234567890123
1234567890123456789012345678901212345678901234567890123456789012123456789012345678901234567890121234567890123
1234567890123456789012345678901212345678901234567890123456789012123456789012345678901234567890121234
12345678901234567890123456789012123456789012345678901234567890121234567890123456789012345678901212345678901234567890123456789012123
123456789012345678901234567890121234567890123456789012345678901212345678901234567890123
123456789012345678901234567890121234567890123456789012345678901212
T

1234567890123456789012345678901212345678901234567890123456789012123456789012345678901234567890121234567890123
1234567890123456789012345678901212345678901234567890123456789012123456789012345678901234567890121234
12345678901234567890123456789012123456789012345678901234567890121234567890123456789012345678901212345678901234567890123
12345678901234567890123456789012123456789012345678901234567890121234567890123456789012345678901212345678901234567890123456789012123
123456789012345678901234567890121234567890123456789012345678901212345678901234567890123
AM

123456789012345678901234567890121234567890123456789012345678901212
1234567890123456789012345678901212345678901234567890123456789012123456789012345678901234567890121234567890123
1234567890123456789012345678901212345678901234567890123456789012123456789012345678901234567890121234
12345678901234567890123456789012123456789012345678901234567890121234567890123456789012345678901212345678901234567890123
12345678901234567890123456789012123456789012345678901234567890121234567890123456789012345678901212345678901234567890123456789012123
123456789012345678901234567890121234567890123456789012345678901212345678901234567890123
H

1. ‹¬O^Î¼ †³òH›ø WO\÷ =òY=ò L`Çë~¡=ò"³á„¬ô#䛽 H›^ΰ. 1) „¬_È=°~¡ 4 H÷.g°. 2) PöQ߆ǰO 4 H÷.g°.
M
AM

P"³° WO\÷ #°O_ |†Ç°@䛽 =zó u#ßQê 10 g°. u#ßQê 3) `Ƕ~¡°æ 4 H÷.g°. 4) "Œ†Çò=¼O 4 H÷.g°.
IG

#_z, Z_È=°"³á„¬ô#䛽 uiy 25 g°. "³o¤Ok. P"³° 3. 'M' 䛽 ^ÎH÷Æ}OQê#¶, Q 䛽 `Ƕ~¡°æ"³á„¬ô# 'N' L#ßk.
KH

JH›ø_È ä›½_"³á„¬ô䛽 uiy 5 g°. #_z# `Ç~ŒÞ`Ç =°~¡ J~ò`Í J^Î$‘“ ¼ 'M' †³òH›ø k‰×
R

䛽_"³á„¬ô䛽 uiy 25 g°. #_z#k. P"³° |†Ç°°^Íi# (A.P.Constable Mains -2016)

‹¬ÖO #°O_ Q®=°¼ªÖ<ŒxH÷ Q® ^ζ~¡O (g°.…Õ) 1) PöQ߆ǰ=ò 2) D‰§#¼=ò


(A.P.Constable Prelims -2016) 3) "Œ†Çò=¼=ò 4) <³á~¡°u
1) 5 2) 10 3) 15 4) 20 4. ~ŒA `Ç# WO\÷ #°O_ |†Ç°°^Íi 30 H÷.g°. ^ÎH÷Æ}O"³á„¬ô
2. XH› ƒì°_È° J`Çx ‹¬¶ø° #°O_ |†Ç°°^Íi ^ÎH÷Æ}O #_z<Œ_È°. Z_È=°"³á„¬ô䛽 uiy 7 H÷.g°. #_KŒ_È°. =°~¡
"³á„¬ô#䛽 5 H÷.g°. ㄬ†Ç¶}÷OKŒ_È°. `Ç~¡°"Œ`Ç „¬~¡O„¬~¡Qê JH›ø_È #°O_ 6 H÷.g°. L`Çë~¡ k䛽øQê #_z P„¦Ô‹¹
ï~O_È°ª~¡°Á 䛽_"³á„¬ô#䛽 uiy, =~¡°‹¬Qê 6 H÷.g°., 5 KÍ~¡°ä›½<Œß_È°. P„¦Ô‹¹ #°O_ J`Çx WO\÷H÷ Q® k‰×,
H÷.g°. #_KŒ_È°. z=~¡Qê JH›ø_È #°O_ 2 H÷.g°. `Ƕ~¡°æ ^ζ~¡=ò (A.P.Constable Mains -2016)

"³á„¬ô#䛽 #_KŒ_È°. J„¬C_È° ‹¬¶ø° #°O_ Q®=°¼ªÖ<ŒxH÷ 1) PöQ߆ǰO 25 H÷.g°. 2) "Œ†Çò=¼O 26 H÷.g°.
Q® k‰×, ^ζ~¡=ò (A.P.Constable Mains -2016) 3) <³á~¡°u 25 H÷.g°. 4) "Œ†Çò=¼O 25 H÷.g°.

REASONING ABILITY (VERBAL) ○ ○ ○ ○ ○ ○ ○ ○ ○


104
○ ○ ○ ○ ○ ○ ○ ○ ○ ○ ○ ○ ○ ○ ○ ○ ○ ○ ○
Study Material
5. XH› J=¶à~ò XH› k䛽ø…Õ 6 H÷.g°. #_z# `Ç~ŒÞ`Ç iv) `Ç^ΰ„¬i L`Çë~¡k‰×…Õ 30 g°. =°i†Çò
Z_È=°"³á„¬ô qiy =°~ùH› 8 H÷.g°. ㄬ†Ç¶}÷Oz#KË, v) J#O`Ç~¡O `Ƕ~¡°æk‰×…Õ 60 g°. #_È°ªë_È°. J`Ç#°
ㄬ ‹ ¬ ° ë ` Ç = ò P"³ ° L#ß ªÖ < ŒxH÷ =°i†Ç ò "³ ò ^Î @ #_ÈH› ㇐~¡OaOz# aO^ΰ=ô, #_ÈH› „¬îiëK͋²#
|†Ç°°^Íi# ªÖ<ŒxH÷ L#ß ^ζ~¡O ZO`Ç? aO^ΰ=ô =°^Î¼ ^ζ~¡=ò (g°.…Õ)
(T.S.Constable Mains -2016) (TS.S.I.Prelims -2016)

1) 12 H÷.g°. 2) 10 H÷.g°. 1) 30 2) 50 3) 60 4) 120


3) 9 H÷.g°. 4) 14 H÷.g°. 12. XH› =¼H÷ë `Ƕ~¡°æk‰×…Õ 250 g°. #_È°ªë_È°. J„¬C_È°
6. Q®_†  ¶Ç ~¡O 10.00 ‹¬=°†Ç°=ò ‹¬¶zOz#„¬ô_È° Q®O@ L`Çë~¡ k‰×…Õ 50 g°. #_È°ªë_È°. J„¬C_È° „¬_È=°~¡ k‰×…Õ
=ò°Á L`Ç~ë k¡ ‰× "³„á ô¬ ‹¬¶zÀ‹ë, U ‹¬=°†Ç¶xH÷ Q®O@ 300 g°. #_È°ªë_°È . J`Ç#° #_ÈH› ㇐~¡OaOz# aO^ΰ=ô,
=ò°Á <³|á °°u k‰×#° ‹¬¶z‹¬°Oë k. (T.S.Const. Mains -'16) #_ÈH› „¬îiëK͋²# aO^ΰ=ô =°^Î¼ ^ζ~¡=ò (g°.…Õ)
1) 5 : 15 2) 5 : 30 3) 4 : 00 4) 4 : 30 (TS.S.I.Prelims -2016)

Y
7. XH› =°x+² ㇐~¡Oƒ’ ªÖ#O #°O_ L`Çë~¡O䛽 3 H÷.g°. 1) 30 2)   3)   4)  
JH›ø_È #°O_ „¬_È=°~¡ "³á„¬ô䛽 2 H÷.g°. #_KŒ_È°. JH›ø_È 13. `Ƕ~¡°æ䛽 Ja=òYOQê L#ß =¼H÷ë `Ƕ~¡°æ k䛽øQê 30

M
#°O_ L`Ç~ë O¡ "³„á ô¬ 䛽 1 H÷.g°. JH›ø_È #°O_ `Ƕ~¡°æ"³„á ô¬ #䛽 g°@~¡°Á #_KŒH› ^ÎH÷Æ}O "³á„¬ô䛽 40 g°.° #_z P̄á

E
5 H÷.g°. #_À‹ë J`Ç#° ㇐~¡Oƒ’ªÖ#O #°O_ ZO`Ç „¬_È=°~¡ "³á„¬ô 60 g°.° #_KŒ_È°. "³ò^Î\ ÷ aO^ΰ=ô
^ζ~¡O…Õ LO\ì_È°. #°O_ J`Ç_³O`Ç ^ζ~¡O…Õ L<Œß_È° (g°.…Õ)

D
9
(A.P.- S.I. Prelims -2016)

59
1) 11 H÷.g°. 2) 5 H÷.g°. 3) 10 H÷.g°. 4) 8 H÷.g°.
CA (TS.S.I.Prelims -2016)

95
8. B J<Í aO^ΰ=ô 'A' aO^ΰ=ô䛽 3 g°@~¡Á ^ζ~¡O…Õ 1) 50 2) 60 3) 70 4) 130

50
L`Çë~¡ k‰×…Õ LOk, C aO^ΰ=ô B 䛽 3 g°. ^ζ~¡O…Õ 14. XH›`Ç#° „¬_È=°~¡ k䛽ø䛽 Ja=òYOQê x|_ L<Œß_È°.
98
A
„¬_È=°~¡ k‰×…Õ, D aO^ΰ=ô C 䛽 5 g°. ^ζ~¡O…Õ J`Ç#° ㄬ^ÎH÷Æ} k‰×…Õ 45° uiy, `Ç~ŒÞ`Ç J^Í k‰×…Õ
:8
^ÎH÷Æ} k‰×…Õ, E aO^ΰ=ô D 䛽 7 g°. ^ζ~¡O…Õ 180° uiy, P̄á Jㄬ^ÎH÷Æ} k‰×…Õ 270° uiQê_È°.
E
h

`Ƕ~¡°æk‰×…Õ L<Œß~ò. B aO^ΰ=ô ^Î$‘“ ¼, E aO^ΰ=ô W„¬C_È° J`ÇxH÷ Ja=òYOQê L#ß k‰×
.P
IC

U k‰×…Õ LOk. (A.P.- S.I. Prelims -2016) (TS. S.I.Mains -2016)


AD

1) `Ƕ~¡°æ 2) ^ÎH÷Æ}=ò 1) ^ÎH÷Æ}O 2) "Œ†Çò=¼O 3) „¬_È=°~¡ 4) <³á~¡°u


AB
O

3) PöQ߆ǰ=ò 4) „¬_È=°~¡ 15. XH› =¼H÷ë ãH÷O^Îx=Þ|_# k‰×…Õ #_È°‹¬°ë<Œß_È°.


ER

9. AB XH› ‹¬~¡ˆ×ö~Y. XH› „¬ô~¡°Q®° A #°O_ |†Ç°°^Íi i) 50 g°. `Ƕ~¡°æ"³á„¬ô


CH

T~¡ÞÖ =òYOQê ̄áH÷ 4 ̋O.g°° H›ke, =°m¤ 䛽_"„á³ ô¬ ii) `Ç~¡°"Œ`Ç 50 g°. ^ÎH÷Æ}O "³á„¬ô
YD

H÷Æu[ ‹¬=¶O`Ç~¡OQê 3 ̋O.g°° H›ke =°m¤ ̄áH÷ iii) P̄á 120 g°. „¬_È=°~¡ "³á„¬ô
,H

T~¡Þœ =òYOQê 2 ̋O.g°° H›ke, uiy 䛽_"„á³ ô¬ H÷uÆ [ iv) `Ç^ΰ„¬i 25 g°. L`Çë~¡O "³á„¬ô
T
AM

‹¬=¶O`Ç~¡OQê 2 ̋O.g°° H›ke z=~¡Qê ãH÷OkH÷ v) z=~¡Qê 70 g°. `Ƕ~¡°æ"³á„¬ô


H
M

J^Ë=òYOQê #_z B x KÍiOk. J„¬C_È° U~¡æ_ÈÛ „¬@„¬ô J„¬ô_È° "³ò^Î\ ÷ aO^ΰ=ô #°O_ z=i aO^ΰ=ô䛽 Q®
AM
IG

"³‰á §¼=ò (KÇ^~Î „¡ ô¬ ̋O.g°…Õ) (TS.S.I.Prelims -2016) ^ζ~¡O (g°@~¡Á…Õ) (TS. S.I.Mains -2016)
KH

1) 24 2) 26 3) 39 4) 144 1) 25 2) 30 3) 40 4) 50
R

10. XH› =¼H÷ë D‰§#¼ k‰×# 40 g°. ^ζ~¡=ò #_z, `Ç# 16. ~˂¬ì<£ L`Çë~¡ k‰×Qê 40 g°@~¡°Á "³o¤, Z_È=°"³á„¬ô uiy
Z_È=°"³á„¬ô 90° uiy 40 g°. ^ζ~¡=ò #_È°ªë_È°. 20 g°@~¡°Á #_KŒ_È°. P `Ç~ŒÞ`Ç =°m¤ Z_È=°"³á„¬ô䛽
J„¬C_È° ^ÎH÷Æ}k‰×#   ^ÎH÷Æ} k‰×# g°. ^ζ~¡=ò uiy 40 g°@~¡°Á #_KŒ_È°. J`Ç#° |†Ç°°^Íi# aO^ΰ=ô
#_È°ªë_È°. #_ÈH› ㇐~¡OaOz# aO^ΰ=ô, #_ÈH› „¬îiëK͋²# #°O_ ^ζ~¡O, k‰×° =~¡°‹¬Qê (TS. S.I.Mains -2016)

aO^ΰ=ô =°^Î¼ ^ζ~¡=ò (g°.…Õ) 1) 20 g°@~¡°Á, `Ƕ~¡°æ 2) 20 g°@~¡°Á, L`Çë~¡O


(TS.S.I.Prelims -2016) 3) 20 g°@~¡°Á, „¬_È=°~¡ 4) 100 g°@~¡°Á, ^ÎH÷Æ}O
1) 0 2) 60 3)   4)   17. Jq°`Ÿ `Ƕ~¡°æ k‰×Qê 30 g°@~¡°Á #_z, 䛽_"³á„¬ô uiy
11. XH› =¼H÷ë ãH÷Ok Wzó# k䛽ø…Õ #_È°ªë_È°. 40 g°@~¡°Á #_KŒ_È°. P `Ç~ŒÞ`Ç Z_È=°"³á„¬ô䛽 uiy 30
i) `Ƕ~¡°æ k‰×…Õ 60 g°. g°@~¡°Á #_KŒ_È°. J`Ç#° |†Ç°°^Íi# aO^ΰ=ô ^Î$‘“ ¼
ii) `Ç~¡°"Œ`Ç ^ÎH÷Æ} k‰×…Õ 80 g°. W„¬C_È° L#ß k‰× (TS. S.I.Mains -2016)

iii) PÌ„á „¬_È=°~¡ k‰×…Õ 120 g°. 1) D‰§#¼O 2) `Ƕ~¡°æ 3) PöQ߆ǰO 4) ^ÎH÷Æ}O
REASONING ABILITY (VERBAL) ○ ○ ○ ○ ○ ○ ○ ○ ○
105
○ ○ ○ ○ ○ ○ ○ ○ ○ ○ ○ ○ ○ ○ ○ ○ ○ ○ ○
Study Material
12345678901234567890123456789012123456789012345678901234567890121234567890123456789012345678901212345678901234567890123
1234567890123456789012 123456789012345678901234

q=~¡}ì`ÇàH› ["Œ|°°
12345678901234567890123456789012123456789012345678901234567890121234567890123456789012345678901212345678901234567890123
1234567890123456789012 123456789012345678901234
12345678901234567890123456789012123456789012345678901234567890121234567890123456789012345678901212345678901234567890123
1234567890123456789012 123456789012345678901234

1234567890123456789012
PREVIOUS BITS :
12345678901234567890123456789012123456789012345678901234567890121234567890123456789012345678901212345678901234567890123
1234567890123456789012 123456789012345678901234
12345678901234567890123456789012123456789012345678901234567890121234567890123456789012345678901212345678901234567890123
1234567890123456789012 123456789012345678901234
12345678901234567890123456789012123456789012345678901234567890121234567890123456789012345678901212345678901234567890123
1234567890123456789012 123456789012345678901234
12345678901234567890123456789012123456789012345678901234567890121234567890123456789012345678901212345678901234567890123
123456789012345678901234

1. (3) ^Î`ŒëO‰×O ㄬHê~¡O XH› ~¡„¦¹ ƒç=°à w†Ç°Qê 5. (2) XH›"Íˆ× P J=¶à~ò `Ƕ~¡°æk䛽ø "³á„¬ô ㄬ†Ç¶}÷Oz#k
" Œ­P_½Ï !X‚±Â M J#°Hùx#
21!h±JJ …­ aÈ· $
36!h±JJ _Ï 7!Iø/h±/
# $
" #
6!h±JJ 6!h±JJ 9!Iø/h±/
36!h±JJ
& % Hê=‹²# ^ζ~¡O, AC = "#


#$

‹¬O^Î¼ |†Ç°°^Íi# ‹¬ÖO #°O_ Q®=°¼ªÖ<ŒxH÷ 
 



Y
=°^Î¼Q® ^ζ~¡O « AE = AB + BE = 10 + 5
= 15 g°II.
       H÷.g°.

M
2. (1) ^Î`ŒëO‰×O ㄬHê~¡O ƒì°x H›^ÎeH›° D ãH÷Ok ƒç=°à
6. (1) Q®_†Ç¶~¡O…Õ Q®O@=ò°Á 10:00 Q®O@„¬ô_È°

E
^¥Þ~Œ K³„¬æ|_¨Û~ò. L`Çë~¡ k‰×"³á„¬ô LO^ÎO>è P Q®_†Ç¶~¡O…Õx ‹¬=°†Ç°O
12:00 Qê 2 Q®O@° `DŽ¬CQê LO_È=KÇ°ó.

D
9
3!Iø/h±JJ …­`É>±¢

59
% & " Œ­·ù‚± J^Íq^ÎOQê 7:15 xIIä›½ Q®O@=ò°Á <³á~¡°u "³á„¬ô
CA
95
6!Iø/h±JJ 6!Iø/h±JJ LO@°Ok.

50
Hêx Hê=‹²# ‹¬i†³Ø°# ‹¬=°†Ç°O
$ #
98
A
7!Iø/h±JJ « 7 : 15 - 2 : 00
:8
k‰× → „¬_È=°~¡ « 5 : 15.
E
h

^ζ~¡O → AE = AD – DE = 6 – 2 = 4 H÷.g°. 7. (2) ^Î`ŒëO‰×O ㄬHê~¡O =°x+² H›^ÎeH›° D ãH÷Ok q^ÎOQê


.P
IC

3. (2) ^Î`ŒëO‰×O ㄬHê~¡O ƒç=°à ~¡¶„¬O…Õ w†Ç°Qê


AD

# M E
. BF = 5 – 2 = 3 km
AB
O

…­ aÈ· % LN &


=´â
_Ï Q EG = EF + FG
ER

LN
CH

= DC + FG $ LN # LN '


YD

2 / = 1 + 3 = 4 km LN N
L
,H


AG = 3 km
Q ^Î$‘“ ¼ 'M' D‰§#¼=ò…Õ H›_È°.
T

" (
AM

4. (4) ^Î`ŒëO‰×O ㄬHê~¡O XH› z`Ç°ë ƒç=°à#° w†Ç°Qê


∴ Hê=‹²# ^ζ~¡O = AE LN
H
M


" # M E = "(
(&

AM
IG

…­ aÈ·
# 8!Iø/h±JJ & =´â   
_Ï Q
=
KH

41!Iø/h±JJ 7!Iø/h±JJ
      km
R

=
$ 8!Iø/h±JJ % 8. (1) ^Î`ŒëO‰×O ㄬHê~¡O J=°iH› #
 
$ &
P„¦Ô‹¹ #°O_ ~ŒA WO\÷ †³òH›ø k‰× → "Œ†Çò=¼O B 䛽
`Ƕ~¡°æ k‰×…Õ H›^ΰ. 
'E' 

^ζ~¡=ò AE = ? (3 { 3 { 1 « 7 H÷.g°.)  "


%
AB = 30 – 6 = 24 km 9. (1) ^Î`ŒëO‰×O P^¥~¡OQê

BE = 7 km % $


AE = "#
#&

&  
  
 


" #
       H÷.g°.  

REASONING ABILITY (VERBAL) ○ ○ ○ ○ ○ ○ ○ ○ ○


106
○ ○ ○ ○ ○ ○ ○ ○ ○ ○ ○ ○ ○ ○ ○ ○ ○ ○ ○
Study Material
"³ò`ÇëO „¬@ "³á‰§¼O « AB × BC = 5 × 6
13. (2) " 
#
= 30 KÇ.̋O.g°.
 
Hê=‹²# "³á‰§¼O « 30 – (3 × 2)
% $
= 30 – 6 = 24 KÇ.̋O.g°.  

(1) $ 51!h
10.
±/ ∴ Hê=‹²# ^ζ~¡O «  
  ¡ #
$`ÉI!œ …­ïjì     g°II.
/
LΌ³$!«‘×$P " 51!h± ‘×$
P
14. (4) Hê=‹²# k‰× « „¬_È=°~¡H÷ (270° – 180° – 450°)
“!«
¢P„
‘

(J„¬‹¬=¼k‰×…Õ)
äˆ

̄á ㄬ‰×ß…Õ 90° uiQê_È° J#Qê JH›ø_È O|HË} = 45° J„¬‹¬=¼k‰×…Õ

Y
ãuƒ’°[O U~¡æ_È°`Ç°Ok. „¬_È=°~¡H÷ 45° J„¬‹¬=¼k‰× J#Qê <³á~¡°u
̄á^Š¥Q®~¡‹¹ ‹²^¥œO`ÇO ㄬHê~¡O 15. (1) ^Î`ŒëO‰×O ㄬHê~¡O

M
H›~¡âO2 « (Z.ƒ’°)2 { (P.ƒ’°)2 " 
#

E

= 402 + 402 ' 
( 

D
9
H›~¡âO «         

59
& $
 % 
J#Qê J`Ç_È° #_ÈH› ㇐~¡OaOz# ªÖ<ŒxH÷ KÍi<Œ_È°.
CA
95


50
11. (2) " 71!h±/
# Hê=‹²# ^ζ~¡O « AG = 50 – 25 = 25 g°.
98
A
61!h±/ 16. (3) ^Î`ŒëO‰×O ㄬHê~¡O XH› z`Ç°ë ƒç=°à w†Ç°Qê
&
:8
' 91!h±/ 91!h±/
71!h±/
$ 31!h±/ #
E
h

% $
.P

231!h±/ 51!h±/ 51!h±/


IC

% "
AD

∴ AF = 80 – 30 = 50 g°II.
Hê=‹²# ^ζ~¡O « AD = BC = 20 g°.

AB
O

(4) &  % 


$ k‰× → 'A' 䛽 „¬_È=°~¡k‰×…Õ
ER

12.
CH

17. (3) ^Î`ŒëO‰×O ㄬHê~¡O XH› z`Ç°ë ƒç=°à w†Ç°Qê


YD

 
41!h±/ # E
#
,H

" 51!h±/
T

=ấ Q
AM

∴ Hê=‹²# ^ζ~¡O « H›~¡âO


H

41!h±/
M

«     
AM

∴ Hê=‹²# k‰× - PöQ߆ǰO.


IG
KH
R

REASONING ABILITY (VERBAL) ○ ○ ○ ○ ○ ○ ○ ○ ○


107
○ ○ ○ ○ ○ ○ ○ ○ ○ ○ ○ ○ ○ ○ ○ ○ ○ ○ ○
Study Material
#O|~Ÿ, ~Œ¼OH÷OQ· Ê >ÿØ"£° ‹ÔïHÞ<£ž >ÿ‹¹“
8 (Number, Ranking & Time Sequence Test)

=òMì¼O‰§°...
G D qƒìQ®O…Õ Jƒ’¼iÖx =ü_È° g°^Πㄬ‰×߁° J_ÈQ®_ÈO [~¡°Q®°`Ç°Ok.
'Concepts'

G D qƒìQ®O…Õ "³ò`ÇëO '3Ñ ~¡Hꁰ H›=ô.


Jq 1) #O|~Ÿ >ÿ‹¹“ (Number Test)

2) ~Œ¼OH÷OQ· >ÿ‹¹“

Y
(Ranking Test)

3) >ÿØ"£° ‹ÔÞïH<£ž >ÿ‹¹“

M
(Time Sequence Test)

E
TYPE - I : #O|~Ÿ >ÿ‹¹“

D
9
59
D qƒìQ®O…Õx ㄬ‰×߁…Õ Wzó# ‹¬OY¼ ãH›=°=ò…Õ XH› xiœ+¬“"³°Ø# x|O^Î##° J#°‹¬iOz, P x|O^Î# ㄬHê~¡O P CA
95
+
‹¬OY¼° Zxߪ~¡°Á =KŒó~ò J#ßk `³°‹¬°HË"Œe.

50
+ ‹¬OY¼ ãH›=°O…Õx JOïH#° `Œ~¡°=¶~¡° K͋² …è^¥ JOïH#° =¶ió x[ ãH›=¶xH÷ =°i†Çò =¶ió# ãH›=¶xH÷ ‹¬O|OkOz# 98
A
ㄬ‰×߁#° J_È°Q®°`Œ~¡°.
:8

D qƒìQ®O…Õx ‹¬=°‹¬¼° ªkOKÇ_¨xH÷ Jƒ’¼iÖH÷ ‹¬i‹¬OY¼°, ƒè‹²‹¬OY¼° ‹¬O†ÇòH›ë ‹¬OY¼°, ㄬ^¥# ‹¬OY¼°....
E

+
h
.P

=O\÷ "Œ\÷Ì„á „¬îië J=Qꂬì# =ôO_¨e.


IC

‹¬=°‹¬¼#° ªkOKÇ°#„¬ô_È° ㄬ‰§ß„¬ã`ÇO…Õx JOïH#° ~¡„¦¹À„„¬~ŸÌ„á ~Œ‹¬°ä›½x ̄xž…˜ L„¬†³¶yOz Q®°~¡°ë° ̄@°“䛽x
AD

+
ªkOKÇ_ÈO "Í°°. ㄬ‰§ß„¬ã`ÇO…Õx JOïHÌ„á ̄xž…˜ Q®°~¡°ë° "Œ_È~Œ^ΰ.
AB
O
ER
CH

=¶ki ㄬ‰×߁°
YD
,H
T
AM

MODEL - 1
H
M

8 6 3 1 5 8 7 6 8 4 3 8 7 8 5 6 2 1

=òO^ΰ =°i†Çò `Ç~¡°"Œ`Ç ã„¬^¥#‹¬OY¼° H›ey#


AM
IG

1. 3 9 2 4 3 9 2 3 9 3 9 2 3 9 2 9 3
'8Ñ =ü_È°ª~¡°Á =zó#k.
KH

̄á ã‰õ}÷…Õ =òO^ΰ 3 LO_ `Ç~¡°"Œ`Ç 2 LO_Í 9 °


3. D ãH÷O^Î W=Þ|_# ‹¬OY¼ ãH›=°O…Õ =òO^ΰ 8
R

Zxß H›=ô?
=ôO_ `Ç~¡°"Œ`Ç 4 …èx 7° Zxß L<Œß~ò ?
ª^Î#: 392 4 392 39 392 392 93 6 5 8 7 4 5 9 8 7 4 3 1 0 8 7 6 5 2
J@°=O\÷ 9 ° <Œ°Q®° H›=ô. 5 1 8 7 6
2. D ãH÷O^Î W=Þ|_# ‹¬OY¼ ãH›=°O…Õ =òO^ΰ =°i†Çò ª^Î#: 6 5 8 7 4 5 9 8 7 4 3

`Ç ~ ¡ ° "Œ`Ç ã„¬ ^  ¥ #‹¬ O Y¼° H› e y# 8 Zxߪ~¡ ° Á 1 0 8 7 6 5 2 5 1 8 7 6

„¬ô#~Œ=$`ÇO J~ò#k ?
7 3 8 6 7 5 9 1 0 8 6 7 3 2 4 9 8 6 4
Wzó# ‹¬OY¼ ãH›=°O#° „¬ijeOz#, =òO^ΰ 8
2 8 6 3 1 5 8 7 6 8 4 3 8 7 8 5 6 2 1 H›ey=ôO_ `Ç~¡°"Œ`Ç 4 …èx 7° ï~O_È°ª~¡°Á =KŒó~ò.
ª^Î#: 7 3 8 6 7 5 9 1 0 8 6 7 3 2 4 9 8 6 4 2

REASONING ABILITY (VERBAL) ○ ○ ○ ○ ○ ○ ○ ○ ○


108
○ ○ ○ ○ ○ ○ ○ ○ ○ ○ ○ ○ ○ ○ ○ ○ ○ ○ ○
Study Material
MODEL - 2
1. D ãH÷O^Î W=Þ|_# ‹¬OY¼ ãH›=°O…Õ Ju Z䛽ø=ª~¡°Á =°i†Çò Ju `Ç䛽ø=ª~¡°Á „¬ô#~Œ=$`Ç=°~ò# JOïH° Uq ?
7 3 9 5 8 6 7 8 4 9 1 0 8 4 3 7 6 5 4 8 7 9 3 0
1 2 8 4 7 9 2 1 0 8 4 3 7 6 5 2 1 8 3 7 9 5 4 2
ª^Î#: 0 1 2 3 4 5 6 7 8 9

↓ ↓ ↓ ↓ ↓ ↓ ↓ ↓ ↓ ↓
3 4 4 5 6 4 3 7 7 5

7 =°i†Çò 8 Z䛽ø=ª~¡°Á (7 ª~¡°Á) =°i†Çò 0 `Ç䛽ø=ª~¡°Á (3 ª~¡°Á) „¬ô#~Œ=$`ÇO J~ò#q.


2. 5 8 3 9 6 7 4 5 6 7 3 6 6 5 7 8 JOïH ãH›=°O…Õx JOïH ªÖ<Œ#° `Œ~¡°=¶~¡° K͆ǰQê Zxß JOïH° J^Í
ªÖ#O…Õ =ôO\ì~ò?

Y
ª^Î#: 5 8 3 9 6 7 4 5 6 7 3 6 6 5 7 8

M
8 7 5 6 6 3 7 6 5 4 7 6 9 3 8 5

Wzó# ãH›=°O#° =°i†Çò JOïH ªÖ<Œ#° `Œ~¡°=¶~¡° K͆ǰQê U~¡æ_# ãH›=°O#° „¬ijeOz# ï~O_È° JOïH°

E
(6 =°i†Çò 6) J^Í ªÖ<Œ…Õ „¬ô#~Œ=$`ÇO J=ô`Œ~ò.

D
9
59
123456789012345678901234567890121234567890
123456789012345678901234567890121234567890123456789012345678901212
12345678901234567890123456789012123456789012345678901234567890121234567890123456789012345678901212345678901234567890123
1234567890123456789012345678901212345678901234567890123456789012123456789012345678901234567890121234567890123
1234567890123456789012345678901212345678901234567890123456789012123456789012345678901234567890121234
123456789012345678901234567890121234567890123456789012345678901212
123456789012345678901234567890121234567890
12345678901234567890123456789012123456789012345678901234567890121234567890123456789012345678901212345678901234567890123456789012123
1234567890123456789012345678901212345678901234567890123456789012123456789012345678901234567890121234

#O|~Ÿ >ÿ‹¹“ : TYPE - I : PRACTICE TEST


123456789012345678901234567890121234567890123456789012345678901212
CA
12345678901234567890123456789012123456789012345678901234567890121234567890123456789012345678901212345678901234567890123
1234567890123456789012345678901212345678901234567890123456789012123456789012345678901234567890121234567890123

95
12345678901234567890123456789012123456789012345678901234567890121234567890123456789012345678901212345678901234567890123456789012123
12345678901234567890123456789012123456789012345678901234567890121234567890123456789012345678901212345678901234567890123
1234567890123456789012345678901212345678901234567890123456789012123456789012345678901234567890121234567890123
1234567890123456789012345678901212345678901234567890123456789012123456789012345678901234567890121234
123456789012345678901234567890121234567890
12345678901234567890123456789012123456789012345678901234567890121234567890123456789012345678901212345678901234567890123456789012123
123456789012345678901234567890121234567890123456789012345678901212
12345678901234567890123456789012123456789012345678901234567890121234567890123456789012345678901212345678901234567890123
1234567890123456789012345678901212345678901234567890123456789012123456789012345678901234567890121234567890123
1234567890123456789012345678901212345678901234567890123456789012123456789012345678901234567890121234
12345678901234567890123456789012123456789012345678901234567890121234567890123456789012345678901212345678901234567890123456789012123
12345678901234567890123456789012123456789012345678901234567890121234567890123456789012345678901212345678901234567890123
1234567890123456789012345678901212345678901234567890123456789012123456789012345678901234567890121234567890123
1234567890123456789012345678901212345678901234567890123456789012123456789012345678901234567890121234
123456789012345678901234567890121234567890
123456789012345678901234567890121234567890123456789012345678901212
12345678901234567890123456789012123456789012345678901234567890121234567890123456789012345678901212345678901234567890123456789012123
12345678901234567890123456789012123456789012345678901234567890121234567890123456789012345678901212345678901234567890123
1234567890123456789012345678901212345678901234567890123456789012123456789012345678901234567890121234567890123
1234567890123456789012345678901212345678901234567890123456789012123456789012345678901234567890121234
12345678901234567890123456789012123456789012345678901234567890121234567890123456789012345678901212345678901234567890123456789012123
123456789012345678901234567890121234567890123456789012345678901212
12345678901234567890123456789012123456789012345678901234567890121234567890123456789012345678901212345678901234567890123456789012123
12345678901234567890123456789012123456789012345678901234567890121234567890123456789012345678901212345678901234567890123
1234567890123456789012345678901212345678901234567890123456789012123456789012345678901234567890121234567890123
1234567890123456789012345678901212345678901234567890123456789012123456789012345678901234567890121234

50
12345678901234567890123456789012123456789012345678901234567890121234567890123456789012345678901212345678901234567890123456789012123
12345678901234567890123456789012123456789012345678901234567890121234567890123456789012345678901212345678901234567890123
1234567890123456789012345678901212345678901234567890123456789012123456789012345678901234567890121234567890123
1234567890123456789012345678901212345678901234567890123456789012123456789012345678901234567890121234

98
A
1. D ãH÷Ok ‹²s‹¹…Õ Zxß 7ä›½ =òO^ΰ 9 `Ç~¡°"Œ`Ç 9. D ãH÷Ok ‹Ôi‹¹…Õ =òO^ΰ ‹¬i‹¬OY¼ LO_, `Ç~¡°"Œ`Ç
:8
'6с° L<Œß~ò. ƒè‹² ‹¬OY¼ LO_Í '8с° Zxß L<Œß~ò.
E

7897653428972459297647
h

58285483787878482382858483
.P
IC

1) 2 2) 3 3) 4 4) 5
AD

1) 2 2) 3 3) 4 4) 5

2. 1 #°O_ 45 =~¡ä›½ Q® <³O|~Ÿ…Õ 3KÍ ƒìyOKÇ|_Í=hß


10. 2386368563683963526836783 J#° ã‰õ}÷…Õ
P~˂¬ì}ì ãH›=°O…Õ J=°ió# ̄á #°O_ 9= ªÖ#O…Õ
AB
O

=òO^ΰ 3 LO_ `Ç~¡°"Œ`Ç 3 …èx '6с° Zxß H›=ô.


LO_Í <³O|~Ÿ Uk?
ER
CH

1) 2 2) 1 3) 3 4) 4
1) 18 2) 21 3) 24 4) 27

11. 576084352908123465795 JOï H  ãH› = °O…Õ


YD

3. 11 #°O_ 50 =°^Î¼Q® <³O|~Ÿ…Õ 7KÍ ƒìyOKÇ|_ 2


KÍ ƒìyOKÇ|_Èx ‹¬OY¼° Zxß H›=ô? Z䛽ø=ª~¡°Á „¬ô#~Œ=$`ÇO J~ò# JOïH =°i†Çò `Ç䛽ø=
,H

ª~¡°Á „¬ô#~Œ=$`ÇO J~ò# JOïH =°^Î¼ ƒè^ÎO


T

1) 2 2) 4 3) 3 4) 6
AM

4. 9 #°O_ 54 =°^Î¼ Q® <³O|~Ÿ…Õ 9KÍ ƒìyOKÇ|_ 3KÍ


H

1) 3 2) 4 3) 5 4) 6

ƒìyOKÇ|_Èx ‹¬OY¼° Zxß?


M

12. 7 0 1 3 5 4 6 9 7 3 8 6 4 2 8 9 6 5 2 JOïH
Uq°…è=ô
AM
IG

1) 8 2) 6 3) 5 4)
ãH›=°O…Õ =òO^ΰ ‹¬i‹¬OY¼ =°i†Çò `Ç~¡°"Œ`Ç ‹¬i‹¬OY¼
5. 1 #°O_ 100 =~¡ä›½ Q® <³O|~Ÿ……Õ 4KÍ ƒìyOKÇ|_
KH

H›ey# ‹¬i‹¬OY¼ ‹¬OY¼


4#° XH› JOïHQê H›e¾# ‹¬O^Î~ŒÄ° Zxß H›=ô.
R

1) 3 2) 1 3) 4 4) 2
1) 7 2) 10 3) 20 4) 21

6. ãH÷Ok =~¡°‹¬…Õ Q® JOïH…Õ '7Ñ `Ç~ŒÞ`Ç =KÍó '3Ñ 13. 3 8 9 7 6 5 4 0 3 5 7 6 3 4 0 6 5 8 4 JOïH ãH›=°O…Õ
Hê䛽O_¨ '8ÑH÷ `Ç~ŒÞ`Ç =KÍó '7Ñ ° Zxß ? Ju `Ç䛽ø=ª~¡°Á „¬ô#~Œ=$`ÇO J~ò# JOïH ‹¬OY¼
898762263269732872778737794 1) 7 2) 8 3) 5 4) 9

1) 10 2) 3 3) 2 4) 0 14. 6 7 8 9 4 3 2 1 8 4 6 8 0 5 9 3 2 1 JOïH
7. 7 3 4 5 6 7 8 9 3 4 7 8 9 10 12 45 J#° ã‰õ}÷…Õ ãH›=°O…Õx JOïH ªÖ<Œ#° `Œ~¡°=¶~¡°K͆ǰQê U~¡æ_#
‹¬i‹¬OY¼ =°^Î¼Q® '7Ñ ° Zxß? JOïH ãH›=°O…Õ Zxß JOïH° J^Í ªÖ#O…Õ =ôO\ì~ò?
1) 3 2) 2 3) 4 4) 5
1) 2 2) 4 3) 1 4) 3

8. D ãH÷Ok ‹²s‹¹…Õ N J<Í JH›Æ~¡O `Ç~ŒÞ`Ç X LO@°Ok. 15. 759643952 JOïH ãH›=°O…Õx JOïH#° P~˂¬ì}
Hêx X `Ç~¡°"Œ`Ç T …è䛽O_¨ LO_Í 'N' ‹¬OY¼ ZO`Ç?
ãH›=°O…Õ J=°iÀ‹ë Zxß JOïH° J^Í ªÖ#O…Õ =ôO\ì~ò?
NXNTQMNXTMXNXCNQMNNXQNXTXNAMXNXM
1) 1 2) 2 3) 3 4) 0
1) 2 2) 4 3) 5 4) 7

REASONING ABILITY (VERBAL) ○ ○ ○ ○ ○ ○ ○ ○ ○


109
○ ○ ○ ○ ○ ○ ○ ○ ○ ○ ○ ○ ○ ○ ○ ○ ○ ○ ○
Study Material
12345678901234567890123456789012
12345678901234567890123456789012123456789012345678901234567890121234567890123456789012345678901212345678901234567890123
12345678901234567890123456789012

q=~¡}ì`ÇàH› ["Œ|°°
12345678901234567890123456789012123456789012345678901234567890121234567890123456789012345678901212345678901234567890123
12345678901234567890123456789012 12345678901234567890123456789012
12345678901234567890123456789012123456789012345678901234567890121234567890123456789012345678901212345678901234567890123
12345678901234567890123456789012 12345678901234567890123456789012
12345678901234567890123456789012
12345678901234567890123456789012123456789012345678901234567890121234567890123456789012345678901212345678901234567890123
12345678901234567890123456789012
12345678901234567890123456789012123456789012345678901234567890121234567890123456789012345678901212345678901234567890123
12345678901234567890123456789012 12345678901234567890123456789012
12345678901234567890123456789012123456789012345678901234567890121234567890123456789012345678901212345678901234567890123
12345678901234567890123456789012 12345678901234567890123456789012
12345678901234567890123456789012123456789012345678901234567890121234567890123456789012345678901212345678901234567890123
12345678901234567890123456789012 12345678901234567890123456789012

1. (1) x†Ç°=°O  ∴ Hê=‹²# ° '4Ñ H›=ô.


'N'

7 8  5 3 4 2 8 9 7 2 4 5 9 2  4 7 9. (3) 10. (3)


̄á ã‰õ}÷…Õ Hê=‹²# '7с° ï~O_È° H›=ô. 11. (2) 6 7 6 0 8 4 3 6 2 9 0 8 2 2 3
2. (4) 1 #°O_ =~¡ä›½ '3ÑKÍ ƒìyO„¬|_È° ‹¬OY¼° 4 6 6 7 9 6
P~˂¬ì}ãH›=°O Z䛽ø=ª~¡°Á „¬ô#~Œ=$`ÇëO J~ò# JOïH « 5
3, 6, 9, 12, 15, 18, 21, 24, 27, 30, 33, `Ç䛽ø=ª~¡°Á „¬ô#~Œ=$`ÇO J~ò# JOïH « 1
36, 39, 42, 45 ƒè^ÎO « 5 - 1 « 4
'9Ñ= ªÖ#O…Õ =KÇ°ó ‹¬OY¼ « 27. 12. (1) 701354697386428
(3) 11 #°O_ 50 =°^Î¼…Õ '7ÑKÍ ƒìyOKÇ|_, 2KÍ 9 6 5 2

Y
∴ =òO^ΰ, `Ç~¡°"Œ`Ç ‹¬i‹¬OY¼ H›ey# ‹¬i‹¬OY¼
3.

ƒìyOKÇ|_Èx ‹¬OY¼° ‹¬OY¼ 3.

M
21, 35, 49
9 #°O_ 54 =°^Î¼ Q® ‹¬OY¼…Õ '9ÑKÍ ƒìyOKÇ|_, 13. (4) 389765403576340
.

(4) 6 5 8 4.

E
4.

3KÍ ƒìyO„¬|_#q Ug° LO_È=ô. 0 1 2 3 4 5 6 7 8 9

D
9
Hê=‹²# ‹¬OY¼° « 7.

59
(1) ↓ ↓ ↓ ↓ ↓ ↓ ↓ ↓ ↓ ↓
2 0 0 3 3 3 3 2 2 1
5.

Jq 4, 24, 40, 44, 48, 64, 84. CA


95
x†Ç°=°O  1, 2 ° J‹¬° W=Þ|_ȅè^ΰ. Hê=ô# JxßO\÷ H›<Œß
50
(3)
`Ç䛽ø=ª~¡°Á „¬ô#~Œ=$`ÇO J~ò# ‹¬OY¼ 9.
6.

Wzó# ã‰õ}÷ 98
A
8 9  2 2 6 3 2 6 9 7 3 2  14. (1) 678943218468059321
:8

778737794 123950864812349876
E
h

Wzó# JOïH ãH=› °O#° =°i†Çò JOïH#° `Œ~¡°=¶~¡°


.P

Hê=‹²# '7с° 2 H›=ô.


IC

^Î`Çëã‰õ}÷…Õ Hê=‹²# '7Ñ  ï~O_È° H›=ô. Jq K͆ǰQê =zó# JOïH ãH›=°O#° „¬ijeOKÇQê ï~O_È°
AD

(2)
JOïH° J^Í ªÖ<Œ…Õ =ô<Œß~ò.
7.
AB
O

7 3 4 5 
x†Ç°=°O /95
9 3  9 10 12 45
15. (4) 7 5 9 6 4 3 9 5 2
ER

P~˂¬ì} ãH›=°O…Õ ~Œ†Ç°Qê


CH

8. (2)
Wzó# ã‰õ}÷ 2 3 4 5 5 6 7 9 9
YD

/9/ /9$ Wzó# ãH›=°O 7 5 9 6 4 3 9 5 2


,H

T Q M N X T M X N Q M N

/92 /9. U XH›ø JOïH 䛀_¨ J^ͪÖ#O…Õ Hù#ªQ®…è^ΰ.


T
AM

N X T X N A M X .

123456789012345678901234567890121234567890
123456789012345678901234567890121234567890123456789012345678901212
H

123456789012345678901234567890121234567890
123456789012345678901234567890121234567890123456789012345678901212
12345678901234567890123456789012123456789012345678901234567890121234567890123456789012345678901212345678901234567890123
1234567890123456789012345678901212345678901234567890123456789012123456789012345678901234567890121234567890123
1234567890123456789012345678901212345678901234567890123456789012123456789012345678901234567890121234
12345678901234567890123456789012123456789012345678901234567890121234567890123456789012345678901212345678901234567890123456789012123
M

12345678901234567890123456789012123456789012345678901234567890121234567890123456789012345678901212345678901234567890123
1234567890123456789012345678901212345678901234567890123456789012123456789012345678901234567890121234567890123
1234567890123456789012345678901212345678901234567890123456789012123456789012345678901234567890121234
123456789012345678901234567890121234567890
123456789012345678901234567890121234567890123456789012345678901212
12345678901234567890123456789012123456789012345678901234567890121234567890123456789012345678901212345678901234567890123456789012123
12345678901234567890123456789012123456789012345678901234567890121234567890123456789012345678901212345678901234567890123
1234567890123456789012345678901212345678901234567890123456789012123456789012345678901234567890121234567890123
1234567890123456789012345678901212345678901234567890123456789012123456789012345678901234567890121234
12345678901234567890123456789012123456789012345678901234567890121234567890123456789012345678901212345678901234567890123456789012123
123456789012345678901234567890121234567890123456789012345678901212
PREVIOUS BITS on TYPE - I
12345678901234567890123456789012123456789012345678901234567890121234567890123456789012345678901212345678901234567890123
1234567890123456789012345678901212345678901234567890123456789012123456789012345678901234567890121234567890123
1234567890123456789012345678901212345678901234567890123456789012123456789012345678901234567890121234
123456789012345678901234567890121234567890
12345678901234567890123456789012123456789012345678901234567890121234567890123456789012345678901212345678901234567890123456789012123
AM

12345678901234567890123456789012123456789012345678901234567890121234567890123456789012345678901212345678901234567890123456789012123
12345678901234567890123456789012123456789012345678901234567890121234567890123456789012345678901212345678901234567890123
1234567890123456789012345678901212345678901234567890123456789012123456789012345678901234567890121234567890123
1234567890123456789012345678901212345678901234567890123456789012123456789012345678901234567890121234
123456789012345678901234567890121234567890123456789012345678901212
IG

12345678901234567890123456789012123456789012345678901234567890121234567890123456789012345678901212345678901234567890123456789012123
12345678901234567890123456789012123456789012345678901234567890121234567890123456789012345678901212345678901234567890123
1234567890123456789012345678901212345678901234567890123456789012123456789012345678901234567890121234567890123
1234567890123456789012345678901212345678901234567890123456789012123456789012345678901234567890121234
123456789012345678901234567890121234567890123456789012345678901212
12345678901234567890123456789012123456789012345678901234567890121234567890123456789012345678901212345678901234567890123456789012123
12345678901234567890123456789012123456789012345678901234567890121234567890123456789012345678901212345678901234567890123
1234567890123456789012345678901212345678901234567890123456789012123456789012345678901234567890121234567890123
1234567890123456789012345678901212345678901234567890123456789012123456789012345678901234567890121234

ãH÷O^Î L#ß ‹¬OMì¼ ã‰õ}÷x „¬ijeOz


KH

‹¬¶KÇ# : (1 =°i†Çò 2) 4. D ãH÷Ok W=Þ|_# JOïH ãH›=°O…Õ 6䛽 =òO^ΰ


ãH÷Ok ㄬ‰×߁䛽 ‹¬=¶^¥<Œ° H›#°Qù#°=ò.
R

…è^¥ 7䛽 `Ç~¡°"Œ`Ç Zxß 5° H›=ô ? (S.I - 2011)


7897653428972459297647 3124567565724756657
(H›=ü¼xöH+¬<£ S.I - 2012) 1) 1 2) 3 3) 2 4) 4 5) 5
1. ã‰õ}÷…Õ L#ß JOïH "³ò`ÇëO ZO`Ç ? 5. ãH÷Ok ã‰õ}÷…Õ 4䛽 =òO^ΰQêx, 3䛽 `Ç~¡°"Œ`Ç Q® 6°
1) 130 2) 140 3) 120 4) 129 Zxß H›=ô ? (S.I - 2010)
2. "³ò^Î\ ÷ ‹¬Q®O ã‰õ}÷…Õx JOïH "³ò`ŒëxH÷, ï~O_È= ‹¬Q®O 6 6 4 2 6 4 6 3 6 43 3 4 6 3 6 46 3
ã‰õ}÷…Õx JOïH "³ò`ŒëxH÷ Q® `Í_¨ ZO`Ç ? 1) 1 2) 2 3) 3 4) 4
1) 5 2) 4 3) 7 4) 6 6. ãH÷Ok ã‰õ}÷…Õ "³O@<Í =ü_È° …è䛽O_¨, J~ò`Í "³O@<Í
3. ãH÷Ok W=Þ|_# ã‰õ}÷…Õ Zxß ª~¡°Á =~¡°‹¬ ‹¬OY¼ Zxq°k Q®, 7 ‹¬OY¼ Zxß H›^ΰ ?
=°^Î¼ `Í_¨ ï~O_È° LOk. (‡éb‹¹ HêxÀ‹“|°…˜ - 2012) 8987622632697328727787377
641222874215358679 (‡éb‹¹ HêxÀ‹“|°…˜ - 2009)
1) 3 2) 2 3) 4 4) 6 1) 2 2) 3 3) 4 4) 6
REASONING ABILITY (VERBAL) ○ ○ ○ ○ ○ ○ ○ ○ ○
110
○ ○ ○ ○ ○ ○ ○ ○ ○ ○ ○ ○ ○ ○ ○ ○ ○ ○ ○
Study Material
7. ãH÷O^Î À„~ùø#ß =~¡°‹¬ãH›=°O…Õ 6 =òO^ΰQê …èx, 8H÷ 6795697687678694677695763 (S.I - 2008)
"³<³ÞO@<Í …èx 3 ‹¬OY¼ Zxß H›=ô ? 1) 1 2) 2 3) 3 4) 4
936639537891639639 (‡éb‹¹ HêxÀ‹“|°…˜ - 2009) 10. ãH÷O^Î À„~ùø#ß ‹¬OMì¼`ÇOãu…Õ Zxß 2° 6 H›O>è
1) XH›\ ÷ 2) ï~O_È° 3) =ü_È° 4) <Œ°Q®° =òO^ΰQê#°, 5 `Ç~ŒÞ`Ç Qê#° =KÇ°ó#° ?
8. ‹¬OMì¼ ã‰õ}÷x KÇkq `Ç~¡°"Œ`Ç ã„¬‰×ß䛽 ["Œ|°e=òà 253486250126262501 (‡éb‹¹ HêxÀ‹“|°…˜ 2008)
7897653428972459297647 1) 0 2) 1 3) 2 4) 3
̄á ã‰õ}÷…Õ 7䛽 =òO^ΰ 9#° H›ey, 7䛽 "³<³ÞO@<Í 6#° 11. ãH÷O^Î W=Þ|_# ã‰õ}÷…Õ Zxߪ~¡°Á =~¡‹¬ ‹¬OY¼ =°^Î¼
H›ey# 7° Zxß H›=ô (S.I - 2008) `Í_¨ ï~O_È° LOk ? (S.I - 2006)
1) 2 2) 3 3) 4 4) 5 641228742153862171413285
9. D ãH÷Ok ‹¬OMì¼ ã‰õ}÷…Õ 6䛽 =òO^ΰ 7#° H›ey, 6䛽 1) XH›\ ÷ 2) ï~O_È° 3) =ü_È° 4) S^ΰ
"³<³ÞO@<Í 9 ~Œä›½O_¨ LO_Í 6° Zxß H›=ô ?
12345678901234567890123456789012123456789012345678901234567890121234567890123456789012345678901212345678901234567890123
1234567890123456789012 123456789012345678901234

q=~¡}ì`ÇàH› ["Œ|°°
12345678901234567890123456789012123456789012345678901234567890121234567890123456789012345678901212345678901234567890123
1234567890123456789012 123456789012345678901234

Y
1234567890123456789012 123456789012345678901234
12345678901234567890123456789012123456789012345678901234567890121234567890123456789012345678901212345678901234567890123

1234567890123456789012
PREVIOUS BITS :
12345678901234567890123456789012123456789012345678901234567890121234567890123456789012345678901212345678901234567890123
1234567890123456789012 123456789012345678901234
12345678901234567890123456789012123456789012345678901234567890121234567890123456789012345678901212345678901234567890123
1234567890123456789012 123456789012345678901234
12345678901234567890123456789012123456789012345678901234567890121234567890123456789012345678901212345678901234567890123
1234567890123456789012 123456789012345678901234
12345678901234567890123456789012123456789012345678901234567890121234567890123456789012345678901212345678901234567890123
123456789012345678901234

M
(1) JOïH "³ò`ÇëO (2)
7{8{9{7{6{5{3{4{2{8
1. 5.

^Î`Çëã‰õ}÷ Hê=‹² '7с° 3 H›=ô.

E
(2)
{9{7{2{4{5{9{2{9{7{
6.

D
9
6«4{7    

59
8 9 8 6 2 2 6 3 2 6 9 7 3 2 8 2 7 7 3 7.

« 130 (2)
CA  

95
7. 6 3 9 8 9 1 6 3 9 6 3 9

(4) Hê=‹²# ƒè^ÎO (1) 

50
« (7 { 8 { 9 { 7 { 6 { 5 { 3 { 4 { 2 {
2. 8.

8 { 9) - (7 { 2 { 4 { 5 { 9 { 2 { 9 { 7 98
A
7 8  5 3 4 2 8 9 7 2 4 5 9 2  4 7

{ 6 { 4 { 7) Hê=‹²# '7с° 2 H›=ô.


:8

« 68 - 62 « 6
E

(3)
h


6 - 4 « 2, 4 - 2 « 2,
9.
.P
IC

(4)
5 - 3 « 2, 5 - 3 « 2,  
3.
AD

6 7 9 5 6 9 8 6 9 4 6 7 7 6 9

8 - 6 « 2, 9 - 7 « 2. 
AB
O

∴ 6 ª~¡°Á. Hê=‹²# '6с° 3 H›=ô.


ER
CH

4. (3)  10. (1) 


^Î`Çëã‰õ}÷…Õ Hê=‹²# '2с° Ug°…è=ô.
YD

3 1 2 4 5 6  5 7 2 4  6 5 7

Hê=‹²# '5с 2 H›=ô.


,H

11. (4) 6 4 1 2 2 8 7 4 2 1 5 3 8 6 2 1 7 1 4 1 3 2 8 5
T
AM

~Œ¼OH÷OQ· >ÿ‹¹“ =¶ki ㄬ‰×߁°


H

TYPE - II :
M
AM
IG

1. XH› `Ç~¡Q®u…Õ Ja~ŒO †³òH›ø ~Œ¼O䛽 "³ò^Î\ ÷ #°O_ ª^Î#: "³ò`ÇëO « 50


KH

10= ~Œ¼O䛽 z=i #°O_ 10= ~Œ¼O䛽 J~ò`Í P


R

8 « ‰§~¡^Î
`Ç~¡Q®u…Õx "³ò`ÇëO q^¥¼~¡°Ö ‹¬OY¼ ZO`Ç? R B
ª^Î#: ï~O_È° "³á„¬ô…ì Wzó# ~Œ¼O䛽#° H›e„² "Œ\÷ #°O_ '1Ñ 10 25

f‹²#KË "³ò`ÇëO q^¥¼~¡°Ö ‹¬OY¼ =‹¬°ëOk. 15 =°Ok

(10 { 10) - 1 « 20 - 1 « 19 ~Œ=ò䛽  ƒì°ä›½ =°^Î¼…Õ 15 =°Ok H›~¡°.


2. XH› =~¡°‹¬…Õ =òO^ΰ #°O_ ~Œ=ò †³òH›ø ~Œ¼OH± Hê=ô# 8 †³òH›ø ªÖ#=ò "Œi =°^Î¼ 8= ªÖ#=ò
10Qê#°  "³#°H› #°O_ ƒì° †³òH›ø ~Œ¼OH± 25 Qê#° J=ô`Ç°Ok.
=°i†Çò ‰§~¡^Î "Œi^ÎíiH÷ ‹¬iQê¾ =°^Î¼…Õ =ôO_<Œ  ‰§~¡^Î ªÖ#=ò =òO^ΰ #°Oz « 10 { 8 «
P =~¡°‹¬…Õ "³ò`ÇëO 50 =°Ok H›~¡°. J~ò<Œ ‰§~¡^Î 18= ªÖ#=ò
=òO^ΰ #°O_ U ªÖ#=ò…Õ H›^ΰ. ‰§~¡^Î ªÖ#=ò "³#°H› #°Oz « 25 { 8 « 33=
ªÖ#=ò
REASONING ABILITY (VERBAL) ○ ○ ○ ○ ○ ○ ○ ○ ○
111
○ ○ ○ ○ ○ ○ ○ ○ ○ ○ ○ ○ ○ ○ ○ ○ ○ ○ ○
Study Material
123456789012345678901234567890121234567890
123456789012345678901234567890121234567890
123456789012345678901234567890121234567890123456789012345678901212
1234567890123456789012345678901212345678901234567890123456789012123456789012345678901234567890121234567890123
1234567890123456789012345678901212345678901234567890123456789012123456789012345678901234567890121234
12345678901234567890123456789012123456789012345678901234567890121234567890123456789012345678901212345678901234567890123456789012123
12345678901234567890123456789012123456789012345678901234567890121234567890123456789012345678901212345678901234567890123
1234567890123456789012345678901212345678901234567890123456789012123456789012345678901234567890121234567890123
1234567890123456789012345678901212345678901234567890123456789012123456789012345678901234567890121234
123456789012345678901234567890121234567890
123456789012345678901234567890121234567890123456789012345678901212
~Œ¼H÷OQ· >ÿ‹¹“ : TYPE - II : PRACTICE TEST
12345678901234567890123456789012123456789012345678901234567890121234567890123456789012345678901212345678901234567890123456789012123
12345678901234567890123456789012123456789012345678901234567890121234567890123456789012345678901212345678901234567890123
1234567890123456789012345678901212345678901234567890123456789012123456789012345678901234567890121234567890123
1234567890123456789012345678901212345678901234567890123456789012123456789012345678901234567890121234
12345678901234567890123456789012123456789012345678901234567890121234567890123456789012345678901212345678901234567890123456789012123
123456789012345678901234567890121234567890123456789012345678901212
12345678901234567890123456789012123456789012345678901234567890121234567890123456789012345678901212345678901234567890123
1234567890123456789012345678901212345678901234567890123456789012123456789012345678901234567890121234567890123
1234567890123456789012345678901212345678901234567890123456789012123456789012345678901234567890121234
123456789012345678901234567890121234567890
12345678901234567890123456789012123456789012345678901234567890121234567890123456789012345678901212345678901234567890123456789012123
123456789012345678901234567890121234567890123456789012345678901212
12345678901234567890123456789012123456789012345678901234567890121234567890123456789012345678901212345678901234567890123
1234567890123456789012345678901212345678901234567890123456789012123456789012345678901234567890121234567890123
1234567890123456789012345678901212345678901234567890123456789012123456789012345678901234567890121234
12345678901234567890123456789012123456789012345678901234567890121234567890123456789012345678901212345678901234567890123456789012123
12345678901234567890123456789012123456789012345678901234567890121234567890123456789012345678901212345678901234567890123
1234567890123456789012345678901212345678901234567890123456789012123456789012345678901234567890121234567890123
1234567890123456789012345678901212345678901234567890123456789012123456789012345678901234567890121234
123456789012345678901234567890121234567890
12345678901234567890123456789012123456789012345678901234567890121234567890123456789012345678901212345678901234567890123456789012123
123456789012345678901234567890121234567890123456789012345678901212
12345678901234567890123456789012123456789012345678901234567890121234567890123456789012345678901212345678901234567890123456789012123
12345678901234567890123456789012123456789012345678901234567890121234567890123456789012345678901212345678901234567890123
1234567890123456789012345678901212345678901234567890123456789012123456789012345678901234567890121234567890123
1234567890123456789012345678901212345678901234567890123456789012123456789012345678901234567890121234
123456789012345678901234567890121234567890123456789012345678901212
12345678901234567890123456789012123456789012345678901234567890121234567890123456789012345678901212345678901234567890123456789012123
12345678901234567890123456789012123456789012345678901234567890121234567890123456789012345678901212345678901234567890123
1234567890123456789012345678901212345678901234567890123456789012123456789012345678901234567890121234567890123
1234567890123456789012345678901212345678901234567890123456789012123456789012345678901234567890121234
12345678901234567890123456789012123456789012345678901234567890121234567890123456789012345678901212345678901234567890123456789012123

1. XH› =~¡°‹¬…Õ "³¶‚¬ì<£ †³òH›ø ªÖ#=ò Z@°#°Oz KǶ‹²# 6. ~¡­ìhž †³òH›ø ~Œ¼O䛽 ̄á #°O_ 23 =°i†Çò ãH÷O^Î
17= ªÖ#=ò  Qˇ…˜ 䛽_"„ á³ ô¬ #°O_ 18 J~ò# Z_È=° #°O_ 42 J~ò# "³ò`ÇëO ZO`Ç =°Ok H›~¡°?
"³„
á ô¬ #°O_ Qˇ…˜ ~Œ¼OH± ZO`Ç? 1) 65 2) 64 3) 63 4) ̄á=x߆Çò
1) 15 2) 16 3) 17 4) 18 7. XH› `Ç~¡Q®u…Õ "³ò`Çë=ò q^¥¼~¡°Ö ‹¬OY¼ 48 =°i†Çò
2. XH› =~¡°‹¬…Õ ~Œ=ò †³òH›ø ~Œ¼OH± Z_È=° #°O_ 23 =Oj †³òH›ø ~Œ¼O䛽 "³ò^Î\ ÷ #°O_ 15 J~ò# z=i
=°i†Çò 䛽_ #°O_ 32 J~ò# P =~¡°‹¬…Õ "³ò`ÇëO #°O_ J`Çx ~Œ¼O䛽 ZO`Ç?
ZO`Ç =°Ok H›~¡°. UnHê^ΰ
UnHê^ΰ
1) 34 2) 33 3) 25 4)

1) 55 2) 54 3) 45 4)
8. XH› =~¡°‹¬…Õ Pk`Ǽ =òO^ΰ #°O_ 10= ªÖ#O…Õ
3. XH› `Ç~¡Q®u Q®k…Õ =°=°`Ç †³òH›ø ~Œ¼OH± 16  =°‚¬ðH÷;

Y
Ja "³#°ä›½ #°O_ 25= ªÖ#O…Õ, QËqO^£ J#° =¼H÷ë
†³òH›ø ~Œ¼OH± z=i #°O_ 11. H÷; †³òH›ø ~Œ¼OH± Pk`Ǽ, Ja =°^Î¼…Õ H›_È°. "³ò`ÇëO 50 =°Ok L#ß

M
=°=°`Ç `Ç~¡°"Œ`Ç 5 =°i†Çò H÷; =°=°`Ç, =°‚¬ðH÷; =~¡°‹¬…Õ QËqO^£ =òO^ΰ #°O_ Z#ß= ªÖ#O…Õ H›_È°.
=°^Î¼…Õ =ô#ßk. J~ò`Í P `Ç~¡Q®u…Õ "³ò`ÇëO q^¥¼~¡°Ö°

E
1) 20 2) 18 3) 19 4) 17

ZO`Ç =°Ok H›~¡°.

D
9. XH› ƒìeH› =~¡°‹¬…Õ ãÀ„=° Z_È=° #°O_ 7= ªÖ#O…Õ

9
UnHê^ΰ

59
1) 36 2) 63 3) 11 4)
=°i†Çò ~¡=°¼ 䛽_"³á„¬ô #°O_ 12= ªÖ#O…Õ L<Œß~¡°.
CA
95
4. XH› =~¡°‹¬…Õ x|_# =¼H÷ë †³òH›ø ªÖ#=ò Z@°"³á„¬ô
`Ç~ŒÞ`Ç "Œ~¡° `Ç=° ªÖ<Œ#° „¬~¡‹¬æ~¡O =¶~¡°óHùx#,
#°O_ KǶ‹²<Œ J`Çx ªÖ#=ò ''13ÑÑ J~ò# P =~¡°‹¬…Õ

50
ãÀ„=° Z_È=° #°O_ 22= ªÖ#O…Õ LO@°Ok. J~ò<Œ
ZO`Ç=°Ok H›~¡°?
98
A
UnHê^ΰ P =~¡°‹¬…Õ ZO`Ç=°Ok ƒìeH›° H›~¡°?
:8
1) 25 2) 24 3) 26 4)

K³„¬æ…è=ò
E

5. XH› =~¡°‹¬…Õ x|_# q^¥¼~¡°Ö……Õ Z_È=°"³á„¬ô #°O_


1) 33 2) 19 3) 31 4)
h

10. POQ®Á JH›Æ~¡=¶…Õ z=i #°Oz 21 JH›Æ~Œ°, =°i†Çò


.P

Q®OQê^Î~Ÿ †³òH›ø ªÖ#=ò 13 =°i†Çò 䛽_"³á„¬ô #°O_


IC

"³ò^Î\ ÷ #°Oz 20 JH›Æ~Œ° f‹¬°ä›½O>è "Œ\÷…Õ =°^Î¼…Õ


AD

J`Çx ªÖ#=ò 15 J~ò`Í WOHê ZO`Ç=°Ok H›e„²`Í


"³ò`Çë=ò 32 =°Ok J=ô`Œ~¡°. LO_Í JH›Æ~¡O Uk?
AB
O

UnHê^ΰ 1) N 2) L 3) K 4) M
ER
CH

1) 3 2) 5 3) 2 4)

12345678901234567890123456789012
12345678901234567890123456789012123456789012345678901234567890121234567890123456789012345678901212345678901234567890123
12345678901234567890123456789012
12345678901234567890123456789012123456789012345678901234567890121234567890123456789012345678901212345678901234567890123
12345678901234567890123456789012 12345678901234567890123456789012

q=~¡}ì`ÇàH› ["Œ|°°
YD

12345678901234567890123456789012
12345678901234567890123456789012123456789012345678901234567890121234567890123456789012345678901212345678901234567890123
12345678901234567890123456789012 12345678901234567890123456789012
12345678901234567890123456789012 12345678901234567890123456789012
12345678901234567890123456789012123456789012345678901234567890121234567890123456789012345678901212345678901234567890123
12345678901234567890123456789012
12345678901234567890123456789012123456789012345678901234567890121234567890123456789012345678901212345678901234567890123
12345678901234567890123456789012
12345678901234567890123456789012123456789012345678901234567890121234567890123456789012345678901212345678901234567890123
12345678901234567890123456789012 12345678901234567890123456789012
12345678901234567890123456789012
12345678901234567890123456789012123456789012345678901234567890121234567890123456789012345678901212345678901234567890123
,H
T

"³ò`Çë=ò « 2 þ 17 - 1 "³ò`Çë=ò « 2 þ 13 - 1
AM

1. (2) 4. (1)
« 34 - 1 « 33 « 26 - 1
H
M

Qˇ…˜ ~Œ¼OH± 䛽_"³á„¬ô « 18 « 25


AM
IG

Qˇ…˜ ~Œ¼OH± Z_È=°"³á„¬ô « 33 - 18 { 1 (2) "³ò`Çë=ò « 13 { 15 - 1


KH

5.

« 16 « 28 - 1
R

2. (2) "³ò`Çë=ò q^¥¼~¡°Ö ‹¬OY¼ « 23 { 32 - 1 « 27


« 55 - 1 32 =°Ok JQ®°@䛽 WOHê#° '5Ñ =°Okx H›„¬=…ÿ#°.
« 54 6. (2) "³ò`Çë=ò « 23 { 42 - 1
3. (1) H÷; « 65 - 1
{1 « 64
{4
21 {4 (1) "³ò`ÇëO q^¥¼~¡°Ö ‹¬OY¼ « 48
16
7.

=°=°`Ç =°‚¬ðH÷; 11 "³ò^Î\÷ #°O_ ~Œ¼OH± « 15


z=i #°O_ ~Œ¼OH± « 48 - 15 { 1
"³ò`Çë=ò « 16 { 5 { 11 { 4 « 49 - 15
« 36 « 34
REASONING ABILITY (VERBAL) ○ ○ ○ ○ ○ ○ ○ ○ ○
112
○ ○ ○ ○ ○ ○ ○ ○ ○ ○ ○ ○ ○ ○ ○ ○ ○ ○ ○
Study Material
8. (2) Pk`Ǽ, Ja =°^Î¼Q® =¼ä›½ë ‹¬OY¼ ~¡=°¼ †³òH›ø „¬î~¡Þ„¬ô ªÖ#O. J~ò# P ƒìeH›
« 50-(10{25)«15 =~¡°‹¬ « 21{1{11«33 =°Ok ƒìeH›°.
QËqO^£ D 15 =°Ok =°^Î¼…Õ H›_È°. 10. (4) z=i #°O_ 21 "³ò^Î\÷ #°O_ 20
Hê=ô# QËqO^£ ªÖ#O Pk`Ǽ #°O_ 8=k. J#Qê =òO^ΰ ' 5
#°O_ 18=k.
A B C D E G H I J K L M N O P Q R S U

(1) ãÀ„=° †³òH›ø ㄬ‹¬°ë`Ç ªÖ#O Z_È=° #°O_ 22=k.


V W X Y Z

9.
 =°^Î¼…Õ L#ß JH›Æ~¡O «
Hêh, D ªÖ#O 䛽_"³á„¬ô #°O_ 12= ªÖ#O…Õ L#ß
F, T M

123456789012345678901234567890121234567890
123456789012345678901234567890121234567890123456789012345678901212
123456789012345678901234567890121234567890
12345678901234567890123456789012123456789012345678901234567890121234567890123456789012345678901212345678901234567890123
1234567890123456789012345678901212345678901234567890123456789012123456789012345678901234567890121234567890123
1234567890123456789012345678901212345678901234567890123456789012123456789012345678901234567890121234
123456789012345678901234567890121234567890123456789012345678901212
12345678901234567890123456789012123456789012345678901234567890121234567890123456789012345678901212345678901234567890123456789012123
12345678901234567890123456789012123456789012345678901234567890121234567890123456789012345678901212345678901234567890123
1234567890123456789012345678901212345678901234567890123456789012123456789012345678901234567890121234567890123
1234567890123456789012345678901212345678901234567890123456789012123456789012345678901234567890121234
123456789012345678901234567890121234567890
12345678901234567890123456789012123456789012345678901234567890121234567890123456789012345678901212345678901234567890123456789012123
123456789012345678901234567890121234567890123456789012345678901212
12345678901234567890123456789012123456789012345678901234567890121234567890123456789012345678901212345678901234567890123
1234567890123456789012345678901212345678901234567890123456789012123456789012345678901234567890121234567890123
1234567890123456789012345678901212345678901234567890123456789012123456789012345678901234567890121234
12345678901234567890123456789012123456789012345678901234567890121234567890123456789012345678901212345678901234567890123456789012123
PREVIOUS BITS on TYPE - II
123456789012345678901234567890121234567890123456789012345678901212
12345678901234567890123456789012123456789012345678901234567890121234567890123456789012345678901212345678901234567890123
1234567890123456789012345678901212345678901234567890123456789012123456789012345678901234567890121234567890123
1234567890123456789012345678901212345678901234567890123456789012123456789012345678901234567890121234
123456789012345678901234567890121234567890
12345678901234567890123456789012123456789012345678901234567890121234567890123456789012345678901212345678901234567890123456789012123
12345678901234567890123456789012123456789012345678901234567890121234567890123456789012345678901212345678901234567890123456789012123
12345678901234567890123456789012123456789012345678901234567890121234567890123456789012345678901212345678901234567890123
1234567890123456789012345678901212345678901234567890123456789012123456789012345678901234567890121234567890123
1234567890123456789012345678901212345678901234567890123456789012123456789012345678901234567890121234
123456789012345678901234567890121234567890123456789012345678901212
12345678901234567890123456789012123456789012345678901234567890121234567890123456789012345678901212345678901234567890123456789012123
12345678901234567890123456789012123456789012345678901234567890121234567890123456789012345678901212345678901234567890123
1234567890123456789012345678901212345678901234567890123456789012123456789012345678901234567890121234567890123
1234567890123456789012345678901212345678901234567890123456789012123456789012345678901234567890121234
12345678901234567890123456789012123456789012345678901234567890121234567890123456789012345678901212345678901234567890123456789012123
1234567890123456789012345678901212345678901234567890123456789012123456789012345678901234567890121234567890123
1234567890123456789012345678901212345678901234567890123456789012123456789012345678901234567890121234

1. XH› ‹¬=¶"͉×=ò#䛽 ‚¬ð[~¡=ô@䛽 ̋ãH›@iQê~¡° 10.10 9,3,6,6,3,9,5,3,7,8,9,1,6,3,9,6,3,9 (PC-2009)


1) 1 2) 2 3) 3

Y
am 䛽 =KŒó~¡°. P†Ç°# K³á~Ÿ„¬~¡ž<£ H›O>è 30 xII =òO^ΰ 4) 4

M
=KŒó~¡°. K³á~Ÿ„¬~¡ž<£ Qê~¡° ‹¬=¶"Í‰× xsâ`Ç ‹¬=°†Ç¶xH÷ 5. XH› K³@Á =~¡°‹¬…Õ XH› K³@°“ z=i #°O_, "³ò^Î\ ÷ #°O_
10 xII° P‹¬¼OQê =KŒó~¡°. ‹¬=¶"Í‰× xsâ`Ç ‹¬=°†Ç°O 5= ªÖ#O…Õ LOk J~ò`Í P =~¡°‹¬…Õ L#ß K³@Á ‹¬OY¼

E
ZO`Ç? (TS.Const. Mains -2016) ZO`Ç? (‡éb‹¹ HêxÀ‹“|°…˜ž-2009)

1) 2) 1) 8 2) 9 3) 10 4) 11

D
9
59
10.40 am 10.10 am

3) 4) 6. J[†Ÿ ° WO\÷ #°O_ |ª“ „ ¹ ä › ½ ~ËA H› O >è 15xII


CA
95
10.30 am 10.20 am

2. XH› ^ùOQ® XH› Hê~¡°#° =°^¥¼‚¬ìßO 1 Q®O@䛽 ^ùOyeOz =òO^ΰQê<Í |†Ç°°^Í~Œ_È°. |ª“„¹ KÍ~¡_¨xH÷ 10 xII

50
^¥xx Q®O@䛽 60 H÷.g°. "ÍQ®O`Ë XH› ~¡‚¬ì^¥iÌ„á „¬_È°`Ç°Ok. J`Ç_È° 8.45 xIIä›½ |ª“„¹ KÍ~Œ_È°. J~ò`Í
98
A
#_È „ ¬ ª ï Q #°. =°^ ¥ ¼‚¬ ì ßO 2 Q® O @ä› ½ Hê~¡ ° J`Ç_È° WO\÷ #°O_ U ‹¬=°†Ç°O…Õ |†Ç°°^Í~Œ_È°?
:8
(‡éb‹¹ HêxÀ‹“|°…˜ž-2009)
^ùOyeOKÇ|_#^Îx x~ŒœxOKÇ°Hùx XH› ‡éb‹¬° ƒÿá䛽̄á
1) 8.30 2) 8.45
E
h

80 H÷.g°. "ÍQ®O`Ë "³O|_À‹ë Zxß Q®O@ ‹¬=°†Ç°O…Õ


.P

am am

3) 8.00 4) 8.35
IC

‡éb‹¬° ^ùOQ®#° ^¥@Q®_È°? (‡éb‹¹ HêxÀ‹“|°…˜ž-2013) am am


AD

1) 3.30 2) 4.00 7. XH› ƒì°~¡ =~¡°‹¬…Õ, ~Œ=ò Z_È=°"³á„¬ô #°O_ 10=


AB
O

p.m. p.m.

3) 5.00 p.m. 4) 6.00 p.m.


ªÖ#O…Õ#¶ 䛽_"³á„¬ô #°O_ 17= ªÖ#O…Õ#° H›^ΰ.
ER

J~ò`Í =~¡°‹¬…Õ Q® "³ò`Çë=ò ƒì°~¡ ‹¬OY¼?


CH

3. ãH÷Ok ã‰õ}…
÷ Õ 5䛽 =òO^ΰ 3䛽 `Ç~ŒÞ`Ç Zxß 7° H›=ô?
(ZïHá ž*˜ HêxÀ‹“|°…˜ž-2013)
YD

77537573773357375753
1) 25 2) 26 3) 27 4) 17
,H

(H›=ü¼xöH+¬<£ HêxÀ‹“|°…˜ž-2010)

1) 1 2) 2 3) 3 4) 4 8. 70 =°Ok Q® XH› `Ç~¡Q®u…Õ ̄á ~Œ¼OH± #°O_ P‹²„¦¹


T
AM

30= ªÖ#=ò…Õ H›^ΰ. kQ®°= #°O_ J`Çx ~Œ¼OH±?


4. D ãH÷Ok =~¡°‹¬…Õ 9H÷ =òO^ΰ 6 `Ç~ŒÞ`Ç =KÍó '3с°
H
M

(‡éb‹¹ HêxÀ‹“|°…˜ž-2013)
Zxß H›=ô?
1) 39 2) 40 3) 41 4) 100
AM
IG

12345678901234567890123456789012123456789012345678901234567890121234567890123456789012345678901212345678901234567890123
1234567890123456789012 123456789012345678901234
KH

12345678901234567890123456789012123456789012345678901234567890121234567890123456789012345678901212345678901234567890123
1234567890123456789012 123456789012345678901234
1234567890123456789012
1234567890123456789012
1234567890123456789012 PREVIOUS BITS : q=~¡}ì`ÇàH› ["Œ|°°
12345678901234567890123456789012123456789012345678901234567890121234567890123456789012345678901212345678901234567890123
123456789012345678901234
123456789012345678901234
12345678901234567890123456789012123456789012345678901234567890121234567890123456789012345678901212345678901234567890123
123456789012345678901234
12345678901234567890123456789012123456789012345678901234567890121234567890123456789012345678901212345678901234567890123
R

1234567890123456789012
1234567890123456789012
123456789012345678901234
12345678901234567890123456789012123456789012345678901234567890121234567890123456789012345678901212345678901234567890123
123456789012345678901234
12345678901234567890123456789012123456789012345678901234567890121234567890123456789012345678901212345678901234567890123

1. Sol : (3) 4. (3)

‹¬=¶"Í‰× xsâ`Ç ‹¬=°†Ç°=ò « 9 3 6 6  9 5 3 7 8 9 1 6  9 6  9

5.
10:30 am

(2)
K³á~Ÿ„¬~¡ž<£ Qê~¡° =zóOk #䛽
"³ò^Î\÷ #°O_ z=i #°O_
= 10 : 10

̋ãH›@s Qê~¡° =zóOk « 10 : 40 #䛽      


2. (3) 2pm + 3 hours = 5 pm
P =~¡°‹¬…Õ "³ò`ÇëO '9Ñ K³@°Á L<Œß~ò.
3. (2) ^Î`Çëã‰õ}÷ 6. (4)
7. (2) "³ò`ÇëO ƒì°~¡ ‹¬OY¼ 10{17-1 « 26
7 7 5 3 7 5 7 3 7 7 3 3 5 7 3 7 5 7 5 3
8. (3) kQ®°= #°O_ ~Œ¼OH± 70-30{1 « 41
REASONING ABILITY (VERBAL) ○ ○ ○ ○ ○ ○ ○ ○ ○
113
○ ○ ○ ○ ○ ○ ○ ○ ○ ○ ○ ○ ○ ○ ○ ○ ○ ○ ○
Study Material
TYPE - III : >ÿØ"£° ‹ÔÞïH<£ž >ÿ‹¹“ =¶ki ㄬ‰×߁°
1. _bÁ ï~á…èÞÀ‹“+¬<£ #°O_ ㄬu J~¡Ö Q®O@䛽 XH› ï~ၰ 2. "³¶‚¬ì<£ J<Í =¼H÷ë D ~ËA䛽 ‹²x=¶#° KǶz 9 ~ËA°
|†Ç ° °^Í ~ ¡ ° `Ç ° Ok. J~ò`Í ZOH› Þ ~òs H› Á ~ Ÿ ø
J=ô`Ç°Ok. =°i†Çò J`Ç#° ‹²x=¶ä›½ "³ˆõÁk öH=O
ㄬ†Ç¶}÷䛽ä›½ D q^ÎOQê K³„²æ<Œ_È°. XH› ï~ၰ À‹“+¬<£
#°O_ |†Ç ° °^Í i W„¬ æ \÷ H ÷ 10 xII° J~ò#k Q®°~¡°"Œ~¡=ò =¶ã`Ç"Í° J~ò`Í D ~ËA U "Œ~¡
=°i†Çò `Ç~¡°"Œ`Ç ï~ၰ À‹“+¬<£ #°O_ Q®O@ 9:35 =°Q®°#°?
xIIä› ½ |†Ç ° °^Í ~ ¡ ° #°. J~ò`Í ZOH› Þ ~òs H› Á ~ Ÿ ø ª^Î#: 7) 9 (1
ㄬ † Ç ¶ }÷ ä › ½ ä› ½ U<ñ<£ ž (Announce) KÍ ‹ ² #
‹¬=°†Ç°=ò#° H›#°Qù#°=ò.
7
ª^Î#: « 9:35 - 30
(R.R.B.-2002)
2
« 9.05 { 10 xII° ‰õ+¬=ò 2 Hê|\÷“ Q®°~¡°"Œ~¡=ò #°O_ ï~O_È= ~ËA.

Y
« Q®O. 9 :15 xII° ∴ ‰×x"Œ~¡=ò.

M
U<ñ<£ž K͋²# ‹¬=°†Ç°=ò : 9 Q®O@ 15 xII°
123456789012345678901234567890121234567890
123456789012345678901234567890121234567890
123456789012345678901234567890121234567890123456789012345678901212

E
12345678901234567890123456789012123456789012345678901234567890121234567890123456789012345678901212345678901234567890123
1234567890123456789012345678901212345678901234567890123456789012123456789012345678901234567890121234567890123
1234567890123456789012345678901212345678901234567890123456789012123456789012345678901234567890121234
12345678901234567890123456789012123456789012345678901234567890121234567890123456789012345678901212345678901234567890123456789012123

~Œ¼OH÷OQ· >ÿ‹“¹
123456789012345678901234567890121234567890123456789012345678901212
12345678901234567890123456789012123456789012345678901234567890121234567890123456789012345678901212345678901234567890123456789012123
12345678901234567890123456789012123456789012345678901234567890121234567890123456789012345678901212345678901234567890123
1234567890123456789012345678901212345678901234567890123456789012123456789012345678901234567890121234567890123
1234567890123456789012345678901212345678901234567890123456789012123456789012345678901234567890121234
123456789012345678901234567890121234567890
12345678901234567890123456789012123456789012345678901234567890121234567890123456789012345678901212345678901234567890123456789012123
12345678901234567890123456789012123456789012345678901234567890121234567890123456789012345678901212345678901234567890123
1234567890123456789012345678901212345678901234567890123456789012123456789012345678901234567890121234567890123
1234567890123456789012345678901212345678901234567890123456789012123456789012345678901234567890121234
123456789012345678901234567890121234567890123456789012345678901212
(II)
12345678901234567890123456789012123456789012345678901234567890121234567890123456789012345678901212345678901234567890123456789012123
12345678901234567890123456789012123456789012345678901234567890121234567890123456789012345678901212345678901234567890123
1234567890123456789012345678901212345678901234567890123456789012123456789012345678901234567890121234567890123
1234567890123456789012345678901212345678901234567890123456789012123456789012345678901234567890121234
123456789012345678901234567890121234567890
123456789012345678901234567890121234567890123456789012345678901212
12345678901234567890123456789012123456789012345678901234567890121234567890123456789012345678901212345678901234567890123456789012123
12345678901234567890123456789012123456789012345678901234567890121234567890123456789012345678901212345678901234567890123
1234567890123456789012345678901212345678901234567890123456789012123456789012345678901234567890121234567890123
1234567890123456789012345678901212345678901234567890123456789012123456789012345678901234567890121234
12345678901234567890123456789012123456789012345678901234567890121234567890123456789012345678901212345678901234567890123456789012123

D
9
123456789012345678901234567890121234567890123456789012345678901212
12345678901234567890123456789012123456789012345678901234567890121234567890123456789012345678901212345678901234567890123
1234567890123456789012345678901212345678901234567890123456789012123456789012345678901234567890121234567890123
1234567890123456789012345678901212345678901234567890123456789012123456789012345678901234567890121234
123456789012345678901234567890121234567890
12345678901234567890123456789012123456789012345678901234567890121234567890123456789012345678901212345678901234567890123456789012123
12345678901234567890123456789012123456789012345678901234567890121234567890123456789012345678901212345678901234567890123
1234567890123456789012345678901212345678901234567890123456789012123456789012345678901234567890121234567890123
1234567890123456789012345678901212345678901234567890123456789012123456789012345678901234567890121234
12345678901234567890123456789012123456789012345678901234567890121234567890123456789012345678901212345678901234567890123456789012123
123456789012345678901234567890121234567890123456789012345678901212

59
12345678901234567890123456789012123456789012345678901234567890121234567890123456789012345678901212345678901234567890123
1234567890123456789012345678901212345678901234567890123456789012123456789012345678901234567890121234567890123
1234567890123456789012345678901212345678901234567890123456789012123456789012345678901234567890121234
12345678901234567890123456789012123456789012345678901234567890121234567890123456789012345678901212345678901234567890123456789012123

CA
95
1. XH› ‡é\© … Õ 5 =°Ok ƒì°~¡ ° ‡…ç¾ x i. ~˂² ì `Ÿ 1) HË~¡°“ 2) ‡éb‹¬° (SI-2011)

50
‹¬O[†Ÿ°ä›½ `Ç䛽ø= `Ç~¡=ò…Õ L<Œß_È°. qHꋹ, k<Í+¹ 3) <Œ¼†Ç°"Œk 4) xOk`Ç°_È°
98
A
H›<Œß Z䛽ø= `Ç~¡=ò…Õ L<Œß_È°. H›=°…˜ †³òH›ø ~Œ¼OH± 4. 5 l…ìÁ…Õ „¦¬`̈́¬î~Ÿ H›<Œß JH›Ä~Ÿ„¬î~Ÿ z#ßk. ‡"£°
:8
~˂²ì`Ÿä›½, qHꋹ䛽 =°^Î¼ H›^ΰ. Z=~¡° Z䛽ø= H›<Œß ^Î<£ƒì^£ ̄^Îík =°i†Çò „¦¬`̈́¬î~Ÿ H›<Œß ƒì~Œ|°H÷
E
h

`Ç~¡=ò…Õ L<Œß~¡°.? ̄^Îík Hê^ΰ Jū^Îí l…ìÁ Uk? (SI-2008)


.P

(SI-2011)
IC

1) k<Í+¹ 2) ‹¬O[†Ÿ° 3) qHꋹ 4) H›=°…˜ 1) ƒì~Œ|OH÷ 2) „¦¬`̈́¬î~Ÿ 3) ª"£° 4) ^Î<£ƒì^£


AD

2. J‚¬ì=°à^£, ‹¬b"£° H›<Œß ‡Ú_È=ôQê L<Œß_È°. ‹¬b"£° 5. Jx`Ç, ‹¬°*ì`Ç H›<Œß ‡Ú_È°Q®° Hêh 䛽=¶i H›<Œß ‡Ú\÷“
AB
O

J‚¬ì=°à^£ H›<Œß ‡Ú_È°Q®° Hê^ΰ Hêx JH›Ä~Ÿ H›<Œß ‡Ú_È"³á# =°i†Çò ‹¬°*ì`Ç, HêOKÇ# ‹¬=¶#"³°Ø# ‡Ú_È=ô Q® "Œ~¡°
ER
CH

Hêh ‹¬°*ì`Ç =x`Ç H›<Œß ‡Ú_È°Q®° J~ò#„¬C_È° HêOKÇ#


"Œ_È° Hê^ΰ. Z=~¡° Z䛽ø= ‡Ú_È"³á#"Œ_È°? (SI-2011)
1) ‹¬b"£° 2) JH›Ä~Ÿ
YD

(Group-I-2012)

3) ªé‚¬ì<£ =°i†Çò JH›Ä~Ÿ 4) J‚¬ìà^£ 1) Jx`Ç H›<Œß ‡Ú\÷“ 2) Jx`Ç`Ë ‹¬=¶#=ò


,H
T

3) 䛽=¶i H›<Œß ‡Ú_È°Q®° 4) =x`Ç H›<Œß ‡Ú\÷“


AM

3. =°~¡}÷Oz# "Œx H›<Œß xO^ΰ`Ç°_È° JkH› ‰×H÷ë Q®"Œ_È°.


6. S^ΰQ®°~¡° ƒì°~¡…Õ M H›O>è V ‡Ú_È=ô Hêh R JO`Ç
H

‡éb‹¬° HË~¡°“ H›<Œß `Ç䛽ø= ‰×H÷ë H›"Œ_È° Hêx <Œ¼†Ç°"Œk


M

‡Ú_È=ô Hê^ΰ. J, D H›O>è ‡Ú_È"Í Hêx M H›O>è ‡Ú\÷“,


AM

H›<Œß Z䛽ø= ‰×H÷ë H›"Œ_È°. xO^ΰ`Ç°_È° ‡éb‹¬° ‹¬=°H›ÆO…Õ


IG

g\÷…Õ J`ǼO`Ç ‡Ú_È"³á# =¼H÷ë? (Group-I-2012)


`Ç# `ǁ#° kOKÇ°HË=…ÿ#°. Z=~¡° Z䛽ø= ‰×H÷ë=°O`Ç°_È°?
KH

1) R 2) M 3) V 4) x~ŒœiOKDžè=ò
R

12345678901234567890123456789012123456789012345678901234567890121234567890123456789012345678901212345678901234567890123
12345678901234567890123456789012123456 1234567890123456789012345678901212345678

q=~¡}ì`ÇàH› ["Œ|°°
12345678901234567890123456789012123456789012345678901234567890121234567890123456789012345678901212345678901234567890123
12345678901234567890123456789012123456 1234567890123456789012345678901212345678
12345678901234567890123456789012123456789012345678901234567890121234567890123456789012345678901212345678901234567890123
12345678901234567890123456789012123456 1234567890123456789012345678901212345678
12345678901234567890123456789012123456789012345678901234567890121234567890123456789012345678901212345678901234567890123
12345678901234567890123456789012123456 1234567890123456789012345678901212345678
12345678901234567890123456789012123456789012345678901234567890121234567890123456789012345678901212345678901234567890123
12345678901234567890123456789012123456 1234567890123456789012345678901212345678
12345678901234567890123456789012123456789012345678901234567890121234567890123456789012345678901212345678901234567890123
12345678901234567890123456789012123456 1234567890123456789012345678901212345678
12345678901234567890123456789012123456789012345678901234567890121234567890123456789012345678901212345678901234567890123
12345678901234567890123456789012123456 1234567890123456789012345678901212345678

1. (2) Wzó# ‹¬=¶KŒ~¡O ㄬHê~¡O 4. (2)


‹¬O[†Ÿ° > ~˂²ì`Ÿ > H›=°…˜ > qHꋹ > k<Í+¹
5. (1) HêOKÇ#, ‹¬°*ì`ǁ° ‹¬=¶#"³°Ø# ‡Ú_È=ô Q®"Œ~¡°.
∴ Z䛽ø= `Ç~¡O…Õ ‹¬O[†Ÿ° L<Œß_È°.
2. (4) ‹¬°*ì`Ç, Jx`Ç H›<Œß ‡Ú\÷“, Hê=ô# HêOKÇ# 䛀_¨
3. (1) Wzó# ‹¬=¶KŒ~¡O ㄬHê~¡O (‰×H÷ë q+¬†Ç°O…Õ) Jx`Ç H›<Œß ‡Ú\÷“.
HË~¡°“ > ‡éb‹¹ > <Œ¼†Ç°"Œk > xOk`Ç°_È° > 6. (1) R>V>M>J>D

=°~¡}÷Oz# "Œ_È° ̄á #°O_ J`ǼO`Ç ‡Ú_È"³á# =¼H÷ë « R

∴ J`ǼO`Ç ‰×H÷ë=O`Ç"³°Ø# "Œ_È° 'HË~¡°“Ñ.

REASONING ABILITY (VERBAL) ○ ○ ○ ○ ○ ○ ○ ○ ○


114
○ ○ ○ ○ ○ ○ ○ ○ ○ ○ ○ ○ ○ ○ ○ ○ ○ ○ ○
Study Material
‹Ô\÷OQ· / À„Á‹²OQ· Jö~*˜"³°O\˜ž
9 (Seating / Placing Arrangements)

=òMì¼O‰§°...
G D qƒìOQ®O…Õx ㄬ‰×߁…Õ Hùxß =‹¬°ë=ô#° Hêh …è^¥ =¼ä›½ë#° Hêh XH› =~¡°‹¬…Õ …è^¥ XH› =$`ŒëHê~¡O…Õ …è^¥ >è|°…˜
KÇ°@¶“....... 䛀~Ëóƒÿ@“_ÈO …è^¥ xƒÿ@“_ÈO [~¡°Q®°`Ç°Ok. J~ò# "Œix P†Ç¶ =~¡°‹¬…Õ Z…ì J=°~Œó…Õ Hùxß ‹¬¶KÇ#°
ㄬ‰×ß…Õ W=Þ|_È`Œ~ò. "Œ\÷x P^¥~¡OQê K͋¬°ä›½x Wzó# "Œi ªÖ<Œ#° Q®°iOz J_y# ㄬ‰×߁䛽 ‹¬ï~á# ["Œ|°°
Q®°iëOKŒe.

Y
M
=¶ki ㄬ‰×߁°

E
D
9
59
MODEL - 1
CA
K¢±~ 

95
=$`ŒëHê~¡O…Õ ‹²\÷“OQ· Jï~O*˜"³°O\˜

50
䊨>~ø u>

1. #°Q®°~¡° „¬ô~°¡ +¬µ° A, B, C, D, ° #°Q®°~¡° =°‚²ìˆ×°


98
A
P, Q, R, S ° =$`ŒëHê~¡O…Õ 䛀~ùóx L<Œß~¡°. "Œi…Õ
:8

U W^Îí~¡° =°‚²ìˆ×° …è^¥ U W^Îí~¡° „¬ô~¡°+¬µ° ㄬH›ø


¢r 圾>· 
E
h

ㄬH›ø<Í …è~¡°. P J#° =°‚²ìˆ× B #䛽 䛽_"³á„¬ô# H›^ΰ.


.P

LÈ¢~ 
IC

R J#° =°‚²ìˆ×, Q J#° =°‚²ìˆ×䛽 Z^ΰ~¡°Qê H›^ΰ


Z) ~¡q䛽 䛽_"³á„¬ô# KÇ~¡}Ÿ H›_È°.
AD

=°i†Çò A J#° „¬ô~¡°+¬µxH÷ Z_È=°"³á„¬ô# H›^ΰ. C a) t=䛽 Z_È=°"³á„¬ô# ï~O_È=ªÖ#O…Õ KÇ~¡}Ÿ H›_È°.
AB
O

J#° =¼H÷ë S J#° =°‚²ìˆ×䛽 䛽_"³á„¬ô# H›^ΰ. J~ò<Œ ‹²) KÇ~¡}Ÿä›½ 䛽_"³á„¬ô# 䛽=¶~Ÿ, t= =ô<Œß~¡°.
ER
CH

‹¬¶KÇ#° (3-5) : U_È°Q®°~¡° ã̄¦O_£ž ªöH`Ÿ, Jx`Ç, À‹ß‚¬ì,


D J#° =¼H÷ëH÷ W~¡°"³á„¬ô…ì Z=ï~=~¡° H›~¡°?
YD

t=, 䛽=¶~Ÿ, H©;, ~¡q° XH› =$`ŒëHê~¡ |Á KÇ°@¶“


ª^Î#: "
,H

4 3 䛀~ùóxi.
T

䛽=¶~Ÿ XH›ø_È° `Ç„æ¬ q°ye# "Œ~¡° ‹¬=¶# ^ζ~¡O…Õ H›~¡°.


AM

i)
$ %
H

ii) Jx`Ç, ~¡qH÷ Z^ΰ~¡°Qê H›^ΰ =°i†Çò 䛽=¶~ŸH÷ Z_È=°


M

ㄬH›ø<Í H›^ΰ.
AM

2 1
IG

# iii) t=䛽 䛽_ ㄬH›ø<Í H©; H›^ΰ.


KH

zã`ÇO ㄬHê~¡O, D J#° =¼H÷ëH÷ W~¡°"³á„¬ô…ì iv) H©;䛽 䛽_ „¬H›ø<Í =ü_È= ªÖ#O…Õ À‹ß‚¬ì H›^ΰ.
P, R J#° =°‚²ìˆ×° LO\ì~¡°.
R

v) ªöH`Ÿ, 䛽=¶~Ÿ #°O_ 90°, Jx`Ç #°O_ 120° HË}O


2. P~¡°Q®°~¡° =¼ä›½ë° =$`ŒëHê~¡O…Õ 䛀~¡°óx =ô<Œß~¡°. K͆Çò#°.
"Œi…Õ KÇ~¡}Ÿ J#° =¼H÷ë ~¡q, 䛽=¶~Ÿ =°^Î¼…Õ H›_È°. Ì„á ‹¬=¶KŒ~¡O P^¥~¡OQê ãH÷Ok ㄬ‰ß
× ä›½ ["Œ|°° W=ÞO_.

J~¡°}Ÿ J#° =¼H÷ë ㉧=}÷, t= =°^¼Î …Õ H›^ΰ. ㉧=}÷, ¢r «‘÷Ia 
~¡qH÷ Z_È=° ㄬH›ø# H›^ΰ. J~ò#
Z) ~¡q䛽 䛽_"³á„¬ô# Z=~¡°H›~¡° ?
60°
u> 60° 60°
ÁŒàƒ­í
a) t=䛽 Z_È=°"³„á ô¬ # ~ï O_È=ªÖ#O…Õ Z=~¡° =ô<Œß~¡°?
30°
60°
60° 30° 圾>· 
‹²) KÇ~¡}Ÿä›½ 䛽_"³á„¬ô# Q® W^Îí~¡° Z=~¡° ? ‚Iª<
KyaÈ
ª^Î#: Wzó# ‹¬¶KÇ# ㄬHê~¡O "Œ~¡O^Î~¡° ㄬH›øzã`ÇO…Õ 3. Jx`Ç =°i†Çò 䛽=¶~Ÿ =°^Î¼ HË}O ZO`Ç ?
KǶ„²#@°Á 䛀~¡°óx =ôO\ì~¡°. „¬@O #°O_, ª^Î#: 30°.
REASONING ABILITY (VERBAL) ○ ○ ○ ○ ○ ○ ○ ○ ○
115
○ ○ ○ ○ ○ ○ ○ ○ ○ ○ ○ ○ ○ ○ ○ ○ ○ ○ ○
Study Material
4. Jx`Ç, À‹ß‚¬ì =°^Î¼ Z=~¡° H›~¡° ? 6. ̄á ㄬ‰×ß#° ªkOKÇ°@䛽 W=Þ|_# ‹¬=¶KŒ~¡O…Õ
ª^Î#: 䛽=¶~Ÿ. J=‹¬~¡O …èxk Uk ?
5. ~¡q #°O_ KǶÀ‹ë ‹¬=¼k‰×…Õ 䛽=¶~Ÿ K͆Çò HË}O ª^Î#: Jxß q=~¡}° J=‹¬~¡O Jq#"Í.
ZO`Ç ?
ª^Î#: (60° + 60° + 30° = 150°) MODEL - 3
MODEL - 2 n~¡É KÇ`Ç°~¡ãªHê~¡ (…è^¥) KÇ`Ç°~¡ãªHê~¡
=~¡°‹¬ãH›=°O…Õ ‹²\÷“OQ· Jï~O*˜"³°O\˜ ‹²\÷“OQ· Jï~O*˜"³°O\˜
1. 5Q®°~¡° =¼ä›½ë° XH› =~¡°‹¬…Õ 䛀~¡°ó<Œß~¡°. "Œi…Õ 1. #°Q®°~¡° À‹ß‚²ì`Ç°° XH› KÇ`Ç°~¡ãªHê~¡ >è|°…˜ KÇ°@¶“
XH› z=~¡ `³eq Q® =¼H÷ë =°~ùH› z=~¡ JO^Î"³°Ø# 䛀~¡°óx =ô<Œß~¡°. =°h+¹ J#°"Œ_È° ª‚²ì…˜ä½› 䛽_"„ á³ ô¬ #
=¼H÷ë H›~¡°. ‹¬#ß\÷ =¼H÷ëH÷ 䛽_"³á„¬ô# …ì=ô‡\÷ =¼H÷ë =ô<Œß_È°.‹¬°Ì‚á션 J#°"Œ_È° H›$+¬äâ ½› Z_È=°"³„
á ô¬ # 䛀~¡°óx

Y
H›~¡°. ‡Ú_È"Œ\÷ =¼H÷ë JO^Î"³°Ø# =¼H÷ëH÷ Z_È=°"³á„¬ô =ô<Œß_È°, H›$+¬â J#°"Œ_È° ª‚²ì…˜ä½› Z_È=°"³„ á ô¬ # 䛀~¡°óx

M
H›_È°. ‹¬#ß\÷ =¼H÷ë `³eqQ®, …ì=ô‡\÷ =¼ä›½ë =°^Î¼ =ô<Œß_È°. J~ò#
H›_È°. J~ò<Œ, "Œi ªÖ<Œ° U =~¡°‹¬ ãH›=°O…Õ Z) ‹¬°Ì‚á션䛽 W~¡°"³á„¬ô…ì =ô#ß"Œ~¡° Z=~¡° ?

E
L<Œß~ò? a) ª‚²ì…˜, ‹¬°Ì‚á션 =°^Î¼…Õ =ô#ßk Z=~¡° ?
`³eqQ® =¼H÷ë ‹¬#ßx =¼H÷ë …ì=ô‡\÷ =¼H÷ë

D
9
ª^Î#: ‹²) H›$+¬â䛽 Z^ΰ~¡°Qê =ô#ßk Z=~¡° ?

59
‹¬°Ì‚á션
CA
95
‡Ú_È"Œ\÷ =¼H÷ë JO^Î"³°Ø# =¼H÷ë

50
H›$+¬â =°h+¹
98
A
‹¬¶KÇ#° (2-6) : P, Q, R, X, Y, Z =°i†Çò A°
:8
`Ƕ~¡°æ#䛽 Ja=òYOQê XH› QË_È̄á 䛀~ùóxi.
E
h

X H÷ 䛽_ „¬H›ø<Í R H›_È°. ª‚²ì…˜


.P

i)
IC

Q =~¡°‹¬…Õ XH› z=~¡# H›_È° =°i†Çò J`Ç_ „¬H›ø# Y ª^Î#: Wzó# ‹¬¶KÇ# ㄬHê~¡O "Œ~¡O^Î~¡° ̄ᄬ@O…Õ
AD

ii)

H›_È°. KǶ„²#@°Á 䛀~¡°óx =ôO\ì~¡°. „¬@#O #°O_,


AB
O

iii) Y =°i†Çò Z  =°^Î¼…Õ A LO_È°#°.


Z) ‹¬°Ì‚á션䛽 W~¡°"³á„¬ô…ì H›$+¬â, =°h+¹ =ô<Œß~¡°.
ER

a) ª‚²ì…˜, ‹¬°Ì‚á션䛽 =°^Î¼…Õ XH› "³á„¬ô H›$+¬â, =°~ùH›


CH

iv) ^ÎH÷Æ} z=~¡ #°O_ X =ü_È= ªÖ#O…Õ H›_È°.


"³á„¬ô# =°h+¹ =ô<Œß~¡°.
YD

Ì„á ‹¬ = ¶KŒ~¡ O P^ ¥ ~¡ O Qê ãH÷ O k W=Þ|_ # ㄬ ‰ × ß ä› ½

‹²) H›$+¬â䛽 Z^ΰ~¡°Qê =°h+¹ L<Œß_È°.


,H

‹¬=¶^¥<Œ° W=ÞO_.
T

ª^Î#: (2-6) W=Þ|_# ‹¬=¶KŒ~¡O P^¥~¡OQê J=°iH›K͋²#


AM

‹¬¶KÇ#° (2-4) : XH› n~¡É KÇ`Ç°~¡ãªHê~¡ |Á †³òH›ø


®
D ãH÷Ok q^ÎOQê LO_È°#°.
H

Q ‡Ú_È"Œ\÷ JOKÇ°…Õ A, B, C, D, E, F ° JOKÇ°H÷ =òQ®°~


¾ °¡
M

Y ®
AM

Kù„¬C# H›~¡°.
IG

A ®
® `Ƕ~¡°æ F H÷ Z_È=° „¬H›ø# 2 ªÖ#O…Õ D H›_È°.
KH

i)
Z
® B, F ° „¬H›ø<Í LO_È~¡°.
R

X ii)

R ® iii) E U z=~¡# LO_È^ΰ. " & $


P ® iv) E „¬H›ø<Í LO_È° 'C' J<Í ƒìeH› D H÷
X X X

2. L`Çë~¡ z=~¡ #°O_ =ü_È= ªÖ#O…Õ Z=~¡° LO_È°#°? H›~¡â=üO…Õ Z^ΰ~¡°Qê LO_È°#°.
ª^Î#: L`Çë~¡ z=i #°O_ =ü_È= ªÖ#O…Õ H›_È°.
'A' X
%
X
#
X
'
3. X Z=i =°^Î¼ H›_È° ? 2. E Z=i =°^Î¼ H›_È° ?
ª^Î#: 'X' J#°"Œ_È° =°i†Çò =°^Î¼# H›_È°.
Z R ª^Î#: A & C.
4. Y H÷ 䛽_ „¬H›ø# Z=~¡° H›~¡° ? 3. F H÷ H›~¡â=üO…Õ Z^ΰ~¡°Qê Z=~¡° H›~¡° ?
ª^Î#: 'Y' H÷ 䛽_ „¬H›ø# H›_È°.
'A' ª^Î#: A
5. W~¡°z=~¡…ÕÁ LOk Z=~¡° ? 4. A H÷ Z^ΰ~¡°Qê Z=~¡° H›~¡° ?
ª^Î#: W~¡° z=~¡Á…Õ =°i†Çò ° H›~¡°.
Q P ª^Î#: D
REASONING ABILITY (VERBAL) ○ ○ ○ ○ ○ ○ ○ ○ ○
116
○ ○ ○ ○ ○ ○ ○ ○ ○ ○ ○ ○ ○ ○ ○ ○ ○ ○ ○
Study Material
MODEL - 4 l#ν,º K{=¤
+¬_Ȱĝ*ìHê~¡ >è|°…˜ KÇ°@¶“ ‹²\÷“OQ· Jï~O*˜"³°O\˜
1. P~¡°Q®°~¡° À‹ß‚²ì`Ç°° =¶†Ç¶OH±, Jz<£, kyÞ[†Ÿ°, >±‡È·PI² yv=¤
‹¬z<£ k"ͼ+¹ =°i†Çò xu<£° XH› +¬_Ȱĝ*ìHê~¡ >è|°…˜
KÇ°@¶“ 䛀~¡°ó<Œß~¡°. k"ÍÞ+¹ä›½ Z_È=°"³á„¬ô# <Œ¾="Œ_È°
Œ­{=¤ lzß\‡ ±
‹¬z<£, Jz<£ J#°"Œ_È° xu<£ =°i†Çò k"ͼ+¹ä›½
=°^Î¼# =ô<Œß_È°. k"ͼ+¹ =°i†Çò ‹¬z<£° XH›iHùH›~¡° ª^Î#: Wzó# ‹¬¶KÇ# ã„H¬ ê~¡O "Œ~¡O^Î~°¡ ã„H¬ ø› „¬@O…Õ KǶ„²#
Z^ΰ~¡°Qê …è~°¡ . =°†Ç¶OH± =°i†Çò xu<£° XH›iHùH›~°¡ q^ÎOQê 䛀~¡°óx =ôO\ì~¡°. „¬@#O #°O_
Z^ΰï~^ΰ~¡°Qê =ô<Œß~¡°. J~ò# Z) ‹¬z<£ä›½ 䛽_ ㄬH›ø<Í Q® "³ò^Î\ ÷ =¼H÷ë kyÞ[†Ÿ°
Z) ‹¬z<£ä›½ 䛽_ã„¬H›ø<Í Q® "³ò^Î\ ÷ =¼H÷ë Z=~¡° ? a) kyÞ[†Ÿ°ä›½ W~¡°"³„á ô¬ …ì ‹¬z<£ =°i†Çò xu<£ =ô<Œß~¡°

Y
a) kyÞ[†Ÿ°ä›½ W~¡°"³á„¬ô…ì Q® "Œ~¡° Z=~¡° ? ‹²) k"ͼ+¹ä›½ Z^ΰ~¡°Qê 䛀~¡°ó#ß =¼H÷ë kyÞ[†Ÿ°
_) Jz<£ä›½ 䛽_"³á„¬ô# ï~O_È==¼H÷ë =°†Ç¶OH±

M
‹²) k"ÍÞ+¹ä›½ Z^ΰ~¡°Qê =ô#ß=¼H÷ë Z=~¡° ?
_) Jz<£ä›½ 䛽_"³á„¬ô ï~O_È==¼H÷ë Z=~¡° ?

E
123456789012345678901234567890121234567890123456789012345678901212
12345678901234567890123456789012123456789012345678901234567890121234567890123456789012345678901212345678901234567890123456789012123
1234567890123456789012345678901212345678901234567890123456789012123456789012345678901234567890
12345678901234567890123456789012123456789012345678901234567890121234567890123456789012345678901212345678901234567890123
1234567890123456789012345678901212345678901234567890123456789012123456789012345678901234567890121234567890123
1234567890123456789012345678901212345678901234567890123456789012123456789012345678901234567890121234
123456789012345678901234567890121234567890123456789012345678901212
12345678901234567890123456789012123456789012345678901234567890121234567890123456789012345678901212345678901234567890123456789012123
1234567890123456789012345678901212345678901234567890123456789012123456789012345678901234567890
12345678901234567890123456789012123456789012345678901234567890121234567890123456789012345678901212345678901234567890123
1234567890123456789012345678901212345678901234567890123456789012123456789012345678901234567890121234567890123
1234567890123456789012345678901212345678901234567890123456789012123456789012345678901234567890121234
12345678901234567890123456789012123456789012345678901234567890121234567890123456789012345678901212345678901234567890123456789012123
1234567890123456789012345678901212345678901234567890123456789012123456789012345678901234567890

D
9
123456789012345678901234567890121234567890123456789012345678901212
12345678901234567890123456789012123456789012345678901234567890121234567890123456789012345678901212345678901234567890123
1234567890123456789012345678901212345678901234567890123456789012123456789012345678901234567890121234567890123
1234567890123456789012345678901212345678901234567890123456789012123456789012345678901234567890121234
PRACTICE TEST - 1
12345678901234567890123456789012123456789012345678901234567890121234567890123456789012345678901212345678901234567890123456789012123
1234567890123456789012345678901212345678901234567890123456789012123456789012345678901234567890
12345678901234567890123456789012123456789012345678901234567890121234567890123456789012345678901212345678901234567890123
1234567890123456789012345678901212345678901234567890123456789012123456789012345678901234567890121234567890123
1234567890123456789012345678901212345678901234567890123456789012123456789012345678901234567890121234
12345678901234567890123456789012123456789012345678901234567890121234567890123456789012345678901212345678901234567890123456789012123
1234567890123456789012345678901212345678901234567890123456789012123456789012345678901234567890
123456789012345678901234567890121234567890123456789012345678901212

59
12345678901234567890123456789012123456789012345678901234567890121234567890123456789012345678901212345678901234567890123
1234567890123456789012345678901212345678901234567890123456789012123456789012345678901234567890121234567890123
1234567890123456789012345678901212345678901234567890123456789012123456789012345678901234567890121234
12345678901234567890123456789012123456789012345678901234567890121234567890123456789012345678901212345678901234567890123456789012123
1234567890123456789012345678901212345678901234567890123456789012123456789012345678901234567890
123456789012345678901234567890121234567890123456789012345678901212
12345678901234567890123456789012123456789012345678901234567890121234567890123456789012345678901212345678901234567890123456789012123
1234567890123456789012345678901212345678901234567890123456789012123456789012345678901234567890
12345678901234567890123456789012123456789012345678901234567890121234567890123456789012345678901212345678901234567890123
1234567890123456789012345678901212345678901234567890123456789012123456789012345678901234567890121234567890123
1234567890123456789012345678901212345678901234567890123456789012123456789012345678901234567890121234
12345678901234567890123456789012123456789012345678901234567890121234567890123456789012345678901212345678901234567890123456789012123
1234567890123456789012345678901212345678901234567890123456789012123456789012345678901234567890

CA
95
. (1 - 7) P~¡°Q®~¡° =¼ä›½ë° =$`ŒëHê~¡O…Õ Z^ΰï~^ΰ~¡°Qê . (8 - 12) A,B,C,D,E,F,G,H J#° =¼ä›½
II ë ° XH› =~¡°‹¬…Õ

50
I

䛀~¡°ó<Œß~¡°. J[†Ÿ° À‚ì=°O`Ÿ#䛽 Z^ΰ~¡°Qê H›_È°. L`Ç~ë ¡ k䛽ø#° KǶ‹¬¶ë x°ó<Œß~¡°. B,G #䛽 U ㄬHø› ##¶
98
A
À‚ì=°O`Ÿ J~¡qO^£#䛽 Z_È=°"³„á ô¬ ‹¬O[†Ÿ°#䛽 䛽_"„
á³ ô¬ # …è_È°. F,G #䛽 䛽_"³á„¬ô ㄬH›ø<Í H›_È° =°i†Çò E #䛽
:8
H›_È°. ‹¬°=°<£ J~¡qO^£#䛽 䛽_"³á„¬ô# H›_È°. ㄬH›ø<Í H›_È°. G J#° =¼H÷ë U z=~¡#¶ …è_È°. A J#°
E
h
.P

(Ì„á ‹¬=¶KŒ~¡O P^¥~¡OQê ‹¬=¶^¥<Œe=òà) =¼H÷ë E #䛽 Z_È=°"³á„¬ô #°O_ S^Ë ªÖ#O…Õ H›_È°. H
IC

J#° =¼H÷ë C #䛽 䛽_"³á„¬ô #°O_ ï~O_Ë ªÖ#O…Õ H›_È°.


AD

1. J~¡qO^£ä›½ Z^ΰ~¡°Qê 䛀~¡°ó#ß =¼H÷ë Z=~¡° ?


1) =°<Ë*˜ 2) À‚ì=°O`Ÿ 3) ‹¬O[†Ÿ° 4) J[†Ÿ° (8 #°O_ 12 =~¡ä›½ Q® ㄬ‰×߁䛽 Ì„á ‹¬=¶KŒ~¡O
AB
O

2. J[†Ÿ° =°i†Çò ‹¬O[†Ÿ°ä›½ =°^Î¼…Õ 䛀~¡°ó#ß =¼H÷ë P^¥~¡OQê ‹¬=¶^¥<Œe=òà)


ER
CH

Z=~¡° ? 8. D ãH÷Ok"Œx…Õ ㄬH›ø ㄬH›ø<Í Q® =¼ä›½ë° Z=~¡° ?


1) C, A 2) F, H 3) C, G 4) A, B
YD

1) J~¡qO^£ 2) À‚ì=°O`Ÿ 3) ‹¬°=°<£ 4) =°<Ë*˜


,H

3. ‹¬O[†Ÿ°ä›½ Z^ΰ~¡°Qê 䛀~¡°ó#ß =¼H÷ë Z=~¡° ? 9. D †³òH›ø ªÖ#O


1) G 䛽 ㄬH›ø<Í =ü_È=ªÖ#O
T

1) À‚ì=°O`Ÿ 2) J~¡qO^£ 3) ‹¬°=°<£ 4) =°<Ë*˜


AM



2) A 䛽 ㄬH›ø# ï~O_È=ªÖ#O
H

4. À‚ì=°O`Ÿ =°<Ë*˜ =°i†Çò J~¡qO^£ ‹¬O[†Ÿ°°


M

3) B 䛽 ㄬH›ø# ï~O_È=ªÖ#O


AM

`Ç=° ªÖ<Œ#° „¬~‹


¡ æ¬ ~¡O =¶~¡°óHùx# =°<Ë*˜ä½› 䛽_"„
á³ ô¬ #
IG

Q® =¼H÷ë Z=~¡° ? 4) F 䛽 ㄬH›ø# "³ò^Î\ ÷ ªÖ#O


KH

1) À‚ì=°O`Ÿ 2) ‹¬O[†Ÿ° 3) ‹¬°=°<£ 4) J~¡qO^£ 10. G 䛽 䛽_ ㄬH›ø<Í Q® =¼H÷ë
R

5. J~¡qO^£

=°<Ë*˜° =¶ã`Ç"°Í `Ç=° ªÖ<Œ#° „¬~‹ ¡ æ¬ ~¡O 1) E 2) A 3) C 4) F


=¶~¡°óHùx# J~¡qO^£ä½› W~¡°"³„
á ô¬ …ì Q® =¼ä›½
ë ° Z=~¡° ? 11. ï~O_È°"³á„¬ô…ì z=~¡# H›"Œï~=~¡° ?
1) ‹¬°=°<£ Ê À‚ì=°O`Ÿ 2) ‹¬O[†Ÿ° Ê À‚ì=°O`Ÿ 1) A & H 2) C & E 3) C & B 4) E & B
3) J[†Ÿ° Ê ‹¬O[†Ÿ° 4) J[†Ÿ° Ê ‹¬°=°<£ 12. D ãH÷Ok "Œx…Õ ㄬH›øㄬH›ø<Í …èx =¼ä›½ë° Z=~¡° ?
1) H & D 2) F & B 3) E & F 4) C & A
6. J[†Ÿ°

À‚ì=°O`Ÿ° =¶ã`Ç"°Í `Ç=° ªÖ<Œ#° „¬~‹ ¡ æ¬ ~¡O


III . (13 - 16) P~¡°, Q®$‚¬ð° XöH =~¡°‹¬…Õ H›=ô. …ì=}¼
=¶~¡°óHùx# J[†Ÿ°ä›½ 䛽_"„
á³ ô¬ # Q® ~ï O_È= =¼H÷ë Z=~¡° ?
1) ‹¬°=°<£ 2) ‹¬O[†Ÿ° 3) =°<Ë*˜ 4) J~¡qO^£ Q®$‚¬ì=ò#䛽 W~¡°"³„á ô¬ …ì ƒì=#, ‹¬`¼Ç  Q®$‚¬ì=ò°#ßq.
…심¼ Q®$‚¬ì=ò#䛽 W~¡°"³á„¬ô…ì w`Ç, ㉧=¼ Q®$‚¬ì=ô°#ßq.
7. JO^Î~¶
¡ "Œi"Œi J‹¬° ªÖ<Œ…Õ<Í =ô#ß@Á~ò`Í ‹¬°=°<£
w`Ç Q®$‚¬ì=ò ƒì=#, ‹¬`¼Ç  †³òH›ø Q®$‚¬ì=òä›½ ‡Ú~¡°Q®°#°
=°i†Çò =°<Ë*˜ä›½ =°^Î¼# =ô#ßk Z=~¡° ?
…è^ΰ. ㉧=¼ Q®$‚¬ì=ò ‹¬`Ǽ Q®$‚¬ì=ò#䛽 ㄬH›ø# …è^ΰ.
1) J~¡qO^£ 2) ‹¬O[†Ÿ° 3) À‚ì=°O`Ÿ 4) J[†Ÿ°
REASONING ABILITY (VERBAL) ○ ○ ○ ○ ○ ○ ○ ○ ○
117
○ ○ ○ ○ ○ ○ ○ ○ ○ ○ ○ ○ ○ ○ ○ ○ ○ ○ ○
Study Material
(13 #°O_ 16 =~¡ä›½ Q® ㄬ‰×߁䛽 Ì„á ‹¬=¶KŒ~¡O IV. (17 - 20) 5 Q®°~¡° =°‚²ìˆ×° XH› =~¡°‹¬…Õ x|_
P^¥~¡OQê ‹¬=¶^¥<Œe=òà) =ô<Œß~¡°. H›æ#, =°$^ΰä›½ Z_È=°"³„á ô¬ # H›^ΰ. =°OA
13. ƒì=# Q®$‚¬ì=ò#䛽 ‡Ú~¡°Q®°# Z=i Q®$‚¬ì=ò° H›=ô ? J~¡ó#䛽 䛽_"³á„¬ô# H›^ΰ. ‹¬Ow`Ç =°OAä›½ =°i†Çò
1) ㉧=¼ =°i†Çò …심¼ 2) …심¼ =°i†Çò w`Ç H›æ#䛽 =°^Î¼…Õ H›^ΰ.
3) ‹¬`Ǽ =°i†Çò …심¼ 4) …ì=}¼ =°i†Çò ㉧=¼ (17 #°O_ 20 =~¡ä›½ Q® ㄬ‰×߁䛽 Ì„á ‹¬=¶KŒ~¡O #°O_
14. P =~¡°‹¬…Õ =°^Î¼…Õ =ô#ß Q®$‚¬ìO/ Q®$‚¬ð° Z=iq ? ‹¬=¶^¥<Œe=òà)
1) ㉧=¼ =°i†Çò …심¼ 2) …ì=}¼ =°i†Çò ƒì=# 17. P =~¡°‹¬…Õ JO^Îi…ÕH÷ =°^Î¼# =ô#ßk Z=~¡° ?
3) ƒì=# =°i†Çò ㉧=¼ 4) ‹¬`Ǽ =°i†Çò w`Ç 1) J~¡ó# 2) =°$^ΰ 3) ‹¬Ow`Ç 4) =°OA
15. P =~¡°‹¬…Õ 䛽_ =°i†Çò Z_È=° z=~¡ Q® Q®$‚¬ð° 18. P =~¡°‹¬…Õ Z_È=°z=~¡ =ô#ßk Z=~¡° ?
Z=iq ? 1) =°$^ΰ 2) H›æ# 3) ‹¬Ow`Ç 4) J~¡ó#
1) ‹¬`Ǽ =°i†Çò w`Ç 2) …ì=}¼ =°i†Çò …심¼ 19. P =~¡°‹¬…Õ 䛽_ #°O_ Z_È=°ä›½ <Œ¾= ªÖ#O…Õ =ô#ßk
3) ‹¬`Ǽ =°i†Çò …심¼ 4) ㉧=¼ =°i†Çò w`Ç

Y
Z=~¡° ?
1) J~¡ó# 2) =°OA 3) =°$^ΰ 4) ‹¬Ow`Ç

M
16. ‹¬`Ǽ =°i†Çò ƒì=# †³òH›ø Q®$‚¬ð =°^Î¼# =ô#ß Q®$‚¬ìO
Z=ik ? 20. P =~¡°‹¬…Õ 䛽_z=~¡ =ô#ßk Z=~¡° ?
1) ㉧=¼ 2) …심¼ 3) …ì=}¼ 4) w`Ç 1) =°OA 2) H›æ# 3) =°$^ΰ 4) ‹¬Ow`Ç

E
D
9
1234567890123456789012
12345678901234567890123456789012123456789012345678901234567890121234567890123456789012345678901212345678901234567890123
12345678901234567890123

q=~¡}ì`ÇàH› ["Œ|°°
12345678901234567890123456789012123456789012345678901234567890121234567890123456789012345678901212345678901234567890123
1234567890123456789012 12345678901234567890123

59
1234567890123456789012
12345678901234567890123456789012123456789012345678901234567890121234567890123456789012345678901212345678901234567890123
1234567890123456789012 12345678901234567890123
12345678901234567890123456789012123456789012345678901234567890121234567890123456789012345678901212345678901234567890123
12345678901234567890123
1234567890123456789012 PRACTICE TEST - 1 :
12345678901234567890123456789012123456789012345678901234567890121234567890123456789012345678901212345678901234567890123
12345678901234567890123
1234567890123456789012
CA
12345678901234567890123
12345678901234567890123456789012123456789012345678901234567890121234567890123456789012345678901212345678901234567890123

95
1234567890123456789012 12345678901234567890123
12345678901234567890123456789012123456789012345678901234567890121234567890123456789012345678901212345678901234567890123

„¬@O #°O_, 6. J[†Ÿ° "Í°=°O`Ÿ° =¶ã`Ç"Í° `Ç=° ªÖ<Œ#° „¬~¡‹¬æ~¡O


50
I. (1 - 7) : K¢rP_¤ (1)

=¶~¡°óHù#Qê U~¡æ_# „¬@O ㄬH›ø# KǶ„¬|_Ok.


98
A
Œ­±>±=¤ Áƒí>±Pa 
:8
K¢rP_¤
E
h

Œ­±>±=¤ K\‡ ±
.P

2 1
IC

K\‡ ± Œ­P\‡ ±
AD

>±=Ì+™
AB
O

1. J~¡qO^£ Z^ΰ~¡°Qê 䛀~¡°ó#ß =¼H÷ë =°<Ë*˜ Œ­P\‡ ±


J[†Ÿ°ä›½ 䛽_"³á„¬ô# Áƒí>±Pa  >±=Ì+™
(1)
ER
CH

2. (4) J[†Ÿ° =°i†Çò ‹¬O[†Ÿ°ä›½ =°^Î¼…Õ 䛀~¡°ó#ß =¼H÷ë


=°<Ë*˜ ï~O_È= =¼H÷ë ‹¬°=°<£.
YD

3. ‹¬O[†Ÿ°ä›½ Z^ΰ~¡°Qê 䛀~¡°ó#ß =¼H÷ë ‹¬°=°<£ 7. JO^Î~¡¶ "Œi "Œi ªÖ<Œ…Õ =ô#ß, ‹¬°=°<£ =°i†Çò
,H

(3) (4)

4. À‚ì=°O`Ÿ =°<Ë*˜ =°i†Çò J~¡qO^£ ‹¬O[†Ÿ°° =°<Ë*˜ä›½ =°^Î¼# =ô#ß =¼H÷ë J[†Ÿ°.
T
AM

(2)

„¬~¡‹¬æ~¡O `Ç=° ªÖ<Œ#° =¶~¡°óHù#Qê U~¡æ_ „¬@O . (8 - 12) : „¬@O #°O_,


H
M

II

D ãH÷O^Î KǶ„¬|_#k.
AM

Œ­P\‡ ±
C A H D G F E B
IG

>±=Ì+™
Œ­±>±=¤
KH

8. C & A ° ㄬH›ø ㄬH›ø<Í Q® =¼ä›½ë°


R

(1)

„¬@O #°O_ =°<Ë*˜ä›½ K\‡ ± Áƒí>±Pa  K¢rP_¤ 9. (2) D †³òH›ø ªÖ#O A 䛽 ㄬH›ø<Í ï~O_È= ªÖ#O
䛽_"³á„¬ô# Q® =¼H÷ë ‹¬O[†Ÿ° 10. (4) G 䛽 䛽_ ㄬH›ø<Í F =ô<Œß_È°
5. J~¡qO^£ =°<Ë*˜° =¶ã`Ç"Í° `Ç=°ªÖ<Œ#° „¬~¡‹¬æ~¡O 11.
(3)
(3) C & B ° ï~O_È° "³á„¬ô…ì z=~¡ Q® =¼ä›½ë°
=¶~¡°óHù#Qê U~¡æ_# „¬@O D ãH÷O^Î KǶ„¬|_#k. 12. (2) F =°i†Çò B ° ㄬH›øㄬH›ø<Í …è~¡°.
>±=Ì+™ III . (13 - 16) :
Œ­±>±=¤ Áƒí>±Pa  13. (4) …ì=}¼ Ê ã‰§=¼
„¬@O #°O_, J~¡qO^£ä›½ 14. (2) ƒì=# Ê ã‰§=¼
W~¡°"³á„¬ô…ì Q® =¼ä›½ë° 15. (1) ‹¬`Ǽ Ê w`Ç
J[†Ÿ° =°i†Çò ‹¬O[†Ÿ°. K\‡ ±
K¢rP_¤
Œ­P\‡ ±
16. (3) …ì=}¼ ‹¬`¼Ç …ì=}¼ ƒì=# ㉧=¼ …심¼ w`Ç

REASONING ABILITY (VERBAL) ○ ○ ○ ○ ○ ○ ○ ○ ○


118
○ ○ ○ ○ ○ ○ ○ ○ ○ ○ ○ ○ ○ ○ ○ ○ ○ ○ ○
Study Material
IV . (17 - 20) : J~¡ó# =°OA ‹¬Ow`Ç H›æ# =°$^ΰ
17. (3) ‹¬Ow`Ç 19. (2) =°OA
18. (4) J~¡ó# 20. (3) =°$^ΰ (Z_È=°z=~¡) (=°^Î¼#) (䛽_z=~¡)
123456789012345678901234567890121234567890123456789012345678901212
1234567890123456789012345678901212345678901234567890123456789012123456789012345678901234567890121234567890123
1234567890123456789012345678901212345678901234567890123456789012123456789012345678901234567890121234
12345678901234567890123456789012123456789012345678901234567890121234567890123456789012345678901212345678901234567890123456789012123
1234567890123456789012345678901212345678901234567890123456789012123456789012345678901234567890
123456789012345678901234567890121234567890123456789012345678901212
12345678901234567890123456789012123456789012345678901234567890121234567890123456789012345678901212345678901234567890123
1234567890123456789012345678901212345678901234567890123456789012123456789012345678901234567890121234567890123
1234567890123456789012345678901212345678901234567890123456789012123456789012345678901234567890121234
12345678901234567890123456789012123456789012345678901234567890121234567890123456789012345678901212345678901234567890123456789012123
1234567890123456789012345678901212345678901234567890123456789012123456789012345678901234567890
12345678901234567890123456789012123456789012345678901234567890121234567890123456789012345678901212345678901234567890123
1234567890123456789012345678901212345678901234567890123456789012123456789012345678901234567890121234567890123
1234567890123456789012345678901212345678901234567890123456789012123456789012345678901234567890121234
123456789012345678901234567890121234567890123456789012345678901212
12345678901234567890123456789012123456789012345678901234567890121234567890123456789012345678901212345678901234567890123456789012123
1234567890123456789012345678901212345678901234567890123456789012123456789012345678901234567890
PRACTICE TEST - 2
12345678901234567890123456789012123456789012345678901234567890121234567890123456789012345678901212345678901234567890123
1234567890123456789012345678901212345678901234567890123456789012123456789012345678901234567890121234567890123
1234567890123456789012345678901212345678901234567890123456789012123456789012345678901234567890121234
12345678901234567890123456789012123456789012345678901234567890121234567890123456789012345678901212345678901234567890123456789012123
1234567890123456789012345678901212345678901234567890123456789012123456789012345678901234567890
123456789012345678901234567890121234567890123456789012345678901212
12345678901234567890123456789012123456789012345678901234567890121234567890123456789012345678901212345678901234567890123
1234567890123456789012345678901212345678901234567890123456789012123456789012345678901234567890121234567890123
1234567890123456789012345678901212345678901234567890123456789012123456789012345678901234567890121234
12345678901234567890123456789012123456789012345678901234567890121234567890123456789012345678901212345678901234567890123456789012123
1234567890123456789012345678901212345678901234567890123456789012123456789012345678901234567890
12345678901234567890123456789012123456789012345678901234567890121234567890123456789012345678901212345678901234567890123
1234567890123456789012345678901212345678901234567890123456789012123456789012345678901234567890121234567890123
1234567890123456789012345678901212345678901234567890123456789012123456789012345678901234567890121234
123456789012345678901234567890121234567890123456789012345678901212
12345678901234567890123456789012123456789012345678901234567890121234567890123456789012345678901212345678901234567890123456789012123
1234567890123456789012345678901212345678901234567890123456789012123456789012345678901234567890
12345678901234567890123456789012123456789012345678901234567890121234567890123456789012345678901212345678901234567890123456789012123
1234567890123456789012345678901212345678901234567890123456789012123456789012345678901234567890
12345678901234567890123456789012123456789012345678901234567890121234567890123456789012345678901212345678901234567890123
1234567890123456789012345678901212345678901234567890123456789012123456789012345678901234567890121234567890123
1234567890123456789012345678901212345678901234567890123456789012123456789012345678901234567890121234

‹¬¶KÇ# : (1-3) ^ÎH÷Æ} =òYOQê x|_ XH› À‹“lÌ„á ‡@° 5. W~¡° z=~¡…Õ LOk Z=~¡°?
‡_È°`Ç°#ß U_È°Q®°~¡° Qê†Ç°h=°}°° x°ó#ß q^¥#O 1) E, F 2) D, C 3) B, E 4) B, A
D q^ÎOQê LOk? 6. L`Çë~¡ z=~¡ #°O_ =ü_È= ªÖ#O…Õ Z=~¡° LO_È°#°?
Z) C H÷ 䛽_ „¬H›ø# D H›^ΰ. 1) G 2) C 3) D 4) F
a) G „¬H›ø# F H›^ΰ. 7. D Z=i =°^Î¼ H›_È°?
4) UnHê^ΰ

Y
‹²) F H÷ Z_È=° „¬H›ø# B H›^ΰ. 1) A, B 2) F, C 3) C, D

M
_) C =°i†Çò B ä›½ =°^Î¼…Õ XH› Qê†Ç°x H›^ΰ. 8. ̄á ㄬ‰×ß#° ªkOKÇ°@䛽 W=Þ|_# ㄬH›@#…Õ J=‹¬~¡O
W) A =°i†Çò D ä›½ =°^Î¼…Õ XH› Qê†Ç°x H›^ΰ. …èxk Uk?
3) D 4) Jhß J=‹¬~¡"Í°

E
Z„¦¹) P =~¡°‹¬…Õ J`ǼO`Ç Z_È=° z=~¡ E H›^ΰ. 1) A 2) B

D
9
1. "Œi…Õ =°^Î¼‹¬ÖOQê L#ßk Z=~¡°? E, F, G, H, I, J, K, L

59
‹¬¶KÇ# : (9-10) J#° 8 =°Ok

1) A 2) B 3) C 4) D
CA
95
=¼ä›½ë° XH› KÇ`Ç°~¡ãªHê~¡„¬ô >è|°…˜ KÇ°@¶“, XHùøH›ø "³á„¬ô

2. B, C ä›½ =°^Î¼…Õ Z=~¡° H›~¡°? W^Îí~¡° Kù„¬C# 䛀~¡°ó<Œß~¡°.

50
1) B 2) D 3) C 4) E ) "Œi…Õ =òQ®°¾~¡° =°‚²ìˆ×° H›~¡°. "Œ~¡° =òQ®°¾~¡°
i

98
A
3. P_†Ç°<£ž #°O_ KǶÀ‹ë "Œi…Õ J`ǼO`Ç ä›½_ z=~¡# ㄬH›ø ㄬH›ø# 䛀~Ëó…è^ΰ.
:8
L#ßk Z=~¡°? ii) J J#° =¼H÷ë L, F ä›½ =°^Î¼…Õ H›~¡°.
E
h

1) D 2) C 3) E 4) F iii) G J#° =¼H÷ë I, F #䛽 =°^Î¼…Õ H›~¡°.


.P
IC

A, B, C, D, E, F, G ° iv) H J#° =°‚²ìˆ× J #䛽 Z_È=°"³á„¬ô #°Oz 2=


AD

‹¬¶KÇ# : (4-8) `Ƕ~¡°æ#䛽 Ja=òYOQê

x|_i. ªÖ#O…Õ H›^ΰ.


AB
O

Z) D H÷ 䛽_ „¬H›ø<Í C H›^ΰ. v) F J#° =¼H÷ë E J#° =°‚²ìˆ×佛 Z^ΰ~¡°Qê 䛀~¡°ó<Œß_È°.


ER
CH

a) B P =~¡°‹¬…Õ XH› z=~¡# H›_È° =°i†Çò J`Ç_ vi) F =°i†Çò I 䛽 =°^Î¼…Õ XH› =°‚²ìˆ× H›^ΰ.

„¬H›ø# E LO_È°#°.
YD

Ì„á ‹¬¶KÇ# P^¥~¡OQê ãH÷Ok ㄬ‰×߁䛽 ‹¬=¶^¥<Œ° ~Œ†Ç°O_.

‹²) E =°i†Çò F  =°^Î¼…Õ G LO_È°#°. ̄á "Œi…Õ =°‚²ìˆ×° Z=ï~=~¡°<Œß~¡°?


,H

9.

_) ^ÎH÷Æ} z=~¡ #°O_ D =ü_È= ªÖ#O…Õ H›_È°. 1) H, E, G 2) E, G, J 3) G, H, J 4) HFI


T
AM

ãH÷Ok "Œx…Õ ‹¬i†³Ø°#k Uk?


H

10.

1) J XH› =°‚²ìˆ× 2) J J#°#k Z=~Ë K³„¬æ…è=ò


Ì„á ‹¬¶KÇ# P^¥~¡OQê ãH÷Ok ㄬ‰×߁䛽 ‹¬=¶^¥<Œe=òà.
M

4. E H÷ 䛽_ „¬H›ø# Z=~¡° H›~¡°?


AM

3) J XH› „¬ô~°¡ +¬µ_È° 4) J †³òH›ø ªÖ#O Q®°iOz K³„æ¬ …è=ò


IG

1) A 2) C 3) G 4) B
KH

1234567890123456789012
12345678901234567890123456789012123456789012345678901234567890121234567890123456789012345678901212345678901234567890123
12345678901234567890123

q=~¡}ì`ÇàH› ["Œ|°°
12345678901234567890123456789012123456789012345678901234567890121234567890123456789012345678901212345678901234567890123
1234567890123456789012
1234567890123456789012 12345678901234567890123
12345678901234567890123
12345678901234567890123456789012123456789012345678901234567890121234567890123456789012345678901212345678901234567890123
1234567890123456789012
PRACTICE TEST - 2 : 12345678901234567890123
12345678901234567890123456789012123456789012345678901234567890121234567890123456789012345678901212345678901234567890123
R

1234567890123456789012
12345678901234567890123456789012123456789012345678901234567890121234567890123456789012345678901212345678901234567890123
1234567890123456789012 12345678901234567890123
12345678901234567890123456789012123456789012345678901234567890121234567890123456789012345678901212345678901234567890123
1234567890123456789012
12345678901234567890123
12345678901234567890123
12345678901234567890123456789012123456789012345678901234567890121234567890123456789012345678901212345678901234567890123

(1–3) ('#%$"& )…­õ*!* ( )>±*

1. 4 2. 2 3. 3
)>±*!& …­õ! '

# )…­õ*!,
(4 – 8)
&
)…­õ*!+
( dŒÔ
dŒÔ_ Œ•@¨
'

b’Ìh·
%
$ )>±*!) - )…­õ*
" 9. 1 10. 3
4. 3 5. 4 6. 1 7. 2 8. 4

(9 – 13)
REASONING ABILITY (VERBAL) ○ ○ ○ ○ ○ ○ ○ ○ ○
119
○ ○ ○ ○ ○ ○ ○ ○ ○ ○ ○ ○ ○ ○ ○ ○ ○ ○ ○
Study Material
123456789012345678901234567890121234567890123456789012345678901212
12345678901234567890123456789012123456789012345678901234567890121234567890123456789012345678901212345678901234567890123
1234567890123456789012345678901212345678901234567890123456789012123456789012345678901234567890121234567890123
1234567890123456789012345678901212345678901234567890123456789012123456789012345678901234567890121234
12345678901234567890123456789012123456789012345678901234567890121234567890123456789012345678901212345678901234567890123456789012123
123456789012345678901234567890121234567890123456789012345678901212345678901234567890123
12345678901234567890123456789012123456789012345678901234567890121234567890123456789012345678901212345678901234567890123
1234567890123456789012345678901212345678901234567890123456789012123456789012345678901234567890121234567890123
1234567890123456789012345678901212345678901234567890123456789012123456789012345678901234567890121234
123456789012345678901234567890121234567890123456789012345678901212
12345678901234567890123456789012123456789012345678901234567890121234567890123456789012345678901212345678901234567890123456789012123
123456789012345678901234567890121234567890123456789012345678901212345678901234567890123
12345678901234567890123456789012123456789012345678901234567890121234567890123456789012345678901212345678901234567890123
1234567890123456789012345678901212345678901234567890123456789012123456789012345678901234567890121234567890123
1234567890123456789012345678901212345678901234567890123456789012123456789012345678901234567890121234
12345678901234567890123456789012123456789012345678901234567890121234567890123456789012345678901212345678901234567890123456789012123
123456789012345678901234567890121234567890123456789012345678901212345678901234567890123
123456789012345678901234567890121234567890123456789012345678901212
PREVIOUS BITS
12345678901234567890123456789012123456789012345678901234567890121234567890123456789012345678901212345678901234567890123
1234567890123456789012345678901212345678901234567890123456789012123456789012345678901234567890121234567890123
1234567890123456789012345678901212345678901234567890123456789012123456789012345678901234567890121234
12345678901234567890123456789012123456789012345678901234567890121234567890123456789012345678901212345678901234567890123456789012123
123456789012345678901234567890121234567890123456789012345678901212345678901234567890123
123456789012345678901234567890121234567890123456789012345678901212
12345678901234567890123456789012123456789012345678901234567890121234567890123456789012345678901212345678901234567890123
1234567890123456789012345678901212345678901234567890123456789012123456789012345678901234567890121234567890123
1234567890123456789012345678901212345678901234567890123456789012123456789012345678901234567890121234
12345678901234567890123456789012123456789012345678901234567890121234567890123456789012345678901212345678901234567890123456789012123
123456789012345678901234567890121234567890123456789012345678901212345678901234567890123
12345678901234567890123456789012123456789012345678901234567890121234567890123456789012345678901212345678901234567890123
1234567890123456789012345678901212345678901234567890123456789012123456789012345678901234567890121234567890123
1234567890123456789012345678901212345678901234567890123456789012123456789012345678901234567890121234
12345678901234567890123456789012123456789012345678901234567890121234567890123456789012345678901212345678901234567890123456789012123
123456789012345678901234567890121234567890123456789012345678901212345678901234567890123
123456789012345678901234567890121234567890123456789012345678901212
12345678901234567890123456789012123456789012345678901234567890121234567890123456789012345678901212345678901234567890123456789012123
123456789012345678901234567890121234567890123456789012345678901212345678901234567890123
12345678901234567890123456789012123456789012345678901234567890121234567890123456789012345678901212345678901234567890123
1234567890123456789012345678901212345678901234567890123456789012123456789012345678901234567890121234567890123
1234567890123456789012345678901212345678901234567890123456789012123456789012345678901234567890121234

ㄬH›~¡}=ò ㄬ‰×ß ‹¬O. 1 #°O_ 5 : 3. =~¡°‹¬…Õ =ü_È= ªÖ#O…Õ 䛀~¡°ó#ßk Z=~¡°?


(AP. Panchyat Secretary Screening -2016) 1) =°OA 2) ~¡q 3) NH›~Ÿ 4) H©ië
=òQ®°¾~¡° ƒì°~¡° =°i†Çò =òQ®°¾~¡° ƒìeH›° (NH›~Ÿ, ~¡q, 4. =~¡°‹¬…Õ ï~O_È= ªÖ#O…Õ 䛀~¡°ó#ßk Z=~¡°?
~Œ=ò, =°OA, ‰Õƒ’ =°i†Çò H©ië) XH› =~¡°‹¬…Õ, L`Çë~¡=ò #°O_ 1) =°OA 2) ‰Õƒ’ 3) ~¡q 4) NH›~Ÿ
^ÎH÷Æ}=ò "³á#°, JO^Î~¡¶ `Ƕ~¡°æ =òY=òQê 䛀~¡°ó<Œß~¡°. =~¡°‹¬ 5. ~¡q, H©iëH÷ =°^Î¼# Z=~¡° 䛀~¡°ó<Œß~¡°?
z@“ z=~¡°…Õ 'S' JH›Æ~¡=ò`Ë "³ò^΁†Í°¼ À„~¡° (POQ®Á=ò…Õ 1) ~Œ=ò 2) ‰Õƒ’ 3) =°OA 4) NH›~Ÿ
À„~¡° ã"Œ‹²#„¬C_È°) H›"Œ~¡° 䛀~ùó#…è^°Î . =°OA 'R' JH›~Æ =¡ ò`Ë 6. Ì„á ‹²Ö`Ç°ä›½ ‹¬i„¬_È° XöH XH› ª^Î¼"³°Ø# J=°iH› ãH÷Ok
"³ò^΁†Í°¼ À„~¡° (POQ®Á=ò…Õ ã"Œ‹²#„¬C_È°) H› =¼H÷ëH÷ Z_È=°"³á„¬ô "Œx…Õ Uk?
1) uH±ä›½ Z_È=°"³á„¬ô x`Í ä›€~Ëó=_ÈO
(SI-2008)

䛀~ùó#ßk. ~¡q =~¡°‹¬…Õ z@“z=~¡Qê 䛀~ùó#…è^°Î . W^Î~í °¡ ƒìeH› 2) w`Ç, „¦¬~¡¶H± =°^Î¼ `Ç ä›€~Ëó=_ÈO
=°^¼Î # 'R' J<Í JH›~Æ =¡ ò`Ë "³ò^΁†Í°¼ (POQ®=Á ò…Õ ã"Œ‹²#„¬C_È°)

Y
3) KÇO„¬ ‹¬iQê¾ w`Ç䛽 Ja=òYOQê 䛀~Ëó=_ÈO
À„~¡° Q® ƒì°_È° 䛀~ùó<Œß_È°. =~¡°‹¬…Õ XH› z@“ z=~¡# XH› 4) KÇO„¬ã„¬H›ø# „¦¬~¡¶H± 䛀~Ëó=_ÈO

M
ƒìeH› 䛀~ùó<Œß_È°. =~¡°‹¬…Õ XH› z@“z=~¡# XH› ƒìeH› 7. P~¡°Q®°~¡° ƒì°~¡°, =$`ŒëHê~¡OQê öHOã^Î=ò KÇ°@¶“
䛀~ùó#ßk. H©ië, ‰Õƒ’䛽 䛽_ ㄬH›ø# 䛀~ùó#ßk. ‰Õƒ’ 'R' J<Í

E
x°KÇ°<Œß~¡°. J‰ÕH±, ㄬƒ’°H÷ Z_È=° "³á„¬ô# x°KÇ°<Œß_È°.
JH›Æ~¡=ò`Ë "³ò^΁†Í°¼ À„~¡° (POQ®Á=ò…Õ ã"Œ‹²#„¬C_È°) H›

D
9
PHꉘ =°i†Çò ㄬHꉘ =°^Î¼ ‹¬°^¥H›~Ÿ L<Œß_È°. ‚¬ìi

59
ƒì°xH÷ 䛽_ "³á„¬ô# 䛀~ùó#ßk. L`Çë~¡„¬ô "³á„¬ô #°O_ ï~O_È= J‰ÕH±ä½› =°i†Çò PHꉘ佛 =°^¼Î L<Œß_È°. ㄬHꉘ佛 Z_È=°
CA
95
ªÖ#=ò…Õ 䛀~ùó#ß ƒì°x À„~¡° 'R' `Ë "³ò^΁° Hê^ΰ "³á„¬ô# Z=~¡°<Œß~¡°? (Group-I-2012)

(POQ®=Á ò…Õ ã"Œ‹²#„¬C_È°). NH›~Ÿ =°i†Çò ~¡q =°^¼Î XH› ƒìeH› 1) ‚¬ìi 2) ㄬƒ’° 3) PHꉘ 4) ‹¬°^¥H›~Ÿ
50
L#ßk. ~¡q =°i†Çò H©ië =°^Î¼ XH› ƒìeH› L#ßk? 98
A
8. ö H Oã^Î ªÖ # =ò#ä› ½ Ja = òYOQê XH› =$`Ç ë = ò…Õ
:8
A,B,C,D,E =°i†Çò F J#° P~¡°Q®°~¡° À‹ß‚²ì`Ç°°
1. =~¡°‹¬…Õ ^ÎH÷Æ}O "³á„¬ô z@“ z=~¡ 䛀~¡°ó#ßk Z=~¡°?
E

1) ~Œ=ò 2) ‰Õƒ’ 3) H©ië 4) =°OA 䛀~ùóx L<Œß~¡°. A,B =°i†Çò E H÷ =°^Î¼ H›^ΰ. C,D
h
.P

=°i†Çò F 䛽 =°^Î¼ H›^ΰ. E,D 䛽 "³O@<Í ä›½_"³á„¬ô#


IC

2. =~¡°‹¬…Õ L`Çë~¡O"³á„¬ô z@“ z=~¡ 䛀~¡°ó#ßk Z=~¡°?


AD

1) ~¡q 2) ~Œ=ò 3) =°OA 4) ‰Õƒ’ H›_È°. E =°i†Çò C H÷ =°^¼Î Z=~¡° H›~¡°? (SI - 2011)
1) D 2) B 3) C 4) x~¡â~òO„¬|_ȅèxk
AB
O

12345678901234567890123456789012123456789012345678901234567890121234567890123456789012345678901212345678901234567890123
1234567890123456789012 123456789012345678901234

q=~¡}ì`ÇàH› ["Œ|°°
12345678901234567890123456789012123456789012345678901234567890121234567890123456789012345678901212345678901234567890123
1234567890123456789012 123456789012345678901234
1234567890123456789012 123456789012345678901234
12345678901234567890123456789012123456789012345678901234567890121234567890123456789012345678901212345678901234567890123
ER
CH

1234567890123456789012
1234567890123456789012
1234567890123456789012 PREVIOUS BITS : 123456789012345678901234
12345678901234567890123456789012123456789012345678901234567890121234567890123456789012345678901212345678901234567890123
123456789012345678901234
12345678901234567890123456789012123456789012345678901234567890121234567890123456789012345678901212345678901234567890123
123456789012345678901234
12345678901234567890123456789012123456789012345678901234567890121234567890123456789012345678901212345678901234567890123
1234567890123456789012 123456789012345678901234
12345678901234567890123456789012123456789012345678901234567890121234567890123456789012345678901212345678901234567890123
YD

(1-5) ㄬ‰×ß…Õ W=Þ|_# ‹¬=¶KŒ~¡O P^¥~¡OQê J=°iH›° 6. (3)


,H

K͆ǰQê=KÇ°ó J=°iH›° D ãH÷Ok q^ÎOQê LO\ì~ò. 7. (2) Wzó# ‹¬=¶KŒ~¡O P^¥~¡OQê P~¡° =¼ä›½ë° x°ó#ß
T

q^¥#=ò#° D ãH÷Ok
AM

(ㄬ‰×ß…Õ =¼ä›½ë°, NH›~Ÿ, ~¡q, ~Œ=ò, =°OA, ‰Õƒ’, ‹¬°^¥H›~Ÿ


H

H©ië) „¬@O `³°æ`Ç°Ok. PHꉘ


M

/ ㄬHꉘ
AM
IG

/ 8 &
KH

>ó
OIœ  4 ‚¬ìi ㄬƒ°’
R

>±PB J‰ÕH±
¢r
)aÈ·¢±ç!>óZP* Ì„á „¬@O #°O_ ã„H¬ ꉘ佛 Z_È=° "³„á ô¬ # '㄃¬ °’ Ñ L<Œß_È°.
ŠÖ„“ Jx `³°‹¬°ëOk.
Iªjì 8. (1) P~¡°Q®°~¡° =¼ä›½ë° Wzó# ‹¬=¶KŒ~¡O ㄬHê~¡O
4
P^¥~¡OQê =$`ŒëHê~¡ |Á̄á 䛀~ùó#° q^ÎOQê ㄬH›ø#
1. (3) ^ÎH÷Æ}O "³á„¬ô z@“ z=~¡ 䛀~ùó#ßk - H©ië KǶ„¬|_#k. #
2. (2) L`Çë~¡O "³á„¬ô z@“z=~¡ 䛀~¡°ó#ß =¼H÷ë - ~Œ=ò " '
3. (1) =~¡°‹¬…Õ =ü_È= ªÖ#O…Õ 䛀~¡°ó#ßk '=°OAÑ & $
4. (4) =~¡°‹¬…Õ ï~O_È= ªÖ#O…Õ 䛀~¡°ó#ßk 'NH›~ŸÑ %
5. (2) ~¡q, H©ië =°^Î¼ ‰Õƒ’ 䛀~¡°ó#ßk. Ì„á „¬@O #°O_ E =°i†Çò C  =°^Î¼ 'D' H›^ΰ.
REASONING ABILITY (VERBAL) ○ ○ ○ ○ ○ ○ ○ ○ ○
120
○ ○ ○ ○ ○ ○ ○ ○ ○ ○ ○ ○ ○ ○ ○ ○ ○ ○ ○
Study Material
10 „¬l…˜ >ÿ‹¹“ (Puzzle Test)

=òMì¼O‰§°...
G „¬l…˜ >ÿ‹¹“ qƒìQ®O…Õx ㄬ‰×߁䛽 ‹¬=¶^¥<Œ° D ãH÷Ok „¬^Μ`Ç° ^¥Þ~Œ ªkOKÇ=KÇ°ó.
1) "³ò^Î@ Wzó# ‹¬=¶KŒ~Œxß xt`ÇOQê, 䛀O䛽+¬OQê KÇ^Î"Œe.
2) ‹¬=¶KŒ~¡O…Õ W=Þ|_# q+¬†Ç¶xß „¬îiëQê J=Qꂬì# K͋¬°HË"Œe.
3) q+¬†Ç° J=Qꂬì# `Ç~¡°"Œ`Ç ‹¬=°‹¬¼…Õ U q+¬†Ç¶xH÷ ‹¬O|OkOz# ‹¬=°‹¬¼ WKŒó~Ë Q®°iëOKŒe.
4) Wzó# ‹¬=¶KŒ~Œxß =s¾H›iOKÇ°HË"Œe. D =s¾H›~¡}, „¬\ ÷“H› ~¡¶„¬O…Õ f‹¬°ä›½O>è ‹¬=¶^¥#O ~Œ|@“_ÈO KŒ…ì

Y
‹¬°ƒ’`Ç~¡O J=ô`Ç°Ok.

M
5) ‹¬=¶KŒ~¡O…Õ Wzó# =‹¬°=ë ô …è^¥ =¼ä›½ë P^¥~¡OQê "Œi ‹²uÖ Q®`°Ç #° (L^¥ : LO_È°@, …è佛 O_È°@, W+¬O“ , J~ò+¬O“ ,
K͆òÇ @, K͆°Ç H›‡é=ô@, =KÇ°ó@, ~ŒH›‡é=ô@, P_È°@, P_ÈH‡› é=ô@...) 3ÿ(\÷H)± =°i†Çò þ (WO@°) Q®°~¡°ë `Ë KǶ„¬=…ÿ#°.

E
L^¥ : ~Œ„¦¬°= ‚¬ðH© P_È°`Œ_È°. H›#°H› „¬\ ÷“H›…Õ ‚¬ðH©H÷ ‹¬O|OkOz# =~¡°‹¬…Õ ~Œ„¦¬°=H÷ Z^ΰ~¡°Qê 3(\÷H±) =¶~¡°ø

D
9
59
KǶ„¬ô`Œ=ò. [†Ç°Ou ‚¬ðH© P_È^ΰ. H›#°H› „¬\ ÷“H›…Õ ‚¬ðH©H÷ ‹¬O|OkOz# =~¡°‹¬…Õ [†Ç°OuH÷ Z^ΰ~¡°Qê þ (WO@°)
CA
95
=¶~¡°ø KǶ„¬=…ÿ#°.
6) D q^OÎ Qê ‹¬=¶KŒ~Œxß q‰õ+Á O² z „¬\H“÷ › ~¡¶„¬O…Õ ~Œ‹²# `Ç~°¡ "Œ`Ç ã„¬‰ß×  P^¥~¡OQê ‹¬=¶^¥<Œ° H›#°Qù#_ÈO ‹¬°ƒ`’ ~Ç O¡

50
J=ô`Ç°Ok.
98
A
G „¬l…˜ >ÿ‹¹“ qƒìQ®O…Õ Jƒ’¼~¡°Ö „¬ij<Œ ‰×H÷ë, J=Qꂬì<Œ ‰×H÷ë, *ìý„¬H›‰×H÷ë, J#ކǰ‰×H÷ë, P…ÕKÇ<Œ‰×H÷ë, J~¡ÖO K͋¬°Hù<Í ‰×H÷ë
:8
E

=°i†Çò „¬iq°`Ç°ä›½ …Õ|_ x~¡â†Ç¶° f‹¬°ä›½<Í ‰×H÷ëx „¬ijeªë~¡°.


h

G
.P

„¬l…˜>ÿ‹¹“ qƒìQ®O…Õ =s¾H›~¡} >ÿ؄¬ô ㄬ‰×߁ qƒìQ®O KŒ…ì =òY¼"³°Ø#k. ㄬ‰×߁…Õ Z䛽ø= ƒìQ®O D qƒìQ®O#öH K³Ok#q
IC
AD

J~ò =ôO\ì~ò.
G D qƒìQ®O…Õ ‹¬=¶KŒ~¡O "³ò`ÇëO „¬° "Œ¼Y¼° ~¡¶„¬O…Õ Wªë~¡°. Wzó# ‹¬=¶KŒ~¡O P^¥~¡OQê ㄬ‰×߁° W=Þ_ÈO
AB
O

[~¡°Q®°`Ç°Ok. Jƒ’¼iÖ Wzó# ‹¬=¶KŒ~Œxß ä›Æ½}âOQê „¬ijeOz, J~¡ÖO K͋¬°ä›½x ㄬ‰×߁䛽 ‹¬i†³Ø°# ‹¬=¶^¥<Œ° W"ŒÞe.
ER
CH

G D qƒìQ®O…Õx ㄬ‰×߁ ‹¬=¶^¥<Œ Hù~¡ä›½ Z@°=O\÷ ‹¬¶ã`Œ° =°i†Çò ‘~Ÿ“H›\˜ž L„¬†³¶yOKÇ=‹²# J=‹¬~¡O …è^ΰ.
YD

<Í~¡°Qê Wzó# ‹¬=¶KŒ~¡O …è^¥ „¬\ ÷“H› ^¥Þ~Œ<Í H›#°Qù#=KÇ°ó#°.


,H

G Hùxß ã„¬‰×߁䛽 ‹¬=¶^¥<Œ° Wzó# ‹¬=¶KŒ~¡O #°O_ T‚²ìOz H›#°Qù<Œež =ôO@°Ok. Hùxß ã„¬‰×߁䛽 ‹¬=¶^¥<Œ°
T
AM

‹¬=¶KŒ~Œxß …Õ`Ç°Qê J^Î¼†Ç°#O K͋² q‰õÁ+²Oz ‹¬=¶KŒ~¡O…Õx "Œ‹¬ë"Œ #°O_ `ŒiøH›OQê P…ÕzOz XH› Jaã‡†Ç¶xß
H

U~¡æ~¡KÇ°Hùx K³„¬æ=‹² =ôO@°Ok.


M

G
AM

Wzó# ㄬ‰×߁䛽 ["Œ|°° ~Œ|@°“@ öH=O Jƒ’¼iÖ J=Qꂬì#, „¬ij#, q‰õÁ+¬}, "Í°^ŠÎ‹¬°žÌ„á# =¶ã`Ç"Í° P^¥~¡„¬_
IG

LO@°Ok.
KH

G „¬°=¶~¡°Á ‹¬=°‹¬¼#° Jƒì¼‹¬O K͆Çò@=# ㄬ‰×߁ ‹¬~¡o, ㄬ‰×߁ "³áq^Î¼O `³°‹¬°ëOk. ª^Î<Œ <³á„¬ô}¼O ̄~¡°Q®°`Ç°Ok.
R

=¶ki ㄬ‰×߁°
MODEL - 1 II) J`Ç#° 'Q' H›ŌhH÷ "³ˆ×¤H›=òO^Í 'N' H›ŌhH÷
"³ˆìë_È°.
G XH› À‹…˜ž iㄬ[O>èq"£ M, N, P, Q, R =°i†Çò S J#° III) J`Ç#° =ü_È=~ËA "³o¤# H›Ōh 'P'.
H›Ōh#° ~ËA䛽 XH› H›Ōh Kù„¬C# P~¡° ~ËA…Õ Ì„á ‹¬=¶KŒ~¡O P^¥~¡OQê D ãH÷Ok ㄬ‰×߁䛽 ‹¬=¶^¥#O
P~¡° H›Ōhä›½ "³ˆì¤x ‡Á<£ K͋¬°Hù<Œß_È°. W=ÞO_.
I) J`Ç#° 'N' =°i†Çò 'R' H›Ōhä›½ "³ˆ×¤H›=òO^Í
H›Ōh 'M' 䛽 "³ˆìë_È°.
REASONING ABILITY (VERBAL) ○ ○ ○ ○ ○ ○ ○ ○ ○
121
○ ○ ○ ○ ○ ○ ○ ○ ○ ○ ○ ○ ○ ○ ○ ○ ○ ○ ○
Study Material
1. P À‹…˜ž iㄬ[O>è\ "
÷ £ Ì+_ȶ¼…˜ ㄬHê~¡O, D ãH÷Ok "Œx…Õ 3. JO^Îi…ÕH÷ =†Ç°‹¬°ž „¬~¡OQê =°^Î¼# =ô#ßk?
‹¬i†³Ø°#k. ª^Î#: A>G>
M >R>K
N
ª^Î#: Wzó# ‹¬=¶KŒ~¡O ㄬHê~¡O, J`Ç#° H›O>è =òO^ΰ M JO^Îi…ÕH÷ =†Ç°‹¬°ž „¬~¡OQê =°^Î¼# =ô#ßk =¶…ì.
䛽 =°i†Çò Q H›O>è =òO^ΰ N 䛽 "³ˆìë_°È . Hê=ô# ‹¬æ+¬O“ Qê 4. JO^Îi…ÕH÷ Z䛽ø= =†Ç°‹¬°ž H›ey#k?
J`Ç#° Q H›O>è =òO^ΰ M䛽 "³ˆìë_È°. ª^Î#: A >G>M>R>K
2. À‹…˜ž iㄬ[O>è\ " ÷ £ H›Ōh 'S' 䛽 "³ò^Î@Qê "³o¤# J`Ç#° =†Ç°‹¬°ž „¬~¡OQê JO^Îi…ÕH÷ ̄^Îík J…ìø.
ï~O_È=~ËA "³ˆ×ä H›Ōh Uk? 5. JO^Îi…ÕH÷ z#ß =†Ç°‹¬°ž H›ey#k?
ª^Î#: J`Ç#° H›Ōh 'S' #° "³ò^Î\ ÷~ËA ^ÎiÅOz#, H›Ōh ª^Î#: A>G>M>R> K

'P' #° =ü_È=~ËA ^ÎiÅOz# M,N,Q =°i†Çò R° =†Ç°‹¬°ž „¬~¡OQê JO^Îi…ÕH÷ z#ß"Œ_È° H›„²…˜.
J#°‹¬iªë~ò. Hê=ô# ~ï O_È=~ËA J`Ç#° ^ÎiÅOKÇ° H›Ōh
‹¬æ+¬“OQê M. MODEL - 3

Y
3. À‹…˜ž iㄬ[O>è\ "
÷ £ H›Ōh 'P' `Ç~°¡ "Œ`Ç "³O@<Í `Ç~°¡ "Œ`Ç I) P =°i†Çò Q „¦¬ô\˜ƒì…˜, ‚¬ðH© P_È°`Œ~¡°.

M
~ËA U H›Ōhx ^ÎiÅOKDžè_È°? II) R =°i†Çò S ƒì¼_àO@<£, ãH÷ïH\˜ P_È°`Œ~¡°.
'P' M,N,Q; R III) Q =°i†Çò R ãH÷ïH\˜, „¦¬ô\˜ƒì…˜ P_È°`Œ~¡°.

E
ª^Î#: =ü_È= ~ËA =°i†Çò ° J#°‹¬iªë~ò.
H›#°H› J`Ç#° H›Ōh 'P' #° ^ÎiÅOKǰ䛽#ß `Ç~¡°"Œ`Ç ~ËA IV) P =°i†Çò S ƒì¼_àO@<£, „¦¬ô\˜ƒì…˜ =°i†Çò ‚¬ðH©

D
9
59
H›Ōh M `DŽ¬æ q°ye# U H›Ōh<³á<Œ ^ÎiÅOKÇQ®_È°. P_È°`Œ~¡°.
CA
95
H›Ōh M "³ò^Î\ ÷ …è^¥ ï~O_È= ªÖ#O…Õ =ôO@°Ok. Ì„á ‹¬=¶KŒ~¡O P^¥~¡OQê D ãH÷Ok ㄬ‰×߁䛽 ‹¬=¶^¥#O

50
ZO^ΰH›#Qê Q, R =°i†Çò N  `Ç~¡°"Œ`Ç M =ôO_È^ΰ. W=ÞO_.
98
A
G Wzó# ‹¬=¶KŒ~Œxß „¬\ ÷“H› ~¡¶„¬O…Õ ~Œ†Ç°Qê U~¡æ_#
:8
MODEL - 2 „¬\ ÷“H› ãH÷O^Î KǶ„¬|_#k.
E
h

G J…ìø, =¶…ì H›<Œß ̄^Îík. Qˇ…˜ =¶…ì H›<Œß ̄^Îí"Œ_È°


.P
IC

=¼H÷ë ‚¬ðH© „¦¬ô\˜ƒì…˜ ƒì¼_àO@<£ ãH÷ïH\˜


Hêx J…ìø H›<Œß z#ß"Œ_È°. H›„…² ˜, ~Œ"£° =°i†Çò =¶…ì
AD

H›<Œß z#ß"Œ_È°. =¶…ì, ~Œ"£° H›<Œß ̄^Îík. P 3 3 3 þ


AB
O

Q 3 3 þ 3
R
ER

(R.R.B. - 2015)

3 3 3
CH

Ì„á ‹¬=¶KŒ~¡O P^¥~¡OQê D ãH÷Ok ㄬ‰×߁䛽 ‹¬=¶^¥#O þ


S 3 3 3 3
YD

W=ÞO_.
Ì„á „¬\ ÷“H› #°O_,
,H

Wzó# ‹¬=¶KŒ~¡O ㄬHê~¡O,


J…ìø (A), =¶…ì (M), H›„²…˜ (K), ~Œ"£° (R)
T
AM

1. ƒì¼_àO@<£ P_Èx"Œ~¡° Z=~¡°?


=†Ç°‹¬°ž° A > M, A > G > M, R > K, M > K =°i†Çò Q ƒì¼_àO@<£ P_È_È°.
H

ª^Î#:
M

M > R.
AM

2. ãH÷ïH\˜ P_Èx"Œ~¡° Z=~¡°?


IG

g@xßO\÷h H›°„¬Qê A > G > M > R > K =KÇ°ó#°. ª^Î#: P ãH÷ïH\˜ P_È_È°.
KH

D ãH›=°O #°O_ ㄬ‰×߁䛽 ‹¬=¶^¥#O ~Œ|@“=KÇ°ó#°. 3. ãH÷Hï \˜, „¦ô¬ \˜ƒì…˜ =°i†Çò ‚¬ðH© =¶ã`Ç"°Í P_Í"Œ~¡° Z=~¡°?
R

1. Qˇ…˜ =°i†Çò ~Œ"£° =†Ç°‹¬°ž H›<Œß =°^Î¼ =†Ç°‹¬°ž


ª^Î#: ãH÷ïH\˜, „¦¬ô\˜ƒì…˜ =°i†Çò ‚¬ðH© =¶ã`Ç"Í° P_È°"Œ_È° Q.
H›e¾#k?
4. ƒì¼_àO@<£, „¦¬ô\˜ƒì…˜ =°i†Çò ãH÷ïH\˜ =¶ã`Ç"Í° P_Í"Œ~¡°
ª^Î#:A>G> M >R>K Z=~¡°?
Qˇ…˜ =°i†Çò ~Œ"£° =†Ç°‹¬°žä›½ =°^Î¼ =†Ç°‹¬°ž
H›e¾#k =¶…ì. ª^Î#: ƒì¼_àO@<£, „¦¬ô\˜ƒì…˜ =°i†Çò ãH÷ïH\˜ =¶ã`Ç"Í° P_È°"Œ_È°
2. =¶…ì =°i†Çò H›„²…˜ =†Ç°‹¬°ž H›<Œß =°^Î¼ =†Ç°‹¬°ž
R.
H›e¾#k? 5. ƒì¼_àO@<£, „¦¬ô\˜ƒì…˜ =°i†Çò ‚¬ðH© =¶ã`Ç"Í° P_Í"Œ~¡°
ª^Î#:A>G>M> R >K Z=~¡°?
=¶…ì =°i†Çò H›„…² ˜ =†Ç°‹¬°žä›½ =°^¼Î =†Ç°‹¬°ž H›e#¾ k ª^Î#: ƒì¼_àO@<£, „¦¬ô\˜ƒì…˜ =°i†Çò ‚¬ðH© =¶ã`Ç"Í° P_È°"Œ_È°
~Œ"£°. P.
REASONING ABILITY (VERBAL) ○ ○ ○ ○ ○ ○ ○ ○ ○
122
○ ○ ○ ○ ○ ○ ○ ○ ○ ○ ○ ○ ○ ○ ○ ○ ○ ○ ○
Study Material
123456789012345678901234567890121234567890123456789012345678901212
123456789012345678901234567890121234567890123456789012345678901212
12345678901234567890123456789012123456789012345678901234567890121234567890123456789012345678901212345678901234567890123456789012123
1234567890123456789012345678901212345678901234567890123456789012123456789012345678901234567890
12345678901234567890123456789012123456789012345678901234567890121234567890123456789012345678901212345678901234567890123
1234567890123456789012345678901212345678901234567890123456789012123456789012345678901234567890121234567890123
1234567890123456789012345678901212345678901234567890123456789012123456789012345678901234567890121234
12345678901234567890123456789012123456789012345678901234567890121234567890123456789012345678901212345678901234567890123456789012123
1234567890123456789012345678901212345678901234567890123456789012123456789012345678901234567890
12345678901234567890123456789012123456789012345678901234567890121234567890123456789012345678901212345678901234567890123
1234567890123456789012345678901212345678901234567890123456789012123456789012345678901234567890121234567890123
1234567890123456789012345678901212345678901234567890123456789012123456789012345678901234567890121234
123456789012345678901234567890121234567890123456789012345678901212
12345678901234567890123456789012123456789012345678901234567890121234567890123456789012345678901212345678901234567890123456789012123
1234567890123456789012345678901212345678901234567890123456789012123456789012345678901234567890
12345678901234567890123456789012123456789012345678901234567890121234567890123456789012345678901212345678901234567890123
1234567890123456789012345678901212345678901234567890123456789012123456789012345678901234567890121234567890123
1234567890123456789012345678901212345678901234567890123456789012123456789012345678901234567890121234
PRACTICE TEST - 1
12345678901234567890123456789012123456789012345678901234567890121234567890123456789012345678901212345678901234567890123456789012123
1234567890123456789012345678901212345678901234567890123456789012123456789012345678901234567890
123456789012345678901234567890121234567890123456789012345678901212
12345678901234567890123456789012123456789012345678901234567890121234567890123456789012345678901212345678901234567890123
1234567890123456789012345678901212345678901234567890123456789012123456789012345678901234567890121234567890123
1234567890123456789012345678901212345678901234567890123456789012123456789012345678901234567890121234
12345678901234567890123456789012123456789012345678901234567890121234567890123456789012345678901212345678901234567890123456789012123
1234567890123456789012345678901212345678901234567890123456789012123456789012345678901234567890
123456789012345678901234567890121234567890123456789012345678901212
12345678901234567890123456789012123456789012345678901234567890121234567890123456789012345678901212345678901234567890123
1234567890123456789012345678901212345678901234567890123456789012123456789012345678901234567890121234567890123
1234567890123456789012345678901212345678901234567890123456789012123456789012345678901234567890121234
12345678901234567890123456789012123456789012345678901234567890121234567890123456789012345678901212345678901234567890123456789012123
1234567890123456789012345678901212345678901234567890123456789012123456789012345678901234567890
12345678901234567890123456789012123456789012345678901234567890121234567890123456789012345678901212345678901234567890123
1234567890123456789012345678901212345678901234567890123456789012123456789012345678901234567890121234567890123
1234567890123456789012345678901212345678901234567890123456789012123456789012345678901234567890121234
12345678901234567890123456789012123456789012345678901234567890121234567890123456789012345678901212345678901234567890123456789012123
1234567890123456789012345678901212345678901234567890123456789012123456789012345678901234567890
123456789012345678901234567890121234567890123456789012345678901212
12345678901234567890123456789012123456789012345678901234567890121234567890123456789012345678901212345678901234567890123456789012123
1234567890123456789012345678901212345678901234567890123456789012123456789012345678901234567890
1234567890123456789012345678901212345678901234567890123456789012123456789012345678901234567890121234567890123
1234567890123456789012345678901212345678901234567890123456789012123456789012345678901234567890121234

; ‹¬¶KÇ# (1-5) : JaÀ+H±, =~¡°}Ÿ, J[†Ÿ°, r=<£, kb„¹ 12. ªÞu, ‰õÞ`Ç H›O>è ‡Ú_È°ïQá#k. ‰§ex, "³á‰§e H›O>è ‡Ú\÷“.
J#° 5Q®°~¡° À‹ß‚²ì`Ç°° H›~¡°. gi…Õ JaÀ+H± =~¡°}Ÿ H©i,ë ‰§ex H›O>è ‡Ú_È°Q
ï # á §e Z`Ç°ë 5"3 L#ß@Á~ò`Í
á k. "³‰
H›O>è ‡Ú\÷“"Œ_È°, kb„¹ H›O>è ‡Ú_È"³á#"Œ_È°. J[†Ÿ° JO^Îi…Õ#° ‡Ú_È°Q®° Z=~¡°?
JO^ÎiH›O>è ‡Ú_È"³á# "Œ_È°. r=<£, =~¡°}Ÿ H›O>è Z`Ç°ë 1) H©ië 2) ªÞu
`Ç䛽ø=, JaÀ+H± H›O>è Z`Ç°ë H›"Œ_È°. 3) ‰õÞ`Ç 4) ‹¬=¶KŒ~¡O ‹¬i‡é^ΰ
(Ì„á ‹¬ = ¶KŒ~¡ O P^ ¥ ~¡ O Qê D ãH÷ O k ㄬ ‰ × ß ä› ½ 13. Qˇ…˜, "³¶‚¬ì<£H›O>è ̄^Îí"Œ_È° Hêx ~Œ"£° H›O>è
‹¬=¶^¥<Œ° W=ÞO_.) z#ß"Œ_È°. "³¶‚¬ì<£, ªé‚¬ì<£ H›O>è ̄^Îí"Œ_È°, Hêx ~Œ"£°
1. JO^Îi…ÕH÷ ‡Ú\÷“ =¼H÷ë Z=~¡°? H›O>è z#ß"Œ_È°. JO^ÎiH›O>è ̄^Îí"Œ_È° Z=~¡°?
1) =~¡°}Ÿ 2) J[†Ÿ° 3) kb„¹ 4) r=<£ 1) Qˇ…˜ 2) "³¶‚¬ì<£ 3) ~Œ"£° 4) ªé‚¬ì<£
2. "Œi"Œi Z`Ç°ë ã„¬Hê~¡O x°óO>è P =~¡°‹¬…Õ =°^Î¼…Õ

Y
14. P~¡° „¬@“}=ò…Õ 1) ^ζe†Ç¶, J=°~Ÿ<Í~Ÿ H›<Œß ̄^Îík.
=ôO_Í =¼H÷ë Z=~¡°? 2) N~ŒO„¬î~Ÿ, <Œ‹²H± H›<Œß ̄^Îík. 3) [QêO,

M
1) JaÀ+H± 2) r=<£ 3) =~¡°}Ÿ 4) kb„¹ N~ŒO„¬î~Ÿ H›<Œß ̄^Îk
í Hê#„¬æ\÷H÷ J=°~Ÿ<~
Í HŸ <
› Œß ̄^Îk
í .
3. r=<£ H›O>è ‡Ú_È"³á#, J[†Ÿ° H›O>è ‡Ú\÷“Qê =ô#ß =¼H÷ë

E
4) J=°~Ÿ<Í~Ÿ, <Œ‹²H±H›<Œß z#ßk, Hêh =°<Œà_£ H›<Œß
Z=~¡°?
̄^Îík. Ju z#ß^³á# „¬@“}=ò Uk?

D
9
1) =~¡°}Ÿ 2) kb„¹ 3) JaÀ+H± 4) J[†Ÿ°

59
1) J=°~Ÿ<Í~Ÿ 2) <Œ‹²H±
4. JO^Îi…ÕH÷ ‡Ú_È"³á#"Œ_È° Z=~¡°?
CA
95
1) kb„¹ 2) r=<£ 3) =~¡°}Ÿ 4) J[†Ÿ° 3) N~ŒO„¬î~Ÿ 4) =°<Œà_£
15. XH›"Íˆ× ‰×Ãã䛽_È° JOQê~¡ä›½x H›<Œß HêOu=O`Ç"³°Ø`Í,

50
5. ‡Ú_È=ô „¬~¡OQê ï~O_È= Ju `Ç䛽ø= ‡Ú_È=ô Q®e¾# =¼H÷ë
JOQê~¡ä›½_È° |$‚¬ì‹¬æu H›<Œß HêOu=O`Ç°_È° Hê#@Á~ò`Í,
98
A
Z=~¡°?
|$‚¬ì‹¬æu ‰×Ãã䛽xH›<Œß HêOu=O`Ç°_³<
á Œ „¬î\
Á Õ H›<Œß HêOu
:8
1) r=<£ 2) =~¡°}Ÿ 3) JaÀ+H± 4) kb„¹
E

‹¬¶KÇ# (6-10) : XH› H›Oªe =^Îí "Íö~Þ~¡° |~¡°=ô° ‚Ôì#°_³á`Í JO^ÎiH›<Œß HêOu=O`Ç°_È° Z=~¡°?
h

;
1) |$‚¬ì‹¬æu 2) JOQê~¡ä›½_È°
.P

H›e¾# A, B, C, D =°i†Çò E J#° J~ò^ΰ W#°„¬


IC

3) ‰×Ãã䛽_È° 4) „¬îÁ\Õ
AD

=‹¬°ë=ô° H›=ô.
i) A †³òH›ø |~¡°=ô B |~¡°=ô H›<Œß ï~\÷“O„¬ô. ‹¬¶KÇ#° (16-18): D ãH÷O^Î "ŒY¼#° „¬ijeOz ‹¬=¶^¥#=ò°
AB
O

ii) B †³òH›ø |~¡°=ô C |~¡°=ô H›<Œß <Œ°Q®°#ß~¡ ï~@°Á H›#°Qù#°=ò.


ER
CH

JkH›O. ~Œ"£°, ~¡‚ÔìO H›<Œß "ÍQ®OQê „¬~¡°ïQ`Ç°ë`Œ_È°. Hêx *ÿá<£ H›<Œß


YD

iii) C †³òH›ø |~¡°=ô D |~¡°=ô…Õ ‹¬Q®O. =°O^ÎHù_Qê „¬~¡°ïQ`Íë „Ô@~Ÿ H›<Œß "ÍQ®OQê „¬~¡°ïQ`Çë…è_È°.
iv) D †³òH›ø |~¡°=ô E |~¡°=ô…Õ ‹¬Q®O.
,H

16. J`ǼO`Ç "ÍQ®OQê „¬~¡°ïQ`Íë"Œ_È° Z=~¡°?


T

v) E †³òH›ø |~¡°=ô A H›<Œß `Ç佛 ø= =°i†Çò C H›<Œß Z䛽ø=. 1) *ÿá<£ 2) „Ô@~Ÿ 3) ~Œ"£° 4) ~¡‚ÔìO
AM

17. Ju <³=°àkQê „¬~¡°ïQ`Íë"Œ_È° Z=~¡°?


H

6. JxßO\÷…ÕH÷ `Ç䛽ø= |~¡°=ô H›ey#k?


M

1) E 2) D 3) B 4) C 1) *ÿá<£ 2) „Ô@~Ÿ 3) ~Œ"£° 4) ~¡‚ÔìO


AM
IG

18. „¬~¡°Q®° ‡é\©…Õ ï~O_È= ªÖ#O…Õ Z=~¡° x°ªë~¡°?


7. †³òH›ø |~¡°=ô D ãH÷Ok "Œx…Õ "Í\ ÷H›<Œß `Ç䛽ø=?
KH

1) A & B 2) A & C 3) D & C 4) B & C 1) *ÿá<£ 2) „Ô@~Ÿ 3) ~Œ"£° 4) ~¡‚ÔìO


R

8. E †³òH›ø |~¡°=ô D ãH÷Ok "Œx…Õ "Í\ ÷H›<Œß Z䛽ø=? 19. S^ΰ =°Ok JƒìÄ~ò°<Œß~¡°. =°<˂¬ì~Ÿ H›<Œß =‹¬O`Ÿ
1) A & B 2) D & C 3) A & D 4) B & C ‡Ú_ÈQi ® Hêx ~ŒA JO`Ç ‡Ú_È=ô Hê^ΰ. ^Î`Œë H›<Œß [†Ç°O`Ÿ
9. JxßO\÷…ÕH÷ |~¡°"³á#k ? ‡Ú_È"³á#"Œ_È°, Hêx =°<˂¬ì~Ÿ H›<Œß ‡Ú\÷“. Z=~¡° P
1) E 2) C 3) B 4) A ãQ®¶„¬ô…Õ ‡Ú_È"³á#"Œ_È° ?
10. |~¡°=ô „¬~¡OQê J=~˂¬ì} ãH›=°O#° ‹¬¶zOKÇ°#k? 1) ~ŒA 2) =°<˂¬ì~Ÿ 3) =‹¬O`Ÿ 4) ^Î`Œë
1) A, B, D, E, C 2) E, C, D, A, B 20. S^ΰ l…ìÁ…ÕÁ „¦¬`̈́¬î~Ÿ H›<Œß JH›Ä~Ÿ„¬î~Ÿ z#ßk. ‡=ò
3) B, D, A, E, C 4) A, B, E, D, C H›<Œß ^Î<£ƒì^£ ̄^Îík =°i†Çò „¦¬`̈́¬î~Ÿ H›<Œß ƒì~Œ|OH÷
11. B H›O>è A ^Î#=O`Ç°_È°. A H›O>è C ^Î#=O`Ç°_È°. C ̄^Îík Hêx ‡=ò JO`Ç Ì„^ÎíkHê^ΰ. Jū^Îí l…ìÁ
H›<Œß D ^#
Î =O`Ç°_È° JO^ÎiH›<Œß E ^#
Î =O`Ç°_È°. WO^ΰ…Õ Uk?
=°^Î¼ ªÖ#=ò#° ‡ÚOk#^³=~¡°? 1) ƒì~Œ|OH÷ 2) „¦¬`̈́¬î~Ÿ
1) A 2) C 3) B 4) D 3) ‡=ò 4) ^Î<£ƒì^£
REASONING ABILITY (VERBAL) ○ ○ ○ ○ ○ ○ ○ ○ ○
123
○ ○ ○ ○ ○ ○ ○ ○ ○ ○ ○ ○ ○ ○ ○ ○ ○ ○ ○
Study Material
1234567890123456789012
12345678901234567890123456789012123456789012345678901234567890121234567890123456789012345678901212345678901234567890123
12345678901234567890123
12345678901234567890123456789012123456789012345678901234567890121234567890123456789012345678901212345678901234567890123
1234567890123456789012 12345678901234567890123

q=~¡}ì`ÇàH› ["Œ|°°
1234567890123456789012
12345678901234567890123456789012123456789012345678901234567890121234567890123456789012345678901212345678901234567890123
12345678901234567890123
12345678901234567890123456789012123456789012345678901234567890121234567890123456789012345678901212345678901234567890123
1234567890123456789012 12345678901234567890123
12345678901234567890123456789012123456789012345678901234567890121234567890123456789012345678901212345678901234567890123
1234567890123456789012
12345678901234567890123456789012123456789012345678901234567890121234567890123456789012345678901212345678901234567890123
1234567890123456789012 PRACTICE TEST - 1 : 12345678901234567890123
12345678901234567890123
12345678901234567890123456789012123456789012345678901234567890121234567890123456789012345678901212345678901234567890123
1234567890123456789012 12345678901234567890123
12345678901234567890123456789012123456789012345678901234567890121234567890123456789012345678901212345678901234567890123
1234567890123456789012 12345678901234567890123

; ‹¬¶KÇ# (1-5) : Wzó# ‹¬=¶KŒ~¡O ㄬHê~¡O, Z`Ç°ë 12. (4) ‰õÞ`Ç < ªÞu   ‰§ex < "³á‰§e   ‰§ex < H©ië̄á
ãH›=°O ~Œ†Ç°Qê D ãH÷Ok q^ÎOQê =ôO@°Ok. ‹¬=¶KŒ~¡O ‹¬i‡é^ΰ.
J[†Ÿ° > =~¡°}Ÿ > r=<£ > JaÀ+H± > kb„¹ (…èH›) 13. (3) "³¶‚¬ì<£ < Qˇ…˜ < ~Œ"£°  
kb„¹ < JaÀ+H± < r=<£ < =~¡°}Ÿ < J[†Ÿ°. ªé‚¬ì<£ < "³¶‚¬ì<£ < ~Œ"£°
̄á# KǶ„¬|_# ãH›=°O #°O_, ªé‚¬ì<£ < "³¶‚¬ì<£ < Qˇ…˜ < ~Œ"£°
1. JO^Îi…ÕH÷ ‡Ú\÷“ =¼H÷ë kb„¹.
(3) JO^Îi…Õ ̄^Îí"Œ_È° ~Œ"£°.
2. ̄á# KǶ„¬|_# ãH›=°O…Õ Z`Ç°ë ã„¬Hê~¡O =~¡°‹¬…Õ
(2) 14. (4) J=°~Ÿ<Í~Ÿ < ^ζe†Ç¶   <Œ‹²H± < N~ŒO„¬î~Ÿ  
=°^Î¼# Q® =¼H÷ë r=<£. J=°~Ÿ<Í~Ÿ < [QêO < N~ŒO„¬î~Ÿ
3. J[†Ÿ° > =~¡°}Ÿ > r=<£.
(1) =°<Œà_£ < J=°~Ÿ<Í~Ÿ < <Œ‹²H±
4. JO^Îi…ÕH÷ ‡Ú_È"³á# "Œ_È° J[†Ÿ°.
(4) =°<Œà_£ < J=°~Ÿ<Í~Ÿ =°i†Çò J=°~¡<Í~Ÿ H›<Œß
5. J[†Ÿ° > =~¡°}Ÿ > r=<£ > JaÀ+H± > kb„¹ ^ζe†Ç¶, N~ŒO„¬î~Ÿ, <Œ‹²H±, [QêO° ̄^Îíq.

Y
(3)

Ju `Ç䛽ø= ‡Ú_È=ô H›e¾# "³ò^Î\ ÷"Œ_È° kb„¹. Juz#ß „¬@“}O =°<Œà_£.


ï~O_È= Ju `Ç䛽ø= ‡Ú_È=ô H›e¾#"Œ_È° JaÀ+H±. 15. JOQê~¡ä›½_È° < ‰×Ãã䛽_È°   JOQê~¡ä›½_È° < |$‚¬ì‹¬æu  

M
(4)

; ‹¬¶KÇ# (6-10) : Wzó# ‹¬=¶KŒ~Œxß ãH›=°~¡¶„¬O…Õ ‰×Ãã䛽_È° < |$‚¬ì‹¬æu < „¬î\Á Õ   < JOQê~¡ä½› _È° < ‰×Ãã䛽_È°
~Œ†Ç°Qê A > B > E > D > C =KÇ°ó#°. < |$‚¬ì‹¬æu < „¦î¬ \Á Õ   JO^ÎiH›<Œß HêOu=O`Ç°_È° „¦î¬ \Á Õ.

E
̄á# KǶ„¬|_# ãH›=°O #°O_,

D
‹¬¶KÇ#° (16-18): ~¡‚ÔìO ~Œ=ò „Ô@~Ÿ *ÿá<£.

9
< < <

A > B > E > D > C #°O_, JxßO\÷H›<Œß `Ç䛽ø= 16.

59
6. (4) (1) J`ǼO`Ç "ÍQ®OQê „¬~¡°ïQ`Ç°ë"Œ_È° *ÿá<£.
17. CA
95
|~¡°=ô Q®k .
'C' (4) Ju <³=°àkQê „¬~¡°ïQ`Ç°ë"Œ_È° ~¡‚ÔìO.
7. A > B > E #°O_, 18. ï~O_È= ªÖ#O…Õ x°KÇ° =¼H÷ë „Ô@~Ÿ.

50
(1) (2)

\ E †³òH›ø |~¡°=ô A =°i†Çò B H›<Œß `Ç䛽ø=. 19. ~ŒA > =‹¬O`Ÿ > =°<˂¬ì~Ÿ 
98
A
(1)

8. (2) E > D > C #°O_, =°<˂¬ì~Ÿ > [†Ç°O`Ÿ > ^Î`Œë


:8
\ E †³òH›ø |~¡°=ô D =°i†Çò C H›<Œß Z䛽ø=. \ ~ŒA > =‹¬O`Ÿ > =°<˂¬ì~Ÿ > [†Ç°O`Ÿ > ^Î`Œë
E

9. A >B>E>D> C #°O_ JxßO\÷H›<Œß |~¡°"³á#k ® ~ŒA ‡Ú_È"³á#"Œ_È°.


h

'A'.
.P

(4)

10. Z䛽ø= ® `Ç䛽ø= « J=~˂¬ì}. A, B, E, D, C. 20. „¦¬`̈́¬î~Ÿ < JH›Ä~Ÿ„¬î~Ÿ   ^Î<£ƒì^£ < ‡O  
IC

(4) (4)

11. A > B : C > A : D > C, E JO^ÎiH›<Œß ^Î#=O`Ç°_È°. ƒì~Œ|OH÷ < „¦¬`̈́¬î~Ÿ


AD

(2)

E > D > C > A > B J=ô`Ç°Ok. =°^Î¼ªÖ#O ^<Î ƒ£ ì^£ < ‡O < ƒì~¡O|OH© < „¦`¬ „Í î¬ ~Ÿ < JH›Ä~Ÿ„î¬ ~Ÿ.
AB
O

‡ÚOk#"Œ_È° C. Jū^Îí l…ìÁ ^Î<£ƒì^£.


ER
CH

1234567890123456789012345678901212345678901234567890123456789012123
12345678901234567890123456789012123456789012345678901234567890121234567890123456789012345678901212345678901234567890123456789012123
123456789012345678901234567890121234567890123456789012345678901212345678901234567890123
12345678901234567890123456789012123456789012345678901234567890121234567890123456789012345678901212345678901234567890123
123456789012345678901234567890121234567890123456789012345678901212345678901234567890123456789012123456789012
1234567890123456789012345678901212345678901234567890123456789012123456789012345678901234567890121234
12345678901234567890123456789012123456789012345678901234567890121234567890123456789012345678901212345678901234567890123456789012123
123456789012345678901234567890121234567890123456789012345678901212345678901234567890123
1234567890123456789012345678901212345678901234567890123456789012123
12345678901234567890123456789012123456789012345678901234567890121234567890123456789012345678901212345678901234567890123
123456789012345678901234567890121234567890123456789012345678901212345678901234567890123456789012123456789012
1234567890123456789012345678901212345678901234567890123456789012123456789012345678901234567890121234
12345678901234567890123456789012123456789012345678901234567890121234567890123456789012345678901212345678901234567890123456789012123
123456789012345678901234567890121234567890123456789012345678901212345678901234567890123
12345678901234567890123456789012123456789012345678901234567890121234567890123456789012345678901212345678901234567890123
123456789012345678901234567890121234567890123456789012345678901212345678901234567890123456789012123456789012
1234567890123456789012345678901212345678901234567890123456789012123456789012345678901234567890121234
PREVIOUS BITS
1234567890123456789012345678901212345678901234567890123456789012123
12345678901234567890123456789012123456789012345678901234567890121234567890123456789012345678901212345678901234567890123456789012123
123456789012345678901234567890121234567890123456789012345678901212345678901234567890123
YD

12345678901234567890123456789012123456789012345678901234567890121234567890123456789012345678901212345678901234567890123
123456789012345678901234567890121234567890123456789012345678901212345678901234567890123456789012123456789012
1234567890123456789012345678901212345678901234567890123456789012123456789012345678901234567890121234
12345678901234567890123456789012123456789012345678901234567890121234567890123456789012345678901212345678901234567890123456789012123
123456789012345678901234567890121234567890123456789012345678901212345678901234567890123
1234567890123456789012345678901212345678901234567890123456789012123
12345678901234567890123456789012123456789012345678901234567890121234567890123456789012345678901212345678901234567890123
123456789012345678901234567890121234567890123456789012345678901212345678901234567890123456789012123456789012
1234567890123456789012345678901212345678901234567890123456789012123456789012345678901234567890121234
12345678901234567890123456789012123456789012345678901234567890121234567890123456789012345678901212345678901234567890123456789012123
123456789012345678901234567890121234567890123456789012345678901212345678901234567890123
1234567890123456789012345678901212345678901234567890123456789012123
12345678901234567890123456789012123456789012345678901234567890121234567890123456789012345678901212345678901234567890123
123456789012345678901234567890121234567890123456789012345678901212345678901234567890123456789012123456789012
1234567890123456789012345678901212345678901234567890123456789012123456789012345678901234567890121234
12345678901234567890123456789012123456789012345678901234567890121234567890123456789012345678901212345678901234567890123456789012123
123456789012345678901234567890121234567890123456789012345678901212345678901234567890123
12345678901234567890123456789012123456789012345678901234567890121234567890123456789012345678901212345678901234567890123456789012123
123456789012345678901234567890121234567890123456789012345678901212345678901234567890123
,H

(1-5) XH› „¬ô‹¬ëHê Ì+…˜æ…Õ '6Ñ „¬ô‹¬ëHꁰ L<Œß~ò. 5. WO^ΰ…Õ U „¬ô‹¬ëH›O ‡`Ç Q®l\˜ ?
T

1) E 2) D 3) B 4) A
AM

(A.P. S.I.Mains-2017)

A, B, C, D, E =°i†Çò F JO^ΰ…Õ ANSWERS


H
M

B, C, E „¬ô‹¬ëHꁰ P䛽„¬KÇó ~¡OQ®° H›=~Ÿ L#ßq, q°Q®`Œ (1-5) Wzó# ㄬ‰×ß#° *ìãQ®`ÇëQê „¬ijeÀ‹ë „¬ô‹¬ëHꁰ, Hù`Çëq,
AM
IG

=ü_È° „¬‹¬°„¬ô„¬KÇó ~¡OQ®° H›=~Ÿ L#ßq. ‡`Çq, „¬ô‹¬ëHê H›=~Ÿ ~¡OQ®°, =°i†Çò "Œ\÷ ‹¬ƒÿûH±“
A, B, D° Hù`Çë „¬ô‹Hë¬ ê° =°i†Çò q°Q®`Œ =ü_È° ‡`Ç „¬ô‹Hë¬ ê°.
KH

W=Þ_È"³°Ø#k. JO^ΰ=Á q^¥¼iÖ "Œ\÷x D H÷Ok


A, C, B ° …ì x"ÍkH›° =°i†Çò q°Q®`Œ =ü_È° Q®l\˜°.
R

q^ÎOQê J=°~¡°óHË"Œe.
1. D ãH÷Ok "Œ\÷…Õ U „¬ô‹¬ëH›O Hù`Çëk, „¬‹¬°„¬ô„¬KÇó H›=~Ÿ
Q®l\˜ …ì ‡`Çq Hù`Çëq P䛽„¬KÇó „¬‹¬°„¬ô
H›ey LO_ Q®l\˜ J~ò LO_È°#°?
1) B 2) D 3) E 4) F x"ÍkH›°
2. WO^ΰ…Õx U ï~O_È° „¬ô‹¬ëHꁰ, ‡`Ç Q®l\˜° J~ò A × 3 × 3 × 3
P䛽„¬KÇó ~¡OQ®° H›=~Ÿ L<Œß~ò? B × 3 × 3 3 ×
1) B, C 2) B, E 3) B, F 4) E XH›ø>è C × 3 3 × 3 ×
3. WO^ΰ…Õx U „¬ô‹¬ëH›O P䛽„¬KÇó ~¡OQ®° H›=~Ÿ LO_, ‡`Ç^³á D 3 × × 3 × 3
LOk. …ì x"ÍkH› Hê"Œe? E 3 × 3 × 3 ×
1) A 2) B 3) C 4) D F 3 × 3 × × 3
4. WO^ΰ…Õ Uk „¬‹¬°„¬ô„¬KÇó H›=~Ÿ, Hù`ÇëQê LO_, …ì x"ÍkH›
J~ò LO_È°#°? 1. (2) 2. (4) 3. (3)
1) A 2) B 3) C 4) D 4. (1) 5. (1)
REASONING ABILITY (VERBAL) ○ ○ ○ ○ ○ ○ ○ ○ ○
124
○ ○ ○ ○ ○ ○ ○ ○ ○ ○ ○ ○ ○ ○ ○ ○ ○ ○ ○
Study Material
…ìlH›…˜ "³<£ zã`Œ°
11 (Logical Venn Diagrams)

=òMì¼O‰§°...
G D qƒìQ®O…Õ ㄬ‰×߁䛽 ‹¬=¶^¥<Œ° H›#°Qù#\ìxH÷ Q®}÷`Ç *ìý#O J=‹¬~¡O …è^ΰ. D qƒìQ®O…Õ ㄬ‰×߁° Jƒ’¼iÖ †³òH›ø
q‰õÁ+¬} ª=°~¡Ö¼=ò#° JOKÇ<Œ "͆Çò@䛽 `Ë_Èæ_È`Œ~ò. Jƒ’¼iÖ P…ÕKÇ<Œ ‰×H÷ë, q‰õÁ+¬} ª=°~¡Ö¼O`Ë ‹¬°°=ôQê ‹¬=¶^¥<Œ°
H›#°Qù#=KÇ°ó.
=¶ki ㄬ‰×߁°

Y
M
TYPE - 1 TYPE - 2

E
D
9
59
"³ò^Î\÷ ~¡H›O …ìlH›…˜ "³<£ zã`Œ°
ï~O_È= ~¡H›O …ìlH›…˜ "³<£ zã`Œ°

G D „¬^uœÎ …Õ Hùxß zã`Œ° Wzó, ^¥x ãHO÷ k Hùxß „¬^¥° G D ~¡HO› ㄉ¬ ß× …ÕÁ XH› "³<£ zã`OÇ Wªë~°¡ . P "³<£ zã`OÇ …Õ
CA
95
…è^¥ "ŒH꼁° Wªë~¡°. ^¥xH÷ ‹¬=¶^¥#OQê P „¬^¥ L#ß qq^Î *ì¼q°f†Ç° zã`Œ° qq^Î JO‰§ä›½

50
‹¬=ü‚¬ìO#䛽 ‹¬i‡é†Í° zã`ÇO#° "³<£ zã`Œ #°O_ K³Ok#qQê LO\ì~ò. P "³<£ zã`Œxß „¬îiëQê
98
A
Q®°iëOKÇ=…ÿ#°. J=Qꂬì# K͋¬°Hùx, ^¥xH÷ ‹¬O|OkOz# ㄬ‰×߁䛽
:8
`Çy# ‹¬=¶^¥#O Q®°iëOKŒe.
E
h

L^¥‚¬ì~¡}° :
.P

L^¥‚¬ì~¡}° :
IC

D ãH÷Ok "³<£ zã`ÇO…Õ 3 ~¡Hê =°#°+¬µ° L<Œß~¡°.


AD

ãuƒ’°[O ‹¬¶ø…˜ \©KÇ~¡Á#°, KÇ`Ç°~¡ã‹¬O ̄ˆ×¤~ò# "Œix,


AB
O

=$`ÇOë L=°à_ 䛽@°O|O…Õ x=‹²OKÍ "Œix ‹¬¶zªë~ò.


ER
CH

(A) (B)
F
YD

E B C
,H

D
A
T
AM

1. L=°à_ 䛽@°O|O…Õ LO@¶ q"Œ‚¬ì"³°Ø, ‹¬¶ø° \©KÇ~¡°Á


H
M

Hêx "Œix ‹¬¶zOKÍ ã‡O`ÇO Uk?


(C) (D) (E)
D ㄉ¬ ß× …Õ L=°à_ 䛽@°O|O…Õ L#ß q"Œ‚¬ì"³°#Ø JO>è
AM
IG

̄á zã`Œ #°O_ ãH÷Ok Wzó# ㄬ‰×߁䛽 ‹¬i‡é†Í° ª^Î#:


KÇ`Ç°~¡ã‹¬O…Õ, =$`ÇëO…Õ LO_¨e. Hêx "Œ~¡° \©KÇ~¡°Á
KH

zã`ÇO#° Q®°iëOKÇO_. Hê~Œ^ΰ. JO>è ãuƒ’°[O…Õ LO_È䛀_È^ΰ. P ㇐O`ÇO


R

1. †Ç¶„²…˜, =¶q°_, ^¥x=°à 'D' J=ô`Ç°Ok.


ª^Î#: †Ç¶„²…˜, =¶q°_, ^¥x=°à =ü_È° "Í~ö Þ~¡° „¬O_ÈÁ ~¡Hꁰ 2. L=°à_ 䛽@°O|O…Õ LO@¶ ̄o¤Hêx "Œ~¡°, ‹¬¶ø°

Hê=ô# zã`ÇO ‹¬i J~ò#k. \©KÇ~¡°Á Hêx "Œix ‹¬¶zOKÍ ã‡O`ÇO Uk?
D ㄬ‰×ß…Õ Ì„o¤Hêx"Œ~¡°, \©KÇ~¡°Á Hêx "Œ~¡° JO>è
'E'

ª^Î#:
2.
KÇ`Ç°~¡ã‹¬O…Õ Hêh, ãuƒ’°[O…ÕQê<Í LO_È䛀_È^ΰ. JO>è
P‹²†Ç¶, _bÁ, WO_†Ç¶

ª^Î#: P‹²†Ç¶…Õ WO_†Ç¶ LOk. WO_†Ç¶…Õ _bÁ LOk. =$`ÇëO…Õ =¶ã`Ç"Í° LO_Í ã‡O`ÇO J=ô`Ç°Ok.
Hê=ô# zã`ÇO ‹¬i J~ò#k.
'A'
3. L=°à_ 䛽@°O|O…Õ LO@¶, q"Œ‚¬ì"³°#
Ø L‡^¥¼†Çò#°
'E'

3. …ì†Ç°~¡°Á, ¢‹Ô끰, _¨H›“~¡°Á


‹¬¶zOKÇ° ㇐O`ÇO Uk?
ª^Î#: =°‚²ìˆ×° HùO^Î~¡° …ì†Ç°~¡°ÁQê, _¨H›“~¡°ÁQê ƒì^Î¼`ǁ° ª^Î#: D ㄬ‰×ß…Õ L=°à_ 䛽@°O|O…Õ LO@¶ q"Œ‚¬ì"³°Ø#
„¬OKǰ䛽O\ì~¡°. Hê=ô# zã`ÇO ‹¬i J~ò#k.
'B' L‡^¥¼†Çò° JO>è =ü_È° ㇐O`Œ° H›e‹²Ok. JO>è
J=ô`Ç°Ok.
'B'

REASONING ABILITY (VERBAL) ○ ○ ○ ○ ○ ○ ○ ○ ○


125
○ ○ ○ ○ ○ ○ ○ ○ ○ ○ ○ ○ ○ ○ ○ ○ ○ ○ ○
Study Material
123456789012345678901234567890121234567890123456789012345678901212
12345678901234567890123456789012123456789012345678901234567890121234567890123456789012345678901212345678901234567890123
1234567890123456789012345678901212345678901234567890123456789012123456789012345678901234567890121234567890123
1234567890123456789012345678901212345678901234567890123456789012123456789012345678901234567890121234
12345678901234567890123456789012123456789012345678901234567890121234567890123456789012345678901212345678901234567890123456789012123
1234567890123456789012345678901212345678901234567890123456789012123456789012345678901234567890
12345678901234567890123456789012123456789012345678901234567890121234567890123456789012345678901212345678901234567890123
1234567890123456789012345678901212345678901234567890123456789012123456789012345678901234567890121234567890123
1234567890123456789012345678901212345678901234567890123456789012123456789012345678901234567890121234
123456789012345678901234567890121234567890123456789012345678901212
12345678901234567890123456789012123456789012345678901234567890121234567890123456789012345678901212345678901234567890123456789012123
1234567890123456789012345678901212345678901234567890123456789012123456789012345678901234567890
12345678901234567890123456789012123456789012345678901234567890121234567890123456789012345678901212345678901234567890123
1234567890123456789012345678901212345678901234567890123456789012123456789012345678901234567890121234567890123
1234567890123456789012345678901212345678901234567890123456789012123456789012345678901234567890121234
12345678901234567890123456789012123456789012345678901234567890121234567890123456789012345678901212345678901234567890123456789012123
1234567890123456789012345678901212345678901234567890123456789012123456789012345678901234567890
123456789012345678901234567890121234567890123456789012345678901212
PRACTICE TEST - 1
12345678901234567890123456789012123456789012345678901234567890121234567890123456789012345678901212345678901234567890123
1234567890123456789012345678901212345678901234567890123456789012123456789012345678901234567890121234567890123
1234567890123456789012345678901212345678901234567890123456789012123456789012345678901234567890121234
12345678901234567890123456789012123456789012345678901234567890121234567890123456789012345678901212345678901234567890123456789012123
1234567890123456789012345678901212345678901234567890123456789012123456789012345678901234567890
123456789012345678901234567890121234567890123456789012345678901212
12345678901234567890123456789012123456789012345678901234567890121234567890123456789012345678901212345678901234567890123
1234567890123456789012345678901212345678901234567890123456789012123456789012345678901234567890121234567890123
1234567890123456789012345678901212345678901234567890123456789012123456789012345678901234567890121234
12345678901234567890123456789012123456789012345678901234567890121234567890123456789012345678901212345678901234567890123456789012123
1234567890123456789012345678901212345678901234567890123456789012123456789012345678901234567890
12345678901234567890123456789012123456789012345678901234567890121234567890123456789012345678901212345678901234567890123
1234567890123456789012345678901212345678901234567890123456789012123456789012345678901234567890121234567890123
1234567890123456789012345678901212345678901234567890123456789012123456789012345678901234567890121234
12345678901234567890123456789012123456789012345678901234567890121234567890123456789012345678901212345678901234567890123456789012123
1234567890123456789012345678901212345678901234567890123456789012123456789012345678901234567890
123456789012345678901234567890121234567890123456789012345678901212
12345678901234567890123456789012123456789012345678901234567890121234567890123456789012345678901212345678901234567890123456789012123
1234567890123456789012345678901212345678901234567890123456789012123456789012345678901234567890
12345678901234567890123456789012123456789012345678901234567890121234567890123456789012345678901212345678901234567890123
1234567890123456789012345678901212345678901234567890123456789012123456789012345678901234567890121234567890123
1234567890123456789012345678901212345678901234567890123456789012123456789012345678901234567890121234

‹¬¶KÇ# : (1-5) D ãH÷Ok W=Þ|_# ㄬ‰×߁†Ç°O^ΰ 3 „¬^Î=ò° 4. |‚¬•ƒ’°l, n~¡ÉKÇ`Ç°~¡ã‹¬=ò, KÇ`Ç°~¡ã‹¬=ò


W=Þ|_#q. ‹¬=¶^¥#=ò…Õ <Œ°Q®° "³<£ zã`Ç=ò°
W=Þ|_#q. „¬^Î=òä›½ ‹¬O|OkOz# "³<£ zã`Ç=ò#° 1) 2)
Q®°iëOKÇO_.

3) 4)
1. ƒì°~¡°, ãH©_¨Hê~¡°°, WO[h~¡°Á

1) 2) 3) 4) 5. ‡ÚQ®ã`ŒQ®°"Œ~¡°, <Œ¼†Ç°"Œ^ΰ°, ‡ÚQ® ã`ŒQ®x"Œ~¡°

2. „¬ô~¡°+¬µ°, ¢‹Ô끰, Q®°=°ªë°

1) 2)

Y
1) 2) 3) 4)

M
3. =òY¼=°Oãu, ‰§‹¬#‹¬ƒ’°¼_È°, ~Œ[¼‹¬ƒ’ ‹¬ƒ’°¼_È°

3) 4)

E
1) 2) 3) 4)

D
9
59
1234567890123456789012
12345678901234567890123456789012123456789012345678901234567890121234567890123456789012345678901212345678901234567890123
12345678901234567890123

q=~¡}ì`ÇàH› ["Œ|°°
12345678901234567890123456789012123456789012345678901234567890121234567890123456789012345678901212345678901234567890123
1234567890123456789012
1234567890123456789012 12345678901234567890123
12345678901234567890123456789012123456789012345678901234567890121234567890123456789012345678901212345678901234567890123
12345678901234567890123
1234567890123456789012
PRACTICE TEST - 1 :
CA
12345678901234567890123
12345678901234567890123456789012123456789012345678901234567890121234567890123456789012345678901212345678901234567890123

95
1234567890123456789012
12345678901234567890123456789012123456789012345678901234567890121234567890123456789012345678901212345678901234567890123
1234567890123456789012 12345678901234567890123
1234567890123456789012 12345678901234567890123
12345678901234567890123456789012123456789012345678901234567890121234567890123456789012345678901212345678901234567890123
12345678901234567890123
12345678901234567890123456789012123456789012345678901234567890121234567890123456789012345678901212345678901234567890123

HùO^Î~¡° ƒì°~¡° ãH©_¨Hê~¡°Á, ~Œ[¼‹¬ƒ’ ‹¬ƒ°’ ¼_È° ‡~¡"Á °³ O\˜…Õ Š¨Œ­$Œ­„“±½`ɱ \½Œ­„“
50
1. (4)
98
A
WO[h~¡°Á J=ô`Œ~¡°. HùO^Î~¡°l l ƒìQ®=ò Hê=ô# „¬@=ò (2)
XP\i¢±Â „킱¢±
:8
WO[h~¡Á…Õ ƒì°~¡°, ãH©_¨Hê~¡° ‹¬i†³Ø°#k.
E
h

°O^ΰ~¡°. ãH©_¨Hê~¡°…ÕÁ HùO^Î~¡° (1) |‚¬•ƒ°’ A…Õ n~¡K É `Ç °Ç ~¡ã‹O¬ ,


.P

>óZ½>±Päv
IC

ƒì°~¡°, WO[h~¡°ÁO^ΰ~¡°. Hê=ô#


4.

KÇ`Ç°~¡ã‹¬O LO\ì~ò. Hùxß LÈaȱ¢äŒ­P }ƒ­–„“±m


AD

n
„¬@=ò (4) ‹¬i†³Ø°#k. äIª`©I뢱‚±
n~¡ÉKÇ`Ç°~¡ãª° KÇ`Ç°~¡ãª°
AB
O

(3) „¬ô~°¡ +¬µ°, ¢‹


ëÔ ° eOQ®„~¬ O¡ Qê 䛀_¨ J=ô`Œ~ò. Hê=ô#
ER
CH

2.

"Í~¡°. Q®°=°ªë…Õ HùO^Î~¡° „¬@=ò (1) ‹¬i†³Ø°#k.


R¯±>±«‘삱
YD

l
o¢ÊLÈaȱ¢äŒ­P
¢‹Ô끰, =°iHùO^Î~¡° „¬ô~¡°+¬µ (2) ‡ÚQ®ã`ŒQ®°"Œ~¡°, ã`ŒQ®x"Œ~¡°
,H

°O\ì~¡°. Hê=ô# „¬@=ò (3)


5.

qƒ’#ß =¼ä›½ë°.
T

=½‡È±#_ϱ‚±
AM

n n
‹¬i†³Ø°#k. <Œ¼†Ç°"Œ^ΰ…Õ HùO^Î~¡° l
H

£ŒÕ삱 …­õ ¢ ±,­¶ ‚±


M

‡ÚQ®ã`ŒQ®°"Œ~¡°, HùO^Î~¡°
AM

(2) ‰§‹¬# ‹¬ƒ’°¼…Õ XH›~¡°


IG

3.
ã`ŒQ®x "Œ~¡°O^ΰ~¡° Hê=ô# ˆÛR¯äaR¯n±#¢± ˆÛR¯n
=òY¼=°Oãu J=ô`Œ_È ° . „¬@=ò (2) ‹¬i†³Ø°#k.
KH

äaR¯y#¢±
123456789012345678901234567890121234567890123456789012345678901212
R

12345678901234567890123456789012123456789012345678901234567890121234567890123456789012345678901212345678901234567890123456789012123
1234567890123456789012345678901212345678901234567890123456789012123456789012345678901234567890
123456789012345678901234567890121234567890123456789012345678901212
12345678901234567890123456789012123456789012345678901234567890121234567890123456789012345678901212345678901234567890123
1234567890123456789012345678901212345678901234567890123456789012123456789012345678901234567890121234567890123
1234567890123456789012345678901212345678901234567890123456789012123456789012345678901234567890121234
12345678901234567890123456789012123456789012345678901234567890121234567890123456789012345678901212345678901234567890123456789012123
1234567890123456789012345678901212345678901234567890123456789012123456789012345678901234567890
12345678901234567890123456789012123456789012345678901234567890121234567890123456789012345678901212345678901234567890123
1234567890123456789012345678901212345678901234567890123456789012123456789012345678901234567890121234567890123
1234567890123456789012345678901212345678901234567890123456789012123456789012345678901234567890121234
123456789012345678901234567890121234567890123456789012345678901212
12345678901234567890123456789012123456789012345678901234567890121234567890123456789012345678901212345678901234567890123456789012123
1234567890123456789012345678901212345678901234567890123456789012123456789012345678901234567890
12345678901234567890123456789012123456789012345678901234567890121234567890123456789012345678901212345678901234567890123
1234567890123456789012345678901212345678901234567890123456789012123456789012345678901234567890121234567890123
1234567890123456789012345678901212345678901234567890123456789012123456789012345678901234567890121234
PRACTICE TEST - 2
12345678901234567890123456789012123456789012345678901234567890121234567890123456789012345678901212345678901234567890123456789012123
1234567890123456789012345678901212345678901234567890123456789012123456789012345678901234567890
123456789012345678901234567890121234567890123456789012345678901212
12345678901234567890123456789012123456789012345678901234567890121234567890123456789012345678901212345678901234567890123
1234567890123456789012345678901212345678901234567890123456789012123456789012345678901234567890121234567890123
1234567890123456789012345678901212345678901234567890123456789012123456789012345678901234567890121234
12345678901234567890123456789012123456789012345678901234567890121234567890123456789012345678901212345678901234567890123456789012123
1234567890123456789012345678901212345678901234567890123456789012123456789012345678901234567890
123456789012345678901234567890121234567890123456789012345678901212
12345678901234567890123456789012123456789012345678901234567890121234567890123456789012345678901212345678901234567890123456789012123
1234567890123456789012345678901212345678901234567890123456789012123456789012345678901234567890
12345678901234567890123456789012123456789012345678901234567890121234567890123456789012345678901212345678901234567890123
1234567890123456789012345678901212345678901234567890123456789012123456789012345678901234567890121234567890123
1234567890123456789012345678901212345678901234567890123456789012123456789012345678901234567890121234
12345678901234567890123456789012123456789012345678901234567890121234567890123456789012345678901212345678901234567890123456789012123
1234567890123456789012345678901212345678901234567890123456789012123456789012345678901234567890
12345678901234567890123456789012123456789012345678901234567890121234567890123456789012345678901212345678901234567890123
1234567890123456789012345678901212345678901234567890123456789012123456789012345678901234567890121234567890123
1234567890123456789012345678901212345678901234567890123456789012123456789012345678901234567890121234
12345678901234567890123456789012123456789012345678901234567890121234567890123456789012345678901212345678901234567890123456789012123
1234567890123456789012345678901212345678901234567890123456789012123456789012345678901234567890

‹¬¶KÇ# : (1-5) D kQ®°= W=Þ|_# ㄬu ㄬ‰×߅Õ#° =ü_͋² 1. KÇOã^ΰ_È°, ƒ’¶q°, q‰×Þ=ò.

„¬^¥° W=Þ_È"³°Ø#k. Jƒ’¼~¡°Ö° D =ü_È° „¬^¥ =°^Î¼ 2. WO_†Ç¶, ‡H÷ªë<£, P‹²†Ç¶.


3. „¬ô‹¬ëH›=ò, ̄<£, ̄xž…˜.
‹¬O|O^Î=ò#° ‹¬¶zOKÍ =$`Çë ‹¬=ü‚¬ì=ò#° Wzó# (1),
4. 䛽H›ø°, „²°Á°, ̄O„¬ô_È° [O`Ç°=ô°.
(2), (3), (4), (5)…Õx =$`Çë ‹¬=ü‚¬ì=ò #°Oz
5. Q®°_ÈÁQ®¶|°, „¬ä›Æ½°, U#°Q®°°.
Z#°ßHùx ‹¬i†³Ø°# ‹¬=¶^¥#=ò#° ‹¬¶zOKÇ=…ÿ#°.
‹¬¶KÇ# : (6-10) D kQ®°= W=Þ|_# ㄬu ㄬ‰×߅Õ#° =ü_͋²
„¬^¥° W=Þ_È"³°Ø#k. Jƒ’¼~¡°Ö° D =ü_È° „¬^¥ =°^Î¼

(1) (2) (3) (4) (5) ‹¬O|O^=


Î ò#° ‹¬¶zOKÍ =$`Çë ‹¬=ü‚¬ì=ò#°, Wzó# (1),

REASONING ABILITY (VERBAL) ○ ○ ○ ○ ○ ○ ○ ○ ○


126
○ ○ ○ ○ ○ ○ ○ ○ ○ ○ ○ ○ ○ ○ ○ ○ ○ ○ ○
Study Material
(2), (3), (4), (5), (6)…Õx =$`Çë ‹¬=ü‚¬ì=ò #°Oz 6. ã‡é\ì<£, Z¢Hê“<£, „¬~¡=¶}°=ô°
Z#°ßHùx "Œ\÷H÷ ‹¬=¶^¥#=ò#° ‹¬¶zOKÇ=…ÿ#°.
7. ‡éb‹¹ P„¦Ô‹¬~Ÿž, ¢‹Ô끰, r„¹
8. ‰×x, ãQ®‚¬ð°, ƒ’¶q°
(1) (2) (3)
9. „¬~¡=¶}°=ô, #¶¼ã\ì<£, #¶¼H÷Á†Ç°‹¹
10. ̋á<£ž, „¦²lH±ž, ïHq°¢‹Ô“
(4) (5) (6)

1234567890123456789012
12345678901234567890123456789012123456789012345678901234567890121234567890123456789012345678901212345678901234567890123
12345678901234567890123
12345678901234567890123456789012123456789012345678901234567890121234567890123456789012345678901212345678901234567890123
1234567890123456789012 12345678901234567890123
1234567890123456789012
1234567890123456789012
q=~¡}ì`ÇàH› ["Œ|°°
12345678901234567890123456789012123456789012345678901234567890121234567890123456789012345678901212345678901234567890123
12345678901234567890123
12345678901234567890123456789012123456789012345678901234567890121234567890123456789012345678901212345678901234567890123
PRACTICE TEST - 2 :
12345678901234567890123
12345678901234567890123456789012123456789012345678901234567890121234567890123456789012345678901212345678901234567890123
1234567890123456789012
12345678901234567890123456789012123456789012345678901234567890121234567890123456789012345678901212345678901234567890123
1234567890123456789012
12345678901234567890123
12345678901234567890123
12345678901234567890123456789012123456789012345678901234567890121234567890123456789012345678901212345678901234567890123
1234567890123456789012 12345678901234567890123

KÇOã^ΰ_È°, ƒ’¶q° J<Í ï~O_È° "Íö~Þ~¡° ãQ®¶„¬ôä›½ ã‡é\ì<£ J#Qê ^Î#q^ΰ¼^¥"͉×=ò, Z¢Hê“<£ J#Qê

Y
1. (2) 6. (4)
‹¬O|OkOz#q. Hêx P ï~O_È° „¬^¥° 'q‰×Þ=òÑ |°°}q^ΰ¼^¥"͉×=ò Q® H›}쁰 H›#°H› Wq ï~O_È°

M
ãQ® ¶ „¬ ô ä› ½ ‹¬ O |Ok O z#q. JO^Î ° =# =°# "Íö~Þ~¡°. Hêx Wq ï~O_È° H›e„² „¬~¡=¶}°=ô ãQ®¶„¬ô#䛽
‹¬=¶^¥#=ò (2) J=ô`Ç°Ok. ‹¬O|OkOz#q. JO^ΰ=# =°# ‹¬=¶^¥#=ò (4)

E
J=ô`Ç°Ok.

D
9
„“·r±

59
LÈPä_ϱ`ɱ
rŠØß>ó …­¢>·~±>õ
CA
95
äˆê]í=¤
(2) WO_†¶Ç , ‡H÷ªë<£ J<Í ~ï O_È° „¬^¥° "Í~ö Þ~¡° ^͉§xH÷ [‚£Ië”=¤

50
2.

‹¬O|OkOz#q. Hêx P ï~O_È° ^͉§° P‹²†Ç¶ (6) HùO`Ç=°Ok ¢‹Ô끰 ‡éb‹¹ P„¦Ô‹¬~¡°Á J=ô`Œ~¡°. Hêx r„¹
98
A
7.

ƒ¶’ ƒìQêxH÷ ‹¬O|OkOz#q. H›#°H› =°# ‹¬=¶^¥#=ò J<Ík D ï~O_O\÷H÷ ‹¬O|OkOz# ãQ®¶„¬ô Hê^ΰ.
:8

(2) J=ô`Ç°Ok. JO^ΰ=# ‹¬=¶^¥#=ò (6) J=ô`Ç°Ok.


E
h
.P
IC

ˆ‘Iø«‘ì=¤ ˆêcŒ­±!Q…§ÕŒ­ Ÿ
AD

XP`Ž‡È·
£ŒÕ‚
ì ±

QŒ³‡È·
AB
O

„¬ô‹¬ëH›=ò, ̄<£, ̄xž…˜ J<Í D =ü_È° „¬^¥° "Íö~Þ~¡°


s…º

3. (5) (4) ‰×x, ƒ’¶q° J<Í ï~O_È° „¬^¥° "Íö~Þ~¡° ãQ®‚¬ðä›½


ER
CH

=‹¬°ë=ô° H›#°H› "Íö~Þ~¡° ãQ®¶„¬ô#䛽 ‹¬O|OkOz#q.


8.

‹¬O|OkOz#q. Hêx Wq ï~O_È° ãQ®‚¬ð ãQ®¶„¬ô䛽


YD

JO^ΰ=# =°# ‹¬=¶^¥#=ò (5) J=ô`Ç°Ok. K³Ok#q. JO^ΰ=# ‹¬=¶^¥#=ò (4) J=ô`Ç°Ok.
,H
T
AM

ͅ=¤
äR¯ƒ ­ñ‚±
ŠØy
H

…­õŒ­ìIœ>ó
„“·r±
M

ͅyŸ†™

䛽H›ø°, „²°Á° J<Í ï~O_È° „¬^¥° "Íö~Þ~¡° *ì`Ç°ä›½ D ㄉ¬ ß× ä›½ ‹¬=¶^¥#=ò H›#°Qù#°@䛽 HùOK³=ò ̋<á ž£
AM

(4) (5)
IG

9.

<Œ…ÿ_û J=‹¬~¡=ò. #¶¼ã\ì<£° #¶¼H÷Á†Ç°‹¹…Õ J=°i


4.

‹¬O|OkOz#q. Hêx Hùxß ä›½H›ø°, Hùxß „²°Á°


KH

̄O„¬ô_È° [O`Ç°=ôä›½ ‹¬O|OkOz#q. JO^ΰ=# LO\ì~ò. #¶¼H÷ Á † Ç ° ‹¹ „¬ ~ ¡ = ¶}°=ô…Õ J=°i


R

‹¬=¶^¥#=ò (4) J=ô`Ç°Ok. JO@°Ok. JO^ΰ=# ‹¬=¶^¥#=ò (5) J=ô`Ç°Ok.


…­¢>·~±>õ
ͅP…­õ`ɱ!\Paȱ>õ‚± $·½ä]í=¤ $·½Iø‡ȱŒº
圾Iœù‚± …³‚±Â‚±
(4) „¦l² H±ž, Hï q°¢‹“Ô J<Í ~ï O_È° „¬^¥° ̋<á ž£ 䛽 ‹¬O|OkOz#
Q®°_ÈÁQ®¶|° J<Íq „¬ä›Æ½ *ìuH÷ ‹¬O|OkOz#q.
10.

5. (3) "Í~ö Þ~¡° ‹¬ƒäûÿ ½› “ °. Hêh D ~ï O_È° „¬^¥° ̋<á ž£ ãQ¶® „¬ô佛
Hêx U#°Q®°° J<Íq D ï~O_O\÷H÷ ‹¬O|OkOz# ‹¬O|OkOz#q. JO^ΰ=# ‹¬=¶^¥#=ò (4)
ãQ¶® „¬ô Hê^ΰ. JO^ΰ=# ‹¬=¶^¥#=ò (3) J=ô`°Ç Ok. J=ô`Ç°Ok.
V$±R¯±‚±
R¯±`ÉÂR¯·}‚± …§³mI²Ÿ
͌â=¤Ÿ

…­åœÇ¾‚± ðI r±£ŒÕ”

REASONING ABILITY (VERBAL) ○ ○ ○ ○ ○ ○ ○ ○ ○


127
○ ○ ○ ○ ○ ○ ○ ○ ○ ○ ○ ○ ○ ○ ○ ○ ○ ○ ○
Study Material
123456789012345678901234567890121234567890123456789012345678901212
123456789012345678901234567890121234567890123456789012345678901212
12345678901234567890123456789012123456789012345678901234567890121234567890123456789012345678901212345678901234567890123456789012123
123456789012345678901234567890121234567890123456789012345678901212345678901234567890123
12345678901234567890123456789012123456789012345678901234567890121234567890123456789012345678901212345678901234567890123
1234567890123456789012345678901212345678901234567890123456789012123456789012345678901234567890121234567890123
1234567890123456789012345678901212345678901234567890123456789012123456789012345678901234567890121234
12345678901234567890123456789012123456789012345678901234567890121234567890123456789012345678901212345678901234567890123456789012123
123456789012345678901234567890121234567890123456789012345678901212345678901234567890123
12345678901234567890123456789012123456789012345678901234567890121234567890123456789012345678901212345678901234567890123
1234567890123456789012345678901212345678901234567890123456789012123456789012345678901234567890121234567890123
1234567890123456789012345678901212345678901234567890123456789012123456789012345678901234567890121234
123456789012345678901234567890121234567890123456789012345678901212
12345678901234567890123456789012123456789012345678901234567890121234567890123456789012345678901212345678901234567890123456789012123
123456789012345678901234567890121234567890123456789012345678901212345678901234567890123
12345678901234567890123456789012123456789012345678901234567890121234567890123456789012345678901212345678901234567890123
1234567890123456789012345678901212345678901234567890123456789012123456789012345678901234567890121234567890123
1234567890123456789012345678901212345678901234567890123456789012123456789012345678901234567890121234
PREVIOUS BITS
12345678901234567890123456789012123456789012345678901234567890121234567890123456789012345678901212345678901234567890123456789012123
123456789012345678901234567890121234567890123456789012345678901212345678901234567890123
123456789012345678901234567890121234567890123456789012345678901212
12345678901234567890123456789012123456789012345678901234567890121234567890123456789012345678901212345678901234567890123
1234567890123456789012345678901212345678901234567890123456789012123456789012345678901234567890121234567890123
1234567890123456789012345678901212345678901234567890123456789012123456789012345678901234567890121234
12345678901234567890123456789012123456789012345678901234567890121234567890123456789012345678901212345678901234567890123456789012123
123456789012345678901234567890121234567890123456789012345678901212345678901234567890123
123456789012345678901234567890121234567890123456789012345678901212
12345678901234567890123456789012123456789012345678901234567890121234567890123456789012345678901212345678901234567890123
1234567890123456789012345678901212345678901234567890123456789012123456789012345678901234567890121234567890123
1234567890123456789012345678901212345678901234567890123456789012123456789012345678901234567890121234
12345678901234567890123456789012123456789012345678901234567890121234567890123456789012345678901212345678901234567890123456789012123
123456789012345678901234567890121234567890123456789012345678901212345678901234567890123
12345678901234567890123456789012123456789012345678901234567890121234567890123456789012345678901212345678901234567890123
1234567890123456789012345678901212345678901234567890123456789012123456789012345678901234567890121234567890123
1234567890123456789012345678901212345678901234567890123456789012123456789012345678901234567890121234
12345678901234567890123456789012123456789012345678901234567890121234567890123456789012345678901212345678901234567890123456789012123
123456789012345678901234567890121234567890123456789012345678901212345678901234567890123
123456789012345678901234567890121234567890123456789012345678901212
12345678901234567890123456789012123456789012345678901234567890121234567890123456789012345678901212345678901234567890123456789012123
123456789012345678901234567890121234567890123456789012345678901212345678901234567890123
1234567890123456789012345678901212345678901234567890123456789012123456789012345678901234567890121234567890123
1234567890123456789012345678901212345678901234567890123456789012123456789012345678901234567890121234

(1-4) ãH÷Ok „¬\ìxß *ìãQ®`ÇëQê J^Î¼†Ç°#O K͋² 1 #°O_ 4 5. XH› qO^ΰ…Õ K̈́
¬ °, =¶O‹¬O =_O
Û KŒ~¡°. HùO^Î~°¡ K̈́
¬ °
=~¡ä›½ Q® ㄬ‰×߁䛽 ‹¬=¶^¥#=ò#° ~Œ†Çò=ò. =¶ã`Ç"°Í u#Qê, =°iHùO^Î~°¡ =¶O‹¬O =¶ã`Ç"°Í uO\ì~¡°.
(A.P.Constable Prelims -2016)
JO`ÍQêH› D ï~O_O\÷h JOwH›iOKÇx ‰§M사ð~¡°¶
" "
HùO^Î~¡°<Œß~¡°. q°ye# "Œ~¡° ï~O_O\÷h uO\ì~¡°. D
%
(
' - ) # „¬i‹²Öux ㄬuaOaOKÍ ‹¬i†³Ø°# zã`Ç~¡¶„¬O
(
(T.S S.I. Mains -2016)
"

WH›ø_È
1) ̄^Îí ãuƒ’°[=ò H›ˆìHê~¡°#° ‹¬¶zOKÇ°#°.
1) 2) 3) 4)

Y
2) z#ß ãuƒ’°[=ò ‰§G"Í`Çë#° ‹¬¶zOKÇ°#°. 6. HùO^Î~¡° ãH÷ïH\˜ P@Qêˆ×ä >ÿxߋ¹ P@Qêˆ×ä, HùO^Î~¡°

M
3) KÇ`Ç°~¡ã‹¬=ò #$`ǼHê~¡°#° ‹¬¶zOKÇ°#°. >ÿxߋ¹ P@Qêˆ×ä ‚¬ðH© P@Qêˆ×ä, U ãH÷ïH\˜ P@Qê_ȶ
4) =$`Çë=ò "³á^ΰ¼#° ‹¬¶zOKÇ°#°. ‚¬ðH© P@Qê_È° Hê^ΰ.

E
1. H›ˆìHê~¡°…Õ Q® "³á^ΰ¼° =°i†Çò #$`ǼHê~¡°#° ̄á ㄬ=KÇ<Œ#° ‹¬¶zOKÍ ‹¬i†³Ø°# zã`ÇO

D
9
=¶ã`Ç"Í° ‹¬¶zOKÇ° JH›Æ~¡=ò
1) 2) 3) 4)
(T.S S.I. Mains -2016)

59
A D G H

CA
95
2. H›ˆìHê~¡°…Õ ‰§G"Í`Ç끶, "³á^ΰ¼¶ Hêx "Œix ‹¬¶zOKÍ 1) 2) 3) 4)

50
JH›Æ~¡=ò°
1) 2) 3) 4) 98
A
A, B A, L B, G L, H
7. Jxß *ì¼q°f†Ç° zã`Œ…ÕÁ#¶ L#ß ‹¬OY¼
:8
3. H›ˆìHê~¡°…Õ #$`ǼHê~¡°¶ =°i†Çò "³á^ΰ¼¶ J~ò# (T.S S.I. Mains -2016)
E
h

"Œix ‹¬¶zOKÍ JH›Æ~¡=ò°



.P

1) 2) 3) 4)
IC

   
A, D C, A C, D G, H

AD

4. ‰§G"Í`Ç끰 J~ò=ôO_, H›ˆìHê~¡°° Hê#@°=O\÷ "Œix


 
AB
O

‹¬¶zOKÇ° JH›Æ~¡=ò.
1) 2) 3) 4) 1) 4 2) 5 3) 8 4) 3
ER
CH

B D F L

12345678901234567890123456789012
12345678901234567890123456789012123456789012345678901234567890121234567890123456789012345678901212345678901234567890123
12345678901234567890123456789012
YD

12345678901234567890123456789012123456789012345678901234567890121234567890123456789012345678901212345678901234567890123
12345678901234567890123456789012 12345678901234567890123456789012
12345678901234567890123456789012
12345678901234567890123456789012
12345678901234567890123456789012
12345678901234567890123456789012 q=~¡}ì`ÇàH› ["Œ|°° 12345678901234567890123456789012
12345678901234567890123456789012123456789012345678901234567890121234567890123456789012345678901212345678901234567890123
12345678901234567890123456789012
12345678901234567890123456789012123456789012345678901234567890121234567890123456789012345678901212345678901234567890123
12345678901234567890123456789012
12345678901234567890123456789012123456789012345678901234567890121234567890123456789012345678901212345678901234567890123
12345678901234567890123456789012
12345678901234567890123456789012123456789012345678901234567890121234567890123456789012345678901212345678901234567890123
,H

12345678901234567890123456789012 12345678901234567890123456789012
12345678901234567890123456789012123456789012345678901234567890121234567890123456789012345678901212345678901234567890123
T

̄á^¥x…Õ ̄^Îí ãuƒ’°[=ò H›ˆìHê~¡°#°, =°iHùO`Ç =°Ok K̈́¬ ° =¶ã`"Ç °Í u<Œß~¡°. WOHùO^Î~°¡
AM
H

v z#ß ãuƒ’°[=ò ‰§G"Í`Çë#°, ï~O_O\÷h u<Œß~¡°.


" "
M

%
qO^ΰ…Õ HùO`Ç=°Ok ‰§M사ð~¡°° 䛀_¨ L<Œß~¡°.
AM

v KÇ`Ç°~¡ã‹¬=ò #$`ǼHê~¡°#°,
(
IG

' - ) #
(

v =$`Çë=ò "³á^ΰ¼#° ‹¬¶zªë~ò. Hê|\÷“ g@O`Ç\÷H÷ ‹¬i„¬_È° "³<£ zã`ÇO


KH

"

(3) H›ˆìHê~¡°° J#Qê ̄^Îí ãuƒ’°[=ò…Õ„¬e "³á^ΰ¼°


R

1.
ðP`ŽP]øi!v=Î#¢±
J#Qê =$`Çë=ò…Õ„¬, #$`ǼHê~¡°°, KÇ`Ç°~¡ã‹¬=ò…Õ >·PŒ­P! >·P«‘ƒ­ñ¢±‚±
Q® D =ü_O\÷ H›~òH›…Õ Q® JH›Æ~¡=ò 'G' v$±#¢± L΅­‚±!v=Î#¢±
\ Hê=‹²# JH›Æ~¡=ò 'G'.

(1) H›ˆìHê~¡°…Õ ‰§G"Í`Ç끶, "³á^ΰ¼°Hêx "Œi J#Qê


Š¨N탭ñ¢±‚±
2.

̄^Îí ãuƒ°’ [=ò…Õ LO_, z#ß ãuƒ°’ [=ò (‰§G"Í`ëÇ °), (3) W=Þ|_# ‹¬=¶KŒ~ŒxH÷, ‹¬iQê¾ ‹¬i„¬_È° zã`ÇO
=$`Ç=ë ò ("³^á °Î ¼) …Õ …è佛 O_¨ LO_È°#@°=O\÷ L=°à_ 6.

JH›Æ~Œ° A, B. äIøðI]™ ?yàŒº ƒ­ñ Iª

(4) (3)
n~¡ÉKÇ`Ç°~¡ã‹¬O, =$`Çë=ò =°i†Çò ãuƒ’°[O JxßO\÷
3. 4.

5. (1) qO^ΰ…Õ K̈́¬°, =¶O‹¬O =_ÛOKŒ~¡°. 7. (4)


gi…Õ HùO`Ç=°Ok =¶O‹¬O =¶ã`Ç"Í° u<Œß~¡°. H›~òH›…Õ (JxßO\÷…Õ) L#ß ‹¬OY¼ « 3.
REASONING ABILITY (VERBAL) ○ ○ ○ ○ ○ ○ ○ ○ ○
128
○ ○ ○ ○ ○ ○ ○ ○ ○ ○ ○ ○ ○ ○ ○ ○ ○ ○ ○
Study Material
=¶¼^ŠÎ"³°\÷H›…˜ ̋<£ž >ÿ‹¹“
12 (Mathematical Sense Test)

=òMì¼O‰§°...
G D qƒìQ®O…Õ Wzó# Q®}÷`Ç ã„¬=KÇ#O…Õ Wq°_†Çò#ß …ìlH±#° J~¡ÖO K͋¬°ä›½x J_y# ㄬ‰×ß䛽 ‹¬=¶^¥#O ~Œ|\ì“e.

=¶ki ㄬ‰×߁°

Y
´ÿ ´ÿ

M
1. 1052 = 4, 2173 = 6 =°i†Çò 1557 = 10 J~ò# ii) 64 9 16 = 96

2483 = ? Þÿ ´ÿ ´ÿ

E
 ×  ×  = 8 3 4 = 96

ª^Î# : Þÿ 
J^Í q^ÎOQê 16þ49þ25 Þ  ×  × 

D
9
1052 = 4 + 2 = 2 + 2 = 4


59
« 4 þ 7 þ 5 « 140
CA
95
2173 = 6 Þÿ  + 3 = 3 + 3 = 6
4. 52 + 38 + 26 = 46 =°i†Çò 24 + 36 + 52 = 36

50
98
A
Þÿ  J~ò# 79 + 55 + 28 q°= ZO`Ç ? (S.I. 2008)
:8
1557 = 10 + 7 = 3 + 7 = 10

ª^Î# : i) 52 + 38 + 26 = 46
E

Þÿ ´ÿ ´ÿ ´ÿ
h

\ÿ 
.P

(5 2) + (3 8) + (2 6)
IC

2483 = + 3 = 3 + 3 = 6

AD

= 10 + 24 + 12 = 46
2. 64 + 144 – 81 = 11, 225 + 625 – 676 = 14
AB
O

ii) 24 + 36 + 52 = 36
=°i†Çò
324 + 441 – 169 = 26 J~ò# 576 + 256 – 196=? Þÿ ´ÿ ´ÿ ´ÿ
ER

(2 4) + (3 6) + (5 2)
CH

ª^Î# : Þÿ = 8 + 18 + 10 = 36
YD

64 + 144 – 81 = 11  +  − 
J^Í q^ÎOQê, 79 { 55 { 28
,H

« (7 þ 9) { (5 þ 5) { (2 þ 8)
= 8 + 12 – 9 = 11

Þÿ
T

 +  − 


AM

« 63 { 25 { 16
225 + 625 – 676 = 14
H

« 104
M

= 15 + 25 – 26 = 14

Þÿ
AM
IG

324 + 441 – 169 = 26  +  − 


5. 20 þ 5 « 4 =°i†Çò 48 þ 12 « 4 J~ò#
KH

= 18 + 21 – 13 = 26

\ÿ 77 þ 11 q°= ZO`Ç ?
R

576 + 256 – 196 (S.I. 2008)

Þ
Þ
 +  − 
24 + 16 – 14
ª^Î# : 20 þ 5 « 4 Þÿ  « 4
Þÿ 26.
48 þ 12 « 4 Þÿ 

«4
3. 81 þ 4 þ 36 « 108   64 þ 9 þ 16 « 96
J~ò# 16 þ 49 þ 25 q°= ZO`Ç ? J^Í q^ÎOQê 77 þ 11 « 
«7
ª^Î# : i) 81 ´ÿ 4 ´ÿ36 = 108 

Þÿ  ×  ×  = 9 ´ÿ ´ÿ
2 6 = 108

REASONING ABILITY (VERBAL) ○ ○ ○ ○ ○ ○ ○ ○ ○


129
○ ○ ○ ○ ○ ○ ○ ○ ○ ○ ○ ○ ○ ○ ○ ○ ○ ○ ○
Study Material
123456789012345678901234567890121234567890123456789012345678901212
123456789012345678901234567890121234567890123456789012345678901212
12345678901234567890123456789012123456789012345678901234567890121234567890123456789012345678901212345678901234567890123456789012123
1234567890123456789012345678901212345678901234567890123456789012123456789012345678901234567890
12345678901234567890123456789012123456789012345678901234567890121234567890123456789012345678901212345678901234567890123
1234567890123456789012345678901212345678901234567890123456789012123456789012345678901234567890121234567890123
1234567890123456789012345678901212345678901234567890123456789012123456789012345678901234567890121234
12345678901234567890123456789012123456789012345678901234567890121234567890123456789012345678901212345678901234567890123456789012123
1234567890123456789012345678901212345678901234567890123456789012123456789012345678901234567890
12345678901234567890123456789012123456789012345678901234567890121234567890123456789012345678901212345678901234567890123
1234567890123456789012345678901212345678901234567890123456789012123456789012345678901234567890121234567890123
1234567890123456789012345678901212345678901234567890123456789012123456789012345678901234567890121234
123456789012345678901234567890121234567890123456789012345678901212
12345678901234567890123456789012123456789012345678901234567890121234567890123456789012345678901212345678901234567890123456789012123
1234567890123456789012345678901212345678901234567890123456789012123456789012345678901234567890
12345678901234567890123456789012123456789012345678901234567890121234567890123456789012345678901212345678901234567890123
1234567890123456789012345678901212345678901234567890123456789012123456789012345678901234567890121234567890123
1234567890123456789012345678901212345678901234567890123456789012123456789012345678901234567890121234
PRACTICE TEST
12345678901234567890123456789012123456789012345678901234567890121234567890123456789012345678901212345678901234567890123456789012123
1234567890123456789012345678901212345678901234567890123456789012123456789012345678901234567890
123456789012345678901234567890121234567890123456789012345678901212
12345678901234567890123456789012123456789012345678901234567890121234567890123456789012345678901212345678901234567890123
1234567890123456789012345678901212345678901234567890123456789012123456789012345678901234567890121234567890123
1234567890123456789012345678901212345678901234567890123456789012123456789012345678901234567890121234
12345678901234567890123456789012123456789012345678901234567890121234567890123456789012345678901212345678901234567890123456789012123
1234567890123456789012345678901212345678901234567890123456789012123456789012345678901234567890
123456789012345678901234567890121234567890123456789012345678901212
12345678901234567890123456789012123456789012345678901234567890121234567890123456789012345678901212345678901234567890123
1234567890123456789012345678901212345678901234567890123456789012123456789012345678901234567890121234567890123
1234567890123456789012345678901212345678901234567890123456789012123456789012345678901234567890121234
12345678901234567890123456789012123456789012345678901234567890121234567890123456789012345678901212345678901234567890123456789012123
1234567890123456789012345678901212345678901234567890123456789012123456789012345678901234567890
12345678901234567890123456789012123456789012345678901234567890121234567890123456789012345678901212345678901234567890123
1234567890123456789012345678901212345678901234567890123456789012123456789012345678901234567890121234567890123
1234567890123456789012345678901212345678901234567890123456789012123456789012345678901234567890121234
12345678901234567890123456789012123456789012345678901234567890121234567890123456789012345678901212345678901234567890123456789012123
1234567890123456789012345678901212345678901234567890123456789012123456789012345678901234567890
123456789012345678901234567890121234567890123456789012345678901212
12345678901234567890123456789012123456789012345678901234567890121234567890123456789012345678901212345678901234567890123456789012123
1234567890123456789012345678901212345678901234567890123456789012123456789012345678901234567890
1234567890123456789012345678901212345678901234567890123456789012123456789012345678901234567890121234567890123
1234567890123456789012345678901212345678901234567890123456789012123456789012345678901234567890121234

1. 3 ´ÿ5 = 375, 4 ´ÿ3 = 108, 6 ´ÿ7 = 2058 J~ò# 8. 5 + 2 + 1 = 64, 6 + 2 + 7 = 225 J~ò# 7 + 5 + 3=
8 ´ÿ5 = 1) 225 2) 245 3) 384 4) 216

1) 675 2) 1000 3) 825 4) 1275


9. 1 ´ÿ7 = 50, 2 ´ÿ6 = 40 J~ò# 3 ´ÿ7 =
2. 1 ´ÿ3 = 26, 2 ´ÿ4 = 48, 1 ´ÿ8 = 36 J~ò# 2 ´ÿ6 = 1) 48 2) 52 3) 54 4) 58

10. 2 – 3 – 4 = 1, 3 – 4 – 5 = 174 J~ò# 2 – 3 – 5 =


1) 48 2) 72 3) 52 4) 54

3. 2 ´ÿ3 = 827, 2 ´ÿ4 - 864, 3 ´ÿ5 = 27125 J~ò#


1) 0 2) 125 3) 74 4) 36
8 ´ÿ6 =
1) 512216 2) 384216 3) 216144 4) 343512
11. 7 ´ÿ 6 = 307, 8 ´ÿ 4 = 496, 9 ´ÿ 5 = 704 J~ò#
4. (3 + 5) ´ÿ(1 + 2) = 170, (2 + 3) ´ÿ(2 + 5) = 377 7 ´ÿ7 =
J~ò# (4 + 3) ´ÿ(1 + 5) =

Y
1) 304 2) 286 3) 294 4) 275

1) 576 2) 650 3) 136 4) 224 12. 3 + 5 + 7 = 59, 4 + 6 + 2 = 102 J~ò# 3 + 6 + 9=

M
5. 1 ´ÿ2 ´ÿ3 ´ÿ4 = 10, 2 ´ÿ3 ´ÿ4 ´ÿ5 = 29, 3 ´ÿ4 ´ÿ5 1) 64 2) 72 3) 96 4) 84

´ÿ6 = 66 J~ò# 4 ´ÿ5 ´ÿ6 ´ÿ7 = 13. 2 ´ÿ3 = 25, 3 ´ÿ4 = 49, 2 ´ÿ6 = 64 J~ò# 4 ´ÿ8 =

E
1) 86 2) 47 3) 127 4) 97 1) 144 2) 125 3) 225 4) 121

D
9
6. 5 + 3 – 1 = 33, 6 + 4 – 2 = 48 J~ò# 7 + 3 – 5 = 14. 643 = 27, 862 = 50, 563 = 33 J~ò# 931 «

59
CA
95
1) 45 2) 43 3) 35 4) 33
1) 30 2) 32 3) 42 4) 28
7. 2 ´ÿ 3 = 30, 3 ´ÿ 5 = 144, 5 ´ÿ 7 = 456 J~ò#
15. 1 + 3 + 7 = 371, 2 + 5 + 8 = 645 J~ò# 7 + 1 + 6=

50
3 ´ÿ8 =
98
A
1) 484 2) 496 3) 560 4) 516
1) 508 2) 488 3) 528 4) 514
:8
1234567890123456789012
12345678901234567890123456789012123456789012345678901234567890121234567890123456789012345678901212345678901234567890123
12345678901234567890123

q=~¡}ì`ÇàH› ["Œ|°°
12345678901234567890123456789012123456789012345678901234567890121234567890123456789012345678901212345678901234567890123
1234567890123456789012 12345678901234567890123
E

1234567890123456789012 12345678901234567890123
12345678901234567890123456789012123456789012345678901234567890121234567890123456789012345678901212345678901234567890123
1234567890123456789012
PRACTICE TEST : 12345678901234567890123
12345678901234567890123456789012123456789012345678901234567890121234567890123456789012345678901212345678901234567890123
h

1234567890123456789012
12345678901234567890123456789012123456789012345678901234567890121234567890123456789012345678901212345678901234567890123
1234567890123456789012 12345678901234567890123
12345678901234567890123456789012123456789012345678901234567890121234567890123456789012345678901212345678901234567890123
.P

1234567890123456789012 12345678901234567890123
12345678901234567890123
12345678901234567890123456789012123456789012345678901234567890121234567890123456789012345678901212345678901234567890123
IC

´ÿ Þÿ ´ÿ ´ÿ ´ÿ ´ÿ ´ÿ Þÿ ´ÿ ´ÿ
AD

1. (2) 3 5 = 375 3 5
3
= 3 125 = 375 5. (3) 1 2 3 4 = 10 (1 2 3) + 4 = 6 + 4 = 10

´ÿ Þÿ ´ÿ ´ÿ ´ÿ ´ÿ ´ÿ Þÿ ´ÿ ´ÿ
AB
O

3
4 3 = 81 4 3 = 4 27 = 108 2 3 4 5 = 29 (2 3 4) + 5 = 24 + 5 = 29

´ÿ Þÿ ´ÿ ´ÿ ´ÿ ´ÿ ´ÿ Þÿ ´ÿ ´ÿ
ER
CH

3
6 7 = 2058 6 7 = 6 343 = 2058 3 4 5 6 = 66 (3 4 5) + 6 = 60 + 6 = 66

\ÿ ´ÿ ´ÿ ´ÿ 3
\ÿ ´ÿ ´ÿ ´ÿ ´ÿ ´ÿ
YD

8 5 = 8 5 = 8 125 = 1000 4 5 6 7 = (4 5 6) + 7 = 120 + 7 = 127

2. (3) ´ÿ Þÿ ´ÿ 6. (4) Þÿ
5 + 3 – 1 = 33 5
2
+ 3
2
– 1
2
= 25 + 9 – 1 = 33
,H

Þÿ
1 3 = 26 13 2 = 26

´ÿ Þÿ ´ÿ 2 2 2
T

6 + 4 – = 48 6 + 4 – 2 = 36 + 16 – 4 = 48

\ÿ
AM

2 4 = 48 24 2 = 48

´ÿ Þÿ ´ÿ 7 + 3 – 5 = 7
2
+ 3
2
– 5
2
= 49 + 9 – 25
H

1 8 = 36 18 2 = 36

\ÿ ´ÿ ´ÿ
M

= 58 – 25 = 33

´ÿ Þÿ
2 6 = 26 2 = 52
AM

3. (1) ´ÿ Þÿ
IG

3 3 7. (3) 2 3 = 30
3 3
(2 + 3 ) – (2 + 3) = 8 + 27 – 5 = 30

´ÿ Þÿ
2 3 = 827 (2 ) (3 ) = (8) (27) = 827

´ÿ Þÿ
KH

3 3
3 3 3 5 = 144 (3 + 5 ) – (3 + 5)
2 4 = 864 (2 ) (4 ) = (8) (64) = 864

´ÿ Þÿ
R

3 3 = 27 + 125 – 8 = 144

´ÿ Þÿ
3 5 = 27125 (3 ) (5 ) = (27) (125) = 27125

\ÿ ´ÿ 8
3
6 = (8 ) (6 ) = 512216
3 5 7 = 456 (5
3 3
+ 7 ) – (5 + 7)

4. (2) ´ÿ Þÿ 2 2
´ÿ 2 2 = 125 + 343 – 12 = 456

J^Í q^ÎOQê
(3 + 5) (1 + 2) = 170 (3 + 5 ) (1 + 2 )

´ÿ
´ÿ
= (9 + 25) (1 + 4)

´ÿ 3
3 3
8 = (3 + 8 ) – (3 + 8) = 27 + 512 – 11 = 528

Þÿ ´ÿ
= 34 5 = 170

´ÿ Þÿ 2 2
´ÿ 2 2 8. (1) 5 + 2 + 1 = 64 5 + 2 + 1 = 8 8 = 64

Þÿ ´ÿ
(2 + 3) (2 + 5) (2 + 3 ) (2 + 5 )

´ÿ 6 + 2 + 7 = 225 6 + 2 + 7 = 15 15 = 225

\ÿ ´ÿ
= (4 + 9) (4 + 25)

´ÿ 7 + 5 + 3 = 15 15 = 225

9. (4) ´ÿ Þÿ
= 13 29 = 377

\ÿ ´ÿ 2 2
´ÿ 2 2 1 7 = 50 1
2
+ 7
2
= 1 + 49 = 50

´ÿ Þÿ
(4 + 3) (1 + 5) = (4 + 3 ) (1 + 5)

´ÿ 2 6 = 40 2
2
+ 6
2
= 4 + 36 = 40

\ÿ ´ÿ
= (16 + 9) (1 + 25)

= 25 ´ÿ 26 = 650
3 7 = 3
2
+ 7
2
= 9 + 49 = 58

REASONING ABILITY (VERBAL) ○ ○ ○ ○ ○ ○ ○ ○ ○


130
○ ○ ○ ○ ○ ○ ○ ○ ○ ○ ○ ○ ○ ○ ○ ○ ○ ○ ○
Study Material
10.  (1) 2 – 3 – 4 – 1 Þÿ 5 3 1 13. (1) ´ÿ Þÿ 2 3 = 25 (2 + 3)
2
= 5 = 25
2

´ÿ Þÿ
2 – 3 – 4 = 32 – 27 – 4 = 1

Þÿ 5 3 1 4 3 = 49 (4 + 3)
2
= 7
2
= 49

´ÿ Þÿ
3 – 4 – 5 –174 3 – 4 – 5 = 243 – 64 – 5 = 174

J^Í q^ÎOQê 5 3 1 2 6 = 64 (2 + 6)
2
= 8
2
= 64

\ÿ ´ÿ
2 – 3 = 5 = 2 – 3 – 5
2 2
4 8 = (4 + 8) = 12 = 144

Þÿ ´ÿ
= 32 – 27 – 5 = 0

11. (3) 7´ÿ 6 = 307 Þÿ 7


3
– 6
2
= 343 – 36 = 307
14. (4) 643 = 27

Þÿ ´ÿ
(6 4) + 3 = 24 + 3 = 27

8´ÿ 4 = 496 Þÿ 8
3
– 4
2
= 512 – 16 = 496
862 = 50

Þÿ ´ÿ
(8 6) + 2 = 48 + 2 = 50

9´ÿ 5 = 704 Þÿ 9
3
– 5
2
= 729 – 25 = 704
´ÿ
563 = 33 (5 6) + 3 = 30 + 3 = 33

\ÿ ´ÿ7 7 = 7
3
– 7
2
= 343 – 49 = 294
15. (3)
931 = (9

Þÿ
3) + 1 = 27 + 1 = 28

Þÿ
3 3 3
1 + 3 + 7 = 371 1 + 3 + 7 = 1 + 27 + 343 – 371
12. (2) 3 + 5 + 7 = 59

4 + 6 + 2 = 102 Þÿ
3
3

3
+ 5
2

2
+ 7 = 27 + 25 + 7 = 59

4 + 6 + 2 = 64 + 36 + 2 = 102
Þÿ
2 + 5 + 8 = 645 2
3
+ 5
3
+ 8
3

\ÿ
= 8 + 125 + 512 = 645

3 + 6 + 9 = 3
3
+ 6
2
+ 9 = 27 + 36 + 9 = 72
\ÿ 3
7 + 1 + 6 = 7 + 1 + 6 = 343 + 1 + 216 = 560
3 3

Y
123456789012345678901234567890121234567890123456789012345678901212
12345678901234567890123456789012123456789012345678901234567890121234567890123456789012345678901212345678901234567890123
1234567890123456789012345678901212345678901234567890123456789012123456789012345678901234567890121234567890123
1234567890123456789012345678901212345678901234567890123456789012123456789012345678901234567890121234
123456789012345678901234567890121234567890123456789012345678901212
12345678901234567890123456789012123456789012345678901234567890121234567890123456789012345678901212345678901234567890123456789012123
123456789012345678901234567890121234567890123456789012345678901212345678901234567890123
12345678901234567890123456789012123456789012345678901234567890121234567890123456789012345678901212345678901234567890123
1234567890123456789012345678901212345678901234567890123456789012123456789012345678901234567890121234567890123
1234567890123456789012345678901212345678901234567890123456789012123456789012345678901234567890121234
12345678901234567890123456789012123456789012345678901234567890121234567890123456789012345678901212345678901234567890123456789012123
123456789012345678901234567890121234567890123456789012345678901212345678901234567890123
123456789012345678901234567890121234567890123456789012345678901212
12345678901234567890123456789012123456789012345678901234567890121234567890123456789012345678901212345678901234567890123
1234567890123456789012345678901212345678901234567890123456789012123456789012345678901234567890121234567890123
1234567890123456789012345678901212345678901234567890123456789012123456789012345678901234567890121234
12345678901234567890123456789012123456789012345678901234567890121234567890123456789012345678901212345678901234567890123456789012123
123456789012345678901234567890121234567890123456789012345678901212345678901234567890123
PREVIOUS BITS

M
123456789012345678901234567890121234567890123456789012345678901212
12345678901234567890123456789012123456789012345678901234567890121234567890123456789012345678901212345678901234567890123456789012123
123456789012345678901234567890121234567890123456789012345678901212345678901234567890123
12345678901234567890123456789012123456789012345678901234567890121234567890123456789012345678901212345678901234567890123
1234567890123456789012345678901212345678901234567890123456789012123456789012345678901234567890121234567890123
1234567890123456789012345678901212345678901234567890123456789012123456789012345678901234567890121234
12345678901234567890123456789012123456789012345678901234567890121234567890123456789012345678901212345678901234567890123456789012123
123456789012345678901234567890121234567890123456789012345678901212345678901234567890123
12345678901234567890123456789012123456789012345678901234567890121234567890123456789012345678901212345678901234567890123
1234567890123456789012345678901212345678901234567890123456789012123456789012345678901234567890121234567890123
1234567890123456789012345678901212345678901234567890123456789012123456789012345678901234567890121234
123456789012345678901234567890121234567890123456789012345678901212
12345678901234567890123456789012123456789012345678901234567890121234567890123456789012345678901212345678901234567890123456789012123
123456789012345678901234567890121234567890123456789012345678901212345678901234567890123
12345678901234567890123456789012123456789012345678901234567890121234567890123456789012345678901212345678901234567890123
1234567890123456789012345678901212345678901234567890123456789012123456789012345678901234567890121234567890123
1234567890123456789012345678901212345678901234567890123456789012123456789012345678901234567890121234
12345678901234567890123456789012123456789012345678901234567890121234567890123456789012345678901212345678901234567890123456789012123
123456789012345678901234567890121234567890123456789012345678901212345678901234567890123

E
1. L J<Íq + x, M J<Ík - x, N J<Ík þ x, P J<Ík Ó a*b Þ a>b
x ‹¬¶zÀ‹ë J„¬C_È°

D
a$b Þ a<b

9
59
[(14N10) L (42P2)] M8 (T.S. S.I.Mains -2016) a#b Þ a³b
1) 153 2) 216 3) 248 4) 251 CA
95
a@b Þ a£b
2. 20-10 JO>è 200 Jh, 8 Ó 4 JO>è 12 Jh, 6 þ 2

50
au b Þ a=b
JO>è 4 Jx J~¡Ö=°~ò`Í J„¬C_È° 3. P @ Q u R * S Þ
98
A
[(100–10) × (1000 Ó 1000)] Ó (100 – 10) =.... 1) 2) 3) 4)
:8
P * Q R $ S Q * S Q $ S

4. P # Q @ R $ S Þ
E
h

(T.S. S.I.Mains -2016)

1) 20 2) 1090 3) 1900 4) 0 1) 2) 3) 4)
.P
IC

P $ Q Q $ S Q @ S R @ S

(3-5) =~¡ä›½ Q® ㄬ‰×߁䛽 ãH÷O^Î W=Þ|_# ‹¬=¶KŒ~Œxß 5. P # Q # R u S * T J~ò`Í, J„¬C_È° ãH÷Ok SzóHꁅÕ
AD

L„¬†³¶yOz ‹¬=¶^¥#=ò° ~Œ|@“O_ Uk J‹¬`Ǽ=ò?


AB
O

(A.P.Constable Prelims -2016)

(A.P. Constable Mains -2016) 1) P # R 2) Q # S 3) Q * T 4) Q $ T


ER
CH

12345678901234567890123456789012
12345678901234567890123456789012123456789012345678901234567890121234567890123456789012345678901212345678901234567890123
12345678901234567890123456789012

q=~¡}ì`ÇàH› ["Œ|°°
12345678901234567890123456789012123456789012345678901234567890121234567890123456789012345678901212345678901234567890123
12345678901234567890123456789012
12345678901234567890123456789012 12345678901234567890123456789012
12345678901234567890123456789012
12345678901234567890123456789012123456789012345678901234567890121234567890123456789012345678901212345678901234567890123
YD

12345678901234567890123456789012
12345678901234567890123456789012123456789012345678901234567890121234567890123456789012345678901212345678901234567890123
12345678901234567890123456789012 12345678901234567890123456789012
12345678901234567890123456789012 12345678901234567890123456789012
12345678901234567890123456789012123456789012345678901234567890121234567890123456789012345678901212345678901234567890123
12345678901234567890123456789012
12345678901234567890123456789012123456789012345678901234567890121234567890123456789012345678901212345678901234567890123
12345678901234567890123456789012 12345678901234567890123456789012
12345678901234567890123456789012123456789012345678901234567890121234567890123456789012345678901212345678901234567890123
,H

…ÿH›ø ㄬHê~¡O (3-5) ㄬ‰×ß ã„¬Hê~¡O


T

(1)
Þÿ Þÿ³ÿ Þ £ Þÿ
AM

1.

Þ Þÿ Þÿ Þ Ó * > ; $

Þÿ
< ; # ; @
H

L + ; M – ; N × ; P

^Î`Çë ‹¬=¶‹¬=ò u
M

Þÿ £
AM

(3) u
IG

3. P @ Q R * S P Q = R > S
[(14N10) L (42P2)] M8

̄á Q®°~¡°ë#° D ‹¬=¶‹¬O…Õ ㄬuöHƄ²OKÇQê Ì„á ‹¬=¶‹¬O #°O_


KH

Ó Þÿ \ ‹¬`Ǽ=ò
R

Q = R > S Q > S
[(14 × 10) + (42


21)] – 8
Þÿ Q * S

=   4. (2) P # Q @ R $ S

³ £
P Q R < S

2. (4) 20 – 10
= 140 + 21 – 8 = 161 – 8 = 153.

Þ J#Qê -ÿÞÿþ Þÿ
200 20 × 10 = 200
Ì„á ‹¬=¶‹¬O #°O_ Þÿ ‹¬`Ǽ=ò. Q < S Q $ S

Ó Þ J#Qê ÓÿÞÿ{ Þÿ 5. (4) u


P # Q # R S
8 4 12

Þ J#Qê þÿÞÿ- Þÿ
8 × 4 = 12
³ ³
P Q R = S, S > T
6 × 2

^Î`Ç달=¶‹¬=ò
4 6 – 2 = 4
³Q S > T

Ó Ó \ Þ Q > T Q * T
[(100–10) × (1000

̄á Q®°~¡°ë#° D ‹¬g°H›~¡}O…Õ ㄬuöHƄ²OKÇQê


1000)] (100 – 10)
Þ Q < T

Þ Q $ T
[(100×10) – (1000 + 1000)] + (100 × 10)

2000 – 2000 = 0
\ J‹¬`Ǽ=ò.
REASONING ABILITY (VERBAL) ○ ○ ○ ○ ○ ○ ○ ○ ○
131
○ ○ ○ ○ ○ ○ ○ ○ ○ ○ ○ ○ ○ ○ ○ ○ ○ ○ ○
Study Material
q°‹²žOQ· Hê¼ï~H›“~Ÿ
13 (Missing Character)

=òMì¼O‰§°...
G q°‹²žOQ· Hê¼ï~H›“~Ÿä›½ ‹¬O|OkOz# ㄬ‰×߁x߆Çò ãuƒ’°*ìHê~¡O…Õ …è^¥ =¶¼ã\÷H±ž ~¡¶„¬O…Õ …è^¥ =$`ŒëHê~¡ ~¡¶„¬O…Õ …è^¥
qq^Î ~¡Hê ƒç=°à#° Wzó JO^ΰ…Õ #O|~¡Á#° …è^¥ POQ®Á JH›Æ~Œ#° „¬îiOz XH› Q®_x ㄬ‰§ß~¡ÖH›O (?) Q®°~¡°ë`Ë =ke
"͆ǰ_ÈO [~¡°Q®°`Ç°Ok.
G

Y
D ~¡H›O ㄬ‰×߁#° ªkOKÇ°@䛽 ‹¬OY¼ †³òH›ø =~¡¾=ò°, „¦¬°#=ò°, Q®°}Hê~¡O, ƒìQꂬð~¡O, P…ì权ÿ\˜ž Q®°iOz
‹¬i†³Ø°# J=Qꂬì# LO_È=…ÿ#°.

M
G „¬@O U^³á#„¬æ\÷H© P ‹¬OY¼° …è^¥ JH›Æ~Œ =°^Î¼Q® x†Ç°=¶xß H›#°Qùx ㄬ‰§ß~¡ÖH›O (?) ªÖ#O…Õ U ‹¬OY¼ …è^¥ JH›Æ~¡O

E
LO_¨…Õ H›#°Qù#=…ÿ#°.
G

D
9
D ~¡H›O ㄬ‰×߁ ‹¬=¶^¥#O Q®°iëOKÇ°@䛽 logic ㄬ^¥#O. logic `³°‹¬°ä›½O>è ‹¬=¶^¥#O Q®°iëOz#>èÁ.

59
CA
95
=¶ki ㄬ‰×߁°
50
98
A
:8
MODEL - 1 ª^Î#: „¬@O…Õ, 22 { 2 « 4 { 2 « 6
E
h

32 { 3 « 9 { 3 « 12
.P

1. D ãH÷O^Î W=Þ|_# ‹¬OY¼ ãH›=°O…Õ q°‹²žOQ· #O|~Ÿ


IC

42 { 4 « 16 { 4 « 20
AD

(?) q°= ZO`Ç ?


52 { 5 « 25 { 5 « 30
AB
O

\ Hê=ô# ? « 12.
ER
CH
YD

ª^Î#: "³ò^Î\ ÷ „¬^ÎO #°O_ (6 { 4 { 8) { 2 « 20 4. …Õ '?Ñ Q® ° ~¡ ° ë


,H

ï~O_È= „¬@O #°O_ (7 { 9 { 8) { 2 « 26


T
AM

=ü_È= „¬@O #°O_ (6 { 5 { 12) { 2 « 25 ªÖ#O…Õ =KÇ°ó ‹¬OY¼#° H›#°Qù#°=ò? (Gr. IV - 1996)

\ Hê=ô# q°‹²žOQ· #O|~Ÿ « 25.


H

ª^Î#: W=Þ|_# „¬@=ò…Õ 2


M

4 = 16
AM

2. D ãH÷Ok „¬\O …Õx q°‹²žOQ· #O|~Ÿ (?)#° H›#°Qù#O_. 2


IG

16 = 256

2
KH

(Communication S.I. - 2012) 256 = 65536


R

5. …Õ '?Ñ Q®°~¡°ë ªÖ#O…Õ ~Œ=‹²#


ª^Î#: „¬@O…Õ, 5 þ 2 « 10
7 þ 2 « 14
9 þ 2 « 18 ‹¬OY¼#° H›#°Qù#°=ò ? (Dy. Jailer - 2013)
3. D ãH÷Ok „¬\O …Õx q°‹²žOQ· #O|~Ÿ (?)#° H›#°Qù#O_.
ª^Î#: „¬@O…Õ, 2 þ 7 « 14
7 þ 9 « 63
15 þ 2 « 30
\ 9 þ 15 « 135.

REASONING ABILITY (VERBAL) ○ ○ ○ ○ ○ ○ ○ ○ ○


132
○ ○ ○ ○ ○ ○ ○ ○ ○ ○ ○ ○ ○ ○ ○ ○ ○ ○ ○
Study Material
‹¬¶KÇ# (6-8) :
MODEL - 2
6. D ãH÷Ok „¬\쁅Õx q°‹²žOQ· #O|~Ÿ (?)#° H›#°Qù#O_.
1. D ãH÷O^Î W=Þ|_# „¬\ì…Õ q°‹²žOQ· #O|~Ÿ (?)
q°= ZO`Ç ?

ª^Î#:
A B C D E F G H I J K L M

1 2 3 4 5 6 7 8 9 10 11 12 13 ª^Î#: „¬@O (i) #°O_ (3 þ 2) - 1 «5

Y
N O P Q R S T U V W X Y Z „¬@O (ii) #°O_ (4 þ 8) - 1 « 31
\ÿÿ„¬@O (iii) #°O_ (9 þ 3) - 1 « 26.

M
14 15 16 17 18 19 20 21 22 23 24 25 26

2. D ãH÷Ok „¬\O …Õx q°‹²žOQ· #O|~Ÿ (?)#° H›#°Qù#O_.


Ì„á „¬\ ÷“H› #°O_

E
R(18) × S(19) = 342
     

D
9
F(6) × Q(17) = 102

59
V(22) × D(4) = 88
(Dy. Jailor - 2013)

ÿÿÿÿÿÿÿÿÿÿ\ CA
95
 
K(11) × Z(26) = 286.
ª^Î#: "³ò^Î\ ÷ „¬@O…Õ 4 × 3 = 12 ÿÞÿ 12
2 = 144

50
ï~O_È= „¬@O…Õ 11 × 9 = 99 = 9801 ÿÞÿ 99
2

98
A
7.
\ J^Íq^ÎOQê, =ü_È= „¬@O…Õ 15 × 6 = 90
:8
ÿÞÿ 2
E

90 = 8100
h
.P
IC
AD

ª^Î#:
AB
O

3.
ER

A B C D E F G H I J K L M
CH

26 25 24 23 22 21 20 19 18 17 16 15 14
YD

N O P Q R S T U V W X Y Z
,H

ª^Î#: Wzó# „¬@O…Õ 133 = 2197 ; 153 = 3375 ; 173 = 4913


T

13 12 11 10 9 8 7 6 5 4 3 2 1
AM

POQ®Á JH›Æ~Œä›½ z=i #°O_ ãH›=°‹¬OY¼…Õ J=°~¡óQê \ÿÿ 3 Þÿ


H

x = 12167 x = 23
M

L = 15, U = 6, Q = 10 =°i†Çò H = 19.


AM

4.
IG

\ q°‹²žOQ· #O|~Ÿ « 19.


KH
R

8.

(Communication S.I. 2012)

2 2 2 2
ª^Î#: "³ò^Î\ ÷ „¬@O…Õ, 93 – (27 + 63) = 3
ª^Î#: (5 + 4 ) – (3 + 2 ) = 41 – 13 = 28
"³ò^Î\ ÷ „¬@O…Õ 79 – (38 + 37) = 4
J^Íq^ÎOQê, (82 + 42) – (82 + x2) = 12
\ J…ìöQ, 67 – (16 + 42) = 9
Þÿ 16 – x
2 = 12

Þÿ 2 x = 16 – 12 = 4 5.
Y  

REASONING ABILITY (VERBAL) ○ ○ ○ ○ ○ ○ ○ ○ ○


133
○ ○ ○ ○ ○ ○ ○ ○ ○ ○ ○ ○ ○ ○ ○ ○ ○ ○ ○
Study Material
ª^Î#:        ("³ò^Î\ ÷ „¬@O…Õ) ª^Î#: „¬@O (1) #°O_ 32 « 9 =°i†Çò 42 « 16 ® 916
       (ï~O_È= „¬@O…Õ) „¬@O (2) #°O_ 2 « 4 =°i†Çò 3 « 9 ® 49
2 2

J^Íq^ÎOQê, „¬@O (3) #°O_ 12 « 1 =°i†Çò 52 « 25 ® 125


‹¬¶KÇ# (5-8) : D ãH÷Ok „¬\ì…Õ '?Ñ Q®°~¡°ë ªÖ#O…Õ =KÇ°ó
       (=ü_È= „¬@O…Õ)
‹¬OY¼#° H›#°Qù#°=ò?
MODEL - 3
5.
1. D ãH÷Ok „¬\ì #°O_ q°‹²žOQ· #O|~Ÿ (?) q°=
ZO`Ç ?
ª^Î#: "³ò^Î\ ÷ „¬@O…Õ (5 þ 3) { 4 « 15 { 4 « 19
ï~O_È= „¬@O…Õ (6 þ 4) { 5 « 24 { 5 « 29
\ J^Íq^ÎOQê =ü_È= „¬@O…Õ (7 þ 5) { 6

Y
« 35 { 6 « 41.

M
ª^Î#: „¬@O #°O_ (7 þ 6) { 3 « 45
(i)
6.
„¬@O (ii) #°O_ (5 þ 4) { 6 « 26

E
\ÿÿ„¬@O (iii) …Õ (7 þ 3) { 8 « 29.

D
9
Hê=ô# q°‹²žOQ· #O|~Ÿ « 29

59
2. D ãH÷Ok „¬\O…Õx q°‹²žOQ· #O|~Ÿ (?) q°= ZO`Ç?
CA
95
(H›=ü¼xöH+¬<£ S.I.- 2012)
ª^Î#:

50
F(6) + P(16) = 22

98
A
G(7) + N(14) = 21
:8
E(5) + J(10) = 15
E

J^Íq^ÎOQê K(11) + P(16) = 27.


h
.P
IC

ª^Î#: Wzó# „¬@O #°O_, (7 þ 2) { 1 « 15 7.


AD

(15 þ 2) { 1 « 31
AB
O

 
(31 þ 2) { 1 « 63
(63 þ 2) { 1 « 127.
ER


CH

  
J^Í ã„¬Hê~¡O, (127 þ 2) { 1 « 255
YD

\ Hê=ô# q°‹²žOQ· #O|~Ÿ « 255.  


,H

3. D ãH÷Ok „¬\쁅Õx q°‹²žOQ· #O|~Ÿ (?)#° H›#°Qù#O_. ª^Î#: "³ò^Î\ ÷ „¬@O…Õ       


T
AM

(Communication S.I. - 2012) ï~O_È= „¬@O…Õ       


H

\ J^Íq^ÎOQê =ü_È= „¬@O…Õ


M
AM
IG

      
KH

  
®ÿ"³ò^Î\ ÷ „¬@O…Õ
R

ª^Î#: 2 2 2
5 = 3 + 4      
2 2 2 ®ÿ"³ò^Î\ ÷ „¬@O…Õ 8.
13 = 5 + 12
 
\ J…ìöQ 3= „¬@O…Õ 2 17 = 15
2 + x
2
     
Þÿ 2 2 2 x = 17 – 15 = 289 – 225 = 64
  
Þÿ Y    Y     ª^Î#: "³ò^Î\ ÷ „¬@O…Õ (3 { 8 { 11) þ (5 { 2 { 9)
4. D ãH÷Ok „¬\쁅Õx q°‹²žOQ· #O|~Ÿ (?)#° H›#°Qù#O_. « 22 þ 16 « 352
(Communication S.I. - 2012) ï~O_È= „¬@O…Õ (9 { 4 { 7) þ (5 { 8 { 12)
« 20 þ 25 « 500
\ J^Íq^ÎOQê =ü_È= „¬@O…Õ
(3 { 5 { 2) þ (4 { 12 { 10)
« 10 þ 26 « 260
REASONING ABILITY (VERBAL) ○ ○ ○ ○ ○ ○ ○ ○ ○
134
○ ○ ○ ○ ○ ○ ○ ○ ○ ○ ○ ○ ○ ○ ○ ○ ○ ○ ○
Study Material
MODEL - 4 5.
1. D ãH÷Ok „¬\ì #°O_ q°‹²žOQ· #O|~Ÿ (?) q°=
ZO`Ç ?
 
ª^Î#:            
   
 
   
 
  
     
6.  
ª^Î#: „¬@O (i)#°O_ (5 { 6) - (7 { 4) « 0
     
„¬@O (ii) #°O_ (7 { 6) - (8 { 4) « 1   

Y
\ÿÿ„¬@O (iii) #°O_ (11 { 2) - (0 { 2) « 11. ª^Î#: 915 - 364 « 551
789 - 543 « 246

M
\ Hê=ô# q°‹²žOQ· #O|~Ÿ « 11 J^Íq^ÎOQê 863 - 241 « 622.
2. D ãH÷Ok „¬\O …Õx q°‹²žOQ· #O|~Ÿ (?)#° H›#°Qù#O_.

E
D
9
59
7.
CA
95
50
ª^Î#: (4 þ 5) { (2 þ 3) { (7 þ 6) « 20 { 6 { 42 « 68
98
A
ª^Î#: (8 × 3) + 6 = 24 + 6 = 30
(2 þ 5) { (6 þ 3) { (1 þ 4) « 10 { 18 { 4 « 32
:8
\ÿÿ(1 þ 2) { (3 þ 2) { (3 þ 4) « 2 { 6 { 12
(8 × 30) + 6 = 240 + 6 = 246
E
h

(8 × 246) + 6 = 1968 + 6 = 1974


.P

\ÿ « 20
IC

(8 × 1974) + 6 = 15792 + 6 = 15798


AD

‹¬¶KÇ# (3-8) :
AB
O

3. D ãH÷Ok „¬\쁅Õx q°‹²žOQ· #O|~Ÿ (?)#° H›#°Qù#O_. 8.


ER
CH
YD

ª^Î#: (7 { 4) - (5 { 6) « 11 - 11 « 0
(7 { 6) - (8 { 4) « 13 - 12 « 1
,H

J^Íq^ÎOQê, (11 { 2) - (2 { 0) « 13 - 2 « 11.


T
AM

Þÿ Þÿ
H

ª^Î#: MODEL - 5
M

3 × 4 = 12 12 – 6 = 6 5 × 2 = 10 10 – 6 = 4

Þÿ Þÿ
AM
IG

3 × 4 = 12 12 – 6 = 6 4 × 3 = 12 12 – 6 = 6

2 × 3 = 6 Þÿ 6 – 6 = 0 4 × 3 = 12 Þÿ 12 – 6 = 6
1. D ãH÷Ok „¬\ ÷“H›…Õ q°‹²žOQ· #O|~Ÿ (?) q°= ZO`Ç ?
KH

Þÿ   
R

2 × 5 = 10 10 – 6 = 4  
3 × 3 = 9 Þÿ 9 – 6 = 3  



3 × 4 = 12 Þÿ 12 – 6 = 6  


ª^Î#: Wzó# „¬\ ÷“H› #°O_,


 
4. "³ò^Î\ ÷ J_È°Û=~¡°‹¬…Õ    
 
ï~O_È= J_È°Û=~¡°‹¬…Õ  
(S.I. - 2011)  
ª^Î#: (14 + 9 + 6 + 2) + 25 = 56
\ÿÿ=ü_È= J_È°Û =~¡°‹¬…Õ    
(12 + 11 + 5 + 3) + 25 = 56

(11 + 20 + 4 + 10) + 25 = 70 Hê=ô# q°‹²žOQ· #O|~Ÿ « 

REASONING ABILITY (VERBAL) ○ ○ ○ ○ ○ ○ ○ ○ ○


135
○ ○ ○ ○ ○ ○ ○ ○ ○ ○ ○ ○ ○ ○ ○ ○ ○ ○ ○
Study Material
2. D ãH÷Ok „¬\O …Õx q°‹²žOQ· #O|~Ÿ (?)#° H›#°Qù#O_.

ª^Î#: "³ò^Î\ ÷ J_È°Û =~¡°‹¬…Õ 3 + 6 + 8 = 17

ï~O_È= J_È°Û =~¡°‹¬…Õ 5 + 8 + 4 = 17 6.


\ J^Íq^ÎOQê, =ü_È= J_È°Û =~¡°‹¬…Õ 4 + 7 + @ = 17

Þÿ @ = 6.

3.

Y
ª^Î#: 7
2 – 5
2 = 49 – 25 = 24

M
11
2 – 8
2 = 121 – 64 = 57

13
2 – 11
2 = 169 – 121 = 48

E
3 3
ª^Î#: "³ò^Î\ ÷ J_È°Û=~¡°‹¬…Õ 3 {4 {5 « 216   3 \ 15
2 – 10
2 = 225 – 100 = 125.

D
9
3 3 3
ï~O_È= J_È°Û =~¡°‹¬…Õ 7 {8 {9 «1584  

59
7.
=ü_È= J_È°Û =~¡°‹¬…Õ 83{113{123 « 3571
CA
95
\ J^Íq^ÎOQê, <Œ¾= J_È°Û =~¡°‹¬…Õ 43{153{183

50
ª^Î#: A, E, I, O, U
« 9271.
98
A
POQ®Á JH›Æ~Œ#° Z = 1, Y = 2, X = 3 ........ A = 26
4. D ãH÷Ok „¬@O…Õx q°‹²žOQ· „¬^ÎO#° H›#°Qù#O_.
:8
ãH›=°O…Õ ~Œ†Ç°Qê U = 6 =KÇ°ó#°.
E
h

6
.P
IC

Hê=ô# q°‹²žOQ· „¬^ÎO « 


AD
AB
O

ª^Î#: "³ò^Î\ ÷ J_È°Û =~¡°‹¬…Õ XHùøH›ø POQ®Á JH›Æ~¡O =^΁|_


8.
`Ç~¡°"Œ`Ç Jk ã"Œ†Ç°|_#k =°i†Çò ãH÷Ok JOïH°
ER
CH

䛀_È|_#q, J^Íq^ÎOQê =ü_È= J_È°Û =~¡°‹¬ 䛀_¨


YD

xiàOKÇ|_#k.
,H

Hê=ô# q°‹²žOQ· „¬^ÎO « K3+5 = K8.   


   
T

ª^Î#: "³ò^Î\ ÷ J_È°Û =~¡°‹¬…Õ


AM

‹¬¶KÇ# (5-9) :  
H

5. D ãH÷Ok ㄬ‰×߁…Õ q°‹²žOQ· „¬^¥#° H›#°Qù#O_.   


   
M

ï~O_È= J_È°Û =~¡°‹¬…Õ


AM

 
IG

  
J^Íq^OÎ Qê, =ü_È= J_È°Û =~¡°‹¬…Õ    
KH

 
R

ª^Î#: (_„¬î¼\÷ *ÿá~Ÿž 2012) 9.


A B C D E F G H I J K L M

1 2 3 4 5 6 7 8 9 10 11 12 13

N O P Q R S T U V W X Y Z
  
14 15 16 17 18 19 20 21 22 23 24 25 26
ª^Î#:    
  
   
1. 






  
   

REASONING ABILITY (VERBAL) ○ ○ ○ ○ ○ ○ ○ ○ ○


136
○ ○ ○ ○ ○ ○ ○ ○ ○ ○ ○ ○ ○ ○ ○ ○ ○ ○ ○
Study Material
MODEL - 6 MODEL - 7
1. D ãH÷Ok „¬\ì…Õ q°‹²žOQ· #O|~Ÿ (?) q°= ZO`Ç? ‹¬¶KÇ# (1-8) :
1. D ãH÷Ok „¬\쁅Õx q°‹²žOQ· #O|~Ÿ (?)#° H›#°Qù#O_.
(Communication S.I. - 2012)

ª^Î#: „¬@O #°O_ (4 þ 4) { (5 þ 6) « 16 { 30


(i)

« 46
„¬@O (ii) #°O_ (5 þ 5) { (7 { 9) « 25 { 63
« 88
\ÿÿ„¬@O (iii) #°O_ (8 þ 5) { (6 þ 7) ª^Î#: ㄬH›ø „¬@O…Õ

Y
« 40 { 42 « 82 (3 × 2) + 1 = 7

\ Hê=ô# q°‹²žOQ· #O|~Ÿ (?) « 82

M
(5 × 2) + 1 = 11

2. D ãH÷Ok „¬\쁅Õx q°‹²žOQ· #O|~Ÿ (?)#° H›#°Qù#O_. (7 × 2) + 1 = 15

E
\ (2 × 2) + 1 = 5

D
9
  
    

59
  
CA
95
     
2.
  

50
ª^Î#:
98
A
(2 + 3 + 4) (2 + 5 + 7) = (9) (14) = 126
:8
(5 + 4 + 3) (3 + 8 + 7) = (12) (18) = 216

ª^Î#: Ì„á „¬@O…Õ


E

(1 + 5 + x) (6 + 4 + 2) = 144 5
4 – 50 = 625 – 50 = 575
h

ÿÞÿ
.P

(6 + x) (12) = 144 4
IC

ÿÞÿ
6 – 50 = 1296 – 50 = 1246
AD

6 + x = 12 4
ÿÞÿ
7 – 50 = 2401 – 50 = 2351
x = 6.
\ 4
AB
O

8 – 50 = 4096 – 50 = 4046
‹¬¶KÇ# (3-4) :
ER
CH

3. D ãH÷Ok „¬\쁅Õx q°‹²žOQ· #O|~Ÿ (?)#° H›#°Qù#O_? 3.


YD
,H
T
AM

ª^Î#: „¬@O #°O_ 9 { 4 « 13 =°i†Çò 8 { 5 « 13 ª^Î#: A = 1, B = 2, = 3 ......... Z = 26 #° J#°‹¬iOz


(i)
H
M

„¬@O (ii) #°O_ 12 { 7 « 19 =°i†Çò 10 { 9 « 19 C = 3 ; I = 9 ; R = 18. J^Í q^ÎOQê X = 24.


AM
IG

„¬@O (iii) #°O_ 12 { 8 « 20 =°i†Çò 4.


16 { x « 20 Þÿx = 4.
KH
R

4. ª^Î#: Ì„á „¬@O…Õ 10 × 20 = 200

20 × 30 = 600
(S.I. 2012)
\ 10 × 30 = 300.
ª^Î#: "³ò^Î\ ÷ „¬@O…Õ (56 { 15) - (22 { 8)
« 71 - 30 « 41 5. (Group IV 2012)

ï~O_È= „¬@O…Õ (46 { 9) - (10 { 6)


« 55 - 16 « 39
J^Íq^ÎOQê, (34{11) - (14{6) « 45-20 « 25. ª^Î#: Ì„á „¬@O…Õ (8 þ 2) þ 2 « 16 þ 2 « 32
(6 þ 7) þ 2 « 42 þ 2 « 84
REASONING ABILITY (VERBAL) ○ ○ ○ ○ ○ ○ ○ ○ ○
137
○ ○ ○ ○ ○ ○ ○ ○ ○ ○ ○ ○ ○ ○ ○ ○ ○ ○ ○
Study Material
(1 þ 5) þ 2 « 5 þ 2 « 10 3.
J^Íq^ÎOQê (4 þ 3) þ 2 « 12 þ 2 « 24
\ q°‹²žOQ· #O|~Ÿ « 24. ª^Î#: 4 + 5 = 9(I)

6.
9 + 5 = 14(N)

14 + 5 = 19(S)
m 
4
19 + 5 = 24 (X)

ª^Î#: "³ò^Î\ ÷ „¬@O…Õ 27 { 68 « 95 « 5 þ 19 4.


ï~O_È= „¬@O…Õ 51 { 84 « 135 « 5 þ 27
J^Íq^ÎOQê, 43 { 62 « 105 « 5 þ 21.
ª^Î#: " 
 + 
  1

7. 
, 
 $ 
  4

Y

% 
 , 
  5

M
ª^Î#: "³ò^Î\ ÷ „¬@O…Õx ‹¬OY¼x߆Çò 7 †³òH›ø Q®°}÷*쁰 

E
ï~O_È= „¬@O…Õx ‹¬OY¼x߆Çò 5 †³òH›ø Q®°}÷*쁰
5.

D
=ü_È= „¬@O…Õx ‹¬OY¼x߆Çò 11 †³òH›ø Q®°}÷*쁰.

9
59
\ q°‹²žOQ· #O|~Ÿ « 11
CA
95
(Communication S.I. 2012)

  
  ª^Î#: "³ò^Î\ ÷ „¬@O…Õ  

50
8.
  
98
A
  

ï~O_È= „¬@O…Õ   
:8
ª^Î#: "³ò^Î\ ÷ „¬@O…Õ
E

(758 – 432) × 3 = 978


J^Íq^ÎOQê, =ü_È= „¬@O…Õ
h

\ÿÿJ^Íq^ÎOQê ï~O_È= „¬@O…Õ


.P

Y  
   Y      
IC

(968 – 128) × 3
.
AD

= 2520  

MODEL - 9
AB
O

MODEL - 8
ER

‹¬¶KÇ# (1-4) :
CH

‹¬¶KÇ# (1-5) : D ãH÷Ok "Œx…Õ '?Ñ Q®°~¡°ë ªÖ#O…Õ =KÇ°ó 1. D ãH÷Ok „¬\쁅Õx q°‹²žOQ· #O|~Ÿ / JH›Æ~Œxß (?)#°
YD

JH›Æ~Œxß ‹¬OY¼x H›#°Qù#°=ò. H›#°Qù#O_.


,H

1.
T
AM
H
M
AM
IG

ª^Î#: A = 1, B = 2, C = 3 ........... Z = 26 #°O_


KH

F(6) + I(9) = O(15) ª^Î#: „¬@O #°O_ 12 { 12 { 22 { 32


(i)

« 1 { 1 { 4 { 9 « 15
R

A(1) + J(10) = K(11)

J^Íq^ÎOQê E(5) + M(13) = R(18) „¬@O (ii) #°O_ 12 { 22 { 32 { 42


« 1 { 4 { 9 { 16 « 30
2. „¬@O (iii) #°O_ 12 { 52 { 22 { 42
« 1 { 25 { 4 { 16 « 46
\ÿ? « 46
ª^Î#: A = 1, B = 2, C = 3 .............. Z = 26 #° J#°‹¬iOz 2.
T(20) × D(4) = Þÿ 80 × 2 = 160 ; F(6) × I(9) = 54

Þÿ 54 × 2 = 108

\ÿÿ R(18) × E(5) = 90 Þÿ 90 × 2 = 180. ª^Î#: ㄬu J_È°Û =~¡°‹¬…Õ A, B =°i†Çò C ° U^Ë XH› ªÖ#O…Õ
=ô<Œß~ò. =°i†Çò J_È°Û =~¡°‹¬…Õx "³ò^Î\ ÷ ~ï O_È° JH›~Æ Œ
REASONING ABILITY (VERBAL) ○ ○ ○ ○ ○ ○ ○ ○ ○
138
○ ○ ○ ○ ○ ○ ○ ○ ○ ○ ○ ○ ○ ○ ○ ○ ○ ○ ○
Study Material
H÷ O ^Î W=Þ|_ # ‹¬ O Y¼ "³ ò `Ç ë O =ü_È = JH› Æ ~ ¡ O MODEL - 10
H›ey†Çò#ßk.
Hê=ô# q°‹²žOQ· „¬^ÎO « A9+3 = A12 1. 1, 2, 4, 8, , 32 =~¡°‹¬ ãH›=°O…Õ q°‹²žOQ· #O|~Ÿ#°
3. H›#°Qù#O_.
ª^Î#: 1, 2, 4, 8, @ , 32 ã‰õ}÷…Õ "³ò^Î\ ÷ ‹¬OY¼ `DŽ¬æ
q°ye#q Jx߆Çò "Œ\÷ =òO^ΰ ‹¬OY¼ä›½ ï~\÷“O„¬ô.
Hê=ô# '?Ñ ªÖ#O…Õx ‹¬OY¼ 16.
ª^Î#: Wzó# „¬\ ÷“H› †³òH›ø x°=ô =~¡°‹¬#° Q®=°xOKÇQê
2. 5, 9, 17, 29, 45,
"; %8 (5

#:
 
&7
 
)4

ª^Î#: Wzó# ãH›=°O {4, {8, {12, {16......... „¬^Μux
     
$9 '6 *3 J#°‹¬i‹¬°ëOk.
\ÿ? « HS Hê=ô# q°‹²žOQ· #O|~Ÿ « 45 { 20 « 65.

Y
4. 3. 1, 2, 3, 5, 8,
ª^Î#: Wzó# ãH› = °O…Õ "³ ò ^Î \ ÷ ï ~ O_È ° „¬ ^ ¥° `Ç „ ¬ æ

M
q°ye#=x߆Çò "Œ\÷ =òO^ΰ ï~O_È° „¬^¥ "³ò`ŒëxH÷

E
ª^Î#: ㄬu J_È°Û =~¡°‹¬…Õ A, B =°i†Çò C ° U^Ë XH› ªÖ#O…Õ ‹¬=¶#O.
Hê=ô# q°‹²žOQ· #O|~Ÿ « 5 { 8 « 13.

D
=ô<Œß~ò. =°i†Çò ㄬu x°=ô =~¡°‹¬…Õx "³ò^Î\ ÷

9
59
=°i†Çò =ü_È= ‹¬OY¼ |œO =°^Î¼ JH›Æ~¡O J=ô`Ç°Ok. 4. 18, 17, 19, 20, 15, 21, 14
CA
95
J#Qê 2 þ 4 « 8. ª^Î#: Wzó# ãH›=°O -1, {2, -3, {4, -5, {6....... „¬^Μux
\ q°‹²žOQ· „¬^ÎO « 8C
50
J#°‹¬i‹¬°ëOk.
98
A
Hê=ô# q°‹²žOQ· #O|~Ÿ « 19 - 3 « 16.
123456789012345678901234567890121234567890123456789012345678901212
:8
123456789012345678901234567890121234567890123456789012345678901212
12345678901234567890123456789012123456789012345678901234567890121234567890123456789012345678901212345678901234567890123456789012123
1234567890123456789012345678901212345678901234567890123456789012123456789012345678901234567890
12345678901234567890123456789012123456789012345678901234567890121234567890123456789012345678901212345678901234567890123
1234567890123456789012345678901212345678901234567890123456789012123456789012345678901234567890121234567890123
1234567890123456789012345678901212345678901234567890123456789012123456789012345678901234567890121234
12345678901234567890123456789012123456789012345678901234567890121234567890123456789012345678901212345678901234567890123456789012123
1234567890123456789012345678901212345678901234567890123456789012123456789012345678901234567890
12345678901234567890123456789012123456789012345678901234567890121234567890123456789012345678901212345678901234567890123
1234567890123456789012345678901212345678901234567890123456789012123456789012345678901234567890121234567890123
1234567890123456789012345678901212345678901234567890123456789012123456789012345678901234567890121234
123456789012345678901234567890121234567890123456789012345678901212
12345678901234567890123456789012123456789012345678901234567890121234567890123456789012345678901212345678901234567890123456789012123
1234567890123456789012345678901212345678901234567890123456789012123456789012345678901234567890
12345678901234567890123456789012123456789012345678901234567890121234567890123456789012345678901212345678901234567890123
1234567890123456789012345678901212345678901234567890123456789012123456789012345678901234567890121234567890123
1234567890123456789012345678901212345678901234567890123456789012123456789012345678901234567890121234
PRACTICE TEST
12345678901234567890123456789012123456789012345678901234567890121234567890123456789012345678901212345678901234567890123456789012123
1234567890123456789012345678901212345678901234567890123456789012123456789012345678901234567890
123456789012345678901234567890121234567890123456789012345678901212
E

12345678901234567890123456789012123456789012345678901234567890121234567890123456789012345678901212345678901234567890123
1234567890123456789012345678901212345678901234567890123456789012123456789012345678901234567890121234567890123
1234567890123456789012345678901212345678901234567890123456789012123456789012345678901234567890121234
12345678901234567890123456789012123456789012345678901234567890121234567890123456789012345678901212345678901234567890123456789012123
1234567890123456789012345678901212345678901234567890123456789012123456789012345678901234567890
123456789012345678901234567890121234567890123456789012345678901212
12345678901234567890123456789012123456789012345678901234567890121234567890123456789012345678901212345678901234567890123
1234567890123456789012345678901212345678901234567890123456789012123456789012345678901234567890121234567890123
1234567890123456789012345678901212345678901234567890123456789012123456789012345678901234567890121234
h

12345678901234567890123456789012123456789012345678901234567890121234567890123456789012345678901212345678901234567890123456789012123
1234567890123456789012345678901212345678901234567890123456789012123456789012345678901234567890
12345678901234567890123456789012123456789012345678901234567890121234567890123456789012345678901212345678901234567890123
1234567890123456789012345678901212345678901234567890123456789012123456789012345678901234567890121234567890123
1234567890123456789012345678901212345678901234567890123456789012123456789012345678901234567890121234
12345678901234567890123456789012123456789012345678901234567890121234567890123456789012345678901212345678901234567890123456789012123
1234567890123456789012345678901212345678901234567890123456789012123456789012345678901234567890
12345678901234567890123456789012123456789012345678901234567890121234567890123456789012345678901212345678901234567890123456789012123
1234567890123456789012345678901212345678901234567890123456789012123456789012345678901234567890
.P
IC
AD

‹¬¶KÇ# (1-20) : D ãH÷Ok "Œx…Õ ㄬ‰§ß~¡ÖH›O L#ßKË@ =KÇ°ó 1 2 3 6


3 7 ?
‹¬OY¼#° H›#°Qù#°=ò?
AB
O

6.
4 5
ER
CH

1 2 2 3 3 4 1) 11 2) 12 3) 13 4) 14
18 96 ?
1.
YD

3 4 5
,H

1) 300 2) 200 3) 100 4) 50


7. 7 37 (GROUP-I-2004)
T
AM

1 2 2 3 3 4
12 72 ?
2.
H

3 4 5
M

9 8
AM

1) 240 2) 180 3) 120 4) 60


IG

43 ?
KH

? 2 1) 40 2) 41 3) 39 4) 32
R

9 3
3. 1 3 5
7 4
5 5 8. 5 19
2 4 6
1) 10 2) 11 3) 12 4) 13
1) 11 2) 30 3) 31 4) 41

1 2 2 3 3 4
4. 6 24 ?
4
3 4 5 9. (GROUP-I-2004)
3 64 5
1) 11 2) 12 3) 35 4) 60 27 125
? 216
1 2 2 3 3 4 8 343 6
5. 6 9 ?
7
3 4 5
1) 11 2) 12 3) 35 4) 60
1) 512 2) 215 3) 64 4) UnHê^ΰ
REASONING ABILITY (VERBAL) ○ ○ ○ ○ ○ ○ ○ ○ ○
139
○ ○ ○ ○ ○ ○ ○ ○ ○ ○ ○ ○ ○ ○ ○ ○ ○ ○ ○
Study Material
1 3 5 0
? 4
10. 2 12 ?
15. 6 3
2 4 6
1) 11 2) 30 3) 22 4) 32 2
5
1 3 5 9
1) 1 2) 8 3) 7 4) UnHê^ΰ
3 7 ?
11. 16.    (S.I.-2007)
2 4 6   
1) 15 2) 11 3) 16 4) 21
 
  
8 11
1) 11 2) 6 3) 3 4) 2

Y
12. 8 32 5 7 ? 5 (J.L.-2004)
1 2
17.

M
9 8 ? 1
1) 2 2) 4 3) 8 4) ̄á=x߆Çò

E
1) 35 2) 53 3) 77 4) 88 ?

D
9
49

59
? 25 9 3
18.

CA
95
13. 2 4 5

50
1) 15 2) 17 3) 19 4) 25

98
A
5 81
:8
7 ? 2
E

19.
5 3
h

1) 9 2) 36 3) 6 4) 64
.P
IC

1) 7 2) 9 3) 8 4) 1
AD

1
14. 15 2 17 (S.I.-2007) ?
AB
O

? 16
4 6
3 18 5 4 2
ER

20.
CH

19
YD

3
1) 13 2) 14 3) 20 4) 21
,H

1) 5 2) 6 3) 7 4) 1
T

1234567890123456789012
12345678901234567890123456789012123456789012345678901234567890121234567890123456789012345678901212345678901234567890123
12345678901234567890123
AM

q=~¡}ì`ÇàH› ["Œ|°°
12345678901234567890123456789012123456789012345678901234567890121234567890123456789012345678901212345678901234567890123
1234567890123456789012
1234567890123456789012 12345678901234567890123
12345678901234567890123456789012123456789012345678901234567890121234567890123456789012345678901212345678901234567890123
1234567890123456789012
12345678901234567890123
12345678901234567890123
12345678901234567890123456789012123456789012345678901234567890121234567890123456789012345678901212345678901234567890123
PRACTICE TEST :
H

1234567890123456789012
12345678901234567890123456789012123456789012345678901234567890121234567890123456789012345678901212345678901234567890123
1234567890123456789012 12345678901234567890123
12345678901234567890123
12345678901234567890123456789012123456789012345678901234567890121234567890123456789012345678901212345678901234567890123
M

1234567890123456789012 12345678901234567890123
12345678901234567890123456789012123456789012345678901234567890121234567890123456789012345678901212345678901234567890123
AM
IG

1. (1) 3 þ 4 þ (5)2 « 3 þ 4 þ 25 « 300. 37


KH

2. (1) 3 þ (4)2 þ 5 « 3 þ 16 þ 5 « 240. 7


R

? 2
2þ2{1«5 7. (1)
9 8
9 3 3þ2{1«7 43 ?
3. (2)
4þ2{1«9
7 4
5 þ 2 { 1 « 11 7 + 1 = 8 37 + 3 = 
5 5
8 + 1 = 9 40 + 3 = 43

2{1«3 5{2«7 8. (4) 5{ (6)2 « 5 { 36 « 41.


3{1«4 7{2«9
4{1«5 9 { 2 « 
4
3 64 5
4. (4) 3 þ 4 þ 5 « 60. 9. (1)
27
?
125
216
83 « 512
3 { 4 { 5 « 12. 8 343 6
5. (2)
7
6. (1) 5 { 6 « 11.
REASONING ABILITY (VERBAL) ○ ○ ○ ○ ○ ○ ○ ○ ○
140
○ ○ ○ ○ ○ ○ ○ ○ ○ ○ ○ ○ ○ ○ ○ ○ ○ ○ ○
Study Material
10. (2) 5 þ 6 « 30. 15. (3) Z^ΰï~^ΰ~¡° "Œ\÷x 䛀_`Í 9 =‹¬°ëOk.
11. (2) 5 { 6 « 11.  ×    ×  
8 11 16. (3) = =  = =  J^Í q^ÎOQê
   
12. (2) 8 32 5 7 5  × Y 
53 = = Y = 
 
9 8
8 þ 9 « 72 11 þ 8 « 88 Y
8 þ 5 « -40 7 þ 5 « -35 17. (4) 1H÷ Z^ΰ~¡°Qê 1 …èH› 12«1 …èH› 13«1 J>èÁ 2䛽
32 53 Z^ΰ~¡°Qê 2 …èH› 22«4 …èH› 23«8.
13. (1) ‹¬OY¼ä›½ Z^ΰ~¡° "Œ\÷ =~Œ¾° H›=ô. 18. (4) 3䛽 Z^ΰ~¡°Qê 32«9 J>èÁ 5䛽 Z^ΰ~¡°Qê 52«25.
14. (2)
1 19. (1) 2, 3, 5° "³ò^Î\ ÷ =ü_È° ㄬ^¥ # ‹¬OY¼°. <Œ°Q®=k

Y
15
?
2
16
17 7.

M
3
4
18
6
5 20. (4) 2, 2{1«3, 3{1«4, 4{1«5 …èH› 1, 2, 3, 4
19 =~¡°‹¬ ‹¬OY¼°. 1, 5° ["Œ|°°. J~ò`Í 1x

E
"³ò^Î\ ÷ ["Œ|°Qê f‹¬°HË"Œe.

D
16 + 6 = 22 18 + 4 = 22 15 + 1 = 16

9
59
17 + 5 = 22 19 + 3 = 22 2 + x = 16

CA
95
x = 16 – 2

Y  

50
123456789012345678901234567890121234567890123456789012345678901212
98
A
12345678901234567890123456789012123456789012345678901234567890121234567890123456789012345678901212345678901234567890123
1234567890123456789012345678901212345678901234567890123456789012123456789012345678901234567890121234567890123
1234567890123456789012345678901212345678901234567890123456789012123456789012345678901234567890121234
12345678901234567890123456789012123456789012345678901234567890121234567890123456789012345678901212345678901234567890123456789012123
123456789012345678901234567890121234567890123456789012345678901212345678901234567890123
12345678901234567890123456789012123456789012345678901234567890121234567890123456789012345678901212345678901234567890123
1234567890123456789012345678901212345678901234567890123456789012123456789012345678901234567890121234567890123
1234567890123456789012345678901212345678901234567890123456789012123456789012345678901234567890121234
123456789012345678901234567890121234567890123456789012345678901212
12345678901234567890123456789012123456789012345678901234567890121234567890123456789012345678901212345678901234567890123456789012123
123456789012345678901234567890121234567890123456789012345678901212345678901234567890123
12345678901234567890123456789012123456789012345678901234567890121234567890123456789012345678901212345678901234567890123
1234567890123456789012345678901212345678901234567890123456789012123456789012345678901234567890121234567890123
1234567890123456789012345678901212345678901234567890123456789012123456789012345678901234567890121234
12345678901234567890123456789012123456789012345678901234567890121234567890123456789012345678901212345678901234567890123456789012123
123456789012345678901234567890121234567890123456789012345678901212345678901234567890123
123456789012345678901234567890121234567890123456789012345678901212
PREVIOUS BITS
12345678901234567890123456789012123456789012345678901234567890121234567890123456789012345678901212345678901234567890123456789012123
123456789012345678901234567890121234567890123456789012345678901212345678901234567890123
12345678901234567890123456789012123456789012345678901234567890121234567890123456789012345678901212345678901234567890123
1234567890123456789012345678901212345678901234567890123456789012123456789012345678901234567890121234567890123
1234567890123456789012345678901212345678901234567890123456789012123456789012345678901234567890121234
:8
123456789012345678901234567890121234567890123456789012345678901212
12345678901234567890123456789012123456789012345678901234567890121234567890123456789012345678901212345678901234567890123456789012123
123456789012345678901234567890121234567890123456789012345678901212345678901234567890123
12345678901234567890123456789012123456789012345678901234567890121234567890123456789012345678901212345678901234567890123
1234567890123456789012345678901212345678901234567890123456789012123456789012345678901234567890121234567890123
1234567890123456789012345678901212345678901234567890123456789012123456789012345678901234567890121234
12345678901234567890123456789012123456789012345678901234567890121234567890123456789012345678901212345678901234567890123456789012123
123456789012345678901234567890121234567890123456789012345678901212345678901234567890123
12345678901234567890123456789012123456789012345678901234567890121234567890123456789012345678901212345678901234567890123
1234567890123456789012345678901212345678901234567890123456789012123456789012345678901234567890121234567890123
1234567890123456789012345678901212345678901234567890123456789012123456789012345678901234567890121234
123456789012345678901234567890121234567890123456789012345678901212
12345678901234567890123456789012123456789012345678901234567890121234567890123456789012345678901212345678901234567890123456789012123
123456789012345678901234567890121234567890123456789012345678901212345678901234567890123
12345678901234567890123456789012123456789012345678901234567890121234567890123456789012345678901212345678901234567890123
1234567890123456789012345678901212345678901234567890123456789012123456789012345678901234567890121234567890123
1234567890123456789012345678901212345678901234567890123456789012123456789012345678901234567890121234
12345678901234567890123456789012123456789012345678901234567890121234567890123456789012345678901212345678901234567890123456789012123
123456789012345678901234567890121234567890123456789012345678901212345678901234567890123
E
h
.P

1. ãH÷O^Î Wzó# =ü_È= zã`ÇO…Õ '?Ñ Q®°~¡°ë L#ß ªÖ#O…Õ 5. ãH÷ O k „¬ @ =ò#° „¬ i jeOz P, Q q°=#°
IC
AD

…Õ„²Oz# ‹¬OY¼ (A.P.Constable Prelims -2016) H›#°Qù#O_. (TS. Constable Mains -2016)
AB
O

  
    
       1
ER
CH

      2
YD

1) 155 2) 165 3) 175 4) 185 1) 4, 8 2) 5, 7 3) 5, 6 4) 4, 7


2. ãH÷Ok „¬\ ÷“H›…Õ …Õ„²Oz# ‹¬OY¼
,H

(A.P.Const. Mains-'16)
ãH÷Ok Wzó# 6, 7 ㄬ‰×߁ „¬@=ò…Õ ‹¬OY¼° XH›
T
AM

   ãH›=° „¬^Μu…Õ L<Œß~ò. (?) ̄\÷“#KË@ =KÍó ‹¬OY¼


H

   H›#°HËøO_.
M

(TS. S.I. Prelims -2016)


AM

        
IG

6.
       
1) 9 2) 10 3) 11 4) 12
KH

3. ㄬ‰§ß~¡HÖ =
› ò (?) KË@ LO_È=‹²# ‹¬OY¼#° H›#°Qù#O_.
1) 9 2) 8 3) 7 4) 6
R

(TS. Constable Mains -2016) 7.       


       
   1) 16 2) 8 3) 10 4) 2



 
ãH÷Ok Wzó# „¬@=ò…Õ ‹¬OY¼° XH› ãH›=°„¬^Μu…Õ
  
L<Œß~ò. D ‹¬=¶KŒ~¡=ò L„¬†³¶yOz, ㄬ‰×߁° 8, 9,
1) 21 2) 22 3) 29 4) 20
10 …Õ (?) ̄\÷“# KË@ =KÍó ‹¬OY¼#° H›#°HËøO_.
4. ãH÷Ok"Œx…Õ '?Ñ L#ßKË@ ~Œ=‹²# ^¥xx H›#°HËøO_.
(TS. S.I. Prelims -2016)
(TS. Constable Mains -2016)

     8.  
     

1) 6 2) 15 3) 7 4) 9 1) 26 2) 28 3) 30 4) 32
REASONING ABILITY (VERBAL) ○ ○ ○ ○ ○ ○ ○ ○ ○
141
○ ○ ○ ○ ○ ○ ○ ○ ○ ○ ○ ○ ○ ○ ○ ○ ○ ○ ○
Study Material
17. 4 9 9 16 16 ? (S.I-2012)

9.   

6 12 20
1) 3 2) 4 3) 5 4) 6
 1) 45 2) 35 3) 21 4) 25

10.   18.
 4 7 (S.I-2012)

5 1 64 3
 
1) 2 2) 3) 4) 1 11 27 ? 8
 
11. D ãH÷Ok "Œx…Õ `DŽ²æ‡é~ò# ^¥xx H›#°Qù#O_? 8 2

Y
(R.R.B. -2014)
1) 81 2) 125 3) 216 4) 127

M
- ( )
19. 13 12 5 (S.I-2012)

$ 8 "

E
17 15 8
+ 0

D
9
25 24 ?

59
1) A 2) B 3) D 4) G
29 21 20
CA
95
12. D ãH÷Ok "Œx…Õ `DŽ²æ‡é~ò# JH›Æ~Œxß H›#°Qù#O_?
1) 15 2) 11 3) 9 4) 7

50
(R.R.B. -2014)

20. ? ‹¬OöH`Ç=ò =^Îí U ‹¬OY¼#° LOKÇ=…ÿ#°?


98
A
; # 9 % ' 5
:8
" : $ 8 & 6 ( (H›=ü¼xöH+¬<£ S.I-2011)
E

2 4 3 9 1 5
h

1) V 2) F 3) G 4) Y
.P
IC

13. DãH÷Ok "Œx…Õ `DŽ²æ‡é~ò# JH›Æ~Œxß H›#°HËøO_?


AD

(R.R.B. -2014)
90 ?
AB
O

20
" 1 : $ 7 $ ( : , 1) 20 2) 25 3) 26 4) 75
ER
CH

6 4 & ( #
‹¬¶KÇ# (21-23) : ㄬ‰×ß Q®°~¡°ë (?) 䛽 |^ΰ°Qê ãH÷Ok
YD

1) L 2) K 3) J 4) I
ㄬ`Œ¼=¶ß†Ç°O #°O_ `Çy# ["Œ=ò#° Z#°ßHù#°=ò?
,H

14. ãH÷Ok"Œx…Õ `DŽ²æ‡é~ò# ^¥xx H›#°HËøO_?


T
AM

(S.I-2008)
(R.R.B. -2014) 4 3 2 9 8
H

21.
M

# , ' 5 2 ? 35
AM
IG

( "
1) 5 2) 6 3) 11 4) 8
- 2
KH

1) 2) 3) 4) 22. 6 18 15
R

S V T U

15. D ãH÷Ok"Œx…Õ `DŽ²æ‡é~ò# ‹¬OY¼#° H›#°HËøO_? 3 2 5


(R.R.B. -2014)
4 3 ?
     8 27 9
     
1) 11 2) 6 3) 3 4) 2
1) 0 2) 3 3) 5 4) 7
23. 1
16. (S.I-2012) 13 217
? 16
56 22 46 10 34 14
41 4 6
39 ? 3 18 5
15 8 9 6 11 6
19
1) 25 2) 48 3) 12 4) 52
1) 13 2) 12 3) 20 4) 21
REASONING ABILITY (VERBAL) ○ ○ ○ ○ ○ ○ ○ ○ ○
142
○ ○ ○ ○ ○ ○ ○ ○ ○ ○ ○ ○ ○ ○ ○ ○ ○ ○ ○
Study Material
‹¬¶KÇ# (24-32):`DŽ²æ# ‹¬OY¼#° Q®°iëOKÇ°=ò (S.I-2007) 32. ㄬ‰§ß~¡ÖH›O L#ßKË@ LO_È=‹²# ‹¬OY¼? (DL-2012)

24. ㄬ‰§ß~¡Ö¡œH›O L#ßKË@ LO_È=‹²# ‹¬OY¼? (PL-2013) 3


1089 12
5 7 18
6 93 15 9 ? 5 4 50 1
363 39
3 6 8
?
1) 5 2) 19 3) 27 4) 89 1) 140 2) 165 3) 112 4) 120
25. ? 1 2 33. ㄬ‰§ß~¡œH›O L#ßKË@ LO_È=‹²# ‹¬OY¼? (DL-2012)

21 22 40
6 8 ?
1 2 5
29 19 42 22 49 23

Y
20 23 43
1) 8 2) 7 3) 9 4) 11

M
1) 5 2) 14 3) 3 4) 2
26. 6 4
34. ㄬ‰§ß~¡ÖH›O L#ßKË@ LO_È=‹²# ‹¬OY¼? (PL-2012)
7 5

E
5 3 ? 29
19 ? 8
5

D
9
4 6 9 72

59
1) 25 2) 37 3) 41 4) 47 42 5
CA
95
27. A 22 I 12 ? (Group-I-2012) 6 30

50
26 E 18 O ?
1) 100 2) 80 3) 90 4) 94
98
A
1) U/6 2) Q/15 3) P/13 4) R/14
35. ㄬ‰§ß~¡ÖH›O L#ßKË@ LO_È=‹²# ‹¬OY¼?
:8
(PL-2012)
28. 8 4 5 6 9
E

3 (Group-I-2012)
3 4 4
h
.P

7 5 3 6 9
IC

2
169 112 ?
AD

4 8 8 5 5 6
AB
O

67 29 ?
2 1 4
ER

1) 47 2) 67 3) 87 4) 57
CH

1) 156 2) 192 3) 174 4) 173


29. „¬\ ÷“H›…Õ °„¬ë"³°Ø# ‹¬OY¼?
YD

(Group-II-2012)
36. ㄬ‰§ß~¡ÖH›O L#ßKË@ LO_È=‹²# ‹¬OY¼?
? 13 49
,H

7 (Ass. Engg in AP& ME-2012)


T

9 17 69
AM

56 1) 120 2) 125
4 28 ? 8
H

13 11 59
M

60 3) 130 4) 135
1) 5 2) 10 3) 11 4) 1
AM
IG

15
30. D ãH÷O^Î W=Þ|_# „¬@O…Õ °„¬ë"³°Ø# ‹¬OY¼
KH

37. ㄬ‰§ß~¡œH›O L#ßKË@ LO_È=‹²# ‹¬OY¼?


14 18 (Group-II-2012)
R

E M (Ass. Directors in E&S-2012)


? 51 25 15
K 55 52 D
12 33 31 31 1) 63 2) 78
F 72 ? G
13 20
L I 3) 45 4) 54
1) 11 2) 13 3) 15 4) 17
31. ㄬ‰§ß~¡ÖH›O L#ßKË@ LO_È=‹²# ‹¬OY¼? (DL-2012)
38. ㄬ‰§ß~¡Ö¡œH›O L#ßKË@ LO_È=‹²# ‹¬OY¼?
(Junior A/c in Muncipal-2012)
4 7 6 1) 96
5 34 6 9 61 6 7 ? 5 2) 72 82 62 74 32 63 45
3) 81 80 55
1) 37 2) 41 3) 47 4) 45 ?
4) 84
REASONING ABILITY (VERBAL) ○ ○ ○ ○ ○ ○ ○ ○ ○
143
○ ○ ○ ○ ○ ○ ○ ○ ○ ○ ○ ○ ○ ○ ○ ○ ○ ○ ○
Study Material
12345678901234567890123456789012123456789012345678901234567890121234567890123456789012345678901212345678901234567890123
1234567890123456789012 123456789012345678901234

q=~¡}ì`ÇàH› ["Œ|°°
12345678901234567890123456789012123456789012345678901234567890121234567890123456789012345678901212345678901234567890123
1234567890123456789012 123456789012345678901234
12345678901234567890123456789012123456789012345678901234567890121234567890123456789012345678901212345678901234567890123
1234567890123456789012 123456789012345678901234

1234567890123456789012
PREVIOUS BITS :
12345678901234567890123456789012123456789012345678901234567890121234567890123456789012345678901212345678901234567890123
1234567890123456789012 123456789012345678901234
12345678901234567890123456789012123456789012345678901234567890121234567890123456789012345678901212345678901234567890123
1234567890123456789012 123456789012345678901234
12345678901234567890123456789012123456789012345678901234567890121234567890123456789012345678901212345678901234567890123
1234567890123456789012 123456789012345678901234
12345678901234567890123456789012123456789012345678901234567890121234567890123456789012345678901212345678901234567890123
123456789012345678901234

 s  s   III : 12 × 3 = 36
1. (2) "³ò^Î\ ÷ „¬@O…Õ    
  12 3 = 4 ¸
 s  s   s  J^Íq^ÎOQê
ï~O_È= „¬@O…Õ  
  Y
28 × 3 = 84 IV :    Y  

J^Íq^ÎOQê =ü_È= „¬@O…Õ \ 10 × 2 = 20
 s  s  8. (1) W=Þ|_# „¬@O…Õ
  
 (6 þ 3) { (4 þ 2) « 18 { 8 « 26
2. (3) Wzó# „¬\ ÷“H›…Õ \ ? « 26
   9. (2) W=Þ|_# „¬@O…Õ (4 þ 1) { (5 þ x) « 24

Y
"³ò^Î\ ÷ x°=ô =~¡°‹¬…Õ   
  4 { 5x « 24

M
   5x « 20
ï~O_È= x°= =~¡°‹¬…Õ    
  ÿÿ\ x « 4

E
  
J^Íq^ÎOQê =ü_È= x°=ô =    ¥ ´
10. (3) „¬@O…Õ ¦ s µ  s Y
 

D
9
  § ¶

59
Þ 6 + 12x = 12
2 2 2
3. (1) "³ò^Î\ ÷ ƒç=°à…Õ « 1 { 3 { 5 « 35
CA
95
ï~O_È= ƒç=°à…Õ « 22 { 32 { 42 « 29 Þÿ12x = 6

50
J^Íq^ÎOQê =ü_È= ƒç=°à…Õ « 12 { 42 { 22
Þÿ Y    
98
A
« 1 { 16 { 4 « 21
\ ? = 21
:8
11. (2) W=Þ|_# JH›Æ~Œ…Õx "³ò^Î\ ÷, z=i JH›Æ~Œ †³òH›ø
E

4. (3) 2 = 4 ; 3 = 9
2 2 POQ®Á =~¡â=¶ J#°~¡¶„¬ ‹¬OY¼#° 䛀_ÈQê =zó#
h
.P

42 = 16 ; 62 = 36 ‹¬OY¼ä›½ J„¬‹=
¬ ¼k‰×…Õx =~¡=
â ¶…Õ U JH›~Æ O¡ J~ò`Í
IC
AD

52 = 25 ; 72 = 49 LO@°O^Ë ^¥x =°^Î¼…Õ ã"Œ†Ç¶e.


\ ?=7 J#Qê L = 12 ; H = 8
AB
O

5. (4) I : 4 + 3 – 1 = 6 12 + 8 = 20 = G (J„¬‹¬=¼k‰×…Õ ªÖ#O)


ER
CH

II : 8 + 9 – 2 = 15 C=3;A=1
YD

III : 5 + 8 – 3 = 10 3 + 1 = 4 = W (J„¬‹¬=¼k‰×…Õ ªÖ#O)


J…ìöQ
,H

IV : (7 + P) – 4 = Q ...... (1)
T

Ì„á ‹¬g°H›~¡}ìxß `Ç$„²ë„¬iKÍ P = 4 J + 0 = 10 + 15 = 25 = B (J„¬‹¬=¼k‰×…Õ ªÖ#O)


AM

Q=7 12. (1) Wzó# „¬\ ÷“H›…Õ ㄬu x°=ô =~¡°‹¬…Õ JH›Æ~¡O, ^¥x
H
M

6. (2) I : 3 + 2 = 9 + 4 = 13
2 2 =¼uö~H› JH›Æ~Œ° W=Þ|_¨Û~ò. nx ㄬHê~¡O KǶÀ‹ë
AM
IG

3 2 – 22 = 9 – 4 = 5 E H÷ =¼uö~H› JH›Æ~¡O 'V'.


KH

II : 72 + 32 = 49 + 9 = 58 13. (4) D ƒç=°à…Õ Z_È=°#°O_ 䛽_H÷ ‡é=ô Hùk JH›~Æ Œ


†³òH›ø ªÖ<Œä›½, 2, 3, 4° H›„¬|_È°`Ç°<Œß~ò. nx
R

72 – 32 = 49 – 9 = 40
III : 52 + 42 = 25 + 16 = 41 ㄬHê~¡O KǶÀ‹ë
52 – 42 = 25 – 16 = 9 ?=I
IV : x2 + 52 = 89 14. (3) Wzó# „¬\H“÷ …
› Õ x°=ô =~¡°‹¬…Õx ‹¬OY¼#° Q®=°xOz#
x2 = 89 – 25 = 64 32 – 3 = 9 – 3 = 6
x=8 82 – 8 = 64 – 8 = 56
\ 82 – 52 = 64 – 25 = 39 102 – 10 = 100 – 10 = 90
\ ?=8 22 – 2 = 4 – 2 = 2
7. (3) I : 4×2=8 12 – 1 = 1 – 1 = 0
¸
8 2=3 x2 – x = 20 ....... (1)
II : 6 × 2 = 12 (1) x `Ç$„²ë„¬iKÍ q°= '5'
¸
6 2=3 x=5 \
REASONING ABILITY (VERBAL) ○ ○ ○ ○ ○ ○ ○ ○ ○
144
○ ○ ○ ○ ○ ○ ○ ○ ○ ○ ○ ○ ○ ○ ○ ○ ○ ○ ○
Study Material
15. (2) Wzó# ƒç=°à…Õ "³ò^Î@Qê Z_È=°"³á„¬ô, ̄á#Qê Q® ï~O_È= „¬@O…Õ → 4{52 « 4{25«29
JH›~Æ O¡ …Õ "³ò^΁°Ì„\÷“ J„¬‹=
¬ ¼k‰×…Õ '5Ñ Ì„OKǰ䛽O@¶ J^Íq^ÎOQê
‡é`Í `Ç~ŒÞu JH›Æ~¡O =‹¬°ëOk. =ü_È= „¬@O…Õ → 6{92« x → x = 81+ 6 = 87
B +5 G +5 L +5 Q 29. (1) „¬\ ÷“H›…Õ
A +5 F ï~O_È= J_È°Û=~¡°‹¬…Õ → 2þ9{3þ17«69
\?Q +5 V =ü_È= J_È°Û=~¡°‹¬…Õ → 2þ13{3þ11 « 59
16. (1) 17. (4) 18. (3) 19. (4) J^Í q^ÎOQê
20. (3) "³ò^Î\ ÷ „¬@=ò…Õ 22{42 « 4{16«20 "³ò^Î\ ÷ J_È°Û=~¡°‹¬…Õ → 2þ x {3þ13« 49
ï~O_È= „¬@=ò…Õ 32{92 « 9{81«90 → 2 x = 49 − 39
J^Íq^ÎOQê → 2 x = 10 ⇒ x = 5
=ü_È= „¬@O…Õ 12{52 « 1{25«26 30. (4) Wzó# „¬@=ò#° ä›Æ½}âOQê „¬ijeOz#

Y
21. (1) 12{13«25Ñ 20{31«51Ñ 18{15«23

M
22. (3) „¬@=ò…Õx ∴ x + 14 = 31
"³ò^Î\ ÷ x°=ô =~¡°‹¬…Õ → (6 Ó 3) þ 4 « 2þ4«8 ⇒ x = 31 − 14 = 17

E
ï~O_È= x°=ô =~¡°‹¬…Õ → (18 Ó 2)þ3 31. (2)

D
9
« 9þ3 «27 "³ò^Î\ ÷ „¬@O…Õ → (5þ6){4 « 30{4 « 34

59
J^Í q^ÎOQê =ü_È= x°=ô =~¡°‹¬…Õ
CA ï~O_È= „¬@O…Õ → (9þ6){7 « 54{7 « 61

95
(15Ó5)þ x «9 =ü_È= „¬@O…Õ → (7þ5){6 « 35{6 « 41

50
⇒ 3× x = 9 32. (4) (39þ3){3 « 117{3«120
98
A
⇒x=3 33. (3)
:8
23. (2)
(29 + 19) = 48 = 6
E
h

24. (4) Wzó# „¬@O…Õ "³ò^Î\ ÷ „¬@O…Õ →


.P
IC

8 8
5þ15{3þ6 « 75{18 « 93
AD

42 + 22 64
7þ5{6þ9 « 35{64« 89 ï~O_È= „¬@O…Ë → = =8
AB
O

8 8
18þ1 { 8þ4 « 18{32 « 50 J^Íq^ÎOQê
ER
CH

25. (4) ï~O_È= x°=ô =~¡°‹¬…Õ → (22{2)-1


49 + 23 72
= =9
YD

« 24-1 « 23 =ü_È= „¬@O…Õ →


8 8
=ü_È= x°=ô =~¡°‹¬…Õ → (40{5)-2
,H

34. (3) Wzó# „¬@O…Õ


T

« 45-2 « 43
AM

5þ(5{1) « 5þ3«30
J^Í q^ÎOQê
H
M

"³ò^Î\ ÷ x°=ô =~¡°‹¬…Õ → (21{1)þ x «20 6þ(6{1) « 6þ7«42


AM
IG

→ 22þ x « 20 8þ(8{1) « 8þ9«72


KH

→ x « 22-20 « 2 9þ(9{1) « 9þ10«90


R

26. (3) 35. (2) (9þ4þ2) { (5þ4þ6) « 192


"³ò^Î\ ÷ „¬@O…Õ → (5þ3){4 « 15{4«19 36. (1) Wzó# „¬@O…Õ
=ü_È= „¬@O…Õ → (6þ4){5 « 24{5«29
7þ8«56' 4þ15«60' 7þ4«28
J^Íq^ÎOQê
∴ 15þ8 « 120
ï~O_È= „¬@O…Õ → (7þ5){6 « 35{6 « 41
27. (1) 37. (1) „¬@O…Õ M=13; D=4 → 13x4 = 52
J„¬‹¬=¼ k‰×…Õx POQ®Á =~¡â=¶ ㄬHê~¡O K=11 ; E=5 → 11x5 = 55
A=26, E=22, I=18, O=12 F=6; L=12 → 12x6 = 72
Pq^ÎOQê KǶÀ‹ë U=6. G=7; I=9 → 7x9 = 63
28. (3) 38. (3)
"³ò^Î\ ÷ „¬@O…Õ → 3{82 « 3{64 « 67

REASONING ABILITY (VERBAL) ○ ○ ○ ○ ○ ○ ○ ○ ○


145
○ ○ ○ ○ ○ ○ ○ ○ ○ ○ ○ ○ ○ ○ ○ ○ ○ ○ ○
Study Material
1 ㄬH›@#°
ㄬH›@#°

(Statements - Conclusions)
-
-
f~Œà<Œ°
f~Œà<Œ°

=òMì¼O‰§°...
G XH› "ŒH›¼=ò …è^¥ À„~Œ…Õx q+¬†Ç¶ #°O_ ãQ®‚²ìOz# q+¬†Ç¶#° 'f~Œà<Œ°Ñ JO\ì~¡°.
G D qƒìQ®O…Õ XH› ㄬH›@# =°i†Çò ^¥xH÷O^Î ï~O_È° f~Œà<Œ° W=Þ|_È`Œ~ò. Wzó# f~Œà<Œ…Õ ㄬH›@#䛽 ‹¬i‡é†Í°
f~Œà<Œxß ZOKÇ°HË=…ÿ#°.

Y
G Wzó# ㄬH›@# ㄬHê~¡O Uk ‹¬`ǼO Jx Jaã‡†Ç°„¬_È`Œ"³¶ J^Í f~Œà#O.

M
G ㄬH›@##° ä›Æ½}âOQê KÇkq J~¡ÖO K͋¬°HË"Œe. ㄬH›@#…Õx H©H› „¬^¥°, "Œ\÷ J~ŒÖ° `³°‹¬°HË"Œe. ㄬH›@#䛽 ãH÷O^Î
Wzó# ï~O_È° f~Œà<Œ#° 䛀_¨ ä›Æ½}âOQê KÇkq J~¡ÖO K͋¬°HË"Œe.

E
G Hùxß f~Œà<Œ° x[rq`ÇO…Õ J‹¬ž° ‹¬ï~á#q Hê^Îx Jx檐ë~ò. J~ò#„¬æ\÷H© XH›"Íˆ× Wzó# ㄬH›@##° x[OQê J#°‹¬iÀ‹ë

D
9
P f~Œà#O ‹¬ï~á#k Jx Q®°iëOKŒe.

59
G CA
Wzó# f~Œà<Œ° x[OQê ㄬH›@#䛽 ‹¬O|OkOz =ôO^¥ …è^¥ Jx Q®°iëOKŒe. Wk KŒ…ì =òY¼O. ZO^ΰH›O>è Hùxß

95
f~Œà<Œ#° x[rq`ÇO…Õ ‹¬ï~á#q J~ò#„¬æ\÷H© ㄬH›@##° J#°‹¬iOz#@°=O\÷ f~Œà#O ‹¬ï~á#k Hê^ΰ Jx Q®°iëOKŒe.

50
G XH› PiÖH› ‹¬O|O^Î"³°Ø# "Œ¼‹¬=ò …è^¥ XH› i‡é~¡°“ …è^¥ P…ÕKÇ##° ãÀ„ö~„²O„¬K͆Çò U^³á<Œ XH› JO‰×=ò䛽 ‹¬O|OkOz#
98
A
"ŒMì¼#O W=Þ|_È°#°. "Œ\÷H÷ ‹¬O|OkOz ㄬ`ǼH›ÆOQê Qêx …è^¥ „¬~ËH›ÆOQêQêx ãQ®‚²ìO„¬|_# f~Œà<Œ° Hùxß =òY¼"³°Ø#
:8
„¬^¥ P^¥~¡OQê ãQ®‚²ìO„¬|_È°#°. W=Þ|_# f~Œà<Œ#° q=~¡OQê q‰õÁ+²Oz JO`ÇsÁ#OQê =ô#ß JO‰§#° J~¡ÖO K͋¬°ä›½x
E
h
.P

`ŒiøH›OQê q‰õÁ+²OKŒe.
IC
AD

=¶ki ㄬ‰×߁°
AB
O
ER
CH

MODEL - 1
YD

ZO^ΰH›#Qê =¶~¡°æ J#°#k ㄬH$› u †³òH›ø ‹¬‚ì¬ [ ^~Î à¡ O.


^¥x ㄬHê~¡"Í° JO^Î~¡¶ #_È°KÇ°HùO\ì~¡°. ÿ Hê=ô#
,H

G Wzó# ㄬH›@# ãH÷Ok ï~O_È° f~Œà<Œ° I =°i†Çò II


T

‹¬=¶^¥#O (4).
AM

Wzó#„¬C_È° "³ò^Î\ ÷ f~Œà#O =¶ã`Ç"Í° ‹¬i†³Ø°#^³á`Í


› # : XH› H›Ōh †Ç¶[=¶#¼O `Ç# H›Ōh…Õx
H

2. ㄬH@
M

‹¬=¶^¥#O (1)Qê#¶, ï~O_È= f~Œà#O =¶ã`Ç"Í°


#°Q®°~¡° L^˼Q®°#° „²ez "Œ~¡O^Îih ö~„¬\ ÷…ÕQê
AM
IG

‹¬i†³Ø°#^³á`Í ‹¬=¶^¥#O (2)Qê#¶, "³ò^Î\ ÷ =°i†Çò


~Œr<Œ=¶ K͋² "³o¤‡é"Œx K³„²æ#k. …è#@Á~ò#
ï~O_È° f~Œà<Œ° ï~O_ȶ ‹¬i†³Ø°#"³á`Í ‹¬=¶^¥#O
KH

`Ç~¡°"Œu ~ËA `Œ"Í° "Œix q^Î° #°O_ `DŽ²æªë=°x


(3)Qê#¶, "³ò^Î\ ÷ =°i†Çò ï~O_È= f~Œà<Œ° ï~O_ȶ
R

K³„²æOk. J^Í~ËA =òQ®°¾~¡° L^˼Q®°° ~Œr<Œ=¶ K͋²


‹¬i†³Ø°#q HêH›‡é`Í ‹¬=¶^¥#O (4)Qê#¶, "³ò^Î\ ÷
"³o¤‡é†Ç¶~¡°.
…è^¥ ï~O_È= f~Œà<Œ…Õ U^Ë XH›\ ÷ ‹¬ï~á#^Î~ò`Í
f~Œà<Œ° : I) <Œ¾ = L^˼y `Ç ~ ¡ ° "Œ`Ç ~ËA
‹¬=¶^¥#O (5)Qê#¶ Q®°iëOKŒe. ~Œr<Œ=¶ Kͪë_È°.
1. ㄬH›@# : „¬ô~Œ`Ç# „¬^Μ`Ç°° H›#°=°~¡°ïQá ãHù`Çë II) `Ç~¡°"Œ`Ç ~ËA †Ç¶[=¶#¼O <Œ¾=
„¬^Μ`Ç°° J=°°…ÕH÷ =KŒó~ò. L^˼yx q^Î° #°O_ `DŽ²æªë~¡°.
f~Œà<Œ° : I) ㄬ‹°¬ ` ë `
Ç ~Ç O¡ "ŒiH÷ „¬ô~Œ`Ç# „¬^`
œÎ °Ç ° ª^Î#: †Ç¶[=¶#¼O K³„²æ#^¥x ㄬHê~¡O ~Œr<Œ=¶ K³†Ç¶¼e.
z~Œä›½ `³„²æOKŒ~ò. …è^ÎO>è q^Î° #°O_ `DŽ²æªë~¡°. Hê=ô# U^ËXH›\ ÷
II) „¬ô~Œ`Ç# „¬^Μ`Ç°° ‡\÷OKÇ_ÈO KŒ…ì
`DŽ¬æx‹¬i. Hê|\÷“ f~Œà#O-I …è^¥ f~Œà#O-II ……Õ
H›+¬“O. U^ËXH›\ ÷ ‹¬i†³Ø°#k. Hê=ô# ‹¬=¶^¥#O (5).
ª^Î#: f~Œà#O-I =°i†Çò f~Œà#O-II ~ ï O_ȶ ‹¬i†³°Ø #q Hê=ô.

REASONING ABILITY (LOGICAL) ○ ○ ○ ○ ○ ○ ○ ○


146
○ ○ ○ ○ ○ ○ ○ ○ ○ ○ ○ ○ ○ ○ ○ ○ ○ ○ ○
Study Material
2. ㄬH›@# : QË~¡°"³KóÇ x h~¡° =°i†Çò ‹¬|°Ä L„¬†¶
³ yOz
MODEL - 2
À+qOQ· K͋²# =°$^ΰ=ôQê À+"£ K͋¬°Hù#
G XH› ㄬH›@# ãH÷O^Î 3 …è^¥ 4 f~Œà<Œ° Wzó#„¬ô_È°, =KÇ°ó#°.
ãH÷O^Î W=Þ|_# P„¬Â<£…Õ Uk ‹¬i†³Ø°#^Ë ZOKÇ°Hùx f~Œà<Œ° : I) ‹¬|°Ä L„¬†³¶yOKÇ䛽O_¨ À+qOQ· K͆ǰ
‹¬=¶^¥#O Q®°iëOKŒe. 䛀_È^ΰ.
1. ㄬH›@# : =°Oã`Ç°O^Î~¡¶ "Œi"Œi ‹¬ÞO`Ç Hê~¡Á…Õ II) KǁÁx h~¡° L„¬†³¶yOz À+qOQ· K͆ǰ
|‚²ì~¡OQ® ‹¬=¶"͉§xH÷ ‚¬ð[ï~á<Œ~¡°. 䛀_È^ΰ.
f~Œà<Œ° : I) =°Oã`Ç°O^Î~¡¶ ^Î#=O`Ç°°.
III) À+qOQ· K͆ǰ_¨xH÷ W^ùH›ø>è J#°"³á#
II) =°Oã`Ç°O^ÎiH© Hê~¡°Á =ô<Œß~ò.
„¬^Μu.
III) =°Oã`Ç°° |‚²ì~¡OQ® ‹¬=¶"͉§xH÷
1) I =¶ã`Ç"Í° ‹¬i†³Ø°#k
‚¬ð[ï~á<Œ~¡°. 2) II =¶ã`Ç"Í° ‹¬i†³Ø°#k.
1) UnHê^ΰ. 2) I, II =¶ã`Ç"Í° ‹¬i†³Ø°#q.

Y
3) II, III =¶ã`Ç"Í° ‹¬i†³Ø°#q.
3) I, III =¶ã`Ç"°Í ‹¬i†³°Ø #q. 4) I, II, III =¶ã`Ç"°Í ‹¬i†³°Ø #q.

M
4) I, II =¶ã`Ç"Í° ‹¬i†³Ø°#q.
5) f~Œà#O II, III =¶ã`Ç"Í° ‹¬i†³Ø°#q.
5) I,II, III =¶ã`Ç"Í° ‹¬i†³Ø°#q Hê^ΰ.
ª^Î#: =°Oã`Ç°O^Î~¡¶ ^Î#=O`Ç°x =°#O f~ŒàxOKDžè=ò.

E
ª^Î#: ㄬH›@#…Õ =°$^ΰ=ôQê À+"£ K͋¬°Hù<Í XH› „¬^Μu Q®°iOz
Hêx Wzó# ㄬH›@# ㄬHê~¡O =°Oã`Ç°O^ÎiH© ‹¬ÞO`Ç

D
9
Hê~¡°Á =ô<Œß†Ç°x x~ŒœiO„¬|_#k =°i†Çò "Œ~¡° =¶ã`Ç"Í° K³„¬æ|_#k.

59
f~Œà<Œ°-I, II, III…Õ Un ㄬH›@#䛽 ‹¬O|OkOz#k
|‚²ì~¡OQ® ‹¬=¶"͉§xH÷ ‚¬ð[ï~á#@°Á 䛀_¨ x~ŒœiO„¬
CA
95
|_#k. Hê=ô# f~Œà#O-II =°i†Çò f~Œà#O-III° Hê^ΰ.

50
‹¬i†³Ø°#q. Hê=ô# ‹¬=¶^¥#O (5). \ Hê=ô# ‹¬=¶^¥#O (5).
98
A
:8
123456789012345678901234567890121234567890123456789012345678901212
1234567890123456789012345678901212345678901234567890123456789012123456789012345678901234567890121234567890123
1234567890123456789012345678901212345678901234567890123456789012123456789012345678901234567890121234
12345678901234567890123456789012123456789012345678901234567890121234567890123456789012345678901212345678901234567890123456789012123
1234567890123456789012345678901212345678901234567890123456789012123456789012345678901234567890
12345678901234567890123456789012123456789012345678901234567890121234567890123456789012345678901212345678901234567890123
1234567890123456789012345678901212345678901234567890123456789012123456789012345678901234567890121234567890123
1234567890123456789012345678901212345678901234567890123456789012123456789012345678901234567890121234
123456789012345678901234567890121234567890123456789012345678901212
12345678901234567890123456789012123456789012345678901234567890121234567890123456789012345678901212345678901234567890123456789012123
1234567890123456789012345678901212345678901234567890123456789012123456789012345678901234567890
12345678901234567890123456789012123456789012345678901234567890121234567890123456789012345678901212345678901234567890123
1234567890123456789012345678901212345678901234567890123456789012123456789012345678901234567890121234567890123
1234567890123456789012345678901212345678901234567890123456789012123456789012345678901234567890121234
12345678901234567890123456789012123456789012345678901234567890121234567890123456789012345678901212345678901234567890123456789012123
1234567890123456789012345678901212345678901234567890123456789012123456789012345678901234567890
123456789012345678901234567890121234567890123456789012345678901212
E

PRACTICE TEST - 1
12345678901234567890123456789012123456789012345678901234567890121234567890123456789012345678901212345678901234567890123
1234567890123456789012345678901212345678901234567890123456789012123456789012345678901234567890121234567890123
1234567890123456789012345678901212345678901234567890123456789012123456789012345678901234567890121234
12345678901234567890123456789012123456789012345678901234567890121234567890123456789012345678901212345678901234567890123456789012123
1234567890123456789012345678901212345678901234567890123456789012123456789012345678901234567890
123456789012345678901234567890121234567890123456789012345678901212
12345678901234567890123456789012123456789012345678901234567890121234567890123456789012345678901212345678901234567890123
1234567890123456789012345678901212345678901234567890123456789012123456789012345678901234567890121234567890123
1234567890123456789012345678901212345678901234567890123456789012123456789012345678901234567890121234
h

12345678901234567890123456789012123456789012345678901234567890121234567890123456789012345678901212345678901234567890123456789012123
1234567890123456789012345678901212345678901234567890123456789012123456789012345678901234567890
12345678901234567890123456789012123456789012345678901234567890121234567890123456789012345678901212345678901234567890123
1234567890123456789012345678901212345678901234567890123456789012123456789012345678901234567890121234567890123
1234567890123456789012345678901212345678901234567890123456789012123456789012345678901234567890121234
12345678901234567890123456789012123456789012345678901234567890121234567890123456789012345678901212345678901234567890123456789012123
1234567890123456789012345678901212345678901234567890123456789012123456789012345678901234567890
123456789012345678901234567890121234567890123456789012345678901212
12345678901234567890123456789012123456789012345678901234567890121234567890123456789012345678901212345678901234567890123456789012123
1234567890123456789012345678901212345678901234567890123456789012123456789012345678901234567890
12345678901234567890123456789012123456789012345678901234567890121234567890123456789012345678901212345678901234567890123
1234567890123456789012345678901212345678901234567890123456789012123456789012345678901234567890121234567890123
1234567890123456789012345678901212345678901234567890123456789012123456789012345678901234567890121234
.P
IC

G
AD

Wzó# ㄬH›@# ãH÷Ok ï~O_È° f~Œà<Œ° I =°i†Çò II =°^Î¼^ζ~¡O ‹¬=òã^Î =¶~¡¾O ^¥Þ~Œ 200
Wzó#„¬C_È° "³ò^Î\ ÷ f~Œà#O =¶ã`Ç"Í° ‹¬i†³Ø°#^³á`Í H÷.g°. `Çy¾#k. nx=# ‹¬O=`Ǟ~ŒxH÷
AB
O

‹¬=¶^¥#O (1)Qê#¶, ï~O_È= f~Œà#O =¶ã`Ç"Í° WO^Î# Y~¡°ó 8 HË@Á ~¡¶‡†Ç°° `Çy¾#k.
ER
CH

‹¬i†³Ø°#^³á`Í ‹¬=¶^¥#O (2)Qê#¶, "³ò^Î\ ÷ =°i†Çò f~Œà<Œ° : I) ~Ë_È°Û =¶~¡¾O ^¥Þ~Œ J†Í°¼ ㄬ†Ç¶}Y~¡°ó
YD

ï~O_È° f~Œà<Œ° ï~O_ȶ ‹¬i†³Ø°#"³á`Í ‹¬=¶^¥#O H› < Œß ‹¬ = òã^Î = ¶~¡ ¾ O ^¥Þ~Œ J†Í ° ¼
,H

(3)Qê#¶, "³ò^Î\ ÷ =°i†Çò ï~O_È= f~Œà<Œ° ï~O_ȶ ㄬ†Ç¶} Y~¡°ó KŒ…ì KñH›.
T
AM

‹¬i†³Ø°#q HêH›‡é`Í ‹¬=¶^¥#O (4)Qê#¶, "³ò^Î\ ÷ II) WO^# Î qx†³¶Q®O JkH› "³ò`ÇOë …Õ ‡Ú^ΰ„¬ô
H

…è^¥ ï~O_È= f~Œà<Œ…Õ U^Ë XH›\ ÷ ‹¬ï~á#^Î~ò`Í K͆ǰ|_È°`Ç°Ok.


M
AM

‹¬=¶^¥#O (5)Qê#¶ Q®°iëOKŒe. 4. ㄬH›@# : ƒì~¡`^Ç ‰Í × ã„¬ƒ°’ `ÇÞ Hê~Œ¼†Ç¶…Õ i[ö~Þ+¬<£


IG

1. ㄬH›@# : ‡éb‹¹ JkHê~¡° #°O_ ㄬ[° =òY¼OQê q^ ¥ #O ‹¬ = °~¡ Ö = O`Ç " ³ ° Ø # L^˼Q® ° #°
KH

PtOKÍk x*ì~òf. =¶ P„¦Ô‹¬~Ÿž JO^ÎiH© JOkOKÇ_ÈO …è^ΰ.


R

D q+¬†°Ç O `³°‹¬°-J=°~Œ=uD.I.G. ㄬH@ › #. f~Œà<Œ° : I) i[ö~Þ+¬<£ q^¥#O ^¥Þ~Œ L^˼Qê#°


f~Œà<Œ° : I) J=°~Œ=u D.I.G. ㄬH@ › # „¬îiëQê ‹¬`¼Ç O. ‹¬O‡kOz# "Œ~¡O^Î~¶¡ ‹¬=°~¡` Ö Ç …èx"Œö~.
II) J=°~Œ=u D.I.G. ㄬH@› # „¬îiëQê J‹¬`¼Ç O. II) i[ö~Þ+¬<£ q^¥#O =# `Ç䛽ø= ㄬuƒ’

2. ㄬH›@# : „¬ô‹¬ëHꁰ À‹ß‚²ì`Ç°=O\÷q. ï~O_ȶ ‰§‰×Þ`Ç =ô#ß"Œ~¡° 䛀_¨ L^˼Qêä›½ ZO„²H›


ㄬ†³¶[<Œ#° JOkªë~ò. K͆ǰ |_È°`Ç°<Œß~¡°.
f~Œà<Œ° : I) À‹ß‚²ì`Ç°#° KǶ‹² Z#°ßHË"Œe. 5. ㄬH›@# : J^Î$+¬“O ^³á~¡¼=O`Ç°_ "³#ßO>è =ôO@°Ok.
II) „¬ô‹¬H
ë ê=# =°#O ƒìs ㄬ†¶³ [<Œ#° f~Œà<Œ° : I) q[†Ç°ª^Î#…Õ H›‘“° Z^ΰ~ùø#@O
‡ÚO^ΰ`Ç°<ŒßO. `Ç„æ¬ x‹¬i. (Bank of Baroda P.O. - 2008)
3. ㄬH›@# : K³<ßá³ =°i†Çò =òOƒì~ò #Q®~Œ =°^¼Î ~Ë_È°Û II) „²iH÷"Œˆ×ä "Œi =°~¡}ìxH÷ =òO^Í J<ÍH›

=¶~¡¾ ^ζ~¡O 800 H÷.g°. Hêx D #Q®~Œ ª~¡°Á =°~¡}÷ªë~¡°.

REASONING ABILITY (LOGICAL) ○ ○ ○ ○ ○ ○ ○ ○


147
○ ○ ○ ○ ○ ○ ○ ○ ○ ○ ○ ○ ○ ○ ○ ○ ○ ○ ○
Study Material
6. ㄬH›@# : †³òH›ø „¬xf~¡° Q®°iOz =¶‹¬“~Ÿ
Insat 2-D 11. ㄬH›@# : Z_¨~¡°…Õ XO>ÿ ^¥Þ~Œ Hê䛽O_¨ "Í~
ö q^OÎ Qê
H›Oã\Յ˜ ªÏH›~¼¡ O ^¥Þ~Œ ‰§G"Í` ëÇ ° =°~ùH›ªi XH›KË\÷ #°O_ =°~ùH›KË\÷H÷ "³ˆ¤× _ÈO Jª^¼Î O.
„¬î~ŒÞƒì¼‹¬O K͉§~¡°. f~Œà<Œ° : I) Z_¨~¡°…Õ XO>ÿ ^¥Þ~Œ ~¡"Œ}ì ‹¬°ƒ’
f~Œà<Œ° : I) H›$ãu=° L„¬ãQ®‚ð¬ xß JO`ÇiH›O Æ …ÕxH÷ „¬OÀ„ `Ç~¡"³°Ø# „¬^Μu.
=òO^ΰ i‚¬ð~¡ž…˜ž `DŽ¬æx‹¬i. II) Z_¨~¡°…Õ XO>ÿ° ‹¬=°$kœQê aªë~ò.
II) ‰§G"Í`Ç끰 JO`Ç=òO^ΰ ㄬ†³¶yOz# 12. ㄬH›@# : XH› K³_ÈÛ =¼H÷ë =°`Nj¬O|O^Î q+¬†Ç¶#° `Ç#
H›$ãu=° L„¬ãQ®‚¬ð#° JO`ÇiH›ÆO…ÕxH÷ ‹¬Þㄬ†¶³ [<Œ Hˋ¬O L„¬†¶³ yOKǰ䛽O\ì~¡°.
„¬OÀ„=°O^ΰ 䛀_¨ W…ì<Í i‚¬ð~¡ž…˜ž f~Œà<Œ° : I) K³_ÈÛ =°#‹¬ë`ÇÞO H›"Œ~¡° J‚¬ìOƒì=O`Ë
Kͪ~¡°. „¬xKͪë~¡°.
7. ㄬH›@# : ''=¶ ㄬƒ’°`ÇÞO, ï~á`Ç°° =°i†Çò ãQêg°}
II) x["³ ° Ø # *ìý # O =°`Ç ƒÕ^ Î # =<Í
㇐O`Œ…Õ Q® À„^Î"Œi †³òH›ø Ja=$kœH÷
J=_È°`Ç°Ok.
„¬îië ªÖ~ò…Õ H›$+² K͋°¬ Oë kÑÑ - ㄬ^¥ #=°Oãu

Y
13. ㄬH›@# : XH› =°x+² =°Oz ‡é~Œ@ †³¶^°Î _È° J"ŒÞO>è
#ö~Oã^Î "³¶k.
J`Ç#° q"ÍH= › O`Ç°_È° J†Çò¼O_¨e. ‡é~Œ@

M
f~Œà<Œ° : I) Q®`Ç ã„¬ƒ’°`ÇÞO ï~á`Ç°° =°i†Çò ãQêg°}
㇐O`Œ…Õ Q® À„^Î"Œi Ja=$kœH÷ †³¶^°Î O^Î~¶¡ =¶@Hê~¡°° =°i†Çò q‹¬°Q®°

E
„¬îi몐Ö~ò H›$+² K͆ǰ…è^ΰ. „¬ô\÷ª
“ ë~°¡ .
: I) q"ÍH›=O`Ç°O^Î~¡¶ q‹¬°Q®°„¬ô\÷“ªë~¡°.

D
f~Œà<Œ°

9
II) D ㄬƒ’°`ÇÞO „¬@“} ㇐O`Œ…Õ Q®

59
II) q"ÍH›=O`Ç°O^Î~¡¶ ‡é~Œ@†³¶^Î°…è.
À„^Î"Œi Ja=$kœ Hˋ¬O H›$+² K͆ǰ^ΰ.
CA
95
8. ㄬH›@# : ‰×`Œaœ ZH±žã̄‹¹…Õ ㄬ†¶ Ç }÷OKÇ°@䛽 i[ö~Þ+¬<£ 14. ㄬH›@# : XH› Hê#æï~<£žH÷ ㄬ„¬OKÇO…Õx "Í°\÷ ãH÷ïH\˜

50
KÍ~òOKǰ䛽x "Íz†ÇòO_Í *ìa`Œ…Õ Q® ƒì¼\˜ž"³°<£#° =¶ã`Ç"Í° P‚¬ðÞxOKŒ~¡°.
98
A
Zxq°k =°OkH÷ Z@“öHä›½ ‹Ô@° i[ö~Þ+¬<£ f~Œà<Œ° : I) Hê#æ~ ï <£žH÷ ‚¬ð[ï~#á ãH÷Hï \˜ P@Qêˆ×¤O^Î~¶¡
:8
öH\ì~òOKÇ|_#q. "Œi ^͉×O `Ç~¡„¬ô# P_# P@…ÕÁ =°Oz
E

<³á„¬ô}¼`Ç#° ㄬ^ÎiÅOKŒ~¡°.
h

(Syndicate Bank P.O. - 2008)


.P
IC

f~Œà<Œ° : ª^¥~¡}OQê ‰×`Œaœ ZH±žã̄‹¹ †³òH›ø


I) II) ‹¬Q®@°# XHùøH›ø =¶¼KŸ…Õ 8 „¬~¡°Q®°°
AD

"³~ò\÷OQ· e‹¹“…Õ À„~¡° =ô#ß"ŒiH÷ ‹Ô@° K͋#² ƒì¼\˜ž"³°<£ P Hê#æ~ï <£žH÷ ‚¬ð[ï~<á Œ~¡°.
AB
O

^ù~¡H›_ÈO KŒ…ì H›+¬“O. 15. ㄬH›@# : Q®`Ç 5 ‹¬O=`Ǟ~Œ°Qê ㄬu 䛽@°O|O †³òH›ø
II) ‰×`Œaœ ZH±žã̄‹¹ KŒ…ì `Ç䛽ø= ƒÿ~Ÿë#° q^ΰ¼`Ÿ qx†³¶Q®O ï~\÷“Ōá#k.
ER
CH

H›ey =ôO@°Ok. f~Œà<Œ° : I) ‹¬=¶[O ƒìQê Ja=$kœ K³Ok#k.


YD

9. ㄬH›@# : ïH#~Œ ƒì¼O䛽 †³òH›ø Jxß ‰§Y…Õ H›e„² II) q^ΰ¼`Ÿ KŒsû° ƒìQê KñH›Qê =ô#ßq.
,H

Q®i+¬»OQê 400 ‡é‹¹“° Mìm° =ô<Œß~ò. 16. ㄬH›@# : ªÏO^Î~¼¡ ª^< Î Œ#° L„¬†¶³ yOKÇ_OÈ KÇ~ŒàxH÷
T
AM

‹¬ï~á# Jƒ’¼~¡°Ö° Hê=…ÿ#° - ïH#~Œƒì¼OH± ‚¬ðxH›~¡O.


H

ㄬH›@#. : I) ªÏO^Î ~ ¡ ¼ ª^ Î < Œ "Œ_È H › O =#


M

f~Œà<Œ°
f~Œà<Œ° : I) ㄬ ‹ ¬ ° ë ` Ç O ‹² | ÄOkx x†Ç ° q°OKÇ ° ä› ½ <Í
AM

^ΰ¢+¬æƒì"Œ° H›°Q®°`Œ~ò.
IG

ㄬãH÷†Ç°…Õ ïH#~Œƒì¼O䛽 400 H›O>è II) ªÏO^Î~¡¼ª^Î<Œ° KŒ…ì Ys^³á#q.


KH

`Ç䛽ø= =°Ok ‹²|ÄOkx f‹¬°HË=KÇ°ó#°. 17. ㄬH›@# : W„¬C_È° VCR Hù#°Qˁ° K͆°Ç _¨xH÷ kQ®°=°u
R

II) ㄬ ‹ ¬ ° ë ` Ç O ‹² | ÄOkx x†Ç ° q°OKÇ ° ä› ½ <Í


…ÿá̋<£ž ‡ÚO^¥ež# J=‹¬~¡O …è^ΰ.
ㄬãH÷†Ç°…Õ ïH#~Œƒì¼O䛽 400 =°Ok
f~Œà<Œ° : I) W„¬C_È° VCR ° ‹¬Þ^Íj „¬i*ìý#O`Ë<Í
‹²|ÄOkx f‹¬°HË=KÇ°ó#°.
`džǶ~¡°K͋¬°ë<Œß~¡°.
10. ㄬH›@# : ~ŒƒÕ†Í° HêO…Õ Ja=$kœ K³O^ΰ`Ç°#ß KŒ…ì
II) W„¬ C _È ° VCR° À‹ÞKÇ ó  Q ê kQ® ° =°u
^͉§…Õ [<Œƒì q„¬s`ÇOQê ̄iy =#~¡°°
„¬i=¶}O ƒìsQê `ÇöQ¾ J=Hê‰×O =ôOk. K͋¬°ä›½<Í ªÏH›~¡¼O H›eæOKŒ~¡°.
f~Œà<Œ° : I) ƒq ’ +¬¼`Ç°… ë Õ Ja=$kœ K³O^ΰ`Ç°#ß ^͉§…Õ 18. ㄬH›@# : D ㄬ„O ¬ KÇO K³_n ÛÈ Hê^ΰ, =°OznHê^ΰ, ㄬu
[<Œƒì ̄~¡°Q®°^΁ =ôO_È^ΰ. =°x+² `Ç# ‹¬ÞO`Ç ã„¬„¬OKŒxß `Œ<Í ‹¬$+²“OKÇ°
II) Ja=$kœ K³O^ΰ`Ç°#ß ^͉§…Õ ㄬƒ°’ `ŒÞä›½
䛽<Œß_È°.
"Œi ^Í‰× ã„¬[ r=#O ª„¦QÔ ê =ôO_Í@°Á f~Œà<Œ° : I) HùO`Ç = °Ok D ㄬ „ ¬ O KŒxß KŒ…ì
K͆°Ç _ÈO H›+ª “¬ ^¼Î O J=ô`Ç°Ok. =°OzkQê H›#°Qù<Œß~¡°.

REASONING ABILITY (LOGICAL) ○ ○ ○ ○ ○ ○ ○ ○


148
○ ○ ○ ○ ○ ○ ○ ○ ○ ○ ○ ○ ○ ○ ○ ○ ○ ○ ○
Study Material
HùO`Ç=°Ok D ㄬ„O¬ KŒxß KŒ…ì K³_k
II) ÛÈ Qê Q®`ÇO…Õ =°#^͉×O…Õ PiÖH› L`Œæ^ÎH›`Ç
II)

H›#°Qù<Œß~¡°. JO‰×O x~¡ÁH›Æ ¼O =‚²ìOKÇ|_#k.


19. ㄬH›@# : ƒì~¡`Ç^͉×O…Õ ㇐^ŠÎq°H› =°i†Çò ‹¬ÞKÇó"³°Ø# 20. ㄬH›@# : ''<Í#° #\÷Oz#O^ΰ䛽, <Œã‰×=°ä›½ `ÇQ®¾ ㄬu
q*ìý<Œx H›<Œß =¼=ª†Ç°O, WO[hiOQ· „¦¬…ìxß „¬îiëQê f‹¬°ä›½O\ì#° …è^¥ J‹¬ž°
=°i†Çò >ÿHê߁rä›½ JkH› ㇐^¥#¼`Ç Ug° f‹¬°HË#°ÑÑ - XH› À‹“*˜ ‘é…Õ ㄬ‹O¬ y‹¬¶ë
=ôOk. #@°_È° ~¡[hHêO`Ÿ K͋²# "Œ¼Mì¼#O.
f~Œà<Œ° : I) ƒì~¡`Ç^͉×O W„¬æ\÷öH ㇐^ŠÎq°H› =°i†Çò f~Œà<Œ° : I) ~¡[hHêO`Ÿ P À‹“*˜‘é K͋²#O^ΰ䛽 „¬îië
‹¬ÞKÇó"
 °³ #
Ø q*ìý#~¡OQ®O…Õ ZO`Ë „¬ô~ËQ®u "Í`Ç<Œxß ‡ÚO^¥_È°.
II) ~¡ [ hHêO`Ÿ P À‹“ * ˜ ‘ é KÍ ‹ ² # O^Î ° ä› ½
ªkOz#k.
Z…ìO\÷ "Í`Ç#O ‡ÚO^΅è^ΰ.
1234567890123456789012
12345678901234567890123456789012123456789012345678901234567890121234567890123456789012345678901212345678901234567890123
12345678901234567890123
12345678901234567890123456789012123456789012345678901234567890121234567890123456789012345678901212345678901234567890123
1234567890123456789012 12345678901234567890123
12345678901234567890123456789012123456789012345678901234567890121234567890123456789012345678901212345678901234567890123
1234567890123456789012
q=~¡}ì`ÇàH› ["Œ|°°
12345678901234567890123

Y
1234567890123456789012
12345678901234567890123456789012123456789012345678901234567890121234567890123456789012345678901212345678901234567890123
12345678901234567890123
12345678901234567890123456789012123456789012345678901234567890121234567890123456789012345678901212345678901234567890123
1234567890123456789012
12345678901234567890123456789012123456789012345678901234567890121234567890123456789012345678901212345678901234567890123
1234567890123456789012PRACTICE TEST - 1 : 12345678901234567890123
12345678901234567890123
12345678901234567890123456789012123456789012345678901234567890121234567890123456789012345678901212345678901234567890123
1234567890123456789012 12345678901234567890123

M
1. (1) f~Œà#O-I ㄬH›@##° ‹¬=°i֋¬°ëOk. ‡é=KÇ°ó. Hê=ô# ï~O_È° f~Œà<Œ…Õ U^Ë XH›\ ÷

E
2. (2) À‹ß‚²ì`Ç° =# Z…ìO\÷ ㄬ†³¶[<Œ°O\솳¶ ‹¬i†³°Ø #k J=ô`Ç°Ok.

D
„¬ô‹¬Hë ê =# 䛀_¨ J…ìO\÷ ㄬ†¶³ [<Œ°O\ì†Ç°<Í^Í 10. (2) f~Œà#O- I ‹¬i†³Ø°#k Hê^ΰ. ƒ’q+¬¼`Ç°ë…Õ [iöQ

9
59
ㄬH@› # J~¡OÖ . Hê=ô# f~Œà#O-II =¶ã`Ç"°Í ‹¬i†³°Ø #k. „¬i}ì=¶#° =°#O =òO^ΰQê<Í f~ŒàxOKDžè=ò.
CA
95
3. (2) WO^Š Î # O Y~¡ ° ó `Ç y ¾ # @°Á =¶ã`Ç " Í ° ㄬ H › @ # f~Œà#O-II ㄬH›@# ㄬHê~¡O ‹¬i†³Ø°#k. =#~¡°

50
`³e†Ç°*苰¬ Oë k. Hê=ô# f~Œà#O-II ‹¬i†³°Ø #k. ㄬ†¶Ç } „¬i=¶}O ƒìsQê `Çy¾#@Á~ò`Í ã„¬[O^Îi rq`Œ°
98
A
Y~¡°ó ㄬªë=# ㄬH›@#…Õ …è^ΰ. Hê=ô# f~Œà#O-II ª„¦ÔQê =ôO_Í@°Á ㄬƒ’°`ÇÞO K͆ǰ_ÈO KŒ…ì H›+¬“O.
:8
=¶ã`Ç"Í° ‹¬i†³Ø°#k. 11. (1) Z_¨~¡°…Õ Hêe#_ÈH›# ㄬ†Ç¶}÷OKDžè=ò. "³¶\ì~¡°
E
h

4. (3) Wzó# ㄬH@ › # ㄬHê~¡O f~Œà#O-I =°i†Çò f~Œà#O-


.P

"Œ‚¬ì<Œ…Õ ㄬ†Ç¶}÷OKDžèO. XO>ÿ̄á ㄬ†Ç¶}÷OKÇ_ÈO


IC

II ï~O_ȶ ‹¬i†³Ø°#"Í.
AD

XH›ø>è ‹¬°°"³á# „¬^Μu. Hê=ô# f~Œà#O-I ㄬH›@#


5. (1) H›‘“° =zó#„¬ô_È° ^³~á ¼¡ OQê "Œ\÷x Z^ΰ~Ëø=@O =°#O ㄬ H ê~¡ O ‹¬ i †³ Ø ° #k. f~Œà#O- II `Ë ã„¬ H › @ #ä› ½
AB
O

㇐~¡OaÀ‹ë J^Î$+¬O“ 䛀_¨ =°#䛽 H›e‹² =‹¬°Oë k. Hê=ô# ‹¬O|O^ÎO …è^ΰ.


ER
CH

f~Œà#O- I ‹¬ i †³ Ø ° #k. f~Œà#O- II ㄬ H › @ #ä› ½


12. (1) f~Œà#O-I ‹¬i†³Ø°#k.
YD

‹¬O|O^ÎO…èx q+¬†Ç°O.
K³ _ È Û " Œ~¡ ° "Œi ‹¬ Þ ã„¬ † ³ ¶ [<Œ Hˋ¬ O Z…ìO\÷
6. (3) f~Œà#O-I =°i†Çò f~Œà#O- II ï~O_ȶ ㄬH›@#
,H

hKŒxïHá<Œ kQ®*ì~¡°`Œ~¡°. f~Œà#O-II ㄬH›@#`Ë


T

ㄬHê~¡O ‹¬i†³Ø°#"Í.
AM

‹¬O|O^ÎO …èxk.
H

U L„¬ãQ®‚¬ð<³áß<Œ JO`ÇiH›ÆO…ÕxH÷ „¬OÀ„ =òO^ΰ ^¥x


M

13. (4) J~ò`Í, J†ÇòO_¨e.... J<Íq Yzó`Ç`ŒÞxß W=ޅè=ô.


„¬xf~¡°#° `DŽ¬æH› =òO^ΰQê<Í „¬sH÷ƪë~¡°.
AM
IG

7. (1) f~Œà#O-I ㄬH›@# ㄬHê~¡O ‹¬i†³Ø°#k. ãQêg°} ‡é~Œ@ †³¶^ŠÎ°ä›½ q"ÍH›O =ôO_¨x K³„¬æ|_#k.
KH

㇐O`Œ…Õ Q® À„^Î"Œi Ja=$kœH÷ H›$+² Kͪë=ò Hêx q"ÍH›=O`Ç°O^Î~¡¶ ‡é~Œ@ †³¶^ŠÎ°x K³„¬æ|_È
R

JO>è „¬@“} ㇐O`ÇO "Œi Ja=$kœH÷ H›$+² K͆ǰO …è^ΰ. Hê=ô# f~Œà#O-II ‹¬iHê^ΰ. f~Œà#O-I <Í~¡°Qê
Jx Hê^ΰ. ㄬH›@# ㄬHê~¡O `DŽ¬C.
8. (1) Jxß ï~áˆ×¤…ìQê<Í ‰×`Œaœ ZH±žã̄‹¹…Õ 䛀_¨ ‹¬i„¬_¨ 14. (1) "Í°\÷ ãH÷ïH\˜ P@Qêˆ×¤…Õ ‹¬Q®@° '8Ñ Q®"Œ~¡° =ôO_È~¡°.
ƒÿ~Ÿë ‹¬OY¼ =ôO\ì~ò. Hê=ô# f~Œà#O-II ‹¬i†³Ø°#k ‹¬Q®@° '8Ñ H›ey# ƒì¼\˜ž"³°<£#° q„¦¬"³°Ø# P@Qêˆ×ä
Hê^ΰ. ㄬH›@#…Õ ‹Ô@° ^ù~¡H›_ÈO Qù„¬æ q+¬†Ç°"Í° JO\ì~¡°. Hê=ô# f~Œà#O-II ‹¬iHê^ΰ. f~Œà#O-I
J#ß@°ÁQê W=Þ|_#k. Hê=ô# f~Œà#O-I ‹¬i†³Ø°#k. ㄬH›@# ㄬHê~¡O ‹¬i†³Ø°#k.
9. (5) ïH#~Œƒì¼OH± "ŒiH÷ Q®i+¬»OQê 400 =°Ok Jƒ’¼~¡°Ö° 15. (4) f~Œà#O-I =°i†Çò f~Œà#O-II ï~O_ȶ ㄬH›@#`Ë
Hê=…ÿ#°. WO@~¡¶¼ä›½ =zó#"Œ~¡O^Î~¶¡ x†Ç°q°O„¬|_È ‹¬O|O^ÎO …èx"Í. ㄬH›@#…Õ q^ΰ¼`Ÿ qx†³¶Q®O
=KÇ°ó …è^¥ x†Ç°q°O„¬|_ÈH› ‡é=KÇ°ó. 400 ‡é‹¹“ä›½ Q®°iOz =¶ã`Ç"Í° K³„¬æ|_#k. Hêx f~Œà<Œ°Qê
Jƒ’¼~¡°Ö° ‹¬i†³Ø°#"Œ~¡° ^ù~¡H›=KÇ°ó …è^¥ ^ù~¡H›H› W=Þ|_# ï~O_ȶ T‚¬ì#…è. Hê=ô# ‹¬=¶^¥#O (d).

REASONING ABILITY (LOGICAL) ○ ○ ○ ○ ○ ○ ○ ○


149
○ ○ ○ ○ ○ ○ ○ ○ ○ ○ ○ ○ ○ ○ ○ ○ ○ ○ ○
Study Material
16. (1) ㄬH›@# ㄬHê~¡O f~Œà#O-I ‹¬i†³Ø°#k. ^Î~¡ Q®°iOz ㄬH@
› #…Õ ㄬ„O¬ KÇO †³òH›ø ‹¬Þƒì"Œxß q~¡°^Μ ƒì=#°
ㄬH›@#…Õ W=ޅè^ΰ. Hê=ô# f~Œà#O-II ‹¬i†³Ø°#k H›ey# ㄬ[ ƒì"Œ Q®°iOz K³„¬æ|_#k. Hê=ô#
Hê^ΰ. f~Œà#O-I =°i†Çò f~Œà#O-II ï~O_ȶ ‹¬i†³Ø°#"Í.
17. (2) '‹¬Þ^Íj „¬i*ìý#O`Ë `džǶ~¡°K͆ǰ_ÈO =#Ñ J#°#k 19. (2) ㄬH›@#…Õ „¬ô~ËQ®u Q®°iOz K³„¬æ|_ȅè^ΰ. Hê=ô#
T‚¬ì#"Í°. Hê|\÷“ f~Œà#O-I ‹¬iHê^ΰ. kQ®°=°u …ÿÌá ‹<£ž f~Œà#O-I ‹¬i†³Ø°#k Hê^ΰ. f~Œà#O-II ㄬH›@#
J=‹¬~¡O …è^ΰ. Jx =ô#ßk Hê|\÷“ f~Œà#O-II ㄬHê~¡O ‹¬i†³Ø°#k.
‹¬i†³Ø°#k. 20. (5) U^³á<Œ [~¡Q®=KÇ°ó J<Ík ㄬH›@#…Õ =ô#ßk. Hê=ô#
18. (3) ï~O_È° f~Œà<Œ¶ ㄬH›@# ㄬHê~¡O ‹¬i†³Ø°#q. f~Œà#O- I …è ^ ¥ f~Œà#O- II …Õ U^Ë XH› \ ÷
‹¬i†³Ø°#k.
123456789012345678901234567890121234567890123456789012345678901212
1234567890123456789012345678901212345678901234567890123456789012123456789012345678901234567890121234567890123
1234567890123456789012345678901212345678901234567890123456789012123456789012345678901234567890121234
12345678901234567890123456789012123456789012345678901234567890121234567890123456789012345678901212345678901234567890123456789012123
1234567890123456789012345678901212345678901234567890123456789012123456789012345678901234567890
12345678901234567890123456789012123456789012345678901234567890121234567890123456789012345678901212345678901234567890123
1234567890123456789012345678901212345678901234567890123456789012123456789012345678901234567890121234567890123
1234567890123456789012345678901212345678901234567890123456789012123456789012345678901234567890121234
123456789012345678901234567890121234567890123456789012345678901212
12345678901234567890123456789012123456789012345678901234567890121234567890123456789012345678901212345678901234567890123456789012123
1234567890123456789012345678901212345678901234567890123456789012123456789012345678901234567890
12345678901234567890123456789012123456789012345678901234567890121234567890123456789012345678901212345678901234567890123
1234567890123456789012345678901212345678901234567890123456789012123456789012345678901234567890121234567890123
1234567890123456789012345678901212345678901234567890123456789012123456789012345678901234567890121234
123456789012345678901234567890121234567890123456789012345678901212

Y
12345678901234567890123456789012123456789012345678901234567890121234567890123456789012345678901212345678901234567890123456789012123
1234567890123456789012345678901212345678901234567890123456789012123456789012345678901234567890
PRACTICE TEST - 2
12345678901234567890123456789012123456789012345678901234567890121234567890123456789012345678901212345678901234567890123
1234567890123456789012345678901212345678901234567890123456789012123456789012345678901234567890121234567890123
1234567890123456789012345678901212345678901234567890123456789012123456789012345678901234567890121234
12345678901234567890123456789012123456789012345678901234567890121234567890123456789012345678901212345678901234567890123456789012123
1234567890123456789012345678901212345678901234567890123456789012123456789012345678901234567890
123456789012345678901234567890121234567890123456789012345678901212
12345678901234567890123456789012123456789012345678901234567890121234567890123456789012345678901212345678901234567890123
1234567890123456789012345678901212345678901234567890123456789012123456789012345678901234567890121234567890123
1234567890123456789012345678901212345678901234567890123456789012123456789012345678901234567890121234
12345678901234567890123456789012123456789012345678901234567890121234567890123456789012345678901212345678901234567890123456789012123
1234567890123456789012345678901212345678901234567890123456789012123456789012345678901234567890
12345678901234567890123456789012123456789012345678901234567890121234567890123456789012345678901212345678901234567890123
1234567890123456789012345678901212345678901234567890123456789012123456789012345678901234567890121234567890123
1234567890123456789012345678901212345678901234567890123456789012123456789012345678901234567890121234
123456789012345678901234567890121234567890123456789012345678901212
12345678901234567890123456789012123456789012345678901234567890121234567890123456789012345678901212345678901234567890123456789012123
1234567890123456789012345678901212345678901234567890123456789012123456789012345678901234567890
12345678901234567890123456789012123456789012345678901234567890121234567890123456789012345678901212345678901234567890123
1234567890123456789012345678901212345678901234567890123456789012123456789012345678901234567890121234567890123
1234567890123456789012345678901212345678901234567890123456789012123456789012345678901234567890121234
12345678901234567890123456789012123456789012345678901234567890121234567890123456789012345678901212345678901234567890123456789012123
1234567890123456789012345678901212345678901234567890123456789012123456789012345678901234567890

M
‹¬¶KÇ#(1-15) : XH› ㄬH›@# ãH÷O^Î 3 …è^¥ 4 f~Œà<Œ° 3. ㄬH›@# : 1. "³¶‚¬ì<£ 䛽@°O| ‹¬ƒ°’ ¼O^Î~¶
¡ x*ì~òf

E
Wzó#„¬ô_È°, ãH÷O^Î W=Þ|_# P„¬Â<£…Õ Uk ‹¬i†³°Ø #^Ë „¬~¡°…è.
2. "³ ¶ ‚¬ ì <£ ä› ½ @°O|O…Õx HùO^Î ~ ¡ °

D
9
ZOKÇ°Hùx ‹¬=¶^¥#O Q®°iëOKŒe.

59
1. ㄬH›@# : 1. HùO`Ç=°Ok Q®}`
÷ ‰Ç §GO…Õ ‹¬ƒäûÿ ½› “ „¬~O¡ Qê x~¡°^˼Q®°°.
CA
95
|‚Ôì#°°. 3. HùO`Ç=°Ok L^˼Q®‹°¬  ë ° x*ì~òf„¬~°¡ °
Hê^ΰ.

50
2. Q®}÷`lj§GO…Õ |‚Ôì#°O^Î~¡¶ ‹¬Ow`Ç
4. HùO`Ç=°Ok x*ì~òf„¬~°¡ ° x~¡°^˼Q®°°.
98
A
q^¥ÞO‹¬°°.
f~Œà<Œ° : I) "³ ¶ ‚¬ ì <£ ä› ½ @°O| ‹¬ ƒ  ’ ° ¼O^Î ~ ¡ ¶
:8
f~Œà<Œ° : I) HùO`Ç=°Ok ‹¬Ow`Ç q^¥ÞO‹¬°° Q®}÷`Ç
E

‰§GO…Õ |‚Ôì#°°. L^˼Q®‹¬°ë…è.


h
.P

II) "³¶‚¬ì<£ 䛽@°O|O…Õx L^˼Q®‹°¬ 


ë O^Î~¶¡
IC

II) ‹¬Ow`Ç q^¥ÞO‹¬°O^Î~¡¶ Q®}÷`lj§GO…Õ


AD

|‚Ôì#°…è. x*ì~òf„¬~¡°…è.
III) "³ ¶ ‚¬ ì <£ ä› ½ @°O|O…Õx x*ì~òf
AB
O

III) Q®}÷`lj§GO…Õ JO^Î~¡¶ |‚Ôì#°…è.


„¬~¡°O^Î~¡¶ x~¡°^˼Q®°…è.
1) I =°i†Çò II =¶ã`Ç"Í° ‹¬i†³Ø°#q.
ER
CH

IV) "³¶‚¬ì<£ 䛽@°O|O…Õx L^˼Q®‹°¬ ë O^Î~¶¡


2) Un ‹¬i†³Ø°#k Hê^ΰ.
YD

x*ì~òf …èx"Œö~.
3) II =°i†Çò III =¶ã`Ç"Í° ‹¬i†³Ø°#q.
1) I,II, III =¶ã`Ç"Í° ‹¬i†³Ø°#qHê=ô.
,H

4) Jhß ‹¬i†³Ø°#"Í.
T

2) I =¶ã`Ç"Í° ‹¬i†³Ø°#k. 3) II =¶ã`Ç"Í° ‹¬i†³Ø°#k


AM

5) I =¶ã`Ç"Í° ‹¬i†³Ø°#k.
4) III =¶ã`Ç"Í° ‹¬i†³Ø°#k 5) Jx߆Çü ‹¬i†³Ø°#"Í
H
M

2. ㄬH›@# : J"³°iHê…Õ „¬xK͆ÇòKÇ°#ß ‰§G"Í`ÇëO^Î~¡¶ 4. ㄬH›@# : ^Î#=O`Ç°° `DŽ¬æ =°ï~=~¡¶ q=¶#O…Õ


AM
IG

ㄬuƒ’ H›"Œö~. HùO`Ç=°Ok ƒì~¡`^Ç ‰Í × ‰§G"Í`ëÇ ° ㄬ†Ç¶}÷OKDžè~¡°. q=¶#O…Õ ㄬ†Ç¶}÷OKÍ


KH

J"³°iHê…Õ „¬xK͆ÇòKÇ°<Œß~¡°. "Œi…Õ HùO^Î~¡° J<Œ~ËQ®¼O ‡†Ç¶¼~¡°.


f~Œà<Œ° : I) ƒì~¡`^ Ç ‰Í × ‰§G"Í`
ëÇ O^Î~¶¡ ㄬuƒ’ …èx"Œö~.
R

f~Œà<Œ° : I) q=¶#O…Õ ㄬ†Ç¶}÷OKÇQ®eöQ "Œ~¡O^Î~¡¶


II) ㄬ u ƒ ’ H› e y# HùO`Ç = °Ok ƒ ì ~¡ f †Ç ° ^Î#=O`Ç°…è.
‰§G"Í`Ç끰 J"³°iHê䛽 =‹¬ "³ˆì¤~¡°. II) ^#Î =O`Ç°O^Î~¶¡ J<Œ~ËQ®¼O ‡†Ç¶¼~¡°.
III) ㄬuƒH ’ e› y# ‰§G"Í` ëÇ O^Î~¶¡ ƒì~¡f†Çò…è. III) ^#
Î =O`Ç°O^Î~¶¡ q=¶#O…Õ ㄬ†¶Ç }÷ªë~°¡ .
IV) HùO`Ç=°Ok ƒì~¡f†Ç° ‰§G"Í`Ç끰 ㄬuƒ’ IV) Z=ï~á`Í q=¶#O…Õ ㄬ†Ç¶}÷ªë~Ë "Œ~¡°
H›"Œ~¡°. J<Œ~ËQ®¼O ‡=ô`Œ~¡°.
1) II =°i†Çò IV =¶ã`Ç"Í° ‹¬i†³Ø°#q. 1) I =°i†Çò III =¶ã`Ç"Í° ‹¬i†³Ø°#q.

2) IV =¶ã`Ç"Í° ‹¬i†³Ø°#k. 2) I =¶ã`Ç"Í° ‹¬i†³Ø°#k.

3) Jhß ‹¬i†³Ø°#"Í. 3) I =°i†Çò IV =¶ã`Ç"Í° ‹¬i†³Ø°#q.

4) Un ‹¬i†³Ø°#k Hê^ΰ. 4) II =°i†Çò III =¶ã`Ç"Í° ‹¬i†³Ø°#q.

5) II, III =°i†Çò IV =¶ã`Ç"Í° ‹¬i†³Ø°#q. 5) Un ‹¬i†³Ø°#k Hê^ΰ.

REASONING ABILITY (LOGICAL) ○ ○ ○ ○ ○ ○ ○ ○


150
○ ○ ○ ○ ○ ○ ○ ○ ○ ○ ○ ○ ○ ○ ○ ○ ○ ○ ○
Study Material
5. ㄬH›@# : <Œ HêÁ‹°¬ …Õx JO^Î~°¡ q^¥¼~¡°
Ö ° `³e"³#á "Œö~. 8. ㄬH›@# : ~Œ[H© † Ç ° <Œ†Ç ° ä› ½ …Õ Zä› ½ ø= =°Ok
=°h+¹ `³e"³á# q^¥¼iÖ Hê^ΰ. J|^¥œ° K³|°`Œ~¡°. Q®°}‰õY~Ÿ „¬zó J|^¥œ
f~Œà<Œ° : I) <Œ HêÁ‹¬°…Õx HùO`Ç =°Ok q^¥¼~¡°Ö° HË~¡° =°i†Çò ~Œ[=°¿o J‹¬° J|^¥œ…è
=¶ã`Ç"Í° `³e"³á#"Œ~¡°. K³„¬æ_È°.
II) =°h+¹ KŒ…ì H›$+² K͆Ƕež =ôOk. f~Œà<Œ° : I) Q®°}‰õY~Ÿ XH› ~Œ[H©†Ç° <Œ†Ç°ä›½_È°.
III) `³eq…èx "Œ~¡° q^¥¼~¡°Ö° Hê~¡°. II) ~Œ[=°¿o Z„¬æ\÷H© ~Œ[H©†Ç° <Œ†Ç°ä›½_È°
IV) =°h+¹ <Œ HêÁ‹¬°…Õx q^¥¼iÖ Hê^ΰ.
Hê…è^ΰ.
1) IV =¶ã`Ç"Í° ‹¬i†³Ø°#k.
III) J|^¥œ…ì_¨x ~Œ[H©†Ç° <Œ†Ç°ä›½° …è~¡°.
2) I =°i†Çò II =¶ã`Ç"Í° ‹¬i†³Ø°#k.
IV) ~Œ[H©†¶ Ç …Õ J|^¥œ° K³„æ¬ _ÈO ‹¬‚ì¬ [O.
3) Jhß ‹¬i†³Ø°#"Í. 4) Un ‹¬i†³°Ø #k Hê^ΰ.
1) Un ‹¬i†³Ø°#k Hê^ΰ.
5) II =°i†Çò IV =¶ã`Ç"Í° ‹¬i†³Ø°#q.
2) Jhß ‹¬i†³Ø°#"Í.

Y
6. ㄬH›@# : <Í~¡O K͋²# ~Œ[H©†Ç° <Œ†Ç°ä›½O^Î~¡¶
3) IV =¶ã`Ç"Í° ‹¬i†³Ø°#k.
Jï~‹¬“†Ç¶¼~¡°. KÇOã^ΉõY~Ÿ =°i†Çò ~¡"Í°+¹

M
4) I =°i†Çò IV =¶ã`Ç"Í° ‹¬i†³Ø°#q.
Jï~‹¬“~ò# "Œi…Õ L<Œß~¡°.
5) III =°i†Çò IV =¶ã`Ç"Í° ‹¬i†³Ø°#q.

E
f~Œà<Œ° : I) JO^Î~°¡ ~Œ[H©†°Ç <Œ†Ç°ä›½° <Í~O ¡ Kͪ~¡°.
9. ㄬH›@# : ‡¦¼H›“s…Õ „¬xKÍÀ‹ XH› Hêià䛽xH÷ 6Q®°~¡°
II) Jï~‹¬~ “ ò# "Œ~¡O^Î~¶¡ ~Œ[H©†°Ç <Œ†Ç°ä›½…è.

D
9
„²Á°<Œß~¡°. =°i†Í° W`Ç~¡ Hêià䛽¶ 6

59
III) KÇOã^ΉõY~Ÿ =°i†Çò ~¡"Í°+¹ ~Œ[H©†Ç°

CA
95
<Œ†Ç°ä›½° Hê~¡°. Q®°~¡° „²Á#° H›ey†ÇòO_ȅè^ΰ.
f~Œà<Œ° : I) ‡¦¼H›“s…Õx JO^Î~¡° Hêià䛽¶ „²Á#°

50
IV) KÇOã^ΉõY~Ÿ =°i†Çò ~¡"Í°+¹° W^Îí~¡¶
H›ey†Çò<Œß~¡°.
98
A
<Í~¡O K͉§~¡°.
II) ‡¦¼H›s
“ …Õx HùO`Ç=°Ok Hêià䛽ä›½ „²Á ° …è~°¡ .
:8
1) I, II =°i†Çò IV f~Œà<Œ° =¶ã`Ç"Í° ‹¬i†³Ø°#q.
E

2) II =°i†Çò IV =¶ã`Ç"Í° ‹¬i†³Ø°#q. III) XH› Hêià䛽_È° `DŽ¬æ ‡¦¼H›“s…Õx q°ye#


h
.P

Hêià䛽O^Î~¡¶ 䛽@°O| x†Ç°Oã`Ç}


IC

3) IV =¶ã`Ç"Í° ‹¬i†³Ø°#k.
AD

4) I =°i†Çò IV =¶ã`Ç"Í° ‹¬i†³Ø°#q. „¬^Μux ‡\÷OKŒ~¡°.


AB
O

5) Un ‹¬i†³Ø°#k Hê^ΰ. IV) HùO`Ç=°Ok Hêià䛽ä›½ 6Q®°~¡° H›O>è

7. ㄬH›@# : ^͉O × `Ç~°¡ „¬ô# XO„²Hž± …Õ „¬`HÇ ê° ªkOz# Z䛽ø= =°Ok „²Á°<Œß~¡°.
ER
CH

"Œ~¡O^ÎiH© „¬`HÇ êxH÷ H›~Æ ¶¡ ‡†Ç° Kù„¬C# 1) Jhß ‹¬i†³Ø°#"Í


YD

ㄬƒ°’ `ÇÞO #[~Œ<Œ ㄬH\ › O÷ z#k. 2) III =¶ã`Ç"Í° ‹¬i†³Ø°#q.


,H

1) P.T. L+¬ <Œ°Q®° „¬`ÇHꁰ ªkOz#k. 3) I =°i†Çò III =¶ã`Ç"Í° ‹¬i†³Ø°#q.


T
AM

2) JtÞx `Ç$\÷…Õ „¬`ÇHêxß KÍ*ì~¡°ó䛽#ßk. 4) I =¶ã`Ç"Í° ‹¬i†³Ø°#k.


H

3) ̋á<Œ<³‚¬ðޅ˜ XH› „¬`ÇH›O ªkOz#k.


M

5) Un ‹¬i†³Ø°#k Hê^ΰ.
AM

f~Œà<Œ° : I) "³ò`ÇO ë 6 „¬`HÇ ê° ^͉O× `Ç~°¡ „¬ô# Qï eKŒ~¡°.


IG

10. ㄬH›@# : ̋áx䛽° ^͉§xH÷ À‹= Kͪë~¡°.


II) „².\÷.L+¬ <Œ°Q®° H› Æ ~¡¶‡†Ç° #[~Œ<Œ
KH

f~Œà<Œ° : I) „¬ô~°¡ +¬µ° ª^¥~¡}OQê ^͉§xH÷ À‹= Kͪë~°¡ .


ïQ°Kǰ䛽Ok.
R

II) Z=ï ~ á ` Í "Œi ^Í ‰ §xH÷ À‹=KÍ ª ë ~ Ë


III) JtÞx H›~ Æ ¶¡ ‡†Ç° #[~Œ<Œ Qï °Kǰ䛽Ok. "Œ~¡O^Î~¡¶ ̋áx䛽…è.
IV) ̋á<Œ<³‚ð¬ ޅ˜ 2 H› Æ ~¡¶‡†Ç° #[~Œ<Œ III) „¬ô~¡°+¬µ…Õ ̋áx䛽O^Î~¡¶ ^͉§xH÷ À‹=
ïQ°Kǰ䛽#ßk.
Kͪë~¡°.
V) ㄬƒ’°`ÇÞO "³ò`ÇëO 6 H›Æ ~¡¶‡†Ç°#°
IV) =°‚²ìˆ×° ^͉§xH÷ À‹=K͆ǰ~¡°.
#[~Œ<ŒQê JOkOzOk.
1) I =°i†Çò II ‹¬i†³Ø°# f~Œà<Œ°.
1) Un ‹¬i†³Ø°#k Hê^ΰ.
2) I, II =°i†Çò III ‹¬i†³Ø°# f~Œà<Œ°.
2) f~Œà#O-II =¶ã`Ç"Í° ‹¬i†³Ø°#k.
3) III =¶ã`Ç"Í° ‹¬i†³Ø°# f~Œà#O.
3) f~Œà#O-V =¶ã`Ç"Í° ‹¬i†³Ø°#k.
4) Jhß ‹¬i†³Ø°# f~Œà<Œ…è.
4) I =°i†Çò IV f~Œà<Œ° =¶ã`Ç"Í° ‹¬i†³Ø°#q.
5) Un ‹¬i†³Ø°# f~Œà#O Hê^ΰ.
5) III =°i†Çò V f~Œà<Œ° =¶ã`Ç"Í° ‹¬i†³Ø°#q.

REASONING ABILITY (LOGICAL) ○ ○ ○ ○ ○ ○ ○ ○


151
○ ○ ○ ○ ○ ○ ○ ○ ○ ○ ○ ○ ○ ○ ○ ○ ○ ○ ○
Study Material
1234567890123456789012
12345678901234567890123456789012123456789012345678901234567890121234567890123456789012345678901212345678901234567890123
12345678901234567890123

q=~¡}ì`ÇàH› ["Œ|°°
12345678901234567890123456789012123456789012345678901234567890121234567890123456789012345678901212345678901234567890123
1234567890123456789012 12345678901234567890123
12345678901234567890123456789012123456789012345678901234567890121234567890123456789012345678901212345678901234567890123
1234567890123456789012 12345678901234567890123
1234567890123456789012
12345678901234567890123456789012123456789012345678901234567890121234567890123456789012345678901212345678901234567890123
12345678901234567890123
12345678901234567890123456789012123456789012345678901234567890121234567890123456789012345678901212345678901234567890123
1234567890123456789012
12345678901234567890123456789012123456789012345678901234567890121234567890123456789012345678901212345678901234567890123
1234567890123456789012 PRACTICE TEST - 2 : 12345678901234567890123
12345678901234567890123
12345678901234567890123456789012123456789012345678901234567890121234567890123456789012345678901212345678901234567890123
1234567890123456789012 12345678901234567890123

1. (5) Q®}÷`lj§GO…Õ |‚Ôì#°O^Î~¡¶ ‹¬Ow`Ç q^¥ÞO‹¬°…è. ‹¬i†³°Ø #k. KÇOã^ΉYõ ~Ÿ =°i†Çò ~¡"°Í +¹° <Í~O¡ Kͪ~¡°.
Hêx ‹¬ O w`Ç q^¥ÞO‹¬ ° O^Î ~ ¡ ¶ Q® } ÷ ` Ç ‰ §GO…Õ Hê|\÷“ "Œ~¡° Jï~‹¬“†Ç¶¼~¡°.
|‚Ôì#°° Hê~¡°. HùO`Ç=°Ok ‹¬Ow`Ç q^¥ÞO‹¬°° f~Œà#O-IV =¶ã`Ç"Í° ‹¬i†³Ø°#k.
=¶ã`Ç"°Í Q®}` ÷ ‰Ç §GO…Õ |‚Ôì#°°. Hê=ô# f~Œà#O- 7. (2) f~Œà#O-I `DŽ¬C. ZO^ΰH›O>è "³ò`ÇëO „¬`ÇHê ‹¬OY¼
I ‹¬i†³°Ø #k. f~Œà#O-II `DŽC ¬ . f~Œà#O-III ㄬH@ › #…Õ 5. f~Œà#O-II ‹¬i†³Ø°#k. „¬`ÇHêxH÷ H›Æ Kù„¬C#
À„~ùø#|_Èx T‚¬ì#O. <Œ°Q®° „¬`ÇHêä›½ <Œ°Q®° H›Æ°.
2. (1) f~Œà#O-II =°i†Çò f~Œà#O-IV° =¶ã`Ç"Í° Wzó# f~Œà#O-III `DŽ¬C. JtÞx „¬`ÇH›O ïQ=…è^ΰ. `Ç$\÷…Õ
ㄬH›@# ㄬHê~¡O ‹¬i†³Ø°#q. KÍ*ì~¡°ó䛽#ßk. f~Œà#O-IV `DŽ¬C. XH› „¬`ÇHêxH÷

Y
3. (2) "³¶‚¬ì<£ 䛽@°O| ‹¬ƒ°’ ¼O^Î~¶¡ x*ì~òf„¬~°¡ …è Hê|\÷“ H›ÆKù„¬C# ̋á<Œ <³‚¬ðޅ˜ H›Æ~¡¶‡†Ç°#° =¶ã`Ç"Í°
"³ ¶ ‚¬ ì <£ ä› ½ @°O|O…Õx L^˼Q® ‹ ¬ ° ë  O^Î ~ ¡ ¶ ïQ°ó䛽#ßk.

M
x*ì~òf„¬~°¡ …è. \ f~Œà#O-II ‹¬i†³Ø°#k. f~Œà#O-IV `DŽ¬C. "³ò`ÇëO #[~Œ<Œ ªÚ=òà 5

E
f~Œà#O-a `DŽ¬C. ZO^ΰH›O>è "³¶‚¬ì<£ 䛽@°O|O…Õ „¬`ÇHêä›½ 5 H›Æ°.

D
9
HùO^Î~°¡ x~¡°^˼Q®°°. f~Œà#O-III =°i†Çò f~Œà#O- 8. (1) ~Œ[H©†°Ç <Œ†Ç°ä›½…Õ Z䛽ø= =°Ok J|^¥œ° K³|°`Œ~¡°

59
IV° ㄬH›@# ㄬHê~¡O ‹¬i†³Ø°#q Hê=ô. - ㄬH@ › #. Hêx J|^¥œ° K³À„æ"Œ~¡O^Î~¶¡ ~Œ[H©†°Ç
CA
95
4. (2) ^ Î # =O`Ç ° ° `Ç „ ¬ æ =°ï ~ =Þ~¡ ¶ q=¶#O…Õ <Œ†Ç°ä›½° Hê~¡°. Hê=ô# f~¡à#O-I `DŽC¬ . Q®°}‰õY~Ÿ

50
ㄬ†Ç¶}÷OKDžè~¡x ㄬH›@#…Õ =ôOk. Hê=ô# f~Œà#O- J|^¥œ HË~¡° J~ò#O`Ç =¶ã`Œ# J`Ç#° ~Œ[H©†°Ç
98
A
I ‹¬i†³Ø°#^Í. <Œ†Ç°ä›½_È° Hê^ΰ. f~Œà#O-II 䛀_¨ `ÇÀ„æ. ZO^ΰH›O>è
:8
q=¶#O…Õ ㄬ†Ç¶}÷OKÍ "Œi…Õ HùO^Î~¡° =¶ã`Ç"Í° J|^¥œ° K³„¬æx ~Œ[H©†Ç° <Œ†Ç°ä›½° HùO`Ç=°Ok
E
h
.P

J<Œ~ËQ®¼O‡†Ç¶¼~¡°. Hê=ô# JO^Î~¡° ^Î#=O`Ç°¶ =ô<Œß~¡° (ㄬH@› # ㄬHê~¡O). f~Œà#O-III ‹¬i†³Ø°#k


IC
AD

J<Œ~ËQ®¼O ‡~ò#@°Á Hê^ΰ. f~Œà#O-II `DŽ¬C. Hê^ΰ. ZO^ΰH›O>è JO^Î~¡O ~Œ[H©†Ç° <Œ†Ç°ä›½¶
AB
O

q=¶#O…Õ ㄬ†Ç¶}÷OKÍ"Œ~¡O^Î~¡¶ ^Î#=O`Ç°…è Hêx, J|^¥œ° K³|°`Œ~¡x ㄬH›@#…Õ …è^ΰ. f~Œà#O-IV


^ Î # =O`Ç ° O^Î ~ ¡ ¶ q=¶#O…Õ ㄬ † Ç ¶ }÷ ª ë ~ ¡ x XH› T‚¬ì. Hê=ô# Jxß f~Œà<Œ¶ ‹¬i†³Ø°#q Hê^ΰ.
ER
CH

ㄬH›@#…Õ …è^ΰ. Hê=ô# f~Œà#O-III `DŽC¬ . f~Œà#O- 9. (5) ‡¦¼H›s“ …Õx JO^Î~°¡ Hêià䛽ä›€ „² Á ° =ô<Œß~Ë, …è^Ë
YD

IV ㄬH›@# ㄬHê~¡O `DŽ¬C. ㄬH@› #…Õ W=ޅè^°Î . Hê=ô# f~Œà#O-I ‹¬i†³°Ø #k Hê^ΰ.
,H

5. (1) ㄬH@ › # ㄬHê~¡O HêÁ‹°¬ …Õx JO^Î~°¡ q^¥¼~¡°Ö ¶ `³e"³#á "Œö~ ‡¦¼H›“s…Õx HùO`Ç=°Ok Hêià䛽ä›½ „²Á° …è~¡<Í
T
AM

J<Ík ‹¬æ+¬“O. Hê=ô# ~¡"Í°+¹ P HêÁ‹¬°…Õx q^¥¼iÖ Hê^ΰ. q+¬†Ç°O Q®°iOz ‹¬æ+¬“`Ç ã„¬H›@#…Õ W=Þ|_ȅè^ΰ.
H
M

f~Œà}O- IV ‹¬i†³Ø°#k. Hê=ô# f~Œà#O-II `DŽ¬C.


AM
IG

f~Œà#O-I `DŽC¬ . JO^Î~¶¡ `³e"³# á "Œö~ J<Ík ㄬH@ › #. ä› ½ @°O| x†Ç ° Oã`Ç } ä› ½ ‹¬ O |Ok O z# JO‰× " Í °
KH

Hêx HùO`Ç=°Ok =¶ã`Ç"Í° `³e"³á#"Œö~ J<Ík `DŽ¬C. ㄬH›@#…Õ …è^ΰ. Hê=ô# f~Œà#O-III `DŽ¬C.
R

f~Œà#O-II =°i†Çò IIIä›½ ㄬH@ › #`Ë ‹¬O|O^OÎ …è^°Î . XöH XH›ø Hêià䛽_H÷ 6Q®°~¡° „² Á °<Œß~¡#ß q+¬†°Ç "Í°
6. (3) ㄬH@ › # ㄬHê~¡O <Í~O¡ K͋#² ~Œ[H©†°Ç <Œ†Ç°ä›½O^Î~¶¡ ㄬH@› #…Õ À„~ùø#|_#k. q°ye# Hêià䛽ä›½ Q® „² Á
Jï~‹¬“†Ç¶¼ö~ `DŽ¬æ JO^Î~¡° ~Œ[H©†Ç° <Œ†Ç°ä›½° <Í~¡O ‹¬OY¼ Q®°iOz K³„æ¬ |_ȅ^ è °Î . Hê=ô# f~Œà#O-IV `DŽC¬ .
K͆ǰ…è^ΰ. Hê=ô# f~Œà#O-I `DŽ¬C. \ Jxß f~Œà<Œ° ‹¬i†³Ø°#q Hê=ô.
Jï~‹¬“~ò# "Œ~¡O^Î~¡¶ ~Œ[H©†Ç° <Œ†Ç°ä›½…è Jx 10. (3) Wzó# ㄬ H › @ # ㄬ H ê~¡ O f~Œà#O- III =¶ã`Ç " Í °
ㄬH›@#…Õ …è^ΰ. Hê=ô# f~Œà#O-II `DŽ¬C. ‹¬i†³Ø°#k. f~Œà#O-I ㄬH›@#䛽 ‹¬O|O^ÎO …èxk.
Jï~‹¬“~ò# "Œi…Õ KÇOã^ΉõY~Ÿ =°i†Çò ~¡"Í°+¹° f~Œà#O- II `DŽ¬C. ZO^ΰH›O>è ^͉§xH÷ À‹=KÍÀ‹
=ô<Œß~¡° Hê|\÷“ ㄬH›@# ㄬHê~¡O "Œ~¡° ~Œ[H©†Ç° "Œ~¡O^Î~¡¶ ̋áx䛽…è Jx ㄬH›@#…Õ W=Þ|_ȅè^ΰ.
<Œ†Ç°ä›½…è. Hê=ô# f~Œà#O-III `DŽ¬C. f~Œà#O-IV f~Œà#O-IV XH› T‚¬ì =¶ã`Ç"Í°.

REASONING ABILITY (LOGICAL) ○ ○ ○ ○ ○ ○ ○ ○


152
○ ○ ○ ○ ○ ○ ○ ○ ○ ○ ○ ○ ○ ○ ○ ○ ○ ○ ○
Study Material
123456789012345678901234567890121234567890123456789012345678901212
123456789012345678901234567890121234567890123456789012345678901212
12345678901234567890123456789012123456789012345678901234567890121234567890123456789012345678901212345678901234567890123456789012123
123456789012345678901234567890121234567890123456789012345678901212345678901234567890123
12345678901234567890123456789012123456789012345678901234567890121234567890123456789012345678901212345678901234567890123
1234567890123456789012345678901212345678901234567890123456789012123456789012345678901234567890121234567890123
1234567890123456789012345678901212345678901234567890123456789012123456789012345678901234567890121234
12345678901234567890123456789012123456789012345678901234567890121234567890123456789012345678901212345678901234567890123456789012123
123456789012345678901234567890121234567890123456789012345678901212345678901234567890123
12345678901234567890123456789012123456789012345678901234567890121234567890123456789012345678901212345678901234567890123
1234567890123456789012345678901212345678901234567890123456789012123456789012345678901234567890121234567890123
1234567890123456789012345678901212345678901234567890123456789012123456789012345678901234567890121234
123456789012345678901234567890121234567890123456789012345678901212
12345678901234567890123456789012123456789012345678901234567890121234567890123456789012345678901212345678901234567890123456789012123
123456789012345678901234567890121234567890123456789012345678901212345678901234567890123
12345678901234567890123456789012123456789012345678901234567890121234567890123456789012345678901212345678901234567890123
1234567890123456789012345678901212345678901234567890123456789012123456789012345678901234567890121234567890123
1234567890123456789012345678901212345678901234567890123456789012123456789012345678901234567890121234
PREVIOUS BITS
12345678901234567890123456789012123456789012345678901234567890121234567890123456789012345678901212345678901234567890123456789012123
123456789012345678901234567890121234567890123456789012345678901212345678901234567890123
123456789012345678901234567890121234567890123456789012345678901212
12345678901234567890123456789012123456789012345678901234567890121234567890123456789012345678901212345678901234567890123
1234567890123456789012345678901212345678901234567890123456789012123456789012345678901234567890121234567890123
1234567890123456789012345678901212345678901234567890123456789012123456789012345678901234567890121234
12345678901234567890123456789012123456789012345678901234567890121234567890123456789012345678901212345678901234567890123456789012123
123456789012345678901234567890121234567890123456789012345678901212345678901234567890123
123456789012345678901234567890121234567890123456789012345678901212
12345678901234567890123456789012123456789012345678901234567890121234567890123456789012345678901212345678901234567890123
1234567890123456789012345678901212345678901234567890123456789012123456789012345678901234567890121234567890123
1234567890123456789012345678901212345678901234567890123456789012123456789012345678901234567890121234
12345678901234567890123456789012123456789012345678901234567890121234567890123456789012345678901212345678901234567890123456789012123
123456789012345678901234567890121234567890123456789012345678901212345678901234567890123
12345678901234567890123456789012123456789012345678901234567890121234567890123456789012345678901212345678901234567890123
1234567890123456789012345678901212345678901234567890123456789012123456789012345678901234567890121234567890123
1234567890123456789012345678901212345678901234567890123456789012123456789012345678901234567890121234
12345678901234567890123456789012123456789012345678901234567890121234567890123456789012345678901212345678901234567890123456789012123
123456789012345678901234567890121234567890123456789012345678901212345678901234567890123
123456789012345678901234567890121234567890123456789012345678901212
12345678901234567890123456789012123456789012345678901234567890121234567890123456789012345678901212345678901234567890123456789012123
123456789012345678901234567890121234567890123456789012345678901212345678901234567890123
1234567890123456789012345678901212345678901234567890123456789012123456789012345678901234567890121234567890123
1234567890123456789012345678901212345678901234567890123456789012123456789012345678901234567890121234

G (1-2) kQ®°=# ï~O_È° ㄬ=KÇ#=ò° (Steatements) 5. ㄬH›@# : "Œ`Œ=~¡} „¬i‹²Ö`Ç°° Z…ì L#߄¬æ\÷H© XH›ø
~
ï O_È° x~Œœ~}
¡ ° (Conclusions) W=Þ|_#q. ^Î`ëÇ x~ŒÖ~}
¡ ‡¦é\Õ ä›€_¨ ‡_È=䛽O_¨ ‘\˜ f†Ç°Q®
(Conclusions)  P^¥~¡OQê ‹¬i†³Ø°# SzóH›=ò#° ã‰õ+¬»"³°Ø# #g# #=ü<Œ° Q® ïH"³°~Œ#°
x~¡~
â òOKÇO_? (A.P. Constable Prelims -2016)
`dž¶Ç ~¡°K͆°Ç Q® L#ß`Ç H›Ōh X. (S.I.- '08)
1. ㄬ=KÇ#=ò°: A) x~¡Þ‚¬ì} ª=°~¡Ö¼O Q® =¼ä›½ëO^ÎiH÷ f~Œà<Œ° : I) ïH"³°~Œ „¬iã‰×=°…Õ X H›ŌhH÷ `DŽ¬æ
"Íö~ U H›ŌhH÷ =°Oz À„~¡° …è^ΰ.
q㉧Ou ‹¬=°†Ç°O ^ù~¡°ä›½#°.
II) Z=ï~á<Œ ïH"³°~Œ X`Ë ‚²ì`Ç"³°Ø# ‘@°Á
B) ‹¬°h`Ç佛 JkH"
› °³ #Ø „¬xƒì~¡O L#߄¬æ\÷H,©
f†Ç°Q®~¡°.
q㉧OuH÷ ‹¬=°†Ç°O ^ù~¡°ä›½#°.
6. ㄬH›@# : KŒ…ì =°Ok q^¥¼~¡°Ö =°#‹¬ë`ÇÞO KŒ…ì

Y
f~Œà<Œ° : I) ‹¬°h`Ç x~¡Þ‚¬ì} ª=°~¡Ö¼O Q® =¼H÷ë.
KÇ°~¡°ä›½Qê =ôO@°Ok.

M
II) ‹¬°h`Ç „¬iã‰×q°OKÍ `Ç`ÇÞOQ® =¼H÷ë.
f~Œà<Œ° : I) KÇ ° ~¡ ° ï H á # =°#‹¬ ë ` Ç Þ O H› e y†Ç ò O_È x
1) I =°i†Çò II J#°‹¬iªë~ò.
q^¥¼~¡°Ö° =ôO_È~¡°.

E
2) I =¶ã`Ç"Í° J#°‹¬i‹¬°ëOk.
II) HùO`Ç = °Ok q^¥¼~¡ ° Ö  ° ‹¬ Q ® @ ° H› < Œß

D
9
59
3) II =¶ã`Ç"Í° J#°‹¬i‹¬°ëOk. `Ç䛽ø=Qê xÀ‹ë[OQê P…Õzªë~¡°.
CA
95
4) I, II ° ï~O_ȶ J#°‹¬iOKÇ=ô. (Bank of Baroda P.O. - 2005)

50
2. ㄬ=KÇ#=ò : ^³á~¡¼=O`Ç°#° J^Î$+¬“O =i‹¬°ëOk. 7. ㄬH›@# : †³òH›ø „¬xf~¡° Q®°iOz =¶‹¬“~Ÿ
Insat 2-D

98
A
f~Œà<Œ° : I)q[†Ç°ª^#Π䛽 i‹¹ø f‹¬°HË=_ÈO J=‹¬~O¡ . H›Oã\Յ˜ ªÏH›~¼¡ O ^¥Þ~Œ ‰§G"Í`
ëÇ ° =°~ùH›ªi
:8
II) „² i H÷ " Œ~¡ ° "Œi =°~¡ } ìxH÷ =òO^Î °
„¬î~ŒÞƒì¼‹¬O K͉§~¡°.
E
h

f~Œà<Œ° : I) H›$ãu=° L„¬ãQ®‚ð¬ xß JO`ÇiH›OÆ …ÕxH÷ „¬OÀ„


.P

J<ÍH›ª~¡°Á KÇx‡é`Œ~¡°.
IC

=òO^ΰ i‚¬ð~¡ž…˜ž `DŽ¬æx‹¬i.


AD

1) I =¶ã`Ç"Í° J#°‹¬i‹¬°ëOk.
(Bank of Baroda P.O. - 2008)
2) II =¶ã`Ç"Í° J#°‹¬i‹¬°ëOk.
AB
O

‰§G"Í`Ç끰 JO`Ç=òO^ΰ ㄬ†³¶yOz#


II)
3) I Hêh …è^¥ II Hêh J#°‹¬iªë~ò.
ER
CH

H›$ãu=° L„¬ãQ®‚¬ð#° JO`ÇiH›ÆO…ÕxH÷


4) I, II ° ï~O_ȶ J#°‹¬iOKÇ=ô.
YD

„¬OÀ„=°O^ΰ 䛀_¨ W…ì<Í i‚¬ð~¡ž…˜ž


3. ㄬH›@# : ‹¬=°‹¬¼ #°O_ |†Ç°@„¬_°È @䛽 L`Ç= ë ° =¶~¡=¾ ò Kͪ~¡°.
,H

^¥xx „¬i+¬øiOKÇ°Hù#°@. (Gr. I - 2008)


T

: ‰×`Œaœ ZH±žã̄‹¹…Õ ㄬ†¶


Ç }÷OKÇ°@䛽 i[ö~Þ+¬<£
AM

8. ㄬH›@#
f~Œà<Œ° : I) U ‹¬=°‹¬¼#¶ Z^ΰ~ùø#H›‡é`Í g° rq`ÇO
H

KÍ~òOKǰ䛽x "Íz†ÇòO_Í *ìa`Œ…Õ Q®


M

=°O^ÎHù_Q® `džǶ~¡Q®°#°. Zxq°k =°Ok =°OkH÷ Z@“öHä›½ ‹Ô@°


AM
IG

II) ‹¬=°‹¬¼ #°O_ `DŽ²æOKÇ°Hù#°@䛽, Jxß i[ö~Þ+¬<£ öH\ì~òOKÇ|_#k.


KH

"ׁ͈…Õ g°~¡° „¬i‘ø~¡ =¶~¡¾=ò#° H›e¾


R

(Syndicate Bank P.O. - 2008)

†ÇòO_È=…ÿ#°. f~Œà<Œ° : ª^¥~¡}OQê ‰×`Œaœ ZH±žã̄‹¹ †³òH›ø


I)

4. ㄬH›@# : ƒì~¡`^ Ç ‰Í O× †³òH›ø PiÖH=


› ¼=‹¬Ö J_È=ô ̄á#<Í "³~ò\÷OQ· e‹¹“…Õ À„~¡° =ô#ß"ŒiH÷ ‹Ô@°
P^¥~¡„¬_†Çò#ßk. ^ù~¡H›_ÈO KŒ…ì H›+¬“O.
II) ‰×`Œaœ ZH±žã̄‹¹ KŒ…ì `Ç䛽ø= ƒÿ~Ÿë#°
f~Œà<Œ° : I) ƒì~¡`Ç^͉×O †³òH›ø PiÖH›‹²Öux "³°~¡°Q®°
„¬~¡KÇ°@䛽 K³@Á#° ~¡H÷ÆOKÇ=…ÿ#°. H›ey =ôO@°Ok.
9. ㄬH›@# :L^Άǰ„¬ô #_ÈH›° ‚¬ðxH›~¡=ò.
(Group I - 2008)

ƒì~¡`Ç^͉×O †³òH›ø PiÖH›‹²Öux "³°~¡°Q®°


(Group I - 2008)
II)
f~Œà<Œ° : I) P~ËQ®¼=O`Ç"°³ #Ø JO^Î~°¡ ㄬ[° L^Ά°Ç „¬ô
„¬~K
¡ °Ç Hù#°@䛽 J_È=ô#° „¬i~¡HOÆ÷ Kǰ䛽#ß
#_ÈH›ä›½ "³ˆÜ¤^ΰ~¡°.
KŒ°#°.
II) ª†Ç°Oã`DŽ¬ô #_ÈH›° ‚¬ðxH›~¡=ò.
REASONING ABILITY (LOGICAL) ○ ○ ○ ○ ○ ○ ○ ○
153
○ ○ ○ ○ ○ ○ ○ ○ ○ ○ ○ ○ ○ ○ ○ ○ ○ ○ ○
Study Material
10. ㄬH›@# : x~Œà} „¬#°° J=°°K͆Çò@…Õ J#°ƒ’=O ̄¦~ò…ÿá# "Œ~¡O^Î~¡¶ ï~Q®°¼~ŸQê ‹¬¶ø…˜H÷
II)

=ô#ß ‹¬=°~¡Ö=O`Ç"³°Ø# ‡é\©`Ç`ÇÞO =ô#ß ‚¬ð[ï~á#"Œö~.


HêOã\ìH›“~¡Á #°O_ ‹Ô…˜Û>ÿO_È~¡Á#° P‚¬ðÞxOKÇ_È 12. ㄬH›@# : `ÇeÁ^ÎOã_È°° "Œi „²Áä›½ L#ß`ǪÖ~ò
"³°#Ø k - XH› ¢Ì„á"\
Í ˜ x~Œà}~¡OQ® ‹¬O‹¬Ö <Ë\©‹.¹ q^μ#° JOkOKÇ°@䛽 ZO`Ç Yï~óá <Œ ƒi’ ªë~°¡ .
(M.A.T. - 2005) (M.A.T. - 2005)

11. ㄬH›@# : „¬sH›


Æ …Õ ̄¦~ò…˜ J=Þ_¨xH÷ Hê~¡}O ‹¬¶ø…˜H÷ f~Œà<Œ° : I) D ~ËA…ÕÁ `ÇeÁ^ÎOã_È°O^Î~¡¶ ƒìQê
ï~Q®°¼~ŸQê ‚¬ð[~¡=ÞH›‡é=_È"Í°. HùO`Ç=°Ok _È|°Ä#ß"Œö~.
ï~Q®°¼~Ÿ ‹¬¶“_³O\˜ž 䛀_¨ ̄¦~ò†Ç¶¼~¡°. II) `ÇeÁ^ÎOã_È°O^Î~¡¶ `Ç=° „²Áä›½ =°Oz
(SSC - 2005) ‹¬¶ø° ^¥Þ~Œ L#ß`Ç q^μ JOk "Œ~¡°
f~Œà<Œ° : I) ï~Q®°¼~ŸQê ‹¬¶ø…˜H÷ ‚¬ð[ï~á# "Œ~¡O^Î~¡¶ ‹¬ O „¬ î ~¡ â Ja = $kœ x ‡ÚO^¥x ‹² Ö ~ ¡

Y
‡‹¹ J=ô`Œ~¡°. Ja~¡°zx H›ey†Çò<Œß~¡°.

M
12345678901234567890123456789012123456789012345678901234567890121234567890123456789012345678901212345678901234567890123
1234567890123456789012 123456789012345678901234

q=~¡}ì`ÇàH› ["Œ|°°
12345678901234567890123456789012123456789012345678901234567890121234567890123456789012345678901212345678901234567890123
1234567890123456789012 123456789012345678901234
1234567890123456789012 123456789012345678901234
12345678901234567890123456789012123456789012345678901234567890121234567890123456789012345678901212345678901234567890123
PREVIOUS BITS :

E
12345678901234567890123456789012123456789012345678901234567890121234567890123456789012345678901212345678901234567890123
1234567890123456789012 123456789012345678901234
12345678901234567890123456789012123456789012345678901234567890121234567890123456789012345678901212345678901234567890123
1234567890123456789012 123456789012345678901234
12345678901234567890123456789012123456789012345678901234567890121234567890123456789012345678901212345678901234567890123
1234567890123456789012 123456789012345678901234
12345678901234567890123456789012123456789012345678901234567890121234567890123456789012345678901212345678901234567890123
1234567890123456789012 123456789012345678901234

D
9
59
1. (1) ㄬ=KÇ#=ò° A, B #° ä›Æ½}âOQê „¬ijeÀ‹ë ‹¬°h`Ç 7. (3) f~Œà#O-I =°i†Çò f~Œà#O- II ï~O_ȶ ㄬH›@#
CA
95
ª=°~¡Ö¼O Q® =¼H÷ë H›#°H› `Ç#H÷ q㉧Ou ‹¬=°†Ç°O ㄬHê~¡O ‹¬i†³Ø°#"Í.

50
^ù~¡°ä›½#°. ^¥x`Ë ‡@° ‹¬°h`Ç „¬iã‰×q°OKÍ =¼H÷ë U L„¬ãQ®‚¬ð<³áß<Œ JO`ÇiH›ÆO…ÕxH÷ „¬OÀ„ =òO^ΰ ^¥x
98
A
Jx 䛀_¨ `³°‹¬°ëOk. Hê=ô# ï~O_ȶ x~Œœ~¡}°
„¬xf~¡°#° `DŽ¬æH› =òO^ΰQê<Í „¬sH÷ƪë~¡°.
:8
‹¬i†³Ø°#"Í. Hê=ô# nxH÷ ‹¬i†³Ø°# ‹¬=¶^¥#O (1).
E

8. (1) Jxß ï~áˆ×¤…ìQê<Í ‰×`Œaœ ZH±žã̄‹¹…Õ 䛀_¨ ‹¬i„¬_¨


h

2. (1) q[†Ç°ª^#Î …Õ H›+„ “¬ _¬  ã‰×q°OKÇ_OÈ (i‹¹ø f‹¬°HË=_ÈO)


.P
IC

`DŽ¬æx‹¬i. J„¬C_Í q[†Ç°O =°#eß =i‹¬°ëOk. Hê=ô# ƒÿ~Ÿë ‹¬OY¼ =ôO\ì~ò. Hê=ô# f~Œà#O-II ‹¬i†³Ø°#k
AD

x~Œœ~} ¡ I ‹¬i†³°Ø #^Í. x~Œœ~} ¡ II ㄬ=KÇ#=ò`Ë ‡ÚO`Ç# Hê^ΰ. ㄬH›@#…Õ ‹Ô@° ^ù~¡H›_ÈO Qù„¬æ q+¬†Ç°"Í°
AB
O

…èxk. Hê=ô# Wk ‹¬i†³Ø°#k Hê^ΰ. Hê=ô# nxH÷ J#ß@°ÁQê W=Þ|_#k. Hê=ô# f~Œà#O-I ‹¬i†³Ø°#k.
ER
CH

‹¬i†³Ø°# ["Œ|° (1). 9. (1) Wzó# ㄬH›@# `DŽ¬C. L^Άǰ„¬ô #_ÈH›° ZO`Ë
YD

3. (4) ㄬH›@# #°O_, ‹¬=°‹¬¼ =zó#„¬ô_È° ^¥xx J…ìöQ ㄬ † ³ ¶ [#H› ~ ¡ O . Hê=ô# f~Œà#O I ‹¬ i †³ Ø ° #k.
,H

ªQ®n†Ç°ä›½O_¨ "³O@<Í „¬i+¬øiOKÇ°HË=_È"Í° L`Çë=° ª†Ç°Oã`DŽô¬ #_ÈH › ° ‚¬ðxH›~=


¡ °x =°#O f~ŒàxOKDž=
è ò.
T
AM

=¶~¡¾O Jx `³e†ÇòKÇ°#ßk. f~Œà#O-I =°i†Çò


10. (1) ㄬH›@# ㄬHê~¡O f~Œà#O-I ‹¬i†³Ø°#k. J#°ƒ’=O,
H

f~Œà#O-II ï~O_ȶ ㄬH›@#`Ë ‹¬O|O^ÎO …èx"Í.


M

‹¬=°~¡Ö`Ç, ‡é\©`Ç`ÇÞO =ô#ß"Œö~ >ÿO_È~¡°Á "͆Ƕx


AM

4. (1) ƒì~¡`^ Ç ‰Í O× †³òH›ø PiÖH› =¼=‹¬Ö J_È=ôÌ„á<Í P^¥~¡„_ ¬ 


IG

=ô#ßk. Hêx öH=O J_È=ôÌ„á<Í P^¥~¡„¬_†Çò#ß^Îx ‹¬æ+¬“OQê ㄬH›@#…Õ À„~ùø<Œß~¡°. Hêx, J…ìO\÷ "Œix
KH

H›#°Qù#_ÈO ‹¬°ƒ’"³¶ ? H›+¬“"³¶ ? ㄬH›@#…Õ


R

^¥x J~¡ÖO Hê^ΰ. Hê=ô# f~Œà#O-I ㄬH›@##°


|„¬~¡°‹¬°ëOk. f~Œà#O-II ‹¬i†³Ø°#k Hê^ΰ. `³e†Ç°*è†Ç°…è^ΰ. Hê=ô# f~Œà#O-II ‹¬iHê^ΰ.
5. (2) X-H›Ōh `dž¶Ç ~¡°K͋# ² Hï "³°~Œ° Z…ìO\÷ "Œ`Œ=~¡} 11. (4) f~Œà#O-I =°i†Çò f~Œà#O-II ï~O_ȶ ㄬH›@#
„¬i‹²Ö`Ç°…Õ<³á<Œ =°Oz ‡¦é\Ձ#° f†Ç°Q®=ô J<Ík ㄬHê~¡O `ÇÀ„æ. ï~Q®°¼~Ÿ ‹¬¶“_³O\˜ž ̄¦~ò~ò#@°Á
ㄬH@› # `³e†Ç°*苰¬ Oë k. WH›ø_È "Í~ö U H›Ōh Hï "³°~Œ W=Þ|_ # k =°i†Ç ò ̄¦ ~ ò=_¨xH÷ Hê~¡ } O
Q®°iOz K³„¬æ…è^ΰ. Hê=ô# f~Œà#O-II ‹¬i†³Ø°#k.
ï~Q®°¼~ŸQê ‚¬ð[~¡°HêH›‡é=_È"Í° Jh K³„¬æ|_#k.
6. (2) KŒ…ì =°Ok q^¥¼~¡°Ö° =°#‹¬ë`ÇÞO KŒ…ì KÇ°~¡°ä›½Qê
12. (2) ㄬH›@# ㄬHê~¡O, f~Œà#O-II ‹¬i†³Ø°#k.
LO@°Ok JO>è q°ye# "Œ~¡° KÇ°~¡°ä›½^Î#O HùO`Ç
`Ç䛽ø= =ô#ß"Œö~. Hê=ô# f~Œà#O-II ‹¬i†³Ø°#k. ㄬH›@#…Õ ZH›ø_¨ _È|°Ä#ß"Œ~¡°, _È|°Ä…èx"Œ~¡° Jx
f~Œà#O-I ㄬH›@#䛽 q~¡°^ΜOQê =ô#ßk. q=iOKDžè^ΰ. Hê=ô# f~Œà#O-I ‹¬iHê^ΰ.

REASONING ABILITY (LOGICAL) ○ ○ ○ ○ ○ ○ ○ ○


154
○ ○ ○ ○ ○ ○ ○ ○ ○ ○ ○ ○ ○ ○ ○ ○ ○ ○ ○
Study Material
2 ㄬH›@#°
ㄬH›@#°

(Statements - Assumptions)
-
-
T‚¬ì#°
T‚¬ì#°

=òMì¼O‰§°...
G D qƒìQ®O…Õx ㄬ‰×hß x`Ǽrq`ÇO…Õ =°# KÇ°@¶“ [~¡°Q®°`Ç°#ß Z<Ëß ‹¬O„¦¬°@#Ì„á W=Þ|_È`Œ~ò.
G ㄬH›@#° XH› <Ë\©‹¬°, XH› Jƒ’¼~¡Ö#, XH› J_ÈÞ~Ÿ>ÿØ*˜"³°O\˜ …è^¥ XH› „¬OKŸ…ÿá<£ U^³á<Œ J=Þ=KÇ°ó.
G =¶ió ~Œ†Ç°_ÈO ‹¬i†³Ø°# T‚¬ìHê^ΰ.
G T‚¬ì#=ò J#°#k XH› "Œ¼Y¼…Õ ^¥y=ô#ß ƒìQ®O.

Y
G D qƒìQ®O #O^ΰ XH› ㄬH›@# =°i†Çò ^¥xH÷O^Î ï~O_È° …è^¥ JO`ÇH›<Œß Z䛽ø= J̋O„¬Â<£ž (T‚¬ì#°) =ôO\ì~ò.

M
Wzó# T‚¬ì#…Õ ㄬH›@##° ‹¬O`Ç$„²ë„¬~¡KÍ T‚¬ì##° ZOKÇ°HË"Œe.
G Wzó# ㄬH›@#䛽 =°i†Çò T‚¬ì##° xt`ÇOQê „¬ijeOz ‹¬i†³Ø°# P…ÕKÇ<Œ q^¥#O`Ë ‹¬=¶^¥#O W"ŒÞe.

E
G Wzó# ㄬH›@#…Õx JO‰×=ò#° P^¥~¡O K͋¬°Hùx, ^¥x #°O_ ‹¬¶zOKÇ° ãHù`Çë ㄬu‡^Î#° x[O J†Í°¼@@°Á =ôO>è<Í

D
9
"Œ\÷x T‚¬ì#° JO\ì~¡°.

59
G T‚¬ì#° ㄬH›@#䛽 ‹¬O|OkOz#q =¶ã`Ç"Í° J~ò=ôO_¨e. Wzó# T‚¬ì#° Hê^Îx ãHù`Çë ㄬu‡^Î# K͆ǰ䛀_È^ΰ. CA
95
J…ìO\÷q ‹¬i†³Ø°# ‹¬=¶^¥<Œ° Hê=ô.
50
98
A

=¶ki ㄬ‰×߁°
:8
E
h
.P
IC

2. ㄬH›@# :
AD

ㄬƒ’°`ÇÞO "Œ~¡° ï~ၰ ㄬ=¶^¥…Õ KÇx‡é~ò#

MODEL - 1
AB
O

"Œi 䛽@°Oƒìä›½ H›Æ~¡¶‡†Ç° #+¬“„¬i‚¬ð~¡O

G W"ŒÞx x~¡â~òOKǰ䛽<Œß~¡°.
ER
CH

ㄬH›@# ãH÷O^Î ï~O_È° T‚¬ì#° Wzó#„¬C_È°, "³ò^Î\ ÷

T‚¬ì#O : I) D #+¬“„¬i‚¬ð~¡O Hù~¡ä›½ J†Í°¼ Y~¡°óä›½


T‚¬ì# =¶ã`Ç"Í° ㄬH›@##° ‹¬O`Ç$„²ë „¬iÀ‹ë ‹¬=¶^¥#O
YD

‹¬i„¬_¨ x^Î°° ㄬƒ’°`ÇÞO =^Îí =ô<Œß~ò.


( 1 ) Qê#¶, ï~O_È= T‚¬ì# =¶ã`Ç"Í° ㄬH›@##°
II)
,H

ƒ’q+¬¼`Ç°ë…Õ ï~á…èÞ ã„¬=¶^¥° `ÇöQ¾ J=Hê‰×O

‹¬O`Ç$„²ë„¬iÀ‹ë ‹¬=¶^¥#O ( 2
) Qê#¶, "³ò^Î\ ÷ T‚¬ì#
T

=ôOk.
AM

ª^Î#:
=°i†Çò ï~O_È= T‚¬ì#° ï~O_ȶ Wzó# ㄬH›@##°
H

ㄬƒ°
’ `ÇÞO #+¬“ „¬i‚¬ð~¡O K³eO
Á KŒe Jx x~¡~
â òOKǰ䛽#ßk

3
M

‹¬O`Ç$„²„
ë i
¬ À‹ë ‹¬=¶^¥#O ( ) Qê#¶, ~
ï O_È° T‚¬ì#¶
Hê|\ ÷“ ^¥xH÷ ‹¬i„¬_¨ _È|°Ä ㄬƒ’°`ÇÞO =^Îí =ô#ß^Îx
AM
IG

Wzó# ㄬH›@##° ‹¬O`Ç$„²ë„¬~¡KÇH› ‡é`Í ‹¬=¶^¥#O


ƒìqOKÇ=KÇ°ó. Hê=ô# T‚¬ì#O I ‹¬=°~¡Öh†Ç°"Í°. Hêx

4
KH

( ) Qê#¶, "³ò^Î\ ÷ T‚¬ì#Qêh, …è^¥ ï~O_È=


ï~á…èÞ ã„¬=¶^¥° ƒ’q+¬¼`Ç°ë…Õ `ÇQ®¾=KÇ°ó#x T‚²ìOKÇ_ÈO

T‚¬ì#Qêh U^³á<Œ XH›\ ÷ Wzó# ㄬH›@##°


II
R

Jª^Î¼O Hê=ô# T‚¬ì#O ‹¬=°~¡Öh†Ç°O Hê^ΰ.

‹¬O`Ç$„²ë„¬iÀ‹ë ‹¬=¶^¥#O ( 5 ) Qê#¶ Q®°iëOKŒe.


‹¬=¶^¥#O (1).
1. ㄬH›@# : ‚¬ì~ŸÂ^£ "³°‚¬ì`Œ H›Ōh A …Õ À+~¡°ÁHˋ¬O `Ç#
3. ㄬH›@# : ‹²Q®ï~@°Á ã`ŒQ®@O P„²#"Œi…Õ Z䛽ø= =°Ok

_È|°Ä Y~¡°ó̄@“_ÈO E^ÎO`Ë ‹¬=¶#O |~¡=ô̄~¡°Q®°`Œ~¡°.

T‚¬ì#O : I) ‚¬ì~ŸÂ^£ "³°‚¬ì`Œ #‘“xß ‡ÚO^ΰ`Œ_È°. T‚¬ì#O : I) XH› =¼H÷ë ‹²Q®ï~@°Á ã`ŒQ®@O P„²#@Á~ò`Í

II) ‚¬ì~ŸÂ^£ "³°‚¬ì`Œ …샏ìxß ‡ÚO^ΰ`Œ_È° J`Çx †³òH›ø |~¡°=ô ̄~¡°Q®°`Ç°Ok.

ª^Î#: E^ÎO…Õ ïQ°„¬ô, F@=ò° =òO^ΰQê<Í `³°‹¬°HË=_ÈO II) XH› =¼H÷ë ‹²Q®ï~@°“ ã`ŒQ®@O P„²#@Á~ò`Í

J‹¬Oƒ=
’ O. Hê=ô# J`Ç#° …샏O
’ ‡ÚO^ΰ`Œ_Ë …è^¥ #+¬O
“ J`Çx †³òH›ø |~¡°=ô ̄~¡Q®^ΰ.

‡éO^Î`Œ_Ë =°#䛽 `³e†Ç°^ΰ. Hêx ï~O_O\ ÷…Õ U^Ë ª^Î#: Wzó# ㄬH›@#…Õ 'Z䛽ø= =°OkÑ Q®°iOz K³„¬æ|_Ok.

Hêx 'JO^Î~¡¶Ñ J<Í ƒì=#…è^ΰ. Hê=ô# ㄬu XH›ø~¡¶


XH› T‚¬ì#O x[O J=ô`Ç°Ok. Jk U^Ë=¶ã`ÇO

`³e†Ç°^ΰ. Hê=ô# T‚¬ì#O I Qêh T‚¬ì#O II Qêh


Jx T‚²ìOKÇ䛀_È^ΰ. Hê=ô# T‚¬ì# I =°i†Çò T‚¬ì#

U^Ë XH›\ ÷. ‹¬=¶^¥#O (5). II ï~O_ȶ ‹¬i†³Ø°#q Hê=ô. ‹¬=¶^¥#O (4).

REASONING ABILITY (LOGICAL) ○ ○ ○ ○ ○ ○ ○ ○


155
○ ○ ○ ○ ○ ○ ○ ○ ○ ○ ○ ○ ○ ○ ○ ○ ○ ○ ○
Study Material
1) 2) II,III
MODEL - 2 Jx߆Çü ‹¬i†³Ø°#"Í =¶ã`Ç"Í° ‹¬i†³Ø°#q

3) I =¶ã`Ç"Í° ‹¬i†³Ø°#q 4) I, II =¶ã`Ç"Í° ‹¬i†³Ø°#q

G ㄬH@
› # ãH÷O^Î ~
ï O_È°H›<Œß Z䛽ø= T‚¬ì#° Wzó#„¬ô_È° 5) UnHê^ΰ.

Wzó# ㄬ‰×ß ãH÷O^Î P„¬Â#°#° „¬ijeOz ‹¬i†³Ø°# ª^Î#: †Ç°[=¶x Hê~¡¼ãH›=°O ‹¬iQê [~¡Qê<Í HË~¡°ä›½O\ì_È°.

‹¬=¶^¥#O ZOKÇ°HË"Œe.
Hê=ô# T‚¬ì#- I ㄬH›@##° `Ç$„²ë„¬~¡°‹¬°ëOk. T‚¬ì#-

1. ㄬH›@# : H›^ΰ°`Ç°#ß ƒ‹¬°ž †³òH›ø „¦¬ô\˜‡`Ÿ =^Îí II =°i†Çò T‚¬ì#- III ° ㄬH›@#`Ë ‹¬O|O^ÎO …èx

ㄬ † Ç ¶ }÷ ä › ½ ° Z=~¡ ¶ x|_È ~ Œ^Î ° - |‹¬ ° ž T‚¬ì°. Hê=ô# ‹¬=¶^¥#O (3).

ZöHø@„¬C_È° H›x„²OKÍ <Ë\ ©‹¬°.


3. ㄬH›@# : WO_†°
Ç <£ ãH÷H
ï \˜ \ "
© °
£ P@f~¡° ㄬ‹°
¬ `
ë O
Ç J‹¬ž°

T‚¬ì#° : I) ãH÷O^΄¬_‡é†Í° ㄬ=¶^ÎO =ô#ßk.


ƒìQê…è^ΰ.

II) |‹¬°ž ZöHø ㄬ†Ç¶}÷䛽ä›½ JªÏH›~¡¼O


T‚¬ì#° : I) WO_†Ç¶ ãH÷ïH\˜ \ ©"£° ~¡^ΰíK͆ǰ|_È°`Ç°Ok

Y
III) „¦¬ô\˜‡`Ÿ g°^Î #°OKÇ°#ß"Œ~¡° \ ÷ïH\˜
II) WH› #°O_ WO_†Ç¶ P_Í ã„¬u=¶¼KŸ F_

M
f‹¬°HË#=‹¬~¡O …è^ΰ.
‡é`Ç°Ok.

1) Jx߆Çü ‹¬i†³Ø°#"Í
III) P@Qêˆ×¤ =¼H÷ëQ®`Ç q+¬†Ç¶…è ㄬ‹¬°ë`ÇO \O
©

E
2) II,III =¶ã`Ç"Í° ‹¬i†³Ø°#q
WO_†Ç¶ ^ΰ~¡Ä~¡ ‹²ÖuH÷ Hê~¡}O

3) I,II

D
9
=¶ã`Ç"Í° ‹¬i†³Ø°#q
1) I,II,III

59
° ‹¬i†³Ø°#"Í

4) I, III 2) I
=¶ã`Ç"Í° ‹¬i†³Ø°#q

CA
95
=¶ã`Ç"Í° ‹¬i†³Ø°#k

5) Un ‹¬i†³Ø°#k Hê^ΰ.
3) II

50
=¶ã`Ç"Í° ‹¬i†³Ø°#k.

ª^Î#: T‚¬ì#- I =°i†Çò T‚¬ì#- II ° ï~O_ȶ ㄬH›@##°


4) I, II,III
98
A
‹¬i†³Ø°#q Hê=ô

`Ç$„²„
ë ~
¬ °
¡ ªë~ò. T‚¬ì#- III `DŽC
¬ .|‹¬°ž ZH÷ø# ㄬuXH›ø~¡¶
5) III
:8
=¶ã`Ç"Í° ‹¬i†³Ø°#k

(3).
E

\ ÷ïHø@°“ f‹¬°HË=‹²O^Í. Hê=ô# ‹¬=¶^¥#O


ª^Î#:
h

Wzó# =ü_È° T‚¬ì#¶ ㄬH›@#`Ë ‹¬O|O^ÎO…èx"Í.


.P

2. ㄬH›@# :
IC

''Hê~¡¼ãH›=°O ‹¬iQê [~¡Q®H› ‡é`Í h䛽 _È|°Ä P@f~¡° ƒìQËH› F_‡é~ò#O`Ç =¶ã`Œ# U^͉×O †³òH›ø
AD

W=Þ#°ÑÑ- P~¡<
¾ [
á³ ~Ÿ`Ë †Ç°[=¶x J#ß =¶@°. \ ©"£° ~¡^ΰí K͆ǰ|_È^ΰ. P@#° "³°~¡°Q®°„¬~¡°KÇ°Hùx =°Oz

T‚¬ì#° : I)
AB
O

Hê~¡¼ãH›=°O ‹¬*ì=ôQê [~¡Qêx ㄬ^Î~¡Å##° Wªë~¡°. ㄬ‹¬°ë`ÇO P@…Õ F_‡é~ò#O`Ç


ER
CH

HË~¡°ä›½O@°<Œß~¡°. =¶ã`Œ# ƒ’q+¬¼`Ç°ë…Õ 䛀_¨ F_‡é`Œ~¡x T‚²ìOKDžè=ò.

II)
YD

_È|°ÄZQ®Qù>è“ P…ÕKÇ#`Ë =ô<Œß~¡° ㄬH›@#…Õ P@Qêˆ×¤ =¼H÷ëQ®`Ç ‹¬=°‹¬¼ ㄬªë=<Í …è^ΰ.

III) P~¡¾<³á[~Ÿ =°i†Çò †Ç°[=¶x ‰×ã`Ç°=ô° (4).


,H

Hê=ô# ‹¬=¶^¥#O

123456789012345678901234567890121234567890123456789012345678901212
12345678901234567890123456789012123456789012345678901234567890121234567890123456789012345678901212345678901234567890123
1234567890123456789012345678901212345678901234567890123456789012123456789012345678901234567890121234567890123
1234567890123456789012345678901212345678901234567890123456789012123456789012345678901234567890121234
12345678901234567890123456789012123456789012345678901234567890121234567890123456789012345678901212345678901234567890123456789012123
1234567890123456789012345678901212345678901234567890123456789012123456789012345678901234567890
123456789012345678901234567890121234567890123456789012345678901212
T

12345678901234567890123456789012123456789012345678901234567890121234567890123456789012345678901212345678901234567890123
1234567890123456789012345678901212345678901234567890123456789012123456789012345678901234567890121234567890123
1234567890123456789012345678901212345678901234567890123456789012123456789012345678901234567890121234
12345678901234567890123456789012123456789012345678901234567890121234567890123456789012345678901212345678901234567890123456789012123
1234567890123456789012345678901212345678901234567890123456789012123456789012345678901234567890
AM

123456789012345678901234567890121234567890123456789012345678901212
12345678901234567890123456789012123456789012345678901234567890121234567890123456789012345678901212345678901234567890123
1234567890123456789012345678901212345678901234567890123456789012123456789012345678901234567890121234567890123
1234567890123456789012345678901212345678901234567890123456789012123456789012345678901234567890121234
12345678901234567890123456789012123456789012345678901234567890121234567890123456789012345678901212345678901234567890123456789012123
1234567890123456789012345678901212345678901234567890123456789012123456789012345678901234567890
PRACTICE TEST - 1
12345678901234567890123456789012123456789012345678901234567890121234567890123456789012345678901212345678901234567890123456789012123
1234567890123456789012345678901212345678901234567890123456789012123456789012345678901234567890
12345678901234567890123456789012123456789012345678901234567890121234567890123456789012345678901212345678901234567890123
1234567890123456789012345678901212345678901234567890123456789012123456789012345678901234567890121234567890123
1234567890123456789012345678901212345678901234567890123456789012123456789012345678901234567890121234
123456789012345678901234567890121234567890123456789012345678901212
12345678901234567890123456789012123456789012345678901234567890121234567890123456789012345678901212345678901234567890123456789012123
1234567890123456789012345678901212345678901234567890123456789012123456789012345678901234567890
12345678901234567890123456789012123456789012345678901234567890121234567890123456789012345678901212345678901234567890123
1234567890123456789012345678901212345678901234567890123456789012123456789012345678901234567890121234567890123
1234567890123456789012345678901212345678901234567890123456789012123456789012345678901234567890121234
123456789012345678901234567890121234567890123456789012345678901212
12345678901234567890123456789012123456789012345678901234567890121234567890123456789012345678901212345678901234567890123456789012123
1234567890123456789012345678901212345678901234567890123456789012123456789012345678901234567890
H

12345678901234567890123456789012123456789012345678901234567890121234567890123456789012345678901212345678901234567890123
1234567890123456789012345678901212345678901234567890123456789012123456789012345678901234567890121234567890123
1234567890123456789012345678901212345678901234567890123456789012123456789012345678901234567890121234
12345678901234567890123456789012123456789012345678901234567890121234567890123456789012345678901212345678901234567890123456789012123
1234567890123456789012345678901212345678901234567890123456789012123456789012345678901234567890
12345678901234567890123456789012123456789012345678901234567890121234567890123456789012345678901212345678901234567890123
1234567890123456789012345678901212345678901234567890123456789012123456789012345678901234567890121234567890123
1234567890123456789012345678901212345678901234567890123456789012123456789012345678901234567890121234
12345678901234567890123456789012123456789012345678901234567890121234567890123456789012345678901212345678901234567890123456789012123
1234567890123456789012345678901212345678901234567890123456789012123456789012345678901234567890
M

‹¬¶KÇ# (1-20) :D ãH÷Ok W=Þ|_# ㄬ‰×߁…Õ XH› ㄬH›@#


AM
IG

II) ã̄‹²_³O\˜ J"Œ¼O>è Hê~ŒQê~¡"Œ‹¬O XH›

=°i†Çò ~ï O_È° T‚¬ì#° W=Þ|_È`Œ~ò."Œ\÷…Õ T‚¬ì#


KH

J~¡Ý`Ç.

ã„H¬ @› ##° `Ç$„²„ë i¬ À‹ë ‹¬=¶^¥#O ( ) Qê#¶, T‚¬ì# 2. ㄬH›@# :


R

I 1 <Í # ° ä› ½ ¶=°<Œe…Õ =ô#߄¬ C _È ° hä› ½ ‡¦ é <£

ㄬH›@##° `Ç$„²ë„¬iÀ‹ë ‹¬=¶^¥#O ( ) Qê#¶, ï~O_È°


II 2 K͆ǰ…è#°.

T‚¬ì#¶ ã„H¬ @› ##° `Ç$„²„ë i¬ À‹ë ‹¬=¶^¥#O ( ) Qê#¶, T‚¬ì#° : 3 I) H›¶=°<Œe…Õ ‡¦é<£ ªÏH›~¼
¡ O =ôO_=ôO_È^°
Î .

ï~O_O\÷…Õ XH›ø\© ㄬH›@##° `Ç$„²ë„¬~¡KÇH› ‡é`Í II) †Ç¶ã`ǁ䛽 "³o¤ ZO*솟° KÍÀ‹ ‹¬=°†Ç°O…Õ

‹¬=¶^¥#O ( ) Qê#¶, =°i†Çò T‚¬ì# …è^¥ T‚¬ì#


4 I Z=~¡¶ ‡¦é<£ K³†Ç°¼~¡°.

…Õ U^³á<Œ XH›\÷ ㄬH›@##° `Ç$„²ë„¬iÀ‹ë ‹¬=¶^¥#O 3. ㄬH›@# :


II J<£ Z ~ò_³ _ £ H› ˆ 쉧…Õ „¦ Ô [  ̄O„¬ ô ^Î  ä› ½

( ) Qê#¶ Q®°iëOKŒe.
5 ㄬƒ’°`ÇÞO J#°=°u WzóOk.

1. ㄬH›@# : 27 Uˆ×¤ Hê~ŒQê~¡"Œ‹¬O <³ž<£ =°O_ͅì#°


T‚¬ì#° : I) J<£Z~ò_³_£ H›ˆì‰§° PiÖOH›OQê #‘“…ÕÁ

=ô<Œß~ò.
ã̄‹²_³O\˜#° K͋²Ok.

T‚¬ì#° : I) 27 Uˆ× à ¤ Hê~ŒQê~¡ O …Õ L#ß Z=ï ~ á < Œ II) Z~ò_³_£ H›ˆì‰§…ÕÁ „¦ÔA° ̄OKŒež#

ã̄‹²_³O\˜ J=ô`Œ~¡°. J=‹¬~¡O …è^ΰ.

REASONING ABILITY (LOGICAL) ○ ○ ○ ○ ○ ○ ○ ○


156
○ ○ ○ ○ ○ ○ ○ ○ ○ ○ ○ ○ ○ ○ ○ ○ ○ ○ ○
Study Material
4. ㄬH›@# : ^͉×O…Õ 18 ‹¬O=`Ǟ~Œ ̄á|_# x~¡°^˼Q®° 11. ㄬH›@# : A J#°"Œ_È° B `Ë K³Ì„æ#°. ''Hêjà~Ÿä½
› J=°$`Nj~
¬ Ÿ

O^ÎiH© x~¡^˼Q® ƒ’$ux W"ŒÞe. g°^ΰQê "³ˆ×ä ^ÎQ®¾i=¶~¡¾OÑÑ.

T‚¬ì#° : I) ^͉×O…Õx KŒ…ì =°Ok x~¡°^˼Q®°° T‚¬ì#° : I) B Hêjà~Ÿä›½ "³ˆì¤#°ä›½O@°<Œß_È°.

À„^Î"Œö~. "ŒiH÷ PiÖH› ‹¬‚¬ð†Ç°O II) A JO^ÎiH÷ Lz`Ç ‹¬‚¬ð° JOkªë_È°.

K͆ǰ=‹²# J=‹¬~¡O =ô#ßk. 12. ㄬH›@# : ‡ÚQ®ã`ŒQ®_ÈO P~ËQê¼xH÷ ‚¬ðxH›~¡O

II) x~¡ ° ^˼Q® †Ç ò =ä› ½ ä› ½ Y~¡ ° ó̄@“ _ ¨xH÷ T‚¬ì#° : I) ‡ÚQ®ã`ŒQ®H›‡é=_ÈO =# P~ËQ®¼O

ㄬ ƒ  ’ ° `Ç Þ O ^Î Q ® ¾ ~ ¡ ‹¬ i ‡é†Í ° @O`Ç x^ Î ° ° "³°~¡°Q®=ô`Ç°Ok

=ô<Œß~ò. II) x*ìxH÷, D ̂ìKÇóiH› J=‹¬~¡O.

5. ㄬH›@# : '‡`Çï~Q®°¼…è@~Ÿ ªÖ#O…Õ ãHù`Çë Z¢Hê“xH± 13. ㄬH›@# : Hêià䛽O^Î~¡¶ ‡¦¼H›“s…ÕxH÷ H›zóO`ÇOQê

ï~Q®°¼…è@~Ÿ J=°ió 30 ‰§`ÇO q^ΰ¼`Ÿ P^¥ 8 : 30 A.M H›…ìÁ ~Œ"Œe.

Y
K͆ǰO_Ñ- XH› ㄬH›@# T‚¬ì#° : I) HùO`Ç=°Ok Hêià䛽° ‡¦¼H›“sH÷ P‹¬¼OQê

M
T‚¬ì#° : I) ƒ’q+¬¼`Ç°ë…Õ q^ΰ¼`Ÿ KŒsû° ƒìsQê ̄iöQ ~Œ=KÇ°ó.

J=Hê‰×O LOk. II) Wzó# <Ë\ ‹


© °
¬ #° Hêià䛽° J#°‹¬iOKŒe.

E
II) q^ΰ¼`Ç°#
ë ° P^¥KÍÀ‹ „¬iH›~Œ° "Œ_#@Á~ò`Í 14. ㄬH›@# : |@“#° ‰×ÃハO
’ K͆°
Ç _¨xH÷ _@~
ï O
û @°Á L„¬†¶
³ Q®

D
9
59
H›ï~O@° a°Á `Ç䛽ø= =‹¬°ëOk. „¬_È`Œ~ò.

6. ㄬH›@# :
CA
T‚¬ì#° : I)

95
'g°_È|°Ä#° 5 <³…ÕÁ ï~\ ÷“O„¬ô K͋¬°HËO_Ñ Ð _@ï~ûO@°Á Z䛽ø= #°~¡°Q®° Wªë~ò.

50
XH› ㄬH›@# II) _@ï~ûO@°Á ãwA =°i†Çò =òiH÷x

T‚¬ì#° : I)
98
A
ㄬH›@# x["³°Ø#k Hê^ΰ =keªë~ò.

15. ㄬH›@# :
:8
II) ª^¥~¡}OQê ㄬ[° "Œi ‹¬O‡^Î# ㄬu XH›ø~¡¶ ª‚¬ì‹¬ Qê^ŠÎ° KÇ^Î"Œ<Œß W+¬“
E
h

̄OKÇ°HË=_¨xöH W+¬“„¬_È`Œ~¡°. „¬_È`Œ~¡°.


.P
IC

7. ㄬH›@# : ^Ë=°° Wˆ×¤…ÕxH÷ ~Œä›½O_¨ H÷\ ÷H©ä›½ "³°+¹° T‚¬ì#° : I) KÇk"ÍO^ΰ䛽 ª‚¬ì‹¬Qê^Î° =¶ã`Ç"Í°
AD

U~Œæ@° K͋¬°HË"Œe. JO^ΰƒì@°…Õ =ô<Œß~ò.


AB
O

T‚¬ì#° : I) ^Ë=°° Wˆ×¤…ÕxH÷ ~Œ=_¨xH÷ H÷\ ÷H©H›O>è II) "Í~


ö „¬ô‹¬H
ë ꁰ KÇ^=
Î @O Z=ÞiH© W+¬O
“ …è^°
Î .
ER
CH

"Íö~ =¶~Œ¾° `Çy#q. 16. ㄬH›@# : h=ô XH› W°Á H ù<Œx J#°Hù#߄¬ C _È ° =¶
YD

II) "³°+¹° `ǁ°„¬ôä›½ ayOKDžè=ò. U*ÿO\˜ g°ä›½ ‹¬‚¬ð†Ç°O Kͪë_È°.

8. ㄬH›@# : T‚¬ì#° : I)
,H

‡é‹¬“…˜ ‰§Y"Œ~¡° …Õ@° „¬î_Èó_¨xH÷ ö~@°Á g°~¡° ‹¬Þ†Ç°OQê W°Á Hù#_¨xH÷ J#~¡°Ý°
T
AM

̄OKŒ~¡°. II) WO\ ÷x Hù#_ÈO…Õ =¶ U*ÿO\˜ x„¬ô}°_È°.


H

T‚¬ì#° : I) 17. ㄬH›@# :


M

ㄬ‹¬°ë`ÇO ^Î~¡° KŒ…ì `Ç䛽ø= D „¬ ô ‹¬ ë H › O ZO`Ç Qù„¬ æ Qê =ôO^Î O >è ''XH›


AM
IG

II) ^Î~¡° ̄OKÇH›‡é`Í "Œ~¡° Hù~¡`Ç#° KÇ^ΰ=ô~Œx "Œ_È° 䛀_¨ \ ©KÇ~Ÿ …è䛽O_¨<Í D
KH

JkQ®q°OKÇ …è~¡°. „¬ô‹¬ëH›O KÇkq J~¡ÖO K͋¬°HËQ®_È°ÑÑ.

9. ㄬH›@# : T‚¬ì#° : I)
R

q[†Ç°O ªkOz# `Ç~°


¡ "Œ`Ç ä›€_¨ q#†Ç°OQê<Í KÇ^ΰ=ô~Œx "Œ_È° \ ©KÇ~Ÿ …è䛽O_¨ q^μ#°

=ôO_¨e. <Í~¡óHË"Œ#°ä›½O\ì_È°.

T‚¬ì#° : I) ª^¥~¡}OQê ㄬ[O^Î~¶


¡ U q+¬†°
Ç O…Õ<³<
á Œ II) D „¬ ô ‹¬ ë H › O \ ©KÇ~Ÿä›½ |^Î ° °Qê =°Oz

q[†Ç°O ªkOz<Œ q#†Ç°OQê<Í =ôO\ì~¡°. ㄬ`Œ¼=¶ß†Ç°O

II) ª^¥~¡}OQê ㄬ[O^Î~¡¶ Q®iÞ+¬µ»°. 18. ㄬH›@# : U^³á<Œ ̄@°“|_È°Ì„á x~¡â†Ç°O f‹¬°ä›½<Í =òO^ΰ

10. ㄬH›@# : ㄬ [ ° q<Ë^Î O Hˋ¬ O ‹² x =¶° KÇ ¶ _È @ O ##°ß ‹¬OㄬkOKÇO_.

Jx"Œ~¡¼O. T‚¬ì#° : I) XH›"Íˆ× g°~¡° ##°ß ‹¬OㄬkOKÇH›‡é`Í

T‚¬ì#° : I) q<Ë^ÎO Hù~¡ä›½ ‹²x=¶° `DŽ¬æ "Íö~ =¶~¡¾O `DŽC


¬ _È° x~¡†
â °
Ç O f‹¬°ä›½<Í J=Hê‰×O =ôOk.

…è^ΰ. II) ̄@°“|_È°Ì„á ‹¬ï~á# x~¡â†Ç°O f‹¬°HË=@O

II) ㄬ[° ‹²x=¶#° W+¬“„¬_È`Œ~¡°. KŒ…ì =òY¼O

REASONING ABILITY (LOGICAL) ○ ○ ○ ○ ○ ○ ○ ○


157
○ ○ ○ ○ ○ ○ ○ ○ ○ ○ ○ ○ ○ ○ ○ ○ ○ ○ ○
Study Material
19. ㄬH›@# : ㄬ‹¬°ë`Ç q^¥¼ =¼=‹¬Ö#° ‹¬=°ãQ® „¬ij# 20. ㄬH›@# : ''h=ô ~ŒãuH÷ ~Œã`Í L#ß`Ç ªÖ~òH÷ KÍ~¡…è=ôÑÑ

K͆Ƕež# J=‹¬~¡O ZO`³á<Œ =ô#ßk. T‚¬ì#° :I) ㄬ[° L#ß`ǪÖ~òx KÍ~¡°HË"Œx H›°

T‚¬ì#° : I) ㄬ‹°
¬ `
ë Ç q^¥¼=¼=‹¬Ö ㄬ†¶
³ Qê`ÇàH›OQê …è^°
Î . H›O\ì~¡°.

II) =¶~¡°æ Ja=$kœH÷ Hê~¡}O II) L#ß`ǪÖ~òx KÍ~¡_¨xH÷ Q®\ ÷“ ㄬ†Ç°`ÇßO

K͆Ƕe.

1234567890123456789012
12345678901234567890123456789012123456789012345678901234567890121234567890123456789012345678901212345678901234567890123
12345678901234567890123
12345678901234567890123456789012123456789012345678901234567890121234567890123456789012345678901212345678901234567890123
1234567890123456789012 12345678901234567890123
12345678901234567890123456789012123456789012345678901234567890121234567890123456789012345678901212345678901234567890123
1234567890123456789012 12345678901234567890123
12345678901234567890123456789012123456789012345678901234567890121234567890123456789012345678901212345678901234567890123
1234567890123456789012
PRACTICE TEST - 1 :
12345678901234567890123
12345678901234567890123456789012123456789012345678901234567890121234567890123456789012345678901212345678901234567890123
1234567890123456789012
12345678901234567890123456789012123456789012345678901234567890121234567890123456789012345678901212345678901234567890123
1234567890123456789012
12345678901234567890123
12345678901234567890123
12345678901234567890123456789012123456789012345678901234567890121234567890123456789012345678901212345678901234567890123
1234567890123456789012 12345678901234567890123
q=~¡}ì`ÇàH› ["Œ|°°
1. (4) ^Í ‰ × ªÞ`Ç O ã`Ç ¼ O ª^ Î # Hˋ¬ O <³  ž<£ =°O_Í … ì H÷\ ÷H© Q®°O_¨ ^Ë=°° WO\ ÷…ÕxH÷ ~Œä›½O_¨ *ìãQ®`Çë

Hê~ŒQê~¡O…Õ =ô<Œßö~ `DŽ¬æ P†Ç°# Z…ìO\ ÷ <Í~¡O „¬_=


È °x ㄬH@
› # ª~ŒO‰×O. U =¶~Œ¾ ^¥Þ~Œ ^Ë=°°

Y
KÍ † Ç ° …è ^ Î ° . =°i†Ç ò ã̄‹² _ ³ O \˜ J"ŒÞ<Í P†Ç ° # WO\ ÷…ÕxH÷ ㄬ"ÍtOz<Œ "Œ\ ÷=Á H›eöQ #‘“° XH›>è.

I II

M
ㄬ`ͼH›OQê *ÿၰH÷ "³ˆ×¤…è^ΰ. P†Ç°# ^͉×OHˋ¬O K͋²# T‚¬ ì #- =°i†Ç ò T‚¬ ì # ï ~ O_È ¶ ㄬ H › @ ##°

`Ç$„²ë„¬~¡KDžè=ô.
`Œ¼Qꁰ, J#°ƒq
’ Oz# H›‘“°, JOH÷`ƒ
Ç ì
 =O =°i†Çò

E
^͉ƒ
× H
’ ë÷ P†Ç°##° ã̄‹²_O
³ \˜ J†Í°¼…ì Kͪ~ò.T‚¬ì#-
8. (2) ㄬ‹¬°ë`ÇO Z…ìO\ ÷ ^Î~¡° =ô<Œß Hù~¡`Ç …è^¥ …Õ@°

D
9
I II

59
…è^¥ #‘“…Õ =ô#߄¬ô_Í ^Î~¡° ̄OKŒ#ß P…ÕKÇ#
=°i†Çò T‚¬ì#- ~
ï O_ȶ ㄬH@
› ##° `Ç$„²„
ë ~
¬ K
¡ …
Ç =
è ô.

I II
CA
95
2. (1) U^³á<Œ ㄬ^͉×O…Õ ‡¦é<£ ªÏH›~¡¼O …è#„¬ô_Í JH›ø_ #°O_
=‹¬°ëOk. Hê=ô# T‚¬ì#- ‹¬ï~á#k Hê^ΰ. T‚¬ì#

50
ㄬH›@##° `Ç$„²ë „¬~¡°‹¬°ëOk.
"Œi |O^°
Î =ôä›½ …è^¥ À‹ß‚²ì`Ç°ä›½ …è^¥ `³e‹²# "ŒiH÷

(4) I
98
A
9. T‚¬ì# JOwH›iOKÇ^ÎQ®¾k Hê^ΰ. Z=i ‹¬Þƒì=O
ZO`Ç J=‹¬~¡"³°Ø<Œ ‡¦é<£ K͆ǰ_ÈO 䛽^Î~¡^ΰ. =°i†Çò

II
:8
Z…ìO\ ÷^Ë =°#O T‚²ìOKDžè=ò. T‚¬ì#-
^ζ~¡ã‡O`Œä›½ "³o¤#„¬ô_Í öHÆ=°‹¬=¶KŒ~¡O
E

I
h

J~¡~
Ö ‚
¡ ì
² `ÇO. Hê=ô# Wzó# ~
ï O_È° T‚¬ì#¶ ㄬH@
› ##°
`³e†Ç°*è†Ç°_¨xH÷ `Ç~¡°KÇ°Qê ‡¦é<£ Kͪë~¡°. T‚¬ì#-
.P
IC

ㄬH›@##° `Ç$„²ë „¬~¡°‹¬°ëOk. T‚¬ì#- II ‹¬i†³Ø°#


`Ç$„²ë „¬~¡KDžè=ô.
AD

10. (2) q<Ë^ÎO Hù~¡ä½


› J<ÍH› =¶~Œ¾°<Œß~ò. Hêx JxßO\ …
÷ ÕH©
T‚¬ìHê^ΰ.
AB
O

ª^¥~¡}OQê ㄬ[° ‹²x=¶ ^¥Þ~Œ =KÍó q<Ë^¥xß


3. (1) PiÖH›OQê #‘“…ÕÁ =ô#߄¬ô_Í „¦ÔA° ̄O„¬ô Q®°iOz

I
ER
CH

W+¬“„¬_È`Œ~¡°. Hê=ô# T‚¬ì# Wzó# ㄬH›@##° `Ç$„²ë


P…Õzªë~°
¡ . "Œ~¡° „¦A
Ô ° ̄OKÇH‡
› é`Í ‹¬O‹¬
Ö x~¡Þ‚¬ì}

II
YD

„¬iKÍ@@°Á …è^°
Î . T‚¬ì#- ㄬH@
› ##° `Ç$„²ë „¬~°
¡ ‹¬°O
ë k.
ª^Î¼O Hê^ΰ. Hê=ô# T‚¬ì#- I ㄬH›@##° `Ç$„²ë

(1) B A
,H

11. J#°"Œ_È° Hêjà~Ÿä›½ "³ˆ×Ã`Ç°#ß@°Á `Ë K³„²æ#„¬ô_Í


„¬~¡°‹¬°ëOk.

A,B II
T

H÷ P ‹¬‚¬ð Wªë_È°. Hê=ô# T‚¬ì# Wzó#


AM

4. (1) ㄬƒ’°`ÇÞO =^Îí Z䛽ø= x^Î°° xÞ=ô<Œß J=‹¬~¡O

I
H

ㄬH›@##° `Ç$„²ë„¬iKÍ@°Á …è^ΰ. T‚¬ì# ㄬH›@##°


M

…è ä › ½ O_¨ "Œ\ ÷ x Y~¡ ° óKÍ † Ç ° ~¡ ° . À„^Î " ŒiH÷ PiÖ H ›


AM

`Ç$„²ë „¬~¡°‹¬°ëOk.

I
IG

‹¬‚¬ð†Ç°O JOkKÇ_ÈO ㄬƒ’°`ÇÞ qk. Hê=ô# T‚¬ì#-

12. (2) ‡ÚQ®ã`ŒQ®@O =# P~ËQ®¼O ‡_È=ô`Ç°O^Îx K³‡æö~


KH

ㄬH›@##° `Ç$„²ë „¬~¡°‹¬°ëOk.

Hêx ‡ÚQ®ã`ŒQ®H›‡é=_ÈO =# P~ËQ®¼O


R

5. (2) ƒq
’ +¬¼`Ç°…
ë Õ q^ΰ¼`Ÿ KŒsû ̄~¡°Q®°^΁ …è^¥ `Ç°Q®^
¾ 
Î #°
"³ ° ~¡ ° Q® = ô`Ç ° O^Î x ^¥x J~¡ Ö O Hê^Î ° . D ̂ìKÇ ó iH›

=òO^ΰQê<Í T‚²ìOKDžè=ò. Hêx q^ΰ¼`Ÿ#° P^¥KÍÀ‹


Yzó`ÇOQê J=‹¬~¡O. ZO^ΰH›O>è ‡ÚQ®ã`ŒQ®_ÈO =#

„¬ i H› ~ Œ#° "Œ_ ` Í q^Î ° ¼`Ÿ qx†³ ¶ Q® O `Ç y ¾ a°Á


Jk ‰× s ~¡ O …Õx JO`Ç ~ ¡ O …ÕxH÷ ‡é~ò Hê#ž~Ÿ # °

II
`Ç䛽ø=Qê =‹¬°ëO^Îx =¶ã`ÇO T‚²ìOKÇ=KÇ°ó#°. Hê=ô#
H› e y‹¬ ° ë O k. Jk ㇐}ìO`Ç H › O . ㄬ H › @ # `Ç $ „² ë

T‚¬ì#- II ㄬH›@##° `Ç$„²ë„¬~¡°‹¬°ëOk.


„¬~¡°‹¬°ëOk.

6. (2) ㄬH@
› # x["³¶ Hê^Ë T‚²ìOKDž=
è ò. Hêx ㄬ[O^Î~¶
¡
13. (2) XH› ‹¬O‹¬Ö…Õ Wzó# <Ë\ ©‹¬° JO^Î~¡¶ ‡\ ÷OKŒežO^Í.

"Œi P^¥†Ç°O ̄~¡Qê<Í HË~¡°ä›½O\ì~¡°. T‚¬ì# -I Hê=ô# T‚¬ì#- I `DŽ¬C. T‚¬ì#- II ‹¬ï~á#^Í.

`DŽ¬C. T‚¬ì# - II ㄬH›@##° `Ç$„²ë„¬~¡°‹¬°ëOk.


14. (2) #°~¡°Q®° Z䛽ø=Qê Wzó |@“ =òiH÷x ‰×Ãハ’O K͆ǰx

(4)
7. ª^ ¥ ~¡ } OQê JO^Î ~ ¡ ¶ QêeHˋ¬ O H÷ \ ÷ H ©  ° `³ i KÍ
_@ï~ûO@°Á Z=~¡¶ L„¬†³¶yOKÇ~¡°. Hê=ô# T‚¬ì# II
=ôOKÇ°`Œ~¡°. `ǁ°„¬ô° ZÁ„¬ô_ȶ `³iz =ôOKÇ~¡°. P
‹¬ï~á#^Í.

REASONING ABILITY (LOGICAL) ○ ○ ○ ○ ○ ○ ○ ○


158
○ ○ ○ ○ ○ ○ ○ ○ ○ ○ ○ ○ ○ ○ ○ ○ ○ ○ ○
Study Material
15. (4) T‚¬ì# I T‚¬ì# II ï~O_ȶ Wzó# ㄬH›@##° `Ç$„²ë =‹¬°ëOk ? #+¬“O =‹¬°ëO^¥ ? J#ßk ƒèsA "͋¬°ä›½x

„¬~K
¡ @
Í °Á …è=ô. WH›ø_È =¼ä›½
ë Ja~°
¡ z Q®°iOz =¶ã`Ç"°
Í J_È°Q®° =òO^ΰ䛽 "͆Ƕe. Z=ih ‹¬OㄬkOKÇ䛽O_¨

ㄬªëqOKÇ|_#k.
‹¬ÞO`Ç x~¡â†Ç°O`Ë Ì„@°“|_ ̄_`Í #‘“#°

(2)
16. XH› ~¡OQ®O…Õ J#°ƒ’=O =ô#ß"Œi ‹¬‚¬ð° KŒ…ì
KÇqKǶ_¨ež =‹¬°O
ë k. Hê=ô# T‚¬ì# I =°i†Çò T‚¬ì#

q°"³#
á q. Hê=ô# T‚¬ì# II ㄬH@
› ##° `Ç$„²„
ë ~
¬ °
¡ ‹¬°O
ë k.
II ï~O_ȶ Wzó# ㄬH›@##° `Ç$„²ë„¬~¡°ªë~ò.

ㄬH›@#…Õ ZH›ø_¨ =¼H÷ëx J#~¡°Ý_Qê K³„²æ#@°Á …è^ΰ.


19. (1) Wzó# ㄬH›@#…Õ q=°~¡Å<Œ`ÇàH› ^Ë~¡}÷ =ô#ßk. ㄬ‹¬°ë`Ç

17. (2) Wzó# ㄬH›@#…Õ „¬ô‹¬ëH›O †³òH›ø Qù„¬æ`Ç#O Q®°iOz


q^¥¼=¼=‹¬Ö…Õ …Õ‡° =ô#ß@°Á ƒìqOKÇ_ÈO [iy#k.

`³e†Ç°*艧~¡°. Jk ZO`ÇQù„¬æQê =ôO@°O^ÎO>è

Hê=ô# T‚¬ì#- I ㄬH›@##° `Ç$„²ë„¬~¡°‹¬°ëOk.

KÇ ^ Î ° =ô~Œx "Œ_È ° ä› € _¨ ^¥xx \ ©KÇ~Ÿ ‹¬ ‚ ¬ ð †Ç ° O

20. (2) ㄬ†Ç°`ÇßO …è䛽O_¨ Z=Þ~¡¶ L#ß`Nj²Öux KÍ~¡°H˅è~¡°.

…è䛽O_¨<Í KÇkq J~¡ÖOK͋¬°HËQ®_È°. Hê=ô# T‚¬ì#

Y
II I
Jk XH› ~ËA…Õ<Ë …è^¥ XH› „¬î@…Õ<Ë ª^¼
Î =°†Í°¼k
ㄬH›@##° `Ç$„²ë„¬~¡°‹¬°ëOk. T‚¬ì# ‹¬iHê^ΰ.

Hê^Îx ㄬH›@# J~¡ÖO. ㄬH›@#…Õ XH› =¼H÷ë Q®°iOz

M
18. (3) ª^¥~¡}OQê U "Œ¼‡~¡O…Õ<³á<Œ ̄@“|_È°° ̄>è“

=òO^Î ° JO^Î ° …Õ J#°ƒ ’ = =ò#ß"Œi ‹¬  ‚¬ 𠁰


=¶ã`Ç"Í° ㄬªëqOKÇ@O [iyOk. Hê=ô# T‚¬ì#- II

E
ㄬH›@##° `Ç$„²ë„¬~¡°‹¬°ëOk.
f‹¬°HË"Œe. P "Œ¼‡~¡O…Õ ̄@°“|_ ̄_`Í …샏’O

D
9
123456789012345678901234567890121234567890123456789012345678901212
12345678901234567890123456789012123456789012345678901234567890121234567890123456789012345678901212345678901234567890123
1234567890123456789012345678901212345678901234567890123456789012123456789012345678901234567890121234567890123
1234567890123456789012345678901212345678901234567890123456789012123456789012345678901234567890121234
12345678901234567890123456789012123456789012345678901234567890121234567890123456789012345678901212345678901234567890123456789012123
1234567890123456789012345678901212345678901234567890123456789012123456789012345678901234567890
123456789012345678901234567890121234567890123456789012345678901212

59
12345678901234567890123456789012123456789012345678901234567890121234567890123456789012345678901212345678901234567890123
1234567890123456789012345678901212345678901234567890123456789012123456789012345678901234567890121234567890123
1234567890123456789012345678901212345678901234567890123456789012123456789012345678901234567890121234
12345678901234567890123456789012123456789012345678901234567890121234567890123456789012345678901212345678901234567890123456789012123
1234567890123456789012345678901212345678901234567890123456789012123456789012345678901234567890
123456789012345678901234567890121234567890123456789012345678901212
12345678901234567890123456789012123456789012345678901234567890121234567890123456789012345678901212345678901234567890123456789012123
1234567890123456789012345678901212345678901234567890123456789012123456789012345678901234567890
12345678901234567890123456789012123456789012345678901234567890121234567890123456789012345678901212345678901234567890123
1234567890123456789012345678901212345678901234567890123456789012123456789012345678901234567890121234567890123
1234567890123456789012345678901212345678901234567890123456789012123456789012345678901234567890121234
PRACTICE TEST - 2
12345678901234567890123456789012123456789012345678901234567890121234567890123456789012345678901212345678901234567890123456789012123
1234567890123456789012345678901212345678901234567890123456789012123456789012345678901234567890
12345678901234567890123456789012123456789012345678901234567890121234567890123456789012345678901212345678901234567890123
1234567890123456789012345678901212345678901234567890123456789012123456789012345678901234567890121234567890123
1234567890123456789012345678901212345678901234567890123456789012123456789012345678901234567890121234
123456789012345678901234567890121234567890123456789012345678901212
12345678901234567890123456789012123456789012345678901234567890121234567890123456789012345678901212345678901234567890123456789012123
1234567890123456789012345678901212345678901234567890123456789012123456789012345678901234567890
12345678901234567890123456789012123456789012345678901234567890121234567890123456789012345678901212345678901234567890123
1234567890123456789012345678901212345678901234567890123456789012123456789012345678901234567890121234567890123
1234567890123456789012345678901212345678901234567890123456789012123456789012345678901234567890121234
123456789012345678901234567890121234567890123456789012345678901212
CA
95
12345678901234567890123456789012123456789012345678901234567890121234567890123456789012345678901212345678901234567890123456789012123
1234567890123456789012345678901212345678901234567890123456789012123456789012345678901234567890
12345678901234567890123456789012123456789012345678901234567890121234567890123456789012345678901212345678901234567890123
1234567890123456789012345678901212345678901234567890123456789012123456789012345678901234567890121234567890123
1234567890123456789012345678901212345678901234567890123456789012123456789012345678901234567890121234
12345678901234567890123456789012123456789012345678901234567890121234567890123456789012345678901212345678901234567890123456789012123
1234567890123456789012345678901212345678901234567890123456789012123456789012345678901234567890
12345678901234567890123456789012123456789012345678901234567890121234567890123456789012345678901212345678901234567890123
1234567890123456789012345678901212345678901234567890123456789012123456789012345678901234567890121234567890123
1234567890123456789012345678901212345678901234567890123456789012123456789012345678901234567890121234
12345678901234567890123456789012123456789012345678901234567890121234567890123456789012345678901212345678901234567890123456789012123
1234567890123456789012345678901212345678901234567890123456789012123456789012345678901234567890

50
‹¬¶KÇ# (1-10) :D ãH÷Ok W=Þ|_# ㄬ‰×߁…Õ XH› ㄬH›@# 4. ㄬH›@# :
98
A
‰§OuH÷ =¼uö ~ H› O Qê J<Í H › ^Í ‰ §°#߄¬ æ \ ÷ H ©

=°i†Çò ~ï O_È° T‚¬ì#° W=Þ|_È`Œ~ò."Œ\÷…Õ T‚¬ì#


:8
†Çò^ÎíO#° fã=OQê =¼uö~H÷OKŒe.

ã„H¬ @› ##° `Ç$„²„ë i¬ À‹ë ‹¬=¶^¥#O ( ) Qê#¶, T‚¬ì# T‚¬ì#° : I)


E

I 1
h

†Çò^¥œ…Õ J=¶†Ç°H› ㄬ[° 䛀_¨

ㄬH›@##° `Ç$„²ë„¬iÀ‹ë ‹¬=¶^¥#O ( ) Qê#¶, ï~O_È°


.P
IC

II 2 KÇx‡é`Œ~¡°.

T‚¬ì#¶ ㄬH›@##° `Ç$„²ë„¬iÀ‹ë ‹¬=¶^¥#O ( )


AD

3 II) †Çò^ÎíO [iy#@Á~ò`Í =°#O F_‡é†Í°

Qê#¶, ï~O_O\÷…Õ U^Ë XH›ø\© ㄬH›@##° `Ç$„²ë„¬~¡KÇH›


AB
O

ㄬ=¶^ÎO 䛀_¨ =ô#ßk.

‡é`Í ‹¬=¶^¥#O ( ) Qê#¶, =°i†Çò T‚¬ì# …è^¥ 5. ㄬH›@# :


ER

''ã̄á"Í@° P‹²ë - JuãH›q°Oz#"Œ~¡° tH÷ÆO„¬


CH

4 I

T‚¬ì# …Õ U^³á<Œ XH›\÷ ㄬH›@##° `Ç$„²ë„¬iÀ‹ë |_È`Œ~¡°ÑÑ- XH› ‹¬ÖO =^Îí =ôOKÇ|_# <Ë\ ©‹¹
YD

II

‹¬=¶^¥#O ( ) Qê#¶ Q®°iëOKŒe.


5
T‚¬ì#° : I) P <Ë\ ‹
© ¹ KǶ‹²# "Œï~=~¡¶ P ‹¬
Ö O *ÕeH÷
,H

"³ˆ×¤~¡°.

1. ㄬH›@# :
T
AM

''‹¬O^Î~Å
¡ 䛽° e„¹#
“ ° "Œ_È=KÇóÑÑ- ㄬ=¶^¥ä›½

II) tH› Æ ä › ½ ƒ ’ † Ç ° „¬ _  ㄬ [ …ÿ = ~¡ ¶ P ‹¬ Ö … ìxß


H

ƒì^μ`Ç =¶kHê^ΰ.
M

PãH›q°OKÍ ª‚¬ì‹¬O K͆ǰ~¡°.

T‚¬ì#° : I)
AM

6. ㄬH›@# : V.I.P
e„¦¹“ "Œ_È@O ZÁ„¬C_ȶ ‹¬°~¡H÷Æ`ÇOHê^ΰ.
IG

 Hê~¡°Á ‡iøOQ· K͆ǰ=…‹²# ㄬ^͉×O…Õ

Hùxߪ~¡°Á ㄬ=¶^¥° [~¡Q®=KÇ°ó.


KH

‡~Ÿ ø KÍ ‹ ² # "Í ö ~ Hê~¡ Á # ° ã\섦 ² H ± ‡éb‹¬ ° °

II) ‹¬O^Î~Å
¡ 䛽…ÿ=~¡¶ e„¦"
“¹ Œ_È@Ōá JO`Ç P‹¬Hë÷
R

`ùyOKŒ~¡°.

KǶ„¬~¡°.
T‚¬ì#° : I) HùO`Ç = °Ok =¼ä› ½ ë  ä› ½ ㄬ ` Í ¼ H› ‡iøOQ·

2. ㄬH›@# : J`Ç#° ƒìQê H›+¬“„¬_È°`Ç°#߄¬æ\ ÷H© À„^Î"Œ_Í.


ㄬ^͉§° =ô<Œß~ò.

T‚¬ì#° : I) ƒìQê H›+¬“„¬_Í =°#‹¬ë`ÇÞO =ô<Œß À„^ÎiH›O…Õ


II) V.I.P ä› ½ =°i†Ç ò ª^ ¥ ~¡ } ㄬ [ ä› ½

=ô#ß"Œ~¡° KŒ…ì =°Ok =ô<Œß~¡°.


"Íö~Þ~¡° ‡iøOQ· ㄬ^͉§° Q®=ô.

II) ƒìQê |^ÎíH›‹¬°ë° 䛀_¨ ^Î#=O`Ç°°


7. ㄬH›@# : ‡Ú_È"³á# 䛀¼…Õ x|_È䛽O_¨ g° \ ÷ïH@Á#°

J=ô`Œ~¡°.
WO@ï~ß\˜…Õ |°H± K͋¬°HËO_.

3. ㄬH›@# : PiÖHO
› Qê Z^ΰQ®°^΁ ªkOKŒO>è ㄬ[O^Î~¶
¡
T‚¬ì#° : I) D ~ËA…ÕÁ KŒ…ì =°Ok ㄬ[° WO@ï~ß\˜…Õ

H›+¬“„¬_ „¬xK͆Ƕe. \÷H


ï ø@°Á |°H± K͋°
¬ HË=_¨xH© W+¬„
“ _
¬ °
È `Ç°<Œß~¡°.

T‚¬ì#° : I) PiÖH›OQê Z^ÎQ®@O JO^ÎiH© J=‹¬~¡O. II) WO@ï ~ ß\˜…Õ<Í Hêä› ½ O_¨ \ ÷ ï H ø@°Á W`Ç ~ ¡

II) H›+¬“„¬_ „¬xK͆ǰ_ÈO Jª^Î¼"Í°g° Hê^ΰ. ㄬ^͉§…Õ 䛀_¨ aªë~ò.

REASONING ABILITY (LOGICAL) ○ ○ ○ ○ ○ ○ ○ ○


159
○ ○ ○ ○ ○ ○ ○ ○ ○ ○ ○ ○ ○ ○ ○ ○ ○ ○ ○
Study Material
8. ㄬH›@# : ƒì~¡`^
Ç ‰
Í O
× …Õ ãHù`ÇQ
ë ê q_È°^΁~ò# "A" „¬ô‹¬H
ë O
› #° 10. ㄬH›@# : ''„¦ÔA K³eÁOKÇx q^¥¼~¡°ÖO^Î~¡¶ D <³ 15=

ㄬ[O^Î~¶
¡ qiqQê Hùx KÇ^°
Î =ô`Ç°<Œß~¡°. `ŒsY°…Õ„¬ô H›zó`ÇOQê „¦ÔA° K³eÁOKŒe.ÑÑ -

T‚¬ì#° : I) D „¬ô‹¬H
ë O
› q_È°^΁䛽 =òO^ΰ ƒì~¡`^
Ç ‰
Í O
× …Õ XH› ‹¬¶ø° <Ë\ ©‹¬° ƒÕ~¡°Û…Õ q^¥¼~¡°Öä›½ Wzó#

„¬ô‹¬H
ë › „¬~<
¡” Œ‰×Hë÷ Q®"Œi ‹¬OY¼ JæO. <Ë\ ©‹¹

II) D „¬ô‹¬ëH› ~¡KÇ~ò`Ç ~Œ‹²# „¬ô‹¬ëHêxßO\ ÷ T‚¬ì#° : I) 15 `ŒsY°…Õ„¬ô „¦ÔA K³eÁOKÇx q^¥¼~¡°Ö°

…ÕxH÷ W^Í Ju `Ç䛽ø= ^Î~¡ H›e¾# „¬ô‹¬ëH›O. ‹¬¶ø°H÷ P `Ç~ŒÞ`Ç ~Œ=‹²# J=‹¬~¡O

9. ㄬH›@# : q^¥¼~¡°ÖO^Î~¡¶ SH›=°`ǼO`Ë "³°Qêe. …è^ΰ.

T‚¬ì#° : I) L=°à_ Q ê =ôO>è Z…ìO\ ÷ ‹¬ = °‹¬ ¼ <³ á < Œ II) 15= `ŒsY°…Õ„¬ô „¦ÔA K³eÁOKÇH›‡é`Í „¦ÔA

‹¬°°=ôQê „¬i+¬øiOKÇ°HË=KÇ°ó#°. K³eO


Á z~# "Œix =°i†Çò K³eOKÇx "Œix

II) SH›=°`ǼOQê …èx q^¥¼~¡°


Ö O^Î~¶
¡ ‡~”‰
¡ § "Íö~Þ~¡° `Ç~¡Q®u Q®^ΰ…Õ =ôOKÇ°`Œ~¡°.

Y
#O_ |‚²ì+¬øiO„¬|_È`Œ~¡°.

M
1234567890123456789012
12345678901234567890123456789012123456789012345678901234567890121234567890123456789012345678901212345678901234567890123
12345678901234567890123

q=~¡}ì`ÇàH› ["Œ|°°
12345678901234567890123456789012123456789012345678901234567890121234567890123456789012345678901212345678901234567890123
1234567890123456789012 12345678901234567890123
12345678901234567890123456789012123456789012345678901234567890121234567890123456789012345678901212345678901234567890123
1234567890123456789012 12345678901234567890123
1234567890123456789012
12345678901234567890123456789012123456789012345678901234567890121234567890123456789012345678901212345678901234567890123
12345678901234567890123
PRACTICE TEST - 2 :

E
12345678901234567890123456789012123456789012345678901234567890121234567890123456789012345678901212345678901234567890123
1234567890123456789012 12345678901234567890123
12345678901234567890123456789012123456789012345678901234567890121234567890123456789012345678901212345678901234567890123
1234567890123456789012 12345678901234567890123
12345678901234567890123456789012123456789012345678901234567890121234567890123456789012345678901212345678901234567890123
1234567890123456789012 12345678901234567890123

D
9
59
1. (1) U ‹¬O^Î~¡Å䛽…ÿá<Œ e„¦¹“ "Œ_È\ìxöH W+¬“„¬_È`Œ~¡°. Hê=ô# ‡iøOQ·ä›½ ㄬ`ͼH› ‹¬Ö…쁰 öH\ì~òªë~¡°. Hê=ô# "ŒiH›

CA
95
T‚¬ì#O- II `DŽ¬C. †Ç°[=¶#¼O ZÁ„¬C_ȶ öH\ì~òOz# ㄬ^͉§…Õ ª=¶#¼ ㄬ[° ‡iøOQ·

50
‹¬O^Î~¡Å䛽 =^Îí<Í =ôO_ "Œix H›x̄@°“䛽x e„¦¹“…Õ KÍÀ‹ë P "Œ‚¬ì<Œ#° `ùyOKÇ_ÈO ª^¥~¡}"Í°. JH›ø_

98
A
ㄬ = ¶^Î O [iy`Í Hꇐ_È … è ~ ¡ ° . Hê|\ ÷ “ J@°=O\ ÷ ƒÕ~¡°Û#° KǶ‹² ª=°#¼ ㄬ[° =òO^ΰQê<Í `³°‹¬°
:8
ㄬ=¶^ÎO U^³á<Œ ‡Ú~¡ƒì@°# [iy`Í `Ç=° ƒì^Î¼`Ç HË"Œe. Hê=ô# T‚¬ì#O -I =°i†Çò T‚¬ì#O- II
E

I
h

…è^Î<Í ã„¬H›@# ª~ŒO‰×O. T‚¬ì#O- ㄬH›@##° ï~O_ȶ ㄬH›@##° `Ç$„²ë„¬~¡°ªë~ò.


.P
IC

`Ç$„²ë„¬~¡°‹¬°ëOk. 7. (3) T‚¬ì#O- I =°i†Çò T‚¬ì#O- II ï~O_ȶ Wzó#


AD

2. (1) ƒìQê H›+¬“„¬_Í"Œ~¡O^Î~¡¶ ^Î#=O`Ç°° Hê…è~¡°. ㄬH›@##° `Ç$„²ë„¬iKÍ"Í. \ ÷ïH@°Á \ ÷ïH\˜ HÒO@~ŸO^ΰ
AB
O

L^¥ : ~ËA䛀bH÷ "³ˆõ¤"Œ~¡° ~Ë[O`Œ H›+¬“„¬_ aªë~ò. Hêx 䛀¼…Õ xƒ_Í W|ÄOk =ôO_Èä€
› _È^O
Î >è
ER
CH

„¬xKͪë~°
¡ . Hêx "ŒiH÷ =KÍó ªÚ=òà "Œi uO_™ Q®°_ȁ
Û ä›½ WO@ï ~ ß\˜…Õ \ ÷ ï H @°Á |°H± KÍ ‹ ¬ ° HË"Œx ㄬ H › @ #
YD

䛀_¨ ‹¬i‡é^ΰ. Hêx XH› |^ÎíH›‹¬°ë_È° =¶ã`ÇO Z„¬æ\ ÷H© ª~ŒO‰×O. J…ìöQ D ~ËA…ÕÁ Z䛽ø= =°Ok. ㄬ[°
,H

^Î#=O`Ç°_È° Hê…è_È°. "Œ_È° ^Î#=O`Ç°_³á<Œ ªÚ=òà D q^¥<Œxß J#°‹¬i‹¬°ë<Œß~¡°. Hê|>è“ D ㄬH›@#


T

I
AM

~ËA~ËAH© `Çiy‡é`Ç°`ÇOk. T‚¬ì#O- ㄬH›@##° W=Þ_ÈO [iyOk.


H

`Ç$„²ë„¬~¡°‹¬°ëOk.. 8. (4) „¬ô‹¬ëH›O …Õx JO‰§° =°Ozq, P‹¬H÷ë H›q J~ò`Í<Í


M

I
AM

(3)
IG

3. PiÖH› Z^ΰQ®°^΁䛽 H›+¬“„¬_ „¬xK͆ǰ_ÈO J=‹¬~¡O P „¬ô‹¬ëHêxß ã„¬[° HùO\ì~¡°. Hêx T‚¬ì#O-

II
KH

=°i†Ç ò |^Î í H ÷ O KÇ ä › ½ O_¨ „¬ x KÍ À ‹ë H› + ¬ “ „ ¬ _ È _ È O J^Í T‚¬ì#O- ° ~


ï O_ȶ ㄬH@
› #䛽 q~¡°^ÎO
œ Qê =ô<Œß~ò.

I (1)
R

J"Œ@° J=ô`Ç°Ok. Hê=ô# T‚¬ì#O- =°i†Çò 9. SH›=°`ǼO`Ë =ôO>è Z…ìO\ ÷ ‹¬=°‹¬¼<³á<Œ L=°à_Qê

T‚¬ì#O- II ï~O_ȶ ㄬH›@##° `Ç$„²ë„¬~¡°ªë~ò. „¬i+¬øiOKÇ°HË=KÇó<Í ƒì=<Í ã„¬H›@#…Õ =ôOk.

4. (1) †Çò^ÎíO J<ÍH›=°Ok ㇐}ì#° ‚¬ìi‹¬°ëOk. Hê|\ ÷“ SH›=°`ǼO`Ë =ôO>è Jhß =°Oz „¦¬e`Œ…è =ôO\ì~ò.

T‚¬ì#O- I ㄬH›@##°`Ç$„²ë„¬~¡°‹¬°ëOk. ㄬu^͉§xH÷ WH›ø_È ‡l\ ÷"£Qê =¶ã`Ç"Í° ㄬH›@# WKŒó~¡°. T‚¬ì#O

^¥x ~¡H›Æ} xq°`ÇëO J}ìކÇò^¥°, P†Çò^Î‹¬O„¬uë, - I ㄬH›@##° `Ç$„²ë„¬~¡°‹¬°ëOk. Hêx T‚¬ì#O- II


~¡H›Æ}^Έ×O =ôO\ì~ò. Hê|\ ÷“ T‚¬ì#O- II ㄬH›@##° q~¡°^ΜOQê =ô#ßk.

`Ç$„²ë„¬~¡KÇ^ΰ. 10. (4) <Ë\ © ‹ ¹ KÇ k q# „²  Á  O^Î ~ ¡ ¶ "Œi `Ç e Á ^ Î O ã_È ° ä› ½

5. (3) T‚¬ì#O- I =°i†Çò T‚¬ì#O - II ï~O_ȶ ㄬ@##° q+¬†Ç°O `³e†Ç°*苲 ‹¬ï~á# ‹¬=°†Ç°O…Õ „¦ÔA°

`Ç$„²ë„¬~¡°ªë~ò. K³ e Á ª ë ~ ¡ < Í k ‹¬ ¶ ø° †Ç ¶ [=¶#¼O †³ ò H› ø ƒ ì =#.

6. (3) ‹¬O„¦¬°O…Õ ㄬ*ìㄬux^Î°ä›½ Qù„¬æ"ŒiH÷ D ~ËA…ÕÁ T‚¬ì#O- I =°i†Çò T‚¬ì#O- II ~


ï O_ȶ ㄬH@
› #`Ë

ZH›ø_³á<Œ QÒ~¡=O a‹¬°ëOk. J…ìöQ "Œi "Œ‚¬ì<Œ ‹¬O|O^ÎO …èx T‚¬ì…è.

REASONING ABILITY (LOGICAL) ○ ○ ○ ○ ○ ○ ○ ○


160
○ ○ ○ ○ ○ ○ ○ ○ ○ ○ ○ ○ ○ ○ ○ ○ ○ ○ ○
Study Material
123456789012345678901234567890121234567890123456789012345678901212
12345678901234567890123456789012123456789012345678901234567890121234567890123456789012345678901212345678901234567890123
1234567890123456789012345678901212345678901234567890123456789012123456789012345678901234567890121234567890123
1234567890123456789012345678901212345678901234567890123456789012123456789012345678901234567890121234
12345678901234567890123456789012123456789012345678901234567890121234567890123456789012345678901212345678901234567890123456789012123
1234567890123456789012345678901212345678901234567890123456789012123456789012345678901234567890
12345678901234567890123456789012123456789012345678901234567890121234567890123456789012345678901212345678901234567890123
1234567890123456789012345678901212345678901234567890123456789012123456789012345678901234567890121234567890123
1234567890123456789012345678901212345678901234567890123456789012123456789012345678901234567890121234
123456789012345678901234567890121234567890123456789012345678901212
12345678901234567890123456789012123456789012345678901234567890121234567890123456789012345678901212345678901234567890123456789012123
1234567890123456789012345678901212345678901234567890123456789012123456789012345678901234567890
12345678901234567890123456789012123456789012345678901234567890121234567890123456789012345678901212345678901234567890123
1234567890123456789012345678901212345678901234567890123456789012123456789012345678901234567890121234567890123
1234567890123456789012345678901212345678901234567890123456789012123456789012345678901234567890121234
12345678901234567890123456789012123456789012345678901234567890121234567890123456789012345678901212345678901234567890123456789012123
1234567890123456789012345678901212345678901234567890123456789012123456789012345678901234567890
123456789012345678901234567890121234567890123456789012345678901212
PRACTICE TEST - 3
12345678901234567890123456789012123456789012345678901234567890121234567890123456789012345678901212345678901234567890123
1234567890123456789012345678901212345678901234567890123456789012123456789012345678901234567890121234567890123
1234567890123456789012345678901212345678901234567890123456789012123456789012345678901234567890121234
12345678901234567890123456789012123456789012345678901234567890121234567890123456789012345678901212345678901234567890123456789012123
1234567890123456789012345678901212345678901234567890123456789012123456789012345678901234567890
123456789012345678901234567890121234567890123456789012345678901212
12345678901234567890123456789012123456789012345678901234567890121234567890123456789012345678901212345678901234567890123
1234567890123456789012345678901212345678901234567890123456789012123456789012345678901234567890121234567890123
1234567890123456789012345678901212345678901234567890123456789012123456789012345678901234567890121234
12345678901234567890123456789012123456789012345678901234567890121234567890123456789012345678901212345678901234567890123456789012123
1234567890123456789012345678901212345678901234567890123456789012123456789012345678901234567890
12345678901234567890123456789012123456789012345678901234567890121234567890123456789012345678901212345678901234567890123
1234567890123456789012345678901212345678901234567890123456789012123456789012345678901234567890121234567890123
1234567890123456789012345678901212345678901234567890123456789012123456789012345678901234567890121234
12345678901234567890123456789012123456789012345678901234567890121234567890123456789012345678901212345678901234567890123456789012123
1234567890123456789012345678901212345678901234567890123456789012123456789012345678901234567890
123456789012345678901234567890121234567890123456789012345678901212
12345678901234567890123456789012123456789012345678901234567890121234567890123456789012345678901212345678901234567890123456789012123
1234567890123456789012345678901212345678901234567890123456789012123456789012345678901234567890
12345678901234567890123456789012123456789012345678901234567890121234567890123456789012345678901212345678901234567890123
1234567890123456789012345678901212345678901234567890123456789012123456789012345678901234567890121234567890123
1234567890123456789012345678901212345678901234567890123456789012123456789012345678901234567890121234

1. ㄬH›@# : ''^ΆǰK͋² ãH÷O^Î䛽 köQ@„¬ô_È° e„¦¹“#° 4. ㄬH›@# : P ㄬ^͉×O…Õ …ÿáã|s xiàOKÇ_ÈO =¼~¡ÖO.

L„¬ † ³ ¶ yOKÇ H › O _ Ñ Ñ XH› J‡~Ÿ “ " ³ ° O\˜…Õx T‚¬ì#° : I) P ㄬ^‰


Í O
× …Õ x=tOKÇ°"Œ~¡° x~¡H~
Æ › Œ‹¬°¼°.

e„¦¹“=^Îí JuH÷OKÇ|_# <Ë\ ©‹¹.


II) P ㄬ^‰
Í O
× …Õ x=tOKÇ° ㄬ[° ‹¬=¶KŒ~¡O

T‚¬ì#° : I) H›ï~O@° a°Á JkH›OQê ~Œ=KÇ°ó#°.


`³°‹¬°HË=_¨xH÷ P‹¬Hë÷ KǶ„¬~°
¡ .

III) P ㄬ^͉×O…Õ JO`Ç䛽=òO^Í KŒ…ì


II) ãH÷ O kH÷ kö Q @„¬ ô _È ° e„¦ ¹ “  ° |~¡ ° =ô#°

…ÿáã|s° =ô<Œß~ò.
"³¶†Ç°…è=ô.

1)I =°i†Çò II ‹¬i†³Ø°#q. 2) III =¶ã`Ç"Í° ‹¬i†³Ø°#k.


III) e„¦¬ô“° H›eæOKÇ_ÈO XH› ªÏH›~¡¼"Í° Hêx

3)I =°i†Çò III =¶ã`Ç"Í° ‹¬i†³Ø°#q.


‚¬ì䛽ø Hê^ΰ.

Y
4)II =¶ã`Ç"Í° ‹¬i†³Ø°#k.

1) III =¶ã`Ç"Í° ‹¬i†³Ø°#k.

I II III

M
5) …è^¥ …è^¥ …Õ U^ËXH›\ ÷ ‹¬i†³Ø°#k.

5. ㄬH›@# :
2) Un ‹¬i†³Ø°#k Hê^ΰ.
WO[h~¡ ° J=Þ_È O ‹¬ ° ƒ ’ O J~ò`Í <Í # °

I III

E
3) =°i†Çò =¶ã`Ç"Í° ‹¬i†³Ø°#q
WO[h~¡° J=Þ#°.

T‚¬ì#° :

D
9
4) Jhß ‹¬i†³Ø°#q
I) =¼H÷ë`ÇÞO =ô#ß"Œ~¡° ã‡Ú̄¦+¬#…˜Qê

59
5) I =¶ã`Ç"Í° ‹¬i†³Ø°#k

CA
=ôO_¨#°ä›½O\ì~¡°.

95
2. ㄬH›@# : Z䛽ø= =°Ok q^¥¼~¡°Ö° g_†³¶öQ"£°ž
II) HùO`Ç=°Ok H›+¬“ª^Î¼"³°Ø# „¬#°<Í

50
P_È°@䛽 W+¬“„¬_È`Œ~¡°.
ªkOKŒ#°ä›½O\ì~¡°.

98
A
T‚¬ì#° : I) ‹¬¶ø…˜ž…Õ g_†³¶öQ"£°ž P_OKÇ~¡°. III) WO[h~¡° J=Þ_ÈO KŒ…ì ‹¬°ƒ’O.
:8
II) g_†¶
³ Q
ö "£°ž=# =¼H÷`
ë Þ
Ç O ̄~¡°Q®°`Ç°Ok. I III
E

1) =¶ã`Ç"Í° ‹¬i†³Ø°#k. 2) =¶ã`Ç"Í° ‹¬i†³Ø°#k.


h

II III
.P

III) öQ"£°ž P_È@O „²Áä›½ ‹¬~¡^¥. 3) =°i†Çò ° =¶ã`Ç"Í° ‹¬i†³Ø°#q.


IC

III II
AD

1) =¶ã`Ç"Í° ‹¬i†³Ø°#k. 4) =¶ã`Ç"Í° ‹¬i†³Ø°#k.

I III 6. ㄬH›@# : H›O„¬î¼@~Ÿ ~¡OQ®O #O^ΰ J`Ç°¼`Çë=° tH›Æ},


AB
O

2) =°i†Çò =¶ã`Ç"Í° ‹¬i†³Ø°#k.

I II =¶ ‹¬O‹¬Ö#O^ΰ JOkOKÇ|_È°`Ç°Ok - XH›


ER

3) =°i†Çò =¶ã`Ç"Í° ‹¬i†³Ø°#q.


CH

4) II =¶ã`Ç"Í° ‹¬i†³Ø°#k.
‹¬O‹¬Ö ㄬH›@#.
YD

5) Jhß ‹¬i†³Ø°#"Í.
T‚¬ì#° : I) ㄬ[…Õ KŒ…ì=°Ok H›O„¬î¼@~Ÿ q^μ „¬@Á
,H

3. ㄬH›@# : P‹¬H÷ë H›ey†Çò<Œß~¡°.


T

XöH =‹¬°ë=ô ^Î~¡ =¶ï~ø\˜…Õx ̄^Î푐„¬ô…Õ


AM

II) ㄬ[° J`Ç°¼`Çë=° tH›Æ} HË~¡°ä›½O\ì~¡°.


̂ìKÇ°óQê#¶, ‡Á\˜‡¦~¡O g°^Î …è^¥ z#ß
H

III)
M

H›O„¬î¼@~Ÿ q^μ…Õ x‘â`°


Ç …ÿ#
á L^˼Qꁰ
‘„¬ô…Õ `Ç䛽ø=Qê#¶ LO@°Ok.
AM
IG

T‚¬ì#° :
‹¬°ƒ’OQê ‡ÚO^Î=KÇ°ó#°.
I) z#ß ‘„¬ô…Õ aOKÍ =‹¬°ë=ô°

I
KH

1) =¶ã`Ç"Í° ‹¬i†³Ø°#k.
#=°à^Îy#q Hê=ô.

II III
R

2) =°i†Çò =¶ã`Ç"Í° ‹¬i†³Ø°#k.


II) ̄^Îí ‘„¬ô †Ç°[=¶#°° H›‹¬“=°~¡Á#°

3) ̄á=hß ‹¬i†³Ø°#"Í. 4) II =¶ã`Ç"Í° ‹¬i†³Ø°#k.

"³¶‹¬O K͋¬°ë<Œß~¡°.

7. ㄬH›@# : XH› ‹¬¶ø° ㄬH›@# - '=¶ ‹¬¶ø…˜ #O^ΰ

III) ̄^Îí ̄^Îí ‘„¬ô#° x~¡Þ‚²ìOKÇ_O


È KŒ…ì

|‹¬°ž ‹¬sދ¬°, H›O„¬î¼@~Ÿ HËzOQ·, ªéæ~Ÿ“ ž,

Y~¡°ó`Ë ä›€_Ȱ䛽#ß „¬x. Hê=ô# P ‘„¬ô…Õ

ªO‹¬ø $uH› Hê~¡¼ãH›=¶° =°i†Çò KÇ^ΰ=ô`Ë

aOKÍ =‹¬°=
ë ô ^~
Î 
¡ ° ̄Oz J=òà`Œ~¡°.
‡@° Jxß ~¡ H ê HêO„² \ © + ¬ < £ „¬ s H› Æ  ä› ½

1) I =¶ã`Ç"Í° ‹¬i†³Ø°#k.
HËzOQ· W=Þ|_È°#°.

III
T‚¬ì#° :
2) =¶ã`Ç"Í° ‹¬i†³Ø°#k.
I) D ~ËA…ÕÁ ªO‹¬ø $uH› Hê~¡¼ãH›=¶°

3) II …è^¥ III …Õ U^Ë XH›\ ÷ ‹¬i†³Ø°#k.


q°ye# "Œ\ H
÷ O
› >è =òY¼O Hê=ô# À„ï~O\˜ž

4) Un ‹¬i†³Ø°#k Hê^ΰ.
JO^Î~¡¶ "Œi „²Á#° P ‹¬¶ø…˜…Õ<Í

5) II =¶ã`Ç"Í° ‹¬i†³Ø°#k.
*ì~ò<£ Kͪë~¡°.

REASONING ABILITY (LOGICAL) ○ ○ ○ ○ ○ ○ ○ ○


161
○ ○ ○ ○ ○ ○ ○ ○ ○ ○ ○ ○ ○ ○ ○ ○ ○ ○ ○
Study Material
II) Z䛽ø==°Ok À„ï~O\˜ž Jxß =‹¬`°
Ç ° H›ey# 9. ㄬH›@# : „¬sH›Æ…Õ q[†Ç°O ªkOKŒO>è H›+¬“„¬_¨x

‹¬¶ø…˜…Õ<Í "Œi „²


Á #° *ì~ò<£ Kͪë~°
¡ . q#†Ÿ°ä›½ ~ŒA K³Ì„æ#°.

III) WO\ÕÁ `ÇeÁ^ÎOã_È°° W^Îí~¡¶ L^˼Qꁰ T‚¬ì#° : I) ~ŒA †³òH›ø ‹¬‚¬ð q#†Ÿ° ‡\ ÷ªë_È°.

K͋¬°ë#ß D ~ËA…ÕÁ W…ìO\ ÷ Jxß II) „¬sH›Æ…Õ q[†Ç°O ªkOKÇ°@ Hê"Œe.

=‹¬`Ç°° J=‹¬~¡O.
III) H›i”# Jƒ’¼‹¬#O q[†Ç¶xH÷ ƒì@

1) I =¶ã`Ç"Í° ‹¬i†³Ø°#k.
J=ô`Ç°Ok.

2) II =¶ã`Ç"Í° ‹¬i†³Ø°#k.
1) I =°i†Çò II =¶ã`Ç"Í° ‹¬i†³Ø°#q.

3) I =°i†Çò II =¶ã`Ç"Í° ‹¬i†³Ø°#q.


2) II =°i†Çò III =¶ã`Ç"Í° ‹¬i†³Ø°#q.

4) Jhß ‹¬i†³Ø°#"Í.
3) I =°i†Çò III =¶ã`Ç"Í° ‹¬i†³Ø°#q.

8. ㄬH›@# : =¶ ‘„¬ ô ä› ½ =zó Jxß "³ ï ~ á \ ©  ° 20%


4) Jhß ‹¬i†³Ø°#"Í.

10. ㄬH›@# :

Y
ƒ ì s =~ŒÂ° L#߄¬ æ \ ÷ H © D ‹¬ O =`Ç ž ~¡ O
iƒè@°`Ë Hù#O_ - XH› ‘„¬ô"Œi ㄬH›@#.

T‚¬ì#° :
ã\섦²H± ‹¬=°‹¬¼ JO`ÇQê H›xæOKDžè^ΰ.

M
I) iƒè@° PH›~¡ } =# KŒ…ì =°Ok ‘„¬ô#°

T‚¬ì#° : I) ƒìs=~ŒÂ…Õ =¶ã`Ç"Í° ã\섦²H± ‹¬=°‹¬¼


‹¬O^ÎiŪë~¡°.

E
LO@°Ok.
II) iƒè@° ㄬH›\ ÷OKÇ#@Á~ò`Í H›‹¬“=°~¡°Á "Íö~

II)

D
=~ŒÂ° ã\섦²H± H›^ÎeH›ä›½ U =¶ã`ÇO

9
‘„¬ôä›½ "³ˆìë~¡°.

59
J"ŒO`Ç~¡O Hê^ΰ.

III)

CA
95
H›‹¬“=°~¡Á#° PH›iÂOKÇ_¨xH÷ iƒè@°

III) =~ŒÂHêO "³ò`ÇO


ë KŒ…ì =òO^ΰ *ìãQ®`
ëÇ °

ㄬH›\ ÷OKÇ_ÈO =¶ã`Ç"Í° ‹¬i†³Ø°# „¬^Μu.

50
f‹¬°ä›½x ã\섦H
² #
± ° x~¡Þ‚²ìOKÇ_"
È °
³ #
Ø k.

1) I =¶ã`Ç"Í° ‹¬i†³Ø°#k.
I II
98
A
1) =°i†Çò =¶ã`Ç"Í° ‹¬i†³Ø°#q.

II
:8
I
2) =¶ã`Ç"Í° ‹¬i†³Ø°#k.
2) =¶ã`Ç"Í° ‹¬i†³Ø°#k.

II III
E

II II
3) =°i†Çò =¶ã`Ç"Í° ‹¬i†³Ø°#q.
h

3) =°i†Çò =¶ã`Ç"Í° ‹¬i†³Ø°#q.


.P

I II
IC

4) =°i†Çò =¶ã`Ç"Í° ‹¬i†³Ø°#q.


4) III =¶ã`Ç"Í° ‹¬i†³Ø°#k.
AD

1234567890123456789012
12345678901234567890123456789012123456789012345678901234567890121234567890123456789012345678901212345678901234567890123
12345678901234567890123
12345678901234567890123456789012123456789012345678901234567890121234567890123456789012345678901212345678901234567890123
1234567890123456789012 12345678901234567890123
1234567890123456789012
12345678901234567890123456789012123456789012345678901234567890121234567890123456789012345678901212345678901234567890123
1234567890123456789012 12345678901234567890123
12345678901234567890123456789012123456789012345678901234567890121234567890123456789012345678901212345678901234567890123
PRACTICE TEST - 3 :
12345678901234567890123
q=~¡}ì`ÇàH› ["Œ|°°
AB
O

1234567890123456789012
12345678901234567890123456789012123456789012345678901234567890121234567890123456789012345678901212345678901234567890123
1234567890123456789012 12345678901234567890123
12345678901234567890123456789012123456789012345678901234567890121234567890123456789012345678901212345678901234567890123
1234567890123456789012 12345678901234567890123
12345678901234567890123
12345678901234567890123456789012123456789012345678901234567890121234567890123456789012345678901212345678901234567890123
ER
CH

1. (2) Wzó# =ü_È° T‚¬ì#…Õ Un ㄬH@


› ##° ‹¬O`Ç$„²„
ë ~
¬ K
¡ Í 6. (3) T‚¬ì#hß ã„¬H›@##° `Ç$„²ë„¬~¡KÍ q^ÎOQê =ô<Œß~ò.
YD

q^ÎOQê …è=ô. J‡~Ÿ“"³°O\˜…ÕÁ x=tOKÍ"Œ~¡° HùOK³O 7. (2) T‚¬ ì #- II ㄬ H › @ ##° `Ç $ „² ë „ ¬ ~ ¡ ° ‹¬ ° ë O k. ö H =O
,H

ªO‹¬$øuH› Hê~¡¼ãH›=¶° =ôO_ KÇ^°


Î =ô …èH‡
› é`Í À„ï~O\˜ž
T

H›~
ï O@° a°Áä½
› P…ÕzOKÇ~°
¡ . e„¦
“¹ ° `Çy#O`Ç |~¡°=ô#°
AM

P ‹¬¶ø°…Õ `Ç=° „²Á#° =ôOKÇ~¡°. J…ì<Í T‚¬ì#-


"³¶†Ç°Q®eöQ "Œ\ ÷x ayªë~¡°. J‡~¡°“"³°O@°H÷
H

III
M

‹¬iHê^ΰ.
~ŒH›‡éH›° ªyOKÍ"Œ~¡° e„¦¹“x ª^¥~¡}OQê<Í
AM

I
IG

8. (1) T‚¬ì#- ㄬH›@##° `Ç$„²ë„¬~¡°‹¬°ëOk. ZO^ΰH›#Qê


"Œ_È=KÇ°ó. WH›ø_È "Œä›½ø Q®°iOz ㄬªë=<Í …è^ΰ.
KH

iƒè@° ㄬH›\ ÷Oz#„¬ô_È° H›‹¬“=°~¡°Á ‘„¬ô#° =¶=ü°


Hê=ô<Œ Jxß T‚¬ì#¶ ‹¬i†³Ø°#q Hê=ô.
R

H›O>è Z䛽ø=Qê ‹¬O^ÎiŪë~¡°. Hêh iƒè@° ㄬH›\ ÷À‹ë<Í


2. (1) ‹¬æ+¬“OQê öQ"£°ž P_È@O „²Áä›½ ‹¬~¡^¥. Hê|>è“ "Œ~¡°

‘„¬ô#° ‹¬O^ÎiŪë~¡°. J#°HË=@O ‹¬iHê^ΰ. J…ìöQ


g_†³¶öQ"£°ž P_È\ìxß W+¬“„¬_È`Œ~¡°.

Jk =¶ã`Ç"°
Í H›‹=
“¬ °~¡#
Á ° PH›iÂOKÇ_¨xH÷ ‹¬i†³°
Ø # „¬^u
íÎ
3. (2) T‚¬ì#- I =°i†Çò T‚¬ì#- II ° P^¥~¡O…èxq.

Hê^ΰ. |@“ <Œ}¼`Ç, ö~@°Á, ‘„¬ô…Õx ªÏH›~Œ¼°,

T‚¬ì#- III Wzó# ㄬH›@##° `Ç$„²ë„¬~¡°‹¬°ëOk.

‘„¬ô"Œi ㄬ=~¡ë# JxßO\ ÷x |\ ÷“ H›‹¬“=°~¡°Á ‘„¬ôH÷

4. (5) Wzó# =ü_È° T‚¬ì#…Õ U^Ë XH›\ ÷ x[O


=ªë~¡°.

J~ò=ôO_È=KÇ°ó. Hêx ‹¬æ+¬»`Ç …è^ΰ.


9. (2) XH› ‹¬‚¬ð#° qx#O`Ç =¶ã`Œ# ‡\ ÷OKŒx …è^ΰ.

5. (4) T‚¬ì#- I =°i†Çò T‚¬ì#- II ° ‹¬i†³Ø°#q Hê^ΰ.


Jk qx# =¼H÷ë †³òH›ø ‹¬Þƒì=Ōá P^¥~¡„_
¬  =ôO@°Ok.

Hêx HùO`Ç=°Ok H›+¬“ª^Î¼"³°Ø# „¬#°<Í


Hê=ô# T‚¬ì#- I ㄬH›@##° `Ç$„²ë„¬~¡KÇ^ΰ. T‚¬ì#-

K͆¶
Ç #°ä›½O\ì~¡<k
Í ‹¬i†³°
Ø # TÀ‚ì. Wk ㄬH@
› ##°
II =°i†Çò T‚¬ì#- III ° ï~O_ȶ ㄬH›@##°

`Ç$„²ë„¬~¡°‹¬°ëOk. Hê=ô# T‚¬ì#- II ‹¬i†³Ø°#k.


`Ç$„²ë„¬~¡°ªë~ò.

REASONING ABILITY (LOGICAL) ○ ○ ○ ○ ○ ○ ○ ○


162
○ ○ ○ ○ ○ ○ ○ ○ ○ ○ ○ ○ ○ ○ ○ ○ ○ ○ ○
Study Material
10. (4) ã\섦²H± ‹¬=°‹¬¼ä›½ J<ÍH› Hê~¡}쁰 =ôO\ì~ò. (L^¥‚¬ì~¡}䛽 ‡ÚQ®=°OKÇ°, ㄬ=¶^¥° ....) Hê=ô# T‚¬ì#- I ‹¬i†³Ø°#k

Hê^ΰ. T‚¬ì#- II `DŽ¬C. =~ŒÂ =Á ã\섦²H±ä›½ JO`Ç~Œ†Ç°O ‹¬‚¬ì[O. T‚¬ì#- III ㄬH›@##° `Ç$„²ë„¬~¡°‹¬°ëOk.

123456789012345678901234567890121234567890123456789012345678901212
12345678901234567890123456789012123456789012345678901234567890121234567890123456789012345678901212345678901234567890123
1234567890123456789012345678901212345678901234567890123456789012123456789012345678901234567890121234567890123
1234567890123456789012345678901212345678901234567890123456789012123456789012345678901234567890121234
12345678901234567890123456789012123456789012345678901234567890121234567890123456789012345678901212345678901234567890123456789012123
123456789012345678901234567890121234567890123456789012345678901212345678901234567890123
123456789012345678901234567890121234567890123456789012345678901212
12345678901234567890123456789012123456789012345678901234567890121234567890123456789012345678901212345678901234567890123456789012123
123456789012345678901234567890121234567890123456789012345678901212345678901234567890123
12345678901234567890123456789012123456789012345678901234567890121234567890123456789012345678901212345678901234567890123
1234567890123456789012345678901212345678901234567890123456789012123456789012345678901234567890121234567890123
1234567890123456789012345678901212345678901234567890123456789012123456789012345678901234567890121234
123456789012345678901234567890121234567890123456789012345678901212
12345678901234567890123456789012123456789012345678901234567890121234567890123456789012345678901212345678901234567890123456789012123
123456789012345678901234567890121234567890123456789012345678901212345678901234567890123
12345678901234567890123456789012123456789012345678901234567890121234567890123456789012345678901212345678901234567890123
1234567890123456789012345678901212345678901234567890123456789012123456789012345678901234567890121234567890123
1234567890123456789012345678901212345678901234567890123456789012123456789012345678901234567890121234
PREVIOUS BITS
12345678901234567890123456789012123456789012345678901234567890121234567890123456789012345678901212345678901234567890123456789012123
123456789012345678901234567890121234567890123456789012345678901212345678901234567890123
123456789012345678901234567890121234567890123456789012345678901212
12345678901234567890123456789012123456789012345678901234567890121234567890123456789012345678901212345678901234567890123
1234567890123456789012345678901212345678901234567890123456789012123456789012345678901234567890121234567890123
1234567890123456789012345678901212345678901234567890123456789012123456789012345678901234567890121234
12345678901234567890123456789012123456789012345678901234567890121234567890123456789012345678901212345678901234567890123456789012123
123456789012345678901234567890121234567890123456789012345678901212345678901234567890123
12345678901234567890123456789012123456789012345678901234567890121234567890123456789012345678901212345678901234567890123
1234567890123456789012345678901212345678901234567890123456789012123456789012345678901234567890121234567890123
1234567890123456789012345678901212345678901234567890123456789012123456789012345678901234567890121234
123456789012345678901234567890121234567890123456789012345678901212
12345678901234567890123456789012123456789012345678901234567890121234567890123456789012345678901212345678901234567890123456789012123
123456789012345678901234567890121234567890123456789012345678901212345678901234567890123
12345678901234567890123456789012123456789012345678901234567890121234567890123456789012345678901212345678901234567890123
1234567890123456789012345678901212345678901234567890123456789012123456789012345678901234567890121234567890123
1234567890123456789012345678901212345678901234567890123456789012123456789012345678901234567890121234
12345678901234567890123456789012123456789012345678901234567890121234567890123456789012345678901212345678901234567890123456789012123
123456789012345678901234567890121234567890123456789012345678901212345678901234567890123

1. ㄬH›@# : g}#° g°>è "Œ~¡O^Î~¡¶ Qê†Ç°ä›½…è (Asst. 7. ㄬH›@# : Ja ''‹¬¶Kǁxß KÇkq =ôO>è `Ç^Î#°Q®°}OQê

Endowment Commissioner. - 2011) Hê~¡¼ãH›=¶xß ã‡~¡OaOKÇ°ÑÑ. (S.I. - 2011)

T‚¬ì#° : I) g}#° g°>è "Œ~¡O^Î~¡¶ kÞf†Ç°ªÖ#O T‚¬ì#° : I) Ja ‹¬¶KÇ#xß J~¡ÖO K͋¬°ä›½O\ì_È°.

II) JO^Î~¡° =°‚²ìˆ×° 䛀_¨ kÞf†Ç°ªÖ#O II) Ja Hê~¡¼ãH›=¶xß ‹¬=°~¡O


Ö Qê x~¡Þ‚²ìKÇQ
® _È°.

2. ㄬH›@# : =°Ozh~¡° Z䛽ø= ã`ŒQ®°@ P~ËQê¼xH÷ KŒ…ì 8. ㄬH›@# : `DŽ¬C Q®°iOz ƒì^Î„¬_È=^ΰí. Hêx "Œ\ ÷ #°O_

Y
=°Ozk. (Asst. Endowment Commis- <Í~¡°óHË"Œe. (S.I. - 2011)

T‚¬ì#° : I)

M
sioner. - 2011) U^³ á < Œ XH› „¬ x KÍ À ‹@„¬ C _È ° `Ç „ ¬ C °

T‚¬ì#° : I) P~ËQ®¼=O`Ç°° JO^Î~¶


¡ Z䛽ø= =°Ozh~¡° ~Œ=_ÈO ‹¬‚¬ì[O.

E
ã`Œy<Œ~¡°. II) Q®`ÇO #°O_ aœ‡ÚOk, ㄬ‹¬°ë`ÇO `DŽ¬C°

D
9
II)

59
`Ç䛽ø= h~¡° ã`ŒQ®°@ P~ËQê¼xH÷ ‚¬ðxHê^ΰ. …è䛽O_¨ „¬xK͆ǰQ® "Œö~ ‹¬=°~¡°Ö°.

3. ㄬH›@# : 9. ㄬH›@# :
CA
95
g}° Jhß ä›€_¨ `ÇO*ì=î~¡°…Õ<Í `džǶ~¡° 5 …è^¥ 6 ‹¬OII =†Ç°‹¬°ž…Õ „²Á"Œ_x

KÍ † Ç ° |_È ° `Ç ° <Œß~ò. (Asst. Endowment ‡~”¡‰§…Õ "͆ǰ_ÈO J=‹¬~¡O. (S.I. - 2011)

50
Commissioner. - 2011) T‚¬ì#° : I)
98
A
6 ‹¬O=`Ǟ~Œ `Ç~¡°"Œ`Ç „²Á#°

T‚¬ì#° : I)
:8
g}° `džǶ~¡°K͆Çò@Hù~¡öH `ÇO*ì=î~¡° ‡~”¡‰§…ÕxH÷ J#°=°uOKÇ~¡°.
E

=ôOk. II) P =†Ç°‹¬°ž…Õ „²Á"Œ_È° <Í~¡Þ_¨xH÷


h
.P

II)
IC

g}° `džǶ~¡° K͆Çò@䛽 `ÇO*ì=î~¡° ‹¬O‹²^ΰœ_È° J=ô`Œ_È° =°i†Çò Ja=$kœ


AD

`DŽ¬æ =°i†³òH›ø ㄬ^͉×O …è^ΰ. ªÖ~ò =ôO@°Ok.

4. ㄬH›@# : 10. ㄬH›@# :


AB
O

''‹¬O„¬î~¡â`Ç H›ey# ‹¬Ow`ÇO =°#‰§ÅOux ''=¶`Ë PHê‰×O…Õ Zyi Q®Q®# ㄬ†Ç¶}


ER

(Asst. Endowment Commis-


CH

W‹¬ ° ë O k.ÑÑ J#°ƒ’"Œxß J#°ƒ’qOKÇO_ÑÑ. Wk XH›

sioner. - 2011) (S.I. - 2008)


YD

q=¶#†Ç¶# ‹¬O‹¬Ö ㄬH›@#.

T‚¬ì#° : I) XH› =¼H÷ë ‹¬Ow"Í`Çë HêH›‡é`Í =°#‰§ÅOu T‚¬ì#° : I) D ㄬH›@# KÇkq# `Ç~¡°"Œ`Ç KŒ…ì =°Ok
,H
T

=ôO_È°#°. ㄬ†Ç¶}÷䛽° q=¶#†Ç¶# ‹¬O‹¬Ö ^¥Þ~Œ


AM

II)
H

JO^Î ~ ¡ ° ‹¬ O w`Ç ‰§GAý  ä› ½ ‹¬ = °‹¬ ¼ ° ㄬ†Ç¶}O K͆Çò@䛽 PH›iÂ`Ç°° HêQ®~¡°.


M

=ôO_È=ô. II)
AM

ㄬ[° ª^¥~¡}OQê ‹¬°Mì#°ƒ’=O Q®


IG

5. ㄬH›@# : H›~ŒàQê~¡O †³òH›ø |_³û\˜ D ‹¬O=`Ǟ~¡O ‹¬OöHÆ=° q=¶#=ò#° ZO„²H› K͆Çò^ΰ~¡°.


KH

Hê~¡¼ãH›=¶ä›½ x^Î°° öH\ì~òOKDžè^ΰ. 11. ㄬH›@# : D ‹¬O=`Ǟ~¡O H›O„¬î¼@~Ÿ ̄ī~¡…˜ž #O^ΰ


R

(S.I. - 2011) H›‹¬“"£°ž KŒsû° `Çy¾Oz#k. (S.I. - 2011)

T‚¬ì#° : I) D ‹¬ O =`Ç ž ~¡ O ‹¬ O ö H Æ = ° Hê~¡ ¼ ãH› = ¶#° T‚¬ì#° : I) ‹¬g°„¬O…Õ H›O„¬î¼@~Ÿ ̄ī~¡…˜ž Ys^ΰ

K̈́¬\쓁x J#°HË=_ÈO …è^ΰ. ‹¬Þ^Íj =¶ï~ø\˜…Õ ̄~¡Q®=KÇ°ó.

II) ‹¬OöHÆ=° Hê~¡¼ãH›=¶° K̈́¬@“_¨xH÷ |_³û\˜ II) ‹¬Þ^Íj =¶ï~ø\˜^¥~¡°° D x~¡â†Ç¶xß

#O^ΰ x^Î°° `DŽ¬æx‹¬i =¼uö~H÷OKÇ=KÇ°ó.

6. ㄬH›@# : ㄬ ‹ ¬ ° ë ` Ç HêO…Õ U ^Í ‰ × O „¬ î ië Q ê ‹¬ Þ †Ç ° O 12. ㄬH›@# : =°Q®„²Á° Qêe„¬@O ZQ®°~¡°"͆Çò@䛽 Z䛽ø=

P^¥~¡"°
³ Ø …è^°
Î . (Civil Asst.Surgeon. - 2011) W+¬“„¬_È`Œ~¡°. (S.I. - 2007)

T‚¬ì#° : I) ^Í‰× J=‹¬~ŒxßO\ ÷x ̄O‡ÚOkOKÇ_ÈO, T‚¬ì#° : I) P_È „ ²  Á  ° ƒç=°à`Ë P_È ° @ä› ½ Zä› ½ ø=

L`Çæuë K͆ǰ@O Jª^Î¼O W+¬“„¬_È`Œ~¡°.

II) ^Í‰× ã„¬[° ‹¬~¡Þ ª^¥~¡}OQê |^ΜH›‹¬°ë°. II) P_Ȅ²Á° =°Q®„²Áä›O>è `³e"³á# "Œ~¡°.

REASONING ABILITY (LOGICAL) ○ ○ ○ ○ ○ ○ ○ ○


163
○ ○ ○ ○ ○ ○ ○ ○ ○ ○ ○ ○ ○ ○ ○ ○ ○ ○ ○
Study Material
1. ㄬH›@# : ㄬ[O^Î~¡¶ "Œi x["³°Ø# P^¥†Ç¶ä›½ II) ãHù`Ç ë Q ê ㇐~¡ O a O z# ï ~ á ˆ × ¤ ä› ½ ‹¬ i „¬ _ ¨

x*ì~òfQê „¬#°ßH›\ “÷ ㄬƒ°


’ `ÇÞO "Œ~¡° KÍÀ‹ qq^Î ㄬ†¶
Ç }©ä½
› ° =ô<Œß~¡° =°i†Çò ㄬƒ°
’ `ŒÞxH÷

Ja=$kœ Hê~¡¼ãH›=¶ä›½ ‹¬‚¬ìH›iOKŒx g\ ÷=Á #+¬“O~Œ^ΰ.

ㄬƒ’°`ÇÞO HËi#k. (Bank P.O. - 2005) 3. ㄬH›@# : XH› <Ë\ ©‹¬° : ''P_\Õi†Ç°O…ÕxH÷ ̋…˜‡¦é#°Á

T‚¬ì#° : I) ㄬƒ’°`ÇÞ HËiH› "Í°~¡ä›½ ㄬ[O^Î~¡¶ =°i†Çò À„[~Ÿ° J#°=°uOKÇ|_È=ô.

ㄬƒ°
’ `ŒÞxH÷ „¬#°ß° H›@_
“ O
È ã‡~¡Oaªë~°
¡ . P_\Õi†Ç°O…ÕxH÷ =KÇ°ó ㄬ[O^Î~¶
¡ "Œi =^Îí

II) ƒ’q+¬¼`Ç°ë…Õ ㄬƒ’°`ŒÞxH÷ „¬#°ß ~¡¶„¬O…Õ Q® ̋…˜‡¦é#°Á =°i†Çò À„[~¡°Á ‹²ÞKŸ P„¦¹ K͆Çò

=KÍó P^¥†Ç°O Q®}h†Ç°OQê ̄~¡°Q®°`Ç°Ok. ㇐~¡Ö#ÑÑ. (Bank P.O. - 2004)

2. ㄬH›@# : ï ~ á … è Þ ‰§Y"Œ~¡ ° ãHù`Ç ë Q ê „¬ k ï ~ á ˆ × ¤ #° u„¬ æ _È O T‚¬ì#° : I) ̋…˜‡¦é#°Á =°i†Çò À„[~¡°Á H›ey#

㇐~¡OaOKÇ_O
È `Ë ‡@° „¬^¥ß°Q®° ~
ï ˆ
á ¤
× ~ŒH›‡éH› "Œ~¡O^Î~¡¶ "Œi"Œi ‹Ô@Á#° KÍ~¡°ä›½<Í …Õ„¬ô

Y
‹¬OY¼#° ̄OKŒ~¡°. (I.B.P.S. - 2002) "Œ\ ÷x ‹²ÞKŸ P„¦¹ K͆Ƕe.

T‚¬ì#° : I)

M
J„¬æ\ ÷ =~¡ä½
› =ô#ß ~
ï ˆ
á Á × ‹¬OY¼ ㄬ†¶
Ç }©ä½
› #° II) ª^¥~¡}OQê ‹¬ƒ’ä›½ ‚¬ð[~¡†Í°¼ ㄬ[°

KÍ~"
¡ †
Í ò
Ç @䛽 `Çy# ‹¬OY¼…Õ …è=ô. ̋…˜‡¦é#°Á …è^¥ À„[~Ÿ° f‹¬°ä›½x "³ˆ×¤~¡°.

E
12345678901234567890123456789012123456789012345678901234567890121234567890123456789012345678901212345678901234567890123
1234567890123456789012 123456789012345678901234

q=~¡}ì`ÇàH› ["Œ|°°
12345678901234567890123456789012123456789012345678901234567890121234567890123456789012345678901212345678901234567890123
1234567890123456789012 123456789012345678901234
12345678901234567890123456789012123456789012345678901234567890121234567890123456789012345678901212345678901234567890123
1234567890123456789012 123456789012345678901234
12345678901234567890123456789012123456789012345678901234567890121234567890123456789012345678901212345678901234567890123
1234567890123456789012 123456789012345678901234
PREVIOUS BITS :

D
9
12345678901234567890123456789012123456789012345678901234567890121234567890123456789012345678901212345678901234567890123
1234567890123456789012 123456789012345678901234
12345678901234567890123456789012123456789012345678901234567890121234567890123456789012345678901212345678901234567890123
1234567890123456789012 123456789012345678901234

59
12345678901234567890123456789012123456789012345678901234567890121234567890123456789012345678901212345678901234567890123
1234567890123456789012 123456789012345678901234

CA
95
1. (4) T‚¬ì#O- I =°i†Çò T‚¬ì#O- II ï~O_ȶ ㄬH›@#`Ë 7. (4) ‹¬¶KÇ##° KÇkq J~¡O
Ö K͋°
¬ HË=KÇ°ó. Hêx J~¡"
Ö °
³ #
Ø O`Ç

50
‹¬O|O^ÎO …èx JO‰§…è. =¶ã`Œ# P „¬xx ‹¬=°~¡Ö=O`ÇOQê x~¡Þ‚²ìOKÇQ®~¡x

98
A
2. (4) T‚¬ì#O- I =°i†Çò T‚¬ì#O- II ï~O_ȶ ㄬH›@##° T‚²ìOKDžèO.
:8
L^¥ : XH› „¬ s H› Æ ~ŒÀ‹ =òO^Î ° q^¥¼~¡ ° Ö  O^Î ~ ¡ ¶
E

`Ç$„²„
ë ~
¬ K
¡ =
Ç ô. ZO^ΰH›O>è P~ËQ®¼=O`Ç°O^Î~¶
¡ Z䛽ø=
h
.P

‹¬=¶^¥<Œ° ~ŒÀ‹ =òO^ΰ ㄬ‰§ß„¬ã`ÇO…Õx ‹¬¶KÇ#hß


=°Ozh~¡ ° ã`Œy#@°Á K³ „ ¬ æ …è = ò. =°i†Ç ò `Ç ä › ½ ø=
IC
AD

KÇkq J~¡O
Ö K͋°
¬ HËQ®~¡°. Hêx P ‹¬¶KÇ# ㄬHê~¡"°
Í
=°Ozh~¡° ã`ŒQ®@O P~ËQê¼xH÷ Z…ìO\ ÷ ‚¬ðx K͆°
Ç ^Îh

JO^Î~°
¡ q^¥¼~¡°
Ö ° ‹¬=°~¡=
Ö O`ÇOQê „¬sH›#
Æ ° „¬îië K͆°
Ç …è~°
¡ .
AB
O

K³„¬æ…è=ò.

3. (2) `ÇO*ì=î~¡° #O^Í g}° `džǶ~¡°Kͪë~¡°. J#Qê "Íö~


8. (1) T‚¬ì#O- I ㄬH›@##° `Ç$„²ë „¬~¡°‹¬°ë#ßk. ㄬ‹¬°ë`ÇO
ER
CH

`DŽC
¬ ° …èä½
› O_¨ J<Ík ‹¬~
ï #
á T‚¬ìHê^ΰ. U„¬x†Ç°O^³<
á Œ
U ㄬ^͉×O…Õ "Œ\ ÷x `džǶ~¡° K͆ǰ~¡°. J<Í J~¡ÖO.
YD

=°#䛽 `³e†Ç°ä›½O_¨<Í ‡Ú~¡‡@°Á [~¡°Q®°`Œ~ò.


Hêx `ÇO*ì=î~¡° =ô#ß^Í g}° `džǶ~¡°K͆ǰ_¨xH÷

L^¥ :
,H

XH› ãH÷ïH\˜ =¶¼KŸ…Õ XH›ªi F_‡é~ò# [@°“


J<Í T‚¬ì J~¡Ö~¡‚²ì`ÇO. ZO^ΰH›#Qê `ÇO*ì=î~¡°…Õ
T
AM

`Ç=° …Õ‡#° `³°‹¬°ä›½#ßO`Ç =¶ã`Œ# =°m¤ Jk


g}…è HêH› WOHê W`Ç~¡ =‹¬°ë=ô° 䛀_¨ `džǶ~¡°
H

F_‡é=_ÈO [~¡Q®^¥ ? Hêx Wzó# ㄬH›@#…Õ K͋²#


M

K͆ǰ=KÇ°ó.
AM

`DŽ¬C° #°O_ <Í~¡°óHË"Œe Jx =¶ã`Ç"Í° =ô#ßk.


I II
IG

4. (4) T‚¬ì#O- =°i†Çò T‚¬ì#O- ° ï~O_ȶ Wzó#

9. (2) T‚¬ì#O- I `DŽ¬C. ZO^ΰH›#Qê „²Á =†Ç°‹¬°ž X


KH

ㄬH›@##° `Ç$„²ë„¬~¡KÇ=ô. ZO^ΰH›#Qê ‹¬Ow`Ç"Í`Ç끰

‹¬OII° ^¥\ ÷#O`Ç =¶ã`Œ# ‡~”¡‰§…ÕxH÷ ~Œä›½O_¨


R

HêH›‡é~ò<Œ =°#‰§ÅOu`Ë …èx"Œ~¡° L<Œß~¡°. "ŒiH÷

K͆ǰ@O ZH›ø_¨ [~¡Q®^ΰ. T‚¬ì#O- II ㄬH›@##°

J<ÍH› ~¡Hê ‹¬=°‹¬¼° =ôO\ì~ò. J…ìöQ ‹¬Ow`Ç

`Ç$„²ë„¬~¡°‹¬°ëOk.

‰§GAýO^ÎiH© ‹¬=°‹¬¼° =ôO_È=<Ín ‹¬~


ï #
á T‚¬ì Hê^ΰ.
10. (4) T‚¬ì#O -I =°i†Çò T‚¬ì#O- II ï~O_ȶ ㄬH›@##°

5. (2) T‚¬ì#O- I `Ç$„²ëHê^ΰ. ZO^ΰH›#Qê ‹¬OöHÆ=°


`Ç$„²ë„¬~¡KÇ=ô. J`ǼkH› ㄬ[° q=¶#KŒsû#°

Hê~¡¼ãH›=¶° `ǁ̄\쓁x J#°ä›½<Œß |_³û\˜ x^Î°°


ƒi
’ OKDž~
è °
¡ =°i†Çò |°‹¬°ž#° ZO„²H› K͋°
¬ 䛽#ß@°ÁQê

…èH›‡é`Í P „¬xK͆ǰ~¡°. Hê|\ ÷“ T‚¬ì#O- II q=¶<Œ#° Z=~¡¶ (ㄬ[°) ZO„²H› K͋¬°HË~¡°.

ㄬH›@##° `Ç$„²ë „¬~¡°‹¬°ëOk.


11. (3) T‚¬ì#O- I =°i†Çò T‚¬ì#O - II ï~O_ȶ ㄬ@#`Ë

6. (1) Z…ìO\ ÷ =°^Îí`Ç° P^¥~¡O …è䛽O_¨ Z=~¡¶ U „¬xx


`Ç$„²ë„¬~¡KÍ"Í.

„¬îiëQê "Œ~¡O`Ç@ "Œö~ K͆ǰ…è~¡°. Hê=ô# T‚¬ì#O-


12. (4) T‚¬ì#O- I =°i†Çò T‚¬ì#O- II ï~O_ȶ ㄬH›@#`Ë

I ㄬH›@##° `Ç$„²ë„¬~¡°‹¬°ëOk. Hêx T‚¬ì#O- II `DŽ¬C.


‹¬O|O^ÎO…è#q.

REASONING ABILITY (LOGICAL) ○ ○ ○ ○ ○ ○ ○ ○


164
○ ○ ○ ○ ○ ○ ○ ○ ○ ○ ○ ○ ○ ○ ○ ○ ○ ○ ○
Study Material
3 ㄬH›@#°
ㄬH›@#°

(Statements and Arguments)


=°i†Çò
=°i†Çò
"Œ^Î#°
"Œ^Î#°

=òMì¼O‰§°...
G Jƒ’¼iÖ `ù°`Ç "Œ¼Y¼#° ä›Æ½}âOQê „¬ijeOz, ‹¬O|Ok`Ç "Œ^Î##° P "Œ¼Y¼ P^¥~¡OQê Q®=°xOz, U "Œ^Î# P JO‰§xH÷
‹¬iQê¾ ‹¬i‡é=ô<Ë x~¡â~òOKŒe.
G |"³°Ø# "Œ^Î#°, =òY¼"³°Ø#q =°i†Çò ㄬ‰×ß䛽 ‹¬¶\÷Qê ‹¬O|OkOz#q.
G

Y
|‚Ôì#"³°Ø# "Œ^Î#° ㇐=òY¼`Ç …èxq, ㄬ‰×ß䛽 =ôO@¶, <Œ=°=¶ã`DŽ¬ô "Œ^Î#°Qê =ôO\ì~ò.
G =òY¼"³°Ø# ㄬ‰×߁ q+¬†Ç°=ò…Õ, x~¡â†Ç¶xH÷ =òO^ΰQê |"³°Ø# =°i†Çò |‚Ôì#"³°Ø# "Œ^Î# =°^Î¼ Q® `Í_¨#°

M
Q®°iëOKÇQ®Qêe.

E
G D `Ç~¡‚¬ð ㄬ‰×߁…Õ, ~Œ[H©†Ç°, ªO„¦²°H› …è^¥ PiÖH› JO‰§ä›½ ‹¬O|OkOz# XH› "Œ¼Y¼ ^¥xH÷ ‹¬O|OkOz Hùxß

D
9
J#°ä›€ =°i†Çò Hùxß ã„¬u䛀 "Œ^Î#° W=Þ|_È°`Œ~ò.

59
G CA
95
=°# =¼H÷ëQ®`Ç Jaã‡†Ç¶#° ㄬH›ø#̄\÷“, W=Þ|_# "Œ^Î# P^¥~¡OQê<Í ["Œ|°#° Q®°iëOKŒe.

50
=¶ki ㄬ‰×߁° 98
A
:8
E
h

"Œ^Î# I :
.P

MODEL - 1
IC

…è^°
Î . H›O„¬î¼@sH›~}
¡ 䛽 KŒ…ì Y~¡ó=ô`Ç°Ok.
AD

JO`Ç ªÚ=òà =$^Š¥ K͆ǰ䛀_È^ΰ.

G "Œ^Î# II :
AB
O

J=ô#°. Ja = $kœ K³ O k# ^Í ‰ §hß "Œi


ㄬu ㄬH›@# ãH÷O^Î ï~O_È° "Œ^Î#° I =°i†Çò II

„¬iã‰×=°#° H›O„¬î¼@sH›iOz#„¬ô_È° =°#O


ER

W=Þ|_È`Œ~ò.
CH

["Œ|°#° D q^ÎOQê Q®°iëOKŒe. =¶ã`ÇO ZO^ΰ䛽 K͆ǰ䛀_È^ΰ? =°#O


YD

1) öH=O "Œ^Î# I =¶ã`Ç"Í° |"³°Ø#k. ZO^ΰ䛽 "³#H›|_¨e ?


,H

2) öH=O "Œ^Î# II =¶ã`Ç"Í° |"³°Ø#k. ª^Î#: "Œ^Î# I =°i†Çò "Œ^Î# II ï~O_ȶ |"³°Ø#q Hê^ΰ.
T
AM

3) "Œ^Î# I =°i†Çò "Œ^Î# II ï~O_ȶ |"³°Ø#"Í. ZO^ΰH›#Qê Ja=$kœ „¬^ŠÎO…Õ #_KÍ@„¬ô_È° Y~¡°ó䛽
H
M

4) "Œ^Î# I =°i†Çò "Œ^Î# II ï~O_ȶ |"³°Ø#q Hê=ô. "³#°Hê_È䛀_È^ΰ. JO`ÇY~¡°ó ̄@“…èx À„^Î^͉×O Hê^ΰ
AM
IG

5) "Œ^Î# I …è^¥ "Œ^Î# II…Õ U^ËXH›\ ÷ |"³°Ø#k. =°#^͉×O.

1. ㄬH›@# :
KH

"Œ^Î# II 䛀_¨ |"³°Ø#k Hê^ΰ. ZO^ΰH›#Qê "Í~ùH›


ƒ ì ~¡ ` Ç ^ Í ‰ × O …Õ =°ixß "³ ° _ H › … ˜ Hê…è r °
R

^͉§#° KǶ‹² =°#O "³O|_OKŒež# J=‹¬~¡O …è^ΰ.


ªÖ„²OKŒ…ì ?

"Œ^Î# I : J=ô#°. _¨H›“~Ÿž ㄬ[ x+¬æuë ^Î$‘“ ¼ KŒ…ì


=°# q"ÍH›O, †ÇòH÷ë, Ja=$kœ „¬^ŠÎ =¶~Œ¾° =°#O

‹¬ÞO`ÇOQê H›e¾†ÇòO_¨e. Hê=ô# ‹¬=¶^¥#O (4).


=°Ok x‘â`Ç°…ÿá# _¨H›“~¡°Á J=‹¬~¡O =ôOk.

"Œ^Î# II :
MODEL - 2
…è^°
Î . ZO^ΰH›O>è W„¬æ\H
ö÷ ªÖ„O
² z# Hê…èr…ÕÁ

`Çy#O`Ç ª=¶ãy …è^°


Î . =òO^ΰ =ô#ß"Œ\ x
÷

Ja=$kœK͋² `Ç~¡°"Œ`Ç Hù`Çëq ªÖ„²OKŒe. G ‹¬¶KÇ#° : D ãH÷Ok ㄬ‰ß


× ä›½ ‹¬O|OkOz ㄬu ㄬ‰ß
× ä›½

ª^Î#: ï~O_È° "Œ^Î#¶ |"³°Ø#"Í. U^³á<Œ f‹²Hù#=KÇ°ó#°. XH› "Œ¼Y¼, I, II, III =°i†Çò IV J#° #O|~¡°Á W=Þ|_#

"Œ^Î# I =°i†Çò "Œ^Î# II ï~O_ȶ ‹¬i†³Ø°#"Í. Hê=ô#


"Œ^Î#° =ôO\ì~ò. g°~¡° U "Œ^Î# …è^¥ "Œ^Î#°

|"³°Ø#KË …è^¥ |‚Ôì#"³°Ø#"Ë =°i†Çò J‹¬æ+¬“"³°Ø#"Ë


‹¬=¶^¥#O (3).

2. ㄬH›@# : WO_†Ç¶…Õx „¬iã‰×=°xßO\ ÷h


Q®°iëOKŒe. W=Þ|_# ㄬ`Œ¼=¶ß†Ç¶…Õ ‹¬ï~á#

‹¬=¶^¥<Œxß ZO„²H› K͆Ƕe.


H›O„¬î¼@sH›~¡}O K͆Ƕ…ì ?

REASONING ABILITY (LOGICAL) ○ ○ ○ ○ ○ ○ ○ ○


165
○ ○ ○ ○ ○ ○ ○ ○ ○ ○ ○ ○ ○ ○ ○ ○ ○ ○ ○
Study Material
1. ㄬH›@# : ‹¬"³°à°, Tö~yO„¬ô° "³ò^΁Q®° Jxß ~¡Hê…ÿá#
2. ㄬH›@# : Jxß "Í°<Í*˜"°
³ O\˜ ‹¬O‹¬
Ö #° ㄬƒ°
’ `ÇÞ x†Ç°Oã`Ç}

…ÕxH÷, „¬ik…ÕxH÷ f‹¬°Hùx ~Œ"Œ…ì?


"Œ\ ÷x ㄬƒ’°`ÇÞ=ò |‚²ì+¬øiOKŒ…ì ?

"Œ^Î#° : I)
"Œ^Î#° :
Hê^ΰ. W@°=O\ ÷ ‹¬O‹¬
Ö #° ‹¬=°~¡=
Ö O`ÇOQê

I) J=ô#°. W…ì K͆ǰ_ÈO ^¥Þ~Œ L^˼Q®°#°


x~¡Þ‚²ìOKÇ_¨xH÷ Hê=‹²# x^°
Î ° ㄬƒ°
’  `ÇÞ=ò

ãH›=°tH›Æ}…Õ LOKÇQ®=ò.
=^Îí …è=ô.

II) Hê^ΰ. ‡Ï~¡° =°¿eH› ‚¬ì䛽ø#° ㄬƒ°


’ `ÇÞ=ò II) Hê^ΰ. `Ç#^³á# su…Õ „¬xKÍÀ‹ ‹¬Þ`ÇOã`Ç=ò

=°i†Çò À‹ÞKÇó ㄬu ‹¬O‹¬Ö䛽 LO_¨e.


‚¬ìiOKÇ~Œ^ΰ.

III) J=ô#°. W…ì K͆ǰ_È=ò ^¥Þ~Œ q^¥¼~¡°Ö


III) J=ô#°. W…ì K͆ǰ_ÈO ^¥Þ~Œ „¬x Xuë_

O^ÎiH© L#ß`Ç ã„¬=¶}ì`Ë q^¥¼ƒÕ^Î#

„¬_È䛽O_¨ J`ǼkH› L`Œæ^ÎH›`Ç ªkOKÍ


[~¡°Q®°`Ç°Ok.

J=Hê‰×=ò H›^ΰ.
IV) J=ô#°. W…ì KÍ À ‹ë < Í q^¥¼ ㄬ = ¶}쁰

Y
ª^Î#: ‹¬"³°à° ãH›=°tH›Æ}ì ~Œ‚²ì`Œ¼xß ‹¬¶zOKÇ=ô. g\ ÷ ^¥Þ~Œ "³°~¡°Q®°„¬_È`Œ~ò.

ª^Î#:

M
‡Ï~¡°ä›½ "Í°° [iöQ@@Á~ò`Í, W…ìO\ ÷ ‹¬O‹¬
Ö x~¡Þ‚¬ì}
㉧q°ä›½°, †Ç¶[=¶#¼=ò †³òH›ø ^˄²_ q^¥<Œ #°O_

Hù~¡ä½
› x^°
Î #° ‹¬g°H›iOKÇ_=
È ò ㄬƒ°
’ `ÇÞ=ò K͆°
Ç Q®^ΰ.
II

E
`Ç=°#° ~¡HO
Æ÷ Kǰ䛽O\ì~¡°. H›#°H› = "Œ^Î# ‹¬i‡é`Ç°Ok.

H›#°H› I = "Œ^Î# |"³°Ø#k Hê^ΰ. II = "Œ^Î#

D
9
Hêx = "Œ^Î# J‹¬æ+¬“OQê =ôOk. J…ìöQ Hùxß
q‰×ދ¬h†Ç°OQê …è^ΰ. JO`ÍHêH› ㄬƒ’°`ÇÞ x~¡Þ‚¬ì} =Á

59
‹¬O^Î~ŒÄ…Õ z#ß z#ß q+¬†Ç¶ Hù~¡ä›½ KÍÀ‹ ‹¬"³¶à°,

CA
95
=¶ã`Ç " Í ° D ‹¬ O ‹¬ Ö  f~¡ ° "³ ° ~¡ ° Q® ° =ô`Ç ° O^Î x , q^¥¼

Tö ~ yO„¬ ô ° ㄬ u |O^ Î H ꁰQê =¶i L`Œæ^Î H › ` Ç # ° ㄬ=¶}쁰 ̄~¡°Q®°`Œ†Ç°x K³„¬æ…è=ò. H›#°H›, III =

50
`Çy¾ªë~ò. H›#°H› III = "Œ^Î# 䛀_¨ ‹¬i†³Ø°#^Í.
=°i†Çò IV = "Œ^Î#° x°=…è=ô.

98
A
123456789012345678901234567890121234567890123456789012345678901212
12345678901234567890123456789012123456789012345678901234567890121234567890123456789012345678901212345678901234567890123456789012123
1234567890123456789012345678901212345678901234567890123456789012123456789012345678901234567890
12345678901234567890123456789012123456789012345678901234567890121234567890123456789012345678901212345678901234567890123
1234567890123456789012345678901212345678901234567890123456789012123456789012345678901234567890121234567890123
1234567890123456789012345678901212345678901234567890123456789012123456789012345678901234567890121234
:8
123456789012345678901234567890121234567890123456789012345678901212
12345678901234567890123456789012123456789012345678901234567890121234567890123456789012345678901212345678901234567890123456789012123
1234567890123456789012345678901212345678901234567890123456789012123456789012345678901234567890
12345678901234567890123456789012123456789012345678901234567890121234567890123456789012345678901212345678901234567890123
1234567890123456789012345678901212345678901234567890123456789012123456789012345678901234567890121234567890123
1234567890123456789012345678901212345678901234567890123456789012123456789012345678901234567890121234
12345678901234567890123456789012123456789012345678901234567890121234567890123456789012345678901212345678901234567890123456789012123
1234567890123456789012345678901212345678901234567890123456789012123456789012345678901234567890
12345678901234567890123456789012123456789012345678901234567890121234567890123456789012345678901212345678901234567890123
1234567890123456789012345678901212345678901234567890123456789012123456789012345678901234567890121234567890123
1234567890123456789012345678901212345678901234567890123456789012123456789012345678901234567890121234
123456789012345678901234567890121234567890123456789012345678901212
PRACTICE TEST
12345678901234567890123456789012123456789012345678901234567890121234567890123456789012345678901212345678901234567890123456789012123
1234567890123456789012345678901212345678901234567890123456789012123456789012345678901234567890
12345678901234567890123456789012123456789012345678901234567890121234567890123456789012345678901212345678901234567890123
1234567890123456789012345678901212345678901234567890123456789012123456789012345678901234567890121234567890123
1234567890123456789012345678901212345678901234567890123456789012123456789012345678901234567890121234
12345678901234567890123456789012123456789012345678901234567890121234567890123456789012345678901212345678901234567890123456789012123
1234567890123456789012345678901212345678901234567890123456789012123456789012345678901234567890
123456789012345678901234567890121234567890123456789012345678901212
E

12345678901234567890123456789012123456789012345678901234567890121234567890123456789012345678901212345678901234567890123
1234567890123456789012345678901212345678901234567890123456789012123456789012345678901234567890121234567890123
1234567890123456789012345678901212345678901234567890123456789012123456789012345678901234567890121234
12345678901234567890123456789012123456789012345678901234567890121234567890123456789012345678901212345678901234567890123456789012123
1234567890123456789012345678901212345678901234567890123456789012123456789012345678901234567890
123456789012345678901234567890121234567890123456789012345678901212
12345678901234567890123456789012123456789012345678901234567890121234567890123456789012345678901212345678901234567890123
1234567890123456789012345678901212345678901234567890123456789012123456789012345678901234567890121234567890123
1234567890123456789012345678901212345678901234567890123456789012123456789012345678901234567890121234
h

12345678901234567890123456789012123456789012345678901234567890121234567890123456789012345678901212345678901234567890123456789012123
1234567890123456789012345678901212345678901234567890123456789012123456789012345678901234567890
12345678901234567890123456789012123456789012345678901234567890121234567890123456789012345678901212345678901234567890123
1234567890123456789012345678901212345678901234567890123456789012123456789012345678901234567890121234567890123
1234567890123456789012345678901212345678901234567890123456789012123456789012345678901234567890121234
12345678901234567890123456789012123456789012345678901234567890121234567890123456789012345678901212345678901234567890123456789012123
1234567890123456789012345678901212345678901234567890123456789012123456789012345678901234567890
.P
IC

‹¬¶KÇ# (1-19) : 3. ㄬH›@# :


AD

ㄬu ㄬH›@# ãH÷O^Î ï~O_È° "Œ^Î#° I =°i†Çò =òOƒì~ò…Õ ãHù`Çë „¬iã‰×=°° ªÖ„²OKŒ…ì?

"Œ^Î#° :
AB
O

II W=Þ|_È`Œ~ò. I) J=ô#°. ãHù`ÇëQê „¬iã‰×=°° ªÖ„²OKÇ@O

["Œ|°#° D q^ÎOQê Q®°iëOKŒe.


ER
CH

=# KŒ…ì=°Ok x~¡°^˼Q®°ä›½ L^˼Q®

J=Hꉧ° H›eæOz#@Á=ô`Ç°Ok.
YD

1) öH=O "Œ^Î# I =¶ã`Ç"Í° |"³°Ø#k.

2) II
II) …è^°
Î . ^¥x=# W„¬æ\÷H
ö Hꁰ+¬¼ „¬îi`Ç"°
³ #
Ø
,H

öH=O "Œ^Î# =¶ã`Ç"Í° |"³°Ø#k.


T

=òOƒì~ò =°iO`Ç Hꁰ+¬ ¼ „¬ î i`Ç " ³ ° Ø


3) I II
AM

"Œ^Î# =°i†Çò "Œ^Î# ï~O_ȶ |"³°Ø#"Í.

ㄬ[ä›½ P<Œ~ËQ®¼ ‹¬=°‹¬¼#° ‹¬$+²‹


“ °
¬ O
ë k.
H

4) "Œ^Î# I =°i†Çò "Œ^Î# II ï~O_ȶ |"³°Ø#q Hê=ô.

4. ㄬH›@# :
M

Hê…èr° / †Çüx=iÅ\ ©…Õ q^¥¼iÖ ‹¬O„¦¬¶


AM

5) "Œ^Î# I …è^¥ "Œ^Î# II…Õ U^ËXH›\ ÷ |"³°Ø#k.


IG

1. ㄬH›@# : U~Œæ@° ‹¬=°O[‹¬"Í°<Œ ?


KH

ㄬƒ°
’ `˼^˼Q®°° K͆ò
Ç ‹¬"°
³ àÌ„á „¬îië xÀ+^O
Î

"Œ^Î#° : I) …è^ΰ. nx=# Hê…èr ㇐OQ®}O JO`Œ


R

qkOKŒ…ì ?

"Œ^Î#° :
~Œ[H© † Ç ° "Œ`Œ=~¡ } O <³  Hùx „²  Á  °

I) J=ô#°. nx=# ㄬƒ’°`˼^˼Q®°O^Î~¡¶

KÇ^ΰ=ô̄á ^Î$+²“ ̄@“…è~¡°.

ãH›=°tH›}
Æ `Ë "Œi"Œi „¬#°° K͋°
¬ 䛽O\ì~¡°.

II) J=ô#°. <Í\ ÷ q^¥¼~¡°Ö…è ö~„¬\ ÷ ~Œ[H©†Ç°

II) …è^ΰ. nx=# ㄬ[#° "Œi ㄬ*쪐Þ=°¼


<Œ†Ç°ä›½°. ^Í‰× ã„¬Qu
® "Œi g°^Í P^¥~¡„_
¬ 

‚¬ì䛽ø #°O_ ƒO


’ Q®O H›eyOz#@Á=ô`Ç°Ok.
=ôOk.

2. ㄬH›@# : ƒì~¡`^
Ç ‰
Í O
× †³òH›ø H÷„
Æ }
¬ ° Ja=$kœx PÀ„†Ç¶…ì?
5. ㄬH›@# : Kǁ# zã`Œ ㄬª~Œ#° xe„²"͆Ƕ…ì ?

"Œ^Î#° : I) J=ô#°. U.S.A. HË~¡°ä›½O@°#ßk 䛀_¨ J^Í


"Œ^Î#° : I) J=ô#°. Kǁ#zã`Œ#° gH÷ÆOKÇ_ÈO =#

Hê|\ ÷“ PÀ„†Ç¶e.
†Çò=`Ç Ì„_Èã`Ë= „¬@°“KÇ°#ßk.

II) …è ^ Î ° . ㄬ u ^Í ‰ × O ^¥x ~¡ H › Æ } KÇ ~ ¡ ¼ #°


II) …è^ΰ. ª^¥~¡} ㄬ[#° K³á`Ç#¼=O`Ç°#°

K̈́¬\ì“e. ‹Ôކǰ~¡H›Æ} ㄬ^ŠÎ=° H›~¡ë=¼O.


K͆Çò@䛽 W^ùH›ø>è =¶~¡¾O.

REASONING ABILITY (LOGICAL) ○ ○ ○ ○ ○ ○ ○ ○


166
○ ○ ○ ○ ○ ○ ○ ○ ○ ○ ○ ○ ○ ○ ○ ○ ○ ○ ○
Study Material
6. ㄬH›@# : 䛀$~¡ =°$Qê#° "Í\ì_È@O „¬îiëQê xÀ+kOKŒ…ì? "Œ^Î#° : I) …è ^ Î ° . "Œix tH÷ Æ O KÇ ä › € _È ^ Î ° . „¦ Ô A ä› ½

"Œ^Î#° : I) J=ô#°. "Í\ì_Í"Œ~¡° "Œ\ ÷ ^¥Þ~Œ ƒìQê =°#"Í° ‹¬‚¬ð†Ç°O K͆Ƕe.

^Î#=O`Ç°° J=ô`Ç°<Œß~¡°. II) J=ô#°. nx=# =¶ã`Ç"Í° x~¡H›Æ~Œ‹¬¼`Ç#°

II) …è^ΰ. 䛀$~¡ =°$Qê#° "Í\ì_È@O =°i†Çò x~¡¶àeOKÇ=KÇ°ó#°.

|OkOKÇ_ÈŌá xÀ+^ÎO qkOKÇ_ÈO JO`Ç 14. ㄬH›@# : Z=ï~`


á Í L^͉
í ¼
× „¬î~¡ÞH›OQê Hê䛽O_¨ J#°HË䛽O_¨

L„¬†³¶Q®H›~¡O Hê^ΰ. <Í~¡O Kͪë~Ë "ŒiH÷ ª^¥~¡} ㄬ[…ìQê<Í

7. ㄬH›@# : ƒì~¡`Ç^͉×O…Õ ‡~”¡‰§…ÕÁ Lz`Ç q^μ#° <Œ¼†Ç°O K͆Ƕ…ì ?

JOkOKŒ…ì ? "Œ^Î#° : I) J=ô#°. ZO^Î ° H› O >è ƒì^ Î ¼ `Ç Q ®  ㄬ [ °

"Œ^Î#° : I) J=ô#°. nx=# JH›Æ~Œ‹¬¼`Ç ‰§`ÇO L^Íí‰×¼„¬î~¡ÞH›OQê <Í~Œ° K͆ǰ~¡°.

Q®}}©†Ç°OQê ̄~¡°Q®°`Ç°Ok. II) Hê^ΰ. KÇ@“O =òO^ΰ JO^Î~¡¶ ‹¬=¶#"Í°.

II)

Y
…è^°
Î . W„¬æ\ H
ö÷ ƒì~¡OQê =ô#ß ^Í‰× Y*ì<ŒÌ„á <Í~¡O Uq^ÎOQê K͋²<Œ <Í~¡"Í°.

=°iO`Ç ƒi
’ OKDžx
è ƒì~¡O "³¶„²#@Á=ô`Ç°Ok. 15. ㄬH›@# : ^͉O
× …Õx ~Ë_Èx
Á ßO\ h
÷ J#°‹¬O^¥#O K͆¶
Ç …ì?

M
8. ㄬH›@# : ƒ ì ~¡ ` Ç ^ Í ‰ × O …Õ x~¡ H › Æ ~ Œ‹¬ ° ¼ F@° ‚¬ ì ä› ½ ø#° "Œ^Î#° : I) …è^ΰ. nx=# ãQê=¶…Õx ㄬ‰§O`Ç`Ç

E
`ùyOKŒ…ì ? ^³|ÄuO@°Ok.

"Œ^Î#° : I) II)

D
9
J=ô#°. "Œ~¡° `ÇÞ~¡Qê ㄬ…Õƒìä›½ …Õ<³á J=ô#°. J…ì K͆ǰ_ÈO =# ㄬ†Ç¶}÷OKÍ

59
`DŽ¬C^¥i…Õ _È|°Ä䛽 P‰×„¬_ F@° "ͪë~¡°. "ŒiH÷ ªÏƒ’¼OQê =ôO@°Ok.

CA
95
II) Hê^ΰ. F@° "͆°
Ç _ÈO ~Œ*ì¼OQ®|^Î"
œ °
³ #
Ø [#à 16. ㄬH›@# : ãQêg°} ㇐O`Œ…Õ 䛽\ ©~¡ „¬iã‰×=°#°

50
‚¬ì䛽ø. ã‡é`Ǟ‚²ìOKŒ…ì ?

9. ㄬH›@# : "Œ^Î#° : 98
A
H›~Ÿ T.V. Ì„á qk O KÇ | _È ° tax #° WOHê I) J=ô#°. ãQêg°} ㇐O`Ç ã„¬[° ‹¬$[<Œ`ÇàH›`Ç
:8
̄OKŒež# J=‹¬~¡O =ôO^¥ ? H›"Œ~¡°.
E

"Œ^Î#° :
h

I) J=ô#°. H›~Ÿ T.V. XH› Q®s


û =‹¬°=
ë ô. H
ö =O II) J=ô#°. nx=# x~¡ ° ^˼Q® ‹¬ = °‹¬ ¼ #°
.P
IC

^#
Î =O`Ç°° =¶ã`Ç"°
Í "Œ\ x
÷ Hù#Q®~¡°. HùO`Ç=~¡ä›½ `Çy¾OKÇ=KÇ°ó.
AD

II) …è^ΰ. H›~Ÿ T.V. #° À„^Î"Œ~¡° 䛀_¨ 17. ㄬH›@# : q^͉§…Õ „¬xK͋¬°ë#ß ƒì~¡f†Ç° ‰§G"Í`Çë#°
AB
O

Hù#Q®~¡°. ‹¬Þ^͉§xH÷ f‹¬°ä›½x ~Œ"Œ…ì ?


ER

10. ㄬH›@# : "Œ^Î#° :


CH

̋H±ž Z_È°¼öH+¬<£#° ‡~”¡‰§…Õ ㄬ"͉×̄\쓅ì? I) J=ô#°. "Œ~¡ ° Pq+¬ ø ~¡ } °, QÒ~¡ " Œ°,

"Œ^Î#° :
YD

I) J=ô#°. Jxß Ja = $kœ K³ O k# ^Í ‰ §° ªÏH›~Œ¼° Jhß =^ΰ°ä›½x =òO^ΰ `Ç=°
,H

J=°° K͋¬°ë<Œß~ò. Hê|\ ÷“ =°#O 䛀_¨ =¶`Ç$ƒ’¶q°H÷ À‹=K͆Ƕe.


T

II)
AM

J=°° K͆Ƕe. Hê^ΰ. =°#䛽 ‹¬i„¬_¨ ㄬuƒ’ =ô#ßk. "Œi


H

II) …è ^ Î ° . HË-Z_È ° ¼ö H +¬ < £ ‡~” ¡ ‰ §…ÕÁ nxx W‘“xH÷ J#°Q®°}OQê "Œix LO_Èx"ŒÞe.
M

18. ㄬH›@# :
AM

J=°° K͆ǰ䛀_È^ΰ. ƒì~¡`^


Ç ‰
Í O
× …Õx =òY¼ #Q®~Œ…Õ 15 ‹¬OII° "Œ_ÈHO
›
IG

11. ㄬH›@# : ƒì~¡`Ç^͉×O…Õ L#ß`Ç q^μ#° WOwÁ+¬µ ^¥\ #


÷ "Œ‚¬ì<Œ#° "Œ_Èä½
› O_¨ x~ËkOKŒ…ì?
KH

g°_†Ç°O…Õ J=°° K͆Ƕ…ì ? "Œ^Î#° : I) J=ô#°. nx=# "Œ`Œ=~¡} Hꁰ‘¼xß


R

"Œ^Î#° : I) J=ô#°. WOQ®ÁO_È°, U.S.A. =O\ ÷ Ja=$kœ KŒ…ì =~¡ä›½ x"ŒiOKÇ=KÇ°ó#°.

K³Ok# ^͉§…Õ q^μ WOwÁ+µ


¬ g°_†°
Ç O…Õ<Í II) …è^ΰ. "Œ‚¬ì# qx†³¶Q®^¥~¡°° "Íö~

J=°° K͋¬°ë<Œß~¡°. ㇐O`Œä›½ "³o¤ ‹¬~


ï #
á L^˼Q®O KǶ‹¬°ä›½x

II) J=ô#°. ㄬ„¬OKÇ "Œ¼„¬ëOQê Z䛽ø= =°Ok rqOKÇ_ÈO KŒ…ì H›+¬“O.

=¶\ìÁ_È°`Ç°#ß ƒì+¬ WOwÁ+¬µ. 19. ㄬH›@# : ƒìs „¬iã‰×=°…Õ ƒìQê Z`³åë# ‡ÚQ® Qù\쓁#°

12. ㄬH›@# : WO_†Ç¶…Õ =°`Œ#° x~ËkOKŒ…ì ? U~Œæ@° K͆Ƕ…ì ?

"Œ^Î#° : I) J=ô#°, nx=# ㄬ[° H›e‹² =ôO\ì~¡°. "Œ^Î#° : I) J=ô#°. nx=# ƒ¶
’ ªÖ~ò =^Îí Hꁰ‘¼xß

II) …è^°
Î . =°`ÇO ㄬ[#° H›e‹² =ôO_ͅì K͋°
¬ O
ë k. KŒ…ì=~¡ä›½ `Çy¾OKÇ=KÇ°ó#°.

13. ㄬH›@# : „²Á#° ‡~”¡‰§ä›½ „¬O„¬x `ÇeÁ^ÎOã_È°#° II) Hê^ΰ. nx=# ̄á "Œ`Œ=~¡}O…Õ Hꁰ+¬¼O

tH÷ÆOKŒ…ì ? ̄~¡°Q®°`Ç°Ok.

REASONING ABILITY (LOGICAL) ○ ○ ○ ○ ○ ○ ○ ○


167
○ ○ ○ ○ ○ ○ ○ ○ ○ ○ ○ ○ ○ ○ ○ ○ ○ ○ ○
Study Material
1234567890123456789012
12345678901234567890123456789012123456789012345678901234567890121234567890123456789012345678901212345678901234567890123
12345678901234567890123
12345678901234567890123456789012123456789012345678901234567890121234567890123456789012345678901212345678901234567890123
1234567890123456789012 12345678901234567890123
12345678901234567890123456789012123456789012345678901234567890121234567890123456789012345678901212345678901234567890123
1234567890123456789012
1234567890123456789012
q=~¡}ì`ÇàH› ["Œ|°°
12345678901234567890123
12345678901234567890123456789012123456789012345678901234567890121234567890123456789012345678901212345678901234567890123
PRACTICE TEST :
12345678901234567890123
12345678901234567890123456789012123456789012345678901234567890121234567890123456789012345678901212345678901234567890123
1234567890123456789012
12345678901234567890123456789012123456789012345678901234567890121234567890123456789012345678901212345678901234567890123
1234567890123456789012
12345678901234567890123
12345678901234567890123
12345678901234567890123456789012123456789012345678901234567890121234567890123456789012345678901212345678901234567890123
1234567890123456789012 12345678901234567890123

1. (2) "Œ^Î# I |"³°#


Ø k Hê^ΰ. Wk T‚¬ð[x`ÇO. ZO^ΰH›#Qê 7. (2) Lz`Ç q^μ#° JOkOz#O`Ç =¶ã`Œ# JO^Î~¡¶

L^˼Q®° _=¶O_ÈÁ#° <³~¡"Í~¡óH›‡é`Í "Œ~¡° L^˼Qꁰ ‡~”¡‰§ä›½ =zó KÇkq JH›Æ~Œ‹¬¼`Ç Ì„~¡°Q®°^΁䛽

=¶<ÍÀ‹ J=Hê‰×O =ôOk. …è^¥ q^°


Î ° x~¡ÞiëOKÇä½
› O_¨ `Ë_È æ _È ~ ¡ ° . Wk T‚¬ ð `Ç à H› " ³ ° Ø # k. Hê=ô# "Œ^Î # I

䛀_¨ =ôO_È=KÇ°ó. ^¥x=Á P ‹¬O‹¬ÖöH #+¬“O. "Œ^Î# |"³°#


Ø k Hê^ΰ. Hêx ^͉O
× …Õx ‡~”‰
¡ §° JxßO\ …
÷ Õ

II |"³°Ø#k. ㄬ*쪐Þ=°¼ ^͉×O…Õ ㄬ[ ‚¬ì䛽øä›½ Lz`Ç q^μ#° JOkOKÇ_ÈO J<Ík Jª^Î¼O. \ ©KÇ~¡Á

ƒ’OQ®O H›eyOKÇ䛀_È^ΰ. "Œi ‹¬=°O[‹¬"³°Ø# _=¶O_ÈÁ r`Œ°, Jxß ‡~”¡‰§…ÕÁx ªÏH›~Œ¼° W…ìO\ ÷=hß

Hˋ¬O "Œ~¡° ‹¬"³°à K͆ǰ=KÇ°ó. ƒ’iOKÇ_ÈO Jª^Î¼O. Hê=ô# "Œ^Î# II |"³°Ø#k.

2. (2) "Œ^Î# I J~¡~


Ö ‚
¡ ì
² `ÇO. ZO^ΰH›O>è Z=~Ë HË~¡°ä›½O@°<Œß~¡x 8. (2) F@° "͆ǰ_ÈO 18 ^¥\ ÷# ㄬu =¼H÷ëH© =ôO_Í ‚¬ì䛽ø.

Y
=°# ~¡H}
ƛ KÇ~¼
¡ #° =°#O P„¬ä€
› _È^°
Î . ^¥xx `ùyOKÇ_ÈO J‹¬Oƒ’=O. Hê=ô# "Œ^Î# II

M
"Œ^Î# II |"³°#
Ø k. ㄬu^͉O
× , `Ç=° ‰×ã`Ç°^͉§ #°O_
|"³°Ø#k. "Œ^Î# I T‚¬ð`ÇàH›"³°Ø#k.

`Ç=°x`Œ=ò ~¡H÷ÆOKÇ°HË=_¨xH÷ P†Çò^¥°


9. (4) "Œ^Î# I =°i†Çò "Œ^Î# II ï~O_ȶ |"³°Ø#q Hê=ô.

E
H›ey†ÇòO_È@O `DŽ¬CHê^ΰ.
10. (4) "Œ^Î# -I =°i†Çò "Œ^Î# - II ï~O_ȶ |"³°Ø#q Hê^ΰ.

D
9
(3)

59
3. "Œ^Î# I =°i†Çò "Œ^Î# II ï~O_ȶ |"³°Ø#"Í.
̋䛽žä›½ ‹¬O|OkOz# JO‰§° ~¡‚¬ì‹¬¼OQê

CA
95
4. (3) "Œ^Î# I =°i†Çò "Œ^Î# II ï~O_ȶ |"³°Ø#"Í.
LOKÇ_ÈO=# „²Á…Õ ^¥x̄á `³e†Ç°x P‹¬H÷ë ̄iy

50
ZO^ΰH›#Qê HùO`Ç=°Ok q^¥¼~¡°
Ö ° <Œ†Ç°H›`Þ
Ç "³¶A…Õ
"Œ~¡° 䛀_¨ ^¥xx ~¡‚¬ì‹¬¼OQê<Í `³°‹¬°ä›½x K³_È°

98
A
„¬_ =~Œ¾°Qê U~¡æ_ KÇ^ΰ=ô#° „¬H›ø#ƒÿ\ ÷“ W`Ç~¡°`Ë
ㄬƒì"ŒxH÷ …Õ#=ô`Ç°<Œß~¡°. Hê=ô# ^¥xx 䛀_¨ XH›
:8
Qù_È=° „¬_ rq`ÇO <Œ‰×#O K͋°
¬ 䛽O@°<Œß~¡°. Hê=ô#
‹¬|û䛽“=…ÿ ƒÕkÀ‹ë ^¥xx 䛀_¨ ª^¥~¡} JO‰×O…ì<Í
E

I
h

"Œ^Î# |"³ ° Ø # k. J^Í q ^ Î O Qê q^¥¼iÖ ^Î ‰ × … Õ<Í


„¬iQ®}ª
÷ ë~°
¡ . JO`ÍHêx "Í~
ö ^͉§…Õ J=°°K͋°
¬ <
ë Œß~¡x
.P
IC

~Œ[H©†Ç¶Ì„á J=Qꂬì# U~¡æ_`Í ƒ’q+¬¼`Ç°ë…Õ Jk


=°#O KÍ † Ç ¶ ež# J=‹¬ ~ ¡ O …è ^ Î ° . =°i†Ç ò HË-
AD

ZO`ÇQê<Ë L„¬†³¶Q®„¬_È°`Ç°Ok. W@°=O\ ÷ q^¥¼~¡°Ö…è


Z_È°¼öH+¬<£ ‡~”¡‰§…ÕÁ J=°°K͆ǰ䛀_È^ΰ J#°#k
AB
O

^͉×~Œ[H©†Ç¶…Õ KÇ°~¡°Qê¾ ~Œ}÷ªë~¡°. Hê=ô# "Œ^Î#


䛀_¨ ‹¬iHê^ΰ.
ER
CH

II |"³°Ø#k.
11. (2) "Œ^Î# I |"³°Ø#k Hê^ΰ.
YD

5. (4) "Œ^Î# I H
ö =O T‚¬ð[x`ÇO. Kǁ#zã`Œ#° KǶ_È@O
ㄬ„¬OKÇ "Œ¼„¬ëOQê Z䛽ø==°Ok WOwÁ+¬µ…Õ<Í =¶\ìÁ_È°
,H

=# =°#O J<ÍH› q+¬†Ç¶#° <Í~¡°óHË=KÇ°ó#°. <³áuH›


`Ç°<Œß~¡°. =°i†Çò U L^˼QêxH÷ WO@~¡¶¼H÷ "³o¤<Œ
T
AM

q°=ä›½ ‹¬O|OkOz# J<ÍH› q+¬†Ç¶#° <Í~¡°óHË


WOwÁ+¬µ „¦¬îÁ†³°hž `DŽ¬æx‹¬i. Hê=ô# "Œ^Î# II
H

=KÇ°ó#°. JO`ÍQêH› Kǁ#zã`Œ° ^¥Þ~Œ ‚¬ð‹¬¼ƒi


’ `Ç"°
³ #
Ø
M

|"³°Ø#k.
AM

‹¬ ~ ¡ ^ ¥ ‹¬ x ß"Í ‰ §#° KÇ ¶ ‹² P#OkOKÇ = KÇ ° ó#°. P


(2)
IG

12. "Œ^Î# I J~¡Ö~¡‚²ì`ÇO. =°`Œ#° x~ËkÀ‹ë<Í =°#°+¬µ

KÇ`Ç°~¡`Ç#° ª^¥~¡} r=# ‰ÜáeH÷ `Ë_È°K͋² W`Ç~¡°#°


KH

=°^ Î ¼ JO`Ç ~ Œ° ̄~¡ ° Q® ° `Œ~ò. ZO^Î ° H› O >è =°`Ç

䛀_¨ #qÞOKÇ=KÇ°ó#°. Hê=ô# "Œ^Î# I |"³°Ø#k


R

q‰§Þª° W„¬æ\ ÷öH ̄H›oOKDžèx q^ÎOQê ‡`ǰ䛽

Hê^ΰ. ª^¥~¡} ㄬ[#°, „²


Á #° J<ÍH› =¶~Œ¾ ^¥Þ~Œ
‡é†Ç¶~ò. nx x~Ë^O
Î Jª^¼
Î O. "Œ^Î# II |"³°#
Ø k.

K³`
á #
Ç ¼=O`Ç°#° K͆°
Ç =KÇ°ó#°. Hê=ô# "Œ^Î# II 䛀_¨
XöH =°`ŒxH÷ K³Ok# "Œ~¡° JO^Î~¡¶ H›e‹²"³°e‹²

|"³°Ø#k Hê^ΰ.
=ôO@°<Œß~¡°. "Í~
ö =°`Œ "Œi`Ë ä›€_¨ À‹ß‚¬ìƒì=O`Ë

6. (4) "Œ^Î# I T‚¬ð[x`ÇO. Hê=ô# Wk |"³°Ø# "Œ^Î#


"³°°Q®°`Ç°<Œß~¡°.

Hê^ΰ.
13. (4) "Œ^Î# I =°i†Çò "Œ^Î# II ï~O_ȶ |"³°Ø#q Hê^ΰ.

"Œ^Î# II J~¡~
Ö ‚
¡ ì
² `ÇO. 䛀$~¡=°$Qê#° "Í\ì_È@O =#
Z=i PiÖ H › ªéÖ = °`Ç # °|\ ÷ “ "Œ~¡ ° KÇ ^ Î = _È O …è ^ ¥

ㄬH›$u ‹¬‚¬ì[ „¬~Œ¼=~¡} ‹¬=°`Ç°¼`Ç ^³|ÄuO@°Ok.


KÇ k qOKÇ H › ‡ é=_È O J<Í k =ôO@°Ok. ㄬ * 쪐Þ=°¼

W„¬æ\ H
ö÷ =°$Qꁰ…èx J_È=ô…Õ K³@#
Á ° #iH÷"‹
Í ² J@g

^͉×O…Õ Z=i P…ÕKÇ#° "Œiq. "Œ\ ÷x ƒ’OQ®„¬~¡KÇ

‹¬ O „¬ ^ Î # ° `Ç y ¾ ‹ ¬ ° ë < Œß~¡ ° . Hê=ô# "Œ^Î # II ä› € _¨

䛀_È^ΰ. J…ìöQ "Œ^Î# II T‚¬ð[x`ÇO =¶ã`Ç"Í°.

|"³°Ø#k Hê^ΰ.

REASONING ABILITY (LOGICAL) ○ ○ ○ ○ ○ ○ ○ ○


168
○ ○ ○ ○ ○ ○ ○ ○ ○ ○ ○ ○ ○ ○ ○ ○ ○ ○ ○
Study Material
14. (2) <Í~¡O Z…ì K͋²<Œ, K͋²# =¼H÷ë tHÆê~¡°Ý_Í. KÇ@“O =òO^ΰ =ô#ß"Œix Q®°iëOz "ŒiH÷ ‹¬ï~á# ã‡é`Œž‚¬ìHêOkOz

JO^Î ~ ¡ ¶ ‹¬ = ¶#"Í ° . U ‹¬ = °†Ç ° O…Õ#¶ q"Í H êxß =¶`Ç$^͉§xH÷ À‹=KÍÀ‹…ì ã‡é`Ǟ‚²ìOKŒe. ㄬuƒ’=ô#ß

H˅Õæ䛀_È^ΰ. ㄬu XH›ø~¡¶ "Œi =¶`Ç$^͉§xöH À‹=K͆¶


Ç <Í x|O^#
Î

"Œ^Î# I |"³°Ø#k Hê^ΰ. …è^ΰ.

"Œ^Î# II |"³°Ø#k. 18. (1) "Œ^Î#-I |"³°Ø#k. Z䛽ø= HêO "Œ_# "Œ‚¬ì<Œ°

15. (4) "Œ^Î# I T‚¬ð[x`ÇO. |"³°Ø#k Hê^ΰ. JkH› Hꁰ‘¼xß q_È°^΁Kͪë~ò. Hê=ô# J…ìO\ ÷

"Œ^Î# II |"³°Ø#k Hê^ΰ. W„¬æ\ ÷öH ㄬ†Ç¶} =¶~Œ¾° "Œ‚¬ì<Œ#° "Œ_Èä½


› O_¨ x~ËkOKÇ_O
È =# „¬~Œ¼=~¡}

U~Œæ@° KÍ ‹ ² = ô<Œß~ò. Yzó`Ç O Qê J#°‹¬ O ^ ¥ #O Hꁰ+¬¼O#° JiH›@_


“ O
È `ˇ@° ㄬ[° "Œ¼^°
Î  ƒìi#

K͆Ƕež# J=‹¬~¡"Í° …è^ΰ. „¬_È䛽O_¨ Hꇐ_#@Á=ô`Ç°Ok.

16. (2) "Œ^Î#-I |"³°Ø#k Hê^ΰ. "Œ^Î#-II J~¡Ö~¡‚²ì`ÇO. "Œ‚¬ì#O Hˋ¬O "Íö~㇐O`Œä›½

Y
"Œ^Î # - II |"³ ° Ø # k. nx=# x~¡ ° ^˼Q® ‹¬ = °‹¬ ¼ "³ˆ×¤#=‹¬~¡O …è^ΰ. ^¥xx J"Í°à‹² …è^¥ „¬H›ø# „¬_È"͋²

M
`ÇQ®°¾`Ç°Ok. ãHù`Çëq Hù#°Qˁ° K͆ǰ=KÇ°ó#°.

17. (4) "Œ^Î#- I |"³°Ø#k Hê^ΰ. ZO^ΰH›#Qê Z=iH÷ 19. (1) "Œ^Î#-I |"³°Ø#k. ƒ’¶ªÖ~ò "Œ`Œ=~¡} Hꁰ+¬¼O

E
#zó#KË@ "Œ~¡° „¬xKÍÀ‹ ‚¬ì䛽ø ㄬuXH›øiH© =ô#ßk. =òY¼OQê ㄬ[#° "Œ¼^Î°ä›½ Q®°iK͋¬°ëOk. nx=#

D
9
Hê=ô# "Œix |=O`ÇOQê ‹¬Þ^͉§xH÷ ~¡„²æOKÇ@O ‹¬ Þ KÇ ó  " ³ ° Ø # Qêex „Ô  °óä› ½ <Í J=Hê‰× O =ôO@°Ok.

59
CA "Œ^Î#-II

95
"Œi ‚¬ì䛽øä›½ ƒ’OQ®O H›eyOKÇ_È"Í°. P~ËQ®¼ ‹¬=°‹¬¼° `ÇQ®°¾=òYO „¬_È`Œ~ò.

" Œ ^ Î # - II |"³°Ø#k Hê^ΰ. ZO^ΰH›#Qê ㄬuƒ’ kÞf†Ç°ã‰õ}÷ "Œ^Î#.

50
123456789012345678901234567890121234567890123456789012345678901212
98
A
12345678901234567890123456789012123456789012345678901234567890121234567890123456789012345678901212345678901234567890123
1234567890123456789012345678901212345678901234567890123456789012123456789012345678901234567890121234567890123
1234567890123456789012345678901212345678901234567890123456789012123456789012345678901234567890121234
123456789012345678901234567890121234567890123456789012345678901212
12345678901234567890123456789012123456789012345678901234567890121234567890123456789012345678901212345678901234567890123456789012123
123456789012345678901234567890121234567890123456789012345678901212345678901234567890123
12345678901234567890123456789012123456789012345678901234567890121234567890123456789012345678901212345678901234567890123
1234567890123456789012345678901212345678901234567890123456789012123456789012345678901234567890121234567890123
1234567890123456789012345678901212345678901234567890123456789012123456789012345678901234567890121234
12345678901234567890123456789012123456789012345678901234567890121234567890123456789012345678901212345678901234567890123456789012123
123456789012345678901234567890121234567890123456789012345678901212345678901234567890123
123456789012345678901234567890121234567890123456789012345678901212
12345678901234567890123456789012123456789012345678901234567890121234567890123456789012345678901212345678901234567890123456789012123
123456789012345678901234567890121234567890123456789012345678901212345678901234567890123
12345678901234567890123456789012123456789012345678901234567890121234567890123456789012345678901212345678901234567890123
1234567890123456789012345678901212345678901234567890123456789012123456789012345678901234567890121234567890123
1234567890123456789012345678901212345678901234567890123456789012123456789012345678901234567890121234
PREVIOUS BITS
:8
123456789012345678901234567890121234567890123456789012345678901212
12345678901234567890123456789012123456789012345678901234567890121234567890123456789012345678901212345678901234567890123456789012123
123456789012345678901234567890121234567890123456789012345678901212345678901234567890123
12345678901234567890123456789012123456789012345678901234567890121234567890123456789012345678901212345678901234567890123
1234567890123456789012345678901212345678901234567890123456789012123456789012345678901234567890121234567890123
1234567890123456789012345678901212345678901234567890123456789012123456789012345678901234567890121234
12345678901234567890123456789012123456789012345678901234567890121234567890123456789012345678901212345678901234567890123456789012123
123456789012345678901234567890121234567890123456789012345678901212345678901234567890123
12345678901234567890123456789012123456789012345678901234567890121234567890123456789012345678901212345678901234567890123
1234567890123456789012345678901212345678901234567890123456789012123456789012345678901234567890121234567890123
1234567890123456789012345678901212345678901234567890123456789012123456789012345678901234567890121234
123456789012345678901234567890121234567890123456789012345678901212
12345678901234567890123456789012123456789012345678901234567890121234567890123456789012345678901212345678901234567890123456789012123
123456789012345678901234567890121234567890123456789012345678901212345678901234567890123
12345678901234567890123456789012123456789012345678901234567890121234567890123456789012345678901212345678901234567890123
1234567890123456789012345678901212345678901234567890123456789012123456789012345678901234567890121234567890123
1234567890123456789012345678901212345678901234567890123456789012123456789012345678901234567890121234
12345678901234567890123456789012123456789012345678901234567890121234567890123456789012345678901212345678901234567890123456789012123
123456789012345678901234567890121234567890123456789012345678901212345678901234567890123
E

1234567890123456789012345678901212345678901234567890123456789012123456789012345678901234567890121234567890123
1234567890123456789012345678901212345678901234567890123456789012123456789012345678901234567890121234
h
.P

G "Œ^Î# :
IC

ㄬu ㄬH›@# ãH÷O^Î ï~O_È° "Œ^Î#° I =°i†Çò II I J=ô#°. ZO^Î ° H› O >è Wk ‰§¢‹Ô ë † Ç ° †Ç ò Q® O
AD

W=Þ|_È`Œ~ò. =°i†Çò q^¥¼~¡°Ö° q*ìý# ‰§GO Q®°iOz

1. ㄬH›@# :
AB
O

ƒì~¡`Ç^͉×O…Õ ~Œ[H©†Ç° ‡s“#° `³°‹¬°Hù#=…ÿ#°.

"Œ^Î# :
ER
CH

x~¡¶àeOKŒ…ì ? (S.I. - 2012) II …è^ΰ. ZO^ΰH›O>è Jk ^Î$Q˾KÇ~¡=ò.

"Œ^Î# I :
YD

J=ô#°. Wk ~Œ[H©†Ç° <Œ†Ç°ä›½ä›½ =°Oz †Ç¶OãuH›=òQê ªy‡é=ô#°.

Q®°}‡~”¡O. 5. ㄬH›@# : ‡~”¡‰§…Õ 5= `Ç~¡Q®u =~¡ä›½ Q® q^¥¼~¡°Ö


,H

"Œ^Î# II : …è^ΰ. J…ì K͆ǰ_ÈO =# ㄬ*쪐Þ=°¼O O^Î i H© ‹¬ O =`Ç ž ~ŒO`Ç ~ ¡ „¬ s H› Æ  #°O_
T
AM

JO`Ç=°=ô`Ç°Ok. q°#‚¬ð~òO„¬ô W"ŒÞ…ì ?


H

2. ㄬH›@# :
M

q^¥¼†Ç¶° =°i†Çò H›ˆì‰§…Õ ̋áxH› (Bank PO Exam - 2009)

"Œ^Î# :
AM
IG

tH›Æ}䛽 x~¡ÄO^Î=òQê qkOKÇ=KÇ°ó<Œ ? I J=ô#°. nx=# À„ï~O\˜žÌ„á ƒì~¡O `ÇQ°


® `
¾ °
Ç Ok.

(S.I. - 2008) "Œ^Î# II :


KH

…è^ΰ. nx=# L‡^¥¼†Çò° L^¥‹Ô#°…ÿá

"Œ^Î# I :
R

J=ô#°. J@°=O\ ÷ tH›Æ} q^¥¼~¡°Ö#° ãH›=° LO\ì~¡°.

tH›Æ}Qê =ôOKÇ°#°. 6. ㄬH›@# : ƒì~¡`Ç^͉×O…Õx =¼=ª†Ç¶xß

"Œ^Î# II : …è^ΰ. J\ ÷“ tH›Æ} q^¥¼~¡°Ö#° KÇ^ΰ=ô #°O_ †Ç¶OãfH›iOKŒ…ì? (MAT- 2004)

^Î$+²“ =°o¤OKÇ°#°. "Œ^Î# I : J=ô#°. nx=# L`Çæuë JkH›OQê

3. ㄬH›@# : WO_†¶
Ç …Õ ‡¦¼+¬<£ ã_ȋ°
¬ ž#° xÀ+kOKŒ…ì? ‡ÚO^Î=KÇ°ó.

(S.I.- 2012) "Œ^Î# II : …è^ΰ. J<ÍH›=°Ok ãQê=°ã„¬[° L‡k

"Œ^Î# I : J=ô#°. …èH‡


› é`Í =¢ªéë`æ
Ç uë ‹¬i‡é=_ÈO …è^°
Î . H˅Õæ`Œ~¡°.

"Œ^Î# II : …è ^ Î ° . ^Î ° ‹¬ ° ë  ° =°x+² †³ ò H› ø =¼H÷ ë ` ŒÞxß 7. ㄬH›@# : q^¥¼~¡ ° Ö  ° ~Œ[H© † Ç ¶ …Õ `Ç y # ‡ã`Ç # °

ㄬuaOaªë~ò. =‚²ìO„¬ä½
› O_¨ K͆°
Ç =KÇ°ó<Œ ? (S.I. - 2008)

4. ㄬH›@# : q*ìý # ‰§G=ò#° q^¥¼†Ç ¶ ° =°i†Ç ò "Œ^Î# I : J=ô#°. ZO^ΰH›O>è q^¥¼~¡°


Ö ° „¬i„¬HÞ
› ^Ή#
× °

H›ˆì‰§……Õ x~¡ÄO
 ^Î JO‰×=òQê K͆°
Ç =KÇ°ó<Œ? ‡ÚOk =ôO_È~°
¡ , =°i†Çò ~Œ[H©†°
Ç <Œ†Ç°ä›½

(S.I.- 2008) KÍu…Õ H©°ƒç=°à° J=ô`Œ~¡°.

REASONING ABILITY (LOGICAL) ○ ○ ○ ○ ○ ○ ○ ○


169
○ ○ ○ ○ ○ ○ ○ ○ ○ ○ ○ ○ ○ ○ ○ ○ ○ ○ ○
Study Material
"Œ^Î# II : …è^ΰ. ZO^ΰH›O>è q^¥¼~¡°Ö…è ƒ’q+¬¼ ‡Ï~¡°°, 12. ㄬH›@# : H›uëH›<Œß ̄#°ß ‰×H÷ëQ®^¥? (Bank P.O. - '04)

Hê|\ “÷ ~Œ[H©†¶
Ç …Õ `Çy#‡ã`Ç =‚²ìOKÇ_O
È "Œ^Î# I : J=ô#°. ~¡KÇ~ò`ǁ° ㄬ[ P…ÕKÇ<Œ

^¥Þ~Œ "Œ~¡° ƒ’q+¬¼`Ç°ë…Õx ‡ã`ǁ䛽 `Ç=°ä›½ q^¥#Ōá ㄬƒì=O KǶ„¬Q®~¡°.

`Œ"Í° ãH›=°tH›Æ} WKÇ°óHù#Q®~¡°.


"Œ^Î# II : Hê^ΰ. ƒºuH›"³°Ø# |O`Ë Z=i<³á<Œ ïQ=

8. ㄬH›@# : ƒì~¡`Ç^͉×O…Õx q"Œ‚¬ì"³°Ø# "Œix öH=O


=KÇ°ó#°.

XöH XH›ø „²Áä›½ „¬iq°`ÇO Hê"Œx


13. ㄬH›@# : ƒì~¡`Ç^͉§xH÷ ̋áxH› |O J=‹¬~¡O …è^¥?

xö~œtOKÇ_ÈO ‹¬=°O[‹¬"Í°<Œ ?
(M.A.T. - 2005)
(Bank P.O. - 2005)
"Œ^Î# I : Hê^ΰ. ㄬ„O
¬ KÇO…Õx q°ye# ^͉§° J‚²ìOª

"Œ^Î# I : J=ô#°. nx=# JkH› [<Œƒì#°


=¶~Œ¾xH÷ J#°Q®°}OQê …è~¡°.

x†Ç°OãuOKÇ=KÇ°ó#°. =°i†Çò =ô#ß"Œix


"Œ^Î# II : J=ô#°. J<ÍH›=°Ok ƒì~¡f†Çò° J‚²ìOª

Jxß ªÏH›~Œ¼`Ë KÇH›øQê ̄OKÇ=KÇ°ó#°.


‹²^¥œO`Œxß #=òàKÇ°<Œß~¡°.

"Œ^Î# II : 14. ㄬH›@# :

Y
ㄬ „ ¬ O KÇ O …Õ U W`Ç ~ ¡ ^Í ‰ × O D „¬ ^ Î œ u x
„²
Á ° `Ç=° `Çe^
Á O
Î ã_È°#° =ò‹¬e`Ç#O…Õ

J=OaOKÇ_ÈO …è^ΰ. Hê|\ ÷“ Wk `DŽ¬C.

M
9. ㄬH›@# :
*ìãQ®`Q
ë Ç ê KǶ‹¬°ä›½<Í q^O
Î Qê KÇ@㓠„¬Hê~¡O "Œix

~¡ H › Æ } =¼=‹¬ Ö =…ÿ q^¥¼=¼=‹¬ Ö ä› € _¨


|=O`ÇO K͆¶
Ç …ì ? (SBI PO - 1998)

"Œ^Î# :

E
öHOã^Πㄬƒ’°`ÇÞO Pn#O…Õ =ôOKÇ_ÈO
I J=ô#°. W@°=O\ ÷ q+¬†Ç¶° KÇ@“ㄬHê~¡"Í°

‹¬=°O[‹¬"Í°<Œ? (R.B.I. - 2004)

D
"Œ^Î# :

9
ª^Î¼O.

I
"Œ^Î# :

59
…è^ΰ. q^μ J#°#k ~ŒR ㄬƒ’°`ŒÞxH÷
II J=ô#°. nx=# =ò‹¬e`Ç#O…Õ =ô#ß `ÇeÁ

CA
95
‹¬O|OkOz# JO‰×O Hê|\ ÷“ Wk ~ŒR

^ÎOã_È°ä›½ HùO`Ç L„¬‰×=°#O a‹¬°ëOk.

15. ㄬH›@# :
ㄬƒ’°`ÇÞ Pn#O…Õ<Í LO_¨e.

50
"Œ^Î# II : J=ô#°. D q^ÎOQê K͆ǰ@O=#
=‹¬°ë=ô#° W<£ª“…˜"³°O\˜ „¬^Μu…Õ Hù#_ÈO

98
A
=# Hù#°Qˁ°^¥~¡°xH÷ …샏’=¶ ?
q^섬~¡OQê Jxß ~Œ¢‘“° ‹¬=¶#, UH›su
:8
(M.A.T - 2004)
Ja=$kœ ‡ÚO^Î=KÇ°ó.

"Œ^Î# :
E

10. ㄬH›@# : I
h

J=ô#°. Hù#°Qˁ°^¥~¡°_È° `Ç䛽ø= K³eÁOKÇ


ƒ ì ~¡ ` Ç ^ Í ‰ × O …Õx Jxß q^Í j ƒì¼Oä› ½ #°
.P
IC

=‹² LO@°Ok.
=ü‹²"͆Ƕ…ì ?

"Œ^Î# :
AD

"Œ^Î# I : …è^°
Î . Wk ƒì~¡`^
Ç ‰
Í × PiÖH~
› O
¡ Q®Ōá ㄬu䛀
II Hê^ΰ. W<£ª“…˜"°
³ O\˜ž H›@°“@ =# 䛽@°O|
AB
O

|_³ û \ ˜ ̄iy‡é~ò ä› ½ @°O|O…Õ H›  `Ç  °


„¬i‹²`
Ö °
Ç #° H›e括°O
ë k. (Bank P.O.- 2003)

"Œ^Î# II : "³ò^΅º`Œ~ò.
ER
CH

J=ô#°. J„¬C_Í ƒì~¡f†Ç° ƒì¼O䛽 =°#°Q®_È

ª^Î¼"³°Ø Jq ‹¬O„¬#ß=°=ô`Œ~ò.
16. ㄬH›@# : ï~á…èÞ"Œ~¡° "Œi ‹¬O‹¬Ö…Õ „¬xKÍÀ‹ L^˼Q®°ä›½
YD

11. ㄬH›@# : ㄬu~ŒRO…Õ XH›\ ÷ H›O>è Z䛽ø= ̂áìHË~¡°“°


WKÍó ㄦԇ‹¹#° `ÇH›Æ}"Í° P„²"͆Ƕ…ì ?
,H

=ôO_¨…ì ?
(Bank P.O. - 2003)
T

"Œ^Î# : "Œ^Î# :
AM

I …è^ΰ. nx=# \ìH±ž K³eÁOKÍ"Œi "³ò`ÇëO


I Hê^Î ° . P ‹¬ O ‹¬ Ö … Õ „¬ x KÍ À ‹ L^˼Q® ° ä› ½
H

(IBPS - 2002) Lz`ÇOQê ㄬ†Ç¶}÷OKÍ ‚¬ì䛽ø LOk.


M

ªÚ=òà =$^Š¥ J=ô`Ç°Ok.

"Œ^Î# II : "Œ^Î# II :
AM

J=ô#°. nx=# öH‹¬° „¬i‘ø~¡O…Õ J=ô#°. nx=# ㄬ † Ç ¶ }÷ ä › ½ ä› ½ WOHê


IG

„¬ô~ËQ®u ̄~¡°Q®°`Ç°Ok. =°i†Çò n~¡ÉHêeH› ªÏH›~Œ¼° H›eæOKÍ g°O@°Ok.


KH

"Œ[¼ ‹¬=°‹¬¼° =ôO_È=ô.


R

12345678901234567890123456789012123456789012345678901234567890121234567890123456789012345678901212345678901234567890123
1234567890123456789012 123456789012345678901234

q=~¡}ì`ÇàH› ["Œ|°°
12345678901234567890123456789012123456789012345678901234567890121234567890123456789012345678901212345678901234567890123
1234567890123456789012 123456789012345678901234
12345678901234567890123456789012123456789012345678901234567890121234567890123456789012345678901212345678901234567890123
1234567890123456789012 123456789012345678901234
1234567890123456789012
PREVIOUS BITS :
12345678901234567890123456789012123456789012345678901234567890121234567890123456789012345678901212345678901234567890123
1234567890123456789012 123456789012345678901234
12345678901234567890123456789012123456789012345678901234567890121234567890123456789012345678901212345678901234567890123
1234567890123456789012 123456789012345678901234
12345678901234567890123456789012123456789012345678901234567890121234567890123456789012345678901212345678901234567890123
1234567890123456789012 123456789012345678901234
12345678901234567890123456789012123456789012345678901234567890121234567890123456789012345678901212345678901234567890123
123456789012345678901234

1. (2) "Œ^Î# I T‚¬ð[x`ÇO. Wk |"³°Ø#k Hê^ΰ. J…ì K͋#


² O`Ç =¶ã`Œ# „²
Á ° ãH›=°tH›}
Æ `Ë LO\ì~¡#°

Hêx "Œ^Î# II |"³°#


Ø k. ƒì~¡`^
Ç ‰
Í O
× ã„¬*쪐Þ=°¼ ^͉O
× . HË=_ÈO J~¡~
Ö ‚
¡ ì
² `ÇO. „²
Á ° ‹¬¶ø° =¶<ÍÀ‹ J=Hꉧ…è

XH›"Íˆ× ~Œ[H©†Ç° ‡s“xßO\ ÷h x~¡¶àeÀ‹ë ㄬƒ’°`ÇÞO Z䛽ø=.

#_À„^³=~¡°. Z=i W‘“~Œ[¼O "Œi^³á‡é`Ç°Ok. "Œ^Î# II T‚¬ð[x`ÇO =¶ã`Ç"Í°. Wn |"³°Ø#k Hê^ΰ.

ㄬ*쪐Þ=°¼ „¬îiëQê JO`ÇO J=ô`Ç°Ok. 3. (2) "Œ^Î# I J~¡Ö~¡‚²ì`ÇO =°i†Çò T‚¬ð`ÇàH›O. Hê=ô# Wk

2. (4) "Œ^Î# I |"³°Ø#k Hê^ΰ. ZO^ΰH›#Qê x~¡ÄO^Î q^μ |"³°Ø#k Hê^ΰ. ㄬu =¼H©ë `Ç# W‘“ä›½ J#°Q®°}OQê

J~¡Ö~¡‚²ì`ÇO. Wk q^¥¼~¡°Ö À‹ÞKÇó#° ‚¬ìiO„¬K͋¬°ëOk. ^ΰ‹¬°ë#° ^ÎiOKÇ=KÇ°ó#°. "Œ~¡° ^ÎiOKÍ ^ΰ‹¬°ë…è "Œi

REASONING ABILITY (LOGICAL) ○ ○ ○ ○ ○ ○ ○ ○


170
○ ○ ○ ○ ○ ○ ○ ○ ○ ○ ○ ○ ○ ○ ○ ○ ○ ○ ○
Study Material
=¼H÷`
ë ŒÞxß `³e†Ç°*誐ë~ò. ‡‰§ó`Ǽ ^͉§…Õ =…ÿ =°# `ÇÞ~¡Qê „¬i‘ø~¡O K͆°
Ç =KÇ°ó =°i†Çò XH›i g°^Í ƒì~¡O

^͉×O…Õ „²zó„²zó ^ΰ‹¬°ë° ^ÎiOKÇ~¡°. =°#k =ôOKÇ@O `Çy¾O„¬|_È°`Ç°Ok.

‹¬Oㄬ^¥†Ç° ^͉×O Hê=ô# "Œ^Î# II |"³°Ø#k. 12. (1) "Œ^Î#-I |"³°Ø#k. ㄬ‹¬°ë`Ç †ÇòQ®O…Õ ㄬ[O^Î~¡¶

4. (1) "Œ^Î# I |"³°Ø#k. ZO^ΰH›O>è >ÿHê߁r "ÍQ®OQê "Œ~Œë„¬ãuH› ^¥Þ~Œ, „¬ô‹¬ëH›„¬~”¡#O ^¥Þ~Œ J<ÍH›

q+¬ † Ç ¶ ° <Í ~ ¡ ° óä› ½ x "Œi P…ÕKÇ < Œ q^ ¥ <Œ#°


Ja=$kœ K³O^ΰ`Ç°#ß ã„¬‹°
¬ `
ë Ç †ÇòQ®O…Õ q*ìý#‰§¢ªëxH÷

P^ Î ° xH› O Qê =¶~¡ ° óä› ½ O@°<Œß~¡ ° . K³ á ` Ç # ¼=O`Ç ° Q® °


‹¬O|OkOz# *ìý#O P=‰×¼H›O. "Œ^Î# II T‚¬ð[x`ÇO.

5. (4) "Œ^Î#-I |‚Ôì#"³°Ø#k. D q+¬†Ç°O…Õ À„ï~O\˜žÌ„á


KÇ°<Œß~¡°.

"Œ^Î#- II =ü~¡š„¬ô "Œ^Î#. ƒºuH›|O HùO`Ç=~¡öH


ƒì~¡O Un =ôO_È^°
Î . WOHê „²
Á 䛽 „¬sH›
Æ ° x~¡Þ‚²ìÀ‹ë<Í

L„¬†³¶Q®„¬_È°`Ç°Ok.
À„ï~O\˜ž ‹¬O`Ç$„²ë K³O^ΰ`Œ~¡°. "Œ^Î#-II T‚¬ð`ÇàH›"°
³ #
Ø k.

13. (4) "Œ^Î # - I =°i†Ç ò "Œ^Î # - II ï ~ O_È ¶ |‚Ô ì #"³ ° Ø #


6. (1) =¼=ª†Ç¶xß †Ç°OãfH›iOKÇ_ÈO =# L`Çæuë

Y
"Œ^Î#…è.
̄~¡°Q®°`Ç°Ok. `Ç^¥Þ~Œ x`Œ¼=‹¬~¡ =‹¬°ë=ô ^Î~¡#°

M
ㄬu^͉O
× `Ç# ª~¡Þƒº=°`Œ¼xß Hꇐ_È°HË=_¨xH÷ ^¥x
x†Ç°OãuOKÇ=KÇ°ó. Hê=ô# "Œ^Î#-I |"³°#
Ø k. "Œ^Î#-

~¡H›Æ} =#~¡°#° Jk U~Œæ@°K͋¬°HË"ŒežO^Í. ~¡H›Æ}


II öH=O XH› T‚¬ì =¶ã`Ç"Í°.

E
=¼=‹¬Ö …èxKË „¬~Œ~ò^͉§° ^¥_K͋² PãH›q°OKÇ°
7. (2) "Œ^Î # - I T‚¬ ð [x`Ç O . ö H =O Ju`Ç ä › ½ ø= =°Ok

D
9
HË=_¨xH÷ U=¶ã`ÇO ‹¬OHËzOKÇ=ô.

59
q^¥¼~¡°Ö…è, Jn K³_È° =¼‹¬<Œä›½ …Õ<³á# "Œö~ ~Œ[H©†Ç°

CA
95
"Íö~ ^͉§° J‚²ìOª =¶~¡¾O…Õ =ô<Œß …èH›‡é~ò<Œ,
<Œ†Ç°ä›½° K³„²æ#@°Á P_È`Œ~¡°. "Œ^Î#-I |"³°Ø#k

=°#O J‚²ìOª ‹²^¥œO`Œ<Íß #=òà䛽x =ô<Œß ^͉~


× H
¡ }
ƛ

50
Hê^ΰ.

=¼=‹¬Ö `DŽ¬æx‹¬i.

98
A
"Œ^Î # - II |"³ ° Ø # k. ~Œ[H© † Ç ¶ O>è < Í ã„¬ ` Ç ¼ ~¡ ° Ö 

(4)
:8
14. KÇ@“ㄬHê~¡O |=O`ÇOQê `ÇeÁ^ÎOã_È° ‹¬O~¡H›Æ}#°

Z`Ç°ëä›½ ̄á Z`ǰ끰 "͋¬¶ë H›Æ}H›Æ}O ãHù`ÇëQê P…Õz‹¬¶ë


E

KǶÀ‹@°Á K͆ǰ_ÈO ‹¬=°O[‹¬h†Ç°O Hê^ΰ. `Ç=°#° H›x,


h

<³Q®°¾ä›½~Œ=_ÈO. D `Ç~¡‚¬ð P…ÕKÇ#° q^¥¼~¡°Ö#°


.P
IC

̄Oz L#ß`Ç ª Ö ~ òH÷ f‹¬ ° ä› ½ =zó# `Ç e Á ^ Î O ã_È ° #°

Qù„¬æQê fiók^ΰí`Œ~ò. Hê=ô# "Œ^Î#-II |"³°Ø#k.


AD

H› O \ ÷ H ÷ ï ~ „¬ æ …ì Hꇐ_È ° HË"Œ<Í P…ÕKÇ # „²  Á 

8. (1) "Œ^Î#-I |"³°#


Ø k. ZO^ΰH›#Qê x†Ç°Oãu`Ç ä›½@°O|O
AB
O

=°#‹¬°ž…Õ =ôO_¨e. W…ìO\ ÷q |=O`ÇOQê

=# ƒì^Î¼`ǁ#° ‹¬ãH›=°OQê x~¡Þ‚²ìOKÇ=KÇ°ó. Z=iH©


ER

"Œ^Î#-I "Œ^Î#-II
CH

KÍ~òOKDžè=ò. Hê=ô# =°i†Çò

U …Õ@° ~Œä›½O_¨ ̄OKÇ=KÇ°ó. J^Í ‹¬=°†Ç°O…Õ ^͉×


YD

ï~O_ȶ |‚Ôì# "Œ^Î#…è.

[<Œƒì#° x†Ç°OãuOz#@Á=ô`Ç°Ok. " Œ ^ Î # - II


15. (4) "Œ^Î#-I =°i†Çò "Œ^Î#-II ï~O_ȶ |"³°Ø#q Hê^ΰ.
,H

|‚Ôì#"³°Ø#k.
T

ZO^ΰH›#Qê W<£ª“…˜"³°O\˜ „¬^Μu…Õ =‹¬°ë=ô#°


AM

9. (1) "Œ^Î#-I |"³°Ø#k. q^¥¼=¼=‹¬Ö…Õ ªÖxH›`Ç䛽


H

Hù#°Qˁ° K͋²#„¬ô_È° =‹¬°ë=ô †³òH›ø J‹¬°^Î~¡ H›O>è


M

‹¬O|OkOz# JO‰§° 䛀_¨ =ôO\ì~ò. Wk ~ŒR


JkH› ^Î~¡#° K³eÁOKŒež =ôO@°Ok. Hê=ô# "Œ^Î#-I
AM
IG

ㄬƒ’°`ŒÞxH÷ ‹¬O|OkOz# JO‰×O. öHOã^ÎO D


|"³°Ø#k Hê^ΰ.
KH

q+¬†°
Ç O…Õ ‹¬~
ï #
á <Œ¼†Ç°O K͆° Î . "Œ^Î#-II H
Ç …è^° ö =O
W<£ª“…˜"³°O\˜…Õ =‹¬°ë=ôx Hù#߄¬C_È° <³<³…ì `Ç䛽ø=
R

T‚¬ð[x`ÇO.
"³ ò `Ç ë O …Õ<Í K³ e Á O KŒež =ôO@°Ok. Wk ä› ½ @°O|

10. (4) ƒì~¡`Ç^͉×O `Ç# "Œ}÷[¼ Hê~¡¼H›…쇐#°


|_³\
û ˜Ì„á Z…ìO\ ÷ ㄬƒì
 =O KǶ„¬^°
Î . H›`ǁ䛽 Z…ìO\ ÷

Hù#ªyOKÇ _ ¨xH÷ *ìf†Ç ° ªÖ ~ ò…Õ ƒì¼Oä› ½ #°


Pªø~¡O …è^ΰ. Hê=ô# "Œ^Î#-II |"³°Ø#k Hê^ΰ.

U~Œæ@°K͋¬°ä›½Ok. Hê=ô# "Œ^Î#-I J‹¬æ+¬“O. "Œ^Î#-


16. (4) "Œ^Î#-I |‚Ôì#=°~ò#k. ZO^ΰH›#Qê ï~á…èÞ

II T‚¬ð`ÇàH›"³°Ø#k.
L^˼Q®°ä›½ P ‹¬O‹¬Ö r`Œ° K³eÁ‹¬°ëOk. P _È|°Ä

11. (2) "Œ^Î#-I J~¡Ö~¡‚²ì`ÇO. "Œ^Î#-II |"³°Ø#k. #°O_ "Œ~¡ ° \ ÷ ï H ø@°Á Hù#°Qˁ° KÍ ‹ ² ㄬ † Ç ¶ }O

ㄬ‹¬°ë`Ç <Œ¼†Ç°=¼=‹¬Ö…Õ ̄O_OQ· öH‹¬°° Hùxß HË@°Á. K͆ǰ=KÇ°ó. Yzó`ÇOQê ‡‹¹° W"ŒÞež# J=‹¬~¡O

XH›\ ÷H›O>è Z䛽ø= ̂áìHË~¡°“° =ôO_È@O =# öH‹¬°° …è^ΰ. "Œ^Î#-II T‚¬ð[x`ÇO.

REASONING ABILITY (LOGICAL) ○ ○ ○ ○ ○ ○ ○ ○


171
○ ○ ○ ○ ○ ○ ○ ○ ○ ○ ○ ○ ○ ○ ○ ○ ○ ○ ○
Study Material
xtó`ÇO - Hê~¡}O
4 (Assertion - Reason)
=òMì¼O‰§°...
G xtó`ÇO =°i†Çò Hê~¡}O ï~O_ȶ [#~¡…˜ <Œ…ÿ_£û䛽 ‹¬O|OkOz#q J~ò LO\ì~ò. Hê=ô# q^¥¼iÖH÷ [#~¡…˜ <Œ…ÿ_£û̄á
„¬îië J=Qꂬì# H›ey =ôO_¨e.
G D qƒìQ®O…Õ xtó`ÇO (Assertion) =°i†Çò Hê~¡}O (Reason) J#° ï~O_È° ㄬH›@#° W=Þ|_È`Œ~ò. q^¥¼iÖ
K͆ǰ=‹²#k.

Y
1) xtó`ÇO, Hê~¡}O ï~O_ȶ ‹¬i†³Ø°# ㄬH›@#…ì ? Hê^¥ ? Q®°iëOKŒe.
2) xtó`ÇO, Hê~¡}O ~ï O_ȶ ‹¬i†³°Ø # ㄬH@
› #…è J~ò`Í Hê~¡}O, xtó`ŒxH÷ ‹¬i†³°Ø # q=~¡} W‹¬°Oë ^¥ ? …è^¥ ? Q®=°xOKŒe.

M
G Jxß HêO„²\ ©\ ÷"£ „¬sH›Æ…Õ xtó`ÇO - Hê~¡}O qƒìQ®O…Õx ㄬ‰×߁° H©H› ‡ã`Ç#° ‡é+²‹¬°ë<Œß~ò.
G

E
xtó`ÇO ‹¬i†³Ø°#k, Hê~¡}O `DŽ¬CQê 䛀_¨ LO_È=KÇ°ó#°. ^¥xx Q®=°xOz ‹¬=¶^¥#O `³e†Ç°*è†Ç¶e.
G xtó`ÇO ‹¬i†³Ø°#k Hê䛽O_¨, Hê~¡}O ‹¬i†³Ø°# ㄬH›@# J~ò LO_È=KÇ°ó.

D
9
59
G xtó`ÇO, Hê~¡}O ï~O_ȶ `DŽ¬C ㄬH›@#° 䛀_¨ J~ò LO_È=KÇ°ó.
CA
95
=¶ki ㄬ‰×߁°
50
98
A
:8
G 2. xtó`ÇO :
E

ㄬu ㄬ‰×ß䛽 D ãH÷Ok "Œx #°O_ XH› ‹¬=¶^¥<Œxß (A) H›i#


” [O`Ë ^ΰ‹¬°
ë #° LuH÷#„¬ô_È° "Œ\ ÷
h
.P

#°O_ =òiH÷x „¬îiëQê `ùyOKDžL


è =ò.
IC

ZOKÇ°HË"Œe.

A R Hê~¡}O (R) :
AD

H›i#
” [O…Õ KŒ…ì ~¡Hê Yx[„¬^¥~ŒÖ°
xtó`ÇO =°i†Çò Hê~¡}O J~ò#„¬ô_È°

a) A R =ôO\ì~ò.
AB
O

=°i†Çò ° ï~O_ȶ ‹¬i†³Ø°# ㄬH›@#…è. Hê~¡}O

(R), (A) ª^Î# : ‹¬=¶^¥#O (b). xtó`ÇO (A) =°i†Çò Hê~¡}O (R) °

(R), (A)
ER

xtó`ÇO #䛽 ‹¬i†³Ø°# q=~¡} W‹¬°ëOk.


CH

b) A R
ï~O_ȶ ‹¬i†³Ø°#"Í. Hêx Hê~¡}O xtó`ÇO 䛽
=°i†Çò ° ï~O_ȶ ‹¬i†³Ø°# ㄬH›@#…è. Hêx
YD

(R) (A)
‹¬i†³Ø°# q=~¡} Hê^ΰ.

Hê~¡}O xtó`ÇO 䛽 ‹¬i†³Ø°# q=~¡} Hê^ΰ.


H›i”# [O`Ë ^ΰ‹¬°ë#° LuH÷#„¬ô_È° #°~¡°Q®° U~¡æ_È^ΰ.

c) (A) (R)
,H

xtó`ÇO ‹¬i†³Ø°#^Í Hêx Hê~¡}O ‹¬i†³Ø°#k


T

^¥x Hê~¡}OQê ^ΰ‹¬°ëä›½ JO\ ÷# =òiH÷x „¬îiëQê


AM

Hê^ΰ.
`ùyOKDžè=ò.
H

d) (A) (R)
M

xtó`Ç O ‹¬ i †³ Ø ° #kHê^Î ° Hêx Hê~¡ } O


H›i”#[O…Õ KŒ…ì ~¡Hê Yx[„¬^¥~ŒÖ°O_È°@ 䛀_¨
AM
IG

‹¬i†³Ø°#^Í. ‹¬i†³Ø°# ㄬH›@#†Í°.

e) (A) (R) 3. xtó`ÇO (A) :


KH

xtó`ÇO Hê~¡}O ï~O_ȶ `DŽ¬C ㄬH›@#°. *è"£°ž KŒ_ÞH± Z¢Hê“<£#° H›#°Qù<Œß_È°.

(A) (R) Hê~¡}O (R) :


R

Z¢Hê“<£ P"͉×O `Ç@‹¬ÖO Hê|\ ÷“ ^¥xx


ㄬ‰ß
× …Õ W=Þ|_# xtó`ÇO =°i†Çò Hê~¡}O

H›#°Qù#_ÈO ª^Î¼"³°Ø#k.

(E)
#° ä›Æ½}âOQê „¬ijeOz Wzó# <Œ°Q®° ‹¬=¶^¥<Œ

ª^Î# : ‹¬=¶^¥#O ZO^ΰH›#Qê

J.J.
#°O_ ‹¬i†³Ø°# ‹¬=¶^¥#O#° ZOKÇ°HË"Œe.

Z¢Hê“<£#° ^Š¥=°ž<£ H›#°Qù<Œß_È° =°i†Çò Z¢Hê“<£

1. xtó`ÇO (A) : ã\ì<£ž‡¦~¡à~Ÿ L„¬†³¶yOz "˅è“*˜#° ̋“„¹


†³òH›ø P"͉×O |°°}ì`ÇàH›O.

J„¹ …è^¥ ̋“„¹_ñ<£ K͆ǰ=KÇ°ó.


5. xtó`ÇO (A) : W^Š…
á³ ˜ Pø‚¬ð…˜ (C2H5OH) =°i†Çò h~¡°

Hê~¡}O (R) : ã\ì<£ž‡¦~¡à~Ÿ#° D.C =†Ç¶…Õ =¶ã`Ç"°


Í
(H2O)  q°ã‰×=°O #°O_ "Œ\ x
÷ „¬îiëQê

L„¬†³¶yªë~¡°.
"Í~¡° K͆ǰ=KÇ°ó#°.

ª^Î# : ‹¬ = ¶^ ¥ #O (C). ZO^Î ° H› # Qê "Ë…è “ * ˜ # ° ̄OKÇ ° @ä› ½ Hê~¡}O (R) : h\ ÷ =°~¡°Q®°ªÖ#O 100°C =°i†Çò

=°i†Ç ò `Ç y ¾ O KÇ ° @ä› ½ ã\ì<£ ž ‡¦  ~¡ à ~Ÿ L„¬ † ³ ¶ yªë ~ ¡ ° . Pø‚¬ð…˜ =°~¡°Q®°ªÖ#O 80°C 

ã\ì<£ž‡¦~¡à~Ÿ#° =òY¼OQê A.C =†Ç¶…Õ ª^Î# : ‹¬=¶^¥#O (d). ZO^ΰH›#Qê xtó`ÇO (A) ‹¬i†³Ø°#

L„¬†³¶yªë~¡°.
ㄬH›@# Hê^ΰ. Hêx Hê~¡}O (R) ‹¬i†³Ø°# ㄬH›@#.

REASONING ABILITY (LOGICAL) ○ ○ ○ ○ ○ ○ ○ ○


172
○ ○ ○ ○ ○ ○ ○ ○ ○ ○ ○ ○ ○ ○ ○ ○ ○ ○ ○
Study Material
123456789012345678901234567890121234567890123456789012345678901212
12345678901234567890123456789012123456789012345678901234567890121234567890123456789012345678901212345678901234567890123
1234567890123456789012345678901212345678901234567890123456789012123456789012345678901234567890121234567890123
1234567890123456789012345678901212345678901234567890123456789012123456789012345678901234567890121234
12345678901234567890123456789012123456789012345678901234567890121234567890123456789012345678901212345678901234567890123456789012123
1234567890123456789012345678901212345678901234567890123456789012123456789012345678901234567890
12345678901234567890123456789012123456789012345678901234567890121234567890123456789012345678901212345678901234567890123
1234567890123456789012345678901212345678901234567890123456789012123456789012345678901234567890121234567890123
1234567890123456789012345678901212345678901234567890123456789012123456789012345678901234567890121234
123456789012345678901234567890121234567890123456789012345678901212
12345678901234567890123456789012123456789012345678901234567890121234567890123456789012345678901212345678901234567890123456789012123
1234567890123456789012345678901212345678901234567890123456789012123456789012345678901234567890
12345678901234567890123456789012123456789012345678901234567890121234567890123456789012345678901212345678901234567890123
1234567890123456789012345678901212345678901234567890123456789012123456789012345678901234567890121234567890123
1234567890123456789012345678901212345678901234567890123456789012123456789012345678901234567890121234
12345678901234567890123456789012123456789012345678901234567890121234567890123456789012345678901212345678901234567890123456789012123
1234567890123456789012345678901212345678901234567890123456789012123456789012345678901234567890
123456789012345678901234567890121234567890123456789012345678901212
PRACTICE TEST - 1
12345678901234567890123456789012123456789012345678901234567890121234567890123456789012345678901212345678901234567890123
1234567890123456789012345678901212345678901234567890123456789012123456789012345678901234567890121234567890123
1234567890123456789012345678901212345678901234567890123456789012123456789012345678901234567890121234
12345678901234567890123456789012123456789012345678901234567890121234567890123456789012345678901212345678901234567890123456789012123
1234567890123456789012345678901212345678901234567890123456789012123456789012345678901234567890
123456789012345678901234567890121234567890123456789012345678901212
12345678901234567890123456789012123456789012345678901234567890121234567890123456789012345678901212345678901234567890123
1234567890123456789012345678901212345678901234567890123456789012123456789012345678901234567890121234567890123
1234567890123456789012345678901212345678901234567890123456789012123456789012345678901234567890121234
12345678901234567890123456789012123456789012345678901234567890121234567890123456789012345678901212345678901234567890123456789012123
1234567890123456789012345678901212345678901234567890123456789012123456789012345678901234567890
12345678901234567890123456789012123456789012345678901234567890121234567890123456789012345678901212345678901234567890123
1234567890123456789012345678901212345678901234567890123456789012123456789012345678901234567890121234567890123
1234567890123456789012345678901212345678901234567890123456789012123456789012345678901234567890121234
12345678901234567890123456789012123456789012345678901234567890121234567890123456789012345678901212345678901234567890123456789012123
1234567890123456789012345678901212345678901234567890123456789012123456789012345678901234567890
123456789012345678901234567890121234567890123456789012345678901212
12345678901234567890123456789012123456789012345678901234567890121234567890123456789012345678901212345678901234567890123456789012123
1234567890123456789012345678901212345678901234567890123456789012123456789012345678901234567890
12345678901234567890123456789012123456789012345678901234567890121234567890123456789012345678901212345678901234567890123
1234567890123456789012345678901212345678901234567890123456789012123456789012345678901234567890121234567890123
1234567890123456789012345678901212345678901234567890123456789012123456789012345678901234567890121234

G ㄬu ㄬ‰×ß䛽 D ãH÷Ok "Œx #°O_ XH› ‹¬=¶^¥<Œxß 3. xtó`ÇO : Z~ò_£žä›½ "ŒH÷ž<£ …è^ΰ.

ZOKÇ°HË"Œe. Hê~¡}O : Z~ò_£ž "ŒH÷ž<£ `dž¶


Ç sH÷ KŒ…ì Y~¡°ó J=‹¬~O
¡ .

A xtó`ÇO =°i†Çò R Hê~¡}O J~ò#„¬ô_È°


4. xtó`ÇO : Z`³åë# ㇐O`Œ…Õ P‚¬ð~¡ „¬^¥~ŒÖ#°

a) A =°i†Çò R ° ï~O_ȶ ‹¬i†³Ø°# ㄬH›@#…è. Hê~¡}O


L_H÷OKÇ_¨xH÷ "Œ\ ÷H÷ L„¬C H›°„¬ô`Œ~¡°.

(R), xtó`ÇO (A) #䛽 ‹¬i†³Ø°# q=~¡} W‹¬°ëOk.


Hê~¡}O : Z`³#
ëå ㇐O`Œ…Õ L‘éâãQ®`Ç H›x+¬O
» Qê LO@°Ok.

b) A =°i†Çò R ° ï~O_ȶ ‹¬i†³Ø°# ㄬH›@#…è. Hêx


5. xtó`ÇO : "³°ØãHË`Ç~¡OQꁰ _˅ì†Ç°=¶# q^ΰ¼`Ÿ

Hê~¡}O (R) xtó`ÇO (A) 䛽 ‹¬i†³Ø°# q=~¡} Hê^ΰ.


=°i†Çò J†Ç°ªøO`Ç `Ç~¡OQê#° XH›^¥xH÷

c) xtó`ÇO (A) ‹¬i†³Ø°#^Í Hêx Hê~¡}O (R) ‹¬i†³Ø°#k


=°~ùH›\ ÷ O|OQê H›ey =ôO\ì~ò.

Hê~¡}O :

Y
"³°ãØ HË`Ç~O
¡ Qꁰ q^ΰ¼^Ά°
Ç ªøO`Ç `Ç~O
¡ Qꁰ.
Hê^ΰ.

6. xtó`ÇO :
d) (A) (R)
HêOu `Ç~¡OQê#° ^Î$=}O K͆ǰ=KÇ°ó#°.

M
xtó`Ç O ‹¬ i †³ Ø ° #kHê^Î ° Hêx Hê~¡ } O

Hê~¡}O : HêOu `Ç~¡OQꁰ u~¡¼H± `Ç~¡OQꁰ.

CO2
‹¬i†³Ø°#^Í.

7. xtó`ÇO :

E
e) (A) (R)
"Œ`Œ=~¡ } O…Õx ̄~¡ ° Q® ° ^Î  =#

xtó`ÇO Hê~¡}O ï~O_ȶ `DŽ¬C ㄬH›@#°.

^Î$"Œ =^Îí =°OKÇ° H›iy‡é`Ç°Ok.

D
9
(A) (R) Hê~¡}O :

59
ㄬ‰ß
× …Õ W=Þ|_# xtó`ÇO =°i†Çò Hê~¡}O
QËÁ|…˜ L‘éâãQ®`Ç Ì„~¡°Q®°`Ç°Ok.

CA
8. xtó`ÇO :

95
#° ä›Æ½}âOQê „¬ijeOz Wzó# <Œ°Q®° ‹¬=¶^¥<Œ
^Î$"Œ =^Îí =‹¬°ë=ô° JkH› ƒì~Œxß H›ey

50
#°O_ ‹¬i†³Ø°# ‹¬=¶^¥#O#° ZOKÇ°HË"Œe.
=ôO\ì~ò.

1. xtó`ÇO : Hê~¡}O :
98
A
=¶#= ‰×s~¡O…Õ POã`ÇO XH› q‹¬iû`Ç ^Î$"Œ =^Îí ƒ’¶q° †³òH›ø Q®°~¡°`ÇÞ `ÇÞ~¡}O
:8
J=†Ç°=O. q°= Q®i+¬»O.

9. xtó`ÇO : 'O'
E

Hê~¡}O :
h

POã`ÇO r~¡âãH÷†Ç°…Õ ‡…ç¾#^ΰ. |Á_£ ãQ®¶„¬ô H›ey# "Œix q‰×ÞãQ®‚Ôì`ǁ°


.P

2. xtó`ÇO :
IC

JO\ì~¡°.
ƒì~¡`^
Ç ‰
Í O
× …Õ P_È"Œi ‹¬OY¼ =°Q®"Œi H›O>è

Hê~¡}O : 'O'
AD

|Á_£ ãQ®¶„¹ ㄬu[#Hê#° H›ey†ÇòO_È^°


Î .
JkH›O.

10. xtó`ÇO :
AB
O

Hê~¡}O : P_È"Œ~¡° P‚¬ð~¡ ‹¬=°`Ç°¼`Ç JkH›OQê


„¬ô~Œ`Ç# <ŒQ®iH›`Ç J`ǼkH›OQê #^ΰ =^Îí
ER

Ja=$kœ K³Ok#k.
CH

‡\ ÷ªë~¡°.

Hê~¡}O : =¶#=ôx =òY¼=$uë =¼=ª†Ç°O.


YD

1234567890123456789012
12345678901234567890123456789012123456789012345678901234567890121234567890123456789012345678901212345678901234567890123
12345678901234567890123

q=~¡}ì`ÇàH› ["Œ|°°
12345678901234567890123456789012123456789012345678901234567890121234567890123456789012345678901212345678901234567890123
1234567890123456789012
1234567890123456789012 12345678901234567890123
12345678901234567890123456789012123456789012345678901234567890121234567890123456789012345678901212345678901234567890123
12345678901234567890123
1234567890123456789012 12345678901234567890123
12345678901234567890123456789012123456789012345678901234567890121234567890123456789012345678901212345678901234567890123
PRACTICE TEST - 1 :
,H

1234567890123456789012
12345678901234567890123456789012123456789012345678901234567890121234567890123456789012345678901212345678901234567890123
1234567890123456789012 12345678901234567890123
1234567890123456789012
12345678901234567890123
12345678901234567890123456789012123456789012345678901234567890121234567890123456789012345678901212345678901234567890123
12345678901234567890123456789012123456789012345678901234567890121234567890123456789012345678901212345678901234567890123
12345678901234567890123
T
AM
H

1. (1) xtó`ÇO =°i†Çò Hê~¡}O ï~O_ȶ ‹¬i†³Ø°#"Í =°i†Çò q^ΰ¼^Ά°


Ç ªøO`Ç `Ç~O
¡ Qê…Õ XH›^¥xHùH›\ ÷ O|OQê
M
AM

Hê~¡}O, xtó`ÇO#䛽 ‹¬i†³Ø°# q=~¡}. =ô#ß q^ΰ¼`Ÿ H


ö ãÆ `ÇO =°i†Çò J†Ç°ªøO`Ç H
ö ãÆ `ÇO _ˁ<Œ
IG

2. (5) xtó`ÇO =°i†Çò Hê~¡}O ï~O_ȶ ‹¬i†³Ø°#q Hê^ΰ.


Kǁ#O H›ey =ôO\ì~ò.
KH

q=H›Æ Hê~¡}OQê ƒì~¡`Ç^͉×O…Õ P_È"Œi „¬ô@°“H› ƒìQê


"³°ØãHË`Ç~¡OQꁰ q^ΰ¼^ΆǰªøO`Ç `Ç~¡OQꁰ.
R

`Çy¾‡é~òOk. =°i†Çò P_È"ŒiH÷ P‚¬ð~¡ ‹¬=°`Ç°¼`Ç


6. (1) xtó`ÇO =°i†Çò Hê~¡}O ï~O_ȶ ‹¬i†³Ø°#"Í =°i†Çò

JkH›OQê J=‹¬~¡"³°Ø#„¬æ\ ÷H© "Œ~¡° ^¥xx KÍ~¡°H˅èH›


Hê~¡}O, xtó`ÇO#䛽 ‹¬i†³Ø°# q=~¡}.

‡é`Ç°<Œß~¡°.
u~¡¼H± `Ç~¡OQê#° ^Î$=}O K͆ǰQ®O Hêx J#°^³á~¡É¼

3. (3) xtó`ÇO ‹¬i†³Ø°#k Hêx Hê~¡}O ‹¬i†³Ø°#k Hê^ΰ.


`Ç~¡OQê#° ^Î$=}O K͆ǰ…è=ò.

Z~ò_£žä›½ Hê~¡}"³°#
Ø "³~
á ‹
¡ ¹ ^¥x *ÿ<\
³ H
÷ ± HË_£ ("Œ~¡‹`
¬ Þ
Ç
7. (1) xtó`ÇO =°i†Çò Hê~¡}O ï~O_ȶ ‹¬i†³Ø°#"Í =°i†Çò

HË_£)#° =¶~¡°ó䛽O@¶ =ôO@°Ok. Hê=ô# "Œ¼H÷ž<£


Hê~¡}O, xtó`ÇO#䛽 ‹¬i†³Ø°# q=~¡}.

CO 2
H›#°Qù#|_ȅè^ΰ.

"Œ`Œ=~¡}O…Õ JkH›=°†Í°¼ Hùk Jk

4. (1) xtó`ÇO =°i†Çò Hê~¡}O ï~O_ȶ ‹¬i†³Ø°#"Í Hê~¡}O,

"Œ`Œ=~¡}O…Õx "Í_x
 ã\섹K‹
Í ² "Í_ =°iO`Ç Ì„iöQ@°Á
xtó`ÇO#䛽 ‹¬i†³Ø°# q=~¡}.

KÍ ‹ ² QËÁ | …˜ L‘éâ ã Q® ` Ç # ° ̄iö Q @@°Á KÍ ‹ ¬ ° ë O k. D


5. (1) xtó`ÇO =°i†Çò Hê~¡}O ï~O_ȶ ‹¬i†³Ø°#"Í =°i†Çò

"Í_=# ^Î$"Œ =^Îí =°OKÇ° H›iy‡é`Ç°Ok.


Hê~¡}O, xtó`ÇO#䛽 ‹¬i†³Ø°# q=~¡}.

REASONING ABILITY (LOGICAL) ○ ○ ○ ○ ○ ○ ○ ○


173
○ ○ ○ ○ ○ ○ ○ ○ ○ ○ ○ ○ ○ ○ ○ ○ ○ ○ ○
Study Material
8. (1) xtó`ÇO =°i†Çò Hê~¡}O ï~O_ȶ ‹¬i†³Ø°#"Í =°i†Çò ~¡H›O ~¡Hêëxß U ãQ®¶„¬ô ~¡H›ëO H›ey# "ŒiïHá<Œ ZH÷øOKÇ

Hê~¡}O, xtó`ÇO#䛽 ‹¬i†³Ø°# q=~¡}. =KÇ°ó#°.

9. (4) xtó`ÇO ‹¬i†³Ø°#k Hê^ΰ Hêx Hê~¡}O ‹¬i†³Ø°#^Í. 10. (1) xtó`ÇO =°i†Çò Hê~¡}O ~
ï O_ȶ ‹¬i†³°
Ø #"Í. Hê~¡}O,

|Á _ £ ã Q® ¶ „¬ ô 'O' Q®  "Œix q‰× Þ ^¥`Ç  ° JO\ì~¡ ° . xtó`ÇO#䛽 ‹¬i†³Ø°# q=~¡}.

ZO^ΰH›#Qê 'O' ãQ®¶„¬ô ~¡HO


ë› ã„¬u[#H›O H›ey†ÇòO_È^°
Î ª~¡=O`Ç"°
³ #
Ø <́ =°i†Çò =¼=ª†Ç¶xH÷ J=‹¬~=
¡ °Q®°

=°i†Çò ㄬu ~¡H›ÆHê#° U~¡æ~¡KÇ°HË^ΰ. Hê=ô# D h\ ÷ Hˋ¬O „¬ô~Œ`Ç# =¶#=ô° #^ΰ =^Îí r=#O

ªyOKÍ "Œ~¡°.

123456789012345678901234567890121234567890123456789012345678901212
12345678901234567890123456789012123456789012345678901234567890121234567890123456789012345678901212345678901234567890123
1234567890123456789012345678901212345678901234567890123456789012123456789012345678901234567890121234567890123
1234567890123456789012345678901212345678901234567890123456789012123456789012345678901234567890121234
123456789012345678901234567890121234567890123456789012345678901212
12345678901234567890123456789012123456789012345678901234567890121234567890123456789012345678901212345678901234567890123456789012123
1234567890123456789012345678901212345678901234567890123456789012123456789012345678901234567890
12345678901234567890123456789012123456789012345678901234567890121234567890123456789012345678901212345678901234567890123
1234567890123456789012345678901212345678901234567890123456789012123456789012345678901234567890121234567890123
1234567890123456789012345678901212345678901234567890123456789012123456789012345678901234567890121234
12345678901234567890123456789012123456789012345678901234567890121234567890123456789012345678901212345678901234567890123456789012123
1234567890123456789012345678901212345678901234567890123456789012123456789012345678901234567890
12345678901234567890123456789012123456789012345678901234567890121234567890123456789012345678901212345678901234567890123
1234567890123456789012345678901212345678901234567890123456789012123456789012345678901234567890121234567890123
1234567890123456789012345678901212345678901234567890123456789012123456789012345678901234567890121234
123456789012345678901234567890121234567890123456789012345678901212
12345678901234567890123456789012123456789012345678901234567890121234567890123456789012345678901212345678901234567890123456789012123
1234567890123456789012345678901212345678901234567890123456789012123456789012345678901234567890
PRACTICE TEST - 2
12345678901234567890123456789012123456789012345678901234567890121234567890123456789012345678901212345678901234567890123
1234567890123456789012345678901212345678901234567890123456789012123456789012345678901234567890121234567890123
1234567890123456789012345678901212345678901234567890123456789012123456789012345678901234567890121234
12345678901234567890123456789012123456789012345678901234567890121234567890123456789012345678901212345678901234567890123456789012123
1234567890123456789012345678901212345678901234567890123456789012123456789012345678901234567890
123456789012345678901234567890121234567890123456789012345678901212
12345678901234567890123456789012123456789012345678901234567890121234567890123456789012345678901212345678901234567890123
1234567890123456789012345678901212345678901234567890123456789012123456789012345678901234567890121234567890123
1234567890123456789012345678901212345678901234567890123456789012123456789012345678901234567890121234
12345678901234567890123456789012123456789012345678901234567890121234567890123456789012345678901212345678901234567890123456789012123
1234567890123456789012345678901212345678901234567890123456789012123456789012345678901234567890
123456789012345678901234567890121234567890123456789012345678901212
12345678901234567890123456789012123456789012345678901234567890121234567890123456789012345678901212345678901234567890123
1234567890123456789012345678901212345678901234567890123456789012123456789012345678901234567890121234567890123
1234567890123456789012345678901212345678901234567890123456789012123456789012345678901234567890121234
12345678901234567890123456789012123456789012345678901234567890121234567890123456789012345678901212345678901234567890123456789012123
1234567890123456789012345678901212345678901234567890123456789012123456789012345678901234567890
12345678901234567890123456789012123456789012345678901234567890121234567890123456789012345678901212345678901234567890123456789012123
1234567890123456789012345678901212345678901234567890123456789012123456789012345678901234567890
12345678901234567890123456789012123456789012345678901234567890121234567890123456789012345678901212345678901234567890123
1234567890123456789012345678901212345678901234567890123456789012123456789012345678901234567890121234567890123
1234567890123456789012345678901212345678901234567890123456789012123456789012345678901234567890121234

G ㄬu ㄬ‰×ß䛽 D ãH÷Ok "Œx #°O_ XH› ‹¬=¶^¥<Œxß 6. xtó`ÇO : ~¡ª†Ç°# KÇ~¼
¡ …Õ <³ãá \Õ[<£ "Œ†Çò=ô †³òH›ø

Y
ZOKÇ°HË"Œe. KÇ~Œ¼j`Ç KŒ…ì `Ç䛽ø=.

1. xtó`ÇO : Hê~¡}O :

M
HË_ãQ®°_È°Û̄á |O ㄬ†³¶yOz#„¬ô_È° Jk ª^¥~¡} L‘éâãQ®`Ç =^Îí <³áã\Õ[<£ "Œ†Çò=ô

„¬Q®°°`Ç°Ok. †³òH›ø KÇ~Œ¼j`Ç KŒ…ì `Ç䛽ø=.

E
Hê~¡}O : ㄬ=¶} "³‰
á §¼Ōá ㄬ†¶
³ yO„¬|_# |…ì<Íß 7. xtó`ÇO : ‡ã`ǁ#° =°i†Çò ^ΰ‹¬°ë#° ‰×Ãハ’O

D
9
„Ô_È#O JO\ì~¡°. K͆ǰ_¨xH÷ ª^¥~¡}OQê Wˆ×¤…Õ HÆê~¡

59
2. xtó`ÇO : „¬~¡Þ`ŒÌ„áH÷ "³ˆ×äHùk "Œ`Œ=~¡} „Ô_È#O ‹¬O|Ok`Ç „¬^¥~ŒÖ#° L„¬†³¶yªë~¡°.

CA
95
`ÇQ®°¾`Ç°Ok. Hê~¡}O : HÆê~¡ ‹¬O|Ok`Ç „¬^¥~ŒÖ° hˆ×¤…Õ ‹¬°°=ôQê

50
Hê~¡}O : "Œ`Œ=~¡}O…Õ ̄áH÷ "³ˆÃ
× ¤ Hùk Qêe ‰×Ã^ÎO
œ Qê H›iy‡é`Œ~ò.

98
A
=ôO@°Ok. 8. xtó`ÇO : Wˆ×¤…Õ "Œ_Í iㄦ²lö~@~Ÿ `ǁ°„¬ô f‹²
:8
3. xtó`ÇO : ƒ’¶q° †³òH›ø L`Çë~Œ~¡ÖQˈ×O =^Îí Qꁰ° LOz#„¬C_È° P Q®k "³ò`ÇëO KǁÁ|_È°`Ç°Ok.
E

Hê~¡}O :
h

ä› ½ _ " ³ á „ ¬ ô ä› ½ =°i†Ç ò ^Î H ÷ Æ } ì~¡ Ö Q Ëˆ× O =^Î í ‰×H÷ë XH› ~¡¶„¬O #°O_ =°~ùH› ~¡¶„¬O…ÕxH÷
.P
IC

Qꁰ° Z_È=°"³á„¬ô䛽 ㄬ†Ç¶}÷ªë~ò. =¶~¡°`Ç°Ok.


AD

Hê~¡}O : ƒ’¶ãƒ’=°}O Qꁰ k‰×……Õ =¶~¡°æä›½ 9. xtó`ÇO : ‹¬Þ`ÇOã`Ç ƒì~¡`^


Ç ‰
Í O
× …Õ ãa\ +
÷ ¹ ª~¡Þƒº=°`ÇÞO
AB
O

Hê~¡}O J=ô`Ç°Ok. Hù#ªyOk.

4. xtó`ÇO : Hê~¡}O :
ER
CH

ƒçQ®°¾#° =ã[OQê =¶~¡°ó@ XH› ƒºuH›KÇ~¡¼. ãa\ ÷+¹ ㄬƒ’°`ÇÞO ‹¬Þ`ÇOã`Ç ƒì~¡`Ç^͉×O †³òH›ø

Hê~¡}O :
YD

ƒºuH›KÇ~¡¼…Õ „¬^¥~¡ÖO †³òH›ø "Í°ˆ×#O =¶~¡ó z=i Q®=~¡ß~Ÿ [#~¡…˜#° x†Ç°q°OzOk.

|_È^ΰ. 10. xtó`ÇO : \ì~Ÿó…ÿ\


á 
˜ …Õ „¬~Œ=†Ç° PHê~¡„ô¬ ^Î~æ
¡ }ì#°
,H

5. xtó`ÇO :
T

#¶¼H÷Á†Ç°~Ÿ i†Ç¶H›“~Ÿ…Õ ƒì~¡[…ìxß L„¬†³¶yªë~¡°.


AM

"³¶_È~
ö @~Ÿ =…ÿ L„¬†³¶yªë~¡°. Hê~¡}O : „¬~Œ=†Ç° ^Î~¡æ}쁰 HêOux ‹¬=¶O`Ç~¡
H
M

Hê~¡}O : #¶¼H÷Á†Ç°~Ÿ i†Ç¶H›~¡“~Ÿ…Õ öHOã^ÎH› qzóuë HêOu„¬ôO[ ~¡¶„¬O…Õ ‹²~


Ö ¡ fã=`Ç`Ë „¬~Œ=~¡#
ë O
AM
IG

Hù~¡ä›½ ^ŠÎ~¡à…˜ #¶¼ã\ì<£#° L„¬†³¶yªë~¡°. K³Okªë~ò.


KH

1234567890123456789012
12345678901234567890123456789012123456789012345678901234567890121234567890123456789012345678901212345678901234567890123
12345678901234567890123
12345678901234567890123456789012123456789012345678901234567890121234567890123456789012345678901212345678901234567890123
1234567890123456789012 12345678901234567890123
q=~¡}ì`ÇàH› ["Œ|°°
1234567890123456789012 12345678901234567890123
12345678901234567890123456789012123456789012345678901234567890121234567890123456789012345678901212345678901234567890123
1234567890123456789012 12345678901234567890123
12345678901234567890123456789012123456789012345678901234567890121234567890123456789012345678901212345678901234567890123
R

1234567890123456789012
1234567890123456789012
PRACTICE TEST - 2 :
12345678901234567890123456789012123456789012345678901234567890121234567890123456789012345678901212345678901234567890123
1234567890123456789012 12345678901234567890123
12345678901234567890123
12345678901234567890123456789012123456789012345678901234567890121234567890123456789012345678901212345678901234567890123
12345678901234567890123
12345678901234567890123456789012123456789012345678901234567890121234567890123456789012345678901212345678901234567890123

1. (2) xtó`ÇO =°i†Çò Hê~¡}O ï~O_ȶ ‹¬i†³Ø°#"Í =°i†Çò ƒçQ®°¾#° ª^¥~¡}OQê, ‹¬°ƒ’OQê ãQê̄¦á\˜Qê =¶~¡ó

Hê~¡}O. xtó`ÇO#䛽 ‹¬i†³Ø°# q=~¡} W‹¬°ëOk. =KÇ°ó#°.

2. (4) xtó`ÇO ‹¬i†³Ø°#k Hê^ΰ. Hêx Hê~¡}O ‹¬i†³Ø°#k. ƒçQ® ° ¾ # °, =ã[OQê =¶ió#„¬ ô _È ° (Jk H › L‘éâ ã Q® ` Ç

"Œ`Œ=~¡}O…Õ ̄áH÷ "³ˆ×äHùk „Ô_È#O ̄~¡°Q®°`Ç°Ok. =°i†Çò „Ô_È#O =^Îí) ^¥x…Õx ~¡ª†Ç°# "Í°ˆ×#O…Õ

J…ìöQ Qêe‰×Ã^ΜOQê LO@°Ok. =¶~¡°æ =‹¬°ëOk. Hê=ô# Wk ƒºuH› KÇ~¡¼Hê^ΰ. ƒçQ®°¾,

3. (3) xtó`ÇO ‹¬i†³Ø°#k Hêx Hê~¡}O ‹¬i†³Ø°#k Hê^ΰ. =ã[OQê =¶~¡°@ XH› ‹¬‚¬ì[ ㄬãH÷†Ç°. D ㄬãH÷†Ç°

XH› ㄬ^͉×O…Õx Qêe ㄬª~¡k‰× JH›ø_ "Œ`Œ=~¡} „¬îië JQ®°@䛽 Hùxß q°e†Ç°#Á ‹¬O=`Ǟ~Œ° „¬_°
È `Ç°Ok.

„Ô_È#Ōá P^¥~¡„¬_ LO@°Ok. 5. (1) xtó`ÇO =°i†Çò Hê~¡}O ï~O_ȶ ‹¬i†³Ø°#"Í =°i†Çò

4. (4) xtó`ÇO ‹¬i†³Ø°#k Hê^ΰ. Hêx Hê~¡}O ‹¬i†³Ø°#k. Hê~¡}O xtó`ÇO#䛽 ‹¬i†³Ø°# q=~¡} W‹¬°ëOk.

REASONING ABILITY (LOGICAL) ○ ○ ○ ○ ○ ○ ○ ○


174
○ ○ ○ ○ ○ ○ ○ ○ ○ ○ ○ ○ ○ ○ ○ ○ ○ ○ ○
Study Material
6. (1) xtó`ÇO =°i†Çò Hê~¡}O ï~O_ȶ ‹¬i†³Ø°#"Í =°i†Çò "Í _  x |†Ç ° @ä› ½ q_È ° ^Î  KÍ ‹ ¬ ° ë O k. `Ç  °„¬ ô `³ ~ ¡ z

Hê~¡}O xtó`ÇO#䛽 ‹¬i†³Ø°# q=~¡} W‹¬°ëOk. LOz#„¬ô_È° H›Oã̄+¬~Ÿ ª^¥~¡} ‹²ÖuH›O>è JkH›OQê

7. (2) xtó`ÇO =°i†Çò Hê~¡}O ï~O_ȶ ‹¬i†³Ø°#"Í Hêx „¬xK͋¬¶ë JkH› L‘âxß Q®k…ÕH÷ q_È°^΁ K͋¬°ëOk.

Hê~¡}O, xtó`ÇO#䛽 ‹¬i†³Ø°# q=~¡} Hê^ΰ. 9. (4) xtó`ÇO ‹¬i†³Ø°#k Hê^ΰ. Hêx Hê~¡}O ‹¬i†³Ø°#^Í.

HÆê~¡ ‹¬O|O^Î „¬^¥~ŒÖ° h\ ÷…Õ H›iy #¶<³ …è^¥ „¦¬°# ƒì~¡`^


Ç ‰
Í §xH÷ ‹¬Þ`ÇOã`ǼO =KŒóH› ãa\ +
÷ ¹ ª~¡Þƒº=°`ÇÞO

„¬^¥~ŒÖ#° =°i†Çò ãwA „¬^¥~ŒÖ#° ㄬuöHƄ¬HꁰQê Hù#ªQ®…è^ΰ.

x~¡¶àeªë~ò. Hêx ƒì~¡`^


Ç ‰
Í × z=i Q®=~¡ß~Ÿ [#~¡…˜#° x†Ç°q°OzOk

8. (4) xtó`ÇO ‹¬i†³Ø°#k Hê^ΰ. Hêx Hê~¡}O ‹¬i†³Ø°#^Í. ãa\ ÷+¹ ㄬƒ’°`ÇÞ"Í°.

iㄦ²lö~@~Ÿ `ǁ°„¬ô `³iz=ôOz#„¬ô_È° L+¬âO JkH›OQê 10. (1) xtó`ÇO =°i†Çò Hê~¡}O ï~O_ȶ ‹¬i†³Ø°#"Í =°i†Çò

|†Ç°@䛽 q_È°^΅áÿ Q®k "³Kó


Ç |_È°`Ç°Ok. iㄦl
² ~
ö @~Ÿ…Õx Hê~¡}O, xtó`ÇO#䛽 ‹¬i†³Ø°# q=~¡} W‹¬°ëOk.

H›Oã̄+¬~Ÿ iㄦ²lö~@~Ÿ †³òH›ø …Õ„¬ KǁÁQê =ôOKÇ°`Ƕ

Y
123456789012345678901234567890121234567890123456789012345678901212
12345678901234567890123456789012123456789012345678901234567890121234567890123456789012345678901212345678901234567890123456789012123
1234567890123456789012345678901212345678901234567890123456789012123456789012345678901234567890
12345678901234567890123456789012123456789012345678901234567890121234567890123456789012345678901212345678901234567890123
1234567890123456789012345678901212345678901234567890123456789012123456789012345678901234567890121234567890123
1234567890123456789012345678901212345678901234567890123456789012123456789012345678901234567890121234
123456789012345678901234567890121234567890123456789012345678901212
12345678901234567890123456789012123456789012345678901234567890121234567890123456789012345678901212345678901234567890123456789012123
1234567890123456789012345678901212345678901234567890123456789012123456789012345678901234567890
12345678901234567890123456789012123456789012345678901234567890121234567890123456789012345678901212345678901234567890123
1234567890123456789012345678901212345678901234567890123456789012123456789012345678901234567890121234567890123
1234567890123456789012345678901212345678901234567890123456789012123456789012345678901234567890121234
12345678901234567890123456789012123456789012345678901234567890121234567890123456789012345678901212345678901234567890123456789012123
1234567890123456789012345678901212345678901234567890123456789012123456789012345678901234567890
123456789012345678901234567890121234567890123456789012345678901212
PRACTICE TEST - 3
12345678901234567890123456789012123456789012345678901234567890121234567890123456789012345678901212345678901234567890123
1234567890123456789012345678901212345678901234567890123456789012123456789012345678901234567890121234567890123
1234567890123456789012345678901212345678901234567890123456789012123456789012345678901234567890121234
12345678901234567890123456789012123456789012345678901234567890121234567890123456789012345678901212345678901234567890123456789012123
1234567890123456789012345678901212345678901234567890123456789012123456789012345678901234567890

M
123456789012345678901234567890121234567890123456789012345678901212
12345678901234567890123456789012123456789012345678901234567890121234567890123456789012345678901212345678901234567890123456789012123
1234567890123456789012345678901212345678901234567890123456789012123456789012345678901234567890
12345678901234567890123456789012123456789012345678901234567890121234567890123456789012345678901212345678901234567890123
1234567890123456789012345678901212345678901234567890123456789012123456789012345678901234567890121234567890123
1234567890123456789012345678901212345678901234567890123456789012123456789012345678901234567890121234
12345678901234567890123456789012123456789012345678901234567890121234567890123456789012345678901212345678901234567890123456789012123
1234567890123456789012345678901212345678901234567890123456789012123456789012345678901234567890
12345678901234567890123456789012123456789012345678901234567890121234567890123456789012345678901212345678901234567890123
1234567890123456789012345678901212345678901234567890123456789012123456789012345678901234567890121234567890123
1234567890123456789012345678901212345678901234567890123456789012123456789012345678901234567890121234
123456789012345678901234567890121234567890123456789012345678901212
12345678901234567890123456789012123456789012345678901234567890121234567890123456789012345678901212345678901234567890123456789012123
1234567890123456789012345678901212345678901234567890123456789012123456789012345678901234567890
12345678901234567890123456789012123456789012345678901234567890121234567890123456789012345678901212345678901234567890123
1234567890123456789012345678901212345678901234567890123456789012123456789012345678901234567890121234567890123
1234567890123456789012345678901212345678901234567890123456789012123456789012345678901234567890121234
12345678901234567890123456789012123456789012345678901234567890121234567890123456789012345678901212345678901234567890123456789012123
1234567890123456789012345678901212345678901234567890123456789012123456789012345678901234567890

E
ㄬu ㄬ‰×ß䛽 D ãH÷Ok "Œx #°O_ XH› ‹¬=¶^¥<Œxß 2. xtó`ÇO (A) : ƒì~¡`Ç^͉×O…Õ KÇïHø~¡ „¬iã‰×=°° JkH›OQê

D
9
ZOKÇ°HË"Œe.

59
Q® ~ŒRO L`Çë~¡ã„¬^͉˜.

A R Hê~¡}O :
CA (R)

95
xtó`ÇO =°i†Çò Hê~¡}O J~ò#„¬ô_È° L`Çë~¡ã„¬^͉˜…Õ L`Çæuë JQ®° K³~¡ä›½ <Œ‹²~¡H›O

a) A =°i†Çò R ° ï~O_ȶ ‹¬i†³Ø°# ㄬH›@#…è. Hê~¡}O K³~¡ä›½.

50
(R), xtó`ÇO (A) #䛽 ‹¬i†³Ø°# q=~¡} W‹¬°ëOk. 3. xtó`ÇO (A) : ƒì~¡`Ç^͉×O…Õ ƒçQ®°¾ =#~¡°° J`ǼkH›OQê

98
A
b) A =°i†Çò R ° ï~O_ȶ ‹¬i†³Ø°# ㄬH›@#…è. Hêx
=ô#ß ã‡O`ÇO KË\ì <ŒQ·„¬î~Ÿ.
:8
Hê~¡}O (R) xtó`ÇO (A) 䛽 ‹¬i†³Ø°# q=~¡} Hê^ΰ.
Hê~¡}O (R) :
E

KË\ì <ŒQ·„¬î~Ÿ XH› „Ô~”¡ ƒ’¶q°.

c) (A) (R)
h

xtó`ÇO (A) :
.P

xtó`ÇO ‹¬i†³Ø°#^Í Hêx Hê~¡}O ‹¬i†³Ø°#k


4. 21 =¶ió =°i†Çò 23 ̋̄“O|~¡° ~ËA…Õ
IC
AD

Hê^ΰ.
~Œãu ‹¬=°†Ç°O =°i†Çò „¬Q®\ ÷ ‹¬=°†Ç°O

d) xtó`Ç O (A) ‹¬ i †³ Ø ° #kHê^Î ° Hêx Hê~¡ } O (R)


AB
O

‹¬=¶#OQê =ôO@°Ok.

‹¬i†³Ø°#^Í.
Hê~¡}O (R) : ƒ’¶=°^Î¼ö~Y ï~O_È° ^Î$"Œä›½ ‹¬=¶#
ER

e) (A) (R)
CH

xtó`ÇO Hê~¡}O ï~O_ȶ `DŽ¬C ㄬH›@#°.


^ζ~¡O…Õ =ôO@°Ok.

(A) (R)
YD

ㄬ‰ß
× …Õ W=Þ|_# xtó`ÇO =°i†Çò Hê~¡}O
5. xtó`ÇO (A) : Hù=ôށ#° PH©žH›~¡}O K͋²#„¬ô_È° Jq
,H

#° ä›Æ½}âOQê „¬ijeOz Wzó# <Œ°Q®° ‹¬=¶^¥<Œ


q_È ° ^Î  KÍ † Ç ò ‰× H ÷ ë Hê~ËÄ Ì‚á ì ã_Í \ ˜#°
T
AM

#°O_ ‹¬i†³Ø°# ‹¬=¶^¥#O#° ZOKÇ°HË"Œe.


PH©žH›~¡}O K͋²#„¬ô_È° q_È°^΁†Í°¼ ‰×H÷ë…Õ

1. xtó`ÇO (A) :
H

P#H› @ “  …Õ x°=KÍ ‹ ² # h\ ÷ H ÷ ‹² Ö u ‰× H ÷ ë
M

‹¬Q®O =ôO@°Ok.

Hê~¡}O :
LO@°Ok.
AM

(R)
IG

Hù=ôށ…Õ =ôO_Í PH÷ž[<£, Hê~ËÄ Ì‚áìã_Í\˜…Õ

Hê~¡}O (R) : ‹²u


Ö ‰×Hë÷ =°i†Çò Q®u ‰×H
ë÷ "³ò`ÇO
ë ~¡ª†Ç°#
KH

=ôO_Í PH÷ž[<£ H›O>è `Ç䛽ø=.

‰×H÷ë J=ô`Ç°Ok.

1234567890123456789012
12345678901234567890123456789012123456789012345678901234567890121234567890123456789012345678901212345678901234567890123
12345678901234567890123
R

12345678901234567890123456789012123456789012345678901234567890121234567890123456789012345678901212345678901234567890123
1234567890123456789012 12345678901234567890123
12345678901234567890123456789012123456789012345678901234567890121234567890123456789012345678901212345678901234567890123
1234567890123456789012
1234567890123456789012
1234567890123456789012 q=~¡}ì`ÇàH› ["Œ|°°
12345678901234567890123
12345678901234567890123456789012123456789012345678901234567890121234567890123456789012345678901212345678901234567890123
PRACTICE TEST - 3 :
12345678901234567890123
12345678901234567890123456789012123456789012345678901234567890121234567890123456789012345678901212345678901234567890123
12345678901234567890123456789012123456789012345678901234567890121234567890123456789012345678901212345678901234567890123
1234567890123456789012
1234567890123456789012
12345678901234567890123
12345678901234567890123
12345678901234567890123456789012123456789012345678901234567890121234567890123456789012345678901212345678901234567890123
12345678901234567890123

1. (3) xtó`ÇO (A) ‹¬i†³Ø°#k Hêx Hê~¡}O (R) ‹¬i†³Ø°#k 4. (1) xtó`ÇO (A) =°i†Çò Hê~¡}O (R) ï~O_ȶ ‹¬i†³Ø°#"Í

Hê^ΰ.
=°i†Çò Hê~¡}O, xtó`ÇO#䛽 ‹¬i†³Ø°# q=~¡}.

ƒ’¶ L„¬i`DžìxH÷ HùO`Ç Z`Ç°ë…Õ ‹²Ö~¡OQê =ô#ß =‹¬°ë=ô


D ~
ï O_È° ~ËA…Õ ƒ¶
’ =°^¼
Î ~
ö Y䛽 =°i†Çò ^$
Î "Œä›½

xßO\ ÷…Õ ‹²Öu‰×H÷ë { Q®u‰×H÷ë « †Ç¶OãuH› ‰×H÷ë.


=°^Î¼ ^ζ~¡O ‹¬=¶#O J=ô`Ç°Ok.

2. (2) (A) (R)


5. (5) (A) (R)
xtó`ÇO =°i†Çò Hê~¡}O ° ~
ï O_ȶ ‹¬i†³°
Ø #"Í.
xtó`ÇO =°i†Çò Hê~¡}O ‹¬i†³Ø°#q Hê=ô.

Hêx Hê~¡}O xtó`ÇO#䛽 ‹¬i†³°


Ø # q=~¡} Hê^ΰ.

3. (2) (A) (R)


Hê~ùÄ̂áìã_Í\˜ PH©žH›~}
¡ O =# q_È°^΁†Í°¼ ‰×Hë÷ H›O>è
xtó`ÇO =°i†Çò Hê~¡}O ° ~
ï O_ȶ ‹¬i†³°
Ø #"Í.

ãHù=ôށ PH©žH›~¡}O =#<Í JkH› ‰×H÷ë q_È°^΁Q®°#°.


Hêx Hê~¡}O, xtó`ÇO#䛽 ‹¬i†³°
Ø # q=~¡} Hê^ΰ.

REASONING ABILITY (LOGICAL) ○ ○ ○ ○ ○ ○ ○ ○


175
○ ○ ○ ○ ○ ○ ○ ○ ○ ○ ○ ○ ○ ○ ○ ○ ○ ○ ○
Study Material
123456789012345678901234567890121234567890123456789012345678901212
123456789012345678901234567890121234567890123456789012345678901212
12345678901234567890123456789012123456789012345678901234567890121234567890123456789012345678901212345678901234567890123456789012123
123456789012345678901234567890121234567890123456789012345678901212345678901234567890123
12345678901234567890123456789012123456789012345678901234567890121234567890123456789012345678901212345678901234567890123
1234567890123456789012345678901212345678901234567890123456789012123456789012345678901234567890121234567890123
1234567890123456789012345678901212345678901234567890123456789012123456789012345678901234567890121234
12345678901234567890123456789012123456789012345678901234567890121234567890123456789012345678901212345678901234567890123456789012123
123456789012345678901234567890121234567890123456789012345678901212345678901234567890123
12345678901234567890123456789012123456789012345678901234567890121234567890123456789012345678901212345678901234567890123
1234567890123456789012345678901212345678901234567890123456789012123456789012345678901234567890121234567890123
1234567890123456789012345678901212345678901234567890123456789012123456789012345678901234567890121234
123456789012345678901234567890121234567890123456789012345678901212
12345678901234567890123456789012123456789012345678901234567890121234567890123456789012345678901212345678901234567890123456789012123
123456789012345678901234567890121234567890123456789012345678901212345678901234567890123
12345678901234567890123456789012123456789012345678901234567890121234567890123456789012345678901212345678901234567890123
1234567890123456789012345678901212345678901234567890123456789012123456789012345678901234567890121234567890123
1234567890123456789012345678901212345678901234567890123456789012123456789012345678901234567890121234
PREVIOUS BITS
12345678901234567890123456789012123456789012345678901234567890121234567890123456789012345678901212345678901234567890123456789012123
123456789012345678901234567890121234567890123456789012345678901212345678901234567890123
123456789012345678901234567890121234567890123456789012345678901212
12345678901234567890123456789012123456789012345678901234567890121234567890123456789012345678901212345678901234567890123
1234567890123456789012345678901212345678901234567890123456789012123456789012345678901234567890121234567890123
1234567890123456789012345678901212345678901234567890123456789012123456789012345678901234567890121234
12345678901234567890123456789012123456789012345678901234567890121234567890123456789012345678901212345678901234567890123456789012123
123456789012345678901234567890121234567890123456789012345678901212345678901234567890123
123456789012345678901234567890121234567890123456789012345678901212
12345678901234567890123456789012123456789012345678901234567890121234567890123456789012345678901212345678901234567890123
1234567890123456789012345678901212345678901234567890123456789012123456789012345678901234567890121234567890123
1234567890123456789012345678901212345678901234567890123456789012123456789012345678901234567890121234
12345678901234567890123456789012123456789012345678901234567890121234567890123456789012345678901212345678901234567890123456789012123
123456789012345678901234567890121234567890123456789012345678901212345678901234567890123
12345678901234567890123456789012123456789012345678901234567890121234567890123456789012345678901212345678901234567890123
1234567890123456789012345678901212345678901234567890123456789012123456789012345678901234567890121234567890123
1234567890123456789012345678901212345678901234567890123456789012123456789012345678901234567890121234
12345678901234567890123456789012123456789012345678901234567890121234567890123456789012345678901212345678901234567890123456789012123
123456789012345678901234567890121234567890123456789012345678901212345678901234567890123
12345678901234567890123456789012123456789012345678901234567890121234567890123456789012345678901212345678901234567890123456789012123
123456789012345678901234567890121234567890123456789012345678901212345678901234567890123

G 1 #°O_ 5 =~¡ä½› Q® ㄬ‰ß× ä›½ : ㄬu ㄬ‰ß× …Õ ~ò=Þ|_# G 7 #°O_ 16 =~¡ä›½ Q® ㄬ‰×߁䛽 xö~œ‰×Hꁰ : ãH÷Ok
xtó`Ç`ÇÞO (A), Hê~¡}O (R) ä›½ ãH÷Ok SzóHê…Õ ã„¬ ‰ × ß ä› ½ ㄬ u ^¥x…Õx xtó`Ç ` Ç Þ =ò (A) =°i†Ç ò
xö~œ‰×Hꁰ. Hê~¡}=ò (R) J<Í ï~O_È° ㄬ=KÇ<Œ° LO\ì~ò.
1) (A), (R) ° ï~O_ȶ x[=ò =°i†Çò (A) (R) H÷
1) A =°i†Çò R ‹¬`Ǽ=ò, J=ô`Ƕ (A) H÷ (R) ‹¬ï~á#

‹¬i†³Ø°# q=~¡} J~ò`Í (1),


q=~¡} J~ò`Í (1) Jh 

2) (A), (R) ° ï~O_ȶ x[=ò =°i†Çò (A) (R) H÷


2) A =°i†Çò R ‹¬`Ǽ=ò, J=ô`Ƕ (A) H÷ (R) ‹¬ï~á#

‹¬i†³Ø°# q=~¡} Hê^ΰ J~ò`Í (2),


q=~¡} HêH›‡é`Í (2) Jh 

3) (A) x[=ò, Hêx (R) `DŽ¬C J~ò`Í (3),


3) (A) (R)
4) (A) (R)
‹¬`Ǽ=ò =°i†Çò J‹¬`Ǽ"³°Ø`Í (3) Jh 

4) (A) (R)
`Ç „ ¬ C , Hêx x[=ò J~ò`Í (4), Jx

Y
J‹¬`Ǽ=ò =°i†Çò ‹¬`Ǽ"³°Ø`Í (4) Jh g°

Z#°ßHù#°=ò. (A.P. S.I. Prelims -2016)


(A.P. S.I. Mains -2016)

M
SzóHêxß ZOKÇ°HËO_.

1. xtó`Ç`ÇÞO (A) : 1939…Õ ‹¬°ƒì+¹KO


Ç ã^΃Ջ¹ ƒì~¡`Ç *ìf†Ç°

7. xtó`Ç`ÇÞO (A) : ‚² ì =¶†Ç ° #^Î ° …Õ J`Ç ¼ k H › = ò r=

E
HêOãïQ‹¹ †³òH›ø J^Î¼H›Æ „¬^ÎqH÷ ~Œr<Œ=¶

#^ΰ°.

D
K͉§~¡°.

9
Hê~¡}O : Hê~¡}O (R) :

59
H› ~ ¡ ° Q® ° `Ç ° #ß =°OKÇ ° `Ë Jq xO„¬ | _È °
(R) ‹¬°ƒì+¹KÇOã^΃Ջ¹ ƒì~¡`Ç ªÞ`ÇOã`Ç bQ·#°

CA
95
`Ç°<Œß~ò.
Pö Q ß†Ç ° P‹² † Ç ¶ …Õ ㇐~¡ O a O KÇ _ ¨xH÷ ,

xtó`Ç`ÇÞO :

50
HêOãïQ‹¹#° =kē\ ÷“ "³o¤<Œ~¡°. 8. (A) B~¡ O Q® * è | ° J`Ç x ^Î H › ø <£ q^ ¥ #=ò…Õ

xtó`Ç`ÇÞO :
98
A
2. (A) ‚² ì O^Î ¶ ^Í ‰ × O , L+¬ â 㠄¬ ^ Í ‰ × ~¡ ° `Ç ° „¬ = #O
q„¦¬°_ȆǶ¼_È°.

Hê~¡}O :
:8
=¶ki "Œ`Œ=~¡}O#° H›ey LO@°Ok. (R) B~¡OQ®*|
è ° |°[ûyOKÍ q^¥<Œxß J#°‹¬iOKÇ
E

Hê~¡}O :
h

(R) ‚²ìO^ζ^͉×O, ª†Ç°# ã\Մ²H›…˜ …èH›‡é†Ç¶_È°.


.P
IC

JHÆ ê O‰× = ò =°^ Î ¼ ‹¬ i Qê¾ =¼=‹² Ö ` Ç = ò


9. xtó`Ç`ÇÞO (A) : Hê~Œ¼†Ç°O…Õ ª=¶lH› J#°‹¬O^¥#O
AD

J~ò#k.
J=‹¬~¡O.
AB
O

3. xtó`Ç`ÇÞO (A) : ‹¬¶~¡°¼x #°O_ J`ǼkH› ^ζ~¡O…Õ L#ß Hê~¡}O (R) : ‹¬=¶[O…Õ KŒ…ì "³#°H›|_# `Ç~¡Q®`Ç°°
ER
CH

ãQ®‚¬ìO |°^Î°_È°.
L<Œß~ò.

Hê~¡}O (R) :
YD

xtó`Ç`ÇÞO :
"³ò`ÇëO ‹¬¶~¡¼ä›½@°O|=ò…Õ |°^Î°_È° Ju
10. (A) j`ŒHêO…Õ `³Áx =¢ªë° ^ÎiOKÇ_¨xH÷

z#ß ãQ®‚¬ìO.
,H

ㄬ[° W+¬“„¬_È`Œ~¡°.

xtó`Ç`ÇÞO :
T

4. (A)
Hê~¡}O :
AM

ƒì~¡`^
Ç ‰
Í O
× ã^Î"˼Ä} ‹¬=°‹¬¼#° Z^ΰ~ùøO
(R) `³Áx =¢ªë° "Í_x "³#°H›ä›½ =°ˆ×¤Qù@°“
H

\ÕOk.
M

=°Oz ª^Î<Œ°.

Hê~¡}O (R) : =‹¬°ë=ô ^Î~¡ ̄~¡°Q®°^΁, ã^Î"˼Ä}=ò


AM

xtó`Ç`ÇÞO (A) :
IG

11. „¬x…Õ =°Oz ㄬ^Î~¡Å# `Ç$„²ëx W‹¬°ëOk.


#䛽 Hê~¡}=ò J=ô`Ç°Ok.

Hê~¡}O (R) :
KH

=$uë…Õ ‹¬O`Ç$„²ë „¦¬e`ÇOQê =°Oz ㄬ^Î~¡Å#

5. xtó`Ç`ÇÞO (A) : …Õ‚¬ì=ò°, ƒ’¶q°…Õ„¬ ã^Î=~¡¶„¬O…Õ


R

LO@°Ok.
LO\ì~ò.

Hê~¡}O (R) : ƒ¶


’ q°, ‹¬¶~¡¼ H÷~}
¡ ì#° `Ç#…Õ„¬eH÷ „ԁ°ó
12. xtó`Ç`ÇÞO (A) : |`Œø~¡ ö H ‹¬ ° …Õ <Í ~ ¡ ‹ ¬ ° ë _  „² ` Ç $ `ŒÞxß

䛽O@°Ok.
=°i†Çò Q®°iëO„¬ô x~ŒÖiOKÇ_¨xH÷ 'DNA',
6. xtó`Ç`ÇÞO (A) : P~¡°¼° Q®OQê#k ㇐O`Ç=ò#䛽
"Íe =òã^΁° („¦²OQ®~Ÿ ㄲO\ ÷OQ·) J<Ík

=òY¼ „¬sH›Æ J~ò¼Ok.


q‹¬ëiOKÇ_ÈO…Õ W#°„¬ ªOöHuH› „¬i*ìý#O

=òY¼"³°Ø# ‡ã`Ç#° =‚²ìOzOk.


Hê~¡}O (R) : DNA q‰õ+
Á }
¬ 䛽 "³Oã@°H›°, ZO_# ~¡H=
ë› ò

Hê~¡}O (R) : Q®OQê#k „¬i"Œ‚¬ìH› ㇐O`Ç=°O`Œ


=°i†Çò g~¡¼=ò †³òH›ø ‹¬Þæ #=ü<Œ°

^Î@“"³°Ø# J~¡}¼O LO_Ok. nxß |"³°Ø# (Small samples) ‹¬i‡é`Œ~ò.

…Õ‚¬ì „¬x=ò@Á`Ë =¶ã`Ç"°


Í `ùyOKÇ_¨xH÷
13. xtó`Ç`ÇÞO (A) : \ ©Hꁰ ~ËQê#° x~Ëkªë~ò.

g=ô`Ç°Ok.
Hê~¡}O (R) : \ ©Hê#° „²Áä›½ ~òzó f~Œe.

(T.S. Constable Mains -2016)

REASONING ABILITY (LOGICAL) ○ ○ ○ ○ ○ ○ ○ ○


176
○ ○ ○ ○ ○ ○ ○ ○ ○ ○ ○ ○ ○ ○ ○ ○ ○ ○ ○
Study Material
14. xtó`Ç`ÇÞO (A) : HËÁ~˄¦¬îÁ~Ë Hê~¡Ä#Á "Œ_ÈHêxß D =°^Î¼ ㄬ„¬OKÇ"Œ¼„¬ëOQê xÀ+kOKŒ~¡°.

Hê~¡}O (R) : D ~¡ª†Ç°<Œ Hê~¡}OQê KÇ~¡à Hê¼#ž~Ÿ =‹¬°ëOk.

15. xtó`Ç`ÇÞO (A) : ãQê̄¦á\˜ *ì~¡°_È° ‹¬Þƒì==ò H›k, nxx H›O^³#Qê L„¬†³¶yªë~¡°.

Hê~¡}O (R) : ãQê̄¦á\˜…Õ À‹ÞKŒó Z¢Hê“<£° LO\ì~ò.

16. xtó`Ç`ÇÞO (A) : „ÔKÇ° „¬^¥~¡ÖO =°|^ΜHêxß x~Ëk‹¬°ëOk.

Hê~¡}O (R) : „ÔKÇ° „¬^¥~¡ÖO P‚¬ð~ŒxH÷ PkH›¼`Ç#° ‹¬=°ä›€~¡°ó`Ç°Ok.

12345678901234567890123456789012123456789012345678901234567890121234567890123456789012345678901212345678901234567890123
1234567890123456789012 123456789012345678901234
12345678901234567890123456789012123456789012345678901234567890121234567890123456789012345678901212345678901234567890123
1234567890123456789012 123456789012345678901234
1234567890123456789012
q=~¡}ì`ÇàH› ["Œ|°°
12345678901234567890123456789012123456789012345678901234567890121234567890123456789012345678901212345678901234567890123
123456789012345678901234
12345678901234567890123456789012123456789012345678901234567890121234567890123456789012345678901212345678901234567890123
1234567890123456789012
PREVIOUS BITS :
123456789012345678901234
12345678901234567890123456789012123456789012345678901234567890121234567890123456789012345678901212345678901234567890123
1234567890123456789012
1234567890123456789012
123456789012345678901234
12345678901234567890123456789012123456789012345678901234567890121234567890123456789012345678901212345678901234567890123
123456789012345678901234
12345678901234567890123456789012123456789012345678901234567890121234567890123456789012345678901212345678901234567890123
1234567890123456789012 123456789012345678901234

1. (3) xtó`Ç`Þ
Ç =ò ‹¬i†³°
Ø #k Hêx, Hê~¡}O ‹¬i†³°
Ø #k Hê^ΰ. 9. (2) A, R ° ï~O_ȶ ‹¬`Œ¼…è. A 'R'
H÷ ‹¬i†³Ø°# q=~¡}

Y
Hê=ô# nxH÷ ‹¬=¶^¥#=ò (3). W=Þ^ΰ. Hê=ô# nxH÷ ‹¬=¶^¥#=ò (2).

M
2. (3) xtó`Ç`Þ
Ç =ò ‹¬i†³°
Ø #k Hêx, Hê~¡}O ‹¬i†³°
Ø #k Hê^ΰ.
10. (4) A `DŽ¬C, R x[=ò.

E
‚²ìO^ζ^͉×O, ª†Ç°# ã\Մ²H›…˜ JHÆêO‰×=ò =°^Î¼<Í
`³  Á x ^Î ° ‹¬ ° ë  ° "Í _  x "³ # °H› ä › ½ =°ˆ× ¤ Qù@°“ =°Oz

Hê䛽O_¨ W`Ç~¡ KË@Á 䛀_¨ =¼=‹²Ö`Ç"³°ØOk. Hê=ô#

D
9
ª^Î<Œ°. Hê=ô# nxH÷ ‹¬=¶^¥#=ò (4).

59
nxH÷ ‹¬i†³Ø°# ‹¬=¶^¥#O (3).
11. (3) A x[=ò, Hêx R `DŽ¬C.

3. (4)
CA
95
‹¬¶~¡°¼xH÷ Ju ^ÎQ®¾~¡…Õ L#ß ãQ®‚¬ìO |°^Î°_È°. Hê=ô#
„¬x…Õ =°Oz ㄬ^~
Î Å
¡ # J<ÍH~
› H
¡ ꁰQê =°# ƒq
’ +¬¼`Ç°ä
ë ½
›

'A'

50
`DŽ¬C.
‹¬‚¬ð†Ç°„¬_È°`Ç°Ok. Hê=ô# `Ç$„²ëx W‹¬°ëOk.

98
A
"³ò`ÇëO ‹¬¶~¡¼ä›½@°O|O…Õ |°^Î°_È° (<³„¦¬î“ ¼<£ `Ç~ŒÞ`Ç)
Hê=ô# nxH÷ ‹¬=¶^¥#=ò (3).

'R'
:8
Juz#ß ãQ®‚¬ìO. Hê=ô# x["Í°.
12. (1) A, R A 'R'
E

° ï~O_ȶ ‹¬`Œ¼…è. H÷ ‹¬i†³Ø°# q=~¡}

4. (3)
h

ㄬ‹¬°ë`Ç ƒì~¡`Ç^͉×O ã^Î"˼Ä} ‹¬=°‹¬¼#°


.P

W‹¬°ëOk. Hê=ô# nxH÷ ‹¬=¶^¥#=ò (1).


IC

'A'
13. (2) A, R A 'R'
Z^ΰ~ùøO@°Ok. Hê=ô# x[=ò.
AD

° ~
ï O_ȶ ‹¬`Œ¼…è. Hêx H÷ ‹¬i†³°
Ø # q=~¡}

XH› ø =‹¬ ° ë = ô ^ Î ~ ¡  ̄~¡ ° Q® ° ^Î … è Hê^Î ° . W`Ç ~ ¡ ã `Œ


AB
O

Hê^ΰ. Hê=ô# nxH÷ ‹¬=¶^¥#=ò (2).

14. (3) A
Hê~¡}쁰 䛀_¨ L<Œß~ò. ã^Î"˼Ä}O ̄~¡Q®\ìxH÷
ER

'R'
CH

x[=ò.

CFC
Hê=ô# `DŽ¬C. Hê=ô# nxH÷ ‹¬i†³Ø°# ‹¬=¶^¥#=ò
YD

 "Œ_ÈHêxß ã„¬„¬OKÇ "Œ¼„¬ëOQê x~ËkOKŒ~¡°.

'R'
(3).

5. (3)
Hêx `DŽ¬C.
,H

…Õ‚¬ì=ò°, ƒ’¶q° …Õ„¬ ã^Î=~¡¶„¬O…Õ<Í Z䛽ø=

Environment
T

'A'
"Œ\ ÷=Á ^³|Ä uO@°Ok.
AM

"³ò`ÇëO…Õ ƒ’¼=°=ô`Œ~ò. Hê=ô# x[=ò.

15. (2) A, R
H

° ï~O_ȶ ‹¬`Œ¼…è.
M

ƒ’¶q°, ‹¬¶~¡¼H÷~¡}ì#° „¬~Œ=~¡ë#O K³Ok‹¬°ëOk. Hêx

'R'
AM

ãQê̄¦ á \ ˜ *ì~¡ ° _È ° ‹¬ Þ ƒ ì ==ò H›  k Hê=ô# nxx


IG

„ԁ°óHù#^ΰ. Hê=ô# `DŽ¬C.

H› O ^³ # Qê L„¬ † ³ ¶ yªë ~ ¡ ° . nxH÷ Hê~¡ } O À‹ÞKŒó


KH

Hê=ô# nxH÷ ‹¬i†³Ø°# ‹¬=¶^¥#=ò (3).

6. (1) A, R A 'R' Z¢Hê“<£° Hê^ΰ. Hê=ô# A H÷ 'R' ‹¬i†³Ø°# q=~¡}


R

° ï~O_ȶ x*ì…è, H÷ ‹¬i†³Ø°# q=~¡}

W=Þ^ΰ.
W‹¬°ëOk. Hê=ô# nxH÷ ‹¬=¶^¥#=ò (1).

7. (1) A, R ° ~
ï O_ȶ ‹¬`Œ¼…è =°i†Çò A H÷ 'R' ‹¬i†³°
Ø #
16. (1) A, R ° ï~O_ȶ ‹¬`Œ¼°.

„ÔK°
Ç „¬^¥~¡O
Ö =°|^ÎH
œ êxß x~Ëk‹°
¬ O
ë k. Z@° J#Qê
q=~¡} W‹¬°O
ë k. Hê=ô# nxH÷ ‹¬i„¬_°
È ‹¬=¶^¥#O (1).

8. (1) A, R ° ï~O_ȶ ‹¬`Œ¼…è. A 'R'H÷ ‹¬i†³Ø°# q=~¡}


„ÔKÇ°„¬^¥~¡ÖO P‚¬ð~ŒxH÷ PkH›¼`Ç#° ‹¬=°ä›€~¡°ó`Ç°Ok.

W‹¬°ëOk. Hê=ô# nxH÷ ‹¬=¶^¥#=ò (1).


Hê=ô# A 'R'H÷ ‹¬i†³Ø°# q=~¡} W‹¬°ëOk.

RÿRÿR

REASONING ABILITY (LOGICAL) ○ ○ ○ ○ ○ ○ ○ ○


177
○ ○ ○ ○ ○ ○ ○ ○ ○ ○ ○ ○ ○ ○ ○ ○ ○ ○ ○
Study Material
_¨\ì ‹¬„¦²+²†Ç°hž (^Î`ŒëO‰× †³¶Q®¼`Ç)
5 (Data Sufficiency)

=òMì¼O‰§°...
G D qƒìQ®O…Õx ㄬ‰×߁䛽 ‹¬=¶^¥#O H›#°Qù#°@䛽 XH› ãH›=°|^Μ"³°Ø# q^¥#O#° ‡\÷OKŒe.
G ㄬ‰×߁…Õ Wzó# ^Î`ŒëO‰§xß ä›Æ½}âOQê q‰õÁ+²Oz ‹¬=¶^¥#O x~¡â~òOKŒe.
G W=Þ|_# ㄬH›@#…Õ ㄬH›@# =¶ã`Ç"Í° ㄬ‰×ß䛽 ‹¬=¶^¥#O W=ÞQ®ey`Í ‹¬=¶^¥#O Qê#¶,
I (a)

ㄬH›@# #°O_ =¶ã`Ç"Í° ㄬ‰×ß䛽 „¬îië ‹¬=¶^¥#O ‡ÚO^ÎQ®ey`Í ‹¬=¶^¥#O Qê#¶,

Y
II (b)

ï~O_È° ㄬH›@#¶ ㄬ‰×ß䛽 ‹¬=¶^¥#O Hù~¡ä›½ Yzó`ÇOQê ^Î`ŒëO‰×O#° Wzó#@Á~ò`Í ‹¬=¶^¥#O Qê#¶,

M
(c)

U ㄬH›@<Œ ‹¬=¶^¥<ŒxH÷ ‹¬i„¬_¨ ^Î`ŒëO‰×O#° H›ey†ÇòO_ÈH›‡é`Í ‹¬=¶^¥#O Qê#¶, (d)

ï~O_È° ㄬH›@#…Õ U^Ë XH›\ ÷ J#Qê "³ò^Î\ ÷k …è^¥ ï~O_È=k ‹¬=¶^¥#O W=ÞQ®eöQ ^Î`ŒëO‰×O#° H›ey†ÇòO>è ‹¬=¶^¥#O

E
Qê#¶ `³e†Ç°*è†Ç¶e.

D
9
59
(e)

G ㄬ‰×߁…Õ W=Þ|_# ㄬH›@##° P^¥~¡OQê K͋¬°ä›½x ㄬ‰×ß䛽 ‹¬=¶^¥#O ~Œ|@“Q®=¶ ? …è^¥ ? J<Í q+¬†Ç¶xß =¶ã`Ç"Í°
CA
95
Q®°iëOKŒe.

50
G W=Þ|_# Jxß ã„¬H›@##° L„¬†³¶yOz ㄬ‰×ß䛽 ‹¬=¶^¥#O H›#°Qù<Œež# J=‹¬~¡O …è^ΰ. 98
A
G Wzó# ㄬ‰×ß䛽 ‹¬=¶^¥#O ‡ÚO^ÎQ® ‹¬=¶KŒ~¡O ^Î`ŒëO‰×O…Õ =ôO^Ë ? …è^Ë ? KǶ‹¬°HË"Œe.
:8

G D qƒìQ®O…Õ J~¡Ö"³°\÷H›…˜ s[xOQ·, ~¡H›ë ‹¬O|O^¥°, HË_OQ· - _™HË_OQ·, ‹Ô\ ÷OQ· Jï~O*˜"³°O\˜, Z#…ÿá\ ÷H›…˜ s[xOQ·...
E
h

"³ò^΁Q®° q+¬†Ç¶ä›½ ‹¬O|OkOz# ㄬ‰×߁#° J_ÈQ®=KÇ°ó.


.P
IC
AD

=¶ki ㄬ‰×߁°
AB
O
ER
CH
YD

1. XH› …ÿáã|s…Õ WOwÁ+¬µ, ‚²ìOn =°i†Çò `³°Q®° #=° ㄬH›@# II : ‹¬°ö~+¹Hê~¡°, WO\÷ #°O_ |†Ç°°^Íi#
,H

=¶ã`Ç"°Í =ô#ßq. P …ÿãá |s…Õ Q® `³°Q®° #=° ‹¬OY¼ 45 xq°‘ `Ç~¡°"Œ`Ç 10 :15 a.m.
T
AM

ZO`Ç? 䛽 P„¦Ô‹¹ä›½ KÍi#k.


H

ㄬH›@# I : …ÿáã|s…Õ Q® 1000 #=…Õ 50%


M

ª^Î#: ㄬ‰×ß䛽 ‹¬O|OkOz# U ^Î`ŒëO‰×=ü ㄬH›@#-I H›ey


AM

#=° WOwÁ+µ¬ =°i†Çò ‚²ìOn #=°.


IG

†ÇòO_ȅè^ΰ.

ㄬH›@# II : ‚²ìOn #= ‹¬OY¼ WOwÁ+¬µ #=


KH

ㄬH›@#-II ㄬHê~¡O ‹¬°ö~+¹ WO\ ÷ =^Îí #°O_ P„¦Ô‹¬°ä›½

‹¬OY¼ H›<Œß ï~\÷“O„¬ô.


R

|†Ç°°^Íi# ‹¬=°†Ç°O « 10:15 a.m. - 45 xII°

ª^Î#: ㄬH@
› #- I #°O_ `³°Q®° #= ‹¬OY¼ « 50% (1000)
« 9:30 a.m.
« 500

Hê=ô# ㄬ H › @ #- II ㄬ ‰ × ß ä› ½ ‹¬ = ¶^ ¥ #O W=ÞQ® 

ㄬ H › @ #- II "³ ò `Ç ë O #= "³ ò `Ç ë O ‹¬ O Y¼#° H› e y

^Î`ŒëO‰×O#° H›ey†Çò#ßk.
†ÇòO_ȅ^
è °
Î . Hê=ô# `³°Q®° #= ‹¬OY¼#° H›#°Qù#°@䛽

Hê=ô# ‹¬=¶^¥#O (b).


Hê=‹²# ^Î`ŒëO‰×O ㄬH›@#-II H›ey†ÇòO_ȅè^ΰ.

Hê=ô# ‹¬=¶^¥#O (a).


3. ~¡q <³ r`ÇO `4,000. 䛽=¶~Ÿ †³òH›ø <³r`ÇO ZO`Ç?
2. ‹¬°ö~+¹ WO\÷ #°O_ P„¦Ô‹¬°H÷ U ‹¬=°†Ç°O…Õ |†Ç°° ㄬH›@# I : 䛽=¶~Ÿ †³òH›ø r`ÇO, ~¡q †³òH›ø r`ÇO
^Í~¡`Œ_È°? H›<Œß ` 500 Z䛽ø=.
ㄬH›@# I : ‹¬°ö~+¹ 9 :15 a.m. 䛽 XH› ‡¦é<£Hꅘ ㄬH›@# II : ~¡q =°i†Çò 䛽=¶~Ÿ ‹¬~Œ‹¬i r`ÇO
i‹Ô"£ K͋¬°ä›½<Œß_È°. `4250.

REASONING ABILITY (LOGICAL) ○ ○ ○ ○ ○ ○ ○ ○


178
○ ○ ○ ○ ○ ○ ○ ○ ○ ○ ○ ○ ○ ○ ○ ○ ○ ○ ○
Study Material
ª^Î#: ㄬH›@#-I #°O_ 䛽=¶~Ÿ r`ÇO « ` 4000 { ` 500 ª^Î#: =°OQ®ˆ×"Œ~¡O `ŒsY° W=Þ|_ȅè^ΰ. Hê=ô# H›$‘â~Œ=ô

« ` 4500 „¬ô\ ÷“#~ËA H›#°Qù#°@䛽 Hê=‹²# ^Î`ŒëO‰×O ï~O_È°

5111!|!)圾>· !saÈP* ㄬH›@#° H›ey†ÇòO_ȅè^ΰ.

ㄬH›@#-II #°O_
3 « 4250
Hê=ô# ‹¬=¶^¥#O (d).

䛽=¶~Ÿ r`ÇO « 8500 - 4000 « ` 4500


5. ªO|†Ç°¼ †³òH›ø `ÇOã_ Z=~¡° ?
Hê=ô# ㄬH›@#-I …è^¥ ㄬH›@#-II …Õ U^Ë XH›\ ÷
› # I : ªO|†Ç°¼ =°i†Çò 䛽=¶~Ÿ J#ß^Î=òà°.
ㄬH@
‹¬=¶^¥#O ‡ÚO^ÎQ®eöQ ‹¬=¶KŒ~¡O H›ey†Çò#ßk.
ㄬH›@# II : 䛽=¶~Ÿ †³òH›ø `ÇeÁ H›$+¬â ªé^Î~¡°x †³òH›ø
Hê=ô# ‹¬=¶^¥#O (e). ƒì~¡¼.
4. D ‹¬O=`Ǟ~¡O #=O|~¡° <³…Õ U ~ËA H›$‘â~Œ=ô ª^Î#: Wzó# ㄬH@
› # ㄬHê~¡O, 䛽=¶~Ÿ =°i†Çò ªO|†Ç°¼

†³òH›ø „¬ô\÷“#~ËA ? †³òH›ø `ÇeÁ H›$+¬â ªé^Î~°


¡ x †³òH›ø ƒì~¡¼. Hê=ô#, ªO|†Ç°¼

Y
ㄬH›@# I : H› $ ‘â ~ Œ=ô „¬ ô \÷ “ # ~ËA 8= `ŒsY° `ÇOã_ H›$+¬â ªé^Î~¡°_È°. Wzó# ï~O_È° ㄬH›@#°

`Ç~¡°"Œ`Ç Hêx 11= `ŒsY° …Õ„¬ô.

M
L„¬†³¶yOz ‹¬=¶^¥#O ~Œ|@“|_#k.

ㄬH›@# II : H›$‘â~Œ=ô „¬ô\÷“#~ËA =°OQ®ˆ×"Œ~¡O. Hê=ô# ‹¬=¶^¥#O (c).

E
123456789012345678901234567890121234567890123456789012345678901212
12345678901234567890123456789012123456789012345678901234567890121234567890123456789012345678901212345678901234567890123
1234567890123456789012345678901212345678901234567890123456789012123456789012345678901234567890121234567890123
1234567890123456789012345678901212345678901234567890123456789012123456789012345678901234567890121234
12345678901234567890123456789012123456789012345678901234567890121234567890123456789012345678901212345678901234567890123456789012123
1234567890123456789012345678901212345678901234567890123456789012123456789012345678901234567890
12345678901234567890123456789012123456789012345678901234567890121234567890123456789012345678901212345678901234567890123
1234567890123456789012345678901212345678901234567890123456789012123456789012345678901234567890121234567890123
1234567890123456789012345678901212345678901234567890123456789012123456789012345678901234567890121234
123456789012345678901234567890121234567890123456789012345678901212
12345678901234567890123456789012123456789012345678901234567890121234567890123456789012345678901212345678901234567890123456789012123
1234567890123456789012345678901212345678901234567890123456789012123456789012345678901234567890
12345678901234567890123456789012123456789012345678901234567890121234567890123456789012345678901212345678901234567890123
1234567890123456789012345678901212345678901234567890123456789012123456789012345678901234567890121234567890123
1234567890123456789012345678901212345678901234567890123456789012123456789012345678901234567890121234
123456789012345678901234567890121234567890123456789012345678901212
12345678901234567890123456789012123456789012345678901234567890121234567890123456789012345678901212345678901234567890123456789012123
1234567890123456789012345678901212345678901234567890123456789012123456789012345678901234567890
PRACTICE TEST - 1
12345678901234567890123456789012123456789012345678901234567890121234567890123456789012345678901212345678901234567890123
1234567890123456789012345678901212345678901234567890123456789012123456789012345678901234567890121234567890123
1234567890123456789012345678901212345678901234567890123456789012123456789012345678901234567890121234
12345678901234567890123456789012123456789012345678901234567890121234567890123456789012345678901212345678901234567890123456789012123
1234567890123456789012345678901212345678901234567890123456789012123456789012345678901234567890

D
9
123456789012345678901234567890121234567890123456789012345678901212
12345678901234567890123456789012123456789012345678901234567890121234567890123456789012345678901212345678901234567890123
1234567890123456789012345678901212345678901234567890123456789012123456789012345678901234567890121234567890123
1234567890123456789012345678901212345678901234567890123456789012123456789012345678901234567890121234
12345678901234567890123456789012123456789012345678901234567890121234567890123456789012345678901212345678901234567890123456789012123
1234567890123456789012345678901212345678901234567890123456789012123456789012345678901234567890
12345678901234567890123456789012123456789012345678901234567890121234567890123456789012345678901212345678901234567890123
1234567890123456789012345678901212345678901234567890123456789012123456789012345678901234567890121234567890123
1234567890123456789012345678901212345678901234567890123456789012123456789012345678901234567890121234
12345678901234567890123456789012123456789012345678901234567890121234567890123456789012345678901212345678901234567890123456789012123
1234567890123456789012345678901212345678901234567890123456789012123456789012345678901234567890
123456789012345678901234567890121234567890123456789012345678901212

59
12345678901234567890123456789012123456789012345678901234567890121234567890123456789012345678901212345678901234567890123
1234567890123456789012345678901212345678901234567890123456789012123456789012345678901234567890121234567890123
1234567890123456789012345678901212345678901234567890123456789012123456789012345678901234567890121234
12345678901234567890123456789012123456789012345678901234567890121234567890123456789012345678901212345678901234567890123456789012123
1234567890123456789012345678901212345678901234567890123456789012123456789012345678901234567890

G (1-15) ㄬ‰×ß ãH÷O^Î W=Þ|_# ㄬH›@#…Õ ㄬH›@# CA


95
5. U ~ËA# "³OH›>è+¹ „¬ô\÷“#~ËA J=ô`Ç°Ok?I

=¶ã`Ç"Í° ㄬ‰×ß䛽 ‹¬=¶^¥#O W=ÞQ®ey`Í ‹¬=¶^¥#O


50
ㄬH›@# : I

Qê#¶,
"³ O H› > è + ¹ „¬ ô \ ÷ “ # ~ËA A…ÿ á 17 H› < Œß

98
A
(a) =òO^ΰQê<Í =ôO@°O^Îx ~¡[hHêO`Ÿ

ㄬH›@# #°O_ =¶ã`Ç"Í° ㄬ‰×ß䛽 „¬îië ‹¬=¶^¥#O


:8
II

‡ÚO^ÎQ®ey`Í ‹¬=¶^¥#O Qê#¶,


K³‡æ_È°.
E

ㄬH›@# :
h

(b) II "³OH›>è+¹ „¬ô\ ÷“#~ËA A…ÿá 15 `Ç~ŒÞ`Ç<Í

~ï O_È° ã„H¬ @› #¶ ㄉ¬ ß× ä›½ ‹¬=¶^¥#O Hù~¡ä½› Yzó`ÇOQê


.P
IC

^Î`ŒëO‰×O#° Wzó#@Á~ò`Í ‹¬=¶^¥#O Qê#¶,


=ôO@°O^Îx z~¡Orq K³‡æ_È°.
AD

6. `Ç~¡Q®u…Õ "³ò`ÇëO ZO`Ç=°Ok q^¥¼~¡°Ö° Q®~¡°?


(c)

U ㄬH›@<Œ ‹¬=¶^¥<ŒxH÷ ‹¬i„¬_¨ ^Î`ŒëO‰×O#° ㄬH›@# :


AB
O

H›ey†ÇòO_ÈH›‡é`Í ‹¬=¶^¥#O Qê#¶,


20 H›<Œß Z䛽ø= =°i†Çò 27 H›<Œß `Ç䛽ø=
ER

(d)
CH

ï~O_È° ㄬH›@#…Õ U^Ë XH›\ ÷ J#Qê "³ò^Î\ ÷k …è^¥


=°Ok q^¥¼~¡°Ö° `Ç~¡Q®u…Õ =ô<Œß~¡°.

ㄬH›@# : II `Ç~¡Q®u…Õ 24 H›<Œß Z䛽ø= =°i†Çò 31

ï ~ O_È = k ‹¬ = ¶^ ¥ #O W=ÞQ® e ö Q ^Î ` Œë O ‰× O #°


YD

H›ey†ÇòO>è ‹¬=¶^¥#O Qê#¶ `³e†Ç°*è†Ç¶e.


H› < Œß `Ç ä › ½ ø= =°Ok q^¥¼~¡ ° Ö  ° H›  ~¡ ° .
,H

(e) `Ç~¡Q®u…Õx q^¥¼~¡°ÖO^Îih ãQ®¶„¬ô°Qê


T

1. q=¶#O †³òH›ø ^Î~¡ ZO`Ç ?


AM

qƒl
’ Oz# XHùøH›ø ãQ®¶„¬ô…Õ 5Q®°~¡° q^¥¼~¡°Ö
H

ㄬH›@# I :
M

5 q=¶<Œ ^Î~¡ 50 HË@°Á. °O\ì~¡°.

ㄬH›@# II : 7. D, E, F, G =°i†Çò H° "Íö~Þ~¡° ‡Ú_È=ô° Q® =¼ä›½ë°.


AM
IG

q=¶#O ^Î~¡ Hê~¡° ^Î~¡ä›½ 100 ï~@°Á. XH›

Hê~¡° ^Î~¡ 10 H›Æ°. J~ò# "Œi…Õ JkH› Z`Ç°ëQ® ï~O_È= =¼H÷ë Z=~¡°?
KH

2. |hß †³òH›ø Hê~¡° U ~¡OQ®°…Õ =ô#ßk ? ㄬH›@# I : D G E


J#°"Œ_È° =°i†Çò  H›<Œß =¶ã`Ç"°
Í
R

ㄬH›@# I : |hß †³ ò H› ø Hê~¡ ° ~¡ O Q® ° , K³ ã i †³ ò H› ø ‡Ú_È"#


á³ "Œ_È°. F J#°"Œ_È° J`ǼO`Ç ‡Ú_È"#
á³

q=¶#O ~¡OQ®° XH›>è. "Œ_È° Hê^ΰ.

ㄬH›@# II : ㄲ<£ž †³òH›ø q=¶#O ~¡OQ®° =¶ã`ÇO hO ㄬH›@# II : H J#°"Œ_È° F H›<Œß ‡Ú_È"³á#"Œ_È°. G

Hê^ΰ. J#°"Œ_È° E H›<Œß ‡Ú_È"³á#"Œ_È°, Hêx D

3. ‹¬=¶# |~¡°=ô° Q® 20 ‹¬OKÇ° |~¡°=ô ZO`Ç ? H›<Œß ‡Ú\ ÷“"Œ_È°.

ㄬH›@# I : XH› ‹¬Oz †³òH›ø <Œ¾= =O`Ç° |~¡°=ô 5 8. ^Íg„¬î~Ÿ =°i†Çò ^ΰ~Œ¾„î¬ ~Ÿ =°^¼Î Q® Ju `Ç佛 ø= ^ζ~¡O
H÷…Ձ°. ZO`Ç?
ㄬH›@# II : Jxß ‹¬OKÇ° ‹¬Q®@° |~¡°=ô 300 H÷…Ձ°. ㄬH›@# I : ^ΰ~Œ¾„¬î~Ÿ, ~Œ†Ÿ°„¬î~Ÿ #°O_ 20 H÷.g°.

4. ö~„¬ô Pk"Œ~¡=¶ ? ^ζ~¡O…Õ =ô#ßk.

ㄬH›@# I : D ~ËA =¶ã`ÇO ‰×x"Œ~¡O Hê^ΰ. ㄬH›@# II : ^Íg„¬î~Ÿ, ~Œ†Ÿ°„¬î~Ÿ #°O_ 15 H÷.g°.

ㄬH›@# II : x#ßQêH› "³ò#ß =¶ã`ÇO |°^Î"Œ~¡O. ^ζ~¡O…Õ =ô#ßk.

REASONING ABILITY (LOGICAL) ○ ○ ○ ○ ○ ○ ○ ○


179
○ ○ ○ ○ ○ ○ ○ ○ ○ ○ ○ ○ ○ ○ ○ ○ ○ ○ ○
Study Material
9. =°^Î<£, H›=°…˜ H›<Œß ̄^Îí"Œ_È°. ‰×~¡`Ÿ, J~¡qO^£ H›<Œß 13. HË_£ ƒì+¬…Õ 'No' †³òH›ø J~¡ÖO Uq°\÷?
z#ß"Œ_È°. J~ò# "Œ~¡O^Îi…Õ z#ß"Œ_È° Z=~¡°? ㄬH›@# I : 'Ne Pa Sic Lo' 'But No None J#Qê

ㄬH›@# I : ‰×~¡`Ÿ, =°^Î<£ H›<Œß z#ß"Œ_È°.


And' Jx J~¡O
Ö . =°i†Çò 'Pa Lo Le Ne'

ㄬH›@# II : J~¡qO^£, H›=°…˜ H›<Œß z#ß"Œ_È°.


J#Qê 'If None and But' Jx J~¡ÖO.

10. 5Q®°~¡° =¼ä›½ë° A, B, C, D =°i†Çò E ° XH› =$`ŒëHê~¡ ㄬH›@# II : 'Le Se Ne Sic' J#Qê 'If No None Will'
=†Ç°O…ì =$`Çë öHOã^¥xß KǶ‹¬°ë#ß@°Á 䛀~¡°ó<Œß~¡°.
Jx J~¡ÖO =°i†Çò 'Le Pi Se Be' J#Qê

J~ò# D 䛽 Z_È=°"³á„¬ô# Q® "³ò^Î\ ÷ =¼H÷ë Z=~¡°?


'Not None if All' Jx J~¡ÖO.

ㄬH›@# I : C A J#°"Œ_È° 䛽 Z_È=°"³„


á ô¬ # ~
ï O_È= =¼H÷.
ë

14. ‡¦é\ՅÕx =¼H÷ë`Ë ‹¬°x`Ç䛽 Q® ‹¬O|O^ÎO Uq°\÷?


B =°i†Çò D ° ㄬH›øㄬH›ø<Í ä›€~¡°ó<Œß~¡°.

ㄬH›@# I :
ㄬH›@# :
‡¦é\ՅÕx =¼H÷ë ‹¬°x`Ç †³òH›ø `Œ`džǰ¼ä›½
II D J#°"Œ_È° B 䛽 Z_È=°"³á„¬ô# "³ò^Î\ ÷

Q® UïHáH› Hù_Ȱ䛽.


=¼H÷ë. E J#°"Œ_È° D =°i†Çò B  ㄬH›ø#

Y
=¼H÷ë Hê^ΰ.
ㄬH›@# II : ‡¦é\ՅÕx =¼H÷ë J#ß^Î=òà°Qêh

M
11. 40 =°Ok q^¥¼~¡°Ö° Q® `Ç~¡Q®u…Õ xu<£ †³òH›ø ~Œ¼O䛽 JH›øK³…ÿÁˆ×ä Qêh Z=~¡¶ …è~¡°. Hêx P

̄á #°O_ ZO`Ç ? ‡¦ é \ՅÕx =¼H÷ ë †³ ò H› ø `Ç O ã_ ‹¬ ° x`Ç ä › ½

E
ㄬH›@# I : xu<£ =°i†Çò n„¬H± =°^Î¼ „¬k=°Ok `Œ`džǰ¼.

D
9
q^¥¼~¡°Ö°<Œß~¡°. 15. x‘O`Ÿ Lz`Ç L„¬Hê~¡ "Í`#
Ç O ^¥Þ~Œ q^μ#° Jƒ¼’ ‹²OKÇ°@䛽

59
ㄬH›@# II : J~¡°Ý_¨ ?
CA
95
n„¬H± ̄á #°O_ 20= ~Œ¼O䛽#° ‡ÚO^¥_È°.

12. XH› HË_£ ƒì+¬†Ç°O^ΰ '297Ñ J#Qê 'tie clip button' ㄬH›@# I : 12 ‹¬OII…Õ„¬ô =†Ç°‹¬°ž H›ey "ŒiÂH›

50
Jx J~¡ÖO. J~ò# P HË_£ ƒì+¬…Õ 'button' J~¡ÖO#° „¬sH›Æ…Õ 60% =¶~¡°ø° ‡ÚOk#"ŒiH÷

98
A
WKÇ°ó JOïH Uk?
:8
‹¬¶ø° Lz`Ç L„¬Hê~¡ "Í`#
Ç O ^¥Þ~Œ q^μ#°

ㄬH›@# I : 'clip your


P HË_£ ƒì+¬…Õ '926Ñ J#Qê
E

JOk‹¬°ëOk.
h

tie'
.P

ㄬH›@# :
Jx J~¡ÖO.
II
IC

x‘O`Ÿ "ŒiÂH› „¬sH›Æ…Õ 85% =¶~¡°ø°

ㄬH›@# II : 'hole and


AD

P HË_£ ƒì+¬…Õ '175Ñ J#Qê


ªkOKŒ_È°.

button' Jx J~¡ÖO.
AB
O

1234567890123456789012
12345678901234567890123456789012123456789012345678901234567890121234567890123456789012345678901212345678901234567890123
12345678901234567890123
12345678901234567890123456789012123456789012345678901234567890121234567890123456789012345678901212345678901234567890123
1234567890123456789012 12345678901234567890123
1234567890123456789012
1234567890123456789012
q=~¡}ì`ÇàH› ["Œ|°°
12345678901234567890123456789012123456789012345678901234567890121234567890123456789012345678901212345678901234567890123
12345678901234567890123
12345678901234567890123
12345678901234567890123456789012123456789012345678901234567890121234567890123456789012345678901212345678901234567890123
ER

PRACTICE TEST - 1 :
CH

1234567890123456789012
12345678901234567890123456789012123456789012345678901234567890121234567890123456789012345678901212345678901234567890123
1234567890123456789012 12345678901234567890123
12345678901234567890123456789012123456789012345678901234567890121234567890123456789012345678901212345678901234567890123
1234567890123456789012 12345678901234567890123
12345678901234567890123
12345678901234567890123456789012123456789012345678901234567890121234567890123456789012345678901212345678901234567890123
YD

61!IÌA±Â
,H

‰×ÃãH›"Œ~¡O ~
ö „¬ô ‰×x"Œ~¡O. J#Qê ~
ö „¬ô Pk"Œ~¡O Hê^ΰ.

(e) #-I #°O_

6
1. ㄬH@
› q=¶#O ^~
Î ¡ «
T

~
ï O_È° ㄬH@
› #…Õ ^Íx#°O_³<
á Œ ‹¬=¶^¥#O ‡ÚO^ÎQ
® O.
AM

(c) #-I #°O_ A…ÿá 17 H›<Œß =òO^ΰ 16,15,14...


H

« 10 HË@°Á 5. ㄬH@
›
M

ㄬH@
› # - II #°O_ q=¶#O ^~
Î ¡ « 100 (10 H›
Æ °) ㄬH›@# - II #°O_ A…ÿá 15 `Ç~ŒÞ`Ç 16, 17, 18 ...
AM

\
IG

« 1000 H›Æ° "³OH›>è+¹ „¬ô\ ÷“# ~ËA A…ÿá 16.


KH

« 10 HË@°Á ï~O_È° ㄬH›@# ^¥Þ~Œ ‹¬=¶^¥#O =zó#k.


R

J#Qê U ㄬ H › @ # #°O_³ á < Œ ‹¬ i †³ Ø ° # ‹¬ = ¶^ ¥ #O 6. (c) "³ò^Î\ ÷ ㄬH›@# ㄬHê~¡O `Ç~¡Q®u…Õx q^¥¼~¡°Ö ‹¬OY¼

‡ÚO^Î=KÇ°ó. 21, 22, 23, 24, 25 …è^¥ 26

2. (d) ㄬH@
› #…Õ ~¡OQ®° Q®°iOz q=iOKDž^
è °
Î . Hê=ô# ~
ï O_È° ï~O_È= ㄬH›@# ㄬHê~¡O `Ç~¡Q®u…Õx q^¥¼~¡°Ö ‹¬OY¼

ㄬH›@#¶ ‹¬i†³Ø°# ‹¬=¶^¥#O W=ޅè=ô. 25 …è^¥ 30

3. (e) ㄬH›@# - I #°O_ ‹¬Oz |~¡°=ô 20 H÷…Ձ° ï~O_È° ㄬH›@# #°O_ ‹¬=¶^¥#O 25.

20 ‹¬OKÇ° |~¡°=ô 20 þ 20 « 400 H÷…Ձ° 7. (a) ㄬH›@# - I #°O_

ㄬH›@# - II #°O_ 20 ‹¬OKÇ° |~¡°=ô H > F > D > G > E

« 300 þ 20 « 6000 H÷…Ձ°. ‹¬æ+¬“OQê H JO^ÎiH›<Œß ‡Ú_È"³á#"Œ_È°. F ‡Ú_È"³á#

J#Qê U ㄬH›@# #°O_³á<Œ ‹¬=¶^¥#O ‡ÚO^Î=KÇ°ó. ï~O_È= =¼H÷ë. ㄬH›@# - II #°O_ H > F

4. (e) ㄬH›@# - I #°O_ D ~ËA Pk"Œ~¡O Hê^Î<Ík ‹¬æ+¬“O. D > G > E

ㄬH›@# -II #°O_ "³ò#ß |°^Î"Œ~¡O J~ò`Í D ~ËA


‹¬i†³Ø°# ‹¬=¶^¥#O ‡ÚO^΅è=ò.

REASONING ABILITY (LOGICAL) ○ ○ ○ ○ ○ ○ ○ ○


180
○ ○ ○ ○ ○ ○ ○ ○ ○ ○ ○ ○ ○ ○ ○ ○ ○ ○ ○
Study Material
Hê=ô# ㄬ H › @ # - I ^¥Þ~Œ ‹¬ i †³ Ø ° # ‹¬ = ¶^ ¥ #O 12. (a) 2 9 7 J#Qê tie clip button

‡ÚO^ÎQ®O. 9 2 6 J#Qê clip your tie. - ㄬH›@# I.

8. (d) Wzó# ㄬH›@# ^¥Þ~Œ ‹¬i†³Ø°# ‹¬=¶^¥#O


‹¬æ+¬“OQê button J#° „¬^O
Î #° 7 JOïH ‹¬¶z‹¬°O
ë k.

‡ÚO^΅è=ò.
‹¬=¶^¥#O ㄬH›@#-I #°O_Í =zó#k.

9. (b) ㄬH›@# II #°O_ =°^Î<£ > H›=°…˜ > J~¡qO^£ > ‰×~¡`Ÿ
13. (a) ㄬH›@#-I #°O_

J#Qê ㄬH@
› # - II …Õx ^Î`ŒëO‰×O ^¥Þ~Œ ‹¬=¶^¥#O
Ne Pa Sic Lo J#Qê But No None And =°i†Çò

‡ÚO^Î=KÇ°ó.
Pa Lo Le Ne J#Qê If None And But.
$ %
‹¬æ+¬“OQê No J<Í „¬^¥xH÷ HË_£ Sic Jx `³°‹¬°ëOk.

& #% #
10. (c) "$ ‹¬=¶^¥#O ㄬH›@#-I #°O_Í =zó#k.

14. (e) ㄬH›@#-I #°O_ ‹¬æ+¬“OQê ‡¦é\ՅÕx =¼H÷ë ‹¬°x`Ç䛽


" #% "$ &

Y
`ÇOã_ Jx `³°‹¬°ëOk.

ㄬH›@#-I #°O_ ㄬH›@#-II #°O_

M
ㄬH@
› #-II #°O_ 䛀_¨ ‡¦é\ՅÕx =¼H÷ë ‹¬°x`Çä½
› `ÇOã_

ï~O_È° ㄬH›@# #°O_


Jx `³°‹¬°ëOk.

E
Hê=ô# U ㄬH›@# #°O_³á<Œ ㄬ‰×ß䛽 ‹¬=¶^¥#O ‡ÚO^Î
&

D
9
# =KÇ°ó#°.

59
(d)

CA ㄬH›@#-I Lz`Ç

95
15. L„¬Hê~¡ "Í`Ç#O ‡ÚO^ΰ@䛽 J~¡Ý`Ç

"
% Q®°iOz =¶ã`Ç"Í° `³e†Ç°*苬°ëOk.

50
\ÿÿ D 䛽 Z_È=°"³á„¬ô# Q® "³ò^Î\ ÷ =¼H÷ë A ㄬH›@#-II …Õ x‘O`Ÿ =†Ç°‹¬°ž Q®°iOz# ‹¬=¶KŒ~¡O

98
A
:8
ï~O_È° ㄬH›@#¶ H›e‹² ‹¬=¶^¥#O ~Œ|\ì“~ò. …è^ΰ. Hê=ô# ï~O_È° ㄬH›@#¶ ㄬ‰×ß䛽 ‹¬=¶^¥#O

(d)
E

11. Wzó# ï~O_È° ㄬH›@# #°O_ =°#O ‹¬i†³Ø°# W=ÞQ®eöQ ^Î`ŒëO‰§xß H›e¾†ÇòO_ȅè^ΰ.
h
.P

‹¬=¶^¥#O ~Œ|@“…è=ò.
IC

123456789012345678901234567890121234567890123456789012345678901212
12345678901234567890123456789012123456789012345678901234567890121234567890123456789012345678901212345678901234567890123
1234567890123456789012345678901212345678901234567890123456789012123456789012345678901234567890121234567890123
1234567890123456789012345678901212345678901234567890123456789012123456789012345678901234567890121234
AD

12345678901234567890123456789012123456789012345678901234567890121234567890123456789012345678901212345678901234567890123456789012123
1234567890123456789012345678901212345678901234567890123456789012123456789012345678901234567890
123456789012345678901234567890121234567890123456789012345678901212
12345678901234567890123456789012123456789012345678901234567890121234567890123456789012345678901212345678901234567890123
1234567890123456789012345678901212345678901234567890123456789012123456789012345678901234567890121234567890123
1234567890123456789012345678901212345678901234567890123456789012123456789012345678901234567890121234
12345678901234567890123456789012123456789012345678901234567890121234567890123456789012345678901212345678901234567890123456789012123
1234567890123456789012345678901212345678901234567890123456789012123456789012345678901234567890
12345678901234567890123456789012123456789012345678901234567890121234567890123456789012345678901212345678901234567890123
1234567890123456789012345678901212345678901234567890123456789012123456789012345678901234567890121234567890123
1234567890123456789012345678901212345678901234567890123456789012123456789012345678901234567890121234
12345678901234567890123456789012123456789012345678901234567890121234567890123456789012345678901212345678901234567890123456789012123
1234567890123456789012345678901212345678901234567890123456789012123456789012345678901234567890
123456789012345678901234567890121234567890123456789012345678901212
PRACTICE TEST - 2
12345678901234567890123456789012123456789012345678901234567890121234567890123456789012345678901212345678901234567890123456789012123
1234567890123456789012345678901212345678901234567890123456789012123456789012345678901234567890
12345678901234567890123456789012123456789012345678901234567890121234567890123456789012345678901212345678901234567890123
1234567890123456789012345678901212345678901234567890123456789012123456789012345678901234567890121234567890123
1234567890123456789012345678901212345678901234567890123456789012123456789012345678901234567890121234
123456789012345678901234567890121234567890123456789012345678901212
12345678901234567890123456789012123456789012345678901234567890121234567890123456789012345678901212345678901234567890123456789012123
1234567890123456789012345678901212345678901234567890123456789012123456789012345678901234567890
12345678901234567890123456789012123456789012345678901234567890121234567890123456789012345678901212345678901234567890123
1234567890123456789012345678901212345678901234567890123456789012123456789012345678901234567890121234567890123
1234567890123456789012345678901212345678901234567890123456789012123456789012345678901234567890121234
AB
O

12345678901234567890123456789012123456789012345678901234567890121234567890123456789012345678901212345678901234567890123456789012123
1234567890123456789012345678901212345678901234567890123456789012123456789012345678901234567890
123456789012345678901234567890121234567890123456789012345678901212
12345678901234567890123456789012123456789012345678901234567890121234567890123456789012345678901212345678901234567890123
1234567890123456789012345678901212345678901234567890123456789012123456789012345678901234567890121234567890123
1234567890123456789012345678901212345678901234567890123456789012123456789012345678901234567890121234
12345678901234567890123456789012123456789012345678901234567890121234567890123456789012345678901212345678901234567890123456789012123
1234567890123456789012345678901212345678901234567890123456789012123456789012345678901234567890
ER
CH

1. 1996 ‹¬OII…Õ U ~ËA# "³¶‚¬ì<£ ‡Á\˜#° Hù#°Qˁ° 4. A =°i†Çò B  =°^Î¼ ^ζ~¡O ZO`Ç ?
YD

Kͪ_È°? ㄬH›@# I : A B =°i†Çò  =°^¼


Πㄬ†¶
Ç }O ~
ï 
á ° ^¥Þ~Œ
,H

ㄬH›@# I : 18 _Ì‹O|~Ÿ 1996䛽 `Ç~¡°"Œ`Ç Hêx 15 Kͪë~¡°.


T
AM

_Ì‹O|~Ÿ 1996䛽 =òO^ΰ =¶ã`ÇO Hê^ΰ. ㄬH›@# II : ~


ï …
á Þ
è >ÿO
Ø >è|°…˜ ㄬHê~¡O A #°O_ B䛽
H

ㄬH›@# :
M

II 16 _Ì‹O|~Ÿ 1996䛽 =òO^ΰ Hêx 19 "³ ˆ × Ã ¤ ï~ၰ ㄬ u ~ËE L^Î † Ç ° O 8:10
AM
IG

_Ì‹O|~Ÿ 1996䛽 `Ç~°


¡ "Œ`Ç =¶ã`ÇO Hê^ΰ. a.m.䛽 A #°O_ |†Ç°°^Íi B䛽 L^Ά°
Ç O

2. P ‚¬ìº‹²OQ· HêŌÁH±ž…Õ "³ò`ÇëO Zxß ‡Á@°Á H›=ô ?


KH

10:50 a.m.䛽 KÍ~¡°`Ç°Ok.

5. D HË~¡°ž †³òH›ø H›zó`Ç Hꁄ¬iq°u ZO`Ç ?


R

ㄬH›@# I : ㄬu qOQ·…Õ 16 ‡Á@°Á =ô<Œß~ò. Wk

"³ò`ÇëO aeÛOQ· ‹¬OY¼ä›½ ‹¬=¶#O. ㄬH›@# I : D HË~¡°ž…Õ =ü_È° ̋q°‹¬“~¡°Á LO\ì~ò.

ㄬH›@# II : ㄬu aeÛOQ·…Õ 4 qOQ·ž =ô<Œß~ò. Jq


~
ï O_È= =°i†Çò =ü_È= ̋q°‹¬~
“ °
¡ Á =°^¼
Î …Õ

tH›Æ}…Õ =°e^Î‰× =ôO@°Ok.


A, B, C =°i†Çò D.

3. D #Q®~¡O…Õ ZO`Ç=°Ok _¨H›“~¡°Á ㇐H©“‹¬° K͋¬°ë<Œß~¡° ? ㄬH›@# II : tH›Æ}…Õ =°e^Î‰× †³òH›ø Hꁄ¬iq°u

ㄬH›@# I : ㄬu 700 =°Ok ㄬ[ä›½ XH› _¨H›“~¡°


ã‡Ú̄¦‹¬~Ÿ x~¡â~òªë~¡°.

=ô<Œß~¡°.
6. M 䛽 P`Ë Q® ‹¬O|O^ÎO Uq°\÷?
ㄬH›@# II : D #Q®~¡O…Õ "³ò`ÇëO 16 "Œ~¡°Û°<Œß~ò.
ㄬH›@# I : J 䛽 W^Îí~¡° 䛽=¶~¡°°. "Œi…Õ XH›~¡° P.

XHùøH›ø "Œ~¡°Û…Õ 700 =°Ok ㄬ[°


ㄬH›@# II : M P †³òH›ø ªé^Îi, †³òH›ø `ÇeÁ †³òH›ø

ªé^Îi.
x=t‹¬°ë<Œß~¡°.

REASONING ABILITY (LOGICAL) ○ ○ ○ ○ ○ ○ ○ ○


181
○ ○ ○ ○ ○ ○ ○ ○ ○ ○ ○ ○ ○ ○ ○ ○ ○ ○ ○
Study Material
7. D ZO`Ç =°Ok 䛽=¶~¡°#° H›ey†Çò<Œß_È° ? ㄬH›@# I : Q® ` Ç ‹¬ O =`Ç ž ~¡ O 2935 Hê~¡ ° Û  ° J=òà

ㄬH›@# I : A †³òH›ø `ÇOã_H÷ =òQ®°¾~¡° „²Á°. _ȆǶ¼~ò.

ㄬH›@# II : B A J#°"Œ_È° 䛽 ªé^Î~¡°_È° =°i†Çò D ㄬH›@# II : D ‹¬O=`Ǟ~¡O J=òà_È~ò# ãw\ O


÷ Q· Hê~¡°
Û

†³òH›ø 䛽=¶~¡°_È°. ‹¬OY¼ Q®`Ç ‹¬O=`Ǟ~¡O`Ë ‡éeó`Í 1.2 ~


ï @°Á.

8. R, T, M, K =°i†Çò P…Õ Z=~¡° JO^ÎiH›<Œß ‡Ú_È"³á# 10. buy-back ‹ÔøO ㄬHê~¡O KÇOã^Î=°¿o `Ç# ãHù`Çë Hê~¡°
"Œ~¡°? Hˋ¬O ZO`Ç K³eÁOKŒe ?
ㄬH›@# I : T =°i†Çò K ° P H›<Œß ‡Ú\ ÷“"Œ~¡°. ㄬH›@# I : ãHù`Çë Hê~¡° Ys^ΰ ‡`Ç Hê~¡° ^Î~¡ä›½ =ü_È°

ㄬH›@# II : M, P H›<Œß ‡Ú_È"³á#"Œ_È° Hêx R H›<Œß ï~@°Á.

‡Ú_È=ô `Ç䛽ø=. ㄬH›@# II : buy-back ‹ÔøO ãH÷O^Î KÇOã^Î=°¿o

9. h ‘„¬ô…Õ D ‹¬O=`Ǟ~¡O Zxß #¶¼ W†Ç°~Ÿ ãw\÷OQ· ‡`ÇHê~¡° Ys^ΰ ~¡¶.25,000Qê q°=

Hê~¡°Û° J=òà_ȆǶ¼~ò ? H›@“|_Ok.

Y
1234567890123456789012
12345678901234567890123456789012123456789012345678901234567890121234567890123456789012345678901212345678901234567890123
12345678901234567890123

q=~¡}ì`ÇàH› ["Œ|°°
12345678901234567890123456789012123456789012345678901234567890121234567890123456789012345678901212345678901234567890123
1234567890123456789012 12345678901234567890123
12345678901234567890123456789012123456789012345678901234567890121234567890123456789012345678901212345678901234567890123
1234567890123456789012 12345678901234567890123

M
PRACTICE TEST - 2 :
12345678901234567890123456789012123456789012345678901234567890121234567890123456789012345678901212345678901234567890123
1234567890123456789012 12345678901234567890123
12345678901234567890123456789012123456789012345678901234567890121234567890123456789012345678901212345678901234567890123
1234567890123456789012 12345678901234567890123
12345678901234567890123456789012123456789012345678901234567890121234567890123456789012345678901212345678901234567890123
1234567890123456789012
12345678901234567890123456789012123456789012345678901234567890121234567890123456789012345678901212345678901234567890123
1234567890123456789012
12345678901234567890123
12345678901234567890123

E
1. (c) ï~O_È° ㄬH›@#…Õx ^Î`ŒëO‰×O ㄬHê~¡O "³¶‚¬ì<£ P 7. (d) A †³òH›ø `ÇOã_H÷ =òQ®°¾~¡° „²Á°. "Œi…Õ H›zó`ÇOQê

D
9
59
‡Á\˜#° 17 _Ì‹O|~Ÿ 1996# Hù<Œß_È°. Hê=ô# ï~O_È° A XH›_È°. B, A †³òH›ø ªé^Î~¡°_È° =°i†Çò D †³òH›ø

CA
95
ㄬH›@# ^Î`ŒëO‰×O J=‹¬~¡"Í°.
䛽=¶~¡°_È°. Hê=ô# D =òQ®°¾~¡° „²Áä›€ `ÇeÁ. B =°Q®

50
2. (c) ï~O_È° ㄬH›@# #°O_ ㄬu ƒ’=Ou 4 qOQ·#°
„²Á"Œ_È°, Hêx q°ye# W^Îí~¡° „²Á° P_Ë? =°QË?

98
A
W=Þ|_ȅè^ΰ. Hê=ô# ï~O_È° ㄬH›@#…Õx ^Î`ŒëO‰×O
H›ey†ÇòOk. ㄬu qOQ·…Õ 16 ‡Á@°Á =ô<Œß~ò. "³ò`ÇO
ë
:8
‹¬=¶^¥#O ~Œ|@°“@䛽 ‹¬i‡é^ΰ.
E

16 ƒ’=O`Ç°°.
h

(c)
.P

8. ï~O_È° ㄬH›@#…Õx ^Î`ŒëO‰×O #°O_ ̄iöQ ‡Ú_È=ô


IC

Þ
Hê=ô# "³ò`ÇO
ë ‡Á\˜ ‹¬OY¼ « 4 þ 16 þ 16 «
AD

ãH›=°O ㄬHê~¡O T/K < P < M < R JO^ÎiH›<Œß


1024. Hê=ô# ï~O_È° ㄬH›@#…Õx ^Î`ŒëO‰×O
AB
O

'R' ‡Ú_È"³á# "Œ_È°.


L„¬†³¶yÀ‹ë<Í ã„¬‰×ß䛽 ‹¬=¶^¥#O ~Œ|@“Q®O.
ER

ï~O_È° ㄬH›@#…Õx ^Î`ŒëO‰×O L„¬†³¶yÀ‹ë<Í ã„¬‰×ß䛽


CH

3. (c) ㄬH@
› #-I #°O_, ㄬu 700 =°OkH÷ XH› _¨H›~
“ °
¡ ㇐H©‹
“ ¹
YD

‹¬=¶^¥#O =‹¬°ëOk.

K͋¬°ë<Œß~¡°. ㄬH›@#-II #°O_, XHùøH›ø "Œ~¡°Û…Õ 700

9. (c) ㄬH@
› #-I =°i†Çò ㄬH@
› #-II ~
ï O_O\ …
÷ Õx ^Î`ŒëO‰×O
,H

=°Ok ㄬ[° Q® "³ò`ÇëO 16 "Œ~¡°Û°<Œß~ò.


T

L„¬†³¶yÀ‹ë<Í ã„¬‰×ß䛽 ‹¬=¶^¥#O ~Œ|@“Q®O.


AM

#Q®~¡O…Õ ㇐H©“‹¹ K͋¬°ë#ß _¨H›“~¡Á ‹¬OY¼ « 16.


H

D ‹¬O=`Ǟ~¡O J=òà_È~ò# Hê~¡°


Û ° « (1.2) (2935)
M

ï~O_È° ㄬH›@#…Õx ^Î`ŒëO‰×O L„¬†³¶yÀ‹ë<Í ã„¬‰×ß䛽


AM

« 3522
IG

‹¬=¶^¥#O =‹¬°ëOk.
(c) #-I =°i†Çò ㄬH@ #-II ~
KH

10. ㄬH@
› › ï O_O\ …
÷ Õx ^Î`ŒëO‰×O

4. (d) ~
ï O_È° ㄬH@
› #…Õ ^ζ~¡O Q®°iOz# ‹¬=¶KŒ~¡O …è^°
Î .
R

L„¬†³¶yÀ‹ë<Í ã„¬‰×ß䛽 ‹¬=¶^¥#O =‹¬°ëOk.

Hê=ô# ㄬ‰×ß䛽 ‹¬=¶^¥#O ~Œ|@°“@䛽 ï~O_È°


ㄬ H › @ # - II #°O_ KÇ O ã^Î = °¿o ‡`Ç H ê~¡ ° Ys^Î °

ㄬH@
› #…Õx ^Î`ŒëO‰×O ‹¬i‡é^ΰ.
`. 25,000
5. (d) Hꁄ¬iq°u Q®°iOz ‹¬æ+¬“`Ç …è^ΰ. Hê=ô# ‹¬=¶^¥#O
\ ㄬH›@#-I #°O_ KÇOã^Î=°¿o Hù`ÇëHê~¡° Ys^ΰ

~Œ|@“…è=ò. J#Qê ï~O_È° ㄬH›@#…Õx ^Î`ŒëO‰×O = 3 × 25,000

‹¬=¶^¥#O ~Œ|@°“@䛽 ‹¬i‡é^ΰ. = `. 75,000


6. (d) ‹¬æ+¬“OQê ï~O_È° ㄬH›@#…Õ M P_Ë? =°QË? Hê=ô# KÇOã^Î=°¿o K³eÁOKÇ=‹²# "³ò`ÇëO

W=Þ|_ȅè^ΰ. Hê=ô# ‹¬=¶^¥#O ~Œ|@°“@䛽 ï~O_È° = `. 75,000 – `. 25,000


ㄬH›@#…Õx ^Î`ŒëO‰×O ‹¬i‡é^ΰ. = `. 50,000

REASONING ABILITY (LOGICAL) ○ ○ ○ ○ ○ ○ ○ ○


182
○ ○ ○ ○ ○ ○ ○ ○ ○ ○ ○ ○ ○ ○ ○ ○ ○ ○ ○
Study Material
123456789012345678901234567890121234567890123456789012345678901212
12345678901234567890123456789012123456789012345678901234567890121234567890123456789012345678901212345678901234567890123
1234567890123456789012345678901212345678901234567890123456789012123456789012345678901234567890121234567890123
1234567890123456789012345678901212345678901234567890123456789012123456789012345678901234567890121234
12345678901234567890123456789012123456789012345678901234567890121234567890123456789012345678901212345678901234567890123456789012123
123456789012345678901234567890121234567890123456789012345678901212345678901234567890123
12345678901234567890123456789012123456789012345678901234567890121234567890123456789012345678901212345678901234567890123
1234567890123456789012345678901212345678901234567890123456789012123456789012345678901234567890121234567890123
1234567890123456789012345678901212345678901234567890123456789012123456789012345678901234567890121234
123456789012345678901234567890121234567890123456789012345678901212
12345678901234567890123456789012123456789012345678901234567890121234567890123456789012345678901212345678901234567890123456789012123
123456789012345678901234567890121234567890123456789012345678901212345678901234567890123
12345678901234567890123456789012123456789012345678901234567890121234567890123456789012345678901212345678901234567890123
1234567890123456789012345678901212345678901234567890123456789012123456789012345678901234567890121234567890123
1234567890123456789012345678901212345678901234567890123456789012123456789012345678901234567890121234
12345678901234567890123456789012123456789012345678901234567890121234567890123456789012345678901212345678901234567890123456789012123
123456789012345678901234567890121234567890123456789012345678901212345678901234567890123
123456789012345678901234567890121234567890123456789012345678901212
PREVIOUS BITS
12345678901234567890123456789012123456789012345678901234567890121234567890123456789012345678901212345678901234567890123
1234567890123456789012345678901212345678901234567890123456789012123456789012345678901234567890121234567890123
1234567890123456789012345678901212345678901234567890123456789012123456789012345678901234567890121234
12345678901234567890123456789012123456789012345678901234567890121234567890123456789012345678901212345678901234567890123456789012123
123456789012345678901234567890121234567890123456789012345678901212345678901234567890123
123456789012345678901234567890121234567890123456789012345678901212
12345678901234567890123456789012123456789012345678901234567890121234567890123456789012345678901212345678901234567890123
1234567890123456789012345678901212345678901234567890123456789012123456789012345678901234567890121234567890123
1234567890123456789012345678901212345678901234567890123456789012123456789012345678901234567890121234
12345678901234567890123456789012123456789012345678901234567890121234567890123456789012345678901212345678901234567890123456789012123
123456789012345678901234567890121234567890123456789012345678901212345678901234567890123
12345678901234567890123456789012123456789012345678901234567890121234567890123456789012345678901212345678901234567890123
1234567890123456789012345678901212345678901234567890123456789012123456789012345678901234567890121234567890123
1234567890123456789012345678901212345678901234567890123456789012123456789012345678901234567890121234
12345678901234567890123456789012123456789012345678901234567890121234567890123456789012345678901212345678901234567890123456789012123
123456789012345678901234567890121234567890123456789012345678901212345678901234567890123
123456789012345678901234567890121234567890123456789012345678901212
12345678901234567890123456789012123456789012345678901234567890121234567890123456789012345678901212345678901234567890123456789012123
123456789012345678901234567890121234567890123456789012345678901212345678901234567890123
12345678901234567890123456789012123456789012345678901234567890121234567890123456789012345678901212345678901234567890123
1234567890123456789012345678901212345678901234567890123456789012123456789012345678901234567890121234567890123
1234567890123456789012345678901212345678901234567890123456789012123456789012345678901234567890121234

G 1 #°Oz 4 =~¡ä›½ Q® ㄬu ㄬ‰×߅Õ#¶ XH› ㄬ‰×ß, ^¥x 2. =üië [#àk#=ò "Œ~¡O…Õx U ~ËA# =‹¬°ëOk?
`Ç~ŒÞ`Ç ï~O_È° ㄬ=KÇ<Œ° ° WKŒó~¡°. I, II ㄬH›@# : I =üië [#àk#=ò |°^Î"Œ~¡O `Ç~¡°"Œ`Ç 

ㄬH›@# `Ë =¶ã`Ç"Í° ㄬ‰×ß䛽 ["Œa=ÞQ®ey`Í (1)


I Hêx =°OQ® ˆ × " Œ~¡ O `Ç ~ ¡ ° "Œ`Ç =‹¬ ° ë O ^Î x

Jh  ㄬª^£ä›½ Yzó`ÇOQê Q®°~¡°ëOk.

ㄬH›@# `Ë =¶ã`Ç"Í° ㄬ‰×ß䛽 ["Œa=ÞQ®ey`Í (2)


II ㄬH›@# : II =üië [#àk#=ò ‰×ÃãH›"Œ~ŒxH÷ =òO^ΰ 

Jh  Hêx =°OQ® ˆ × " Œ~¡ O `Ç ~ ¡ ° "Œ`Ç =‹¬ ° ë O ^Î x

ㄬ H › @ # #° H› e „² ` Í =¶ã`Ç " Í ° ㄬ ‰ × ß ä› ½


I, II W=¶"£°ä›½ Yzó`ÇOQê Q®°~¡°ëOk.

["Œa=ÞQ®ey`Í (3) Jh  3. H÷;`Ë K³á#`Ǽ䛽 Q® ‹¬O|O^Î"Í°k?


ã„H¬ @› # ° ~ï O_O\÷h f‹¬°Hù<Œß WOHùO`Ç J^Î#„¬ô
I, II ㄬH›@# : I K³á`Ç#¼ `ÇeÁ, H÷; `ÇOã_ †³òH›ø ªé^Îi.

Y
‹¬=¶KŒ~¡O …è䛽O_¨ ㄬ‰×ß䛽 ["Œa=ޅèH›‡é`Í (4) Jh ㄬH›@# : II K³á`Ç#¼ `Œ`ÇQêi UïHáH› 䛽=¶~¡°x †³òH›ø

g° SzóHêxß Q®°iëOKÇO_.

M
(AP.S.I. Mains-2016) 䛽=¶ï~ë H÷;.

4. qa#ß Z`ǰ끰 H›ey# S^ΰ =°Ok À‹ß‚²ì`Ç°° A, B, C,


1. \º<£ T …Õ ‡éb‹¹ ‡¦é~Ÿž…Õx „¬ô~¡°+¬µ =°i†Çò ¢‹Ôë

E
D, E …Õ, Q®i+¬» ‡Ú_È=ô…Õ ï~O_È="Œ_È° Z=~¡°?
JkHê~¡° x+¬æuë ZO`Ç?

D
9
ㄬH›@# I : D, B

59
E D B
ㄬH›@# I :
 H›O>ÿ ‡Ú_È"³á#"Œ_È°. H›O>ÿ
¢‹Ôë JkHê~¡° ‹¬OY¼ „¬ô~¡°+¬ JkHê~¡°

CA
95
‡Ú\ ÷“"Œ_È°.
‹¬OY¼…Õ ‹¬QêxH›O>è 250 `Ç䛽ø=.

ㄬH›@# II : E C A
ㄬH›@# :

50
H›O>ÿ ‡Ú_È"³á#"Œ_È°. Q®i+¬» ‡Ú_ÈQ®i
II ¢‹Ôë JkHê~¡° ‹¬OY¼, „¬ô~¡°+¬ JkHê~¡°


98
A
Hê^ΰ.

‹¬OY¼…Õ = ƒìQ®O.

:8
12345678901234567890123456789012123456789012345678901234567890121234567890123456789012345678901212345678901234567890123
1234567890123456789012 123456789012345678901234
E

12345678901234567890123456789012123456789012345678901234567890121234567890123456789012345678901212345678901234567890123
1234567890123456789012 123456789012345678901234
1234567890123456789012
q=~¡}ì`ÇàH› ["Œ|°°
123456789012345678901234
12345678901234567890123456789012123456789012345678901234567890121234567890123456789012345678901212345678901234567890123
h

1234567890123456789012 123456789012345678901234
12345678901234567890123456789012123456789012345678901234567890121234567890123456789012345678901212345678901234567890123
1234567890123456789012 PREVIOUS BITS :
12345678901234567890123456789012123456789012345678901234567890121234567890123456789012345678901212345678901234567890123
.P

123456789012345678901234
1234567890123456789012 123456789012345678901234
12345678901234567890123456789012123456789012345678901234567890121234567890123456789012345678901212345678901234567890123
IC

1234567890123456789012 123456789012345678901234
12345678901234567890123456789012123456789012345678901234567890121234567890123456789012345678901212345678901234567890123
AD

„¬ô~¡°+¬µ ‹¬OY¼ « x   ¢‹Ôë ‹¬OY¼ « y J#°Hùx# ㄬH›@#- II #°O_


AB
O

1. (2) ㄬH›@# – I #°O_ =üië [#àk#O =KÇ°ó "Œ~Œ° |°^Î, Q®°~¡°.


ER
CH

Y I, II
 
Hê=ô# ㄬ H › @ #  #°O_ =üië [#àk#O
YD

¢‹Ôë ‹¬OY¼ «
 Q®°~¡°"Œ~¡O J=ô`Ç°Ok.

Y
,H

Z    Hê=ô# ‹¬=¶^¥#=ò ~Œ|@°“@䛽 I, II ㄬ=KÇ<Œ°


T

Þ
AM

J=‹¬~¡O.
x = 2y + 500
H

(1) – I
M

3. ㄬH›@# #°O_
Hê=ô# Wk ‹¬=¶^¥#O ~Œ|@°“@䛽 ‹¬i‡é^ΰ
AM
IG

K³`
á #
Ç ¼ `ÇeÁ - H÷; `ÇOã_ ªé^Îi. J#Qê H÷;, K³`
á #
Ç ¼ä›½
ㄬH›@#- II #°O_
KH

 J`Çë 䛀`Ç°~¡° J=ô`Ç°Ok.

¢‹Ôë ‹¬OY¼ « „¬ô~¡°+¬µ ‹¬OY¼ þ


 II
R

ㄬH›@#- #°O_

 K³`
á #
Ç ¼ `Œ`ÇQêi UïHH
á › 䛽=¶~¡°x 䛽=¶ï~ë J#Qê XH›"ˆ
Í ×

ZYs
 K³eÁ J=KÇ°ó. „¬îië ‹¬=¶KŒ~¡O J=…è^ΰ.

Y 
 Hê=ô# „¬îië ‹¬=¶^¥#O ~Œ|@°“@䛽 ㄬH@
› # I =¶ã`Ç"°
Í

Z 
=   
J=‹¬~¡O.

YZ 4. (4) ㄬH›@# – I #°O_

Hê=ô# ㄬH›@# - II #°O_ =¶ã`Ç"Í° „¬îië ‹¬=¶^¥#O E > D, B & D > B

~Œ|@“Q®=ò. Þ E > D > B

Hê=ô# nxH÷ ‹¬i†³Ø°# ‹¬=¶^¥#O (2). Hê=ô# Wk ‹¬=¶^¥#O ~Œ|@°“@䛽 ‹¬i‡é^ΰ

2. (3) ㄬH›@# – I #°O_ ㄬH›@#- II #°O_

=üië [#àk#O =KÇ°ó "Œ~Œ° Q®°~¡°, ‰×ÃãH›, ‰×x Hê=ô# nxH÷ ‹¬i†³Ø°# ‹¬=¶^¥#O (2).

REASONING ABILITY (LOGICAL) ○ ○ ○ ○ ○ ○ ○ ○


183
○ ○ ○ ○ ○ ○ ○ ○ ○ ○ ○ ○ ○ ○ ○ ○ ○ ○ ○
Study Material
[_û"³°O\˜
6 (Situation Reaction Test)

=òMì¼O‰§°...
G D KŒ„¬“~Ÿ…Õ Jƒ’¼iÖ „¬i‹²Ö`Ç°ä›½ J#°Q®°}OQê Z…ì ‹¬æOkªë_È° J<Ík „¬sH÷ÆOKÇ_ÈO [~¡°Q®°`Ç°Ok. J…ìöQ Jƒ’¼iÖ J#°Hùx
„¬i‹²Ö`Ç°° Z^ΰï~á#„¬ô_È° x~¡â†Ç¶° Z…ì f‹¬°ä›½O\ì_È° J<Ík „¬sH÷ƪë~¡°. nx<Í JO\ì~¡°. ''[_û"³°O\˜ÑÑ

G nx…Õ ㄬ‰×߁䛽 ‹¬=¶^¥#O WKÍó@„¬ô_È° ª#°ä›€ ^Î$H›æ^ŠÎO Q® J„¬Â<£° ‹¬=¶^¥<Œ°Qê Q®°iëOKŒe.

Y
123456789012345678901234567890121234567890123456789012345678901212
12345678901234567890123456789012123456789012345678901234567890121234567890123456789012345678901212345678901234567890123
1234567890123456789012345678901212345678901234567890123456789012123456789012345678901234567890121234567890123
1234567890123456789012345678901212345678901234567890123456789012123456789012345678901234567890121234
123456789012345678901234567890121234567890123456789012345678901212
12345678901234567890123456789012123456789012345678901234567890121234567890123456789012345678901212345678901234567890123456789012123
1234567890123456789012345678901212345678901234567890123456789012123456789012345678901234567890
12345678901234567890123456789012123456789012345678901234567890121234567890123456789012345678901212345678901234567890123
1234567890123456789012345678901212345678901234567890123456789012123456789012345678901234567890121234567890123
1234567890123456789012345678901212345678901234567890123456789012123456789012345678901234567890121234
12345678901234567890123456789012123456789012345678901234567890121234567890123456789012345678901212345678901234567890123456789012123
1234567890123456789012345678901212345678901234567890123456789012123456789012345678901234567890
123456789012345678901234567890121234567890123456789012345678901212
12345678901234567890123456789012123456789012345678901234567890121234567890123456789012345678901212345678901234567890123
1234567890123456789012345678901212345678901234567890123456789012123456789012345678901234567890121234567890123
1234567890123456789012345678901212345678901234567890123456789012123456789012345678901234567890121234
12345678901234567890123456789012123456789012345678901234567890121234567890123456789012345678901212345678901234567890123456789012123
1234567890123456789012345678901212345678901234567890123456789012123456789012345678901234567890

M
PRACTICE TEST
123456789012345678901234567890121234567890123456789012345678901212
12345678901234567890123456789012123456789012345678901234567890121234567890123456789012345678901212345678901234567890123456789012123
1234567890123456789012345678901212345678901234567890123456789012123456789012345678901234567890
12345678901234567890123456789012123456789012345678901234567890121234567890123456789012345678901212345678901234567890123
1234567890123456789012345678901212345678901234567890123456789012123456789012345678901234567890121234567890123
1234567890123456789012345678901212345678901234567890123456789012123456789012345678901234567890121234
12345678901234567890123456789012123456789012345678901234567890121234567890123456789012345678901212345678901234567890123456789012123
1234567890123456789012345678901212345678901234567890123456789012123456789012345678901234567890
12345678901234567890123456789012123456789012345678901234567890121234567890123456789012345678901212345678901234567890123
1234567890123456789012345678901212345678901234567890123456789012123456789012345678901234567890121234567890123
1234567890123456789012345678901212345678901234567890123456789012123456789012345678901234567890121234
123456789012345678901234567890121234567890123456789012345678901212
12345678901234567890123456789012123456789012345678901234567890121234567890123456789012345678901212345678901234567890123456789012123
1234567890123456789012345678901212345678901234567890123456789012123456789012345678901234567890
12345678901234567890123456789012123456789012345678901234567890121234567890123456789012345678901212345678901234567890123
1234567890123456789012345678901212345678901234567890123456789012123456789012345678901234567890121234567890123
1234567890123456789012345678901212345678901234567890123456789012123456789012345678901234567890121234
12345678901234567890123456789012123456789012345678901234567890121234567890123456789012345678901212345678901234567890123456789012123
1234567890123456789012345678901212345678901234567890123456789012123456789012345678901234567890
123456789012345678901234567890121234567890123456789012345678901212
12345678901234567890123456789012123456789012345678901234567890121234567890123456789012345678901212345678901234567890123
1234567890123456789012345678901212345678901234567890123456789012123456789012345678901234567890121234567890123
1234567890123456789012345678901212345678901234567890123456789012123456789012345678901234567890121234
12345678901234567890123456789012123456789012345678901234567890121234567890123456789012345678901212345678901234567890123456789012123
1234567890123456789012345678901212345678901234567890123456789012123456789012345678901234567890

E
1. h ‹¬‚¬ìÙ^˼y XH› ‡s“…Õ J#°HË䛽O_¨ h̄á# 䛀…˜ã_OH± 6. g° H›Ōh Hù`Çë "Í°<Í[~Ÿ *ì~ò<£ J†Ç¶¼_È° J„¬C_È° h=ô

D
9
‡éªë_È° J„¬C_È° h=ô 1) h=ô =¶~¡`Œ=ô

59
1) Jux ã_³‹¹Ì„á 䛀_¨ #°=ôÞ ‡éªë=ô (Revenge) 2) J`Çxx ª^Î~¡OQê P‚¬ðÞxªë=ô
CA
95
2) „¬~¡"Œ…è^ΰ Jx K³‡ë=ô 3) J`Çxx ‡Úy_, ƒq ’ +¬¼`Ÿ…Õ J#u ‹¬‚𬠆Ƕxß HË~¡`Œ=ô

50
3) Hù_È`Œ=ô 4) UnHê^ΰ 4) UnHê^ΰ
98
A
2. h †³òH›ø H›ˆì‰§…Õ ~¡H^ ë› ¥# ta~¡O [~¡°Q®°`Ç°Ok J„¬C_È° 7. h L^˼Q®O…Õ XH› „¬x x#°ß W|ÄOk ̄_È°`Ç°Ok J„¬C_È°
:8
h=ô ? h=ô Uq°Kͪë=ô ?
E
h

1) JO^Îih J_È°Q®°`Œ=ô (UO K³†Ç¶…Õ `ËKÇH›)


.P

1) ~¡H›ë^¥#O Kͪë=ô
IC

2) ‰§O`ÇOQê 䛀~ùóx, ‹¬=°‹¬¼ #°O_ |†Ç°@ „¬_È_¨xH÷


AD

2) ~¡H›ë^¥#O K͆ǰ_ÈO h䛽 W+¬“O LO_È^ΰ


ㄬ†Ç°uߪë=ô
3) h À‹ß‚²ì`Ç°#° ~¡H›ëO W=Þ_¨xH÷ X„²æªë=ô
AB
O

3) P „¬xx =k…誐ë=ô 4) UnHê^ΰ


4) UnHê^ΰ
ER
CH

8. h ‹¬‚¬ìÙ^˼y `Ç# „¬x…Õ Hùxß ‹¬=°‹¬¼°#߄¬ô_È°


3. Hù`Çë ㄬ^͉×O…Õ h䛽 ‘„²OQ· K͆Ƕ#°ä›½<Œß=ô J„¬C_È°
1) J`Çx ƒì^΁ Q®¶ió J‹¬° „¬\ ÷“OKÇ°HË=ô
YD

h=ô
2) J`ÇxH÷ ‹¬‚¬ð†Ç°„¬_È`Œ=ô
,H

1) ‘„²OQ· K͆Ƕ#°ä›½<Í P…ÕKÇ# q~¡q°OKǰ䛽O\ì=ô


3) Hùxß ‹¬¶KÇ#° K³|°`Œ=ô
T
AM

2) P ㄬ^͉§xß Q®°iëOKÇQ®=ô
4) UnHê^ΰ
H

3) W`Ç~¡°#° J_y, ‘„²OQ· ‹¬Ö…ìxH÷ "³ˆìë=ô


M

9. h=ô ~ï 
á °…Õ ㄬ†¶ Ç }÷‹°¬ #
ë ߄¬C_È° h ‹Ô@° ãH÷O^Î ‹¬¶\˜Hö ‹¬°#°
AM

4) h Hˋ¬O W`Ç~¡°#° ‘„²OQ· K͆ǰ=°O\ì=ô


IG

KǶzuq. J„¬C_È° h=ô


4. P^Î~¡Å=O`Ç"³°Ø# "Í°<Í[~Ÿä›½ LO_¨ež# H›Æ}쁰
KH

1) ‡éb‹¬°ä›½ `³e†Ç°*誐ë=ô
1) ~¡¶…˜ž ‡\÷OKŒe
R

2) ‹¬¶@°öH‹¬° `³~¡z JO^ΰ…Õ U=ò<Œß~ò Jx KǶªë=ô


2) #°=ôÞ h…ì =ôO\ì=ô 3) ‹¬¶\˜Hö ‹¬°Ì„á#Q® Jã_ȋ°¬ P^¥~¡OQê J`ÇxH÷ JO^Î*ª è ë=ô
3) W`Ç~°¡ ° Ja#OkOKÍ q^" Î °³ #
Ø x~¡†
â ¶
Ç ° f‹¬°ä›½O\ì~¡° 4) UnHê^ΰ
4) ̋q°<Œ~Ÿž Z䛽ø= W"ŒÞe. 10. h䛽 [Þ~¡O =zó#„¬C_È° h=ô W…ì J#°ä›½O\ì=ô
5. h=ô H›ˆì‰§ä›½ "³ˆ×Ãë#„¬C_È° P‹¬¼O J~òOk U |‹¬°ž 1) „¬x H›<Œß P~ËQ®¼O =òY¼O
~Œ=_ÈO …è^ΰ J„¬C_È° h=ô 2) P„¦Ô‹¬°ä›½ "³ˆì¤ež# „¬x…è^ΰ
1) #_z "³ˆìë=ô 3) q^Î° Q®°iOz P…ÕzOKÇ=ô
2) H›ˆì‰§ä›½ P~ËA "³ˆ×Á=ô 4) `ùO^Î~¡ Q® „¬#°° „¬îië K͋² =ªë=ô
3) "Í~ö ^¥i P…Õzªë=ô (other possible convey- 11. D P^°Î xH› ㄬ„O ¬ KÇO…Õ x*ì~òfH÷ q°= …è^°Î Wk x[O
ance) 1) ZÁ„¬C_ȶ 2) Z䛽ø q+¬†Ç¶…Õ
4) UnHê^ΰ 3) Hùxß q+¬†Ç¶…Õ 4) UnHê^ΰ

REASONING ABILITY (LOGICAL) ○ ○ ○ ○ ○ ○ ○ ○


184
○ ○ ○ ○ ○ ○ ○ ○ ○ ○ ○ ○ ○ ○ ○ ○ ○ ○ ○
Study Material
12. ㄬ[`Ë ‹¬O|O^¥°Q® L^˼yH÷ LO_Í H›Æ}쁰 1) ~Œ[H©†Ç° <Œ†Ç°ä›½ ‹¬‚¬ð†Ç°O "³ˆìë=ô
1) XH› =°Oz PH÷O„¬ôQ® =¼H÷ë 2) W„¬C_È°#ß „¬i‹²Ö`Ç°…ÕÁ Wk =¶=ü…è J#°Hùx OKÇ
2) `ùO^Î~¡Qê x~¡â†Ç¶° f‹¬°HË=_ÈO W=Þ_¨xH÷ JOwH›iªë=ô
3) =°~Œ¼^Î`Ë ä›€_# q#†Ç°O 3) OKÇO WKÍó =òO^ΰ W^Í z=iªi. WOïH„¬C_ȶ
4) ‹¬=°†Ç°‹¬¶æië W=Þ#x J#°ä›½O\ì=ô.
13. h=ô H›ˆì‰§ ‚¬ð‹¬… “ ˜…Õ L#߄¬C_È°, h䛽 =°^¥¼‚¬ìßO J#ßO 4) J…ìO\÷ OKÇO`Ë =KÍó L^˼Qêxß u~¡‹¬øiªë=ô
u<Í@„¬ô_È° „¬„¬C…Õ ~Œˆ×ä =ô<Œß~ò J„¬C_È° h=ô ? 20. h ‹¬Ì‚ú^˼y J`Çx q^Î°#° ‹¬iQê x~¡ÞiëOKÇ_ÈO …è^ΰ
1) „¬„¬C#° u#䛽O_¨ =kē\÷“ "Íö~ „¬…ÿÁO f‹¬°ä›½O\ì=ô J„¬C_È° h=ô
2) ‚¬ð‹¬“…˜ W<£KŒiûH÷ „¦²~Œ¼^ΰKͪë=ô 1) öH=O h q^Î°° h=ô xiÞiëOKÇ°`Œ=ô
3) =O@ "Œx`Ë PãQ®‚¬ìOQê =¶\ìÁ_È`Œ=ô 2) J`Çx „¬x K͆ǰx`Ç<Œxß J‹¬~ŒQê f‹¬°Hùx h=ô
4) h䛽 h=ôQê =O@ K͋¬°ä›½O\ì=ô. =òO^ΰïHˆìë=ô
3) "Í°<Í[~Ÿä›½ „¦²~Œ¼^ΰ Kͪë=ô

Y
14. Z=ï~á<Œ HË~¡^Îyxk x#°ß HËi`Í J„¬C_È° h=ô
4) J`Çx qx†³¶Q®^¥~¡°#° 䛀_¨ ‹¬=°#ކǰ„¬iz

M
1) P =¼H÷ëx ^ζ~¡OQê =ôOKÇ°`Œ=ô
2) P =¼H÷ëH÷ ^ζ~¡OQê =ôO\ì=ô H›Ōh䛽 K³_ÈÛ À„~¡° ~Œä›½O_¨ Kͪë=ô.
21. h=ô #k…Õ D^ΰ`Ç°#߄¬C_È°, J#°HË䛽O_È^¥ XH› ƒìeH›

E
3) J`Ç_È° HËi#k h =Á Hê^Îx K³|°`Œ=ô
h\÷…Õ =ò#Q®°KÇ°O_È°@ KǶzuq. h䛽 D`Ç =zó#KË

D
4) UnHê^ΰ

9
59
h=ô
15. À‹ß‚¬ì=°O>è h †³òH›ø
CA
95
1) P"³°#° ~¡H÷ƪë=ô
1) XH› „¬iq°uH÷ ‹¬O|OkOz# ‹¬O|O^ÎO
2) Z=ï~á<Œ ~¡H÷ƪë~Ë Jx KǶK³^Î=ô

50
2) XH› `³e†Ç°x ƒì+¬ 3) XH› ƒì^Î
3) ã‡Ú̄¦+¬…˜ D`Ç "ŒiïHá Z^ΰ~¡° KǶªë=ô
98
A
4) W^Îí~¡° …èH› =òQ®°¾~¡° =°^Î¼Q® J#°|O^ÎO
4) UnHê^ΰ
:8
16. h=ô h Hê~¡° #_È°„¬ô`Ç°O_ÈQê J^ΰ„¬ô `DŽ²æ, XH› „¬O_ÈÁ
22. h=ô
E
h

"Œ¼‡i †³òH›ø „¬O_ÈÁ ̄áH÷ h Hê~¡° "³o¤Ok. J„¬C_È° h=ô


.P

1) ㄬ[#° q=°iÅOKÇ_¨xH÷ =¼uö~H›O


IC

1) ‡i‡é`Œ=ô (Hê~¡°`Ë ‹¬‚¬ð) 2) Z„¬C_ȶ =¼uö~H÷OKÇ=ô


AD

2) ã`Ë=…Õ ZO^ΰ䛽 ̄\ì“=ô Jx u_È`Œ=ô 3) h=ô `DŽC¬ K͋# ² „¬ô_È° q=°iÅOz# "Œix `DŽC¬ „¬>^ “ÿ =
Î ô
AB
O

3) h=Á [iy# ㄬ=¶^¥xH÷ K³_‡  é~ò# „¬O_ÈäÁ ½› _È|°Ä° 4) `DŽ¬C `³e†Ç°ä›½O_¨ Z=ix q=°iÅOKÇ=ô
ER
CH

K³eÁªë=ô 23. XH› =°x+²…Õx =°Oz H›Æ}O Uk ?


4) h `DŽ¬C Hê^Îx ‹¬=°iÖOKǰ䛽O\ì=ô.
YD

1) W`Ç~¡°#° P#O^Î „¬~¡KÇ_ÈO


17. XH› H˄¬ãQ®‹¬°ë_³á# qx†³¶Q®^¥~¡°_È° J`Ç# ‹¬=°‹¬¼#° 2) H›+¬“„¬_ „¬x K͆ǰ_ÈO 3) x*ì~òf
,H

x"ÍkOKÇ_¨xH÷ ‹Ôx†Ç°~Ÿ "Í°<Í[~Ÿ#° H›"Œ#°ä›½O>è,


T

4) QÒ~¡=OQê =¶\ìÁ_È@O
AM

J„¬C_È° h=ô 24. h †³òH›ø À‹ß‚²ì`Ç°_È°/ À‹ß‚²ì`Ç°~Œ° D =°^Î¼<Í Hù`ÇëQê


H
M

1) h=ô J`Çx`Ë =¶\ìÁ_È`Œ=ô L^˼Q®O…Õ KÍi KŒ…ì xiÞ~Œ=°OQê (ar)Qê =ô<Œß_È°


AM
IG

2) J`Çxx ƒÿkiOz ‰§OuO„¬*誐ë=ô J„¬C_È° h=ô. x#°ß JO`ÇQê „¬\ O “÷ KÇ°H˅è^x


Î `ǁOKÇ°`Œ=ô
KH

3) J`Çx ‹¬=°‹¬¼#° „¬i+¬øiOz, ‰§OuO„¬*誐ë=ô 1) h=ô 䛀_¨ J`Çx/P"³° Q®°iOz „¬\ ÷“OKÇ°HË=ô
R

4) UnHê^ΰ 2) h=ô U"³°Ø<Œ `DŽ¬C K͉§"Œ Jx J#°HùO\ì=ô


18. h=ô XH› KŒiã`Œ`ÇàH› ㄬ^͉§xß ‹¬O^ÎiÅÀ‹ë J„¬C_È° h=ô 3) W^Îí~¡° =¶\ìÁ_È°Hùx Mìm ‹¬=°†Ç¶xß öH\ì~òOKÇ°
1) JH›ø_ "Œ`Œ=~¡}ìxß P#Okªë=ô HùO\ì~¡°.
2) ㄬ`ͼH›`Ç `³°‹¬°ä›½O\ì=ô 4) UnHê^ΰ
3) _ * ÿ á < £ †³ ò H› ø ªO‹¬ ø $uH› , ªO„¦ Ô ° H› q+¬ † Ç ¶  25. XH› =‹¬°=
ë ô Hù#ß`Ç~ŒÞ`Ç P ^ΰHê}O †Ç°[=¶x W=Þ=‹²#
H›#°QùO\ì=ô ^¥xH›O>è 100 ~¡¶II° Z䛽ø= WÀ‹ë J„¬C_È° h=ô
4) UnHê^ΰ 1) h À‹ß‚²ì`Ç°ä›½ qO^ΰ U~Œæ@° Kͪë=ô
19. h †³òH›ø ãQê_È°¼†Í°+¬<£ `Ç~ŒÞ`Ç h䛽 =°Oz r`ÇO`Ë 2) 20~¡¶II° ^Í=ôx ‚¬•O_™…Õ "͋² WOHê J…ì _È|°Ä°
ㄬƒ’°`ÇÞO L^˼Q®O =zóOk. J„¬C_È° h À‹ß‚²ì`Ç°_ùH›_È° ~Œ"Œx Ptªë=ô
OKÇq°=Þ䛽O_¨ *ì~òxOQ· …ÿ@~Ÿ W=Þ~¡° Jx K³‡æ_È° 3) Z䛽ø= Wzó# _È|°Ä#° uiy Wªë=ô
J„¬C_È° h=ô 4) UnHê^ΰ

REASONING ABILITY (LOGICAL) ○ ○ ○ ○ ○ ○ ○ ○


185
○ ○ ○ ○ ○ ○ ○ ○ ○ ○ ○ ○ ○ ○ ○ ○ ○ ○ ○
Study Material
26. XH› ~ËA h=ô Hê~¡°…Õ ㄬ†¶ Ç }÷‹°¬ #
ë ߄¬ô_È° XH› Qê†Ç¶`Ë 2) J=‹¬~O¡ Uq°@x J_y, J=‹¬~Œxß |\÷“ x~¡~â òOK³^= Î ô
=ô#ß ƒìeH›#° P‹¬æãuH÷ f‹¬°ïHˆì¤x XH› =¼H÷ë ‹¬‚¬ð†Ç°O 3) Z…ìO\÷ J=‹¬~O¡ Jx `³°‹¬°HË䛽O_¨, _È|°Ä WK³ó^Î=ô
J_y`Í J„¬C_È° h=ô 4) UnHê^ΰ
1) ##°ß =ke ̄@°“ Jx "³o¤‡é†³°^Î=ô 29. ''ã‰×=°…è^ΰ …ìƒ’O …è^ΰÑÑ (Norisk No gain) J„¬C_È°
2) ‡éb‹¬°ä›½ K³„¬æO_ Jx ‹¬‚¬ð WK³ó^Î=ô h=ô
3) "³O@<Í ‚¬ð‹²æ@…˜ä›½ f‹¬°ïHˆìë=ô 1) ã‰×=° JO>è …ìƒ’O …è^Î#°ä›½O\ì=ô
2) P q"Œ^ÎO ‹¬i†³Ø°#k Jx #=òà`Œ=ô
4) "Íö~ "Œˆ×¤x J_ÈQ®=°x ‹¬‚¬ð Wªë=ô
3) J=‹¬~¡O …èx ã‰×=° „²KËóˆ×ä Kͪë~¡° Jx #=òà`Œ=ô
27. h †³òH›ø WO@~¡¶Þ ¼ H›O>è =òO^ΰ ~ËA h †³òH›ø
4) ‹¬O^Î~ŒÄ° ã‰×=°#° H›e¾ªë†Ç°x #q°à"Œ\÷x ƒ’iOz
À‹ß‚²ì`Ç°_È° WO@~¡¶¼Þ䛽 "³o¤ =zó, WO@~¡°¼ KÍÀ‹ =¼H÷ë
…샏’O ‡ÚO^ΰ`Œ=ô
'H›i”#OQêÑ ã„¬‰×߁° J_Qê_È° Jx K³‡æ_È° J„¬C_È° h=ô 30. XH› i̋„¬Âx‹¹“䛽 LO_¨ež# H›Æ}O
1) ㄬ†Ç°ußOKÇ_ÈO P̄^Î=ô

Y
1) <Œ†Ç°H›`ÇÞO 2) =üië=°`ÇÞO
2) "Íö~ ^¥xH÷ ㄬ†Ç°ußOK³^Î=ô

M
3) F~¡°æ 4) ãH›=°tH›Æ}
3) `ÍeïHá# ㄬ‰×߁° J_ÈQêx ^Í=ôxß ã‡iÖªë=ô
["Œ|°°

E
4) h=ô ‹¬i†³°Ø # ㄲ„~ö¬ +¬<£ (Preparation) `Ë =ô<Œß=ô
1) 2 2) 1 3) 3 4) 3 5) 3

D
Hê=ô# ƒì^΄¬_È=ô

9
59
6) 2 7) 2 8) 2 9) 1 10) 4
28. XH›"Íˆ× h †³òH›ø À‹ß‚²ì`Ç°_È° J=‹¬~Œ~¡ÖO HùOK³ _È|°Ä
CA
95
11) 3 12) 1 13) 2 14) 3 15) 4
J_y`Í, J„¬C_È° h=ô 16) 3 17) 3 18) 3 19) 4 20) 4

50
1) _È|°Ä° W=Þ=ô 21) 1 22) 4 23) 3 24) 3 25) 3
98
A
26) 3 27) 4 28) 3 29) 4 30) 3
:8
123456789012345678901234567890121234567890123456789012345678901212
E

12345678901234567890123456789012123456789012345678901234567890121234567890123456789012345678901212345678901234567890123
1234567890123456789012345678901212345678901234567890123456789012123456789012345678901234567890121234567890123
1234567890123456789012345678901212345678901234567890123456789012123456789012345678901234567890121234
12345678901234567890123456789012123456789012345678901234567890121234567890123456789012345678901212345678901234567890123456789012123
123456789012345678901234567890121234567890123456789012345678901212345678901234567890123
123456789012345678901234567890121234567890123456789012345678901212
12345678901234567890123456789012123456789012345678901234567890121234567890123456789012345678901212345678901234567890123
1234567890123456789012345678901212345678901234567890123456789012123456789012345678901234567890121234567890123
1234567890123456789012345678901212345678901234567890123456789012123456789012345678901234567890121234
h

12345678901234567890123456789012123456789012345678901234567890121234567890123456789012345678901212345678901234567890123456789012123
123456789012345678901234567890121234567890123456789012345678901212345678901234567890123
12345678901234567890123456789012123456789012345678901234567890121234567890123456789012345678901212345678901234567890123
1234567890123456789012345678901212345678901234567890123456789012123456789012345678901234567890121234567890123
1234567890123456789012345678901212345678901234567890123456789012123456789012345678901234567890121234
12345678901234567890123456789012123456789012345678901234567890121234567890123456789012345678901212345678901234567890123456789012123
123456789012345678901234567890121234567890123456789012345678901212345678901234567890123
123456789012345678901234567890121234567890123456789012345678901212
PREVIOUS BITS
12345678901234567890123456789012123456789012345678901234567890121234567890123456789012345678901212345678901234567890123456789012123
123456789012345678901234567890121234567890123456789012345678901212345678901234567890123
12345678901234567890123456789012123456789012345678901234567890121234567890123456789012345678901212345678901234567890123
1234567890123456789012345678901212345678901234567890123456789012123456789012345678901234567890121234567890123
1234567890123456789012345678901212345678901234567890123456789012123456789012345678901234567890121234
.P

123456789012345678901234567890121234567890123456789012345678901212
12345678901234567890123456789012123456789012345678901234567890121234567890123456789012345678901212345678901234567890123456789012123
123456789012345678901234567890121234567890123456789012345678901212345678901234567890123
12345678901234567890123456789012123456789012345678901234567890121234567890123456789012345678901212345678901234567890123
1234567890123456789012345678901212345678901234567890123456789012123456789012345678901234567890121234567890123
1234567890123456789012345678901212345678901234567890123456789012123456789012345678901234567890121234
IC

12345678901234567890123456789012123456789012345678901234567890121234567890123456789012345678901212345678901234567890123456789012123
123456789012345678901234567890121234567890123456789012345678901212345678901234567890123
12345678901234567890123456789012123456789012345678901234567890121234567890123456789012345678901212345678901234567890123
1234567890123456789012345678901212345678901234567890123456789012123456789012345678901234567890121234567890123
1234567890123456789012345678901212345678901234567890123456789012123456789012345678901234567890121234
123456789012345678901234567890121234567890123456789012345678901212
12345678901234567890123456789012123456789012345678901234567890121234567890123456789012345678901212345678901234567890123456789012123
123456789012345678901234567890121234567890123456789012345678901212345678901234567890123
12345678901234567890123456789012123456789012345678901234567890121234567890123456789012345678901212345678901234567890123
1234567890123456789012345678901212345678901234567890123456789012123456789012345678901234567890121234567890123
1234567890123456789012345678901212345678901234567890123456789012123456789012345678901234567890121234
12345678901234567890123456789012123456789012345678901234567890121234567890123456789012345678901212345678901234567890123456789012123
123456789012345678901234567890121234567890123456789012345678901212345678901234567890123
AD

1. h À‹ß‚²ì`Ç°x WO\÷…Õ P_È°`Ç°#߄¬ô_È°, J`Ç_°È XH› PKŒó^#


Î 3) J`Çx "³°_È #°O_ P Q®°_ÈÛ#° "ÍQ®=òQê f‹², H÷O^Î
AB
O

…èx q^ΰ¼`Ÿ fQ® ƒìi# „¬_#„¬C_È° h=ô (S.I. - 2008) „¬_È"͋², ^¥x̄á hˆ×¤#° ‡Ú̋^Î=ô.
ER
CH

1) h KÍ`Ç°`Ë J`Çxx „¬@°“Hùx, ^¥x ƒìi #°O_ "³eH÷ 4) P Q®°_ÈÛ#° "³eH÷f‹² ^ζ~¡=òQê ‡~¡"͆Çò^ΰ=ô
YD

4. h À‹ß‚²ì`Ç°_Èx ƒìqOz# J`Ç_È° x#°ß "³¶‹¬yOK³#x


f†Çò@䛽 ㄬ†Ç°ußOK³^Î=ô.
,H

`³e‹²HùO\÷q. J„¬ô_È° h=ô Uq° K͆Çò^ΰ=ô? (S.I.-'08)


2) P fQ®#° „¬@°“Hùx ^ζ~¡=òQê q‹²i"͆Çò^ΰ=ô.
T
AM

1) J`Çx`Ë `³Q®`³O„¬ô° K͋¬°ä›½O\ì=ô.


3) P fQ®#° K³ä›ø H›ã~¡`Ë …ìy q‹²i"͆Çò^ΰ=ô
H

2) ^³|Ä䛽 ^Î|Ä f†Çò^ΰ=ô.


M

4) "³á^ΰ¼x „²e„²OK³^Î=ô. 3) J`Çx `DŽ¬C `³°‹¬°ä›½<Í@°Á KÍ̋^Î=ô.


AM
IG

2. h À‹ß‚²ì`Ç°_È° J`Çx q"Œ‚¬ì=°‚¬ìÙ`Ǟ==ò#䛽 x#°ß 4) W`Ç~¡ À‹ß‚²ì`Ç°ä›½ J`Çx Q®°iOz K³Ì„æ^Î=ô.


KH

P‚¬ðÞxOKDžè^ΰ h=ô (S.I. - 2008) 5. h=ô |‹¬°ž…Õ L<Œß=ô. ƒìQê Q®°O„¬ôQê L#ßO^ΰ=# h=ô
R

1) J`ÇxH÷ =¼uö~H›=òQê „¬@°“^΁Qê LO_È°@ kQ®=‹²# Q®=°¼ªÖ#=ò#° |‹¬°ž KÍi#„¬ô_È° 䛀_¨. h䛽
2) P L`Ǟ==ò#䛽 ‚¬ð[~¡=ô^ΰ=ô \÷ïH\˜ f†Ç°…èx „¬i‹²Öu Z^ΰï~á#k. J„¬ô_È° h=ô Uq°
3) h ‰×ÏìHêOH›Æ#° „¬Ō^Î=ô K͆Çò^ΰ=ô. (S.I. - 2008)

4) D "³ò`ÇëO =¼=‚¬ð~¡=ò#䛽 x~¡ÁH›Æ ¼=ò K͆Çò^ΰ=ô 1) W|ÄOk #°O_ |†Ç°@„¬_È°@䛽 `ÇÞ~¡Qê |†Ç°@䛽
3. XH› À‹ß‚²ì`Ç°x ‡s“…Õ L#߄¬C_È° h À‹ß‚²ì`Ç°x QùO`ǰ̄á
Q®O`Ç° "͆Çò^ΰ=ô.
2) H›O_ÈH›“~¡°#° „²ez, J`ÇxH÷ _ÈaÄzó, \÷ïH\˜ Wzó
L#ß Q®°_ÈÛ J`Çx "³#°H› Q® |Á̄á Q® ãHù"ùÞuë =#
"͆ǰ=°x K³Ì„æ^Î=ô.
JyßH÷ Z~¡†°Ç Q®°@#° KǶzuq J„¬C_È° h=ô (S.I.- 2008)
3) ^ÎQ®¾~¡Qê 䛀~¡°ó#ß XH›iH÷ P _ÈaÄzó, H›O_ÈH›“~¡°ä›½
1) h À‹ß‚²ì`Ç°x "³#°H›ä›½ KǶ_È=°x J_È°Q®°^Î=ô
Wzó"͆ǰ=°x K³Ì„æ^Î=ô.
2) À‹ß‚²ì`Ç°x `ÇeÁx „²°ó@䛽 „¬~¡°ïQ`³ë^Î=ô 4) ¢_³á"£ä›½ P _È|°Ä WK³ó^Î=ô.

REASONING ABILITY (LOGICAL) ○ ○ ○ ○ ○ ○ ○ ○


186
○ ○ ○ ○ ○ ○ ○ ○ ○ ○ ○ ○ ○ ○ ○ ○ ○ ○ ○
Study Material
6. h=ô ‡~¡°ø…Õ Hêe |O`Œ@#° P_È°KÇ°O\÷q h=ô Hêe`Ë 3) P H›=~¡°#° `³iz KǶz, =°~¡ ‡é‹¹“ K͆Çò^ΰ=ô.
|Oux `Çxß#„¬ô_È° Jk ㄬH›ø# Q® WO\÷…Õx H÷\ ÷H© 4) ^ÎQ®¾~¡…Õ L#ß `LJ…Õ _È|Ä…Õ ‡é‹¹“ K³Ì‹^Î=ô.
J^Îí=ò#䛽 `Çye, „¬_‡é~ò#k J„¬ô_È° h=ô 10. h=ô ZH›ø=‹²# ï~ၰ J„¬C_Í |†Ç°°^Í~¡ƒÕ`Ç°#ß^Î#ß
(S.I. - 2008) q+¬†°Ç =ò h=ô ~ ï …
á Þè À‹“+#
¬ °ä›½ KÍi# "³O@<Í `³e‹²Hù<³^= Î ô.
1) P WO\÷ †Ç°[=¶x =^Îí #°O_ h |Oux uiy W=°àx J„¬ C _È ° KÍ † Ç ° =‹² # ‹¬ i †³ Ø ° # qk . (h=ô \÷ ï H \˜
^³á~¡¼=òQê HËï~^Î=ô. f‹¬°H˅è^ΰ) (S.I. - 2008)

2) h `ÇÀ„æg° …è^Îx K³Ì„æ^Î=ô. 1) P ~ï  á °#° =^΁H›, `Ç~°¡ "Œ`Ç q~Œ=°, ªÖ#=ò…Õ \÷.\÷.S.
3) ~¡‚¬ì‹¬¼=òQê h |Oux `³K³ó^Î=ô. =^Îí \÷Hï \˜ Hù#…èx h P‰×H` ë÷ #
Ç ° `³e†Ç°*è†òÇ @.
4) P WO\÷ †Ç°[=¶xH÷ H›= Æ ¶„¬}° `³e„², P J^Î=
í ò#H÷ 2) ï~ၰ =^Îí䛽 „¬~¡°ïQuë h ㄬ†Ç¶}=ò#° Q®°@°“Qê
|^ΰ° "Í~ùH› ^¥xH÷ ZO`Ç Y~¡óQ®°<Ë ^¥xx =ò@“ ªyOKÇ°@
K³Ì„æ^Î=ô. 3) "³ò^Î\ ÷ \÷ïH@°“#° Hùx, P ï~ၰ JH›ø_Í LO>è Z䛽ø@

Y
7. h ªß<Œ Q®k…Õx 䛽ˆì~ò Hê~¡°`Ƕ, zHê䛽 H›eyOKÇ° 4) H› ^ Î ° °`Ç ° #ß ï ~ á  °#° Zä› ½ ø@ J<Í Q® O _È = ò#°

M
‰×|í=ò#° x~¡O`Ç~¡=òQê K͆ÇòKÇ°#ßk J„¬ô_È° h=ô? Z^ΰ~ùø#°@ H›<Œß P ï~ၰ =ke"͆Çò@.

E
(S.I. - 2008) 11. h=ô XH› ‹¬¶ø@~ŸÌ„á ~¡‚ì¬ ^¥i…Õ ‡é=ô#„¬ô_È° ƒÿHá ̱ „á W~¡°=ô~¡°

D
1) K³=ôÌ„á `ǁQ®_È#° LOKÇ°Hùx xã^·é=ô^ΰ=ô.

9
XH› ¢‹Ôë †³òH›ø |OQê~¡O Qù°‹¬° H›|oOKÇ°@#° KǶK³^Î~¡°

59
2) H÷O^Î XH› ƒìeóx LOK³^Î=ô. J„¬ô_È° h=ô (S.I. - 2008)

CA
95
3) P 䛽ˆì~ò ^¥Þ~¡=ò…Õ XH› a~¡_¨#° LOKÇ°@䛽 1) ¢‹Ôëx F^¥~¡°ó^ΰ=ô

50
ㄬ†Ç°uߪë=ô. 2) "Œix „¬@°“Hù#°@䛽 ƒì°~¡ "³O@„¬_³^Î=ô
98
A
4) XH› 䛽ˆì~ò =°~¡=°à`Ç°ë ^¥ix, =°~¡=°`Ç°ë K͆Çò@䛽 3) P q+¬†Ç°O Q®°iOz ‡éb‹¬°ä›½ `³ē^Î=ô
:8
„²°`Ç°=ô. 4) `Ç~¡°"Œ`Ç Uq° [~¡°Q®°#x x|_ KǶK³^Î=ô
E
h
.P

8. XH› „¬sHÆê ‚¬ð°…Õ, ㄬ‰§ß„¬ã`Ç=ò KŒ…ì H›i# ” =òQê LO_, 12. h=ô K͋²# `³eq `Ç䛽ø= `DŽ¬C° Zuë KǶ„¬|_#q. h=ô
IC

1) #=ôÞ`Ƕ f‹²"͆Çò^ΰ=ô
AD

h=ô ‹¬O`Ç$„²Hë ~
› O
¡ Qê ã"Œ†Ç°…è#„¬ô_È° h=ô K͆°Ç ^ÎQ°® ‹¬i†³°Ø # (S.I. - 2008)

„¬x 2) Hù‹¬yO`Ç°=ô
AB
O

(S.I. - 2008)

1) „¬sH›Æ䛽x`Ë ã„¬‰×߁° ‡~”¡¼„¬ô‹¬ëH›=ò…Õ …è^Îx K³„¬C@ 3) zO`ŒãQ®‹¬°ë_È=ô HêQ®=ô


ER
CH

2) „¬ s H› Æ ã"Œ†Ç ò "Œix ‚¬ 𠁰 #°O_ "³  °„¬ e H÷ 4) H›$`ÇAý_È°Qê ƒìqO`Ç°=ô


YD

~Œ=‹²OkQê ãÀ„ö~„²OKÇ°@ 13. h=ô h À‹ß‚²ì`Ç°x ‡s“…Õ L#߄¬ô_È°, h À‹ß‚²ì`Ç°x =°„¦ ¹ ~Ÿ


,H

3) h ‡Ú~¡°Q®°"Œx =^Îí #°O_ U^³á# `³e‹²Hù#°@䛽 (muffler) H÷ x„¬C JO@°ä›½Ok. J„¬ô_È° h=ô
T
AM

ㄬ†Ç°ußOKÇ°@ (S.I. - 2008)


H
M

4) x^¥#OQê P…ÕzOz, h䛽 `³e‹²#O`Ç =~¡ä›½ P 1) h À‹ß‚²ì`Ç°x „²ez JH›ø_È KǶ_È° Jx K³„¬C^ΰ=ô
AM
IG

ㄬ‰×߁䛽 `Çy# ["Œ|° Hù~¡ä›½ ㄬ†Ç°ußOKÇ°@ 2) h=ô "³O@<Í J`Çx J=°à …è^¥ <Œ#ß䛽 ‡¦é<£ K³Ì‹^Î=ô
KH

9. h=ô ‡~”¡‰§ #°O_ uiy =KÇ°óKÇ°<Œß=ô. h=ô =KÇ°ó 3) `ùO^Î~¡Qê P x„¬C L#ß =°„¬Á~Ÿ J`Çx "³°_È#°O_
R

^¥i…Õ Jã_ȋ¬° =°i†Çò L„¬†³¶yO„¬|_Èx ª“O„¬ô#° f̋^Î=ô, `Ç~¡°"Œ`Ç =°Oz h~¡°x ^¥xÌ„á ‡éK³^Î=ô
H›ey L#ß XH› H›=~¡°#° XH› gk…Õ KǶzuq J„¬ô_È° 4) h=ô =°„¦¬Á~Ÿ f‹² ^ζ~¡=òQê q‹²ï~^Î=ô.
h=ô (S.I. - 2008)

1) ^¥xx ZH›ø_ ªÖ#=ò…Õ JH›ø_Í =ke "³o¤ ‡é=ô^ΰ=ô.


["Œ|°°
1) 3 2) 3 3) 3 4) 3 5) 2
2) P ª“ O „¬ ô #° `ùyOz, P H› = ~¡ ° #° <Œ‰× # O
6) 4 7) 4 8) 4 9) 4 10) 3
K͆Çò^ΰ=ô. 11) 2 12) 4 13) 3

REASONING ABILITY (LOGICAL) ○ ○ ○ ○ ○ ○ ○ ○


187
○ ○ ○ ○ ○ ○ ○ ○ ○ ○ ○ ○ ○ ○ ○ ○ ○ ○ ○
Study Material
7 ãu‡^Î
ãu‡^Î
`Ç~¡ø"Œ^Î=ò
`Ç~¡ø"Œ^Î=ò

(Syllogism)

=òMì¼O‰§°...
G ãu‡^Î `Ç~¡ø"Œ^Î=ò qƒìQ®O#O^ΰ ï~O_È° ㄬH›@#° =°i†Çò ^¥xH÷O^Î ï~O_È° x~¡â†Ç¶° W=Þ|_È`Œ~ò. Wzó# x~¡â†Ç¶…Õ

Uk ‹¬i†³Ø°#^Ë ZOKÇ°HË"Œe.

G ï~O_È° H›O>è Z䛽ø= x~¡â†Ç¶#° Wzó#„¬ô_È° ãH÷O^Î W=Þ|_# P„¬Â<£#°O_ ‹¬i†³Ø°# ^¥xx ZOKÇ°HË"Œe.

Y
Wzó# ㄬH›@#° "Œ‹¬ë"ŒxH÷ q~¡°^ÎíOQê =ô<Œß ^¥x`Ë =°#䛽 ‹¬O|O^ÎO…è^ΰ. ㄬH›@##° P^¥~¡O K͋¬°ä›½x =¶ã`Ç"Í°

M
x~¡â†Ç¶ #°O_ ‹¬ï~á# "Œ\ ÷x ZOKÇ°HË"Œe.

G "³ò^Î\ ÷ x~¡â†Ç°O =¶ã`Ç"Í° ‹¬i†³Ø°#k J~ò`Í ‹¬=¶^¥#O (a) Qê#¶, ï~O_È= x~¡â†Ç°O =¶ã`Ç"Í° ‹¬i†³Ø°#k J~ò`Í

E
‹¬=¶^¥#O (b) Qê#¶, ï~O_È° x~¡â†Ç¶¶ ‹¬i†³Ø°#"Í J~ò`Í ‹¬=¶^¥#O (c) Qê#¶, ï~O_È° x~¡â†Ç¶¶ ‹¬ï~á#q HêH›‡é`Í

D
9
‹¬=¶^¥#O (d) Qê#¶, ï~O_O\ ÷…Õ U^ËXH›\ ÷ ‹¬i†³Ø°#k J~ò`Í ‹¬=¶^¥#O (e) Qê#¶ Q®°iëOKŒe.

59
CA
ㄬH›@#…Õ W=Þ|_# ‹¬=¶KŒ~Œxß =¶ã`Ç"Í° ãQ®‚²ìOKŒe. W`Ç~¡ q=~¡=ò#° f‹¬°HË䛽O_¨ =òyO„¬ô#° P…ÕzOKŒe.

95
G D qƒìQ®O…Õx ㄬ‰×߁#° ªkOKÍ ãH›=°O…Õ, "³ò^Î@ ㄬH›@##° KÇkq J~¡ÖO K͋¬°Hùx ^¥xH›#°Q®°}OQê "³<£ zã`Œxß

50
xiàOz D "³<£ zã`ŒxH÷ J#°Q®°}OQê x~¡â†Ç¶° =ô<Œß†³¶, …è^Ë Q®°iëOKŒe.

G 98
A
‹¬ƒÿû䛽“ (H›~¡ë#°) =¶ã`Ç"Í° ^Î$gH›iOKÇ° ㇐u„¬kH›#° ª~¡ÞãuH›OQê ^Î$gH›iOKÇ|_# ㇐u„¬kH› (A) JO\ì~¡°. ª~¡ÞãuH›OQê
:8
E

^Î$gH›iOKÇ|_# ㇐u„¬kH›ä›½ =¼uö~H›OQê Un LO_È^ΰ.


h

L^¥ :
.P

JO^Î~¡° ƒìeH›° ‡¦¼+¬<£ ã_ȋ¹° ^ÎiOKŒ~¡°.


IC
AD

̄á ㄬH›@# ª~¡ÞãuH›OQê ^Î$gH›iOKÇ|_#k. Hê=ô# nxH÷


AB
O

=¼uö~H›OQê Un =ôO_È^ΰ. J#Qê ‡¦¼+¬<£ ã_ȋ¹ ^ÎiOz#

"Œ~¡O^Î~¡¶ ƒìeH›° J#_ÈO `DŽ¬C.


ER

G
CH

`ŒiøH› À‚ì`Ç°"Œ^¥xH÷ ‹¬O|OkOz D J^¥¼†Ç°O KŒ…ì =òY¼"³°Ø#k.

G
YD

ª~¡ÞãuH›OQê =¼uö~H›"³°Ø# ㇐u„¬kH› (E) ‹¬ƒÿû䛽“ (H›~¡ë)#° =°i†Çò ãH÷†Ç°#° ^Î$gH›iOKÇ°#°.

L^¥ :
,H

ãƒÿ_£ Un u#_¨xH÷ J#°=ôQê …è^ΰ.


T
AM
H
M
AM
IG
KH

ä„`¤ v$_Ïz$r

G
R

xií+¬“OQê =¼uö~H›"³°Ø# ㇐u„¬kH› (I) H›~¡ë#° Qêh, ãH÷†Ç°#°Qêh ^Î$gH›iOKÇ^ΰ.

L^¥ : HùO`Ç=°Ok ãH÷ïH\˜ P_Í"Œ~¡° „¦¬ô\˜ƒì…˜ 䛀_¨ P_È`Œ~¡°. nxH÷ =¼uö~H›OQê 䛀_¨ LO_È=KÇ°ó. J#Qê HùO`Ç=°Ok

„¦¬ô\˜ƒì…˜ P_Í"Œ~¡° ãH÷ïH\˜ 䛀_¨ P_È`Œ~¡°. Wk 䛀_¨ ‹¬i†³Ø°#^Í.

…§­õ]™„톙

äIøðI ]™ Q`Î#¢±

Q`Î#¢±

G xi푐“xH÷ =¼uö~H›"³°Ø# ㇐u„¬kH› (O) ãH÷†Ç°#° =¶ã`Ç"Í° ^Î$gH›iOKÇ°#°.

L^¥ : HùO`Ç=°Ok 䛽‹Ô놳¶^Î°° "³ò~¡@°"Œˆ×ä Hê^ΰ.

REASONING ABILITY (LOGICAL) ○ ○ ○ ○ ○ ○ ○ ○


188
○ ○ ○ ○ ○ ○ ○ ○ ○ ○ ○ ○ ○ ○ ○ ○ ○ ○ ○
Study Material
G U"³á<Œ ï~O_È° =òyO„¬ô° =ô#ßKË "Œ\ ÷x ㄬ`ͼH›"³°Ø# ‹¬O^Î~ŒÄ°Qê „¬iQ®}÷ªë~¡°. J…ìO\ ÷ ‹¬=°†Ç°O…Õ ‹¬=¶^¥#=ò

W=Þ|_# x~¡â†Ç¶…Õ "³ò^Î\ ÷k …è^¥ ï~O_È=k J^Í q^ÎOQê LO@°Ok.

L^¥ : ̄á# q=iO„¬|_# "Œ\ ÷#°O_ E+I J#Qê ª~¡ÞãuH›OQê =¼uö~H›"³°Ø#k { xií+¬“OQê ^Î$gH›iO„¬|_#k (…è^¥)

A+E J#Qê ª~¡ÞãuH›OQê ^Î$gH›iOz#k { ª~¡ÞãuH›OQê =¼uö~H›"³°Ø#k (…è^¥) I+O J#Qê xií+¬“OQê ^Î$gH›iOz#k

{ xií+¬“OQê =¼uö~H›"³°Ø#k.

D q^ÎOQê W=Þ|_# ㄬH›@#ä›½ ‹¬=¶^¥#O x~¡â†Ç¶…Õ U^Ë XH›\ ÷ J=ô`Ç°Ok.

G ㄬ‰×߁° ï~O_È° q^¥°Qê J_È°Q®°`Œ~¡°. Jq

1) ï~O_È° JOwH›$`Ç ã‡^ŠÎq°H› JO‰×=òä›½ ‹¬O|OkOz# "Œ^Î#° =°i†Çò

2) ï~O_È° H›O>è Z䛽ø= JOwH›$`Ç ã‡^ŠÎq°H› JO‰×=òä›½ ‹¬O|OkOz# "Œ^Î#°.

G "³ò^Î\ ÷ ~¡H›O ㄬ‰×߁…Õ ï~O_È° ㄬH›@#° Wzó "Œ\ ÷ ãH÷O^Î ï~O_È° x~¡â†Ç¶° W=Þ|_È`Œ~ò.

Y
ï~O_È= ~¡H›O ㄬ‰×߁…Õ ï~O_È° ㄬH›@#° Wzó "Œ\ ÷ ãH÷O^Î ï~O_È° H›O>è Z䛽ø= x~¡â†Ç¶° W=Þ|_È`Œ~ò.

M
=ü_È=~¡H›O ㄬ‰×߁…Õ ï~O_È° H›<Œß Z䛽ø= ㄬH›@#° W=Þ|_ "Œ\ ÷ ãH÷O^Î ï~O_È° H›<Œß Z䛽ø= x~¡â†Ç¶° W=Þ|_È`Œ~ò.

1234567890123456789012345678901212345678901234567890123456789012123456789012345678901234567890121234567890123
1234567890123456789012345678901212345678901234567890123456789012123456789012345678901234567890121234
123456789012345678901234567890121234567890123456789012345678901212
12345678901234567890123456789012123456789012345678901234567890121234567890123456789012345678901212345678901234567890123456789012123
1234567890123456789012345678901212345678901234567890123456789012123456789012345678901234567890
12345678901234567890123456789012123456789012345678901234567890121234567890123456789012345678901212345678901234567890123
1234567890123456789012345678901212345678901234567890123456789012123456789012345678901234567890121234567890123
1234567890123456789012345678901212345678901234567890123456789012123456789012345678901234567890121234
12345678901234567890123456789012123456789012345678901234567890121234567890123456789012345678901212345678901234567890123456789012123
1234567890123456789012345678901212345678901234567890123456789012123456789012345678901234567890
123456789012345678901234567890121234567890123456789012345678901212

E
12345678901234567890123456789012123456789012345678901234567890121234567890123456789012345678901212345678901234567890123
1234567890123456789012345678901212345678901234567890123456789012123456789012345678901234567890121234567890123
1234567890123456789012345678901212345678901234567890123456789012123456789012345678901234567890121234
12345678901234567890123456789012123456789012345678901234567890121234567890123456789012345678901212345678901234567890123456789012123
1234567890123456789012345678901212345678901234567890123456789012123456789012345678901234567890
123456789012345678901234567890121234567890123456789012345678901212
PRACTICE TEST
12345678901234567890123456789012123456789012345678901234567890121234567890123456789012345678901212345678901234567890123456789012123
1234567890123456789012345678901212345678901234567890123456789012123456789012345678901234567890
12345678901234567890123456789012123456789012345678901234567890121234567890123456789012345678901212345678901234567890123
1234567890123456789012345678901212345678901234567890123456789012123456789012345678901234567890121234567890123
1234567890123456789012345678901212345678901234567890123456789012123456789012345678901234567890121234
12345678901234567890123456789012123456789012345678901234567890121234567890123456789012345678901212345678901234567890123456789012123
1234567890123456789012345678901212345678901234567890123456789012123456789012345678901234567890
12345678901234567890123456789012123456789012345678901234567890121234567890123456789012345678901212345678901234567890123
1234567890123456789012345678901212345678901234567890123456789012123456789012345678901234567890121234567890123
1234567890123456789012345678901212345678901234567890123456789012123456789012345678901234567890121234
123456789012345678901234567890121234567890123456789012345678901212
12345678901234567890123456789012123456789012345678901234567890121234567890123456789012345678901212345678901234567890123456789012123
1234567890123456789012345678901212345678901234567890123456789012123456789012345678901234567890
12345678901234567890123456789012123456789012345678901234567890121234567890123456789012345678901212345678901234567890123
1234567890123456789012345678901212345678901234567890123456789012123456789012345678901234567890121234567890123
1234567890123456789012345678901212345678901234567890123456789012123456789012345678901234567890121234
123456789012345678901234567890121234567890123456789012345678901212
12345678901234567890123456789012123456789012345678901234567890121234567890123456789012345678901212345678901234567890123456789012123
1234567890123456789012345678901212345678901234567890123456789012123456789012345678901234567890
12345678901234567890123456789012123456789012345678901234567890121234567890123456789012345678901212345678901234567890123
1234567890123456789012345678901212345678901234567890123456789012123456789012345678901234567890121234567890123
1234567890123456789012345678901212345678901234567890123456789012123456789012345678901234567890121234
12345678901234567890123456789012123456789012345678901234567890121234567890123456789012345678901212345678901234567890123456789012123
1234567890123456789012345678901212345678901234567890123456789012123456789012345678901234567890

D
9
59
G (1-20) 5. ㄬH›@#° :
CA (BOM PO 2010)

95
ㄬ‰×ß …Õ ㄬH›@#° Wzó ãH÷O^Î x~¡â†Ç¶° 1) Hùxß <Ë@°Á <Œ}聰.

WKŒó~¡°. g°~¡° ‹¬=¶^¥<Œxß D ãH÷Ok q^O


Î Qê Q®°iëOKŒe. 2) U <Œ}ÿ=ü Hê~¡°Û Hê^ΰ

50
"³ò^Î\ ÷ x~¡†
â °
Ç O =¶ã`Ç"°
Í ‹¬i†³°
Ø #k J~ò`Í ‹¬=¶^¥#O x~¡â†Ç¶° : I) Jxß Hê~¡°Û° <Ë@°Á J=Þ=KÇ°ó#°

98
A
(a) Qê#¶, II) Jxß <Ë@°Á <Œ}ÿ¶ Hê=ô, Hê~¡°
Û ¶ Hê=ô
:8
~
ï O_È= x~¡†
â °
Ç O =¶ã`Ç"°
Í ‹¬i†³°
Ø #k J~ò`Í

(b) 6. ㄬH›@#° :
E

1) Hùxß Wˆ×ÃÁ ƒ’=O`Ç°°


‹¬=¶^¥#O Qê#¶, ï~O_È° x~¡â†Ç¶¶ ‹¬i†³Ø°#"Í
h
.P

J~ò`Í ‹¬=¶^¥#O (c) Qê#¶, ï~O_È° x~¡â†Ç¶¶


2) U ƒ’=Ou J‡~Ÿ“"³°O\˜ Hê^ΰ
IC

x~¡â†Ç¶° : I)
AD

‹¬ï~á#q HêH›‡é`Í ‹¬=¶^¥#O (d) Qê#¶, ï~O_O\ ÷…Õ


H›h‹¬O Hùxß ƒ’=O`Ç°° Wˆ×ÃÁ Hê^ΰ

(e) II) Hùxß ƒ’=O`Ç°° J‡~Ÿ“"³°O\˜°


AB
O

U^ËXH›\ ÷ ‹¬i†³Ø°#k J~ò`Í ‹¬=¶^¥#O Qê#¶

Q®°iëOKŒe.
7. ㄬH›@#° : 1) U F_È „¬_È= Hê^ΰ
ER
CH

1. ㄬH›@#° : 1) Jxß [O`Ç°=ô° „¬ä›Æ½°


2) Jxß „¬_È=° ‡ã`ǁ°

x~¡â†Ç¶° : I)
YD

H›h‹¬O Hùxß ‡ã`ǁ° „¬_È=°


2) Hùxß [O`Ç°=ô° =°#°+¬µ°

II)
x~¡â†Ç¶° : I)
,H

U F_È ‡ã`Ç Hê^ΰ


HùO`Ç=°Ok =°#°+¬µ° „¬ä›Æ½°

8. ㄬH›@#° :
T

II)
AM

1) Jxß Qêe„¬\쁶 q=¶<Œ…è


Hùxß „¬ä›Æ½° =°#°+¬µ°

2. ㄬH›@#° :
H

2) U q=¶#=ü „¬H÷Æ Hê^ΰ


M

1) 䛽H›ø° <Œ°Q®° Hêˆ×ä H›ey =ô<Œß~ò

x~¡â†Ç¶° : I)
AM

H›h‹¬O Hùxß Qêe„¬\쁰 „¬ä›Æ½°


IG

II)
2) |Á° <Œ°Q®° Hêˆ×ä H›ey =ô<Œß~ò

x~¡â†Ç¶° : I) U Qêe„¬@=ü „¬H÷ÆHê^ΰ


KH

9. ㄬH›@#° :
|Á° Jhß ä›½H›ø°

II)
1) Hùxß QêÁ‹¹° …Õ‚¬ð°
R

䛽H›ø° Jhß |Á°

3. ㄬH›@#° :
2) Hùxß ã^Î"Œ° …Õ‚¬ð°

x~¡â†Ç¶° : I)
1) QË^Î°=° ~¡OQ®° Z~¡°„¬ô =°i†Çò hO

U ã^Î==ü QêÁ‹¹ Hê^ΰ

II)
~¡OQ®° P䛽„¬KÇó

H›h‹¬O Hùxß QêÁ‹¹° ã^Î"Œ°

10. ㄬH›@#° :
2) P䛽„¬Kó
Ç ~¡OQ®° Q®°…ìa =°i†Çò „¬‹°
¬ „¬ô
1) Jxß ƒì¼O䛽° H›ˆì‰§°

~¡OQ®° Z~¡°„¬ô

x~¡â†Ç¶° : I)
2) Hùxß H›ˆì‰§° ‡~”¡‰§°

„¬‹¬°„¬ô ~¡OQ®° QË^Î°=°


x~¡â†Ç¶° : I)
II)
U ƒì¼O䛽 ‡~”¡‰§ Hê^ΰ

II)
Q®°…ìa ~¡OQ®° hO
Hùxß ‡~”¡‰§° H›ˆì‰§° Hê^ΰ

4. ㄬH›@#° : 1) Hùxß ~Œˆ×ä Qˈ쁰


11. ㄬH›@#° : 1) JO^Î~¡° ~¡KÇ~ò`ǁ¶ …ì†Ç°~Ÿ°

2) Jxß Qˈ쁰 =ò`Œ¼°


2) JO^Î~¡° KÇk"Í"Œ~¡° …ì†Ç°~Ÿ°

x~¡â†Ç¶° : I) Hùxß ~Œˆ×ä =ò`Œ¼°


x~¡â†Ç¶° : I) HùO`Ç=°Ok …ì†Ç°~Ÿ° KÇk"Í"Œ~¡°

II)
II)
Jxß =ò`Œ¼° ~Œˆ×ä
HùO`Ç=°Ok KÇk"Í"Œ~¡° ~¡KÇ~ò`ǁ°

REASONING ABILITY (LOGICAL) ○ ○ ○ ○ ○ ○ ○ ○


189
○ ○ ○ ○ ○ ○ ○ ○ ○ ○ ○ ○ ○ ○ ○ ○ ○ ○ ○
Study Material
12. ㄬH›@#° : 1) Jxß „¬~¡°ž¶ fQ®° x~¡â†Ç¶° : I) Hùxß ‹Ô`ŒHËH›z°H›° „¬ô=ôށ°

2) U fQê |°@“ Hê^ΰ II) Hùxß ‹Ô`ŒHËH›z°H›° „¬ô=ôށ°

x~¡â†Ç¶° : I) U „¬~¡¶ž |°@“Hê^ΰ J=Þ=KÇ°ó.

II) H›h‹¬O Hùxß „¬~°


¡ ž° |°@“° J=ô`Œ~ò
17. ㄬH›@#° : 1) Jxß |¶@°Á K³„¬C°

13. ㄬH›@#° : 1) Jxß Q®_†Ç¶~Œ° |¶@°Á


2) Hùxß K³„¬C° ªO_ȅ˜ž

2) Hùxß Q®_†Ç¶~Œ° LOQ®~Œ°


x~¡â†Ç¶° : I) Hùxß ªO_ȅ˜ž |¶@°Á

(Allahabad Bank Cl - 2011)


II) U ªO_ȅ˜ |¶@° Hê^ΰ

x~¡â†Ç¶° : I) Hùxß LOQ®~Œ° |¶@°Á


18. ㄬH›@#° : 1) Jxß `ǁ°„¬ô¶ H÷\ ÷H©°

II) Jxß LOQ®~Œ¶ |¶@°Á


2) _È<£„¹ XH› `ǁ°„¬ô

14. ㄬH›@#° : 1) Jxß Q®°_¨~Œ¶ Q®^ΰ°


x~¡â†Ç¶° : I) _È<£„¹ XH› H÷\ ÷H©

II)
2) Jxß Q®^ΰ¶ Wˆ×ä

Y
_È<£„¹ XH› `ǁ°„¬ô, H÷\ ÷H© Hê^ΰ

(Allahabad Bank Cl - 2011)


19. ㄬH›@#° :
x~¡â†Ç¶° : I)
1) Jxß ~¡OQ®°¶ hO ~¡OQ®°q

M
Jxß Q®°_¨~Œ¶ Wˆ×ä

II)
2) Hùxß „¬ô‹¬°„¬ô ~¡OQ®°° #°„¬ôq

x~¡â†Ç¶° : I)
Jxß Wˆ×´¤ Q®°_¨~Œ°

E
15. ㄬH›@#° :
Jxß hO ~¡OQ®°¶ #°„¬ôq

II)
1) Hùxß U#°Q®°° Q®°ã~Œ°

D
Hùxß #°„¬ô° ~¡OQ®°°

9
20. ㄬH›@#° :
2) J‰×Þ XH› Q®°ã~¡O

59
x~¡â†Ç¶° : I)
1) Jxß U#°Q®°¶ Qê_^΁°

CA
95
J‰×Þ XH› U#°Q®°

II)
2) Hùxß Qê_^
Î ° #°„¬ô ~¡OQ®°…Õ =ô<Œß~ò.

x~¡â†Ç¶° : I)
J‰×Þ XH› U#°Q®° Hê^ΰ

50
16. ㄬH›@#° : Jxß U#°Q®°¶ #°„¬ô ~¡OQ®°q

II)
1) Hùxß „¬ô=ôށ° ƒçkíOH›°

98
A
Hùxß U#°Q®°° #°„¬ô ~¡OQ®°q
2) Hùxß ƒçkíOH›° ‹Ô`ŒHËH›z°H›°
:8
1234567890123456789012
12345678901234567890123456789012123456789012345678901234567890121234567890123456789012345678901212345678901234567890123
12345678901234567890123
12345678901234567890123456789012123456789012345678901234567890121234567890123456789012345678901212345678901234567890123
1234567890123456789012 12345678901234567890123
q=~¡}ì`ÇàH› ["Œ|°°
E

1234567890123456789012
12345678901234567890123456789012123456789012345678901234567890121234567890123456789012345678901212345678901234567890123
1234567890123456789012 12345678901234567890123
12345678901234567890123
12345678901234567890123456789012123456789012345678901234567890121234567890123456789012345678901212345678901234567890123
h

1234567890123456789012
1234567890123456789012 PRACTICE TEST : 12345678901234567890123
12345678901234567890123456789012123456789012345678901234567890121234567890123456789012345678901212345678901234567890123
12345678901234567890123456789012123456789012345678901234567890121234567890123456789012345678901212345678901234567890123
.P

1234567890123456789012 12345678901234567890123
12345678901234567890123
12345678901234567890123456789012123456789012345678901234567890121234567890123456789012345678901212345678901234567890123
IC
AD

…­åœÇ¾‚±
AB
O

R̉킱
\Paȱ>õ‚±
ER

(c) ‰ØÄ¥
CH

(a)
>óa½‚±
1. 4.
YD
,H

>±$±,­¶‚± "³ < £ z ã`Ç O #°O_ ‹¬ æ +¬ “ O Qê x~¡ ⠆ Ç ° O – I =¶ã`Ç"Í°


T
AM

"³<£ zã`ÇO #°O_ x~¡â†Ç°O –I =°i†Çò x~¡â†Ç°O – II ‹¬i†³Ø°#k.


H

=ÌA±Â
M

ï~O_ȶ ‹¬i†³Ø°#"Í.
AM

(d)
IG

2. Wzó# ㄬ H › @ #…Õ L=°à_ „¬ ^ Î O …è ^ Î ° . Hê=ô#


5. (a) (…è^¥)
KH

x~¡â†Ç¶° ï~O_ȶ ‹¬i†³Ø°#q Hê^ΰ.

RÌ_Ï±>± =ÌA±Â I뢱܂± I뢱܂± =~邱


=~邱
R

Q圾…­LÈô
(i) (ii)
3. (d) [¢±…­õ (…è^¥)

"³<z
£ ã`ÇO (ii) #°O_ x~¡†
â °
Ç O – I ‹¬i†³°Ø #q. "³<z
£ ã`ÇO

i‚P R¯±†íb
(i) =°i†Çò (ii)  #°O_ x~¡â†Ç°O – II `DŽ¬C. J#Qê

…­Œ­±…­õ
(i) (ii) x~¡â†Ç°O –I =¶ã`Ç"Í° ‹¬i†³Ø°#k.

"³<£zã`ÇO (i) #°O_ „¬‹¬°„¬ô~¡OQ®° H›zó`ÇOQê QË^Î°=°


6. (d)
Jx K³„¬æ…è=ò.

"³<£zã`ÇO (ii) #°O_ Q®°…ìa ~¡OQ®° JO`Œ hO Jx „“>Paȱ‚± X‰ØÄ¥ Kˆ‘ ”#´±P]™‚±
K³„¬æ…è=ò.

"³<£zã`ÇO #°O_ x~¡â†Ç°O –I =°i†Çò x~¡â†Ç°O – II


Hê=ô# x~¡â†Ç°O – I =°i†Çò x~¡â†Ç°O – II ï~O_ȶ

ï~O_ȶ ‹¬i†³Ø°#q Hê=ô.


‹¬i†³Ø°#q Hê^ΰ.

REASONING ABILITY (LOGICAL) ○ ○ ○ ○ ○ ○ ○ ○


190
○ ○ ○ ○ ○ ○ ○ ○ ○ ○ ○ ○ ○ ○ ○ ○ ○ ○ ○
Study Material
…­`É> G`É …­`É> }±A”‚±
7. (a) (…è^¥) 12. (a) …­¢±Ÿ‚±
ˆ‘äaȂ± ˆ‘äaȂ± gR¯‚±
G`É "³<£zã`ÇO #°O_ x~¡â†Ç°O –I ‹¬i†³Ø°#k.

R¯`Ž‡È·‚±
–I }·A±Â
"³<£zã`Œ #°O_ ‹¬æ+¬“OQê x~¡â†Ç°O ‹¬i†³Ø°#k
R¯`Ž‡È·‚± MPR¯‚±
"³<£zã`ÇO (i) #°O_ x~¡â†Ç°O – II `DŽ¬C
13. (e) (…è^¥)

Rëf…­]킱 }·A±Â
MPR¯‚±
(b)
8.
…­åœÇ¾‚± (i) (ii)

r>·=‚± "³<£zã`ÇO (i) #°O_ Hùxß LOQ®~Œ° |¶@°Á J#°#k

Y
"³<£zã`ÇO #°O_ ‹¬æ+¬“OQê x~¡â†Ç°O – II =¶ã`Ç"Í°
‹¬i†³Ø°#k.

M
"³<£ zã`ÇO (ii) #°O_ Jxß LOQ®~Œ° |¶@°Á J#°#k
‹¬i†³Ø°#k

‹¬i†³Ø°#k.

E
– I …è^¥ – II …Õ
­íP
Hê=ô# x~¡â†Ç°O x~¡â†Ç°O U^ËXH›\ ÷

R댺 R댺
†Öƒ

D
9
†Öƒ­íP ‹¬i†³Ø°#k.

59
9. (e) (…è^¥)

CA X‰ØÄ¥

95
ä_Ï>P
ä_Ï>P
(a)

50
R¯_±Ï ‚±
14.

98
(i) (ii)
A
:8
"³<£zã`ÇO (ii) #°O_ x~¡â†Ç°O –I ‹¬i†³Ø°#k
E

(i) – II R¯±`©‚±
h

"³<£zã`ÇO #°O_ x~¡â†Ç°O ‹¬i†³Ø°#k


.P
IC

Hêx D ï~O_O\ ÷…Õ U^Ë XH›>è ‹¬i†³Ø°#k. Hê=ô#


"³<£ zã`ÇO #°O_ x~¡†
â °
Ç O – I ‹¬i†³°Ø #k Hêx x~¡†
â °
Ç O
AD

x~¡â†Ç°O – I …è^¥ x~¡â†Ç°O – II …Õ U^Ë XH›\ ÷


– II ‹¬i†³Ø°#k Hê^ΰ.
AB
O

‹¬i†³Ø°#k. KŠØß
KŠØß
ER
CH

ˆ‘ (e)
¢Š• 15. (…è^¥)

¨‚
YD

„í½P圾 ˆ‘•¢Š¨‚ R¯±ä‚± R¯±ä‚±


„í½P圾 V$±R¯±‚±
10. (d) (…è^¥) V$±R¯±‚±
,H

Iœ‰íŠ¨‚ Iœ‰íŠ¨‚ (i) (ii)


T
AM

(i) (ii) "³<£zã`ÇO (i) #°O_ J‰×Þ XH› U#°Q®° J#°#k `DŽ¬C.
H
M

"³<£zã`ÇO (ii) #°O_ J‰×Þ XH› U#°Q®° Hê^ΰ J#°#k


(i) #°O_ – I ‹¬iHê^Îx
AM

"³<£ zã`ÇO x~¡†


â °
Ç O `³°‹¬°O
ë k.
IG

`DŽ¬C.

"³<£zã`ÇO (i) =°i†Çò (ii)  #°O_ x~¡â†Ç°O – II


KH

Hêx ï~O_O\ ÷…Õ U^Ë XH›\ ÷ x[O J"ŒÞe.

‹¬iHê^Îx `³°‹¬°ëOk.
– I – II
R

Hê=ô# x~¡â†Ç°O …è^¥ x~¡â†Ç°O …Õ U^Ë

¢LÈóaȂ± ¢LÈóaȂ± XH›\ ÷ ‹¬i†³Ø°#k.


LÈl#Î#¢± LÈl#Î#¢±
ŒÕaIÌIœ!{‚±Iœ‚± …­õ>õ߂±
11. (a) (…è^¥)
ŒÕaIÌIœ!{‚±Iœ‚±
16. (b) (…è^¥)

„èlPIœ‚±
†í‡È± ‚± †í‡È± ‚± …­õ>õ߂± „èlPIœ‚±
(i) (ii) (i) (ii)

"³<£ zã`Œ #°O_ x~¡â†Ç°O –I ‹¬i†³Ø°#k. Hêx "³<£ "³<£zã`ÇO (i) #°O_ Hùxß ‹Ô`ŒHËH›z°H›° „¬ô=ôށ°

zã`ÇO (i) #°O_ x~¡ ⠆ Ç ° O – II ‹¬ i †³ Ø ° #k Hê^Î x


Jx H›zó`ÇOQê K³„¬æ…è=ò.

`³°‹¬°ëOk.
"³<£zã`ÇO (ii) #°O_ Hùxß ‹Ô`ŒHËH›z°H›° „¬ô=ôށ°

J=Þ=KÇ°ó J#°#k ‹¬i†³Ø°#k.

REASONING ABILITY (LOGICAL) ○ ○ ○ ○ ○ ○ ○ ○


191
○ ○ ○ ○ ○ ○ ○ ○ ○ ○ ○ ○ ○ ○ ○ ○ ○ ○ ○
Study Material
«‘P`Ɇ™Ÿ XH› `ǁ°„¬ô =°i†Çò H÷\ ÷H©
Iø]øIª
L´…­D‚±
Jx ‹¬æ+¬“OQê `³°‹¬°ë#ßk.
aȂ±…­õ
17. (e) (…è^¥)
«‘P`Ɇ™Ÿ `É=¤‚…º
Hê=ô# x~¡â†Ç°O – II `DŽ¬C.
}·A±Â
}·A±Â
L´…­D‚± 19. (d) Wzó# ㄬH@
› #…Õ =°^¼
Î „¬^O
Î …è^°
Î . Hê=ô# x~¡†
â ¶
Ç °

K³°Áƒì@° Hê=ô.
"³<£ zã`ÇO (i) #°O_, Hùxß ªO_ȅ˜ž |¶@°Á J#°#k

…­Œ­±…­õ
‹¬i†³Ø°#k Hê^ΰ
i‚P
(ii) #°O_ $‚±…­õ
"³<£ zã`ÇO U ªO_ȅ˜ |¶@° Hê^ΰ J#°#k
Kià
‹¬i†³Ø°#k Hê^ΰ.

Hêx ~
ï O_È° "³<£ zã`Œ…Õ x~¡†
â ¶
Ç  ㄬHê~¡O U^ËXH›\ ÷
20. (d) Wzó# x~¡â†Ç¶…Õ =°^Î¼„¬^ÎO Qê_^΁° q=iO„¬

Y
x[O J"ŒÞe.
|_ȅè^ΰ. Hê=ô# ï~O_È° x~¡â†Ç¶° ‹¬ï~á#q Hê=ô.

(I) (II)

M
Hê=ô# x~¡â†Ç°O …è^¥ x~¡â†Ç°O …Õ U^Ë

V$±R¯±‚±
XH›\ ÷ ‹¬i†³Ø°#k.

$‚±…­õ

E
18. (a) "³<£ zã`ÇO #°O_, _È<
£ „¹ XH› H÷\ H
÷ © J#°#k ‹¬i†³°
Ø #k. Rë`Ž_ς± Rë`Ž_ς±
(or)

D
9
Hê=ô# x~¡â†Ç°O –I ‹¬i†³Ø°#k $‚±…­õ V$±R¯±‚±

59
CA
95
"³<£ zã`ÇO #°O_ _È<£„¹ J#°#k

123456789012345678901234567890121234567890123456789012345678901212
12345678901234567890123456789012123456789012345678901234567890121234567890123456789012345678901212345678901234567890123456789012123
123456789012345678901234567890121234567890123456789012345678901212345678901234567890123
12345678901234567890123456789012123456789012345678901234567890121234567890123456789012345678901212345678901234567890123
1234567890123456789012345678901212345678901234567890123456789012123456789012345678901234567890121234567890123
1234567890123456789012345678901212345678901234567890123456789012123456789012345678901234567890121234

50
12345678901234567890123456789012123456789012345678901234567890121234567890123456789012345678901212345678901234567890123456789012123
123456789012345678901234567890121234567890123456789012345678901212345678901234567890123
123456789012345678901234567890121234567890123456789012345678901212
12345678901234567890123456789012123456789012345678901234567890121234567890123456789012345678901212345678901234567890123
1234567890123456789012345678901212345678901234567890123456789012123456789012345678901234567890121234567890123
1234567890123456789012345678901212345678901234567890123456789012123456789012345678901234567890121234
12345678901234567890123456789012123456789012345678901234567890121234567890123456789012345678901212345678901234567890123456789012123
123456789012345678901234567890121234567890123456789012345678901212345678901234567890123
12345678901234567890123456789012123456789012345678901234567890121234567890123456789012345678901212345678901234567890123
1234567890123456789012345678901212345678901234567890123456789012123456789012345678901234567890121234567890123
1234567890123456789012345678901212345678901234567890123456789012123456789012345678901234567890121234
123456789012345678901234567890121234567890123456789012345678901212
PREVIOUS BITS
12345678901234567890123456789012123456789012345678901234567890121234567890123456789012345678901212345678901234567890123456789012123
123456789012345678901234567890121234567890123456789012345678901212345678901234567890123
12345678901234567890123456789012123456789012345678901234567890121234567890123456789012345678901212345678901234567890123
1234567890123456789012345678901212345678901234567890123456789012123456789012345678901234567890121234567890123
1234567890123456789012345678901212345678901234567890123456789012123456789012345678901234567890121234
12345678901234567890123456789012123456789012345678901234567890121234567890123456789012345678901212345678901234567890123456789012123
123456789012345678901234567890121234567890123456789012345678901212345678901234567890123
123456789012345678901234567890121234567890123456789012345678901212
98
A
12345678901234567890123456789012123456789012345678901234567890121234567890123456789012345678901212345678901234567890123
1234567890123456789012345678901212345678901234567890123456789012123456789012345678901234567890121234567890123
1234567890123456789012345678901212345678901234567890123456789012123456789012345678901234567890121234
12345678901234567890123456789012123456789012345678901234567890121234567890123456789012345678901212345678901234567890123456789012123
123456789012345678901234567890121234567890123456789012345678901212345678901234567890123
12345678901234567890123456789012123456789012345678901234567890121234567890123456789012345678901212345678901234567890123
1234567890123456789012345678901212345678901234567890123456789012123456789012345678901234567890121234567890123
1234567890123456789012345678901212345678901234567890123456789012123456789012345678901234567890121234
123456789012345678901234567890121234567890123456789012345678901212
12345678901234567890123456789012123456789012345678901234567890121234567890123456789012345678901212345678901234567890123456789012123
123456789012345678901234567890121234567890123456789012345678901212345678901234567890123
1234567890123456789012345678901212345678901234567890123456789012123456789012345678901234567890121234567890123
1234567890123456789012345678901212345678901234567890123456789012123456789012345678901234567890121234
12345678901234567890123456789012123456789012345678901234567890121234567890123456789012345678901212345678901234567890123456789012123
123456789012345678901234567890121234567890123456789012345678901212345678901234567890123
:8
+ 1 #°O_ 3 =~¡ä›½ Q® ㄬ‰×߁䛽, ãH÷Ok ㄬ=KÇ<Œ x~ŒÖ~¡}° :
E

I) 'C' 'A'
h

Hùxß ° ° J=ô`Œ~ò.

P^¥~¡OQê ‹¬i†³Ø°# x~Œœ~¡}#° ZO„²H› K͆ǰO_.


.P
IC

II) Hùxß 'B' ° 'C' ° J=ô`Œ~ò.


AD

(A.P.Constable Prelims -2016) 1) I =¶ã`Ç"Í° ‹¬i†³Ø°#k

1. ㄬ=KÇ#=ò° : 'A' 'B'


AB
O

1) Hùxß ° ° J=ô`Œ~ò.
2) II =¶ã`Ç"Í° ‹¬i†³Ø°#q

'B' 'C'
ER

2) Jxß ° ° J=ô`Œ~ò.
I II
CH

3) …è^¥ ‹¬i†³Ø°#k

3) Hùxß 'C' °, 'D' ° J=ô`Œ~ò.


I II
YD

4) =°i†Çò ‹¬i†³Ø°#q

x~ŒÖ~¡}° : I) Hùxß 'A' ° 'C' ° J=ô`Œ~ò.


4. ㄬH›@#° : (S.I. - '07)
,H

1) Hùxß Qê_^΁° U#°Q®°°

II) Hùxß 'B' ° 'D' ° J=ô`Œ~ò.


T

2) Hùxß U#°Q®°° „²°Á°.


AM

I
x~¡â†Ç¶° :
1) =¶ã`Ç"Í° ‹¬i†³Ø°#k

I)
H

Hùxß „²°Á° Qê_^΁°


II
M

2) =¶ã`Ç"Í° ‹¬i†³Ø°#q

II)
AM

Hùxß Qê_^΁° „²°Á°


I II
IG

3) …è^¥ ‹¬i†³Ø°#k

III) Hùxß U#°Q®°° Qê_^΁°


I II
KH

4) =°i†Çò ‹¬i†³Ø°#q

IV)
2. ㄬ=KÇ#=ò° :
Hùxß „²°Á° U#°Q®°°
'P' 'Q'
R

1) Jxß ° ° J=ô`Œ~ò.

1) Jhß ‹¬i†³Ø°#q
2) 'Q' ° Un 'R' Hê=ô.

2) II & III =¶ã`Ç"Í° ‹¬i†³Ø°#q


3) Hùxß 'R' °, 'S' ° J=ô`Œ~ò.

x~ŒÖ~¡}° : I) 'P' ° Un 'R' Hê=ô.


3) III & IV =¶ã`Ç"Í° ‹¬i†³Ø°#q

II) Hùxß 'S' ° 'Q' ° Hê=ô.


4) Un ‹¬i†³Ø°#k Hê^ΰ

1) I =¶ã`Ç"Í° ‹¬i†³Ø°#k
5. ㄬH›@#° : 1) Jxß Ì„#°ß° ̄xžˆ×ÃÁ (S.I. - 2007)

2) II =¶ã`Ç"Í° ‹¬i†³Ø°#q 2) U ̄xž¶ HËu Hê^ΰ

3) I …è^¥ II ‹¬i†³Ø°#k x~¡â†Ç¶° : I) U ̄#°ß HËu Hê^ΰ

4) I =°i†Çò II ‹¬i†³Ø°#q II) Hùxß Ì„#°ß° HË`Ç°°

3. ㄬ=KÇ#=ò° : 1) Jxß 'A' ° 'B' ° J=ô`Œ~ò. III) Jxß HË`Ç°° ̄#°ß°

2) Jxß 'C' ° 'B' ° J=ô`Œ~ò.


IV) Hùxß HË`Ç°° ̄#°ß°

REASONING ABILITY (LOGICAL) ○ ○ ○ ○ ○ ○ ○ ○


192
○ ○ ○ ○ ○ ○ ○ ○ ○ ○ ○ ○ ○ ○ ○ ○ ○ ○ ○
Study Material
1) I & III =¶ã`Ç"Í° ‹¬i†³Ø°#q 9. ㄬH›@#° : 1) H›=ò° Jx߆Çò „¬ô‹¬ëHꁰ

2) II & III =¶ã`Ç"Í° ‹¬i†³Ø°#q. (S.I. - 2012)

3) I =¶ã`Ç"Í° ‹¬i†³Ø°#k 2) „¬ô‹¬ëH›=òx߆Çò "Í°*ì|Á°

4) Un ‹¬i†³Ø°#k Hê^ΰ x~¡â†Ç¶° : I) H›=ò° Jx߆Çò "Í°*ì|Á°

6. ㄬH›@#° : 1) Hùxß ‹Ôª° ‡h†Ç¶° II) "Í°*ì|Á° Jx߆Çò H›=ò°

2) Jxß ‡h†Ç¶° H›„¬C° III) „¬ô‹¬ëH›=ò° Jx߆Çò H›=ò°

(Commincation S.I. - 2013) IV) Hùxß "Í°*ì|Á° =¶ã`Ç"°


Í „¬ô‹¬H
ë ꁰ

x~¡â†Ç¶° : I) Hùxß ‹Ôª° H›„¬C° 1) I & IV =¶ã`Ç"Í° ‹¬i†³Ø°#q

II) Hùxß H›„¬C° ‡h†Ç¶° 2) II & III =¶ã`Ç"Í° ‹¬i†³Ø°#k

III) I & III

Y
Jxß ‡h†Ç¶° ‹Ôª° 3) =¶ã`Ç"Í° ‹¬i†³Ø°#q

IV) Jxß H›„¬C° ‡h†Ç¶° 4) Jhß ‹¬i†³Ø°#"Í

M
1) I & II =¶ã`Ç"Í° ‹¬i†³Ø°#q 10. ㄬH›@#° : 1) Hùxß ƒìÁOïH@°Á „¬~¡^¥°.

E
2) II & III =¶ã`Ç"Í° ‹¬i†³Ø°#q 2) U „¬~¡^¥ 䛽sóHê^ΰ

D
9
3) II & IV =¶ã`Ç"Í° ‹¬i†³Ø°#q (Jr. Stenographers, Typists in A.P. Ministe-

59
III & IV rial & A.P. Secretarial Sub Services - 2012)
4) =¶ã`Ç"Í° ‹¬i†³Ø°#q

CA
95
7. ㄬH›@#° : 1) Jxß q=¶<Œ° ï~áˆ×ä x~¡â†Ç¶° : I) XH›ø ƒìÁOïH\˜ 䛀_¨ „¬~¡^¥ Hê^ΰ

50
(S.I. - 2012) II)

98
2) Hùxß ï~áˆ×ÃÁ 䛽só° Jxß ƒìÁOïH@°Á 䛽só°
A
x~¡â†Ç¶° : I) III)
:8
Hùxß q=¶<Œ° 䛽só° Hùxß „¬~¡^¥° ƒìÁOïH@°Á
E

II) IV)
h

Hùxß ä›½só° ï~áˆ×ä „¬~¡^¥° ƒìÁOïH@°Á Hê^ΰ


.P
IC

III) Jxß ä›½só° ï~áˆ×ä 1) I & III =¶ã`Ç"Í° ‹¬i†³Ø°#q


AD

IV) Hùxß ä›½só° q=¶<Œ° 2) I …è^¥ IV ‹¬i†³Ø°#q


AB
O

1) I & III =¶ã`Ç"Í° ‹¬i†³Ø°#q 3) II & IV =¶ã`Ç"Í° ‹¬i†³Ø°#q


ER
CH

2) III & IV =¶ã`Ç"Í° ‹¬i†³Ø°#q 4) III =¶ã`Ç"Í° ‹¬i†³Ø°#k

11. ㄬH›@#° :
YD

3) III =¶ã`Ç"Í° ‹¬i†³Ø°#k 1) Hùxß Hê~¡°Á ‹¬¶ø@~¡°Á


,H

4) II =¶ã`Ç"Í° ‹¬i†³Ø°#k 2) U ‹¬¶ø@~¡¶ P\Õ Hê^ΰ.


T

8. ㄬH›@#° :
AM

1) Jxß Hù=°à° „¬ô+¬æ=ò° (S.I.-'07) (Jr. Stenographers, Typists in A.P. Ministe-


H
M

2) Jxß „¬ô+¬æ=ò° P䛽° rial & A.P. Secretarial Sub Serivices - 2012)
AM

x~¡â†Ç¶° : x~¡â†Ç¶° :
IG

I) Jxß Hù=°à° P䛽° I) Hùxß Hê~¡°Á P\Ձ° Hê=ô


KH

II) Jxß P䛽° Hù=°à° II) Jxß P\Ձ° Hê~¡°Á


R

III) Jxß „¬ô+¬æ=ò° Hù=°à° III) Hùxß P\Ձ° Hê~¡°Á

IV) Jxß P䛽° „¬ô+¬æ=ò° IV) Hùxß ‹¬¶ø@~¡°Á Hê~¡°Á

1) II & III =¶ã`Ç"Í° ‹¬i†³Ø°#q 1) II & III =¶ã`Ç"Í° ‹¬i†³Ø°#q

2) I & IV =¶ã`Ç"Í° ‹¬i†³Ø°#q 2) I =°i†Çò II =¶ã`Ç"Í° ‹¬i†³Ø°#q

3) I =¶ã`Ç"Í° ‹¬i†³Ø°#k 3) I & III =¶ã`Ç"Í° ‹¬i†³Ø°#q.

4) II & IV =¶ã`Ç"Í° ‹¬i†³Ø°#q 4) IV =¶ã`Ç"Í° ‹¬i†³Ø°#q.

REASONING ABILITY (LOGICAL) ○ ○ ○ ○ ○ ○ ○ ○


193
○ ○ ○ ○ ○ ○ ○ ○ ○ ○ ○ ○ ○ ○ ○ ○ ○ ○ ○
Study Material
12345678901234567890123456789012123456789012345678901234567890121234567890123456789012345678901212345678901234567890123
1234567890123456789012 123456789012345678901234

q=~¡}ì`ÇàH› ["Œ|°°
12345678901234567890123456789012123456789012345678901234567890121234567890123456789012345678901212345678901234567890123
1234567890123456789012 123456789012345678901234
12345678901234567890123456789012123456789012345678901234567890121234567890123456789012345678901212345678901234567890123
1234567890123456789012 123456789012345678901234
12345678901234567890123456789012123456789012345678901234567890121234567890123456789012345678901212345678901234567890123
1234567890123456789012 123456789012345678901234
1234567890123456789012
1234567890123456789012 PREVIOUS BITS :
12345678901234567890123456789012123456789012345678901234567890121234567890123456789012345678901212345678901234567890123
123456789012345678901234
12345678901234567890123456789012123456789012345678901234567890121234567890123456789012345678901212345678901234567890123
123456789012345678901234
12345678901234567890123456789012123456789012345678901234567890121234567890123456789012345678901212345678901234567890123
1234567890123456789012 123456789012345678901234

$ %
1. (1) " #
#
6. (1)
ŒÕ«‘‚± ˆ‘i‡È·‚±
*
**

I, II  #°O_
Iœ…­D‚±
Hùxß 'A' ° 'C' ° J=ô`Œ~ò.
"³<z
£ ã`ÇO#° Q®=°xOKÇQê x~¡†
â °
Ç O –I =°i†Çò x~¡†
â °
Ç O

Hê=ô# nxH÷ ‹¬i†³Ø°# ‹¬=¶^¥#O (1). – II =¶ã`Ç"Í° ‹¬i†³Ø°#=x ‹¬æ+¬“OQê `³°‹¬°ëOk.

2 3 ðâ‰ØÄ¥

Y
2. (1)
1 4 2 3 3 4

M
(4)
7.
r>·=‚± 圾tô‚±
*
**
***

E
I, II  #°O_

D
9
'P' 'R' – II

59
° Un Hê=ô
"³ < £ zã`Ç O #°O_ ‹¬ æ +¬ “ O x~¡ ⠆ Ç ° O =¶ã`Ç " Í °

'S' 'Q' (Converse)


CA
95
Hùxß ° ° Hê=ô. ‹¬i†³Ø°#^Îx `³°‹¬°ëOk.

Q圾‚±

50
Hê=ô# nxH÷ ‹¬i†³Ø°# ‹¬=¶^¥#O (4).

# …­õ,­ç>ó‚±
98
A
(3)
#
8.
:8
Iú>±á‚±
E

(2)
3.
" $ " $
h
.P
IC
AD

"³<£ zã`ÇO #°O_ x~¡â†Ç°O –I =¶ã`Ç"Í° ‹¬i†³Ø°#^Îx


AB
O

‹¬æ+¬“OQê `³°‹¬°ëOk.
Hùxß 'C' ° 'A' ° J=ô`Œ~ò.
ER
CH

Hùxß 'B' ° 'C' ° J=ô`Œ~ò. #α+í}‚‚±


YD

Hê=ô# nxH÷ ‹¬i†³Ø°# ‹¬=¶^¥#O (4). 9. (1) …­õŒ­Iì 낱


,H

Iœ‚>ó‚±
T

4. (3)
AM
H

…³‚±Â‚±
M

"³<£ zã`ÇO #°O_ x~¡â†Ç°O – I =°i†Çò x~¡â†Ç°O –IV


AM
IG

Rë`Ž_‚
Ï ± V$±R¯±‚± =¶ã`Ç"Í° ‹¬ï~á#=x ‹¬æ+¬“OQê `³°‹¬°ëOk.
KH

™
– I =°i†Çò II `DŽ¬C ðI] …­¢_¦
„íPÂ
"³<£ zã`ÇO #°O_ x~¡â†Ç¶° Jx
R

™
– III =°i†Çò ðI] …­¢_¦
„íÂP 圾tô
‹¬æ+¬O
“ Qê `³°‹¬°O
ë k. =°i†Çò x~¡†
â ¶
Ç °

10. (4) (…è^¥)

IV ° ‹¬i†³Ø°#=x ‹¬æ+¬“O. 圾tô

ͅyŸ‰ØÄ¥
(i) (ii)
(3)
– III
5.
"³<£zã`Œ #°O_ ‹¬æ+¬“OQê x~¡â†Ç°O =¶ã`Ç"Í°

ͅ$±à‚±
IÌv
‹¬i†³Ø°#k.

I뢱   I뢱  
11. (4)
Œ­·ùA
Q]Ö (…è^¥) Œ­·ùA
–I
"³<£ zã`ÇO #°O_ x~¡â†Ç°O =¶ã`Ç"Í° ‹¬i†³Ø°#^Îx
Q]Ö
‹¬æ+¬“OQê `³°‹¬°ëOk.
(i) (ii)
"³<£ zã`Œ #°O_ x~¡â†Ç°O – IV° ‹¬i†³Ø°#k.

REASONING ABILITY (LOGICAL) ○ ○ ○ ○ ○ ○ ○ ○


194
○ ○ ○ ○ ○ ○ ○ ○ ○ ○ ○ ○ ○ ○ ○ ○ ○ ○ ○
Study Material
1 ã‰õ}°°
ã‰õ}°°
(Series)
123456789012345678901234567890121234567890123456789012345678901212
12345678901234567890123456789012123456789012345678901234567890121234567890123456789012345678901212345678901234567890123
1234567890123456789012345678901212345678901234567890123456789012123456789012345678901234567890121234567890123
1234567890123456789012345678901212345678901234567890123456789012123456789012345678901234567890121234
123456789012345678901234567890121234567890123456789012345678901212
12345678901234567890123456789012123456789012345678901234567890121234567890123456789012345678901212345678901234567890123456789012123
1234567890123456789012345678901212345678901234567890123456789012123456789012345678901234567890
12345678901234567890123456789012123456789012345678901234567890121234567890123456789012345678901212345678901234567890123
1234567890123456789012345678901212345678901234567890123456789012123456789012345678901234567890121234567890123
1234567890123456789012345678901212345678901234567890123456789012123456789012345678901234567890121234
12345678901234567890123456789012123456789012345678901234567890121234567890123456789012345678901212345678901234567890123456789012123
1234567890123456789012345678901212345678901234567890123456789012123456789012345678901234567890
12345678901234567890123456789012123456789012345678901234567890121234567890123456789012345678901212345678901234567890123
1234567890123456789012345678901212345678901234567890123456789012123456789012345678901234567890121234567890123
1234567890123456789012345678901212345678901234567890123456789012123456789012345678901234567890121234
123456789012345678901234567890121234567890123456789012345678901212
12345678901234567890123456789012123456789012345678901234567890121234567890123456789012345678901212345678901234567890123456789012123
1234567890123456789012345678901212345678901234567890123456789012123456789012345678901234567890
PRACTICE TEST
12345678901234567890123456789012123456789012345678901234567890121234567890123456789012345678901212345678901234567890123
1234567890123456789012345678901212345678901234567890123456789012123456789012345678901234567890121234567890123
1234567890123456789012345678901212345678901234567890123456789012123456789012345678901234567890121234
12345678901234567890123456789012123456789012345678901234567890121234567890123456789012345678901212345678901234567890123456789012123
1234567890123456789012345678901212345678901234567890123456789012123456789012345678901234567890
123456789012345678901234567890121234567890123456789012345678901212
12345678901234567890123456789012123456789012345678901234567890121234567890123456789012345678901212345678901234567890123
1234567890123456789012345678901212345678901234567890123456789012123456789012345678901234567890121234567890123
1234567890123456789012345678901212345678901234567890123456789012123456789012345678901234567890121234
12345678901234567890123456789012123456789012345678901234567890121234567890123456789012345678901212345678901234567890123456789012123
1234567890123456789012345678901212345678901234567890123456789012123456789012345678901234567890
123456789012345678901234567890121234567890123456789012345678901212
12345678901234567890123456789012123456789012345678901234567890121234567890123456789012345678901212345678901234567890123
1234567890123456789012345678901212345678901234567890123456789012123456789012345678901234567890121234567890123
1234567890123456789012345678901212345678901234567890123456789012123456789012345678901234567890121234
12345678901234567890123456789012123456789012345678901234567890121234567890123456789012345678901212345678901234567890123456789012123
1234567890123456789012345678901212345678901234567890123456789012123456789012345678901234567890
12345678901234567890123456789012123456789012345678901234567890121234567890123456789012345678901212345678901234567890123456789012123
1234567890123456789012345678901212345678901234567890123456789012123456789012345678901234567890
12345678901234567890123456789012123456789012345678901234567890121234567890123456789012345678901212345678901234567890123
1234567890123456789012345678901212345678901234567890123456789012123456789012345678901234567890121234567890123
1234567890123456789012345678901212345678901234567890123456789012123456789012345678901234567890121234

‹¬¶KÇ# (1-35) : D ãH÷Ok ã‰õ}°…Õ, ㄬ‰×ß „¬\ì…Õ <Œ¾= ƒç=°à `Ç~ŒÞ`Ç Hù#ªyO„¬ôQê =KÇ°ó ƒç=°à#° ㄬH›ø<Í W=Þ|_#
‹¬=¶^¥# „¬\ì #°O_ Q®°iëOKÇ°=ò?

Y
ㄬ‰×ß „¬@=ò° ‹¬=¶^¥# „¬@=ò°

M
1.

E
D
9
59
(a) (b) (c) (d) (1) (2) (3) (4) (5)

CA
95
2.

50
98
A
(a) (b) (c) (d) (1) (2) (3) (4) (5)
:8
E
h

3.
.P
IC
AD

(a) (b) (c) (d) (1) (2) (3) (4) (5)


AB
O
ER
CH

4.
YD

(a) (b) (c) (d) (1) (2) (3) (4) (5)


,H
T
AM

5.
H
M
AM
IG

(a) (b) (c) (d) (1) (2) (3) (4) (5)


KH
R

6.

(a) (b) (c) (d) (1) (2) (3) (4) (5)

7.

(a) (b) (c) (d) (1) (2) (3) (4) (5)

8.

(a) (b) (c) (d) (1) (2) (3) (4) (5)

REASONING ABILITY (NON-VERBAL) ○ ○ ○ ○ ○ ○


195
○ ○ ○ ○ ○ ○ ○ ○ ○ ○ ○ ○ ○ ○ ○ ○ ○ ○ ○
Study Material
9.

(a) (b) (c) (d) (1) (2) (3) (4) (5)

10.

(a) (b) (c) (d) (1) (2) (3) (4) (5)

‹¬¶KÇ# (11-35) : D ãH÷Ok ‹¬=°‹¬¼ zã`Œ…Õx '?Ñ Q®°~¡°ë ªÖ#=ò…Õ =òO^ΰ '3Ñ „¬\ìä›½ Hù#ªyO„¬ôQê =KÇ°ó „¬\ìxß ã„¬H›ø<Í
W=Þ|_# ‹¬=¶^¥# „¬\ì #°O_ Q®°iëOKÇO_?

Y
11. ?

M
(1) (2) (3) (4)

E
D
9
12.

59
?
CA
95
(1) (2) (3) (4)

50
98
A
13.
?
:8
E
h

(1) (2) (3) (4)


.P
IC
AD

14. ?
AB
O
ER
CH

(1) (2) (3) (4)


YD

15.
?
,H
T
AM

(1) (2) (3) (4)


H
M
AM
IG

16.
?
KH
R

(1) (2) (3) (4)

17.
?
(1) (2) (3) (4)

18.
?
(1) (2) (3) (4)

REASONING ABILITY (NON-VERBAL) ○ ○ ○ ○ ○ ○


196
○ ○ ○ ○ ○ ○ ○ ○ ○ ○ ○ ○ ○ ○ ○ ○ ○ ○ ○
Study Material
19.
?
(1) (2) (3) (4)

20.
?
(1) (2) (3) (4)

Y
21. ?

M
(1) (2) (3) (4)

E
D
9
59
22. ?
CA
95
50
(1) (2) (3) (4)

98
A
:8
23.
E

?
h
.P
IC
AD

(1) (2) (3) (4)


AB
O
ER
CH

24.
?
YD
,H

(1) (2) (3) (4)


T
AM
H
M

25. ?
AM
IG
KH

(1) (2) (3) (4)


R

26.
?
(1) (2) (3) (4)

27.
?
(1) (2) (3) (4)

REASONING ABILITY (NON-VERBAL) ○ ○ ○ ○ ○ ○


197
○ ○ ○ ○ ○ ○ ○ ○ ○ ○ ○ ○ ○ ○ ○ ○ ○ ○ ○
Study Material
28.

(1) (2) (3) (4)

29.
?
(1) (2) (3) (4)

30.
?

Y
M
(1) (2) (3) (4)

E
31. ?

D
9
59
CA
95
(1) (2) (3) (4)

50
98
A
32.
?
:8
E
h
.P

(1) (2) (3) (4)


IC
AD
AB
O

33.
?
ER
CH
YD

(1) (2) (3) (4)


,H
T
AM

34.
?
H
M
AM
IG

(1) (2) (3) (4)


KH
R

35.
?
(1) (2) (3) (4)

REASONING ABILITY (NON-VERBAL) ○ ○ ○ ○ ○ ○


198
○ ○ ○ ○ ○ ○ ○ ○ ○ ○ ○ ○ ○ ○ ○ ○ ○ ○ ○
Study Material
1234567890123456789012
12345678901234567890123456789012123456789012345678901234567890121234567890123456789012345678901212345678901234567890123
12345678901234567890123
12345678901234567890123456789012123456789012345678901234567890121234567890123456789012345678901212345678901234567890123
1234567890123456789012 12345678901234567890123
12345678901234567890123456789012123456789012345678901234567890121234567890123456789012345678901212345678901234567890123
1234567890123456789012
1234567890123456789012
q=~¡}ì`ÇàH› ["Œ|°°
12345678901234567890123
12345678901234567890123456789012123456789012345678901234567890121234567890123456789012345678901212345678901234567890123
PRACTICE TEST :
12345678901234567890123
12345678901234567890123456789012123456789012345678901234567890121234567890123456789012345678901212345678901234567890123
1234567890123456789012
12345678901234567890123456789012123456789012345678901234567890121234567890123456789012345678901212345678901234567890123
1234567890123456789012
12345678901234567890123
12345678901234567890123
12345678901234567890123456789012123456789012345678901234567890121234567890123456789012345678901212345678901234567890123
1234567890123456789012 12345678901234567890123

1. (1) "³ò^Î\ ÷ „¬@O…Õx PHê~Œ° ~ï O_ȶ u~¡Q® ãu„²æ`Í U~¡æ_È°#k ‹¬=¼k‰×…Õ ハ=’ °}O K³O^ΰ`Ç°Ok. =$`Ç=ë ò J#°#k KÇ`°Ç ~¡ã‹¬
ï~O_È= ƒç=°à…Õx PHê~Œ° `Ç~ŒÞ`Ç "Œ\÷x ^ÎQ®¾~¡Qê KÍió boundary †³òH›ø =¼uö~H› cornerH÷ KÍ~¡°`Ç°Ok. nx
H›e„²`Í =ü_È= „¬@O…Õx PHê~¡O =KÇ°ó#°. =ü_È= ㄬHê~¡O KǶÀ‹ë ã‰õ}÷…Õ `Ç~ŒÞ`Ç =KÇ°ó ƒç=°à ‹¬=¶^¥#
„¬@O…Õx PHê~¡=ò°#° u~¡Q® ãu„²æ`Í (XH›^¥xHùH›\ ÷ „¬@=ò…Õx 4=k.
=¼uö~H›OQê) U~¡æ_È°#k <Œ¾= „¬@O…Õx PHê~¡O D 9. (2) Wzó# ƒç=°à…Õ 'pin' PHê~¡=ò KÇ`Ç°~¡ã‹¬ boundary
q^OÎ Qê „¬^uœÎ Hù#ªy‡é`Ƕ LO@°Ok. D „¬^uœÎ ㄬHê~¡O †³òH›ø ƒ’°[O…Õ ‹¬QêxH÷ J„¬‹¬=¼k‰×…Õ KÍ~¡°`Ç°Ok. ㄬu
KǶÀ‹ë ã‰õ}…÷ Õ `Ç~ŒÞ`Ç =KÇ°ó „¬@=ò ‹¬=¶^¥# „¬@=ò…Õx ̋“„¹…Õ =°i†Çò Wk 180° ‹¬=¼k‰×…Õ =°i†Çò 90°
"³ò^Î\ ÷k. Hê=ô# nxH÷ ‹¬i†³Ø°# ‹¬=¶^¥#„¬ô ƒç=°à (1). ‹¬=¼k‰×…Õ XH›^¥x `Ç~ŒÞ`Ç ãƒ’=°}O K³O^ΰ`Ç°Ok. ƒì}O
2. (3) Wzó# ã‰õ}÷…Õ ㄬu ï~O_È= ̋“„¹…Õ XöH q^Î"³°Ø# PH›$u Q®°~¡°ë =¶ã`ÇO ㄬű“„¹…Õ KÇ`Ç°~¡ã‹¬ boundary †³òH›ø

Y
„¬ô#~Œ=$`Ç=°=ô`Ç°Ok. ㄬuªi XH› xiœ+¬“"³°Ø# PHê~¡O ƒ°’ [O…Õ ‹¬QêxH÷ KÍ~°¡ `Ç°Ok. 135° ‹¬=¼k‰×…Õ nx ㄬHê~¡O
(ƒì}O Q®°~¡°,ë ƒç=°à ãH÷O^Î …ÿ<á )£ =¶ã`ÇO ^¥x ªÖ#O #°O_ KǶÀ‹ë ã‰õ}÷…Õ `Ç~ŒÞ`Ç =KÇ°ó ƒç=°à ‹¬=¶^¥# ƒç=°à…Õx

M
90° ‹¬=¼k‰×…Õ ハ’=°}O K³O^ΰ`Ç°Ok. D q^ÎOQê KǶÀ‹ë 2=k.

E
ã‰õ}÷…Õ `Ç~ŒÞ`Ç =KÇ°ó ƒç=°à ‹¬=¶^¥# „¬@=ò…Õx 10. (4) Wzó# JO‰§…Õx =òY¼"³°#Ø PHê~¡O (JO>è ƒç=°à KÇ°@°“
=ü_È=k. ㄬHø›  L#ß PHê~¡O) J#°#k ㄬu ̋“„… ¹ Õ 90° ‹¬=¼k‰×…Õ

D
9
59
3. (5) Wzó# ã‰õ}…÷ Õx „¬\쁅Õx PHê~Œeß „¬ijeÀ‹ë XHËø ̋„“ …¹ Õ ãƒ’=°}O K³O^ΰ`Ç°Ok. J…ìöQ ƒç=°à …Õx xiœ+¬“ PHê~Œ°
CA
95
XHËø ö~Mì YO_È=ò Z_È=°"³á„¬ô #°O_ =°i†Çò 䛽_ "³á„¬ô (O) =¶ã`ÇO XHËø ªÖ#=ò =òO^ΰ䛽 [~¡°Q®°`Œ~ò.
#°O_ XH›^¥x `Ç~ŒÞ`Ç XH›\ ÷ `ùyOKÇ |_È°`Ç°Ok. P ‹¬=¼k‰×…Õ nx ㄬHê~¡=ò KǶÀ‹ë ã‰õ}÷…Õ `Ç~¡°"Œ~ò =KÇ°ó

50
q^ÎOQê KǶÀ‹ë ã‰õ}÷…Õ `Ç~¡°"Œ`Ç =KÇ°ó ƒç=°à ‹¬=¶^¥# ƒç=°à ‹¬=¶^¥# „¬@=ò…Õx 4=k.
98
A
„¬@=ò…Õ '5Ñ=k. ‹¬=°‹¬¼ zã`Œ…ÕÁ 1,3 XöH…ì L<Œß~ò. 2= ‹¬=°‹¬¼
:8
11. (3)
E

4. „¬@=ò…Õx À+_£ K͆°Ç |_# ƒìQ®=ò XH›\ ,÷ ~ï O_È°, =ü_È°, zã`ÇO…ìQê<Í Mìm Q®_…Õ ƒç=°à LO_¨e. 3= ["Œ|°
h

(4)
.P

<Œ°Q®°..... ªÖ<Œ° =~¡°‹¬Qê J„¬‹¬=¼k‰×…Õ ハ’=°}O zã`ÇO ‹¬iJ~ò#k.


IC
AD

K³O^ΰ`Ç°Ok. nx ㄬHê~¡O KǶÀ‹ë ã‰õ}÷…Õ `Ç~ŒÞ`Ç =KÇ°ó 12. (1) ãuƒ°’ [O |†Ç°@ w`DžÕÁ ‹¬=¼k‰×…Õ =¶~¡°æ [~¡°Q®°`ËOk.
ƒç=°à ‹¬=¶^¥# „¬@=ò…Õx 4=k. ["Œ|° ƒç=°à…Õ 1=k Mìm Q®_…Õ ‹¬i‡é`ËOk.
AB
O

5. (4) Wzó# ƒç=°à…Õ ㄬu ~ï O_È= ̋“„…¹ Õ XöH~¡H"› °³ #Ø PHê~¡=ò 13. (2) ‹¬=°‹¬¼ zã`Œ…ÕÁ ‹¬=¼k‰×…Õ O, { Q®°~¡° ë ° XHùøH›ø =üä›½
ER
CH

„¬ô#~Œ=$`ÇëO J=ô`Ç°Ok. ㄬu ̋“„¹…Õ xiœ+¬“"³°Ø# PHê~¡=ò [~¡°Q®°`Ç°<Œß~ò. D H›Æ}쁰#ß ï~O_È= ["Œ|° ƒç=°à
YD

(‡~Ÿ“) J#°#k u~¡Q® ãu„¬æ |_È°`Ç°Ok. Mìm Q®_…Õ ‹¬i‡é`Ç°Ok.


,H

6. (3) Wzó# ƒç=°à…Õ ㄬu ̋“„¹…Õ KŒ„¬=ò (Arc) J#°#k 14. (4) ‹¬=°‹¬¼ zã`Œ…ÕÁ #ÁKÇ°H›ø XHùøH›ø Q®_H÷ XHùøH›ø\÷ Kù„¬C#
T

90° J„¬ ‹ ¬ = ¼k‰× … Õ ãƒ ’ = °}O K³ O k ^¥x KÇ ` Ç ° ~¡ 㠋¬ ̄~¡°Q®°`ËOk. <Œ°Q®° KÇ°H›ø°#ß 4= ["Œ|° zã`ÇO Mìm
AM

(boundary) †³òH›ø z=~¡Á䛽 KÍ~¡°`Ç°Ok. ‹¬=¼k‰×…Õ Q®_…Õ ‹¬i‡é`Ç°Ok.


H
M

=°i†Çò =$`ÇOë J#°#k J„¬‹=¬ ¼k‰×…Õ ㄬHø› <Í L#ß KÇ`°Ç ~¡ã‹¬ 15. ‹¬=°‹¬¼ zã`Œ…ÕÁ #ÁKÇ°H›ø, `³ÁKÇ°H›ø J„¬‹¬=¼k‰×…Õ
AM

(2)
IG

†³òH›ø ƒ°’ *ì =^Îäí ½› KÍ~°¡ `Ç°Ok. nx ㄬHê~¡O KǶÀ‹ë ã‰õ}…÷ Õ u~¡°Q®°`Ç°<Œß~ò. D H›} Æ ì°#ß 2= ["Œ|° zã`ÇO ‹¬=°‹¬¼
KH

`Ç~ŒÞ`Ç =KÇ°ó „¬@=ò ‹¬=¶^¥# „¬@=ò…Õx 3=k. zã`ÇO Mìm Q®_H÷ ‹¬i‡é`Ç°Ok.
R

7. (4) Wzó# ƒç=°à…Õ "³ò^Î\ ÷ ^¥x…Õ z=~¡Qê L#ß ''XH› 16. (2) 1= ‹¬=°ª¼„¬@O…Õx KÇ`°Ç ~¡ã‹¬O =ü L#ß w`ǁ° ~ï O_È=
KÇ°H›øÑÑx `ùyÀ‹ë =KÇ°ó#k. 2= ƒç=°à…Õx PHê~¡=ò ‹¬=°ª¼ „¬@O…Õ =¼uö~H› =üä›½ "³ˆì¤~ò. =ü_È=
=°i†Çò 2= ƒç=°à…Õx PHê~¡=ò…Õ 2 ~ö Mì YO_È=ò° ‹¬=°ª¼ „¬@O…Õ =°m¤ "³ò^Î\ ÷ „¬@O…Õ L#ß KË@°#öH
`ùyOKÇ|_`Í =KÇ°ó#k 3= ƒç=°à…Õx PHê~¡=ò 3= =KŒó~ò. Mìm ‹¬=°‹¬¼ zã`ÇO…Õ ï~O_È= ‹¬=°ª¼ zã`Ç"Í°
ƒç=°à…Õx 3 KÇ°H›ø° `ùyÀ‹ë U~¡æ_È°#k 4= ƒç=°à…Õx =‹¬°ëOk. JO>è 2= ["Œ|° zã`ÇO ‹¬ï~áOk.
PHê~¡=ò Hê=ô# ã‰õ}÷…Õ `Ç~¡°"Œ`Ç =KÇ°ó ƒç=°à…Õ '4Ñ 17. (3) H›`³ë~¡ ƒç=°à 90O ‹¬=¼k‰×Qê u~¡°Q®°`ËOk. 3= ["Œ|°
~ö MìYO_¨° `ùyOKÇ|_È`Œ~ò. nxß|\÷“ KǶÀ‹ë ‹¬=¶^¥# ƒç=°à Mìm ‹¬=°‹¬¼ zã`Œxß „¬îiOKÍ ƒç=°à.
„¬@=ò…Õx '4Ñ =k ã‰õ}÷…Õ `Ç~ŒÞ`Ç =KÇ°ó ƒç=°à. 18. (3) 1= ‹¬=°ª¼ zã`ÇO =°^Î¼…Õx { Q®°~¡°ë =~¡°‹¬Qê ï~O_È°,
8. (4) Wzó# ƒç=°à…Õ ㄬu ̋„“ …¹ Õ n~¡K É `Ç °Ç ~¡ã‹¬O J#°#k XH›\,÷ =ü_È° ‹¬=°ª¼ zã`Œ…ÕÁ þ Q®°~¡°ëQê, { Q®°~¡°ëQê =¶iOk.
~ï O_È°, =ü_È°, <Œ°Q®°.... ªÖ<Œ° (Mìm°) (ㄬu ªÖ#=ò <Œ°QË ‹¬=°ª¼ zã`ÇO…Õ =°~¡ { Q®°~¡°ë ~Œ"Œe. ‹¬=°ª¼
KÇ`Ç°~¡ã‹¬ boundary †³òH›ø ƒ’°[O…Õ ‹¬QêxH÷ ‹¬=¶#O) zã`ÇO…Õ KÇ`Ç°~¡ã‹¬O |†Ç°@=ô#ß =$`ÇëO, ãuƒ’°[O Q®°~¡°ë°

REASONING ABILITY (NON-VERBAL) ○ ○ ○ ○ ○ ○


199
○ ○ ○ ○ ○ ○ ○ ○ ○ ○ ○ ○ ○ ○ ○ ○ ○ ○ ○
Study Material
‹¬=¼k‰×…Õ XH› =ü#°O_ =¼uö~H› =üä›½ 2, 3 ‹¬=°ª¼ 26. (1) 27. (2)

zã`Œ…ÕÁ =¶~Œ~ò. Mìm ‹¬=°ª¼ zã`ÇO…Õ `³Á ãuƒ’°[O 28. (4) ƒç=°à J„¬‹¬=¼ k‰×…Õ 90O u~¡°Q®°`ËOk. 2= ƒç=°à…Õ
KÇ`°Ç ~¡ã‹¬O ̄ƒá ì Q®O Z_È=°ã„¬Hø› , #Á =$`ÇOë 䛽_ã„¬Hø› ~Œ"Œe. 䛽_KÍ~ò J…ìöQ LO_ Z_È=° KÍ~ò ãH÷O^Î䛽 =¶iOk.
W…ì L#ß ["Œ|° zã`ÇO (3) ‹¬ï~áOk. 4= ‹¬=°ª¼ zã`ÇO…Õ =°~¡ 䛽_K~Í ò J…ìöQ LO_ Z_È=°
19. (3) ‹¬=°ª¼ zã`Œ…ÕÁ ƒì}O Q®°~¡°ë =¼uö~H› k‰×…ÕÁH÷ "³ˆ×Ã`ËOk. KÍ~ò ãH÷O^ÎH÷ uiy`Í, 4= ["Œ|° ƒç=°à ‹¬=°ª¼ zã`ÇO…Õ
=$`ÇëO |†Ç°@ L#ß KÇ°H›ø 90O ‹¬=¼k‰×Qê u~¡°Q®°`ËOk. ‹¬i‡é`Ç°Ok.
3= ["Œ|° zã`ÇO Mìm ƒç=°à䛽 ‹¬ï~á#k. 29. (2) « Q®°~¡°ë XHùøH›ø\÷ Kù„¬C# ̄~¡°Q®°`Ƕ, ‹¬=°ª¼ zã`ÇO
20. (3) KÇ`Ç°~¡ã‹¬O 45O K͋¬¶ë u~¡°Q®°`ËOk. …Õ„¬e þ Q®°~¡°ë { <Œ°Q®° Hê"Œe.
Q®°~¡°Që ê, =°~¡ þ Q®°~¡°Që ê =¶iOk. Mìm ‹¬=°ª¼ zã`ÇO…Õ 30. (3) 1= ‹¬=°ª¼ zã`ÇO…Õ z#ß =$`ÇëO ï~O_³á, ̄^Îí =$`ÇëO…ÕH÷
=°~¡ '{Ñ Q®°~¡°ë ~Œ"Œe. Jk KÇ`°Ç ~¡ãªxH÷ ̄á"„á³ ô¬ LO_¨e. "³ˆì¤~ò. J…ìöQ 3= ‹¬=°ª¼ zã`ÇO…Õx |†Ç°\÷ z#ß
3= ["Œ|° zã`ÇO ‹¬=°ª¼ zã`ÇO…Õx Mìm…Õ ‹¬i‡é†Í° =$`ÇëO ï~O_³á, ̄^Îí =$`ÇëO …Õ„¬eH÷ "³o¤`Í, "³ò`ÇëO <Œ°Q®°
ƒç=°à. z#ß =$`Œë° =ôO\ì~ò.

Y
21. (4) ‹¬=°ª¼zã`Œ…ÕÁ 1,3 XöH q^ÎOQê L<Œß~ò. JO^ΰ=Á 31. (1) Jxß ‹¬=°ª¼ zã`Œ° =~¡°‹¬…Õ 90O HË}O`Ë ãƒ’=°}O

M
Mìm ‹¬=°ª¼zã`ÇO…Õ 2= ‹¬=°ª¼ zã`Ç"Í° ~Œ"Œe. J…ì K͋¬°ë<Œß~ò. Hê=ô# Mìm Q®_…Õ 1= ["Œ|° „¬@O
L#ßk 4= ["Œ|° zã`ÇO. ‹¬i‡é`Ç°Ok.

E
22. (2) ‹¬=°ª¼zã`Œ…ÕÁ 䛽_KÍu "³á„¬ô L#ß ƒì}쁰 XHùøH›ø\÷ 32. (3) ‹¬=°ª¼ zã`Œ° XH› w`Ç =°i†Çò z#ß =$`ÇëO ‹¬=¼k‰×°

D
9
Kù„¬C# XHùøH›ø Q®_…  Õ Ì„~¡°Q®°`Ç°<Œß~ò. Z_È=°KÍu "³„á ô¬ #ß XHùøH›ø\÷Qê ̄~¡°Q®°`Ƕ =‹¬°<ë Œß~ò. Hê=ô# D =¶~¡°æhß

59
ƒì}O XHùøH›ø Q®_…Õ =¼uö~H›k‰×䛽 =¶~¡°`ËOk. Mìm
CA '3Ñ= ["Œ|° „¬@O…Õ L<Œß~ò.

95
‹¬=°ª¼ zã`ÇO…Õ <Œ°Q®° ƒìQꁰ 䛽_KÍu "³á„¬ô LO_¨e. 33. (1) ‹¬=°ª¼ zã`Œ…Õ …Õ„¬ L#ß z#ß =$`ÇëO ‹¬=¼k‰×…Õ

50
Z_È=°KÍu ƒì}O ãH÷O^Î䛽 =¶~Œe. 2= ["Œ|° ƒç=°à =°i†Çò |†Ç°@ L#ß =ãH›ö~Y J„¬‹¬=¼k‰×…Õ `Ç~¡°"Œ`Ç
98
A
H›ï~䛽“. =ü#° KÍ~°¡ 䛽O@°<Œß~ò. Hê=ô# Mìm Q®_… Õ 1= ["Œ|°
:8
23. XHùøH›øQ®_…  Õ - =$`ÇOë , JO^ΰ…Õx Q®°~¡° ë ° 90O ‹¬=¼k‰×…Õ „¬@O ‹¬i‡é`Ç°Ok.
E

(2)
h

u~¡°Q®°`Ƕ =¶~¡°`Ç°<Œß~ò. 34. W=Þ|_# =~¡°‹¬ ‹¬=°ª¼ zã`Œ…Õ j~ŒÂ =^Îí L#ß ƒì}O
.P

(3)
IC

24. #Á=$`ÇëO J„¬‹¬=¼ k‰×…Õ u~¡°Q®°`Ƕ XHùøH›ø =üä›½ Q®°~¡°ë =°i†Çò =ãH›ö~Y ï~O_ȶ J„¬‹¬=¼k‰×…Õ `Ç~¡°"Œ`Ç
AD

(4)

XHùøH›ø ƒç=°à…Õ =¶~¡°`Ç°Ok. `³Á =$`ÇOë ‹¬=¼k‰×…Õ XHùøH›ø j~ŒÂxß KÍ~¡°ä›½O@°<Œß~ò. Hê=ô# MìmQê W=Þ|_# Q®_x
AB
O

=üä›½ =¶~¡°`ËOk. D =¶~¡°æ° [iy, Mìm ‹¬=°ª¼ '3Ñ „¬îi‹¬°ëOk.


ER
CH

zã`ÇO…Õ 4= ["Œ|° zã`ÇO ~Œ"Œe. 35. (2) W=Þ|_# =~¡°‹¬ ‹¬=°ª¼ zã`Œ…ÕÁx ãuƒ’°[=ò =°i†Çò
ƒì}O Q®°~¡°ë J#‹¬=¼k‰×…Õ 90O K͋¶¬ ë u~¡°Q®°`Ç°Ok. ƒì}O KÇ`Ç°~¡ã‹¬=ò ‹¬=¼k‰×…Õ ハ’=°}O K͋¬°ë<Œß~ò. 2= „¬@O
YD

25. (3)

=°^Î¼ w`ǁ° XHùøH›ø\÷ `Çy¾‡é`Ç°<Œß~ò. ‹¬iQê¾ MìmQê ~ò=Þ|_# Q®_x „¬îi‹¬°ëOk.


,H

123456789012345678901234567890121234567890123456789012345678901212
T
AM

123456789012345678901234567890121234567890123456789012345678901212
12345678901234567890123456789012123456789012345678901234567890121234567890123456789012345678901212345678901234567890123
1234567890123456789012345678901212345678901234567890123456789012123456789012345678901234567890121234567890123
1234567890123456789012345678901212345678901234567890123456789012123456789012345678901234567890121234
12345678901234567890123456789012123456789012345678901234567890121234567890123456789012345678901212345678901234567890123456789012123
123456789012345678901234567890121234567890123456789012345678901212345678901234567890123
12345678901234567890123456789012123456789012345678901234567890121234567890123456789012345678901212345678901234567890123456789012123
123456789012345678901234567890121234567890123456789012345678901212345678901234567890123
12345678901234567890123456789012123456789012345678901234567890121234567890123456789012345678901212345678901234567890123
1234567890123456789012345678901212345678901234567890123456789012123456789012345678901234567890121234567890123
1234567890123456789012345678901212345678901234567890123456789012123456789012345678901234567890121234
123456789012345678901234567890121234567890123456789012345678901212
12345678901234567890123456789012123456789012345678901234567890121234567890123456789012345678901212345678901234567890123456789012123
123456789012345678901234567890121234567890123456789012345678901212345678901234567890123
12345678901234567890123456789012123456789012345678901234567890121234567890123456789012345678901212345678901234567890123
1234567890123456789012345678901212345678901234567890123456789012123456789012345678901234567890121234567890123
1234567890123456789012345678901212345678901234567890123456789012123456789012345678901234567890121234
PREVIOUS BITS
123456789012345678901234567890121234567890123456789012345678901212
12345678901234567890123456789012123456789012345678901234567890121234567890123456789012345678901212345678901234567890123456789012123
123456789012345678901234567890121234567890123456789012345678901212345678901234567890123
H

12345678901234567890123456789012123456789012345678901234567890121234567890123456789012345678901212345678901234567890123
1234567890123456789012345678901212345678901234567890123456789012123456789012345678901234567890121234567890123
1234567890123456789012345678901212345678901234567890123456789012123456789012345678901234567890121234
12345678901234567890123456789012123456789012345678901234567890121234567890123456789012345678901212345678901234567890123456789012123
123456789012345678901234567890121234567890123456789012345678901212345678901234567890123
123456789012345678901234567890121234567890123456789012345678901212
12345678901234567890123456789012123456789012345678901234567890121234567890123456789012345678901212345678901234567890123
1234567890123456789012345678901212345678901234567890123456789012123456789012345678901234567890121234567890123
1234567890123456789012345678901212345678901234567890123456789012123456789012345678901234567890121234
12345678901234567890123456789012123456789012345678901234567890121234567890123456789012345678901212345678901234567890123456789012123
123456789012345678901234567890121234567890123456789012345678901212345678901234567890123
M

12345678901234567890123456789012123456789012345678901234567890121234567890123456789012345678901212345678901234567890123
1234567890123456789012345678901212345678901234567890123456789012123456789012345678901234567890121234567890123
1234567890123456789012345678901212345678901234567890123456789012123456789012345678901234567890121234
12345678901234567890123456789012123456789012345678901234567890121234567890123456789012345678901212345678901234567890123456789012123
123456789012345678901234567890121234567890123456789012345678901212345678901234567890123
AM
IG

1. „¬@O…Õ A …Õx Q®°~¡°ë ? H÷ „¬@O B …Õx ‹¬i†³Ø°#k. (T.S. S.I. Mains-2016)


KH
R

Figure - A Figure - B

I II III IV V
1) I 2) II 3) III 4) IV
1. (1) "³ò^Î\ ÷ „¬@O…Õx ƒç=°à#° u~¡Q®u„²æ ㄬH›ø<Í L#ß #Á KÇ°H›ø#° ï~O_È= „¬@O…Õx PHê~¡=ò…Õx ^¥x…Õ ‹¬Q®O À+_£#°
f‹²"͋² LOz# U~¡æ_È°#k. ï~O_È= „¬@O…Õx PHê~¡=ò. P q^ÎOQê 3= ㄬ‰×߅Õx PHê~¡=ò#° =¶~¡°æ° K͆ǰQê
=KÇ°ó#k ã‰õ}÷…Õ '?Ñ Q®°~¡°ë ªÖ#O…Õ =KÇ°ó ƒç=°à. D q^ÎOQê KǶÀ‹ë ã‰õ}÷…Õ `Ç~¡°"Œ~ò ƒç=°à figure B …Õx I ƒç=°à.

REASONING ABILITY (NON-VERBAL) ○ ○ ○ ○ ○ ○


200
○ ○ ○ ○ ○ ○ ○ ○ ○ ○ ○ ○ ○ ○ ○ ○ ○ ○ ○
Study Material
2 ‡éeH› „¬sH›Æ (Analogy)
‹¬¶KÇ# (1-35) : ãH÷O^Î L#ß ã„¬‰×߅ÕÁ =ü_È° ‹¬=°ª¼ zã`Œ°<Œß~ò. `Ç~¡°"Œ`Ç <Œ°QË ^¥xHˋ¬O XH› ㄬ‰§ß~¡ÖH›O LOk. "³ò^Î\ ÷

ï~O_È° ‹¬=°ª¼ zã`Œ =°^Î¼ ‹¬O|O^ÎO LOk. J^Í q^Î"³°Ø# ‹¬O|O^ÎO q°ye# ï~O_È° ‹¬=°ª¼ zã`Œ =°^Î¼ 䛀_¨

LO_¨e. ㄬ‰§ß~¡ÖH›O L#ßKË@ ‹¬i‡é†Í° ["Œ|° zã`Œxß Q®°iëOKÇO_. (‹¬=¶^¥# „¬\ì #°O_)

ㄬ‰×ß „¬@=ò° ‹¬=¶^¥# „¬@=ò°

1.

Y
(1) (2) (3) (4) (5)

M
2.

E
D
9
(1) (2) (3) (4) (5)

59
CA
95
3.

50
(1) (2) (3) (4) (5)

98
A
4.
:8
E
h
.P
IC

(1) (2) (3) (4) (5)


AD

5.
AB
O
ER
CH

(1) (2) (3) (4) (5)


YD

6.
,H
T
AM

(1) (2) (3) (4) (5)


H
M
AM
IG

7.
KH
R

(1) (2) (3) (4) (5)

8.

(1) (2) (3) (4) (5)

9.

(1) (2) (3) (4) (5)

10.

(1) (2) (3) (4) (5)

REASONING ABILITY (NON-VERBAL) ○ ○ ○ ○ ○ ○


201
○ ○ ○ ○ ○ ○ ○ ○ ○ ○ ○ ○ ○ ○ ○ ○ ○ ○ ○
Study Material
11.
?
(1) (2) (3) (4)

12.
?
(1) (2) (3) (4)

13.

Y
?

M
(1) (2) (3) (4)

E
D
9
14.

59
?
CA
95
(1) (2) (3) (4)

50
98
A
:8
15. ?
E
h
.P
IC

(1) (2) (3) (4)


AD
AB
O

16. ?
ER
CH

(1) (2) (3) (4)


YD
,H

17.
T
AM

?
H
M
AM

(1) (2) (3) (4)


IG
KH

18.
?
R

(1) (2) (3) (4)

19.
?
(1) (2) (3) (4)

20.
?
(1) (2) (3) (4)

REASONING ABILITY (NON-VERBAL) ○ ○ ○ ○ ○ ○


202
○ ○ ○ ○ ○ ○ ○ ○ ○ ○ ○ ○ ○ ○ ○ ○ ○ ○ ○
Study Material
21.
?
(1) (2) (3) (4)

22.
?
(1) (2) (3) (4)

23.
?

Y
M
(1) (2) (3) (4)

E
24.
?

D
9
59
CA
95
(1) (2) (3) (4)

50
25.
? 98
A
:8
E

(1) (2) (3) (4)


h
.P
IC
AD

26.
?
AB
O
ER
CH

(1) (2) (3) (4)


YD

27.
,H

?
T
AM
H

(1) (2) (3) (4)


M
AM
IG

28.
KH

?
R

(1) (2) (3) (4)

29.
?
(1) (2) (3) (4)

30.
?
(1) (2) (3) (4)

REASONING ABILITY (NON-VERBAL) ○ ○ ○ ○ ○ ○


203
○ ○ ○ ○ ○ ○ ○ ○ ○ ○ ○ ○ ○ ○ ○ ○ ○ ○ ○
Study Material
31.
?
(1) (2) (3) (4)

32.
?
(1) (2) (3) (4)

33.
?
(1) (2) (3) (4)

Y
M
34. ?

E
(1) (2) (3) (4)

D
9
59
35.
? CA
95
50
(1) (2) (3) (4)

98
A
1234567890123456789012
12345678901234567890123456789012123456789012345678901234567890121234567890123456789012345678901212345678901234567890123
12345678901234567890123
12345678901234567890123456789012123456789012345678901234567890121234567890123456789012345678901212345678901234567890123
1234567890123456789012 12345678901234567890123
1234567890123456789012
q=~¡}ì`ÇàH› ["Œ|°°
12345678901234567890123456789012123456789012345678901234567890121234567890123456789012345678901212345678901234567890123
12345678901234567890123
:8
1234567890123456789012 12345678901234567890123
12345678901234567890123456789012123456789012345678901234567890121234567890123456789012345678901212345678901234567890123
1234567890123456789012
1234567890123456789012 PRACTICE TEST : 12345678901234567890123
12345678901234567890123456789012123456789012345678901234567890121234567890123456789012345678901212345678901234567890123
12345678901234567890123
12345678901234567890123456789012123456789012345678901234567890121234567890123456789012345678901212345678901234567890123
E

1234567890123456789012 12345678901234567890123
12345678901234567890123456789012123456789012345678901234567890121234567890123456789012345678901212345678901234567890123
h
.P
IC

1. (1) "³ò^Î\ ÷ „¬@=ò…Õx PHê~¡=ò#° 180° ãu„²æ XH› ãHù`Çë J#°#k ^¥x †³òH›ø PHê~¡=ò#° `Çy¾OKǰ䛽O@°Ok.
AD

ö~Y#° z=~¡# H›e„²`Í U~¡æ_È°#k ï~O_È= „¬@=ò…Õx KÇ`Ç°~¡ã‹¬ boundary ^ÎQ®¾~¡ Q® =$`Çë=ò ^¥x †³òH›ø
AB
O

PHê~¡=ò. J^Íq^ÎOQê 3= ƒç=°à…Õx PHê~Œxß PHê~¡=ò#° ̄OKǰ䛽x öHOã^ÎO ^ÎQ®¾iH÷ =‹¬°ëOk. ̄á#
=¶~¡°æ° K͋²# U~¡æ_È°#k '?Ñ Q®°~¡°ë ªÖ#=ò…Õ =KÇ°ó
ER

^¥x „¬i=¶}=ò#° `Çy¾OKǰ䛽#ß PH›$u J#°#k D


CH

ƒç=°à nx ㄬHê~¡O KǶÀ‹ë '?Ñ Q®°~¡°ë ªÖ#O…Õ =KÇ°ó#k. =$`ÇëO …Õ„¬eH÷ KÍ~¡°`Ç°Ok. ƒì}O Q®°~¡°ë =¶ã`ÇO 135°
YD

‹¬=¶^¥# „¬@=ò…Õx 1=k. ‹¬=¼k‰×…Õ ハ=’ °}O K³O^ΰ`Ç°Ok. P q^OÎ Qê =¶~¡°æ°


,H

2. "³ò^Î\ ÷ „¬@=ò…Õx PHê~¡=ò#° 90° J„¬‹¬=¼k‰×…Õ KÍ ‹ ² # U~¡ æ _È ° PHê~¡ = ò ï ~ O_È = „¬ @ =ò…Õxk.


T

(3)
AM

ハ’=°}O K³OkOk =°i†Çò ^¥xß x°=ôQê ãu„²æ`Í J^Íq^ÎOQê 3= ƒç=°à…Õx PHê~¡=ò#° K͋²#
H
M

U~¡æ_È°#k ï~O_È= „¬@=ò…Õx PHê~¡=ò. D q^ÎOQê U~¡æ_È°#k '?Ñ Q®°~¡°ë ªÖ#O…Õ =KÇ°ó PHê~¡=ò. nx
AM
IG

=ü_È= „¬@=ò…Õx PHê~Œeß K͋²# U~¡æ_È°#k '?Ñ ã„¬Hê~¡O KǶÀ‹ë ‹¬=¶^¥# „¬@O…Õx 4=k '?Ñ Q®°~¡°ë
KH

Q®°~¡°ë ªÖ#O…Õ =KÇ°ó ƒç=°à. nx ㄬHê~¡O KǶÀ‹ë '?Ñ ªÖ#O…Õ =KÇ°ó ƒç=°à.
Q®°~¡°ë ªÖ#O…Õ =KÇ°ó ƒç=°à ‹¬=¶^¥# „¬@=ò…Õx
R

5. (5) "³ò^Î\ ÷ ƒç=°à…Õx PHê~¡=ò †³òH›ø J^΄ œ ô¬ ㄬuaO|O


3=k.
'2Ñ= ƒç=°à. J^Íq^ÎOQê 3= ƒç=°à…Õx PHê~¡=ò
3. (2) "³ò^Î\ ÷ „¬@=ò…Õx PHê~¡=ò#° ãu„²æ =°i†Çò ƒì}O
†³òH›ø J^Μ„¬ô ㄬuaO|O '?Ñ Q®°~¡°ë ªÖ#O…Õ =KÇ°ó
†³òH›ø `ǁ#° u~¡Q® ãu„²æ`Í U~¡æ_È°#k 2= ƒç=°à…Õx
ƒç=°à. '5Ñ nxH÷ ‹¬i†³Ø°# ‹¬=¶^¥#O.
PHê~¡=ò. D q^OÎ Qê =ü_È= „¬@=ò…Õx PHê~¡=òx
6. (2) "³ò^Î\ ÷ „¬@=ò…Õx ãH÷O^Î L#ß PHê~¡=ò †³òH›ø
K͋²# U~¡æ_È°#k '?Ñ Q®°~¡°ë ªÖ#O…Õ =KÇ°ó#k. nx
Z䛽ø=Qê ̄Oz`Í Jk ƒì‚¬ì¼ PHê~¡OQê =¶~¡°`Ç°Ok.
ㄬHê~¡O KǶÀ‹ë ‹¬=¶^¥# ƒç=°à…Õ 2=k '?Ñ Q®°~¡°ë
ªÖ#O…Õ =KÇ°ó#k. =°i†Çò ̄á PHê~¡=ò †³òH›ø „¬i=¶}ìxß HùkíQê ̄Oz
4. (4) "³ò^Î\ ÷ „¬@=ò…Õx Hö Oã^ÎO ^ÎQ~¾® ¡ PHê~¡O …Õ„¬ L#ß ^¥xß Ì„OKÇ|_# ãH÷O^Î PHê~¡=ò †³òH›ø =°^Î¼…Õ
PHê~¡=ò J#°#k KÇ`°Ç ~¡ã‹¬ boundary †³òH›ø z=iH÷ ̄\ì“e. JO`ÍHê䛽O_¨ ƒì‚¬ì¼OQê L#ß PHê~ŒxH÷
KÍ~¡°`Ç°Ok. =°i†Çò öHOã^ÎO ^ÎQ®¾~¡ Q® PHê~¡=ò WOHË ö~Y#° H›°„¬Qê U~¡æ_È° PHê~Œxß …Õ…Õ„¬Qê

REASONING ABILITY (NON-VERBAL) ○ ○ ○ ○ ○ ○


204
○ ○ ○ ○ ○ ○ ○ ○ ○ ○ ○ ○ ○ ○ ○ ○ ○ ○ ○
Study Material
LOz# U~¡æ_È°#k ㄬ‰×ß…Õ Wzó# ï~O_È= ƒç=°à…Õx =KÇ°ó ƒç=°à nx ㄬHê~¡O KǶÀ‹ë '?Ñ Q®°~¡°ë ªÖ#O…Õ
PHê~¡O P q^OÎ Qê 3= ƒç=°à…Õx PHê~ŒH÷ =¶~¡°æ° ‹¬=¶^¥# „¬\쁅Õx '1Ñ=k =‹¬°ëOk.
K͋²# U~¡æ_È°#k '?Ñ Q®°~¡°ë ªÖ#O…Õ =KÇ°ó ƒç=°à. nx 11. (1) 1= ‹¬=°ª¼ zã`ÇO…Õ ̄^Îí n~¡ÉKÇ`Ç°~¡ã‹¬O, z#ß
ㄬHê~¡O KǶÀ‹ë '?Ñ Q®°~¡°ë ªÖ#O…Õ ‹¬=¶^¥# „¬\쁅Õx n~¡KÉ `Ç °Ç ~¡ã‹¬O `Ç=° ªÖ<Œ° =¶~¡°óHùx, ~ï O_È= ‹¬=°ª¼
2=k =‹¬°ëOk. zã`ÇOQê U~¡æ_¨Û~ò. z#ß n~¡É KÇ`Ç°~¡ã‹¬O…Õ w`Ç
7. (3) "³ò^Î\ ÷ „¬@=ò…Õx PHê~¡O…Õ XH› ƒ’°[O "³á„¬ôQê …è~¡æ_¨Û~ò. J…ìO\÷ =¶~¡°æ…è 3= ‹¬=°ª¼ zã`ÇO…Õ
L#ß ö~Y#° ï~O_È° ªÖ<Œ° J„¬‹¬=¼k‰×…Õ =òO^ΰ䛽 [iy, Z_È=°"³á„¬ô z#ß ƒç=°à…Õ w`ǁ° ~Œ"Œe. 1=
[i„² =°i†Çò „¬@=ò…Õx, "Í~ùH› PHê~¡=ò#° ‹¬=¼ ["Œ|° ƒç=°à H›ï~䛽“.
k‰×…Õ =ü_È° ªÖ<Œ#° =òO^ΰ䛽 [i„²# U~¡æ_È°#k. 12. (1) Mìm KÇ`Ç°~¡ãªxß YO_‹¬¶ë ö~Y° U~¡æ_ÈÛ@°“, Mìm
ï~O_È= „¬@O…Õx PHê~¡O P q^ÎOQê 3= „¬@O…Õx =$`Œëxß YO_‹¬¶ë ö~Y° U~¡æ_¨e. 1= ["Œ|° ƒç=°à
PHê~ŒxH÷ =¶~¡°æ° K͋#² =KÇ°ó#q '?Ñ Q®°~¡°ë ªÖ#O…Õ H›ï~䛽“.

Y
=KÇ°ó PHê~¡O nx ㄬHê~¡O KǶÀ‹ë ‹¬=¶^¥# „¬\쁅Õx 13. (3) 1= ‹¬=°ª¼ „¬@O…Õx KÇ`°Ç ~¡ã‹¬O JOKǰ̄á L#ß w`ǁ°

M
'3Ñ=k '?Ñ Q®°~¡°ë ªÖ#O…Õ =KÇ°ó PHê~¡=ò. ï~O_Ë ‹¬=°ª¼ zã`ÇO…ÕxH÷ =¶~¡°`Ç°<Œß~ò. =$`ÇëO
8. (4) "³ò^Î\ ÷ „¬@=ò…Õx PHê~Œx 90° ãu„²æ "Œ\÷x ~ï O_È° 180O ‹¬=¼k‰×Qê uiyOk. W"Í =¶~¡°æ° 3= ‹¬=°ª¼

E
ƒìQꁰQê q_ÈQù\÷“ P q_ÈQù\÷“# ö~Y k‰×Qê 180° P zã`ÇO…Õ [iy`Í, 3= ["Œ|° zã`ÇO…Õ =…ÿ =¶~¡°æ°

D
9
~ï O_È° ƒìQêxß ãu„²æ XH› #Á\ ÷ =$`Ç=ë òKÍ H›°„¬|_# LO\ì~ò.

59
U~¡æ_È°#k '2Ñ= „¬@O…Õx PHê~¡O. J^Íq^ÎOQê 3=
CA 1= ‹¬=°ª¼ zã`ÇO…Õx PHê~¡=ò †³òH›ø J^Μ„¬ô

95
14. (2)
„¬@O…Õx PHê~Œxß =¶~¡°æ° K͋²# U~¡æ_È°#k '?Ñ ã„¬uaO|O '2Ñ= ƒç=°à…Õx PHê~¡O. J^Íq^ÎOQê '?Ñ

50
Q®°~¡°ë ªÖ#O…Õ =KÇ°ó#k. nx ㄬHê~¡O KǶÀ‹ë ‹¬=¶^¥# Q®°~¡°ë ªÖ#O…Õ =KÇ°ó PHê~¡=ò 3= ƒç=°à J^Μ„¬ô
98
A
„¬\쁅Õx 4=k '?Ñ Q®°~¡°ë ªÖ#O…Õ =KÇ°ó PHê~¡O. ㄬuaO|O '?Ñ ªÖ#O…Õ =KÇ°ó ƒç=°à nx ㄬHê~¡O
:8
9. "³ò^Î@ „¬@=ò…Õ #°„¬ô P䛽 135° J„¬‹¬=¼k‰×…Õ
E

(4)
KǶÀ‹ë ‹¬=¶^¥# „¬@=ò…Õx ï~O_È=k '?Ñ ªÖ#O…Õ
h

ハ’=°}O K³Ok, `³°„¬ô P䛽 90° J„¬‹¬=¼k‰×…Õ


.P

=KÇ°ó#°.
IC

ハ’=°}O K³O^ÎQê U~¡æ_È°#k '2Ñ= „¬@=ò…Õx


AD

15. (2) 1= ‹¬=°ª¼ zã`ÇO…Õx 'þÑ Q®°~¡°ë Qê =¶i, 2=


PHê~¡=ò. J^Íq^ÎOQê 3= ƒç=°à…Õx PHê~ŒH÷
AB
O

‹¬=°ª¼ zã`ÇO U~¡æ_Ok. J…ìöQ 5 n‡° =¶i 5


=¶~¡°æ° K͋²# U~¡æ_È°#k '?Ñ Q®°~¡°ë ªÖ#O…Õ =KÇ°ó Hù"ùÞ`Ç°ë ° U~¡æ_¨e.
ER
CH

ƒç=°à. nxㄬHê~¡O KǶÀ‹ë ‹¬=¶^¥# „¬\쁅Õx 4=k 16. "³ò^Î\ ÷ ‹¬=°ª¼ zã`ÇO…Õx =ü_È° Q®°~¡°ë° =ü_È°
YD

(2)
'?Ñ Q®°~¡°ë ªÖ#O…Õ =‹¬°ëOk. =$`Œë°Qê =¶i, 2= ‹¬=°ª¼ zã`ÇO U~¡æ_Ok.
,H

10. (1) "³ò^Î\ ÷ ƒç=°à…Õx PHê~Œ° D ãH÷Ok q^ÎOQê KÇ`Ç°~¡ã‹¬O ' ÑQê =¶iOk. 3= ‹¬=°ª¼ zã`ÇO…Õx
T
AM

=¶~¡°æ° K³OkOk, P `Ç~ŒÞ`Ç s-PHê~¡„¬ô Q®°~¡°ë lat- ° =$`Œë°Qê =¶i`Í, 2= ["Œ|° zã`Ç"°Í ~¡æ_È°`Ç°Ok.
H

eral Qêãu„¬æ|_È°`Ç°Ok. (P =¶~¡°æ K³Ok# ªÖ#O…Õ<Í)


M

17. (4) XH›@= ‹¬=°ª¼zã`ÇO…Õx ãuƒ’°[O ƒ’¶q°x P#°Hùx


AM

=°i†Ç ò n~¡ É K Ç ` Ç ° ~¡ 㠋¬ O J#°#k 90° ハ ’ = °}O


IG

=°~Ë ãuƒ°’ [O JO>è ~ï O_È° ãuƒ°’ *ì`Ë ~ï O_Ë ‹¬=°ª¼


K³O^ΰ`Ç°Ok. '«Ñ Q®°~¡°ë 䛀_¨ 90° ハ=’ °}O K³O^ΰ`Ç°Ok „¬@O U~¡æ_Ok. J…ìöQ Ìÿ 䛽 =°~Ë Ìÿ H›e‹² Î
KH

=°i†Çò ƒì}O Q®°~¡°ë lateral Qê ãu„¬æ|_È°`Ç°Ok.


R

U~¡æ_¨e.
=$`Çë=ò J#°#k À+_£ K͆ǰ|_È°`Ç°Ok. (P =¶~¡°æ
18. (3)
K³Ok# ªÖ#O…Õ<Í) PHê~Œ° =¶~¡°æ K³O^ΰ f~¡°
19. (4) 1= ‹¬=°ª¼ zã`ÇO…Õx <Œ°Q®° ƒç=°à° 2= ‹¬=°ª¼
(pattern).
zã`ÇO…Õ =¼uö~H›k‰×…ÕÁH÷ "³ˆì¤~ò. W^Í =¶~¡°æ 3=
‹¬=°ª¼ zã`ÇO…Õ [iy`Í, 4= ["Œ|° zã`ÇO…ì
=¶~¡°æ° [~¡°Q®°`Œ~ò.
20. (3)

D q^ÎOQê =¶~¡°æ° K͋²# `Ç~ŒÞ`Ç U~¡æ_È°#k. ㄬ‰×ß 21. (2) ‹¬=°ª¼ zã`ÇO 1H÷ 2= ‹¬=°ª¼ zã`ÇO J^ΜO †³òH›ø
„¬@=ò…Õx ï~O_È=k. J^Íq^Î"³°Ø# =¶~¡°æ° '3Ñ= ㄬuaOƒO. Hê=ô# 3= ‹¬=°ª¼ zã`ŒxH÷ ‹¬i„¬_¨
ƒç=°à…Õx PHê~ŒH÷ K͋#² =KÇ°ó#k '?Ñ Q®°~¡°ë ªÖ#O…Õ ["Œ|° 2.

REASONING ABILITY (NON-VERBAL) ○ ○ ○ ○ ○ ○


205
○ ○ ○ ○ ○ ○ ○ ○ ○ ○ ○ ○ ○ ○ ○ ○ ○ ○ ○
Study Material
22. (3) ‹¬=°ª¼ zã`ÇO 1 #°O_ 2H÷ ƒì}O Q®°~¡°ë ^¥x =¼uö~H› 29. (4) 1= ‹¬=°ª¼ zã`ÇO…Õx ï~O_͋² w`DžÕÁ XHùøH›ø w`Ç
k‰×…Õ L#ß ƒ’°[O =^Îí䛽 =°i†Çò =ü# =ô#ß `Çy¾ ~ï O_Ë ‹¬=°ª¼zã`ÇO U~¡æ_Ok. J…ìöQ 3= zã`ÇO…Õ
=ãH›ö~Y `Ç~¡°"Œ`Ç =üä›½ ‹¬=¼k‰×…Õ "³ˆ×Ãë<Œß~ò. XHùøH›ø w`Ç `Çy¾`Í, Mìm KÇ`Ç°~¡ã‹¬"Í° q°Q®°°`Ç°Ok.
Hê=ô# Mìm Q®_…Õ '3Ñ ["Œ|° ‹¬i‡é`Ç°Ok. 30. (4) 1= ‹¬=°ª¼ zã`ÇO ‹¬=¼k‰×Qê 90O uiy ~ï O_Ë ‹¬=°ª¼
23. (2) 1= ‹¬=°ª¼ zã`ÇO 90O ‹¬=¼k‰×…Õ HË}O uiy 2= zã`ÇO U~¡æ_Ok. J\ìÁöQ 3= ‹¬=°ª¼zã`ÇO uiy 4=
„¬@O U~¡æ_Ok. =°i†Çò =ü# L#ß =ãH›ö~Y ["Œ|° zã`ÇO…ì Mìm ªÖ#O…Õ ƒç=°à U~¡æ_È°`Ç°Ok.
`Ç~¡°"Œ`Ç =üä›½ J„¬‹¬=¼k‰×…Õ uiyOk. Hê=ô# 31. (2) "³ò^Î\ ÷ ƒç=°à…Õx PHê~¡=ò †³òH›ø J^΄
œ ô¬ ㄬuaO|O
Mìm Q®_…Õ 2= ["Œ|° „¬@O U~¡æ_È°`Ç°Ok.
ï ~ O_È = ƒç=°à…Õx PHê~¡ = ò. J^Í q ^ Î O Qê 3=
24. (1) 1= ‹¬=°ª¼ zã`ÇO #°O_ 2= „¬@O…ÕxH÷ …Õ„¬e ƒç=°à
ƒç=°à…Õx PHê~¡=ò †³òH›ø J^Μ„¬ô ㄬuaO|O '?Ñ
|†Ç°@䛽, |†Ç°\÷ ƒç=°à …Õ„¬eH÷ "³ˆì¤~ò. nx ㄬHŒ~¡O
Q®°~¡°ë ªÖ#O…Õ =KÇ°ó#k. Hê=ô# nxH÷ ["Œ|° (2).

Y
3= ƒç=°à…Õ PHê~ŒH÷ =¶~¡°æK͋²# =KÇ°ó#k
"³ò^Î\ ÷ ƒç=°à…Õx PHê~Œ#° "Œ\÷ ªÖ<Œ#°

M
32. (1)
‹¬=¶^¥#O…Õ 1= ƒç=°à.
„¬~¡‹¬æ~¡O =¶~¡°óHùx# =°i†Çò Q®°~¡°ë (þ) =°^Î¼…Õ
1= ‹¬=°ª¼ zã`ÇO…Õx =°^Î¼ ƒç=°à 2…Õ ̄^Îík J~ò

E
25. (3)
=zó# J„¬C_È° U~¡æ_È°#k '2Ñ= ƒç=°à…Õ Q® PHê~¡O.
=°i†Çò ̄á# L#ß ƒì}O Q®°~¡°ë k‰× `Œ~¡°=¶~¡°

D
9
nx ㄬHê~¡O '3Ñ= ƒç=°à…Õx PHê~ŒH÷ =¶~¡°æ°

59
J~òOk. Hê=ô# 3= ƒç=°à…Õ W…ìO\÷ =¶~¡°æ [iy`Í
CA
95
3= ["Œ|° „¬@O U~¡æ_È°`Ç°Ok. K͋²# U~¡æ_È°#k '?Ñ Q®°~¡°ë ªÖ#O…Õ =KÇ°ó PHê~¡=ò.

50
26. (3) 1= ‹¬=°ª¼ zã`ÇO…Õ =$`ÇëO…Õx ‹¬Q®O ƒìQ®O‡é~ò, Hê=ô# nxH÷ ‹¬i†³Ø°# ["Œ|° (1).
98
A
J_ÈÛw`ǁ° x°= w`ǁ°Qê =¶i 2= ‹¬=°ª¼ zã`ÇO 33. (4) "³ò^Î\ ÷ ƒç=°à…Õx PHê~¡=ò †³òH›ø J^΄œ ô¬ ㄬuaO|O,
:8
U~¡æ_Ok. W"Í =¶~¡°æ° [iy KÇ`°Ç ~¡ã‹¬O…Õ ‹¬Qƒ ® ì Q®O ~ï O_È= ƒç=°à…Õx PHê~¡O. J^Íq^OÎ Qê '3Ñ= ƒç=°à…Õx
E
h
.P

‡é~ò, J_ÈÛw`ǁ° x°=ô w`ǁ°Qê U~¡æ_¨e. J…ì PHê~ŒH÷ =¶~¡°æ° K͋#² =KÇ°ó#k '?Ñ Q®°~¡°ë ªÖ#O…Õ
IC
AD

L#ß 3= ["Œ|° ƒç=°à ‹¬ï~á# ["Œ|° ƒç=°à. =KÇ°ó#k. Hê=ô# nxH÷ ‹¬i†³Ø°# ["Œ|° (4).
AB
O

27. (3) 1= ‹¬=°ª¼ zã`ÇO…Õx =$`ÇëO`Ë ‡@°, J^Î#OQê 34. (2) "³ò^Î\ ÷ ƒç=°à…Õx PHê~Œxß '450Ñ ‹¬=¼k‰×…Õ ãu„²æ,
ER

=°~Ë u~¡Q®|_# =$`ÇëO`Ë ï~O_Ë ‹¬=°ª¼ zã`ÇO


CH

=°^Î¼# L#ß '.Ñ Q®°~¡°ë fÀ‹‹² ̄á =zó# PHê~¡=ò…Õ,


U~¡æ_Ok. W…ìO\÷ =¶~¡°æ° [iy`Í 3= ‹¬=°ª¼
YD

ï~O_È° ƒì}O Q®°~¡°ë#° H›°„¬Qê =KÇ°ó#k '2Ñ=


zã`ÇO…Õx n~¡K É `Ç °Ç ~¡ãªxH÷ J^Î#OQê J^Í n~¡K
É `Ç °Ç ~¡ã‹¬O
,H

ƒç=°à…Õ Q® PHê~¡=ò.


T

u~¡Q®|_ <Œ°QË ªÖ#O…Õ ~Œ"Œe. ["Œ|° ƒç=°à…ÕÁ


AM

35. (4) "³ò^Î\ ÷ ƒç=°à…Õx =°^Î¼…Õ Q® ̄^Îí PHê~Œxß


H

3=k H›ï~䛽“.
M

`ùyOKÇ Q ê U~¡ æ _È ° #k. ï ~ O_È = ƒç=°à…Õ Q® 


AM

28. 1= ‹¬=°ª¼ zã`ÇO…Õx Q®°~¡°ë° ‹¬=¼k‰×…Õ uiy


IG

(1)
PHê~¡=ò. P q^ÎOQê '3Ñ= ƒç=°à…Õx PHê~ŒH÷
=üä›½ "³o¤ 2= zã`ÇOQê U~¡æ_¨Û~ò. J…ìöQ
KH

=¶~¡°æ K³OkOz# '?Ñ Q®°~¡°ë ªÖ#O…Õ =KÇ°ó PHê~¡=ò.


R

ãuƒ°’ [O KÇ°@¶“ L#ß Q®°~¡°ë ° ‹¬=¼k‰×Qê uiy`Í ["Œ|° Hê=ô# nxH÷ ‹¬i†³Ø°# ["Œ|° (4).
ƒç=°à (1)…ÕQê Mìm ‹¬=°ª¼ zã`ÇO U~¡æ_¨e.

REASONING ABILITY (NON-VERBAL) ○ ○ ○ ○ ○ ○


206
○ ○ ○ ○ ○ ○ ○ ○ ○ ○ ○ ○ ○ ○ ○ ○ ○ ○ ○
Study Material
123456789012345678901234567890121234567890123456789012345678901212
12345678901234567890123456789012123456789012345678901234567890121234567890123456789012345678901212345678901234567890123
1234567890123456789012345678901212345678901234567890123456789012123456789012345678901234567890121234567890123
1234567890123456789012345678901212345678901234567890123456789012123456789012345678901234567890121234
12345678901234567890123456789012123456789012345678901234567890121234567890123456789012345678901212345678901234567890123456789012123
123456789012345678901234567890121234567890123456789012345678901212345678901234567890123
12345678901234567890123456789012123456789012345678901234567890121234567890123456789012345678901212345678901234567890123
1234567890123456789012345678901212345678901234567890123456789012123456789012345678901234567890121234567890123
1234567890123456789012345678901212345678901234567890123456789012123456789012345678901234567890121234
123456789012345678901234567890121234567890123456789012345678901212
12345678901234567890123456789012123456789012345678901234567890121234567890123456789012345678901212345678901234567890123456789012123
123456789012345678901234567890121234567890123456789012345678901212345678901234567890123
12345678901234567890123456789012123456789012345678901234567890121234567890123456789012345678901212345678901234567890123
1234567890123456789012345678901212345678901234567890123456789012123456789012345678901234567890121234567890123
1234567890123456789012345678901212345678901234567890123456789012123456789012345678901234567890121234
12345678901234567890123456789012123456789012345678901234567890121234567890123456789012345678901212345678901234567890123456789012123
123456789012345678901234567890121234567890123456789012345678901212345678901234567890123
123456789012345678901234567890121234567890123456789012345678901212
PREVIOUS BITS
12345678901234567890123456789012123456789012345678901234567890121234567890123456789012345678901212345678901234567890123
1234567890123456789012345678901212345678901234567890123456789012123456789012345678901234567890121234567890123
1234567890123456789012345678901212345678901234567890123456789012123456789012345678901234567890121234
12345678901234567890123456789012123456789012345678901234567890121234567890123456789012345678901212345678901234567890123456789012123
123456789012345678901234567890121234567890123456789012345678901212345678901234567890123
123456789012345678901234567890121234567890123456789012345678901212
12345678901234567890123456789012123456789012345678901234567890121234567890123456789012345678901212345678901234567890123
1234567890123456789012345678901212345678901234567890123456789012123456789012345678901234567890121234567890123
1234567890123456789012345678901212345678901234567890123456789012123456789012345678901234567890121234
12345678901234567890123456789012123456789012345678901234567890121234567890123456789012345678901212345678901234567890123456789012123
123456789012345678901234567890121234567890123456789012345678901212345678901234567890123
12345678901234567890123456789012123456789012345678901234567890121234567890123456789012345678901212345678901234567890123
1234567890123456789012345678901212345678901234567890123456789012123456789012345678901234567890121234567890123
1234567890123456789012345678901212345678901234567890123456789012123456789012345678901234567890121234
12345678901234567890123456789012123456789012345678901234567890121234567890123456789012345678901212345678901234567890123456789012123
123456789012345678901234567890121234567890123456789012345678901212345678901234567890123
123456789012345678901234567890121234567890123456789012345678901212
12345678901234567890123456789012123456789012345678901234567890121234567890123456789012345678901212345678901234567890123456789012123
123456789012345678901234567890121234567890123456789012345678901212345678901234567890123
12345678901234567890123456789012123456789012345678901234567890121234567890123456789012345678901212345678901234567890123
1234567890123456789012345678901212345678901234567890123456789012123456789012345678901234567890121234567890123
1234567890123456789012345678901212345678901234567890123456789012123456789012345678901234567890121234

‹¬¶KÇ# (1-4) : =~¡ä›½ Q® ㄬu ㄬ‰×ß…Õ ï~O_È° „¬@=ò ‹¬=ò^¥†Ç°=ò°<Œß~ò. A, B, C, D „¬@=ò° ‹¬=°‹¬¼ ‹¬q°ux, 1, 2,

3, 4 „¬@=ò° ‹¬=¶^¥# ‹¬q°ux ‹¬¶zªë~ò. A =°i†Çò B  =°^Î¼ XH› xiœ+¬“ ‹¬O|O^ÎO LOk. J\ìÁO\ ÷ ‹¬O|O^¥<Íß

C …Õ H›ey# „¬\ìxß ‹¬=¶^¥# ‹¬q°u #°Oz, D …Õx ªÖ<ŒxH÷ ZO„²H› K͆ǰO_. (A.P. S.I. Prelims -2016)

1.

Y
2.

M
E
D
9
59
CA
95
3.

50
98
A
:8
E
h
.P
IC
AD

4.
AB
O
ER
CH
YD

„¬@O …Õx Q®°~¡°ë '?Ñ H÷ „¬@O …Õx ‹¬i†³Ø°#k.


,H

5. A B (T.S. S.I. Prelims -2016)


T
AM
H
M
AM
IG
KH
R

1) I 2) II 3) III 4) IV
6. „¬@O (i) H÷ „¬@O (ii) `Ë L#ß ‹¬O|O^ÎO =O\÷^Í, „¬@O (iii) H÷ „¬@O (iv) `Ë LO_Í@@°ÁQê, ㄬ‰×ß Q®°~¡°ë ªÖ#O…Õ
‹¬i†³Ø°# ㄬ`Œ¼=¶ß†Ç°OQê Q® „¬\ìxß, ‹¬=¶^¥#‹¬q°u #°O_ Z#°ßHù#°=ò. (A.P. S.I. Mains -2016)

‹¬=°ª¼ ‹¬q°u ‹¬=¶^¥# ‹¬q°u

REASONING ABILITY (NON-VERBAL) ○ ○ ○ ○ ○ ○


207
○ ○ ○ ○ ○ ○ ○ ○ ○ ○ ○ ○ ○ ○ ○ ○ ○ ○ ○
Study Material
7 =°i†Çò 8 ㄬ‰×߁䛽 xö~œ‰×Hꁰ :
A =°i†Çò B „¬\ì =°^Î¼ XH› xiœ+¬“"³°Ø# ‹¬O|O^ÎO L#ßk. C =°i†Çò D  =°^Î¼ J^Íq^Î"³°Ø# ‹¬O|O^¥xß U~¡æ~¡KÍ
‹¬i†³Ø°# „¬\ìxß ‹¬=¶^¥# ‹¬q°u #°O_ Z#°ßHùx D x xO„¬O_. (A.P. S.I. Mains -2016)

7. ㄬ‰×߁ ‹¬q°u
8. ㄬ‰×߁ ‹¬q°u

["Œ|° ‹¬q°u
["Œ|° ‹¬q°u

Y
M
E
D
9
<Œ°Q®° SzóHꁰ (1) #°Oz (4)…Õ, J‹¬° [`Ç…Õ Q® ‹¬O|O^¥xß ‡ée#\÷“q =ü_È° L<Œß~ò. J‹¬° [`Ç…Õ Q®

59
9.

‹¬O|O^ÎO …ìO\÷k …èx SzóHêxß ZOz, ^¥xx g° ‹¬=¶^¥#OQê ~Œ†Ç°O_.


CA
95
(A.P. S.I. Mains -2016)

J‹¬° [`Ç

50
98
A
:8
E
h
.P
IC

‹¬=¶^¥# [`ǁ°
AD
AB
O
ER
CH
YD
,H

ãH÷Ok ㄬ‰×߅Õx Wzó# <Œ°Q®° ‹¬=°ª¼ „¬\쁅Õ, XH› ãH÷Ok W=Þ|_# 11 =°i†Çò 12 ㄬ‰×߁…Õ, ï~O_È°
T

10. (11-12) :
AM

xiœ+¬“"³°Ø# ª=¶#¼ H›Æ}=ò H›^ΰ. (1) #°Oz (4)…Õx ‹¬=°ª¼ „¬\ì`Ë ‡@°, (1) #°Oz (4)KÍ Q®°iëOKÇ|_#
H
M

‹¬=¶^¥# „¬\ì #°O_, D ª=¶#¼ H›} Æ =ò Q® „¬\ìxß ‹¬=¶^¥# „¬\쁰 L<Œß~ò. ~ï O_È=ô ‹¬=°ª¼ „¬\쁅Õ#¶
AM
IG

Z#°ßHùx g° ‹¬=¶^¥#OQê ~Œ†Ç°O_. Q® ª=¶#¼ H›Æ}ì<Í H›ey L#ß ‹¬=¶^¥# „¬\ìxß
KH

(A.P. S.I. Mains -2016) Z#°ßHùx, g° ‹¬=¶^¥#O ~Œ†Ç°O_.


R

‹¬=°ª¼ „¬@=ò° (A.P. S.I. Mains -2016)

11. ‹¬=°ª¼ „¬@=ò°

‹¬=¶^¥# „¬@=ò° ‹¬=¶^¥# „¬@=ò°

REASONING ABILITY (NON-VERBAL) ○ ○ ○ ○ ○ ○


208
○ ○ ○ ○ ○ ○ ○ ○ ○ ○ ○ ○ ○ ○ ○ ○ ○ ○ ○
Study Material
12. ‹¬=°ª¼ „¬@=ò° ‹¬=¶^¥# „¬@=ò°

12345678901234567890123456789012123456789012345678901234567890121234567890123456789012345678901212345678901234567890123
1234567890123456789012 123456789012345678901234
12345678901234567890123456789012123456789012345678901234567890121234567890123456789012345678901212345678901234567890123
1234567890123456789012
q=~¡}ì`ÇàH› ["Œ|°°
123456789012345678901234
12345678901234567890123456789012123456789012345678901234567890121234567890123456789012345678901212345678901234567890123
1234567890123456789012 123456789012345678901234
12345678901234567890123456789012123456789012345678901234567890121234567890123456789012345678901212345678901234567890123
1234567890123456789012 123456789012345678901234
1234567890123456789012
1234567890123456789012 PREVIOUS BITS :
12345678901234567890123456789012123456789012345678901234567890121234567890123456789012345678901212345678901234567890123
123456789012345678901234
12345678901234567890123456789012123456789012345678901234567890121234567890123456789012345678901212345678901234567890123
123456789012345678901234
12345678901234567890123456789012123456789012345678901234567890121234567890123456789012345678901212345678901234567890123
1234567890123456789012 123456789012345678901234

1. (4) ƒç=°à 'A' …Õx "³°„¬e PHê~¡=ò (n~¡É KÇ`°Ç ~¡ã‹¬=ò) U~¡æ_È°#k (ii)= ƒç=°à. J^Íq^ÎOQê (iii) = ƒç=°à
z#ßk K͆°Ç |_#k, …Õ„¬e PHê~¡=ò (ãuƒ°’ [=ò) ̄^Îkí („¬OKƒÇ °’ l)H÷ =¶~¡°æ° K͆°Ç Qê U~¡æ_È°#k '?Ñ ªÖ#O…Õ
K͆°Ç |_ "³ò^Î@Qê z#ßk K͆°Ç |_# PHê~Œxß D =KÇ°ó ƒç=°à. Hê=ô# nxH÷ ‹¬i†Ç°Q®° ["Œ|°. ‹¬=¶^¥#

Y
ãuƒ°’ [=ò …Õ„¬ LOz ^¥xß À+_£ K͆°Ç Qê U~¡æ_È°#k ‹¬q°u…Õ (1) =k.

M
ƒç=°à 'B' …Õx PHê~¡=ò. D q^OÎ Qê ƒç=°à 'C' …Õx 7. (2) ƒç=°à 'A' …Õx +¬_Ȱĝl, ^¥xH÷ JO@°Hùx L#ß n~¡É
PHê~Œä›½ =¶~¡°æ° K͆ǰQê U~¡æ_È°#k '?Ñ Q®°~¡°ë KÇ`Ç°~¡ã‹¬O ï~O_ȶ '90OÑ J„¬‹¬=¼k‰×…Õ ªÖ#ハ’O‰×O

E
ªÖ#O…Õ =KÇ°ó PHê~¡=ò. Hê=ô# nxH÷ ‹¬i†³Ø°# K³Ok#q =°i†Çò n~¡ÉKÇ`Ç°~¡ã‹¬ PHê~¡O ㄬH›ø<Í L#ß

D
‹¬=¶^¥#=ò, ‹¬=¶^¥# „¬\쁅Õx (4) =k. ƒ°’ *ìxH÷ J„¬‹=¬ ¼k‰×…Õ `Ç# ªÖ<Œxß =¶~¡°óHùx Z_È=°

9
59
2. (1) "³ò^Î\ ÷ 'A' …Õx PHê~¡=ò (+¬_Ȱĝl)…Õ XH› ƒ’°*ìxß ã„¬H›ø# L#ß ƒ’°[=ò †³òH›ø =°^Î¼=° ªÖ#O…Õ J=°i
CA
95
`ùyOKÇQê 'B'…Õ „¬OKǃ’°l U~¡æ_È°`Ç°Ok. J…ìöQ LO@°Ok. W^Íq^Î"³°Ø# =¶~¡°æ° ƒç=°à 'C'…Õx

50
'A'…Õx PHê~¡=ò "³°„¬ ï~O_È° =$`Œë° 'B'…Õx PHê~ŒxH÷ K͋²#@Á~ò`Í U~¡æ_È°#k '?Ñ Q®°~¡°ë ªÖ#O…Õ
98
A
PHê~¡=ò#° "³°„¬Qê LOKŒe. XH›^¥xHùH›\ ÷ =KÇ°ó ƒç=°à. Hê=ô# nxH÷ ‹¬i„¬_È° ‹¬=¶^¥# ƒç=°à
:8
Z^ΰ~¡°Qê =°i†Çò =°^¼Î …Õ XöH À+_£ K͆°Ç |_# =$`Œëxß ["Œ|° ‹¬q°u…Õx ï~O_È=k.
E

LOKÇQê U~¡æ_È°#k 'B'…Õx PHê~¡=ò. W^Í q^Î"³°Ø# 8. ƒç=°à 'A' …Õx ƒì}O Q®°~¡°ë H›e¾ L#ß ö~Y H›~òH›
h

(3)
.P

=¶~¡°æ° 'C' …Õx PHê~¡=ò#䛽 K͋²# U~¡æ_È°#k †³òH›ø PHê~¡=ò J#°#k '45OÑ ‹¬=¼k‰×…Õ ªÖ#ハO’ ‰×O
IC
AD

'?Ñ Q®°~¡°ë ªÖ#=ò…Õ =KÇ°ó PHê~¡=ò. Hê=ô# nxH÷ K³Ok, ^¥x…Õ XH› ö~Y…Õ ‹¬Q®ƒìQ®O H˅Õæ=ô#°. J>èÁ
‹¬i†³Ø°# ‹¬=¶^¥#=ò (1). 'þÑ Q®°~¡°ë 90O ‹¬=¼k‰×…Õ ªÖ#ハO’ ‰×O K³Ok '{Ñ Q®°~¡°Që ê
AB
O

3. ƒç=°à 'A' (ãuƒ’°[O)…Õx PHê~¡=ò#䛽 u~¡Q®ãu„²æ =¶~¡°`Ç°Ok. W@°=O\÷ =¶~¡°æ° ƒç=°à 'C' …Õx
ER

(4)
CH

D ï ~ O_È ° PHê~¡ = ò#° XH› ø ^¥x…Õ WOHùH› \ ÷ PHê~ŒxH÷ H›#°H› K͋²#@Á~ò`Í U~¡æ_È°#k '?Ñ Q®°~¡°ë
YD

J=°~¡óQê U~¡æ_È°#k 2= ƒç=°à…Õx PHê~¡=ò. J…ìöQ ªÖ#=ò…Õ =KÇ°ó PHê~¡=ò. Hê=ô# nxH÷ ‹¬i†³Ø°#
,H

3= ƒç=°à (~ŒO|‹¹)䛽 =¶~¡°æ° K͆ǰQê U~¡æ_È°#k ‹¬=¶^¥#=ò, ["Œ|° ‹¬q°u…Õx (3) =k.
T

'?Ñ Q®°~¡°ë ªÖ#O…Õ =KÇ°ó PHê~¡=ò. Hê=ô# nxH÷ J‹¬° [`Ç…Õ "³ò^Î\ ÷ ãÀ„¦"£°…Õx PHê~Œ…Õ …Õ„¬e
AM

9. (4)

‹¬i†³Ø°# ‹¬=¶^¥#=ò. ‹¬=¶^¥# zã`Œ…Õx 4=k. =$`Çë=ò ̄^ÎíkQê K͋² ãuƒ’°*ìxß =$`ÇëO…Õ„¬ LOz,
H
M

4. ƒç=°à 'A'…Õx ƒì}O `ǁ Q®°~¡°ë Q® PHê~Œxß '90°' n~¡ÉKÇ`Ç°~¡ãªxß ãuƒ’°[O…Õ„¬ LOz# U~¡æ_È°#k 2=
AM

(3)
IG

‹¬=¼k‰×…Õ ãu„²æ ãÀ„¦"£°…Õ Z_È=°"³á„¬ôQê ̄\÷“, q°ye# ƒç=°à…Õx PHê~¡=ò. W^Íq^ÎOQê D ãH›=°=ò#°
KH

ƒìQ®=ò u~¡Qã® u„²æ ‹¬=¼k‰×…Õ ªÖ#ハO’ ‰×O K³OkOKÇQê „¬ijeÀ‹ë ‹¬=¶^¥<Œ…Õ [`ǁ…Õx 4= ƒç=°à ‹¬i†³°Ø #
R

U~¡æ_È°#k 'B' …Õx PHê~¡=ò. W^Í q^Î"³°Ø# =¶~¡°æ° ["Œ|° J=ô`Ç°Ok.


'C' …Õx PHê~¡=ò#䛽 K͋²#KË =KÇ°ó#k '?Ñ Q®°~¡°ë 10. (4) W=Þ|_# Jxß ƒç=°à…Õ Jxß ‹¬=¶# ƒìQꁰQê
ªÖ#O…Õx PHê~¡=ò. Hê=ô# nxH÷ ‹¬i†³Ø°# ["Œ|°. qƒ’lO„¬|_¨Û~ò. Hê=ô# nxH÷ ㄬHê~¡O KǶÀ‹ë
‹¬=¶^¥# „¬\쁅Õx (2) =k. ‹¬=¶^¥<Œ „¬\쁅Õx 4=k Hê=‹²# ‹¬=¶^¥#O.
5. (3) ƒç=°à 'A' …Õx PHê~¡=ò#° u~¡Q®ãu„²æ, z=~¡# L#ß 11. (3) W=Þ|_# "³ò^Î\ ÷ „¬@O…Õ ï~O_È° PHê~Œ° ‹¬=¶#
KÇ°H›ø#° q_Qê ̄áH÷ f‹¬°ä›½=zó# U~¡æ_È°#k 'B'…Õx ƒ°’ *쁰 H›e¾ L<Œß~ò. J>èÁ ~ï O_È= ƒç=°à…Õx PHê~Œ°
PHê~¡=ò. W^Í q^"Î °³ #Ø =¶~¡°æ° 'C'…Õx PHê~¡=ò#䛽 䛀_¨ H›e¾ L<Œß~ò. nx ㄬHê~¡O KǶÀ‹ë ‹¬=¶^¥#
K͋#² KË U~¡æ_È°#k '?Ñ Q®°~¡°ë ªÖ#O…Õ =KÇ°ó#k. Hê=ô# „¬@O 3=k.
nxH÷ ‹¬i†³Ø°# ["Œ|° III = ƒç=°à. 12. (3) "³ò^Î\ ,÷ ~ï O_È= „¬\쁅Õx =$`ÇOë …Õx ‹¬QO® PHê~Œxß
6. (1) u~¡Q® ãu„¬æ|_# ãuƒ°’ [O PHê~Œxß ‹¬i†Ç°Q®° ãH›=°O…Õ H›e¾ L#ß „¬@O 3=k H›#°H›. nxH÷ ‹¬i†³°Ø # ‹¬=¶^¥#O
(x°=ôQê) J=°ió ^¥x †³òH›ø „¬i=¶}O `Çy¾OKÇQê 3=k.

REASONING ABILITY (NON-VERBAL) ○ ○ ○ ○ ○ ○


209
○ ○ ○ ○ ○ ○ ○ ○ ○ ○ ○ ○ ○ ○ ○ ○ ○ ○ ○
Study Material
a#ß"³°Ø# ^¥xx Q®°iëOKÇ°@
3
123456789012345678901234567890121234567890123456789012345678901212
(Odd Man Out)
12345678901234567890123456789012123456789012345678901234567890121234567890123456789012345678901212345678901234567890123
1234567890123456789012345678901212345678901234567890123456789012123456789012345678901234567890121234567890123
1234567890123456789012345678901212345678901234567890123456789012123456789012345678901234567890121234
123456789012345678901234567890121234567890123456789012345678901212
12345678901234567890123456789012123456789012345678901234567890121234567890123456789012345678901212345678901234567890123456789012123
1234567890123456789012345678901212345678901234567890123456789012123456789012345678901234567890
12345678901234567890123456789012123456789012345678901234567890121234567890123456789012345678901212345678901234567890123456789012123
1234567890123456789012345678901212345678901234567890123456789012123456789012345678901234567890
12345678901234567890123456789012123456789012345678901234567890121234567890123456789012345678901212345678901234567890123
1234567890123456789012345678901212345678901234567890123456789012123456789012345678901234567890121234567890123
1234567890123456789012345678901212345678901234567890123456789012123456789012345678901234567890121234
123456789012345678901234567890121234567890123456789012345678901212
12345678901234567890123456789012123456789012345678901234567890121234567890123456789012345678901212345678901234567890123456789012123
1234567890123456789012345678901212345678901234567890123456789012123456789012345678901234567890
12345678901234567890123456789012123456789012345678901234567890121234567890123456789012345678901212345678901234567890123
1234567890123456789012345678901212345678901234567890123456789012123456789012345678901234567890121234567890123
1234567890123456789012345678901212345678901234567890123456789012123456789012345678901234567890121234
PRACTICE TEST - 1
12345678901234567890123456789012123456789012345678901234567890121234567890123456789012345678901212345678901234567890123456789012123
1234567890123456789012345678901212345678901234567890123456789012123456789012345678901234567890
123456789012345678901234567890121234567890123456789012345678901212
12345678901234567890123456789012123456789012345678901234567890121234567890123456789012345678901212345678901234567890123
1234567890123456789012345678901212345678901234567890123456789012123456789012345678901234567890121234567890123
1234567890123456789012345678901212345678901234567890123456789012123456789012345678901234567890121234
12345678901234567890123456789012123456789012345678901234567890121234567890123456789012345678901212345678901234567890123456789012123
1234567890123456789012345678901212345678901234567890123456789012123456789012345678901234567890
12345678901234567890123456789012123456789012345678901234567890121234567890123456789012345678901212345678901234567890123
1234567890123456789012345678901212345678901234567890123456789012123456789012345678901234567890121234567890123
1234567890123456789012345678901212345678901234567890123456789012123456789012345678901234567890121234
123456789012345678901234567890121234567890123456789012345678901212
12345678901234567890123456789012123456789012345678901234567890121234567890123456789012345678901212345678901234567890123456789012123
1234567890123456789012345678901212345678901234567890123456789012123456789012345678901234567890
12345678901234567890123456789012123456789012345678901234567890121234567890123456789012345678901212345678901234567890123
1234567890123456789012345678901212345678901234567890123456789012123456789012345678901234567890121234567890123
1234567890123456789012345678901212345678901234567890123456789012123456789012345678901234567890121234
12345678901234567890123456789012123456789012345678901234567890121234567890123456789012345678901212345678901234567890123456789012123
1234567890123456789012345678901212345678901234567890123456789012123456789012345678901234567890

‹¬¶KÇ# (1-30) : D ãH÷Ok W=Þ|_# ‹¬=°‹¬¼ zã`Œ…Õ a#ß"³°Ø# ^¥xx Q®°iëOKÇO_?


1. 12.

Y
(1) (2) (3) (4) (1) (2) (3) (4)

M
2. 13.

E
(1) (2) (3) (4) (1) (2) (3) (4)

D
9
59
3.
14.
CA
95
(1) (2) (3) (4)

50
98
A
(1) (2) (3) (4)
4.
:8
E

(1) (2) (3) (4) 15.


h
.P
IC

5.
AD

(1) (2) (3) (4)


AB
O

(1) (2) (3) (4)


ER
CH

6. 16.
YD

(1) (2) (3) (4)


,H

(1) (2) (3) (4)


T
AM

7.
17.
H
M

(1) (2) (3) (4)


AM
IG

8. (1) (2) (3) (4)


KH
R

(1) (2) (3) (4) 18.

9. (1) (2) (3) (4)

(1) (2) (3) (4)


19.
10.
(1) (2) (3) (4)
(1) (2) (3) (4)
20.
11.

(1) (2) (3) (4)


(1) (2) (3) (4)

REASONING ABILITY (NON-VERBAL) ○ ○ ○ ○ ○ ○


210
○ ○ ○ ○ ○ ○ ○ ○ ○ ○ ○ ○ ○ ○ ○ ○ ○ ○ ○
Study Material
21. 25.

(1) (2) (3) (4)


(1) (2) (3) (4)

26.
22.
(1) (2) (3) (4)
(1) (2) (3) (4)
27.
23.
(1) (2) (3) (4)
(1) (2) (3) (4)
28.

Y
24.

M
(1) (2) (3) (4)

(1) (2) (3) (4)

E
1234567890123456789012
12345678901234567890123456789012123456789012345678901234567890121234567890123456789012345678901212345678901234567890123
12345678901234567890123

D
q=~¡}ì`ÇàH› ["Œ|°°

9
12345678901234567890123456789012123456789012345678901234567890121234567890123456789012345678901212345678901234567890123
1234567890123456789012
1234567890123456789012 12345678901234567890123
12345678901234567890123456789012123456789012345678901234567890121234567890123456789012345678901212345678901234567890123
12345678901234567890123

59
1234567890123456789012
PRACTICE TEST - 1 :
12345678901234567890123456789012123456789012345678901234567890121234567890123456789012345678901212345678901234567890123
1234567890123456789012 12345678901234567890123
12345678901234567890123456789012123456789012345678901234567890121234567890123456789012345678901212345678901234567890123
1234567890123456789012 12345678901234567890123
12345678901234567890123456789012123456789012345678901234567890121234567890123456789012345678901212345678901234567890123
12345678901234567890123
1234567890123456789012
CA
12345678901234567890123
12345678901234567890123456789012123456789012345678901234567890121234567890123456789012345678901212345678901234567890123

95
ZO^ΰH›O>è ãuƒ’°[O z=~¡ ƒì}O`ǁ LOk.

50
1. (2) 12. (4)

98
A
2. (1) ZO^ΰH›O>è 䛽_"³á„¬ô w`Ç䛽 =$`ÇëO LOk. 13. (4)
:8
3. (1) q°ye# "Œ\÷H÷ Z_È=°"³á„¬ô w`Ç䛽 =$`ÇëO LOk. 14. (4) 4=k =°OKÇ ° ㇐O`Œ…ÕÁ x WQ® ¶ Á , qQ® ` Œ=hß
E
h

4. 1,3,4 ƒç=°à…Õ z#ß =$`ÇëŌá w`Ç LOk. 2…Õ L+¬â㇐O`ÇO.


.P

(2)
IC

=¶ã`ÇO ̄^Îí =$`ÇëO ãH÷O^Î w`Ç LOk. Hê=ô# Wk 15. 1,2,4 ‹¬ÞæHê „¬O@°. 3=^³á# †Ç¶„²…˜-XH› K³@°“
AD

(3)

a#ß"³°Ø#k. †³òH›ø „¦¬=ò.


AB
O

5. 1,2,4 ƒç=°à…Õ ƒì}쁰 XH›^¥xHùH›\ ÷ =¼uö~H› 16. (3) 1,2,4 zã`Œ° ~Ë_ÈÁg°^Î #_KÍq. 3=^³á# „¬_È= hˆ×Á…Õ
ER
CH

(3)

k‰×…Õ L<Œß~ò. 3…Õ =¶ã`ÇO J\ìÁ …è=ô. Hê=ô# Wk #_KÍk.


YD

a#ß"³°Ø#k. 17. (3) 1,2,4 zã`Œ…ÕÁx „¬ä½Æ› ° PHê‰×O…Õ Zyö~q. HË_ Z䛽ø=
,H

Z`Ç°ë䛽 ZQ®~¡…è^ΰ.
T

6. 2,3,4 ƒç=°à…Õ ㄬu ƒç=°à 4 ƒìQê`Ë U~¡æ_Ok.


AM

(1)
18. 4= KÇ`Ç°~¡ã‹¬O…Õ D '-Ñ J_ÈÛw`Džè^ΰ. JO^ΰ=Á Jk
H

1= ƒç=°à =¶ã`ÇO =ü_È° ƒìQê`Ë U~¡æ_È°`Ç°Ok. (4)


M

qa#ß"³°Ø# ƒç=°à.
AM

Hê=ô# W^Í a#ß"³°Ø#k.


IG

1,3,4 ƒç=°à° <́̄á #_KÍ <Œ°Q®° Hêˆ×¤ [O`Ç°=ô°. 19. (3) 1,2,4 ƒç=°à° ‹¬=¼k‰×…Õ u~¡°Q®°`Ƕ U~¡æ_È°`Ç°<Œß~ò.
KH

7. (2)
3= ƒç=°à qa#ßOQê LOk.
2= ƒç=°à z°H› („¬H÷Æ) - Qêe…Õ ZQ®°~¡Q®k. Hê=ô#
R

20. (1) 2,3,4 zã`Œ…ÕÁ ~ï O_È° `³Á =$`Œëä›½ Z^ΰ~¡°Qê, =°^¼Î …Õ


W^Í a#ß"³°Ø#k.
#Á=$`ÇëO LOk. 1= ƒç=°à…Õ ï~O_È° `³Á =$`Œë
8. (2) 1,3,4 ƒç=°à° ƒ°’ *ì`Ë U~¡æ_È°`Ç°<Œß~ò. 2= ƒç=°à
=°^¼Î HêH›, XH› `³Á =$`ÇOë Z^ΰ~¡°Qê<Í #Á=$`ÇOë LOk.
=$`ÇëO. (ƒ’°*쁰 …è=ô). Hê=ô# W^Í a#ß"³°Ø#k.
JO^ΰ=Á 1=k qa#ß"³°Ø# ƒç=°à.
9. (3) 3= ƒç=°à H›=ò. 1,2,4 ƒç=°à° =O@ ª=¶#°°.
21. (1) 2,3,4 ƒç=°à…ÕÁ n~¡ÉKÇ`Ç°~¡ã‹¬O…Õx XH› ‹¬Q®ƒìQ®O XH›
Hê=ô# (3) a#ß"³°Ø#k.
KÇ`Ç°~¡°Ä[OQê, XH› ãuƒ’°[OQê q_ÈQù@“|_¨Û~ò. 1=
10. (1) E,M,W ° 4 w`ǁ`Ë U~¡æ_È°`Ç°<Œß~ò. Hêx 'K' 3
ƒç=°à…Õ ï ~ O_È ° ãuƒ ’ ° *쁰Qê q_È Q ù@“ | _¨Û ~ ò.
w`ǁ`Ë U~¡æ_È°`Ç°<Œß~ò. Hê=ô# W^Í a#ß"³°Ø#k.
JO^ΰ=Á 1=k a#ß"³°Ø# ƒç=°à.
11. (1) 2,3,4 ƒç=°à…ÕÁ J_ÈwÛ `ǁ° ãuƒ°’ *쁰 |†Ç°@ L<Œß~ò. 22. (4) 4 `DŽ¬æ q°ye#q „¬ä›Æ½°.

REASONING ABILITY (NON-VERBAL) ○ ○ ○ ○ ○ ○


211
○ ○ ○ ○ ○ ○ ○ ○ ○ ○ ○ ○ ○ ○ ○ ○ ○ ○ ○
Study Material
23. (4) 4 `DŽ¬æ q°ye# ‹¬=°ª¼ zã`Œhß w`ǁ`Ë xiàOKÇ 26. (4) 1,2,3 ‹¬=°ª¼ „¬\ì…Õ ãuƒ°’ *ì †³òH›ø XH› j~¡Â=ò
|_¨Û~ò. =°i†Çò XH› ƒ°’ [=ò =$`Œëxß `Œä›½`Ç°<Œß~ò. Hêh 4…Õ
24. (4) 1, 2, 3…Õ =$`Œëxß '4Ñ ‹¬=¶# ƒìQꁰ K͆°Ç |_¨Û~ò. ãuƒ ’ ° [=ò †³ ò H› ø j~¡  =ò° =¶ã`Ç " Í ° =$`Œë x ß
`Œä›½`Ç°<Œß~ò.
Hêx '4Ñ=k a#ß"³°Ø#k.
27. (4) 1,2,3 ‹¬=°ª¼ „¬\ì…Õ z#ß =°i†Çò ̄^Îí ƒç=°à°
25. (3) 3 `DŽ¬æ q°ye# ‹¬=°ª¼ zã`Œ…ÕÁ z#ß ƒç=°à ̄^Îí
‹¬=¶#OQê _"³á_£ K͆ǰ|_¨Û~ò. Hêh 4…Õ J…ì …è^ΰ.
ƒç=°à…Õ„¬ LOKÇ|_Ok. Hêh 3…Õ z#ß ƒç=°à ̄^Îí
28. (4) 4 q°#‚¬ð~òOz q°ye# Jxß „¬\쁰 ‹¬=¼k‰×…Õ
ƒç=°à#° YO_‹¬¶ë LOKÇ|_Ok. u~¡°Q®° `Ç°<Œß~ò.
123456789012345678901234567890121234567890123456789012345678901212
12345678901234567890123456789012123456789012345678901234567890121234567890123456789012345678901212345678901234567890123456789012123
1234567890123456789012345678901212345678901234567890123456789012123456789012345678901234567890
12345678901234567890123456789012123456789012345678901234567890121234567890123456789012345678901212345678901234567890123
1234567890123456789012345678901212345678901234567890123456789012123456789012345678901234567890121234567890123
1234567890123456789012345678901212345678901234567890123456789012123456789012345678901234567890121234
123456789012345678901234567890121234567890123456789012345678901212
12345678901234567890123456789012123456789012345678901234567890121234567890123456789012345678901212345678901234567890123456789012123
1234567890123456789012345678901212345678901234567890123456789012123456789012345678901234567890
12345678901234567890123456789012123456789012345678901234567890121234567890123456789012345678901212345678901234567890123
1234567890123456789012345678901212345678901234567890123456789012123456789012345678901234567890121234567890123
1234567890123456789012345678901212345678901234567890123456789012123456789012345678901234567890121234
123456789012345678901234567890121234567890123456789012345678901212

Y
12345678901234567890123456789012123456789012345678901234567890121234567890123456789012345678901212345678901234567890123456789012123
1234567890123456789012345678901212345678901234567890123456789012123456789012345678901234567890
12345678901234567890123456789012123456789012345678901234567890121234567890123456789012345678901212345678901234567890123
1234567890123456789012345678901212345678901234567890123456789012123456789012345678901234567890121234567890123
1234567890123456789012345678901212345678901234567890123456789012123456789012345678901234567890121234
PRACTICE TEST - 2
12345678901234567890123456789012123456789012345678901234567890121234567890123456789012345678901212345678901234567890123456789012123
1234567890123456789012345678901212345678901234567890123456789012123456789012345678901234567890
123456789012345678901234567890121234567890123456789012345678901212
12345678901234567890123456789012123456789012345678901234567890121234567890123456789012345678901212345678901234567890123
1234567890123456789012345678901212345678901234567890123456789012123456789012345678901234567890121234567890123
1234567890123456789012345678901212345678901234567890123456789012123456789012345678901234567890121234
12345678901234567890123456789012123456789012345678901234567890121234567890123456789012345678901212345678901234567890123456789012123
1234567890123456789012345678901212345678901234567890123456789012123456789012345678901234567890
12345678901234567890123456789012123456789012345678901234567890121234567890123456789012345678901212345678901234567890123
1234567890123456789012345678901212345678901234567890123456789012123456789012345678901234567890121234567890123
1234567890123456789012345678901212345678901234567890123456789012123456789012345678901234567890121234
123456789012345678901234567890121234567890123456789012345678901212
12345678901234567890123456789012123456789012345678901234567890121234567890123456789012345678901212345678901234567890123456789012123
1234567890123456789012345678901212345678901234567890123456789012123456789012345678901234567890
12345678901234567890123456789012123456789012345678901234567890121234567890123456789012345678901212345678901234567890123
1234567890123456789012345678901212345678901234567890123456789012123456789012345678901234567890121234567890123
1234567890123456789012345678901212345678901234567890123456789012123456789012345678901234567890121234
12345678901234567890123456789012123456789012345678901234567890121234567890123456789012345678901212345678901234567890123456789012123
1234567890123456789012345678901212345678901234567890123456789012123456789012345678901234567890
1234567890123456789012345678901212345678901234567890123456789012123456789012345678901234567890121234567890123
1234567890123456789012345678901212345678901234567890123456789012123456789012345678901234567890121234
12345678901234567890123456789012123456789012345678901234567890121234567890123456789012345678901212345678901234567890123456789012123
1234567890123456789012345678901212345678901234567890123456789012123456789012345678901234567890

M
‹¬¶KÇ# (1-30) : D ãH÷Ok W=Þ|_# ‹¬=°‹¬¼ zã`Œ…Õ a#ß"³°Ø# ^¥xx Q®°iëOKÇO_?

E
D
9
1.

59
8.

CA
95
(1) (2) (3) (4) (1) (2) (3) (4) (5)

50
98
A
2. 9.
:8
E
h

(1) (2) (3) (4) (1) (2) (3) (4) (5)


.P
IC

3.
AD

10.
AB
O

(1) (2) (3) (4)


ER
CH

(1) (2) (3) (4) (5)


4.
YD

11.
,H

(1) (2) (3) (4)


T
AM
H

(1) (2) (3) (4) (5)


M

5.
AM
IG

12.
KH

(1) (2) (3) (4) (5)


R

(1) (2) (3) (4) (5)


6.
13.
(1) (2) (3) (4) (5)
(1) (2) (3) (4) (5)
7.
14.
(1) (2) (3) (4) (5)
(1) (2) (3) (4) (5)

REASONING ABILITY (NON-VERBAL) ○ ○ ○ ○ ○ ○


212
○ ○ ○ ○ ○ ○ ○ ○ ○ ○ ○ ○ ○ ○ ○ ○ ○ ○ ○
Study Material
15. 23.

(1) (2) (3) (4) (1) (2) (3) (4)

16. 24.

(1) (2) (3) (4)


(1) (2) (3) (4)

17. 25.

Y
(1) (2) (3) (4) (1) (2) (3) (4)

M
18. 26.

E
D
9
(1) (2) (3) (4)

59
(1) (2) (3) (4)

CA
95
19. 27.

50
98
A
(1) (2) (3) (4)
(2) (3) (4)
:8
(1)
E

28.
h

20.
.P
IC
AD

(1) (2) (3) (4)


(1) (2) (3) (4)
AB
O

29.
ER
CH

21.
YD

(1) (2) (3) (4)


,H

(1) (2) (3) (4)


T
AM

30.
H

22.
M
AM
IG

(1) (2) (3) (4)


(1) (2) (3) (4)
KH

1234567890123456789012
12345678901234567890123456789012123456789012345678901234567890121234567890123456789012345678901212345678901234567890123
12345678901234567890123
R

q=~¡}ì`ÇàH› ["Œ|°°
12345678901234567890123456789012123456789012345678901234567890121234567890123456789012345678901212345678901234567890123
1234567890123456789012 12345678901234567890123
12345678901234567890123456789012123456789012345678901234567890121234567890123456789012345678901212345678901234567890123
1234567890123456789012
1234567890123456789012 12345678901234567890123
12345678901234567890123456789012123456789012345678901234567890121234567890123456789012345678901212345678901234567890123
12345678901234567890123
1234567890123456789012 PRACTICE TEST - 2 :
12345678901234567890123456789012123456789012345678901234567890121234567890123456789012345678901212345678901234567890123
12345678901234567890123456789012123456789012345678901234567890121234567890123456789012345678901212345678901234567890123
1234567890123456789012
1234567890123456789012
12345678901234567890123
12345678901234567890123
12345678901234567890123456789012123456789012345678901234567890121234567890123456789012345678901212345678901234567890123
12345678901234567890123

1. (4) ZO^ΰH›O>è XHùøH›ø PHê~¡O ï~O_È°ª~¡°Á w†Ç°_ÈO 6. (3) Wzó# ƒç=°à…Õ JxßO\÷…Õx öHOã^ÎO =^Îí U Q®°~¡°ë°
[iyOk. …è=ô. D ƒç=°à…Õ XH›ø (3)…Õ `DŽ¬æ q°ye#
2. (4) q°ye# "Œ\÷…Õ #°„¬ô ƒìQ®O 4= =O`Ç° LOk. "Œ@xßO\÷…Õ Q®°~¡°ë° XH›^¥x `Ç~ŒÞ`Ç XH›\ ÷ =ªë~ò.
3. (4) 'N' =ü_È° ö~MìYO_¨KÍ U~¡æ_Ok. Hê=ô# (3) a#ß"³°Ø#k.
4. (3) q°ye#q Jhß ‹¬=¼k‰×…Õ u~¡°Q®°KÇ°#ßq. 7. (3) Wzó# ƒç=°à…Õx PHê~Œ…Õ XH›ø (3)…Õ `DŽ¬æ
5. (5) Wzó# ƒç=°à…Õx XH› ø (5)…Õ `Ç „ ¬ æ q°ye# q°ye# "Œ@xßO\÷…Õ ‹¬Q®O P䛽 Q®°~¡°ë H›O>è ö~Y
"Œ@xßO\÷…Õ ï~O_È° ƒì}„¬ô Q®°~¡°ë° 䛽_"³á„¬ôQê ‹¬OY¼ XH›\ ÷ Z䛽ø=Qê LO@°<Œß~ò. Hê=ô# Wk
L<Œß~ò. Hê=ô# Wk a#ß"³°Ø#q. a#ß"³°Ø#k.

REASONING ABILITY (NON-VERBAL) ○ ○ ○ ○ ○ ○


213
○ ○ ○ ○ ○ ○ ○ ○ ○ ○ ○ ○ ○ ○ ○ ○ ○ ○ ○
Study Material
8. (4) Wzó# ƒç=°à…Õ XH› ø (4)…Õ `Ç „ ¬ æ q°ye# 16. (2) 1,3,4 ƒç=°à° ‹¬O=$`Ç „¬\쁰. 1= ƒç=°à J‹¬O=$`Ç
"Œ@xßO\÷ … Õ ï ~ O_ O \÷ … Õ XH› …ÿ á < £ KÇ ° H› ø KÍ „¬@O.
U~¡æ_È°`Ç°Ok. Hê=ô# '4Ñ a#ß"³°Ø#k. 17. (1) 2,3,4 ƒç=°à° ‹¬O=$`Ç „¬\쁰. 1= ƒç=°à J‹¬O=$`Ç
9. (2) Wzó# ƒç=°à…Õ XH› ø (2)…Õ `Ç „ ¬ æ q°ye# „¬@O.
"Œ@xßO\÷…Õ XH› „¬îië KÇ`Ç°~¡ã‹¬ PHê~¡O, =ü_È°
18. (3) 1,2,4 ƒç=°à…ÕÁ ̄^Îí ƒç=°à, z#ß ƒç=°à XöH k‰×…Õ
ƒ’°*쁰 Q®ey# XH› H›„¬C 'U' PHê~¡O Q® =üH›O
L<Œß~ò. 3= ƒç=°à…Õ =¶ã`ÇO ̄^Îí ƒç=°à䛽 =¼uö~H›
=°i†Çò ï~O_È° ƒ’°*쁰 H›ey# PHê~¡„¬ô =üH›O,
k‰×…Õ z#ß ƒç=°à LOk.
XH› ‹¬~¡ˆ×ö~Y H›=ô. Hê=ô# Wk a#ß"³°Ø#k.
19. (3) 1,2,4 =‹¬°ë=ô° ‰×s~¡ J=†Ç°"Œä›½ `ÇyeOz,
10. (3) Wzó# ƒç=°à…Õ XH› ø (3)…Õ `Ç „ ¬ æ q°ye#
"Œ@xßO\÷…Õx XH› xiœ+¬“ =üH›=ò ^¥x †³òH›ø ^ÎiO„¬KÍÀ‹q, Qù_È°Q®° J…ì Hê^ΰ.

Y
J^Μ„¬ô ㄬuaO|O° H›ey L<Œß~ò. Hê=ô# Wk 20. (4) LeÁ, Hêï~\˜, =òÁOy ^ΰO„¬ä›€~¡°. "Í~¡°‰Ü#Q® Hê^ΰ.

M
a#ß"³°Ø#k. 21. (1) 2,3,4 - w`ǁ`Ë U~¡æ_Íq. 1= ƒç=°à䛽 w`DžèHêH›,

E
11. (3) Wzó# ƒç=°à…Õ XH› ø (3)…Õ `Ç „ ¬ æ q°ye# =O„¬ô 䛀_¨ Hê"Œe.

D
9
"Œ@xßO\÷…Õ qq^Î ~¡Hê =üHꁰ XH› =~¡°‹¬…Õ

59
22. (1) 2,3,4 ƒç=°à…Õ M - W…ì w`Ç …Õ„¬…è 'M' PHê~¡O
=ªë~ò. JO>è 3 '«Ñ Q®~¡°ë°, 4 'OÑ Q®°~¡°ë° =°i†Çò
CA
95
LOk. 1= ƒç=°à…Õ 'w' PHê~¡OQê LOk.
5 c-PHê~¡„¬ô =üHꁰ =°i†Çò ƒç=°à…Õ Q® 'x'

50
23. (1) 2,3,4 ƒç=°à…Õ ā° Z^ΰï~^ΰ~¡°Qê L<Œß~ò. 1=
Q®°~¡°ë° Hê=ô# W^Í a#ß"³°Ø#k. 98
A
ƒç=°à…Õ XH›ø ㄬH›øöH ā° L<Œß~ò.
:8
12. (1) Wzó# ƒç=°à…Õ XH› ø (1)…Õ `Ç „ ¬ æ q°ye#
E

1,3,4 ƒç=°à…Õ XH› J~¡Ö=$`ÇëO XH› w`ÇKÍ`Ç ï~O_È°


h

24. (2)
"Œ@xßO\÷…Õx PHê~Œ° XH›^¥xHùH›ø\÷ ハ’=°}O
.P
IC

K³O^ΰ`Ç°<Œß~ò. Hê=ô# Wk a#ß"³°Ø#k. ‹¬=°ƒìQꁰ K͆ǰ|_È°`Ç°Ok.


AD

Wzó# ƒç=°à…Õ XH› ø (2)…Õ `Ç „ ¬ æ q°ye# 25. 1,3,4 zã`Œ…ÕÁ #ÁKÇ°H›ø z#ß ãuƒ’°*ì…ÕÁ LOk. 2 …Õ
AB
O

(2)
13. (2)

"Œ@xßO\÷…Õx PHê~Œ° XH›^¥xHùH›\ ÷ ハ’=°}O z#ß ãuƒ’°[O |†Ç°@ LOk.


ER
CH

K³O^ΰ`Œ~ò. Hê=ô# Wk a#ß"³°Ø#k. 1,2,3 zã`Œ…ÕÁ J_ÈÛw`Ç 1/4 ƒìQ®O…Õ LOk. 4…Õ
YD

26. (4)

Wzó# ƒç=°à…Õ XH› ø (4)…Õ `Ç „ ¬ æ q°ye# ‹¬Q®O…Õ J_ÈÛw`Ç LOk.


,H

14. (4)
T

"Œ@xßO\÷…Õx ï~O_O\÷…Õ XH› ƒì}O öHOã^ÎO "³á„¬ôQê


AM

27. (4) 1,2,3 zã`Œ…ÕÁ 'Y' PHê~¡O XöH ̋áA…Õ L<Œß~ò.


H

LO@°Ok. Hê=ô# Wk a#ß"³°Ø#k. 4…Õ z#ßk LOk.


M
AM
IG

15. (3) ƒç=°à =°^Î¼…Õ w‹²# J_ÈÛ w`Ç䛽 z=~¡ L#ß w`ǁ° 28. (3) 1,2,4 zã`Œ…ÕÁ Z^ΰï~^ΰ~¡° ƒ’°*쁰 ‹¬=¶#OQê
KH

1,2,4 ƒç=°à…ÕÁ `ǁ"³á„¬ô L<Œß~ò. 3= ƒç=°à…Õ L<Œß~ò. 3…Õ Z^ΰï~^ΰ~¡° ƒ’°*쁰 ‹¬=¶#OQê …è=ô.
R

`ËH›"³á„¬ô䛽 D z=i w`ǁ° W\ìÁ L<Œß~ò. . 29. (3) 1,2,4 zã`Œ° ‹¬O=$`Ç „¬\쁰. 3=k J‹¬O=$`Ç „¬@O.
30. (3) 1,2,4 ‰×s~Œxß H›„²æ =ôOKÍq.

REASONING ABILITY (NON-VERBAL) ○ ○ ○ ○ ○ ○


214
○ ○ ○ ○ ○ ○ ○ ○ ○ ○ ○ ○ ○ ○ ○ ○ ○ ○ ○
Study Material
123456789012345678901234567890121234567890123456789012345678901212
12345678901234567890123456789012123456789012345678901234567890121234567890123456789012345678901212345678901234567890123456789012123
123456789012345678901234567890121234567890123456789012345678901212345678901234567890123
12345678901234567890123456789012123456789012345678901234567890121234567890123456789012345678901212345678901234567890123
1234567890123456789012345678901212345678901234567890123456789012123456789012345678901234567890121234567890123
1234567890123456789012345678901212345678901234567890123456789012123456789012345678901234567890121234
123456789012345678901234567890121234567890123456789012345678901212
12345678901234567890123456789012123456789012345678901234567890121234567890123456789012345678901212345678901234567890123456789012123
123456789012345678901234567890121234567890123456789012345678901212345678901234567890123
12345678901234567890123456789012123456789012345678901234567890121234567890123456789012345678901212345678901234567890123
1234567890123456789012345678901212345678901234567890123456789012123456789012345678901234567890121234567890123
1234567890123456789012345678901212345678901234567890123456789012123456789012345678901234567890121234
12345678901234567890123456789012123456789012345678901234567890121234567890123456789012345678901212345678901234567890123456789012123
123456789012345678901234567890121234567890123456789012345678901212345678901234567890123
123456789012345678901234567890121234567890123456789012345678901212
12345678901234567890123456789012123456789012345678901234567890121234567890123456789012345678901212345678901234567890123
1234567890123456789012345678901212345678901234567890123456789012123456789012345678901234567890121234567890123
1234567890123456789012345678901212345678901234567890123456789012123456789012345678901234567890121234
PREVIOUS BITS
12345678901234567890123456789012123456789012345678901234567890121234567890123456789012345678901212345678901234567890123456789012123
123456789012345678901234567890121234567890123456789012345678901212345678901234567890123
123456789012345678901234567890121234567890123456789012345678901212
12345678901234567890123456789012123456789012345678901234567890121234567890123456789012345678901212345678901234567890123
1234567890123456789012345678901212345678901234567890123456789012123456789012345678901234567890121234567890123
1234567890123456789012345678901212345678901234567890123456789012123456789012345678901234567890121234
12345678901234567890123456789012123456789012345678901234567890121234567890123456789012345678901212345678901234567890123456789012123
123456789012345678901234567890121234567890123456789012345678901212345678901234567890123
12345678901234567890123456789012123456789012345678901234567890121234567890123456789012345678901212345678901234567890123
1234567890123456789012345678901212345678901234567890123456789012123456789012345678901234567890121234567890123
1234567890123456789012345678901212345678901234567890123456789012123456789012345678901234567890121234
12345678901234567890123456789012123456789012345678901234567890121234567890123456789012345678901212345678901234567890123456789012123
123456789012345678901234567890121234567890123456789012345678901212345678901234567890123
123456789012345678901234567890121234567890123456789012345678901212
12345678901234567890123456789012123456789012345678901234567890121234567890123456789012345678901212345678901234567890123456789012123
123456789012345678901234567890121234567890123456789012345678901212345678901234567890123
12345678901234567890123456789012123456789012345678901234567890121234567890123456789012345678901212345678901234567890123
1234567890123456789012345678901212345678901234567890123456789012123456789012345678901234567890121234567890123
1234567890123456789012345678901212345678901234567890123456789012123456789012345678901234567890121234
12345678901234567890123456789012123456789012345678901234567890121234567890123456789012345678901212345678901234567890123456789012123
123456789012345678901234567890121234567890123456789012345678901212345678901234567890123

1. ãH÷Ok "Œ\÷…Õ q°Q®`Œ "Œ\÷`Ë ‹¬i ‡éxk Uk? (3) #°Oz (4) =~¡ä›½ Q® ㄬ‰×߁…Õ g°ä›½ <Œ°Q®°
(AP. Panchyat Secretar y Screening -2016) „¬@=ò° (1), (2), (3), (4) W=Þ|_<Œ~ò. "Œ\÷…Õ
XH›\ ÷ q°#‚¬ð q°ye#q Jhß U^Ë XH› q^ÎOQê XöH
1) 2) 3) 4) ª~¡¶‡¼xß H›ey LO\ì~ò. ^Î`ëÇ ‹¬q°u…Õ a#ßOQê L#ß
„¬\ìxß ZO„²H› K͋¬°Hùx, SzóHꁰ (1) #°O_ (4)…Õ
2. Wzó# <Œ°Q®° „¬\ì…Õ W`Ç~¡ =ü_È° „¬\ìä›½ a#ßOQê ^¥x ‹¬OY¼#° ‹¬i†³Ø°# ‹¬=¶^¥#OQê ZO„²H› K͋¬°HËO_.
L#ß „¬\ìxß Z#°ßHù#°=ò. (A.P.S.I. Mains -2016)
(A.P.Constable Prelims -2016)

4.

Y
M
1) 2) 3) 4) 1) 2) 3) 4)
3. ãH÷Ok ㄬ‰×߁…Õ Wzó# (1) #°Oz (4) Q® „¬@=ò…Õ

E
5.
=ü_O\÷…Õ Q® =üH›O (i), =üH›O (ii) `Ë XH›

D
9
59
xiœ+¬“ ‹¬O|O^¥xß H›ey LO@°Ok. =üH›O (i)H÷
CA
95
=üH›O (ii)`Ë J…ìO\÷ ‹¬|O^¥xß H›ey LO_Èx 1) 2) 3) 4)
„¬@O#° H›#°Qùx ‹¬=¶^¥#OQê ~Œ†Ç°O_.

50
98
A
(A.P.S.I. Mains -2016)
:8
E
h
.P
IC

1) 2) 3) 4)
AD
AB
O

12345678901234567890123456789012123456789012345678901234567890121234567890123456789012345678901212345678901234567890123
1234567890123456789012 123456789012345678901234

q=~¡}ì`ÇàH› ["Œ|°°
12345678901234567890123456789012123456789012345678901234567890121234567890123456789012345678901212345678901234567890123
1234567890123456789012 123456789012345678901234
1234567890123456789012 123456789012345678901234
12345678901234567890123456789012123456789012345678901234567890121234567890123456789012345678901212345678901234567890123
1234567890123456789012
1234567890123456789012 PREVIOUS BITS : 123456789012345678901234
12345678901234567890123456789012123456789012345678901234567890121234567890123456789012345678901212345678901234567890123
123456789012345678901234
12345678901234567890123456789012123456789012345678901234567890121234567890123456789012345678901212345678901234567890123
ER
CH

1234567890123456789012
1234567890123456789012 123456789012345678901234
12345678901234567890123456789012123456789012345678901234567890121234567890123456789012345678901212345678901234567890123
123456789012345678901234
12345678901234567890123456789012123456789012345678901234567890121234567890123456789012345678901212345678901234567890123
YD

1. (3) 1, 2, 4 …Õx JO`Ç~¡, ƒì‚¬ì¼=òQê "Íö~Þ~¡° PHê~Œ° 4. (1) XH›ø 1= ƒç=°à…Õ `Ç„æ¬ q°ye# "Œ@xßO\÷…Õ "³°„¬e
,H

L<Œß~ò. Hêx '3Ñ= PHê~¡O nxH÷ q~¡°^ΜO Hê=ô# L#ß PHê~Œ †³òH›ø ƒ°’ *쁰 ̄~¡°Q®°^΁ / `Ç~°¡ Q®°^΁Qê
T
AM

W^Í a#ß"³°Ø#k.
H

H›e¾ L<Œß~ò. Hê=ô# 1=k a#ß"³°Ø#k.


M

2. XH›ø '4Ñ= ƒç=°à XH›ø>è ‹¬=¶# ƒìQꁰQê qƒ’lO„¬


AM

(4)
=$`Œë…Õx XH› ‡=ôƒìQêxß ï~O_È°Qê qƒ’lOz XH›
IG

5. (2)
|_Ok. Hê=ô# '4Ñ XH›ø>è a#ß"³°Ø#k.
KH

‹¬=¶# (   ) ƒìQ®O…Õ KÇ°H›ø LOKŒ~¡°. Jk öH=O


3. XH›ø 2= ƒç=°à…Õ `Ç„æ¬ q°ye# ƒç=°àxßO\÷…Õ (i)=
R

(2)
2= ƒç=°à…Õ =¶ã`Ç"Í°. Hê=ô# Hê=ô# W^Í a#ß"³°Ø#k.
ƒç=°à †³òH›ø PHê~Œxß `ǁãH÷O^ΰ°Qê ãu„²æ ^¥x†³òH›ø
J^΄œ ô¬ ㄬuaO|O`Ë H›°„¬Qê =KÇ°ó#q (ii) = ƒç=°à…Õx q°ye#k nxH÷ q~¡°^ΜO.
PHê~¡=ò. Hê=ô# (2)= ƒç=°à XH›ø>è a#ß"³°Ø#k.

RÿRÿR

REASONING ABILITY (NON-VERBAL) ○ ○ ○ ○ ○ ○


215
○ ○ ○ ○ ○ ○ ○ ○ ○ ○ ○ ○ ○ ○ ○ ○ ○ ○ ○
Study Material
J‹¬O„¬î~¡â zã`Œ#° „¬îiOKÇ°@
4 (Completion of Incomplete Figures)

=òMì¼O‰§°...
G Hê=‹²# zã`Ç ƒìQêxß H›#°Qù#°@䛽 ‹¬°°"³á# „¬^Μu U=°#Qê, =òO^ΰQê Mìm ㄬ^͉×O…Õ Z…ìO\÷ „¬@O =‹¬°ëO^Ë
q°ye# „¬\ìxß Q®=°xOz ̄xž…˜`Ë ~¡„¦¹Qê „¬îiëK͋² Wzó# P„¬Â<£#° „¬ijeOz# ‹¬=¶^¥#O ‹¬°ƒ’OQê Q®°iëOKÇ=KÇ°ó#°.
G D qƒìQ®O…Õ J‹¬O„¬î~¡âOQê =ô#ß XH› zã`ÇO W=Þ|_È°`Ç°Ok. =°i†Çò J‹¬O„¬î~¡âOQê =ô#ß ƒìQêxß ‹¬O„¬î~¡âO K͆Çò@䛽

Y
<Œ°Q®° P„¬Â<
£ ° W=Þ|_È`Œ~ò. "Œ\÷…ÕÁ XH›ø>è ‹¬i†³°Ø # ‹¬O„¬î~¡â zã`Œxß W‹¬°O
ë k. P ‹¬i†³°Ø # ƒìQêxß Q®°iëOz ‹¬=¶^¥#O
W"ŒÞe.

M
=¶ki ㄬ‰×߁°

E
D
9
59
1. Wzó# <Œ°Q®° ㄬ`Œ¼=¶ß†Ç¶ #°O_ ‹¬ï~á# ^¥xx 3. D ãH÷Ok „¬@O…Õx J‹¬O„¬î~¡â ƒìQêxß „¬îiOKÇO_.
CA
95
ZO„²H› K͋² 'X' zã`Ç=ò…Õx „¬^Μux „¬îië K͆ǰO_.

50
ㄬ‰×ß „¬@O
98
A
‹¬=¶^¥# „¬\쁰
:8
E
h
.P
IC

(X) (1) (2) (3) (4)


AD

ª^Î#: zã`Ç=ò (2) „¬^Μux „¬îië K͆ǰO_. ZO^ΰH›#Qê 'X'


AB
O

zã`Ç=ò †³òH›ø Mìm ªÖ<Œxß „¬îi‹¬°ëOk. H›#°H›, ª^Î#:


ER

‹¬=¶^¥#=ò (2) J=ô`Ç°Ok.


CH

„¬@O =°^Î¼…Õ =$`ÇëO „¬îië Hê"Œe. KÇ`Ç°~¡°Ä[O †³òH›ø


YD

ƒ’°[O „¬îië Hê"Œe. ö~MìYO_ÈO „¬îië Hê"Œe =°i†Çò


,H

‹¬ï~á# ªÖ<Œ…Õ ƒì}O Q®°~¡°ë° 䛽_"³á„¬ô䛽 =ôO_¨e.


T
AM
H

2. D ãH÷Ok 1, 2, 3, 4 J#° zã`Œ…Õ U zã`Ç"³°Ø`Í 'X' 4. D ãH÷O^Î W=Þ|_# J‹¬O„¬î~¡â „¬@O…Õx J‹¬O„¬î~¡â
M
AM

zã`Ç=ò †³òH›ø Mìm ‹¬…


Ö ìxß ƒs
’ K
ë ‹
Í ,² „¬^u
œÎ x ‹¬O„¬î~¡=
â ò ƒìQêxß „¬îiOKÇO_.
IG

K͋¬°ëO^Ë ^¥xx ZO„²H› K͆Ƕe.


KH

ㄬ‰×ß „¬@O
R

‹¬=¶^¥# „¬\쁰

(X) (1) (2) (3) (4)


ª^Î#: Jxß ƒìQê…Õ =$`ÇëO…Õ„¬ ãuƒ’°[O =ô#ßk. ãuƒ’°[O
ª^Î#: '2Ñ zã`Ç=ò ‹¬æ+¬“OQê „¬^Μux ‹¬O„¬î~¡âO K͋¬°ëOk. ãH÷O^Î䛽, ̄áH÷ =°i†Çò 䛽_"³á„¬ô䛽 =ô#ßk. q°ye#k
ZO^ΰH›#Qê Jq 'X' zã`Ç=ò †³òH›ø Mìm ‹¬Ö=ò…Õ Z_È=°"³á„¬ô䛽. Hê=ô# ‹¬O„¬î~¡â zã`ÇO ‡ÚO^ΰ@䛽
‹¬i‡é`Ç°Ok. H›#°H› ‹¬=¶^¥#=ò '2Ñ J=ô`Ç°Ok.
Hê=‹²# ƒìQ®O

REASONING ABILITY (NON-VERBAL) ○ ○ ○ ○ ○ ○


216
○ ○ ○ ○ ○ ○ ○ ○ ○ ○ ○ ○ ○ ○ ○ ○ ○ ○ ○
Study Material
123456789012345678901234567890121234567890123456789012345678901212
12345678901234567890123456789012123456789012345678901234567890121234567890123456789012345678901212345678901234567890123
1234567890123456789012345678901212345678901234567890123456789012123456789012345678901234567890121234567890123
1234567890123456789012345678901212345678901234567890123456789012123456789012345678901234567890121234
12345678901234567890123456789012123456789012345678901234567890121234567890123456789012345678901212345678901234567890123456789012123
1234567890123456789012345678901212345678901234567890123456789012123456789012345678901234567890
12345678901234567890123456789012123456789012345678901234567890121234567890123456789012345678901212345678901234567890123
1234567890123456789012345678901212345678901234567890123456789012123456789012345678901234567890121234567890123
1234567890123456789012345678901212345678901234567890123456789012123456789012345678901234567890121234
123456789012345678901234567890121234567890123456789012345678901212
12345678901234567890123456789012123456789012345678901234567890121234567890123456789012345678901212345678901234567890123456789012123
1234567890123456789012345678901212345678901234567890123456789012123456789012345678901234567890
12345678901234567890123456789012123456789012345678901234567890121234567890123456789012345678901212345678901234567890123
1234567890123456789012345678901212345678901234567890123456789012123456789012345678901234567890121234567890123
1234567890123456789012345678901212345678901234567890123456789012123456789012345678901234567890121234
12345678901234567890123456789012123456789012345678901234567890121234567890123456789012345678901212345678901234567890123456789012123
1234567890123456789012345678901212345678901234567890123456789012123456789012345678901234567890
123456789012345678901234567890121234567890123456789012345678901212
PRACTICE TEST - 1
12345678901234567890123456789012123456789012345678901234567890121234567890123456789012345678901212345678901234567890123
1234567890123456789012345678901212345678901234567890123456789012123456789012345678901234567890121234567890123
1234567890123456789012345678901212345678901234567890123456789012123456789012345678901234567890121234
12345678901234567890123456789012123456789012345678901234567890121234567890123456789012345678901212345678901234567890123456789012123
1234567890123456789012345678901212345678901234567890123456789012123456789012345678901234567890
123456789012345678901234567890121234567890123456789012345678901212
12345678901234567890123456789012123456789012345678901234567890121234567890123456789012345678901212345678901234567890123
1234567890123456789012345678901212345678901234567890123456789012123456789012345678901234567890121234567890123
1234567890123456789012345678901212345678901234567890123456789012123456789012345678901234567890121234
12345678901234567890123456789012123456789012345678901234567890121234567890123456789012345678901212345678901234567890123456789012123
1234567890123456789012345678901212345678901234567890123456789012123456789012345678901234567890
12345678901234567890123456789012123456789012345678901234567890121234567890123456789012345678901212345678901234567890123
1234567890123456789012345678901212345678901234567890123456789012123456789012345678901234567890121234567890123
1234567890123456789012345678901212345678901234567890123456789012123456789012345678901234567890121234
12345678901234567890123456789012123456789012345678901234567890121234567890123456789012345678901212345678901234567890123456789012123
1234567890123456789012345678901212345678901234567890123456789012123456789012345678901234567890
123456789012345678901234567890121234567890123456789012345678901212
12345678901234567890123456789012123456789012345678901234567890121234567890123456789012345678901212345678901234567890123456789012123
1234567890123456789012345678901212345678901234567890123456789012123456789012345678901234567890
12345678901234567890123456789012123456789012345678901234567890121234567890123456789012345678901212345678901234567890123
1234567890123456789012345678901212345678901234567890123456789012123456789012345678901234567890121234567890123
1234567890123456789012345678901212345678901234567890123456789012123456789012345678901234567890121234

‹¬¶KÇ# (1-20) : D ãH÷Ok W=Þ|_# ㄬ‰×߄¬@=ò 'X' …Õ '?Ñ Q®°~¡°ë ªÖ#O…Õ =KÇ°ó PHê~Œxß ã„¬H›ø<Í W=Þ|_# ‹¬=¶^¥# „¬\ì
#°O_ H›#°Qù#O_?
ㄬ‰×߄¬@=ò ‹¬=¶^¥# „¬@=ò°

1. (Navodaya Vidyalaya - 2000)

(X) (a) (b) (c) (d)

Y
M
2. (S.S.C. 2002)

E
(X) (a) (b) (c) (d)

D
9
59
CA
95
3.

50
98
A
(X) (a) (b) (c) (d)
:8
E
h
.P
IC

4. (S.S.C. 2002)
AD
AB
O

(X)
(a) (b) (c) (d)
ER
CH
YD
,H

5.
T
AM

(X) (a) (b) (c) (d)


H
M
AM
IG
KH

6. (M.B.A. 2006)
R

(X) (a) (b) (c) (d)

7.

(X) (a) (b) (c) (d)

8. (M.B.A. 2006)

(X) (a) (b) (c) (d)

REASONING ABILITY (NON-VERBAL) ○ ○ ○ ○ ○ ○


217
○ ○ ○ ○ ○ ○ ○ ○ ○ ○ ○ ○ ○ ○ ○ ○ ○ ○ ○
Study Material
9.

(X) (a) (b) (c) (d)

10.

(X) (a) (b) (c) (d)

Y
11. (R.R.B. 2004)

M
(a) (b) (c) (d)

E
(X)

D
9
59
CA
95
12. (Navodaya Vidyalaya - '05)

50
98
A
(X) (a) (b) (c) (d)
:8
E
h
.P

13.
IC
AD

(a) (b) (c) (d)


AB
O

(X)
ER
CH
YD

14. (S.S.C.2005)
,H
T
AM

(X) (a) (b) (c) (d)


H
M
AM
IG

15. (Navodaya Vidyalaya - '05)


KH
R

(X) (a) (b) (c) (d)

16.

(X) (a) (b) (c) (d)

17.

(X) (a) (b) (c) (d)

REASONING ABILITY (NON-VERBAL) ○ ○ ○ ○ ○ ○


218
○ ○ ○ ○ ○ ○ ○ ○ ○ ○ ○ ○ ○ ○ ○ ○ ○ ○ ○
Study Material
18.

(X) (a) (b) (c) (d)

19.

(X) (a) (b) (c) (d)

Y
20. (Navodaya Vidyalaya - '05)

M
(X) (a) (b) (c) (d)

E
1234567890123456789012
12345678901234567890123456789012123456789012345678901234567890121234567890123456789012345678901212345678901234567890123
12345678901234567890123

q=~¡}ì`ÇàH› ["Œ|°°
12345678901234567890123456789012123456789012345678901234567890121234567890123456789012345678901212345678901234567890123
1234567890123456789012 12345678901234567890123
1234567890123456789012
12345678901234567890123456789012123456789012345678901234567890121234567890123456789012345678901212345678901234567890123
12345678901234567890123
12345678901234567890123456789012123456789012345678901234567890121234567890123456789012345678901212345678901234567890123
1234567890123456789012 12345678901234567890123
PRACTICE TEST - 1 :
12345678901234567890123456789012123456789012345678901234567890121234567890123456789012345678901212345678901234567890123
1234567890123456789012 12345678901234567890123

D
9
12345678901234567890123456789012123456789012345678901234567890121234567890123456789012345678901212345678901234567890123
1234567890123456789012 12345678901234567890123
12345678901234567890123456789012123456789012345678901234567890121234567890123456789012345678901212345678901234567890123
1234567890123456789012 12345678901234567890123

59
CA
95
1. (b)
8. (c) 15. (a)

50
98
A
:8
E

9.
h

2.
(a)
.P

(a)
16.
IC

(d)
AD
AB
O

10.
ER

3.
CH

(a) (b)

17.
YD

(d)
,H
T
AM

4. 11. (c)
H

(b)
M

18.
AM

(c)
IG
KH

5. 12.
R

(d)
(d)

19. (a)

6. (b)
13. (c)

20. (c)

14. (d)

7. (d)

REASONING ABILITY (NON-VERBAL) ○ ○ ○ ○ ○ ○


219
○ ○ ○ ○ ○ ○ ○ ○ ○ ○ ○ ○ ○ ○ ○ ○ ○ ○ ○
Study Material
123456789012345678901234567890121234567890123456789012345678901212
12345678901234567890123456789012123456789012345678901234567890121234567890123456789012345678901212345678901234567890123
1234567890123456789012345678901212345678901234567890123456789012123456789012345678901234567890121234567890123
1234567890123456789012345678901212345678901234567890123456789012123456789012345678901234567890121234
12345678901234567890123456789012123456789012345678901234567890121234567890123456789012345678901212345678901234567890123456789012123
1234567890123456789012345678901212345678901234567890123456789012123456789012345678901234567890
12345678901234567890123456789012123456789012345678901234567890121234567890123456789012345678901212345678901234567890123
1234567890123456789012345678901212345678901234567890123456789012123456789012345678901234567890121234567890123
1234567890123456789012345678901212345678901234567890123456789012123456789012345678901234567890121234
123456789012345678901234567890121234567890123456789012345678901212
12345678901234567890123456789012123456789012345678901234567890121234567890123456789012345678901212345678901234567890123456789012123
1234567890123456789012345678901212345678901234567890123456789012123456789012345678901234567890
12345678901234567890123456789012123456789012345678901234567890121234567890123456789012345678901212345678901234567890123
1234567890123456789012345678901212345678901234567890123456789012123456789012345678901234567890121234567890123
1234567890123456789012345678901212345678901234567890123456789012123456789012345678901234567890121234
12345678901234567890123456789012123456789012345678901234567890121234567890123456789012345678901212345678901234567890123456789012123
1234567890123456789012345678901212345678901234567890123456789012123456789012345678901234567890
123456789012345678901234567890121234567890123456789012345678901212
PRACTICE TEST - 2
12345678901234567890123456789012123456789012345678901234567890121234567890123456789012345678901212345678901234567890123
1234567890123456789012345678901212345678901234567890123456789012123456789012345678901234567890121234567890123
1234567890123456789012345678901212345678901234567890123456789012123456789012345678901234567890121234
12345678901234567890123456789012123456789012345678901234567890121234567890123456789012345678901212345678901234567890123456789012123
1234567890123456789012345678901212345678901234567890123456789012123456789012345678901234567890
123456789012345678901234567890121234567890123456789012345678901212
12345678901234567890123456789012123456789012345678901234567890121234567890123456789012345678901212345678901234567890123
1234567890123456789012345678901212345678901234567890123456789012123456789012345678901234567890121234567890123
1234567890123456789012345678901212345678901234567890123456789012123456789012345678901234567890121234
12345678901234567890123456789012123456789012345678901234567890121234567890123456789012345678901212345678901234567890123456789012123
1234567890123456789012345678901212345678901234567890123456789012123456789012345678901234567890
12345678901234567890123456789012123456789012345678901234567890121234567890123456789012345678901212345678901234567890123
1234567890123456789012345678901212345678901234567890123456789012123456789012345678901234567890121234567890123
1234567890123456789012345678901212345678901234567890123456789012123456789012345678901234567890121234
12345678901234567890123456789012123456789012345678901234567890121234567890123456789012345678901212345678901234567890123456789012123
1234567890123456789012345678901212345678901234567890123456789012123456789012345678901234567890
123456789012345678901234567890121234567890123456789012345678901212
12345678901234567890123456789012123456789012345678901234567890121234567890123456789012345678901212345678901234567890123456789012123
1234567890123456789012345678901212345678901234567890123456789012123456789012345678901234567890
12345678901234567890123456789012123456789012345678901234567890121234567890123456789012345678901212345678901234567890123
1234567890123456789012345678901212345678901234567890123456789012123456789012345678901234567890121234567890123
1234567890123456789012345678901212345678901234567890123456789012123456789012345678901234567890121234

‹¬¶KÇ# (1-40) : D ãH÷Ok W=Þ|_# ㄬ‰×߄¬@=ò (X) …Õ '?Ñ Q®°~¡°ë ªÖ#O…Õ =KÇ°ó PHê~Œxß ã„¬H›ø<Í W=Þ|_# ‹¬=¶^¥#
„¬\ì#°O_ H›#°Qù#O_?
ㄬ‰×߄¬@=ò ‹¬=¶^¥# „¬@=ò°

1. (Navodaya Vidyalaya - '01)

(X) (a) (b) (c) (d)

Y
M
2.

E
(X) (a) (b) (c) (d)

D
9
59
CA
95
3.

50
98
A
(X) (a) (b) (c) (d)
:8
E
h
.P
IC

4.
AD

(a) (b) (c) (d)


AB
O

(X)
ER
CH
YD

5. (S.S.C. 2004)
,H
T
AM

(X) (a) (b) (c) (d)


H
M
AM
IG
KH

6. (R.R.B. 2004)
R

(X) (a) (b) (c) (d)

7. (M.B.A. 2006)

(X) (a) (b) (c) (d)

8. (R.R.B. 2004)

(X) (a) (b) (c) (d)

REASONING ABILITY (NON-VERBAL) ○ ○ ○ ○ ○ ○


220
○ ○ ○ ○ ○ ○ ○ ○ ○ ○ ○ ○ ○ ○ ○ ○ ○ ○ ○
Study Material
9. (R.R.B. 2004)

(X) (a) (b) (c) (d)

10.

(X) (a) (b) (c) (d)

Y
11.

M
(a) (b) (c) (d)

E
(X)

D
9
59
CA
95
12. (S.S.C. - 1995)

50
(X) (a) (b) (c) (d)

98
A
:8
E
h
.P

13.
IC
AD

(X) (a) (b) (c) (d)


AB
O
ER
CH
YD

14.
,H
T

(X) (a) (b) (c) (d)


AM
H
M
AM
IG

15.
KH
R

(X) (a) (b) (c) (d)

16.

(X) (a) (b) (c) (d)

17.

(X) (a) (b) (c) (d)

REASONING ABILITY (NON-VERBAL) ○ ○ ○ ○ ○ ○


221
○ ○ ○ ○ ○ ○ ○ ○ ○ ○ ○ ○ ○ ○ ○ ○ ○ ○ ○
Study Material
18.

(X) (a) (b) (c) (d)

19.

(X) (a) (b) (c) (d)

Y
20. (Asstt.Grade 1995)

M
E
(X) (a) (b) (c) (d)

D
9
59
CA
95
21.

50
(a) (b) (c) (d)
98
A
(X)
:8
E
h
.P

22.
IC
AD

(X) (a) (b) (c) (d)


AB
O
ER
CH
YD

23.
,H

(a) (b) (c) (d)


T

(X)
AM
H
M
AM
IG

24.
KH

(a) (b) (c) (d)


R

(X)

25. (U.D.C. - 1995)

(X) (a) (b) (c) (d)

26. (U.D.C. - 1995)

(X) (a) (b) (c) (d)

REASONING ABILITY (NON-VERBAL) ○ ○ ○ ○ ○ ○


222
○ ○ ○ ○ ○ ○ ○ ○ ○ ○ ○ ○ ○ ○ ○ ○ ○ ○ ○
Study Material
27.

(X) (a) (b) (c) (d)

28.

(X) (a) (b) (c) (d)

Y
M
29.

E
(X) (a) (b) (c) (d)

D
9
59
CA
95
30.

50
(a) (b) (c) (d)
98
(X)
A
:8
E
h
.P

31.
IC
AD

(X) (a) (b) (c) (d)


AB
O
ER
CH
YD

32.
,H
T

(X) (a) (b) (c) (d)


AM
H
M
AM
IG

33. (Asstt.Grade - 1995)


KH
R

(X) (a) (b) (c) (d)

34. (Investigators Exam. - 2005)

(X) (a) (b) (c) (d)

35.

(X) (a) (b) (c) (d)

REASONING ABILITY (NON-VERBAL) ○ ○ ○ ○ ○ ○


223
○ ○ ○ ○ ○ ○ ○ ○ ○ ○ ○ ○ ○ ○ ○ ○ ○ ○ ○
Study Material
1234567890123456789012
12345678901234567890123456789012123456789012345678901234567890121234567890123456789012345678901212345678901234567890123
12345678901234567890123
12345678901234567890123456789012123456789012345678901234567890121234567890123456789012345678901212345678901234567890123
1234567890123456789012 12345678901234567890123
12345678901234567890123456789012123456789012345678901234567890121234567890123456789012345678901212345678901234567890123
1234567890123456789012
1234567890123456789012
q=~¡}ì`ÇàH› ["Œ|°° 12345678901234567890123
12345678901234567890123456789012123456789012345678901234567890121234567890123456789012345678901212345678901234567890123
PRACTICE TEST - 2 :
12345678901234567890123
12345678901234567890123456789012123456789012345678901234567890121234567890123456789012345678901212345678901234567890123
1234567890123456789012
12345678901234567890123456789012123456789012345678901234567890121234567890123456789012345678901212345678901234567890123
1234567890123456789012
12345678901234567890123
12345678901234567890123
12345678901234567890123456789012123456789012345678901234567890121234567890123456789012345678901212345678901234567890123
1234567890123456789012 12345678901234567890123

13. (a) 25. (d)

1. (a)

14. (d)
26. (d)

2. (c)

15. (c)
27.
3.
(d)
(c)

Y
M
16. (d)
28.
4. (d)

E
(d)

D
9
59
17.
CA
95
(c)
5. (d) 29. (b)

50
98
A
18.
:8
(c)

6. 30.
E

(b)
(b)
h
.P
IC
AD

19. (d)
7.
AB
O

(c)
31. (c)
ER
CH

8.
YD

(c)

20. (d)
,H

32. (c)
T
AM

9. (b)
H

21.
M

(b)
AM
IG

33. (b)
KH

10. (c)

22.
R

(d)

34.
11.
(a)
(d)
23. (d)

12. (c) 35. (c)

24. (a)

REASONING ABILITY (NON-VERBAL) ○ ○ ○ ○ ○ ○


224
○ ○ ○ ○ ○ ○ ○ ○ ○ ○ ○ ○ ○ ○ ○ ○ ○ ○ ○
Study Material
^¥y†Çò#ß zã`Œxß H›#°Qù#°@
5 (Finding out Hidden/ Embedded Figures)

=òMì¼O‰§°...
G D qƒìQ®O…Õ J_öQ ㄬ‰×߁…Õ XH› ㄬ`ͼH› zã`Œxß Wzó ^¥xx X Qê Q®°iëOz, `Ç~¡°"Œ`Ç <Œ°Q®° zã`Œ° Wzó "Œ\÷H÷ 1,
2, 3 =°i†Çò 4 Qê Q®°~¡°ë° Wªë~¡°. W=Þ|_# X zã`ÇO…Õ 1, 2, 3, 4 …Õ U zã`ÇO ^¥y=ô#ß^Ë Q®°iëOKŒe.
G XH› Q®°~¡°ë Wzó <Œ°Q®° ㄬ`ͼH› zã`Œ° Wzó, P Q®°~¡°ë U zã`ÇO H›ey†ÇòO^Ë H›#°Qù#=°x 䛀_¨ J_È°Q®°`Œ~¡°.

Y
=¶ki ㄬ‰×߁°

M
E
2. D ãH÷O^Î W=Þ|_# <Œ°Q®° zã`Œ…Õ zã`Œxß

D
9
1. D ãH÷O^Î W=Þ|_# <Œ°Q®° zã`Œ…Õ zã`Œxß

59
H›ey†Çò#ß zã`Œxß Q®°iëOKÇO_. H›ey†Çò#ß zã`Œxß Q®°iëOKÇO_.
CA
95
50
98
A
:8
(1) (2) (3) (4)
E

(1) (2) (3) (4)


h
.P
IC

ª^Î#: „¬ @ O (3) #°O_ Q® ° ~¡ ° ë # ° ‹¬ æ +¬ “ O Qê


AD

ª^Î#: „¬@O (3) #°O_ Hê=‹²# Q®°~¡°ë#°


AB
O

Q®=°xOKÇ=KÇ°ó.
‹¬æ+¬“OQê Q®=°xOKÇ=KÇ°ó.
ER
CH

123456789012345678901234567890121234567890123456789012345678901212
12345678901234567890123456789012123456789012345678901234567890121234567890123456789012345678901212345678901234567890123456789012123
1234567890123456789012345678901212345678901234567890123456789012123456789012345678901234567890
12345678901234567890123456789012123456789012345678901234567890121234567890123456789012345678901212345678901234567890123
1234567890123456789012345678901212345678901234567890123456789012123456789012345678901234567890121234567890123
1234567890123456789012345678901212345678901234567890123456789012123456789012345678901234567890121234
123456789012345678901234567890121234567890123456789012345678901212
12345678901234567890123456789012123456789012345678901234567890121234567890123456789012345678901212345678901234567890123456789012123
1234567890123456789012345678901212345678901234567890123456789012123456789012345678901234567890
YD

12345678901234567890123456789012123456789012345678901234567890121234567890123456789012345678901212345678901234567890123
1234567890123456789012345678901212345678901234567890123456789012123456789012345678901234567890121234567890123
1234567890123456789012345678901212345678901234567890123456789012123456789012345678901234567890121234
12345678901234567890123456789012123456789012345678901234567890121234567890123456789012345678901212345678901234567890123456789012123
1234567890123456789012345678901212345678901234567890123456789012123456789012345678901234567890
123456789012345678901234567890121234567890123456789012345678901212
12345678901234567890123456789012123456789012345678901234567890121234567890123456789012345678901212345678901234567890123
1234567890123456789012345678901212345678901234567890123456789012123456789012345678901234567890121234567890123
1234567890123456789012345678901212345678901234567890123456789012123456789012345678901234567890121234
PRACTICE TEST - 1
12345678901234567890123456789012123456789012345678901234567890121234567890123456789012345678901212345678901234567890123456789012123
1234567890123456789012345678901212345678901234567890123456789012123456789012345678901234567890
123456789012345678901234567890121234567890123456789012345678901212
12345678901234567890123456789012123456789012345678901234567890121234567890123456789012345678901212345678901234567890123
1234567890123456789012345678901212345678901234567890123456789012123456789012345678901234567890121234567890123
1234567890123456789012345678901212345678901234567890123456789012123456789012345678901234567890121234
12345678901234567890123456789012123456789012345678901234567890121234567890123456789012345678901212345678901234567890123456789012123
1234567890123456789012345678901212345678901234567890123456789012123456789012345678901234567890
12345678901234567890123456789012123456789012345678901234567890121234567890123456789012345678901212345678901234567890123456789012123
1234567890123456789012345678901212345678901234567890123456789012123456789012345678901234567890
12345678901234567890123456789012123456789012345678901234567890121234567890123456789012345678901212345678901234567890123
1234567890123456789012345678901212345678901234567890123456789012123456789012345678901234567890121234567890123
1234567890123456789012345678901212345678901234567890123456789012123456789012345678901234567890121234
123456789012345678901234567890121234567890123456789012345678901212
12345678901234567890123456789012123456789012345678901234567890121234567890123456789012345678901212345678901234567890123456789012123
1234567890123456789012345678901212345678901234567890123456789012123456789012345678901234567890
12345678901234567890123456789012123456789012345678901234567890121234567890123456789012345678901212345678901234567890123
1234567890123456789012345678901212345678901234567890123456789012123456789012345678901234567890121234567890123
1234567890123456789012345678901212345678901234567890123456789012123456789012345678901234567890121234
,H

12345678901234567890123456789012123456789012345678901234567890121234567890123456789012345678901212345678901234567890123456789012123
1234567890123456789012345678901212345678901234567890123456789012123456789012345678901234567890

D ãHO÷ k ㄉ¬ ß× …Õ „¬@=ò …Õ Q® PHê~¡=ò ‹¬=¶^¥# „¬\ì…Õ ^Íx…Õ ^¥y†Çò#ß^Ë Q®°iëOKÇO_?
T

‹¬¶KÇ# (1-20) :
AM

(X)
H

ㄬ‰×߄¬@=ò ‹¬=¶^¥# „¬@=ò°


M
AM
IG

1.
KH
R

(X) (1) (2) (3) (4)

2.

(X) (1) (2) (3) (4)

3.

(X) (1) (2) (3) (4)

REASONING ABILITY (NON-VERBAL) ○ ○ ○ ○ ○ ○


225
○ ○ ○ ○ ○ ○ ○ ○ ○ ○ ○ ○ ○ ○ ○ ○ ○ ○ ○
Study Material
4.

(X) (1) (2) (3) (4)

5.

(X) (1) (2) (3) (4)

Y
6.

M
E
(X) (1) (2) (3) (4)

D
9
7.

59
CA
95
50
(X) (1) (2) (3) (4)
98
A
:8
8.
E
h
.P
IC
AD

(X) (1) (2) (3) (4)


AB
O
ER
CH

9.
YD
,H
T

(X) (1) (2) (3) (4)


AM
H
M

10.
AM
IG
KH
R

(X) (1) (2) (3) (4)

11.

(X) (1) (2) (3) (4)

12.

(X) (1) (2) (3) (4)

REASONING ABILITY (NON-VERBAL) ○ ○ ○ ○ ○ ○


226
○ ○ ○ ○ ○ ○ ○ ○ ○ ○ ○ ○ ○ ○ ○ ○ ○ ○ ○
Study Material
13.

(X) (1) (2) (3) (4)

14.

(X) (1) (2) (3) (4)

Y
15.

M
E
(X) (1) (2) (3) (4)

D
9
59
16.
CA
95
50
98
A
(X) (1) (2) (3) (4)
:8
E
h

17.
.P
IC
AD
AB
O

(X) (1) (2) (3) (4)


ER
CH
YD

18.
,H
T
AM
H

(X) (1) (2) (3) (4)


M
AM
IG

19.
KH
R

(X) (1) (2) (3) (4)

20.

(X) (1) (2) (3) (4)

REASONING ABILITY (NON-VERBAL) ○ ○ ○ ○ ○ ○


227
○ ○ ○ ○ ○ ○ ○ ○ ○ ○ ○ ○ ○ ○ ○ ○ ○ ○ ○
Study Material
1234567890123456789012
12345678901234567890123456789012123456789012345678901234567890121234567890123456789012345678901212345678901234567890123
12345678901234567890123
12345678901234567890123456789012123456789012345678901234567890121234567890123456789012345678901212345678901234567890123
1234567890123456789012 12345678901234567890123
12345678901234567890123456789012123456789012345678901234567890121234567890123456789012345678901212345678901234567890123
1234567890123456789012
1234567890123456789012
q=~¡}ì`ÇàH› ["Œ|°°
12345678901234567890123
12345678901234567890123456789012123456789012345678901234567890121234567890123456789012345678901212345678901234567890123
PRACTICE TEST - 1 :
12345678901234567890123
12345678901234567890123456789012123456789012345678901234567890121234567890123456789012345678901212345678901234567890123
1234567890123456789012
12345678901234567890123456789012123456789012345678901234567890121234567890123456789012345678901212345678901234567890123
1234567890123456789012
12345678901234567890123
12345678901234567890123
12345678901234567890123456789012123456789012345678901234567890121234567890123456789012345678901212345678901234567890123
1234567890123456789012 12345678901234567890123

1. (2) 6. (2) 11. (1) 16. (2)

2. (1) 7. (4) 12. (4) 17. (4)

3. (4) 8. (1) 13. (1) 18. (3)

Y
M
4. (4)

E
9. (4) 14. (4) 19. (4)

D
9
59
CA
95
5. (2) 15. (4) 20. (4)

50
10. (2)
98
A
:8
E

123456789012345678901234567890121234567890123456789012345678901212
h

12345678901234567890123456789012123456789012345678901234567890121234567890123456789012345678901212345678901234567890123
1234567890123456789012345678901212345678901234567890123456789012123456789012345678901234567890121234567890123
1234567890123456789012345678901212345678901234567890123456789012123456789012345678901234567890121234
12345678901234567890123456789012123456789012345678901234567890121234567890123456789012345678901212345678901234567890123456789012123
1234567890123456789012345678901212345678901234567890123456789012123456789012345678901234567890
123456789012345678901234567890121234567890123456789012345678901212
12345678901234567890123456789012123456789012345678901234567890121234567890123456789012345678901212345678901234567890123456789012123
1234567890123456789012345678901212345678901234567890123456789012123456789012345678901234567890
12345678901234567890123456789012123456789012345678901234567890121234567890123456789012345678901212345678901234567890123
1234567890123456789012345678901212345678901234567890123456789012123456789012345678901234567890121234567890123
1234567890123456789012345678901212345678901234567890123456789012123456789012345678901234567890121234
.P

123456789012345678901234567890121234567890123456789012345678901212
12345678901234567890123456789012123456789012345678901234567890121234567890123456789012345678901212345678901234567890123456789012123
1234567890123456789012345678901212345678901234567890123456789012123456789012345678901234567890
12345678901234567890123456789012123456789012345678901234567890121234567890123456789012345678901212345678901234567890123
1234567890123456789012345678901212345678901234567890123456789012123456789012345678901234567890121234567890123
1234567890123456789012345678901212345678901234567890123456789012123456789012345678901234567890121234
PRACTICE TEST - 2
IC

12345678901234567890123456789012123456789012345678901234567890121234567890123456789012345678901212345678901234567890123456789012123
1234567890123456789012345678901212345678901234567890123456789012123456789012345678901234567890
12345678901234567890123456789012123456789012345678901234567890121234567890123456789012345678901212345678901234567890123
1234567890123456789012345678901212345678901234567890123456789012123456789012345678901234567890121234567890123
1234567890123456789012345678901212345678901234567890123456789012123456789012345678901234567890121234
123456789012345678901234567890121234567890123456789012345678901212
12345678901234567890123456789012123456789012345678901234567890121234567890123456789012345678901212345678901234567890123456789012123
1234567890123456789012345678901212345678901234567890123456789012123456789012345678901234567890
12345678901234567890123456789012123456789012345678901234567890121234567890123456789012345678901212345678901234567890123
1234567890123456789012345678901212345678901234567890123456789012123456789012345678901234567890121234567890123
1234567890123456789012345678901212345678901234567890123456789012123456789012345678901234567890121234
123456789012345678901234567890121234567890123456789012345678901212
12345678901234567890123456789012123456789012345678901234567890121234567890123456789012345678901212345678901234567890123456789012123
1234567890123456789012345678901212345678901234567890123456789012123456789012345678901234567890
12345678901234567890123456789012123456789012345678901234567890121234567890123456789012345678901212345678901234567890123
1234567890123456789012345678901212345678901234567890123456789012123456789012345678901234567890121234567890123
1234567890123456789012345678901212345678901234567890123456789012123456789012345678901234567890121234
AD

12345678901234567890123456789012123456789012345678901234567890121234567890123456789012345678901212345678901234567890123456789012123
1234567890123456789012345678901212345678901234567890123456789012123456789012345678901234567890

‹¬¶KÇ# (1-20) : D ãHO÷ k W=Þ|_# ㄉ¬ ß× „¬@=ò …Õ Q® PHê~¡=ò ‹¬=¶^¥# „¬\ì…Õ ^Íx…Õ ^¥y†Çò#ß^Ë H›#°HËøO_ ?
AB
O

'X'

ㄬ‰×߄¬@=ò ‹¬=¶^¥# „¬@=ò°


ER
CH
YD

1.
,H
T
AM
H

(X) (1) (2) (3) (4)


M
AM
IG

2.
KH
R

(X) (1) (2) (3) (4)

3.

(X) (1) (2) (3) (4)

4.

(X) (1) (2) (3) (4)


REASONING ABILITY (NON-VERBAL) ○ ○ ○ ○ ○ ○
228
○ ○ ○ ○ ○ ○ ○ ○ ○ ○ ○ ○ ○ ○ ○ ○ ○ ○ ○
Study Material
5.

(X) (1) (2) (3) (4)

6.

(X) (1) (2) (3) (4)

Y
7.

M
E
(X) (1) (2) (3) (4)

D
9
59
8.
CA
95
50
98
A
:8
(X) (1) (2) (3) (4)
E
h
.P
IC

9.
AD
AB
O
ER
CH

(X) (1) (2) (3) (4)


YD

10.
,H
T
AM
H
M
AM

(X) (1) (2) (3) (4)


IG
KH

11.
R

(X) (1) (2) (3) (4)

12.

(X) (1) (2) (3) (4)

REASONING ABILITY (NON-VERBAL) ○ ○ ○ ○ ○ ○


229
○ ○ ○ ○ ○ ○ ○ ○ ○ ○ ○ ○ ○ ○ ○ ○ ○ ○ ○
Study Material
13.

(X) (1) (2) (3) (4)

14.

(X) (1) (2) (3) (4)

Y
15.

M
E
(X) (1) (2) (3) (4)

D
9
59
16.
CA
95
50
98
A
(X) (1) (2) (3) (4)
:8
E
h
.P

17.
IC
AD
AB
O

(X) (1) (2) (3) (4)


ER
CH
YD

18.
,H
T
AM
H
M

(X) (1) (2) (3) (4)


AM
IG

19.
KH
R

(X) (1) (2) (3) (4)

20.

(X) (1) (2) (3) (4)

REASONING ABILITY (NON-VERBAL) ○ ○ ○ ○ ○ ○


230
○ ○ ○ ○ ○ ○ ○ ○ ○ ○ ○ ○ ○ ○ ○ ○ ○ ○ ○
Study Material
1234567890123456789012
12345678901234567890123456789012123456789012345678901234567890121234567890123456789012345678901212345678901234567890123
12345678901234567890123
12345678901234567890123456789012123456789012345678901234567890121234567890123456789012345678901212345678901234567890123
1234567890123456789012 12345678901234567890123
12345678901234567890123456789012123456789012345678901234567890121234567890123456789012345678901212345678901234567890123
1234567890123456789012
1234567890123456789012
q=~¡}ì`ÇàH› ["Œ|°°
12345678901234567890123
12345678901234567890123456789012123456789012345678901234567890121234567890123456789012345678901212345678901234567890123
PRACTICE TEST - 2 :
12345678901234567890123
12345678901234567890123456789012123456789012345678901234567890121234567890123456789012345678901212345678901234567890123
1234567890123456789012
12345678901234567890123456789012123456789012345678901234567890121234567890123456789012345678901212345678901234567890123
1234567890123456789012
12345678901234567890123
12345678901234567890123
12345678901234567890123456789012123456789012345678901234567890121234567890123456789012345678901212345678901234567890123
1234567890123456789012 12345678901234567890123

1. (4) 6. (3) 16. (1)


11. (2)

7. (1) 12. (4) 17. (2)


2. (4)

Y
18. (4)

M
3. (4) 8. (3) 13. (3)

E
D
9
59
14. (4)
CA 19. (3)

95
4. (1) 9. (4)

50
98
A
:8
E

15. (4)
h

5. (4) 10. (4) 20. (1)


.P
IC
AD
AB
O

123456789012345678901234567890121234567890123456789012345678901212
12345678901234567890123456789012123456789012345678901234567890121234567890123456789012345678901212345678901234567890123
1234567890123456789012345678901212345678901234567890123456789012123456789012345678901234567890121234567890123
1234567890123456789012345678901212345678901234567890123456789012123456789012345678901234567890121234
12345678901234567890123456789012123456789012345678901234567890121234567890123456789012345678901212345678901234567890123456789012123
123456789012345678901234567890121234567890123456789012345678901212345678901234567890123
12345678901234567890123456789012123456789012345678901234567890121234567890123456789012345678901212345678901234567890123
1234567890123456789012345678901212345678901234567890123456789012123456789012345678901234567890121234567890123
1234567890123456789012345678901212345678901234567890123456789012123456789012345678901234567890121234
123456789012345678901234567890121234567890123456789012345678901212
12345678901234567890123456789012123456789012345678901234567890121234567890123456789012345678901212345678901234567890123456789012123
123456789012345678901234567890121234567890123456789012345678901212345678901234567890123
ER
CH

12345678901234567890123456789012123456789012345678901234567890121234567890123456789012345678901212345678901234567890123456789012123
123456789012345678901234567890121234567890123456789012345678901212345678901234567890123
12345678901234567890123456789012123456789012345678901234567890121234567890123456789012345678901212345678901234567890123
1234567890123456789012345678901212345678901234567890123456789012123456789012345678901234567890121234567890123
1234567890123456789012345678901212345678901234567890123456789012123456789012345678901234567890121234
123456789012345678901234567890121234567890123456789012345678901212
PREVIOUS BITS
12345678901234567890123456789012123456789012345678901234567890121234567890123456789012345678901212345678901234567890123456789012123
123456789012345678901234567890121234567890123456789012345678901212345678901234567890123
12345678901234567890123456789012123456789012345678901234567890121234567890123456789012345678901212345678901234567890123
1234567890123456789012345678901212345678901234567890123456789012123456789012345678901234567890121234567890123
1234567890123456789012345678901212345678901234567890123456789012123456789012345678901234567890121234
12345678901234567890123456789012123456789012345678901234567890121234567890123456789012345678901212345678901234567890123456789012123
123456789012345678901234567890121234567890123456789012345678901212345678901234567890123
123456789012345678901234567890121234567890123456789012345678901212
12345678901234567890123456789012123456789012345678901234567890121234567890123456789012345678901212345678901234567890123
1234567890123456789012345678901212345678901234567890123456789012123456789012345678901234567890121234567890123
1234567890123456789012345678901212345678901234567890123456789012123456789012345678901234567890121234
12345678901234567890123456789012123456789012345678901234567890121234567890123456789012345678901212345678901234567890123456789012123
123456789012345678901234567890121234567890123456789012345678901212345678901234567890123
12345678901234567890123456789012123456789012345678901234567890121234567890123456789012345678901212345678901234567890123
1234567890123456789012345678901212345678901234567890123456789012123456789012345678901234567890121234567890123
1234567890123456789012345678901212345678901234567890123456789012123456789012345678901234567890121234
123456789012345678901234567890121234567890123456789012345678901212
12345678901234567890123456789012123456789012345678901234567890121234567890123456789012345678901212345678901234567890123456789012123
123456789012345678901234567890121234567890123456789012345678901212345678901234567890123
YD

12345678901234567890123456789012123456789012345678901234567890121234567890123456789012345678901212345678901234567890123
1234567890123456789012345678901212345678901234567890123456789012123456789012345678901234567890121234567890123
1234567890123456789012345678901212345678901234567890123456789012123456789012345678901234567890121234
12345678901234567890123456789012123456789012345678901234567890121234567890123456789012345678901212345678901234567890123456789012123
123456789012345678901234567890121234567890123456789012345678901212345678901234567890123
,H

1. ãH÷Ok ㄬ‰×߅Õ, ㄬ‰×߄¬@O…Õ ~ò=Þ|_# ƒìQê#° 2. D kQ®°=# S^ΰ „¬\쁰 ~ò=Þ|_#q. "Œ\÷…Õ XH›
T
AM

JuH÷OKÇQê U~¡æ_Í „¬\ìxß, ‹¬=¶^¥#„¬\ì #°O_ KÇ ` Ç ° ~¡ ã ªxß U~¡ æ _È \ ìxH÷ JuH÷ O KÇ = ‹² # =ü_È °
H

H›#°Qù#°=ò. „¬\쁰.
M

(A.P.S.I.Mains -2016) (A.P.S.I.Mains -2016)


AM

ㄬ‰§ß „¬@O ‹¬=¶^¥# „¬\쁰


IG
KH
R

1) 2) 3) 4)
(A) (B) (C) (D) (E)

1) A, C, D 2) A, B, D 3) B, D, E 4) C, D, E
12345678901234567890123456789012123456789012345678901234567890121234567890123456789012345678901212345678901234567890123
1234567890123456789012 123456789012345678901234
12345678901234567890123456789012123456789012345678901234567890121234567890123456789012345678901212345678901234567890123
1234567890123456789012 123456789012345678901234

q=~¡}ì`ÇàH› ["Œ|°°
12345678901234567890123456789012123456789012345678901234567890121234567890123456789012345678901212345678901234567890123
1234567890123456789012 123456789012345678901234
12345678901234567890123456789012123456789012345678901234567890121234567890123456789012345678901212345678901234567890123
1234567890123456789012 123456789012345678901234
1234567890123456789012
1234567890123456789012 PREVIOUS BITS :
12345678901234567890123456789012123456789012345678901234567890121234567890123456789012345678901212345678901234567890123
123456789012345678901234
12345678901234567890123456789012123456789012345678901234567890121234567890123456789012345678901212345678901234567890123
123456789012345678901234
12345678901234567890123456789012123456789012345678901234567890121234567890123456789012345678901212345678901234567890123
1234567890123456789012 123456789012345678901234
12345678901234567890123456789012123456789012345678901234567890121234567890123456789012345678901212345678901234567890123

1. (2) ㄬ‰×ß „¬@O…Õx PHê~Œ#° L„¬†³¶yOz `džǶ~¡° 2. (1) XH› KÇ ` Ç ° ~¡ 㠋¬ O (Jxß ƒ ’ ° *쁰 ‹¬ = ¶#O)#°
K͆Çò#k ‹¬=¶^¥# „¬\쁅Õx '2Ñ=k. U~¡æ~¡°KÇ°@䛽 =°#䛽 W=Þ|_# ƒç=°à…Õ L„¬†³¶Q®
„¬_È°#k A, C, D. " %
$

REASONING ABILITY (NON-VERBAL) ○ ○ ○ ○ ○ ○


231
○ ○ ○ ○ ○ ○ ○ ○ ○ ○ ○ ○ ○ ○ ○ ○ ○ ○ ○
Study Material
6 J^ÎíO…Õ ㄬuaO|O (Mirror Image)

=òMì¼O‰§°...
G D KŒ„¬“~Ÿ…Õx ㄬ‰×߁° J^ÎíO…Õ U~¡æ_È° ㄬuaO|Ōá P^¥~¡„¬_ LO\ì~ò.
G XH› =‹¬°ë=ô#° J^ÎíO =òO^ΰOz (…è^¥) XH› =‹¬°ë=ô =òO^ΰ J^ÎíO LOz P =‹¬°ë=ô †³òH›ø ㄬuaO|O †³òH›ø PHê~¡O Z…ì
=ôO@°O^Ë H›#°Qù#=°O\ì~¡°.
G J^ÎíO…Õx ㄬuaO|O †³òH›ø Z_È=°ƒìQ®O, "Œ‹¬ë= =‹¬°ë=ô †³òH›ø 䛽_ƒìQ®O. J^ÎíO…Õx ㄬuaO|O †³òH›ø 䛽_ƒìQ®O,

Y
"Œ‹¬ë= =‹¬°ë=ô †³òH›ø Z_È=°ƒìQ®O.

M
G J^ÎíO =òO^ΰ XH› =‹¬°ë=ô J^ÎíO #°O_ ZO`Ç ^ζ~¡O…Õ =ôO^Ë, J^ÎíO…Õ ㄬuaO|O 䛀_¨ J^ÎíO #°O_ JO`Í ^ζ~¡O…Õ

E
LO@°Ok.

D
9
G 0 #°O_ 9 =~¡ä›½ Q® JOïH †³òH›ø ㄬuaOƒì° G POQ®Á JH›Æ~¡=¶…Õ ̄^Îí JH›Æ~Œ° =°i†Çò "Œ\÷

59
J^ÎíO…Õ Z…ì U~¡æ_È`Œ†³¶ KǶ„¬ô „¬\ ÷“H› :
CA
95
ㄬuaOƒì° XöH…ìQê =ôO_È°#q '11Ñ Jq,

50
JOïH ㄬuaO|O JH›Æ~¡O A H I M O T U V W X Y
98
A
0 0
ㄬuaO|O A H I M O T U V W
:8
X Y
1 1
E

G WOwÁ+µ¬ JH›~Æ =¡ ¶…Õx z#ß JH›~Æ Œ †³òH›ø ㄬuaOƒì°


h

2 2
.P
IC

3 3
J^ÎíO…Õ Z…ì U~¡æ_È`Œ†³¶ KǶ„¬ô „¬\ ÷“H› :
AD

4 4
5 5 JH›Æ~¡O ㄬuaO|O JH›Æ~¡O ㄬuaO|O
AB
O

6 6
a a n n
ER
CH

7 7
8 b b o o
YD

8
9 9 c c p p
,H

q q
G JOïH…Õ 0 =°i†Çò 8 †³òH›ø ㄬuaOƒì° J"Í =ôO\ì~ò. d d
T
AM

r r
G WOwÁ+µ¬ JH›~Æ =¡ ¶…Õx ̄^Îí JH›~Æ Œ †³òH›ø ㄬuaOƒì° e e
H

s s
M

J^ÎíO…Õ Z…ì U~¡æ_È`Œ†³¶ KǶ„¬ô „¬\ ÷“H› : f f


t t
AM
IG

g g
JH›Æ~¡O ㄬuaO|O JH›Æ~¡O ㄬuaO|O h
u u
KH

h
A A N N v v
i i
R

B B O O w w
P P j j x x
C C
Q Q k k y y
D D
E E R R l l z z
S S m m –
F F –
T T
G G
H H
U U G POQ®Á JH›Æ~¡=¶…Õ z#ß JH›Æ~Œ° =°i†Çò "Œ\÷
V V
I I W W ㄬuaOƒì° XöH…ìQê =ôO_È°#q '5Ñ Jq,
J J X X
K K Y Y
JH›Æ~¡O i l o v w x
L L Z Z ㄬuaO|O i l o v w x
M M – –

REASONING ABILITY (NON-VERBAL) ○ ○ ○ ○ ○ ○


232
○ ○ ○ ○ ○ ○ ○ ○ ○ ○ ○ ○ ○ ○ ○ ○ ○ ○ ○
Study Material
G Hùxß JH›Æ~Œ° =°i†Çò JOïH q°ã‰×=¶ †³òH›ø G Hùxß q°ã‰×=° ‹¬OY¼ ㄬuaOƒì° J^ÎOí …Õ Z…ì =ôO\솳¶
ㄬuaOƒì° J^ÎOí …Õ Z…ì U~¡æ_È°`Œ†³¶ KǶ„¬ô „¬\ ÷“H› : KǶ„¬ô „¬\ ÷“H› :
JOïH°, JH›Æ~Œ`Ë U~¡æ_# J^ÎíO…Õ ‹¬OY¼° ㄬuaO|=ò
q°ã‰×=° „¬^¥° ㄬuaO|=ò 49631 13694
1B97dAMK543e e345KMAd79B1 28357 75382
uzo698MTJn52 25nJTM896ozu 350 053
3cfoL7H H7Lofc3 680171 171086
1A694PQ7 7QP496A1 16947 74961
11EFRgd385 583dgRFE11 13351 15331

G J^ÎíO…Õ Q®_†Ç¶~Œ ㄬuaOƒì#° KǶ„¬ô „¬\ ÷“H› =°i†Çò ㄬuaO|O…Õ U~¡æ_È° ‹¬=°†Ç°O :

Y
x[ Q®_†Ç¶~¡O ‹¬=°†Ç°O J^ÎíO…Õ ㄬuaO|O ㄬuaO|O G U ‹¬=°†Ç°O…Õ<³á<Œ J^ÎíO…Õ U~¡æ_# Q®_†Ç¶~¡O

M
…Õx ‹¬=°†Ç°O H›#°Qù#°@䛽 x[Q®_†Ç¶~¡O…Õx
‹¬=°†Ç¶xß 12 #°O_ f‹²"† Í ¶Ç e. J#Qê 11:60

E
22 23 2 2 32 22
21 3 3 12 #°O_. L^¥‚¬ì~¡}䛽 x[ Q®_†  ¶Ç ~¡O…Õ ‹¬=°†Ç°O

D
9
3 : 00 9 : 00 3:00 J~ò#„¬ô_È°, ㄬuaO|O…Õ ‹¬=°†Ç°O 12-

59
: 4 4 :
9 5 5 9
CA 3 « 9:00 Q®O@°.

95
8 7 6 6 7 8
x[Q®_†Ç¶~¡O…Õ ‹¬=°†Ç°O 8:15 J~ò#„¬ô_È°,

50
22 23 2 2 32 22 ㄬuaO|O…Õ ‹¬=°†Ç°O 11:60-8:15 « 3:45.
98
A
21 3 3 12 G Q®_†Ç¶~Œä›½ ‹¬O|OkOz ㄬuaOƒì…Õx
:8
: 4 4 : 00 4 : 8 : 00
‹¬ = °†Ç ° O H› # °Qù#°@ä› ½ ö H =O 12 Q® O II
E

9 5 5 9
h

8 7 6 6 7 8 Hê=¶<Œxß =¶ã`Ç"Í° J#°‹¬iOKŒe. a.m


.P
IC

=°i†Çò p.m `Ë ‹¬O|O^ÎO …è^ΰ.


AD

=¶ki ㄬ‰×߁°
AB
O
ER
CH
YD

MODEL - 1 „¬^ÎO J^ΜO ㄬuaO|O


,H

(1-5) D ãH÷O^Î „¬^¥ J^Μ„¬ô ㄬuaOƒìxß H›#°Qù#O_.


T

ª^Î#: (1) EAGER REGAE


AM

1. GUIMP
H
M

1) GUIMP 2) PMIUG 3) GUIMP 4) PMIUG 4. GEOGRAPHY


AM
IG

„¬^ÎO J^ΜO ㄬuaO|O 1) YHPARGOEG 2) YHPARGOEG


KH

ª^Î#: (2) GUIMP PMIUG


3) YHPARGOEG 4) YHPARGOEG
R

„¬^ÎO J^ΜO ㄬuaO|O


2. SHARE
ª^Î#: (1) GEOGRAPHY YHPARGOEG
1) ERAHS 2) ERAHS 3) ERAHS 4) ERAHS
„¬^ÎO J^ΜO ㄬuaO|O 5. EFFECTIVE
ª^Î#: (1) SHARE ERAHS 1) EVITCEFFE 2) EVITCEFFE
3) EFFECTIVE 4) EVITCEFFE
3. EAGER „¬^ÎO J^ΜO ㄬuaO|O
1) REGAE 2) REGAE ª^Î#: (1) EFFECTIVE EVITCEFFE
3) REGAE 4) REDAE
REASONING ABILITY (NON-VERBAL) ○ ○ ○ ○ ○ ○
233
○ ○ ○ ○ ○ ○ ○ ○ ○ ○ ○ ○ ○ ○ ○ ○ ○ ○ ○
Study Material
123456789012345678901234567890121234567890123456789012345678901212
12345678901234567890123456789012123456789012345678901234567890121234567890123456789012345678901212345678901234567890123
1234567890123456789012345678901212345678901234567890123456789012123456789012345678901234567890121234567890123
1234567890123456789012345678901212345678901234567890123456789012123456789012345678901234567890121234
12345678901234567890123456789012123456789012345678901234567890121234567890123456789012345678901212345678901234567890123456789012123
1234567890123456789012345678901212345678901234567890123456789012123456789012345678901234567890
12345678901234567890123456789012123456789012345678901234567890121234567890123456789012345678901212345678901234567890123
1234567890123456789012345678901212345678901234567890123456789012123456789012345678901234567890121234567890123
1234567890123456789012345678901212345678901234567890123456789012123456789012345678901234567890121234
123456789012345678901234567890121234567890123456789012345678901212
12345678901234567890123456789012123456789012345678901234567890121234567890123456789012345678901212345678901234567890123456789012123
1234567890123456789012345678901212345678901234567890123456789012123456789012345678901234567890
12345678901234567890123456789012123456789012345678901234567890121234567890123456789012345678901212345678901234567890123
1234567890123456789012345678901212345678901234567890123456789012123456789012345678901234567890121234567890123
1234567890123456789012345678901212345678901234567890123456789012123456789012345678901234567890121234
12345678901234567890123456789012123456789012345678901234567890121234567890123456789012345678901212345678901234567890123456789012123
1234567890123456789012345678901212345678901234567890123456789012123456789012345678901234567890
123456789012345678901234567890121234567890123456789012345678901212
PRACTICE TEST on MODEL - 1
12345678901234567890123456789012123456789012345678901234567890121234567890123456789012345678901212345678901234567890123
1234567890123456789012345678901212345678901234567890123456789012123456789012345678901234567890121234567890123
1234567890123456789012345678901212345678901234567890123456789012123456789012345678901234567890121234
12345678901234567890123456789012123456789012345678901234567890121234567890123456789012345678901212345678901234567890123456789012123
1234567890123456789012345678901212345678901234567890123456789012123456789012345678901234567890
123456789012345678901234567890121234567890123456789012345678901212
12345678901234567890123456789012123456789012345678901234567890121234567890123456789012345678901212345678901234567890123
1234567890123456789012345678901212345678901234567890123456789012123456789012345678901234567890121234567890123
1234567890123456789012345678901212345678901234567890123456789012123456789012345678901234567890121234
12345678901234567890123456789012123456789012345678901234567890121234567890123456789012345678901212345678901234567890123456789012123
1234567890123456789012345678901212345678901234567890123456789012123456789012345678901234567890
12345678901234567890123456789012123456789012345678901234567890121234567890123456789012345678901212345678901234567890123
1234567890123456789012345678901212345678901234567890123456789012123456789012345678901234567890121234567890123
1234567890123456789012345678901212345678901234567890123456789012123456789012345678901234567890121234
12345678901234567890123456789012123456789012345678901234567890121234567890123456789012345678901212345678901234567890123456789012123
1234567890123456789012345678901212345678901234567890123456789012123456789012345678901234567890
123456789012345678901234567890121234567890123456789012345678901212
12345678901234567890123456789012123456789012345678901234567890121234567890123456789012345678901212345678901234567890123456789012123
1234567890123456789012345678901212345678901234567890123456789012123456789012345678901234567890
12345678901234567890123456789012123456789012345678901234567890121234567890123456789012345678901212345678901234567890123
1234567890123456789012345678901212345678901234567890123456789012123456789012345678901234567890121234567890123
1234567890123456789012345678901212345678901234567890123456789012123456789012345678901234567890121234

‹¬¶KÇ# (1-20) : D ãH÷Ok ㄬ‰×߁…Õ W=Þ|_# „¬^ÎO †³òH›ø J^Μ„¬ô ㄬuaOƒìxß ãH÷O^Î W=Þ|_# ㄬ`Œ¼=¶ß†Ç¶ #°O_
H›#°Qù#O_?
1. DEFENCE
%
11. NATIONAL
1) ECNEFED 2) & $ /& &
)
1) LANOI^AN 2) LANOITAN
3) ECNEFED 4) ECNEFED 3) LANOITAM 4) LANOITAN
2. COMBAT 12. FIXING
1) COM B AT 2) TABMOC 1) GNIXIF 2) FIXING
3) TABMOC 4) T ABMOC 3) GNIXIF 4) GNIXIF
3. SUMMONED 13. BRISK
1) DENOMMUS 2) DENOMMUS 1) KSIRB 2) KSIRB

Y
3) DENOMMUS 4) DENOMMUS 3) KSIRB 4) KSIRB

M
4. SCREEN 14. PAINTED
1) NEERCS
2) NEERCS 1) DETNIAP 2) DETNIAP

E
3) NEERCS 4) NEERCS 3) DETNIAP 4) DETNIAP

D
9
5. HALLOWED 15. INFORMATIONS

59
1) DEWOLLAH 2) DEWOLLAH 1) 2) INFORMATIONS
CA
95
3) DEWOLLAH 4) DEWOLLAH 3) 4) INFORMATIONS

50
6. IRONIC 16. SUPERVISOR
98
A
1) CINORI 2) CINORI 1) ROSIVREPUS 2)
4) CINORI
:8
3) CINORI 3) 4)
E

7. CONNOISSEUR 17. REASONING


h
.P

1) RUESSIONNOC 2) RUESSIONNOC 1) REASONING 2) GNINOSAER


IC

3) RUESSIONNOC 4) RUESSIONNOC 3) REASONING 4) GNINOSAER


AD

8. SECURE 18. MAGAZINE


AB
O

1) ERUCES 2) ERUCES 1) REASONING 2) GNINOSAER


ER
CH

3) ERUCES 4) ERUCES 3) MAGAZINE 4) ENIZAGAM


YD

9. QUALITY 19. qutubgarh


1) QUALITY 2) YTILAUQ 1) p u t u d g a r h 2) C V V E H B I
,H

W U
3) YTILAUQ 4) YTILAUQ 3) h r a g b u a t u q 4) h r a g b u t u p
T
AM

10. TERMINATE
. 20. MALAYALAM
H
M

1) TERMINATE 2) ETANI RET 1) MALAYALAM 2) MALAYALAM


L L
AM

3) ETANIMRET 4) ETANIMRET 3) 4) MA AYA AM


IG

0$/$<$/$0
KH

123456789012345678901234567890121234567890
12345678901234567890123456789012123456789012345678901234567890121234567890123456789012345678901212345678901234567890123
123456789012345678901234567890121234567890

q=~¡}ì`ÇàH› ["Œ|°°
12345678901234567890123456789012123456789012345678901234567890121234567890123456789012345678901212345678901234567890123
123456789012345678901234567890121234567890
123456789012345678901234567890121234567890 123456789012345678901234567890121234567890
12345678901234567890123456789012123456789012345678901234567890121234567890123456789012345678901212345678901234567890123
123456789012345678901234567890121234567890
123456789012345678901234567890121234567890
123456789012345678901234567890121234567890
12345678901234567890123456789012123456789012345678901234567890121234567890123456789012345678901212345678901234567890123
R

123456789012345678901234567890121234567890
12345678901234567890123456789012123456789012345678901234567890121234567890123456789012345678901212345678901234567890123
123456789012345678901234567890121234567890 123456789012345678901234567890121234567890
12345678901234567890123456789012123456789012345678901234567890121234567890123456789012345678901212345678901234567890123
123456789012345678901234567890121234567890
123456789012345678901234567890121234567890
123456789012345678901234567890121234567890
12345678901234567890123456789012123456789012345678901234567890121234567890123456789012345678901212345678901234567890123

„¬^ÎO J^ΜO ㄬuaO|O „¬^ÎO J^ΜO ㄬuaO|O


1. (3) DEFENCE ECNEFED 4. (4) SCREEN NEERCS

„¬^ÎO J^ΜO ㄬuaO|O „¬^ÎO J^ΜO ㄬuaO|O


2. (2) COMBAT TABMOC 5. (2) HALLOWED DEWOLLAH

„¬^ÎO J^ΜO ㄬuaO|O „¬^ÎO J^ΜO ㄬuaO|O


3. (2) SUMMONED DENOMMUS 6. (1) IRONIC CINORI

REASONING ABILITY (NON-VERBAL) ○ ○ ○ ○ ○ ○


234
○ ○ ○ ○ ○ ○ ○ ○ ○ ○ ○ ○ ○ ○ ○ ○ ○ ○ ○
Study Material
„¬^ÎO J^ΜO ㄬuaO|O MODEL - 2
7. (1) CONNOISSEUR RUESSIONNOC
(1-2) D ãH÷Ok „¬@=ò 'X' …Õ 'AB' ªÖ#O…Õ J^Î=œ ò LOz#KË
„¬^ÎO J^ΜO ㄬuaO|O U~¡æ_È° ㄬuaOƒìxß ã„¬Hø› <Í L#ß ã„¬`Œ¼=¶ß†Ç¶ #°O_
8. (2) SECURE ERUCES Q®°iëOKÇO_.

„¬^ÎO J^ΜO ㄬuaO|O 1.


9. (3) QUALITY YTILAUQ
(X) (a) (b) (c) (d)

„¬^ÎO J^ΜO ㄬuaO|O (B.ED. 2002)

10. (3) TERMINATE ETANIMRET ª^Î#:

Y
M
„¬^ÎO J^ΜO ㄬuaO|O (X) (c)

E
11. (2) NATIONAL LANOITAN
2.

D
9
„¬^ÎO J^ΜO ㄬuaO|O

59
CA
95
(X) (a) (b) (c) (d)
12. (4) FIXING GNIXIF

50
ª^Î#:
„¬^ÎO J^ΜO ㄬuaO|O 98
A
:8
13. (4) BRISK KSIRB (X) (b)
E
h
.P

„¬^ÎO J^ΜO ㄬuaO|O MODEL - 3


IC
AD

14. (2) PAINTED DETNIAP (1-2) D ãH÷Ok „¬@=ò 'X'…Õ 'MN' ªÖ#O…Õ J^Î=œ ò#° LOz#
AB
O

U~¡æ_È° ㄬuaOƒìxß ã„¬H›ø<Í L#ß ã„¬`Œ¼=¶ß†Ç¶#°


„¬^ÎO J^ΜO ㄬuaO|O
ER

H›#°Qù#O_.
CH
YD

15. (3) INFORMATIONS 1.


,H

„¬^ÎO J^ΜO ㄬuaO|O


T
AM

(X) (a) (b) (c) (d)


H

16. (1) SUPERVISOR ROSIVREPUS ª^Î#:


M
AM
IG

„¬^ÎO J^ΜO ㄬuaO|O (X) (d)


KH

17. (2) REASONING GNINOSAER


2.
R

„¬^ÎO J^ΜO ㄬuaO|O


18. (4) MAGAZINE ENIZAGAM (X) (a) (b) (c) (d)

„¬^ÎO J^ΜO ㄬuaO|O ª^Î#:


19. (4) q u t u b g a r h hragbutup
(X) (a)

„¬^ÎO J^ΜO ㄬuaO|O


20. (2) MALAYALAM MALAYALAM

REASONING ABILITY (NON-VERBAL) ○ ○ ○ ○ ○ ○


235
○ ○ ○ ○ ○ ○ ○ ○ ○ ○ ○ ○ ○ ○ ○ ○ ○ ○ ○
Study Material
123456789012345678901234567890121234567890123456789012345678901212
1234567890123456789012345678901212345678901234567890123456789012123456789012345678901234567890121234567890123
1234567890123456789012345678901212345678901234567890123456789012123456789012345678901234567890121234
12345678901234567890123456789012123456789012345678901234567890121234567890123456789012345678901212345678901234567890123456789012123
1234567890123456789012345678901212345678901234567890123456789012123456789012345678901234567890
12345678901234567890123456789012123456789012345678901234567890121234567890123456789012345678901212345678901234567890123
1234567890123456789012345678901212345678901234567890123456789012123456789012345678901234567890121234567890123
1234567890123456789012345678901212345678901234567890123456789012123456789012345678901234567890121234
123456789012345678901234567890121234567890123456789012345678901212
12345678901234567890123456789012123456789012345678901234567890121234567890123456789012345678901212345678901234567890123456789012123
1234567890123456789012345678901212345678901234567890123456789012123456789012345678901234567890
12345678901234567890123456789012123456789012345678901234567890121234567890123456789012345678901212345678901234567890123
1234567890123456789012345678901212345678901234567890123456789012123456789012345678901234567890121234567890123
1234567890123456789012345678901212345678901234567890123456789012123456789012345678901234567890121234
12345678901234567890123456789012123456789012345678901234567890121234567890123456789012345678901212345678901234567890123456789012123
1234567890123456789012345678901212345678901234567890123456789012123456789012345678901234567890
123456789012345678901234567890121234567890123456789012345678901212
TEST YOURSELF
12345678901234567890123456789012123456789012345678901234567890121234567890123456789012345678901212345678901234567890123
1234567890123456789012345678901212345678901234567890123456789012123456789012345678901234567890121234567890123
1234567890123456789012345678901212345678901234567890123456789012123456789012345678901234567890121234
12345678901234567890123456789012123456789012345678901234567890121234567890123456789012345678901212345678901234567890123456789012123
1234567890123456789012345678901212345678901234567890123456789012123456789012345678901234567890
123456789012345678901234567890121234567890123456789012345678901212
12345678901234567890123456789012123456789012345678901234567890121234567890123456789012345678901212345678901234567890123
1234567890123456789012345678901212345678901234567890123456789012123456789012345678901234567890121234567890123
1234567890123456789012345678901212345678901234567890123456789012123456789012345678901234567890121234
12345678901234567890123456789012123456789012345678901234567890121234567890123456789012345678901212345678901234567890123456789012123
1234567890123456789012345678901212345678901234567890123456789012123456789012345678901234567890
12345678901234567890123456789012123456789012345678901234567890121234567890123456789012345678901212345678901234567890123
1234567890123456789012345678901212345678901234567890123456789012123456789012345678901234567890121234567890123
1234567890123456789012345678901212345678901234567890123456789012123456789012345678901234567890121234
12345678901234567890123456789012123456789012345678901234567890121234567890123456789012345678901212345678901234567890123456789012123
1234567890123456789012345678901212345678901234567890123456789012123456789012345678901234567890
123456789012345678901234567890121234567890123456789012345678901212
12345678901234567890123456789012123456789012345678901234567890121234567890123456789012345678901212345678901234567890123456789012123
1234567890123456789012345678901212345678901234567890123456789012123456789012345678901234567890
12345678901234567890123456789012123456789012345678901234567890121234567890123456789012345678901212345678901234567890123
1234567890123456789012345678901212345678901234567890123456789012123456789012345678901234567890121234567890123
1234567890123456789012345678901212345678901234567890123456789012123456789012345678901234567890121234

1. 486AMO35 †³òH›ø J^Μ„¬ô ㄬuaOƒìxß H›#°Qù#O_?


1) 553OMA684 2) 53OMA684 10.
3) 3OMA684 4) 5O3MA684
(1) (2) (3) (4)
2. D ãH÷Ok "Œx…Õ J^ÎíO…Õ ‹¬=¶# ㄬuaO|O#° U~¡æ~¡KÇ° (X)

JO‰×=ò Uk?
1) S5OI8MVU 2) IOM808MOI 11.
3) 380VMI5 4) TVMO8FE
‹¬¶KÇ# (3-25) : D ãH÷Ok ㄬ‰×߁…Õ (…è^¥) XY (…è^¥) MN
(X) (1) (2) (3) (4)
ªÖ<Œ…Õ J^Î=œ ò#° LOz# U~¡æ_È° 'ã„u¬ aOƒìxßÑ

Y
AB

ã„H¬ ø› # W=Þ|_# ã„`¬ Œ¼=¶ß†Ç¶ #°O_ H›#°Qù#O_? 12.

M
3. (X) (1) (2) (3) (4)

E
13.

D
9
(X) (1) (2) (3) (4)

59
(SSC- 2012)

CA (1) (2) (3) (4)

95
(X)

4.

50
14.
98
A
:8
(X) (1) (2) (3) (4)
(1) (2) (3) (4)
E

(X)
h

(SSC- 2012)
.P
IC

5. 15.
AD
AB
O

(X) (1) (2) (3) (4) (X) (1) (2) (3) (4)
ER
CH

6. (SSC- 2010)
YD

16.
,H

(X) (1) (2) (3) (4)


T
AM

(X) (1) (2) (3) (4)


H

7.
M

(SSC- 2011)
AM
IG

(X) (1) (2) (3) (4)


17.
KH

(SSC (10 + 2)2012)


R

(X) (1) (2) (3) (4)


8.
(X) (1) (2) (3) (4) 18.
(MBA 2008)

(X) (1) (2) (3) (4)


9. (SSC - 2012)

(X)
B (1) (2) (3) (4) 19.
(SSC- 2011)
(X) (1) (2) (3) (4)
(MBA - 2006)

REASONING ABILITY (NON-VERBAL) ○ ○ ○ ○ ○ ○


236
○ ○ ○ ○ ○ ○ ○ ○ ○ ○ ○ ○ ○ ○ ○ ○ ○ ○ ○
Study Material
20. 23.

(X) (1) (2) (3) (4) (X) (1) (2) (3) (4)
(Police - 2006) (SSC - 2012)

24. XH› Q®_†


 ¶Ç ~¡O †³òH›ø ㄬuaO|O J^ÎOí …Õ KǶ„¬ô ‹¬=°†Ç°O
21.
8:33 J~ò# J‹¬° Q®_†Ç¶~¡O…Õ ‹¬=°†Ç°O
(X) (1) (2) (3) (4)
1) 2:27 2) 3:27 3) 4:27 4) 3:37
(SSC - 2011)
25. XH› Q®_†Ç¶~¡O…Õx ‹¬=°†Ç°O 10 xII° `Ç䛽ø= 10
J~ò#k. J~ò# J^ÎOí …Õ P Q®_†  ¶Ç ~¡O †³òH›ø ㄬuaO|O
22. KǶ„¬ô ‹¬=°†Ç°O

Y
1) 3:10 2) 2:10 3) 2:50 4) 1:10

M
(X) (1) (2) (3) (4)
(SSC - 2008)

E
1234567890123456789012
12345678901234567890123456789012123456789012345678901234567890121234567890123456789012345678901212345678901234567890123
12345678901234567890123

q=~¡}ì`ÇàH› ["Œ|°°
12345678901234567890123456789012123456789012345678901234567890121234567890123456789012345678901212345678901234567890123
1234567890123456789012 12345678901234567890123

D
9
1234567890123456789012
12345678901234567890123456789012123456789012345678901234567890121234567890123456789012345678901212345678901234567890123
12345678901234567890123
1234567890123456789012
TEST YOURSELF :
12345678901234567890123456789012123456789012345678901234567890121234567890123456789012345678901212345678901234567890123
12345678901234567890123

59
1234567890123456789012
12345678901234567890123456789012123456789012345678901234567890121234567890123456789012345678901212345678901234567890123
1234567890123456789012 12345678901234567890123
12345678901234567890123456789012123456789012345678901234567890121234567890123456789012345678901212345678901234567890123
12345678901234567890123
1234567890123456789012
12345678901234567890123456789012123456789012345678901234567890121234567890123456789012345678901212345678901234567890123
12345678901234567890123

CA
95
„¬^ÎO J^ΜO ㄬuaO|O PHê~¡O J^ΜO ㄬuaO|O

50
98
A
1. (2) 486AMO35 53OMA684
:8
7. (1)
E

„¬^ÎO J^ΜO ㄬuaO|O


h
.P
IC

PHê~¡O J^ΜO ㄬuaO|O


AD

2. (2) IOM808MOI IOM808MOI


AB
O

„¬^ÎO J^ΜO ㄬuaO|O


ER

8. (3)
CH

3. (4)
YD

„¬^ÎO J^ΜO ㄬuaO|O


,H

PHê~¡O J^ΜO ㄬuaO|O


T
AM

9. (2)
H
M

4. (2)
AM

PHê~¡O J^ΜO ㄬuaO|O


IG
KH

PHê~¡O J^ΜO ㄬuaO|O


R

10. (2)
5. (4)
PHê~¡O J^ΜO ㄬuaO|O

PHê~¡O J^ΜO ㄬuaO|O 11. (4)

6. (1) PHê~¡O J^ΜO ㄬuaO|O

12. (4)

REASONING ABILITY (NON-VERBAL) ○ ○ ○ ○ ○ ○


237
○ ○ ○ ○ ○ ○ ○ ○ ○ ○ ○ ○ ○ ○ ○ ○ ○ ○ ○
Study Material
‹¬OY¼ J^ΜO ㄬuaO|O PHê~¡O J^ΜO ㄬuaO|O

13. (2) 19. (4)

PHê~¡O J^ΜO ㄬuaO|O


PHê~¡O J^ΜO ㄬuaO|O
14. (1)
20. (2)
PHê~¡O J^ΜO ㄬuaO|O

15. (2) „¬^ÎO J^ΜO ㄬuaO|O

Y
M
PHê~¡O J^ΜO ㄬuaO|O 21. (4)

E
D
9
16. (3) PHê~¡O J^ΜO ㄬuaO|O

59
CA
95
22. (4)

50
PHê~¡O J^ΜO ㄬuaO|O
98
A
:8
E

17. (4)
PHê~¡O J^ΜO ㄬuaO|O
h
.P
IC
AD

23. (3)
PHê~¡O J^ΜO ㄬuaO|O
AB
O
ER
CH

B
18. (2) PHê~¡O J^ΜO ㄬuaO|O
YD

24. (2) 3:27


,H

25. (2) 2:10


T
AM
H

123456789012345678901234567890121234567890123456789012345678901212
M

12345678901234567890123456789012123456789012345678901234567890121234567890123456789012345678901212345678901234567890123
1234567890123456789012345678901212345678901234567890123456789012123456789012345678901234567890121234567890123
1234567890123456789012345678901212345678901234567890123456789012123456789012345678901234567890121234
123456789012345678901234567890121234567890123456789012345678901212
12345678901234567890123456789012123456789012345678901234567890121234567890123456789012345678901212345678901234567890123456789012123
123456789012345678901234567890121234567890123456789012345678901212345678901234567890123
12345678901234567890123456789012123456789012345678901234567890121234567890123456789012345678901212345678901234567890123
1234567890123456789012345678901212345678901234567890123456789012123456789012345678901234567890121234567890123
1234567890123456789012345678901212345678901234567890123456789012123456789012345678901234567890121234
12345678901234567890123456789012123456789012345678901234567890121234567890123456789012345678901212345678901234567890123456789012123
123456789012345678901234567890121234567890123456789012345678901212345678901234567890123
123456789012345678901234567890121234567890123456789012345678901212
12345678901234567890123456789012123456789012345678901234567890121234567890123456789012345678901212345678901234567890123
1234567890123456789012345678901212345678901234567890123456789012123456789012345678901234567890121234567890123
1234567890123456789012345678901212345678901234567890123456789012123456789012345678901234567890121234
12345678901234567890123456789012123456789012345678901234567890121234567890123456789012345678901212345678901234567890123456789012123
123456789012345678901234567890121234567890123456789012345678901212345678901234567890123
PREVIOUS BITS
AM

12345678901234567890123456789012123456789012345678901234567890121234567890123456789012345678901212345678901234567890123456789012123
123456789012345678901234567890121234567890123456789012345678901212345678901234567890123
12345678901234567890123456789012123456789012345678901234567890121234567890123456789012345678901212345678901234567890123
1234567890123456789012345678901212345678901234567890123456789012123456789012345678901234567890121234567890123
1234567890123456789012345678901212345678901234567890123456789012123456789012345678901234567890121234
123456789012345678901234567890121234567890123456789012345678901212
IG

12345678901234567890123456789012123456789012345678901234567890121234567890123456789012345678901212345678901234567890123456789012123
123456789012345678901234567890121234567890123456789012345678901212345678901234567890123
12345678901234567890123456789012123456789012345678901234567890121234567890123456789012345678901212345678901234567890123
1234567890123456789012345678901212345678901234567890123456789012123456789012345678901234567890121234567890123
1234567890123456789012345678901212345678901234567890123456789012123456789012345678901234567890121234
123456789012345678901234567890121234567890123456789012345678901212
12345678901234567890123456789012123456789012345678901234567890121234567890123456789012345678901212345678901234567890123456789012123
123456789012345678901234567890121234567890123456789012345678901212345678901234567890123
12345678901234567890123456789012123456789012345678901234567890121234567890123456789012345678901212345678901234567890123
1234567890123456789012345678901212345678901234567890123456789012123456789012345678901234567890121234567890123
1234567890123456789012345678901212345678901234567890123456789012123456789012345678901234567890121234
12345678901234567890123456789012123456789012345678901234567890121234567890123456789012345678901212345678901234567890123456789012123
123456789012345678901234567890121234567890123456789012345678901212345678901234567890123
KH

1. TERMINATE †³òH›ø ^Î~¡æ} ㄬuaO|O 1. (3)


R

(T.S. S.I. Mains -2016) J^ΜO


1) TERMINATE 2) ETANIMRET TERMINATE ETANIMRET
3) ETANIMRET 4) ETANIMRET

REASONING ABILITY (NON-VERBAL) ○ ○ ○ ○ ○ ○


238
○ ○ ○ ○ ○ ○ ○ ○ ○ ○ ○ ○ ○ ○ ○ ○ ○ ○ ○
Study Material
7 h\÷…Õ ㄬuaO|O (Water Image)
=òMì¼O‰§°...
G XH› JH›Æ~¡O …è^¥ JOïH#° h\÷…Õ KǶ‹²#@Á~ò`Í Jk `ǁãH÷O^ΰ°Qê H›x„²‹¬°ëOk.
G POQ®Á JH›Æ~¡=¶…Õ C, D, E, H, I, K O =°i†Çò X JH›Æ~Œ †³òH›ø h\÷ ㄬuaOƒì° J"Í JH›Æ~Œ#° ‡ée =ôO\ì~ò.
G J^ÎíO…Õ XH› =‹¬°ë=ô †³òH›ø ㄬuaO|O ‡~¡ÅÞ q…Õ=¶xH÷ Q®°ï~á`Í h\÷…Õ XH› =‹¬°ë=ô †³òH›ø ㄬuaO|O `ǁãH÷O^ΰ=ô`Ç°Ok.

Y
L^¥ :

M
=¼H÷ë PHê~¡O

E
h~¡° 12345678901234
12345678901234 h~¡°

D
9
ㄬuaO|O

59
h\÷ ㄬuaO|O
CA
95
50
G 1 #°O_ 9 =~¡ä›½ Q® JOïH †³òH›ø h\÷ ㄬuaOƒì° :
98
A
:8
E
h
.P
IC

G POQ®Á JH›Æ~¡=¶…Õx z#ß JH›Æ~Œ †³òH›ø h\÷ ㄬuaOƒì° :


AD
AB
O

k
ER
CH
YD
,H

G POQ®Á JH›Æ~¡=¶…Õx ̄^Îí JH›Æ~Œ †³òH›ø h\÷ ㄬuaOƒì° :


T
AM
H
M
AM
IG
KH
R

REASONING ABILITY (NON-VERBAL) ○ ○ ○ ○ ○ ○


239
○ ○ ○ ○ ○ ○ ○ ○ ○ ○ ○ ○ ○ ○ ○ ○ ○ ○ ○
Study Material
123456789012345678901234567890121234567890123456789012345678901212
12345678901234567890123456789012123456789012345678901234567890121234567890123456789012345678901212345678901234567890123
1234567890123456789012345678901212345678901234567890123456789012123456789012345678901234567890121234567890123
1234567890123456789012345678901212345678901234567890123456789012123456789012345678901234567890121234
12345678901234567890123456789012123456789012345678901234567890121234567890123456789012345678901212345678901234567890123456789012123
1234567890123456789012345678901212345678901234567890123456789012123456789012345678901234567890
12345678901234567890123456789012123456789012345678901234567890121234567890123456789012345678901212345678901234567890123
1234567890123456789012345678901212345678901234567890123456789012123456789012345678901234567890121234567890123
1234567890123456789012345678901212345678901234567890123456789012123456789012345678901234567890121234
123456789012345678901234567890121234567890123456789012345678901212
12345678901234567890123456789012123456789012345678901234567890121234567890123456789012345678901212345678901234567890123456789012123
1234567890123456789012345678901212345678901234567890123456789012123456789012345678901234567890
12345678901234567890123456789012123456789012345678901234567890121234567890123456789012345678901212345678901234567890123
1234567890123456789012345678901212345678901234567890123456789012123456789012345678901234567890121234567890123
1234567890123456789012345678901212345678901234567890123456789012123456789012345678901234567890121234
12345678901234567890123456789012123456789012345678901234567890121234567890123456789012345678901212345678901234567890123456789012123
1234567890123456789012345678901212345678901234567890123456789012123456789012345678901234567890
123456789012345678901234567890121234567890123456789012345678901212
PRACTICE TEST - 1
12345678901234567890123456789012123456789012345678901234567890121234567890123456789012345678901212345678901234567890123
1234567890123456789012345678901212345678901234567890123456789012123456789012345678901234567890121234567890123
1234567890123456789012345678901212345678901234567890123456789012123456789012345678901234567890121234
12345678901234567890123456789012123456789012345678901234567890121234567890123456789012345678901212345678901234567890123456789012123
1234567890123456789012345678901212345678901234567890123456789012123456789012345678901234567890
123456789012345678901234567890121234567890123456789012345678901212
12345678901234567890123456789012123456789012345678901234567890121234567890123456789012345678901212345678901234567890123
1234567890123456789012345678901212345678901234567890123456789012123456789012345678901234567890121234567890123
1234567890123456789012345678901212345678901234567890123456789012123456789012345678901234567890121234
12345678901234567890123456789012123456789012345678901234567890121234567890123456789012345678901212345678901234567890123456789012123
1234567890123456789012345678901212345678901234567890123456789012123456789012345678901234567890
12345678901234567890123456789012123456789012345678901234567890121234567890123456789012345678901212345678901234567890123
1234567890123456789012345678901212345678901234567890123456789012123456789012345678901234567890121234567890123
1234567890123456789012345678901212345678901234567890123456789012123456789012345678901234567890121234
12345678901234567890123456789012123456789012345678901234567890121234567890123456789012345678901212345678901234567890123456789012123
1234567890123456789012345678901212345678901234567890123456789012123456789012345678901234567890
123456789012345678901234567890121234567890123456789012345678901212
12345678901234567890123456789012123456789012345678901234567890121234567890123456789012345678901212345678901234567890123456789012123
1234567890123456789012345678901212345678901234567890123456789012123456789012345678901234567890
12345678901234567890123456789012123456789012345678901234567890121234567890123456789012345678901212345678901234567890123
1234567890123456789012345678901212345678901234567890123456789012123456789012345678901234567890121234567890123
1234567890123456789012345678901212345678901234567890123456789012123456789012345678901234567890121234

‹¬¶KÇ# (1-25) : D ãH÷O^Î W=Þ|_# „¬\ì †³òH›ø h\÷ ㄬuaOƒì#° W=Þ|_# ‹¬=¶^¥# zã`Œ #°O_ Q®°iëOKÇO_.
‹¬=°ª¼ zã`ÇO ‹¬=¶^¥# zã`Œ°
1.

(a) (b) (c) (d)

2.

Y
M
(a) (b) (c) (d)

3.

E
D
9
59
(a) (b) (c) (d)

CA
95
4.

50
98
A
:8
(a) (b) (c) (d)
E
h
.P
IC

5.
AD
AB
O

(a) (b) (c) (d)


ER
CH

6.
YD
,H

(a) (b) (c) (d)


T
AM
H

7.
M

(SSC CPO Sub-Insepector Exam, 03.09.2006)


AM
IG

(a) (b) (c) (d)


KH
R

8.
(a) (b) (c) (d)

9. (SSC Graduate Level Tier-I Exam, 19.05.2013, (1st Sitting)

(a) (b) (c) (d)

10. (SSC Combined Matric Level (Pre) Exam,

30.07.2006 (2nd Sitting) (Central Zone)


(a) (b) (c) (d)

REASONING ABILITY (NON-VERBAL) ○ ○ ○ ○ ○ ○


240
○ ○ ○ ○ ○ ○ ○ ○ ○ ○ ○ ○ ○ ○ ○ ○ ○ ○ ○
Study Material
11. (SSC Combined Matric Level (Pre) Exam,

30.07.2006 (1st Sitting) (East Zone)

(a) (b) (c) (d)

12. (SSC Combined Matric Level (Pre) Exam,

21.05.2000 (1st Sitting) (Middle Zone)


(a) (b) (c) (d)

13. (SSC Combined Graduate Level Prelim Exam,

04-07-1999 (1st Sitting)


(a) (b) (c) (d)

Y
14.

M
(a) (b) (c) (d)

E
D
9
15.

59
CA
95
50
(a) (b) (c) (d)

98
A
16.
:8
E
h
.P
IC

(a) (b) (c) (d)


AD
AB
O

17. (LIC 2001)


ER
CH
YD

(a) (b) (c) (d)


,H
T
AM

18. (a) (b) (c) (d)


H
M
AM
IG
KH

19. (a) (b) (c) (d)


R

20. (a) (b) (c) (d)

21. (a) (b) (c) (d)

22. (a) (b) (c) (d)

REASONING ABILITY (NON-VERBAL) ○ ○ ○ ○ ○ ○


241
○ ○ ○ ○ ○ ○ ○ ○ ○ ○ ○ ○ ○ ○ ○ ○ ○ ○ ○
Study Material
23. (a) (b) (c) (d)

24. (a) (b) (c) (d)

25. (a) (b) (c) (d)

PRACTICE TEST - 1 : ["Œ|°°

Y
1) d 2) c 3) d 4) c 5) d 6) d 7) c 8) b 9) c 10) d

M
11) d 12) c 13) b 14) c 15) a 16) c 17) b 18) b 19) d 20) a

E
21) a 22) a 23) c 24) b 25) a

D
9
123456789012345678901234567890121234567890123456789012345678901212
12345678901234567890123456789012123456789012345678901234567890121234567890123456789012345678901212345678901234567890123
1234567890123456789012345678901212345678901234567890123456789012123456789012345678901234567890121234567890123
1234567890123456789012345678901212345678901234567890123456789012123456789012345678901234567890121234
12345678901234567890123456789012123456789012345678901234567890121234567890123456789012345678901212345678901234567890123456789012123
1234567890123456789012345678901212345678901234567890123456789012123456789012345678901234567890
123456789012345678901234567890121234567890123456789012345678901212

59
12345678901234567890123456789012123456789012345678901234567890121234567890123456789012345678901212345678901234567890123
1234567890123456789012345678901212345678901234567890123456789012123456789012345678901234567890121234567890123
1234567890123456789012345678901212345678901234567890123456789012123456789012345678901234567890121234
12345678901234567890123456789012123456789012345678901234567890121234567890123456789012345678901212345678901234567890123456789012123
1234567890123456789012345678901212345678901234567890123456789012123456789012345678901234567890
123456789012345678901234567890121234567890123456789012345678901212
12345678901234567890123456789012123456789012345678901234567890121234567890123456789012345678901212345678901234567890123456789012123
1234567890123456789012345678901212345678901234567890123456789012123456789012345678901234567890
12345678901234567890123456789012123456789012345678901234567890121234567890123456789012345678901212345678901234567890123
1234567890123456789012345678901212345678901234567890123456789012123456789012345678901234567890121234567890123
1234567890123456789012345678901212345678901234567890123456789012123456789012345678901234567890121234
PRACTICE TEST - 2
12345678901234567890123456789012123456789012345678901234567890121234567890123456789012345678901212345678901234567890123456789012123
1234567890123456789012345678901212345678901234567890123456789012123456789012345678901234567890
12345678901234567890123456789012123456789012345678901234567890121234567890123456789012345678901212345678901234567890123
1234567890123456789012345678901212345678901234567890123456789012123456789012345678901234567890121234567890123
1234567890123456789012345678901212345678901234567890123456789012123456789012345678901234567890121234
123456789012345678901234567890121234567890123456789012345678901212
12345678901234567890123456789012123456789012345678901234567890121234567890123456789012345678901212345678901234567890123456789012123
1234567890123456789012345678901212345678901234567890123456789012123456789012345678901234567890

CA
12345678901234567890123456789012123456789012345678901234567890121234567890123456789012345678901212345678901234567890123
1234567890123456789012345678901212345678901234567890123456789012123456789012345678901234567890121234567890123
1234567890123456789012345678901212345678901234567890123456789012123456789012345678901234567890121234
123456789012345678901234567890121234567890123456789012345678901212

95
12345678901234567890123456789012123456789012345678901234567890121234567890123456789012345678901212345678901234567890123456789012123
1234567890123456789012345678901212345678901234567890123456789012123456789012345678901234567890
12345678901234567890123456789012123456789012345678901234567890121234567890123456789012345678901212345678901234567890123
1234567890123456789012345678901212345678901234567890123456789012123456789012345678901234567890121234567890123
1234567890123456789012345678901212345678901234567890123456789012123456789012345678901234567890121234
12345678901234567890123456789012123456789012345678901234567890121234567890123456789012345678901212345678901234567890123456789012123
1234567890123456789012345678901212345678901234567890123456789012123456789012345678901234567890
12345678901234567890123456789012123456789012345678901234567890121234567890123456789012345678901212345678901234567890123
1234567890123456789012345678901212345678901234567890123456789012123456789012345678901234567890121234567890123
1234567890123456789012345678901212345678901234567890123456789012123456789012345678901234567890121234
12345678901234567890123456789012123456789012345678901234567890121234567890123456789012345678901212345678901234567890123456789012123
1234567890123456789012345678901212345678901234567890123456789012123456789012345678901234567890

50
‹¬¶KÇ# D ãH÷O^Î W=Þ|_# JO‰×=ò †³òH›ø h\÷ ㄬu 8. ab45CD67
98
(1-25):
A
aOƒì#° W=Þ|_# P„¬Â#° #°O_ ‹¬i†³°Ø # ^¥xx ZOKÇ°HËO_. 1) ab45CD67 2) ap45CD67
:8
3) ab45CD67 4) ab45CD 6 7
E

1. 13add7LM
h
.P

1) 13add7LM 2) 13a dd7L M 9. RAJ589D8


IC

1) RAJ589D8 2) RAJ589D8
AD

3) 1 3a dd7 LM 4) 13adp LM 7
RAJ 89D8
3) RAJ589D8
AB
O

2. Stage 4) 5
2) Stage
ER

10. BK50RP62
CH

1) Stage
4) Stag e 1) BK 2) BK50RP62
YD

3) Stage
26PR05

3. HIDE 3) BK50RP62 4) BK50RP62


,H
T

1) HIDE 2) HIDE 11. PRAISED9743DEVOTE


AM

1) P RAISED9743DEVO T E
H

3) HIDE 4) HIDE
M

2) PRAISED9743DEVOTE
AM

4. COLLECT
IG

1) COLLECT 2) COLLECT 3) PRAISED9743DE V OTE


KH

PRAISED9743DEVOTE
3) CO LLECT 4) COLLECT 4)
R

5. POLICE 12. YUGOSLAVIA


1) POLICE 2) POLICE 1) YUGOSLAVIA 2) YU GOSLAVIA
YUGOSLAVIA UGOSLAVIA
3) POLICE 4) PO L ICE 3) 4) Y
6. DL2CA34 (IMO 2012) 13. DL3R1N2554
1) DL2CA34 2) DL2CA
3
4 1) DL3R1N2554 2) DL3 R1N255 4
DL2CA34 R1N 554 1N255
3) DL2CA34 4) 3) DL3 2 4) DL3R 4
7. QUARREL (NSO 2012) 14. S5L3T8
1) QUARREL 2) QU ARREL 1) S5L3T8 2) S 5 L3T8
S5L T8 S5L
3) QUARREL 4) Q UARREL 3) 3 4) 3T8

REASONING ABILITY (NON-VERBAL) ○ ○ ○ ○ ○ ○


242
○ ○ ○ ○ ○ ○ ○ ○ ○ ○ ○ ○ ○ ○ ○ ○ ○ ○ ○
Study Material
15. HEDIOUS 21. N 4 t Q j 3
1) HEDIOUS 2) HEDIOUS 1) N 4 t Q j 3 2) N 4 t Q j 3
U N tQj3 N4tQj3
3) HEDIOS 4) HEDIO S 3) 4 4)
16. PROEM 22. R A J 5 8 9 D 8
1) P ROEM 2) PROEM 1) RAJ589D8 2) RAJ589D8
PROEM PRO EM RAJ589D8 RAJ589D8
3) 4) 3) 4)
17. ERUDITE 23. 3 8 4 6 0 17
1) ERUDITE 2) ERUDI TE 1) 384601 7
2) 7106403
ERUDI E ER 46
3) T 4) UDITE 3) 38 017 4) 3081706
18. WEAK 24. CONNOISSEUR
1) WEAK 2) W EAK 3) W EAK 4) WEAK

Y
1) CONNOISSEU R
2) CONNOISSEUR

M
19. gentle 3) CONNOISSEU 4) CONNOISSE
R RU

1) g e ntle 2) gentle 3) geutle 4) gentle 25. KIBITZ

E
20. TALKATIVE 1) KIBITZ 2) KIBI^Z
TALKATIVE TALKATI

D
9
1) 2) VE 3) KIBI^Z 4) KIBITZ

59
TALKATIVE TALK TIVE
3) 4) A
CA
95
50
98
A
PRACTICE TEST - 2 : ["Œ|°°
:8
E

1) 1 2) 2 3) 1 4) 3 5) 2 6) 4 7) 1 8) 1 9) 1 10) 1
h
.P
IC

11) 4 12) 3 13) 1 14) 3 15) 1 16) 3 17) 1 18) 4 19) 2 20) 3
AD

21) 1 22) 1 23) 2 24) 4 25) 3


AB
O
ER
CH
YD
,H
T
AM
H
M
AM
IG
KH
R

REASONING ABILITY (NON-VERBAL) ○ ○ ○ ○ ○ ○


243
○ ○ ○ ○ ○ ○ ○ ○ ○ ○ ○ ○ ○ ○ ○ ○ ○ ○ ○
Study Material
8 À„„¬~Ÿ

À„„¬~Ÿ
‡¦éeÛOQ·

‡¦éeÛOQ·

(Paper folding & Cutting)


Ê

Ê
H›\÷“OQ·

H›\÷“OQ·

=òMì¼O‰§°...
G XH› ‡~¡^Î~¡ÅH› KÇ`Ç°~¡ãªHê~¡„¬ô À„„¬~Ÿ#° =°_È°KÇ°@ (‡¦é…˜Û K͆Çò@) :
D `Ç~¡‚¬ð ㄬ‰×߁…Õ XH› KÇ`Ç°~¡ãªHê~¡„¬ô ‡~¡^Î~¡ÅH› À„„¬~Ÿ KǶ„¬|_È°`Ç°Ok. P À„„¬~ŸÌ„á XH› _*ÿá<£ =°i†Çò KÇ°H›ø
w`Ç W=Þ|_È°`Ç°Ok.

Y
P KÇ°H›ø w`Ç̄á À„„¬~Ÿ#° =°_È°=Qê U~¡æ_Í ãHù`Çë _*ÿá<£#° Jƒ’¼iÖ H›#°Qù<Œe.

M
=¶ki ㄬ‰×߁°

E
D
9
59
1. XH› ‡~¡^Î~¡ÅH› À„„¬~Ÿ#° Wzó# KÇ°H›ø w`ǁ "³O|_ "³O|_ =°_z# 1/4= =O`Ç° =$`ÇëO U~¡æ_È°#°. Z
CA
95
ㄬ‰×ß…Õ J_y# q^ÎOQê =°_z ^¥x…Õx U^³á<Œ XH› …Õ KǶ„²# q^ÎOQê H›uëiOz# =$`ŒëxH÷ <Œ°Q®°

50
ƒìQêxß H›uëiOz uiy P À„„¬~Ÿ =°_È`ǁ#° q„²æ# "³á„¬ô…ì J^Í _*ÿá#° =‹¬°ëOk. <Œ°Q®° "³á„¬ô…ì J^Í

98
A
`Ç~¡°"Œ`Ç ƒ’¼=°†Í°¼ PHê~¡O Q®°iëOKÇ°@ : _*ÿá#°Q® „¬@O ‹¬æ+¬“OQê (b).
:8
D ãH÷O^Î W=Þ|_# =ü_È° „¬\쁰 x, y =°i†Çò z 2. D ãH÷O^Î W=Þ|_# KÇ`°Ç ~¡ãªHê~¡„ô¬ ‡~¡^~Î Å¡ H› À„„¬~#
Ÿ °
E
h

#° „¬ijeOz U~¡æ_È° „¬\ìxß H›#°Qù#O_.


.P

KÇ°H›øw`Ç „¬~O
¡ Qê =°_z#„¬ô_È° U~¡æ_È° „¬@O#° Wzó#
IC
AD

P„¬Â<£ #°O_ ‹¬i†³Ø°#^¥xx ZOKÇ°Hù#O_.


AB
O
ER
CH

X Y Z
YD

(X) (a) (b) (c) (d)


,H

(a) (b) (c) (d)

ª^Î#: Wzó# À„„¬~Ÿ#° KÇ°H›øw`Ç „¬~¡OQê =°_z#„¬ô_È°


T
AM

ª^Î#: X XH› =$`ÇëO, ^¥xx KÇ°H›ø w`Ç "³O|_ =°_z# 䛽_"³á„¬ô# Q® KÇ°H›ø Z_È=°"³á„¬ôQ® KÇ°H›ø =°^Î¼ä›½
H
M

J~¡=
Ö $`ÇO
ë U~¡æ_È°#°. Y XH› J~¡=œ $`ÇOë . ^¥xx KÇ°H›øw`Ç =‹¬°O
ë ^Îx ‹¬æ+¬O
“ Qê `³°‹¬°O
ë k. Hê=ô# ‹¬=¶^¥#O (b).
AM
IG

123456789012345678901234567890121234567890123456789012345678901212
12345678901234567890123456789012123456789012345678901234567890121234567890123456789012345678901212345678901234567890123
1234567890123456789012345678901212345678901234567890123456789012123456789012345678901234567890121234567890123
1234567890123456789012345678901212345678901234567890123456789012123456789012345678901234567890121234
12345678901234567890123456789012123456789012345678901234567890121234567890123456789012345678901212345678901234567890123456789012123
1234567890123456789012345678901212345678901234567890123456789012123456789012345678901234567890
12345678901234567890123456789012123456789012345678901234567890121234567890123456789012345678901212345678901234567890123
1234567890123456789012345678901212345678901234567890123456789012123456789012345678901234567890121234567890123
1234567890123456789012345678901212345678901234567890123456789012123456789012345678901234567890121234
123456789012345678901234567890121234567890123456789012345678901212
12345678901234567890123456789012123456789012345678901234567890121234567890123456789012345678901212345678901234567890123456789012123
1234567890123456789012345678901212345678901234567890123456789012123456789012345678901234567890
KH

12345678901234567890123456789012123456789012345678901234567890121234567890123456789012345678901212345678901234567890123
1234567890123456789012345678901212345678901234567890123456789012123456789012345678901234567890121234567890123
1234567890123456789012345678901212345678901234567890123456789012123456789012345678901234567890121234
12345678901234567890123456789012123456789012345678901234567890121234567890123456789012345678901212345678901234567890123456789012123
1234567890123456789012345678901212345678901234567890123456789012123456789012345678901234567890
123456789012345678901234567890121234567890123456789012345678901212
PRACTICE TEST - 1
12345678901234567890123456789012123456789012345678901234567890121234567890123456789012345678901212345678901234567890123
1234567890123456789012345678901212345678901234567890123456789012123456789012345678901234567890121234567890123
1234567890123456789012345678901212345678901234567890123456789012123456789012345678901234567890121234
12345678901234567890123456789012123456789012345678901234567890121234567890123456789012345678901212345678901234567890123456789012123
1234567890123456789012345678901212345678901234567890123456789012123456789012345678901234567890
123456789012345678901234567890121234567890123456789012345678901212
12345678901234567890123456789012123456789012345678901234567890121234567890123456789012345678901212345678901234567890123456789012123
1234567890123456789012345678901212345678901234567890123456789012123456789012345678901234567890
12345678901234567890123456789012123456789012345678901234567890121234567890123456789012345678901212345678901234567890123
1234567890123456789012345678901212345678901234567890123456789012123456789012345678901234567890121234567890123
1234567890123456789012345678901212345678901234567890123456789012123456789012345678901234567890121234
12345678901234567890123456789012123456789012345678901234567890121234567890123456789012345678901212345678901234567890123456789012123
1234567890123456789012345678901212345678901234567890123456789012123456789012345678901234567890
12345678901234567890123456789012123456789012345678901234567890121234567890123456789012345678901212345678901234567890123
1234567890123456789012345678901212345678901234567890123456789012123456789012345678901234567890121234567890123
1234567890123456789012345678901212345678901234567890123456789012123456789012345678901234567890121234
123456789012345678901234567890121234567890123456789012345678901212
R

12345678901234567890123456789012123456789012345678901234567890121234567890123456789012345678901212345678901234567890123456789012123
1234567890123456789012345678901212345678901234567890123456789012123456789012345678901234567890
12345678901234567890123456789012123456789012345678901234567890121234567890123456789012345678901212345678901234567890123
1234567890123456789012345678901212345678901234567890123456789012123456789012345678901234567890121234567890123
1234567890123456789012345678901212345678901234567890123456789012123456789012345678901234567890121234

‹¬¶KÇ# (1-20) : D ãH÷O^Î W=Þ|_# ㄬ‰×߁° À„„¬~Ÿ H›uëiOKÇ°@䛽 ‹¬O|OkOz#q. ãH÷Ok zã`Œ…Õ Wzó# _¨\˜ž ㄬHê~¡O
H›uëiÀ‹ë Z…ìO\÷ zã`ÇO =‹¬°ëO^Ë P À„„¬~Ÿ H›\ ÷OQ·#° T‚²ìOz H›#°Qù<Œe.
‹¬=°ª¼ zã`Œ° ‹¬=¶^¥# zã`Œ°

1.

X Y Z (a) (b) (c) (d)

2.
X Y Z (a) (b) (c) (d)

REASONING ABILITY (NON-VERBAL) ○ ○ ○ ○ ○ ○


244
○ ○ ○ ○ ○ ○ ○ ○ ○ ○ ○ ○ ○ ○ ○ ○ ○ ○ ○
Study Material
3.

X Y Z (a) (b) (c) (d)

4.

X Y Z (a) (b) (c) (d)

5.

X Y Z

Y
(a) (b) (c) (d)

M
6.

E
X Y Z (a) (b) (c) (d)

D
9
59
7.
CA
95
50
X Y Z (a) (b) (c) (d)

98
A
:8
8.
E
h
.P

X Y Z
IC

(a) (b) (c) (d)


AD

9.
AB
O
ER
CH

X Y Z (a) (b) (c) (d)


YD

10.
,H
T
AM

X Y Z
H

(a) (b) (c) (d)


M
AM

11.
IG
KH
R

X Y Z (a) (b) (c) (d)

12.

X Y Z (a) (b) (c) (d)

13. (a) (b) (c) (d)

14. (a) (b) (c) (d)

X Y Z
REASONING ABILITY (NON-VERBAL) ○ ○ ○ ○ ○ ○
245
○ ○ ○ ○ ○ ○ ○ ○ ○ ○ ○ ○ ○ ○ ○ ○ ○ ○ ○
Study Material
1234567890123456789012
12345678901234567890123456789012123456789012345678901234567890121234567890123456789012345678901212345678901234567890123
12345678901234567890123
12345678901234567890123456789012123456789012345678901234567890121234567890123456789012345678901212345678901234567890123
1234567890123456789012 12345678901234567890123
12345678901234567890123456789012123456789012345678901234567890121234567890123456789012345678901212345678901234567890123
1234567890123456789012
1234567890123456789012
q=~¡}ì`ÇàH› ["Œ|°°
12345678901234567890123
12345678901234567890123456789012123456789012345678901234567890121234567890123456789012345678901212345678901234567890123
PRACTICE TEST - 1 :
12345678901234567890123
12345678901234567890123456789012123456789012345678901234567890121234567890123456789012345678901212345678901234567890123
1234567890123456789012
12345678901234567890123456789012123456789012345678901234567890121234567890123456789012345678901212345678901234567890123
1234567890123456789012
12345678901234567890123
12345678901234567890123
12345678901234567890123456789012123456789012345678901234567890121234567890123456789012345678901212345678901234567890123
1234567890123456789012 12345678901234567890123

1. (c) 7. (b)

Z (c) Z (b)

2. (c) 8. (c)

Z (c) Z (c)
9. (d)
3. (b)

Y
Z (d)
Z (b)

M
10. (b)

E
4. (b)
Z (b)

D
9
Z (b)

59
11. (b)
CA
95
5. (c) Z (b)

50
98
A
Z (c) 12. (a)
:8
E

Z (a)
h

6. (a)
.P
IC
AD

Z (a) 13. (d) 14. (d)


AB
O

(d) (d)
ER
CH

123456789012345678901234567890121234567890123456789012345678901212
12345678901234567890123456789012123456789012345678901234567890121234567890123456789012345678901212345678901234567890123456789012123
1234567890123456789012345678901212345678901234567890123456789012123456789012345678901234567890
12345678901234567890123456789012123456789012345678901234567890121234567890123456789012345678901212345678901234567890123
1234567890123456789012345678901212345678901234567890123456789012123456789012345678901234567890121234567890123
1234567890123456789012345678901212345678901234567890123456789012123456789012345678901234567890121234
123456789012345678901234567890121234567890123456789012345678901212
12345678901234567890123456789012123456789012345678901234567890121234567890123456789012345678901212345678901234567890123456789012123
1234567890123456789012345678901212345678901234567890123456789012123456789012345678901234567890
YD

12345678901234567890123456789012123456789012345678901234567890121234567890123456789012345678901212345678901234567890123
1234567890123456789012345678901212345678901234567890123456789012123456789012345678901234567890121234567890123
1234567890123456789012345678901212345678901234567890123456789012123456789012345678901234567890121234
12345678901234567890123456789012123456789012345678901234567890121234567890123456789012345678901212345678901234567890123456789012123
1234567890123456789012345678901212345678901234567890123456789012123456789012345678901234567890
123456789012345678901234567890121234567890123456789012345678901212
12345678901234567890123456789012123456789012345678901234567890121234567890123456789012345678901212345678901234567890123
1234567890123456789012345678901212345678901234567890123456789012123456789012345678901234567890121234567890123
1234567890123456789012345678901212345678901234567890123456789012123456789012345678901234567890121234
PRACTICE TEST - 2
12345678901234567890123456789012123456789012345678901234567890121234567890123456789012345678901212345678901234567890123456789012123
1234567890123456789012345678901212345678901234567890123456789012123456789012345678901234567890
123456789012345678901234567890121234567890123456789012345678901212
12345678901234567890123456789012123456789012345678901234567890121234567890123456789012345678901212345678901234567890123
1234567890123456789012345678901212345678901234567890123456789012123456789012345678901234567890121234567890123
1234567890123456789012345678901212345678901234567890123456789012123456789012345678901234567890121234
12345678901234567890123456789012123456789012345678901234567890121234567890123456789012345678901212345678901234567890123456789012123
1234567890123456789012345678901212345678901234567890123456789012123456789012345678901234567890
12345678901234567890123456789012123456789012345678901234567890121234567890123456789012345678901212345678901234567890123456789012123
1234567890123456789012345678901212345678901234567890123456789012123456789012345678901234567890
12345678901234567890123456789012123456789012345678901234567890121234567890123456789012345678901212345678901234567890123
1234567890123456789012345678901212345678901234567890123456789012123456789012345678901234567890121234567890123
1234567890123456789012345678901212345678901234567890123456789012123456789012345678901234567890121234
123456789012345678901234567890121234567890123456789012345678901212
12345678901234567890123456789012123456789012345678901234567890121234567890123456789012345678901212345678901234567890123456789012123
1234567890123456789012345678901212345678901234567890123456789012123456789012345678901234567890
12345678901234567890123456789012123456789012345678901234567890121234567890123456789012345678901212345678901234567890123
1234567890123456789012345678901212345678901234567890123456789012123456789012345678901234567890121234567890123
1234567890123456789012345678901212345678901234567890123456789012123456789012345678901234567890121234
,H

12345678901234567890123456789012123456789012345678901234567890121234567890123456789012345678901212345678901234567890123456789012123
1234567890123456789012345678901212345678901234567890123456789012123456789012345678901234567890
T

D ãH÷O^Î W=Þ|_# ㄬ‰×߁…Õ X J#°#k XH› ‡~¡^Î~¡ÅH› À„„¬~¡°. ^¥x̄á KÇ°H›øw`Ç W=Þ|_#k. P KÇ°H›ø w`Ç „¬~¡OQê
AM

;
H

À„„¬~Ÿ#° =°_z#„¬ô_È° U~¡æ_Í ãHù`Çë „¬@O#° Wzó# P„¬Â<£ #°O_ H›#°Qù#O_.


M
AM
IG

‹¬=°‹¬¼ zã`ÇO ‹¬=¶^¥# zã`Œ°


KH
R

1.

„¬@=ò (X) (a) (b) (c) (d)

2.

„¬@=ò (X) (a) (b) (c) (d)

3.

„¬@=ò (X) (a) (b) (c) (d)

REASONING ABILITY (NON-VERBAL) ○ ○ ○ ○ ○ ○


246
○ ○ ○ ○ ○ ○ ○ ○ ○ ○ ○ ○ ○ ○ ○ ○ ○ ○ ○
Study Material
4.

„¬@=ò (X) (a) (b) (c) (d)

5.

„¬@=ò (X) (a) (b) (c) (d)

6.

Y
„¬@=ò (X) (a) (b) (c) (d)

M
7.

E
„¬@=ò (X)

D
9
(a) (b) (c) (d)

59
CA
95
8.

50
98
A
„¬@=ò (X) (a) (b) (c) (d)
:8
E
h

9.
.P
IC
AD

„¬@=ò (X) (a) (b) (c) (d)


AB
O
ER
CH

10.
YD

„¬@=ò (X) (a) (b) (c) (d)


,H
T
AM

11.
H
M

„¬@=ò (X)
AM
IG

(a) (b) (c) (d)


KH

12.
R

„¬@=ò (X) (a) (b) (c) (d)

13.

„¬@=ò (X) (a) (b) (c) (d)

14.

„¬@=ò (X) (a) (b) (c) (d)

REASONING ABILITY (NON-VERBAL) ○ ○ ○ ○ ○ ○


247
○ ○ ○ ○ ○ ○ ○ ○ ○ ○ ○ ○ ○ ○ ○ ○ ○ ○ ○
Study Material
15.

„¬@=ò (X) (a) (b) (c) (d)

16.

„¬@=ò (X) (a) (b) (c) (d)

17.

„¬@=ò (X) (a) (b) (c) (d)

Y
M
18.

E
„¬@=ò (X) (a) (b) (c) (d)

D
9
1234567890123456789012
12345678901234567890123456789012123456789012345678901234567890121234567890123456789012345678901212345678901234567890123
12345678901234567890123

q=~¡}ì`ÇàH› ["Œ|°°

59
12345678901234567890123456789012123456789012345678901234567890121234567890123456789012345678901212345678901234567890123
1234567890123456789012
1234567890123456789012 12345678901234567890123
12345678901234567890123456789012123456789012345678901234567890121234567890123456789012345678901212345678901234567890123
12345678901234567890123
PRACTICE TEST - 2 :
1234567890123456789012
12345678901234567890123456789012123456789012345678901234567890121234567890123456789012345678901212345678901234567890123
12345678901234567890123
1234567890123456789012
CA 12345678901234567890123
12345678901234567890123456789012123456789012345678901234567890121234567890123456789012345678901212345678901234567890123

95
1234567890123456789012
12345678901234567890123456789012123456789012345678901234567890121234567890123456789012345678901212345678901234567890123
1234567890123456789012 12345678901234567890123
12345678901234567890123
12345678901234567890123456789012123456789012345678901234567890121234567890123456789012345678901212345678901234567890123

50
(a)
98
A
1. (c) 10.
:8
E
h
.P

(d)
IC

2. 11. (b)
AD
AB
O

3. (a) 12. (d)


ER
CH
YD

(b)
,H

4.
13. (b)
T
AM
H
M

5. (c) (c)
AM

14.
IG
KH
R

6. (b) 15. (d)

7. (b) 16. (a)

8. (a) 17. (b)

9. (d) 18. (a)

REASONING ABILITY (NON-VERBAL) ○ ○ ○ ○ ○ ○


248
○ ○ ○ ○ ○ ○ ○ ○ ○ ○ ○ ○ ○ ○ ○ ○ ○ ○ ○
Study Material
XöH ^Î~Œà° H›e¾# „¬\ì#° Q®°iëOKÇ°@
9 (Finding out Identical figures)

=òMì¼O‰§°...
G D qƒìQ®O…Õ Hùxß „¬\쁰 XH› KÇ`Ç°~¡ãªHê~¡„¬ô „¬@O…Õ KǶ„¬|_È`Œ~ò. "Œ\÷…Õ L=°à_ ^Î~Œà° H›ey# „¬\ì
‹¬=ü‚¬ìO#° Q®°iëOKŒe.

Y
=¶ki ㄬ‰×߁°

M
E
1. „¬@O X H›ey†Çò#ß Q®°~¡°ëhß H›ey†Çò#ß „¬\ìxß A, B, C =°i†Çò D …Õ U „¬@O H›ey†ÇòO^Ë Q®°iëOKÇO_.

D
9
59
CA
95
50
(X)
98
A
(a) (b) (c) (d)
:8
ª^Î#: X#° ä›Æ½}âOQê „¬ijeOz# g°^Î@ X…Õx Q®°~¡°ëhß „¬@O A H›ey†Çò#ßk.
\
E

‹¬=¶^¥#O (a).
h
.P
IC
AD

2. 3.
AB
O
ER
CH
YD
,H
T
AM
H
M
AM

ª^Î#: 1, 6, 9° ãuƒ’°*쁰


IG

ª^Î#: Wzó# „¬\ìxß „¬ijeOKÇQê 4, 7, 9 ° Mìm „¬\쁰.


3, 4, 7° KÇ`Ç°~¡°Ä*쁰
KH

1, 3, 8 ° =ü_È° ö~MìYO_¨KÍ 6 ƒìQꁰQê


2, 5, 8° „¬OKǃ’°*쁰 qƒ’lO„¬|_# „¬\쁰. 2, 5, 6° ï~O_È° O|ö~Mì
R

(q‰õÁ+¬}) YO_¨KÍ 4 ƒìQꁰQê qƒ’lO„¬|_# „¬\쁰.


(q‰õÁ+²OKÇ°@)

REASONING ABILITY (NON-VERBAL) ○ ○ ○ ○ ○ ○


249
○ ○ ○ ○ ○ ○ ○ ○ ○ ○ ○ ○ ○ ○ ○ ○ ○ ○ ○
Study Material
123456789012345678901234567890121234567890123456789012345678901212
12345678901234567890123456789012123456789012345678901234567890121234567890123456789012345678901212345678901234567890123
1234567890123456789012345678901212345678901234567890123456789012123456789012345678901234567890121234567890123
1234567890123456789012345678901212345678901234567890123456789012123456789012345678901234567890121234
12345678901234567890123456789012123456789012345678901234567890121234567890123456789012345678901212345678901234567890123456789012123
1234567890123456789012345678901212345678901234567890123456789012123456789012345678901234567890
12345678901234567890123456789012123456789012345678901234567890121234567890123456789012345678901212345678901234567890123
1234567890123456789012345678901212345678901234567890123456789012123456789012345678901234567890121234567890123
1234567890123456789012345678901212345678901234567890123456789012123456789012345678901234567890121234
123456789012345678901234567890121234567890123456789012345678901212
12345678901234567890123456789012123456789012345678901234567890121234567890123456789012345678901212345678901234567890123456789012123
1234567890123456789012345678901212345678901234567890123456789012123456789012345678901234567890
12345678901234567890123456789012123456789012345678901234567890121234567890123456789012345678901212345678901234567890123
1234567890123456789012345678901212345678901234567890123456789012123456789012345678901234567890121234567890123
1234567890123456789012345678901212345678901234567890123456789012123456789012345678901234567890121234
12345678901234567890123456789012123456789012345678901234567890121234567890123456789012345678901212345678901234567890123456789012123
1234567890123456789012345678901212345678901234567890123456789012123456789012345678901234567890
123456789012345678901234567890121234567890123456789012345678901212
PRACTICE TEST - 1
12345678901234567890123456789012123456789012345678901234567890121234567890123456789012345678901212345678901234567890123
1234567890123456789012345678901212345678901234567890123456789012123456789012345678901234567890121234567890123
1234567890123456789012345678901212345678901234567890123456789012123456789012345678901234567890121234
12345678901234567890123456789012123456789012345678901234567890121234567890123456789012345678901212345678901234567890123456789012123
1234567890123456789012345678901212345678901234567890123456789012123456789012345678901234567890
123456789012345678901234567890121234567890123456789012345678901212
12345678901234567890123456789012123456789012345678901234567890121234567890123456789012345678901212345678901234567890123
1234567890123456789012345678901212345678901234567890123456789012123456789012345678901234567890121234567890123
1234567890123456789012345678901212345678901234567890123456789012123456789012345678901234567890121234
12345678901234567890123456789012123456789012345678901234567890121234567890123456789012345678901212345678901234567890123456789012123
1234567890123456789012345678901212345678901234567890123456789012123456789012345678901234567890
12345678901234567890123456789012123456789012345678901234567890121234567890123456789012345678901212345678901234567890123
1234567890123456789012345678901212345678901234567890123456789012123456789012345678901234567890121234567890123
1234567890123456789012345678901212345678901234567890123456789012123456789012345678901234567890121234
12345678901234567890123456789012123456789012345678901234567890121234567890123456789012345678901212345678901234567890123456789012123
1234567890123456789012345678901212345678901234567890123456789012123456789012345678901234567890
123456789012345678901234567890121234567890123456789012345678901212
12345678901234567890123456789012123456789012345678901234567890121234567890123456789012345678901212345678901234567890123456789012123
1234567890123456789012345678901212345678901234567890123456789012123456789012345678901234567890
12345678901234567890123456789012123456789012345678901234567890121234567890123456789012345678901212345678901234567890123
1234567890123456789012345678901212345678901234567890123456789012123456789012345678901234567890121234567890123
1234567890123456789012345678901212345678901234567890123456789012123456789012345678901234567890121234

; D ãH÷O^Î W=Þ|_# „¬@O …Õ =ô#ß Q®°~¡°ëxßO\÷h


X
5.
H›ey†Çò#ß „¬\ìxß Q®°iëOKÇO_. (1-3 ㄉ¬ ß× ° D ~¡HêxH÷
K³Ok#q)
1.

(X) 1) 1, 5, 7   2, 4, 6   3, 9, 8
2) 1, 5, 7   2, 4, 8   3, 6, 9
3) 1, 4, 7   2, 5, 8   3, 6, 9

Y
4) 1, 7, 9   3, 5, 8   2, 4, 6

M
(a) (b) (c) (d)
6.

E
2.

D
9
59
CA
95
(X)

50
98
A
1) 3, 4, 9   5, 7, 8   1, 2, 6
:8
2) 1, 5, 6   2, 4, 8   3, 7, 9
E
h

3) 4, 6, 8   3, 5, 7   1, 2, 9
.P
IC

(a) (b) (c) (d)


4) 1, 2, 7   3, 5, 9   4, 6, 8
AD

3.
AB
O

7.
ER
CH

(X)
YD
,H
T
AM

(S.S.C. 2002)
H
M

(a) (b) (c) (d)


1) 1, 2, 4   3, 5, 6   7, 8, 9
(4-20) : D ㄬ‰×߁…Õ XöH ^Î~¡àO H›e¾†Çò#ß „¬\ì#°
AM
IG

; 2) 1, 7, 8   3, 5, 6   2, 4, 9
H›#°Qù#O_?
KH

3) 1, 3, 4   2, 8, 9   5, 6, 7
R

4) 1, 7, 8   2, 3, 6   4, 5, 9
4.
8.

(S.S.C. 2002)
1) 1, 3, 7   2, 4, 6   5, 8, 9 1) 7, 8, 9   2, 4, 3   1, 5, 6
2) 1, 4, 6   2, 5, 7   3, 8, 9 2) 1, 3, 2   4, 5, 7   6, 8, 9
3) 1, 4, 8   2, 5, 6   3, 7, 9 3) 1, 6, 8   3, 4, 7   2, 5, 9
4) 1, 4, 8   2, 7, 9   3, 5, 6 4) 1, 6, 9   3, 4, 7   2, 5, 8
REASONING ABILITY (NON-VERBAL) ○ ○ ○ ○ ○ ○
250
○ ○ ○ ○ ○ ○ ○ ○ ○ ○ ○ ○ ○ ○ ○ ○ ○ ○ ○
Study Material
9. 11.

1) 7, 8, 9   2, 3, 4   1, 5, 6 1) 1, 5, 8   2, 6, 7   3, 4, 9
2) 1, 2, 3   4, 5, 7   6, 8, 9 2) 1, 5, 7   2, 6, 8   3, 4, 5
3) 1, 6, 8   3, 4, 7   2, 5, 9 3) 1, 5, 8   2, 4, 7   3, 6, 9
4) 3, 7, 8   1, 2, 5   4, 6, 9
4) 1, 5, 8   2, 6, 9   3, 4, 7
10. 12.

Y
M
E
D
9
59
1) 1, 3, 4   2, 5, 6   7, 8, 9
CA
1) 1, 5, 8   3, 4, 7   2, 6, 9

95
2) 2, 5, 9   1, 3, 4   6, 7, 8 2) 6, 7, 8   1, 3, 7   2, 4, 9

50
3) 1, 4, 7   2, 5, 8   3, 6, 9 3) 1, 3, 6   4, 5, 9   2, 7, 8
98
A
4) 1, 2, 9   3, 5, 8   4, 6, 7 4) 1, 3, 6   2, 5, 7   4, 8, 9
:8
1234567890123456789012
12345678901234567890123456789012123456789012345678901234567890121234567890123456789012345678901212345678901234567890123
12345678901234567890123

q=~¡}ì`ÇàH› ["Œ|°°
E

12345678901234567890123456789012123456789012345678901234567890121234567890123456789012345678901212345678901234567890123
1234567890123456789012
1234567890123456789012 12345678901234567890123
12345678901234567890123
12345678901234567890123456789012123456789012345678901234567890121234567890123456789012345678901212345678901234567890123
h

1234567890123456789012
1234567890123456789012 PRACTICE TEST - 1 : 12345678901234567890123
12345678901234567890123456789012123456789012345678901234567890121234567890123456789012345678901212345678901234567890123
12345678901234567890123456789012123456789012345678901234567890121234567890123456789012345678901212345678901234567890123
.P

1234567890123456789012 12345678901234567890123
12345678901234567890123
12345678901234567890123456789012123456789012345678901234567890121234567890123456789012345678901212345678901234567890123
IC

1234567890123456789012 12345678901234567890123
12345678901234567890123456789012123456789012345678901234567890121234567890123456789012345678901212345678901234567890123
AD

1. (4) „¬@O X =°i†Çò „¬@O 4 „¬ijeOz# ~ï O_È° „¬\쁶 3, 5, 9 „¬\쁰 XH› „¬@O…Õ =°~ùH› „¬@O H›ey
AB
O

‹¬=¶# Q®°~¡°ë#° H›ey†Çò<Œß~ò. †Çò<Œß~ò.


2. (3) „¬@O X =°i†Çò „¬@O 3 „¬ijeOz# ~ï O_È° „¬\ì…Õ 4, 6, 8 ï~O_È° "Íö~Þ~¡° „¬\쁰 ‹¬æiÅOKǰ䛽#ß „¬\쁰.
ER
CH

=$`Ç ë O , KÇ ` Ç ° ~¡ 㠋¬ O , =ü_È ° J~¡ Ö = $`Œë  °, XH› 7. (2) 1, 7, 8 „¬\쁰 ª^¥~¡} *ì¼q°f†Ç° „¬\쁰.
YD

n~¡ÉKÇ`Ç°~¡ã‹¬O, XH› ãuƒ’°[O° =ô<Œß~ò. 2, 4, 9 „¬\쁰 ï~O_È° ö~MìYO_¨° YO_OKǰ䛽#ß


,H

„¬\쁰.
3. (1) „¬@O X =°i†Çò „¬@O 1 „¬ijeOz# =$`ÇOë , #H›ãÆ `ÇO,
T

3, 5, 6 „¬\쁰 XöH ö~MìYO_ÈO H›ey# „¬\쁰.


AM

=ü_È° ö~MìYO_¨° ï~O_È° „¬\ì…Õ =ô<Œß~ò.


8. (4) 1, 6, 9 „¬\쁰 ãuƒ’°*쁰 3, 4, 7° KÇ`Ç°~¡°Ä*쁰
H

4. (3) 1, 4, 8 „¬\쁰 XöH…ì=ô#ß („¬i=¶}O ‹¬=¶#O


M

=°i†Çò 2, 5, 8° „¬OKǃ’°l°.


AM

Hê^ΰ) ï~O_È° „¬\쁰 XH›^¥xx =°~ùH›\ ÷ ‹¬æiÅOKÇ°


IG

9. (4) 3, 7, 8° JO`Ç~¡O…Õ n~¡É=$`Œë° H›ey# „¬\쁰.


䛽<Œß~ò =°i†Çò "Œ\÷ JO`Ç~¡O <Œ°Q®° ƒìQꁰQê
KH

1, 2, 5° JO`Ç~¡O…Õ „¬~Œ=†Ç° …è^¥ =OKÇ|_#


qƒ’lO„¬|_#k. =$`Çëö~Y° H›ey# „¬\쁰.
R

2, 5, 6° =ü_È° *ì¼q°f†Ç° „¬\쁰 H›ey†Çò<Œß~ò. 4, 6, 9° JO`Ç~¡O…Õ ö~MìYO_¨° H›ey# „¬\쁰.
"Œ\÷…Õ ï~O_È° „¬\쁰 XöH…ì=ô<Œß~ò. 10. (4) 1, 2, 9° <Œ°Q®° ƒìQꁰQê qƒ’lO„¬|_# „¬\쁰.
3, 7, 9 ° ï~O_È° „¬\ì#° H›ey†Çò<Œß~ò. Jq 3, 5, 8° =ü_È° ƒìQꁰQê qƒ’lO„¬|_# „¬\쁰.
XöH…ì =ôO_È=KÇ°ó …è^¥ XöH…ì =ôO_ÈH›‡é=KÇ°ó. 4, 6, 7° ï~O_È° ƒìQꁰQê qƒ’lO„¬|_# „¬\쁰.
5. (2) 1, 5, 7 „¬\쁰 XöH…ì=ô#ß „¬\쁰 JO`Ç~¡O…Õ 11. (3) 2, 4, 7° ‰×OMìHê~¡„¬ô ƒìQ®O H›ey# „¬\쁰.
H›ey†Çò#ß „¬\쁰. 1, 5, 8° =$`ÇOë =°i†Çò ãuƒ°’ [O H›ey†Çò#ß „¬\쁰.
2, 4, 8 „¬\쁰 XH› „¬@O…Õ "Í~ùH› „¬@O JO`Ç~¡OQê 3, 6, 9 ° ö~MìYO_¨#° =¶ã`Ç"Í° H›ey†Çò#ß
H›e¾†Çò#ß „¬\쁰. „¬\쁰 =°i†Çò |‚¬•ƒ’°A°.
3, 6, 9° XöH…ì =ô#ß „¬\쁰 JO`Ç~¡O…Õ =ôO_ 12. (4) 2, 5, 7° =ü_ȶ J~¡Ö=$`Œë° H›ey# „¬\쁰.
"Œ\÷ =°^Î¼ Q® ㇐O`ÇO À+_£ K͆ǰ|_# „¬\쁰. 1, 3, 6° =$`Œë#° H›ey# „¬\쁰.
6. (4) 1, 2, 7 „¬\쁰 ª^¥~¡} *ì¼q°f†Ç° „¬\쁰. 4, 8, 9° ãuƒ’°*ì#° H›ey# „¬\쁰.

REASONING ABILITY (NON-VERBAL) ○ ○ ○ ○ ○ ○


251
○ ○ ○ ○ ○ ○ ○ ○ ○ ○ ○ ○ ○ ○ ○ ○ ○ ○ ○
Study Material
123456789012345678901234567890121234567890123456789012345678901212
12345678901234567890123456789012123456789012345678901234567890121234567890123456789012345678901212345678901234567890123
1234567890123456789012345678901212345678901234567890123456789012123456789012345678901234567890121234567890123
1234567890123456789012345678901212345678901234567890123456789012123456789012345678901234567890121234
12345678901234567890123456789012123456789012345678901234567890121234567890123456789012345678901212345678901234567890123456789012123
1234567890123456789012345678901212345678901234567890123456789012123456789012345678901234567890
12345678901234567890123456789012123456789012345678901234567890121234567890123456789012345678901212345678901234567890123
1234567890123456789012345678901212345678901234567890123456789012123456789012345678901234567890121234567890123
1234567890123456789012345678901212345678901234567890123456789012123456789012345678901234567890121234
123456789012345678901234567890121234567890123456789012345678901212
12345678901234567890123456789012123456789012345678901234567890121234567890123456789012345678901212345678901234567890123456789012123
1234567890123456789012345678901212345678901234567890123456789012123456789012345678901234567890
12345678901234567890123456789012123456789012345678901234567890121234567890123456789012345678901212345678901234567890123
1234567890123456789012345678901212345678901234567890123456789012123456789012345678901234567890121234567890123
1234567890123456789012345678901212345678901234567890123456789012123456789012345678901234567890121234
12345678901234567890123456789012123456789012345678901234567890121234567890123456789012345678901212345678901234567890123456789012123
1234567890123456789012345678901212345678901234567890123456789012123456789012345678901234567890
123456789012345678901234567890121234567890123456789012345678901212
PRACTICE TEST - 2
12345678901234567890123456789012123456789012345678901234567890121234567890123456789012345678901212345678901234567890123
1234567890123456789012345678901212345678901234567890123456789012123456789012345678901234567890121234567890123
1234567890123456789012345678901212345678901234567890123456789012123456789012345678901234567890121234
12345678901234567890123456789012123456789012345678901234567890121234567890123456789012345678901212345678901234567890123456789012123
1234567890123456789012345678901212345678901234567890123456789012123456789012345678901234567890
123456789012345678901234567890121234567890123456789012345678901212
12345678901234567890123456789012123456789012345678901234567890121234567890123456789012345678901212345678901234567890123
1234567890123456789012345678901212345678901234567890123456789012123456789012345678901234567890121234567890123
1234567890123456789012345678901212345678901234567890123456789012123456789012345678901234567890121234
12345678901234567890123456789012123456789012345678901234567890121234567890123456789012345678901212345678901234567890123456789012123
1234567890123456789012345678901212345678901234567890123456789012123456789012345678901234567890
12345678901234567890123456789012123456789012345678901234567890121234567890123456789012345678901212345678901234567890123
1234567890123456789012345678901212345678901234567890123456789012123456789012345678901234567890121234567890123
1234567890123456789012345678901212345678901234567890123456789012123456789012345678901234567890121234
12345678901234567890123456789012123456789012345678901234567890121234567890123456789012345678901212345678901234567890123456789012123
1234567890123456789012345678901212345678901234567890123456789012123456789012345678901234567890
123456789012345678901234567890121234567890123456789012345678901212
12345678901234567890123456789012123456789012345678901234567890121234567890123456789012345678901212345678901234567890123456789012123
1234567890123456789012345678901212345678901234567890123456789012123456789012345678901234567890
12345678901234567890123456789012123456789012345678901234567890121234567890123456789012345678901212345678901234567890123
1234567890123456789012345678901212345678901234567890123456789012123456789012345678901234567890121234567890123
1234567890123456789012345678901212345678901234567890123456789012123456789012345678901234567890121234

‹¬¶KÇ# (1-20) : D ãH÷Ok ㄬ‰×ß…Õ W=Þ|_# „¬\ì…Õ XöH 3) 1, 4, 7   2, 5, 8   3, 6, 9


^Î~¡àO H›e¾ L#ß "Œ\÷x Q®°iëOKÇO_ ? 4) 1, 7, 9   3, 5, 8   2, 4, 6

5.
1.

1) 1, 2, 5   6, 7, 9   3, 4, 8

Y
1) 1, 2, 3   4, 5, 8   6, 7, 9 2) 1, 2, 6   3, 4, 5   7, 8, 9
2) 1, 5, 6   2, 3, 4   7, 8, 9

M
3) 1, 3, 8   2, 4, 6   5, 7, 9
3) 1, 3, 5   2, 4, 8   6, 7, 9 4) 1, 8, 9   2, 6, 7   3, 4, 5

E
4) 1, 4, 7   2, 5, 8   3, 6, 9

D
9
6.

59
2.
CA
95
50
98
A
:8
1) 1, 3, 5   2, 6, 8   4, 7, 9
E

2) 3, 2, 7   6, 5, 8   1, 4, 9
h
.P
IC

1) 2, 4, 7   1, 8, 9   3, 5, 6 3) 2, 3, 4   5, 6, 8   1, 7, 9
AD

2) 2, 6, 9   1, 5, 7   3, 4, 8 4) 1, 3, 5   2, 6, 9   4, 7, 8
AB
O

3) 2, 6, 7   1, 5, 8   3, 4, 9
4) 2, 7, 8   1, 5, 9   3, 4, 6
ER

7.
CH
YD

3.
,H
T
AM
H
M

1) 1, 5, 6   2, 3, 4   7, 8, 9
AM
IG

2) 1, 4, 7   2, 5, 8   3, 6, 9
KH

1) 3, 4, 9   5, 7, 8   1, 2, 6
3) 1, 2, 3   4, 5, 8   6, 7, 9
2) 1, 5, 6   2, 4, 8   3, 7, 9
R

4) 1, 3, 5   2, 4, 8   6, 7, 9
3) 4, 6, 8   3, 5, 7   1, 2, 9
4) 1, 2, 7   3, 5, 9   4, 6, 8
8.
4.

1) 2, 4, 6   3, 7, 9   1, 5, 8
2) 2, 4, 6   1, 7, 8   3, 5, 9
1) 1, 5, 7   2, 4, 6   3, 9, 8 3) 1, 3, 8   2, 4, 6   5, 7, 9
2) 1, 5, 7   2, 4, 8   3, 6, 9 4) 1, 2, 3   4, 5, 6   7, 8, 9

REASONING ABILITY (NON-VERBAL) ○ ○ ○ ○ ○ ○


252
○ ○ ○ ○ ○ ○ ○ ○ ○ ○ ○ ○ ○ ○ ○ ○ ○ ○ ○
Study Material
9. 10.

1) 1, 2, 6   3, 4, 7   5
2) 1, 3   2, 6   4, 5, 7 1) 1, 2, 3   4, 5, 6   7, 8, 9
3) 1, 2   6, 7, 3   4, 5 2) 1, 3, 5   2, 4, 6   7, 8, 9
4) 1, 3   2, 4, 5   6, 7 3) 1, 5, 9   3, 6, 2   4, 7, 8
4) 1, 9, 7   2, 8, 5   3, 4, 6
1234567890123456789012
12345678901234567890123456789012123456789012345678901234567890121234567890123456789012345678901212345678901234567890123
12345678901234567890123

Y
12345678901234567890123456789012123456789012345678901234567890121234567890123456789012345678901212345678901234567890123
1234567890123456789012 12345678901234567890123

q=~¡}ì`ÇàH› ["Œ|°°
12345678901234567890123456789012123456789012345678901234567890121234567890123456789012345678901212345678901234567890123
1234567890123456789012 12345678901234567890123
12345678901234567890123456789012123456789012345678901234567890121234567890123456789012345678901212345678901234567890123
1234567890123456789012 12345678901234567890123
PRACTICE TEST - 2 :
12345678901234567890123456789012123456789012345678901234567890121234567890123456789012345678901212345678901234567890123
1234567890123456789012
12345678901234567890123456789012123456789012345678901234567890121234567890123456789012345678901212345678901234567890123
1234567890123456789012
12345678901234567890123
12345678901234567890123

M
12345678901234567890123456789012123456789012345678901234567890121234567890123456789012345678901212345678901234567890123
1234567890123456789012 12345678901234567890123
12345678901234567890123456789012123456789012345678901234567890121234567890123456789012345678901212345678901234567890123

1. (2) 1, 5, 6 „¬ \ 쁰 À+_£ KÍ † Ç ° |_ # ƒ ì Qê#°

E
5, 7, 9 „¬\쁰 "Í~ùH› „¬\ì`Ë ƒì‚¬ì¼OQê ‹¬æiÅOKÇ°
H›ey†Çò<Œß~ò. 䛽<Œß~ò.

D
9
59
7, 8, 9 „¬ \ 쁰 JO`Ç ~ ¡ O Qê "Í ~ ùH› „¬ \ ì 6. (1) 2, 6, 8° ãuƒ’°*쁰.
CA
95
j~ŒÂxßO\÷x ‹¬æiŋ¬°ë<Œß~ò. 4, 7, 9° KÇ`Ç°~¡°Ä*쁰.

50
2, 3, 4 „¬\쁰 JO`Ç~¡OQê ö~MìYO_¨`Ë qƒ’lO„¬ 1, 3, 5° =$`ŒëxH÷ ‹¬O|OkOz#k.
98
A
|_¨Û~ò. 7. (1) 1, 5, 6 „¬\쁰 J"Í „¬\ì#° JO`Ç~¡OQê H›ey
:8
2. (2) 1, 5, 7° JO`Ç~¡OQê "Í~ùH› „¬\ì#° H›ey†ÇòO_# =ô<Œß~ò.
E
h

KÇ`Ç°~¡°Ä*ì…ÿá <Œ°Q®° ƒìQꁰQê qƒ’lO„¬|_¨Û~ò. 2, 3, 4 „¬\쁰 <Œ°Q®° ƒìQꁰQê qƒ’lO„¬|_¨Û~ò.


.P
IC

2, 6, 9° JO`Ç~¡OQê "Í~ùH› „¬\ì#° H›ey†ÇòO_# 7, 8, 9 „¬\쁰 "Í~ùH› „¬\ì#° JO`Ç~¡OQê Jxß
AD

ãuƒ’°*ì…ÿá P~¡° ƒìQꁰQê qƒ’lO„¬|_¨Û~ò. j~ŒÂ#° `Œä›½`Ç°#ß@°Á<Œß~ò.


AB
O

3, 4, 8° JO`Ç~¡OQê „¬\ì#° H›ey†Çò#ß =$`Œë…ÿá 8. (3) 1, 3, 8 „¬\쁰 qƒ’lO„¬|_Èx „¬\쁰.


ER
CH

<Œ°Q®° ƒìQꁰQê qƒ’lO„¬|_¨Û~ò. 2, 4, 6 „¬\쁰 JO`Ç~¡OQê „¬\ì#° H›ey#q.


YD

3. (4) 1, 2 7° Mìm „¬\쁰. 5, 7, 9 „¬\쁰 ƒì‚¬ì¼OQê XH›^¥xx =°~ùH›\ ÷


,H

3, 5, 9° JO`Ç~¡OQê "Í~ùH› „¬\쁰 H›ey# „¬\쁰. ‹¬æiÅOKǰ䛽#ß „¬\쁰.


T
AM

4, 6, 8° ƒì‚¬ì¼OQê "Í~ùH› „¬\ì#° ‹¬æiŋ¬°ë#ß 9. (4) 1, 3 „¬\ì…Õ XöH Q®°~¡°ë JO`Ç~¡OQê =ôOKÇ|_#k.
H

„¬\쁰. 2, 4, 5 „¬ \ ì…Õ J"Í „¬ \ 쁰 JO`Ç ~ ¡ O Qê


M
AM

4. (2) 3, 6, 9 „¬\쁰 =°O^΄¬ô JOKÇ° H›e¾†Çò<Œß~ò.


IG

‹¬æiÅOKŒ~ò.
KH

2, 4, 8 JO`Ç~¡O…Õ "Í~ùH› „¬\쁰 =ô<Œß~ò. 6, 7 „¬\쁰 4 ƒìQꁰQê qƒ’lO„¬|_¨Û~ò.


10. (1)
R

1, 5, 7 „¬\ì †³òH›ø JO`Ç~¡O…Õ J^Í~¡H›„¬ô „¬\쁰 1, 2, 3 „¬\쁰 ~ï O_È° ~ö MìYO_¨#° H›ey =ô<Œß~ò.
"Œ\÷ ƒ’°*ì P^¥~¡OQê ƒìQꁰQê qƒ’lO„¬|_¨Û~ò. 4, 5, 6 „¬\쁰 =ü_È° ~ö MìYO_¨#° H›ey =ô<Œß~ò.
5. (3) 1, 3, 8 „¬\쁰 ï~O_È° ƒìQꁰQê qƒ’lO„¬|_¨Û~ò. 7, 8, 9 „¬\쁰 <Œ°Q®° ö~MìYO_¨#° H›ey
2, 4, 6 „¬\ì j~ŒÂ =^Îí HùO`Çƒì Q®O ~ö MìYO_¨`Ë =ô<Œß~ò.
w†Ç°|_#k.

REASONING ABILITY (NON-VERBAL) ○ ○ ○ ○ ○ ○


253
○ ○ ○ ○ ○ ○ ○ ○ ○ ○ ○ ○ ○ ○ ○ ○ ○ ○ ○
Study Material
10 „¬@ =¶ãuH› (Figure Matrix)

=òMì¼O‰§°...
G D qƒìQ®O…Õx ㄬ‰×߁…Õ 2þ2 …è^¥ 3þ3 =¶ã\ ÷H±ž ~¡¶„¬O…Õ Hùxß „¬\쁰 W=Þ|_È`Œ~ò. D „¬\쁰 J_È°Û=~¡°‹¬…ÕQêh
…è^¥ x°=ô =~¡°‹¬…ÕQêh Hùxß ‹¬=¶# ^Î~Œà° H›ey =ôO\ì~ò. "Œ\÷ #°O_ XH› „¬@O W=Þ|_È^ΰ. P „¬\ìxß ^Î~Œà
P^¥~¡OQê =°#O Q®°iëOKŒe.

Y
G D qƒìQ®O…Õx ㄬ‰×߁#° ªkOKÇ°@䛽 „¬@ƒìQê#° J`ǼO`Ç ä›Æ½}âOQê „¬ijeOKŒe. "Œ\÷ ^Î~Œà#° J~¡ÖOK͋¬°ä›½x
W=Þ|_Èx „¬@O Z…ì =ôO@°O^Ë T‚²ìOKŒe.

M
=¶ki ㄬ‰×ß

E
D
9
59
CA
95
1. D ãH÷O^Î W=Þ|_# 3 þ 3 =¶ã\÷H±ž…Õx „¬\ì…Õ =ü_È= J_È°Û=~¡°‹¬…Õx "³ò^Î\ ÷ „¬@O M #° 180°
ㄬ‰§ß~¡HÖ O
› (?) =ô#ßKË@ U „¬@O =‹¬°O
ë ^Ë H›#°Qù#O_. J„¬‹¬=¼ k‰×…Õ u„¬æQê U~¡æ_È°`Ç°Ok. nxx 90°

50
M

98
A
J„¬‹¬=¼k‰×…Õ u„¬æQê U~¡æ_È°`Ç°Ok. J^Í^Î~¡àO
A

M
A "³ò^Î\ ÷ J_È°Û=~¡°‹¬
A

:8
ㄬHê~¡O ï~O_È= J_È°Û=~¡°‹¬…Õx "³ò^Î\ ÷ „¬^ÎO N #°
E

N N ? ï~O_È= J_È°Û=~¡°‹¬
h

180° J„¬‹¬=¼k‰×…Õ u„¬æQê N U~¡æ_È°`Ç°Ok. nxx


.P
IC

M
=ü_È= J_È°Û=~¡°‹¬
M

M
AD

90° J„¬‹¬=¼ k‰×…Õ u„¬æQê Z U~¡æ_È°`Ç°Ok.


ª^Î#: "³ò^Î\ ÷ J_È°Û=~¡°‹¬…Õx "³ò^Î\ ÷ „¬@O A #° 180°
AB
O

Hê=ô# ? =ô#ßKË@ =ôO_¨e.


Z
J„¬‹¬=¼ k‰×…Õ u„¬æQê U~¡æ_È°`Ç°Ok. nxx 90°
A
ER
CH

J„¬‹¬=¼ k‰×…Õ u„¬æQê U~¡æ_È°`Ç°Ok.


A

YD

123456789012345678901234567890121234567890123456789012345678901212
12345678901234567890123456789012123456789012345678901234567890121234567890123456789012345678901212345678901234567890123
1234567890123456789012345678901212345678901234567890123456789012123456789012345678901234567890121234567890123
1234567890123456789012345678901212345678901234567890123456789012123456789012345678901234567890121234
12345678901234567890123456789012123456789012345678901234567890121234567890123456789012345678901212345678901234567890123456789012123
1234567890123456789012345678901212345678901234567890123456789012123456789012345678901234567890
12345678901234567890123456789012123456789012345678901234567890121234567890123456789012345678901212345678901234567890123456789012123
1234567890123456789012345678901212345678901234567890123456789012123456789012345678901234567890
12345678901234567890123456789012123456789012345678901234567890121234567890123456789012345678901212345678901234567890123
1234567890123456789012345678901212345678901234567890123456789012123456789012345678901234567890121234567890123
1234567890123456789012345678901212345678901234567890123456789012123456789012345678901234567890121234
123456789012345678901234567890121234567890123456789012345678901212
12345678901234567890123456789012123456789012345678901234567890121234567890123456789012345678901212345678901234567890123456789012123
1234567890123456789012345678901212345678901234567890123456789012123456789012345678901234567890
12345678901234567890123456789012123456789012345678901234567890121234567890123456789012345678901212345678901234567890123
1234567890123456789012345678901212345678901234567890123456789012123456789012345678901234567890121234567890123
1234567890123456789012345678901212345678901234567890123456789012123456789012345678901234567890121234
,H

123456789012345678901234567890121234567890123456789012345678901212
PRACTICE TEST - 1
12345678901234567890123456789012123456789012345678901234567890121234567890123456789012345678901212345678901234567890123456789012123
1234567890123456789012345678901212345678901234567890123456789012123456789012345678901234567890
12345678901234567890123456789012123456789012345678901234567890121234567890123456789012345678901212345678901234567890123
1234567890123456789012345678901212345678901234567890123456789012123456789012345678901234567890121234567890123
1234567890123456789012345678901212345678901234567890123456789012123456789012345678901234567890121234
12345678901234567890123456789012123456789012345678901234567890121234567890123456789012345678901212345678901234567890123456789012123
1234567890123456789012345678901212345678901234567890123456789012123456789012345678901234567890
12345678901234567890123456789012123456789012345678901234567890121234567890123456789012345678901212345678901234567890123
1234567890123456789012345678901212345678901234567890123456789012123456789012345678901234567890121234567890123
1234567890123456789012345678901212345678901234567890123456789012123456789012345678901234567890121234
123456789012345678901234567890121234567890123456789012345678901212
12345678901234567890123456789012123456789012345678901234567890121234567890123456789012345678901212345678901234567890123456789012123
1234567890123456789012345678901212345678901234567890123456789012123456789012345678901234567890
12345678901234567890123456789012123456789012345678901234567890121234567890123456789012345678901212345678901234567890123
1234567890123456789012345678901212345678901234567890123456789012123456789012345678901234567890121234567890123
1234567890123456789012345678901212345678901234567890123456789012123456789012345678901234567890121234
12345678901234567890123456789012123456789012345678901234567890121234567890123456789012345678901212345678901234567890123456789012123
1234567890123456789012345678901212345678901234567890123456789012123456789012345678901234567890
123456789012345678901234567890121234567890123456789012345678901212
T

12345678901234567890123456789012123456789012345678901234567890121234567890123456789012345678901212345678901234567890123
1234567890123456789012345678901212345678901234567890123456789012123456789012345678901234567890121234567890123
1234567890123456789012345678901212345678901234567890123456789012123456789012345678901234567890121234
12345678901234567890123456789012123456789012345678901234567890121234567890123456789012345678901212345678901234567890123456789012123
1234567890123456789012345678901212345678901234567890123456789012123456789012345678901234567890
AM

‹¬¶KÇ# (1-20) : D ãH÷Ok W=Þ|_# ‹¬=°‹¬¼ zã`ÇO…Õ '?Ñ Q®°~¡°ë ªÖ#O…Õ =KÇ°ó ƒç=°à#° ㄬH›ø<Í W=Þ|_# ‹¬=¶^¥# zã`Œ#°O_
H
M

Q®°iëOKÇ°=ò ?
AM
IG

‹¬=°‹¬¼ zã`ÇO ‹¬=¶^¥# zã`Œ°


KH

3.
R

1.

(a) (b) (c) (d)

(a) (b) (c) (d) 4.

2. (a) (b) (c) (d)

5.
(a) (b) (c) (d)

(a) (b) (c) (d)

REASONING ABILITY (NON-VERBAL) ○ ○ ○ ○ ○ ○


254
○ ○ ○ ○ ○ ○ ○ ○ ○ ○ ○ ○ ○ ○ ○ ○ ○ ○ ○
Study Material
6. 15.

(a) (b) (c) (d)


(a) (b) (c) (d)

16.
7.
(a) (b) (c) (d)

(a) (b) (c) (d)

17.
8.

Y
M
(a) (b) (c) (d)
(a) (b) (c) (d)

E
18.

D
9
9.

59
CA
95
(a) (b) (c) (d)

50
(a) (b) (c) (d)

98
A
:8
19.
10.
E
h
.P
IC
AD

(a) (b) (c) (d) (a) (b) (c) (d)


AB
O

20.
ER

11.
CH
YD
,H

(a) (b) (c) (d) (a) (b) (c) (d)


T
AM
H

21.
M

12.
AM
IG
KH

(a) (b) (c) (d)


(a) (b) (c) (d)
R

22.
13.
(a) (b) (c) (d) (a) (b) (c) (d)

23.
14.
(a) (b) (c) (d)

(a) (b) (c) (d)

REASONING ABILITY (NON-VERBAL) ○ ○ ○ ○ ○ ○


255
○ ○ ○ ○ ○ ○ ○ ○ ○ ○ ○ ○ ○ ○ ○ ○ ○ ○ ○
Study Material
25.
24.

(a) (b) (c) (d) (a) (b) (c) (d)

123456789012345678901
12345678901234567890123456789012123456789012345678901234567890121234567890123456789012345678901212345678901234567890123
1234567890123456789012
12345678901234567890123456789012123456789012345678901234567890121234567890123456789012345678901212345678901234567890123
123456789012345678901 1234567890123456789012
12345678901234567890123456789012123456789012345678901234567890121234567890123456789012345678901212345678901234567890123
123456789012345678901
123456789012345678901
q=~¡}ì`ÇàH› ["Œ|°°
1234567890123456789012
12345678901234567890123456789012123456789012345678901234567890121234567890123456789012345678901212345678901234567890123
PRACTICE TEST - 1 :
1234567890123456789012
12345678901234567890123456789012123456789012345678901234567890121234567890123456789012345678901212345678901234567890123
123456789012345678901
12345678901234567890123456789012123456789012345678901234567890121234567890123456789012345678901212345678901234567890123
123456789012345678901
1234567890123456789012
1234567890123456789012
12345678901234567890123456789012123456789012345678901234567890121234567890123456789012345678901212345678901234567890123
123456789012345678901 1234567890123456789012

1. (b) ㄬu J_È°Û=~¡°‹¬…Õx ㄬH›øㄬH›ø „¬\쁰 XH›^¥xHùH›\ ÷ 9. (c) ㄬu J_È°=Û ~¡°‹¬…Õ "³ò^Î\ ÷ =°i†Çò ~ï O_È= „¬\쁅Õx
‡~¡ÅÞ q…Õ=°OQê =ôO_ „¬@ ƒìQꁰ ̄~¡°Q®°`Ƕ …è^¥ L=°à_ ö ~ MìYO_È O 3= „¬ @ O…Õ KÇ ¶ „¬ | _ # k.
`ÇQ®°¾`Ƕ =ô<Œß~ò. =ü_È= J_È°Û=~¡°‹¬…Õ „¬@O…Õ„¬e J^Íq^OÎ Qê =ü_È= J_È°= Û ~¡°‹¬…Õ "³ò^Î\ ÷ =°i†Çò

Y
KÇ°H›ø° HùOK³O ̄~¡Qêe. ~ï O_È= „¬\쁅Õx L=°à_ „¬@O .

M
10. (c) ㄬu J_È°Û=~¡°‹¬…Õ =ü_È° ~¡HꁰQê À+_£ K͆ǰ|_#
2. (d) ㄬu J_È°Û=~¡°‹¬…Õx „¬@O#° 90° J„¬‹¬=¼ k‰×…Õ
ãuƒ’°*쁰<Œß~ò. =ü_È= J_È°Û=~¡°‹¬…Õ q°Ì‹á ž#k

E
u„¬æQê `Ç~¡°"Œu „¬@O =‹¬°ëOk. J…ìöQ |†Ç°\÷ w`ǁ°
. ㄬu ãuƒ’°*ìxH÷ =ü_È° ~¡Hê Hêˆ×ä<Œß~ò.
"³ò^Î\ ÷ =°i†Çò ï~O_È= „¬\ì#° H›°„¬Qê =ü_È=

D
9
q°Ì‹á ž# ãuƒ’°[O =ü_È° Hêˆ×¤#° H›ey†ÇòO_¨e.

59
„¬@O H›ey =ôO@°Ok.
CA
J#Qê =°i†Çò z#ß =$`ÇëO XH›^¥x `Ç~ŒÞ`Ç

95
3. (c) ㄬu J_È°Û=~¡°‹¬…Õx =ü_È= „¬@O, "³ò^Î\ ÷ ï~O_È°
=°~ùH› „¬@O…Õ À+_£ K͆ǰ|_#k. J#Qê Hê=‹²#

50
„¬\ì †³òH›ø L=°à_ „¬@O.
=$`ÇëO l.
98
A
ZO^ΰH›#Qê , „¬\ì…Õ l =¶ã`Ç"Í° L=°à_Qê 11. (d) ㄬu J_È°Û=~¡°‹¬…Õx „¬\ì…Õ =ü_È° ~¡Hê ƒì‚¬ì¼
:8
=ô#ßk. „¬\쁰, =ü_È° ~¡Hê JO`Ç~¡ „¬\쁰, =ü_È° ~¡Hê
E
h

4. (c) "³ò^Î\ ÷ J_È°=Û ~¡°‹¬…Õx "³ò^Î\ ÷ „¬@O#° J„¬‹=¬ ¼k‰×…Õ À+_OQ·° =ô<Œß~ò. =ü_È= J_È°Û=~¡°‹¬…Õ q°Ì‹á ž#
.P
IC

90° ° ãu„²æ ̄á# XH› Q®°~¡°ë ㄬ`ͼH›OQê =ôOKÇ|_#k „¬@O À+_OQ· …èx ãuƒ’°[O =°i†Çò JO`Ç~¡ „¬@O.
AD

=°i†Çò ãuƒ’°[O …Õ„¬e À+_OQ· ƒìQ®O 䛽_ #°O_ 12. (d) ㄬu J_È°Û=~¡°‹¬…Õx „¬@O, ^¥x =òO^ΰ „¬@O 90°
AB
O

‹¬ = ¼k‰× … Õ u~¡ ° Q® Q ê U~¡ æ _È ° `Ç ° Ok =°i†Ç ò


Z_È=°ä›½ =¶~¡ó|_#k. J^Í q^ÎOQê 䛽
ER
CH

ㄬu=~¡°‹¬…Õ =ü_È° "Í~ö Þ~¡° ~¡Hê _*<áÿ £ ° =ô<Œß~ò.


"Œ\÷ ㄬHê~¡O Q®=°xOKÇQê ‹¬=¶^¥#O (d).
YD

=iëO„¬K͆ǰQê =KÇ°ó#°.
13. (c) ㄬu J_È°Û=~¡°‹¬…Õ XöH ~¡H›O „¬@O =ô#ßk. Hêx
,H

5. (a) ㄬu J_È°=Û ~¡°‹¬…Õ#¶ "³ò^Î\ ÷ „¬@O…Õx ~ï O_È= ƒì}O „¬i=¶}O ̄~¡°Q®°`Ƕ "³ò^Î\ ÷ ï~O_È° „¬\쁅Õx
T
AM

Q®°~¡°ë =¼uö~H›k‰×…ÕxH÷ =°i†Çò ï~O_È= „¬@O…Õx ƒìQêxßO\÷h H›ey †Çò#ßk.


H

ï~O_È° ƒì}O Q®°~¡°ë¶ =¼uö~H›k‰×…ÕxH÷ =¶~Œ~ò. 14. (d) ㄬu J_È°Û=~¡°‹¬…Õ J^ÎíO…Õx ㄬuaOƒì †³òH›ø
M
AM

J^Íq^ÎOQê =ü_È= J_È°Û=~¡°‹¬…Õx ï~O_È= „¬@O ~¡OQ®°° =¶~¡ó|_¨Û~ò.


IG

↑↓ J=ô`Ç°Ok. 15. (b) "³ò^Î\ ÷ J_È°Û=~¡°‹¬…Õ =$`ÇëO, n~¡É=$`ÇëO J~ò#k


KH

6. (d) ㄬu J_È°Û=~¡°‹¬…Õx "³ò^Î\ ÷ „¬@O, ï~O_È= „¬\쁰 =°i†Çò <Œ°Q®° „¬ˆ×ä #°O_ 3 f‹²"͆ǰ|_ ̄á#
R

`³°„¬ô ~¡OQ®° =°i†Çò #°„¬ô~¡OQ®°° XH›^¥xHùH›\ ÷ XH› H÷Æu[ ‹¬=¶O`Ç~¡ ö~Y w†Ç°|_#k. J^Íq^ÎOQê
=¶~¡°óHË|_# „¬\쁰. ï~O_È= J_È°Û=~¡°‹¬…Õx "³ò^Î\ ÷ „¬^ÎO .
7. (a) ㄬu J_È°Û=~¡°‹¬…Õx "³ò^Î\ ÷ „¬@O =°i†Çò =ü_È= 16. (c) ㄬu J_È°Û=~¡°‹¬…Õ XöH JH›Æ~¡O qq^Î ~¡¶‡…Õ
„¬@O° XH›^¥xH÷ =°~ùH›\ ÷ J^ÎíO…Õx ㄬuaOƒì°. ~Œ†Ç°|_#k. ï~O_È= J_È°Û=~¡°‹¬…Õ q°Ì‹á ž# „¬@O
8. (d) ㄬ u J_È ° Û = ~¡ ° ‹¬ … Õx "³ ò ^Î \ ÷ „¬ @ O#° 135° .
J„¬‹¬=¼k‰×…Õ u„¬æQê ï~O_È= „¬@O U~¡æ_È°`Ç°Ok. 17. (d) ㄬu J_È°=Û ~¡°‹¬…Õ XöH~¡H„› ô¬ „¬@O qq^Î „¬i=¶}쁅Õ
=°i†Çò ï~O_È= „¬@O#° 135° J„¬‹¬=¼k‰×…Õ u„¬æQê =ô#߄¬æ\÷H© XH›\ ÷ MìmQê, XH›\ ÷ x°=ô Q®ˆ×¤`Ë
=ü_È= „¬@O U~¡æ_È°`Ç°Ok. Hê=ô# #° 135° =°i†Çò =°~ùH›\ ÷ ~ï O_È° "³„ á ô¬ …ì w`ǁ`Ë À+_£ K͆°Ç |_
=ô#ßk. J^Íq^ÎOQê =ü_È= J_È°Û=~¡°‹¬…Õ q°Ì‹á ž#k
1234
J„¬‹¬=¼k‰×…Õ u„¬æQê U~¡æ_È°`Ç°Ok. 1234
1234.
1234

REASONING ABILITY (NON-VERBAL) ○ ○ ○ ○ ○ ○


256
○ ○ ○ ○ ○ ○ ○ ○ ○ ○ ○ ○ ○ ○ ○ ○ ○ ○ ○
Study Material
18. (d) ㄬu J_È°Û=~¡°‹¬…Õ ï~O_È= „¬@O #°O_ "³ò^Î\ ÷ 22. (b) Jxß x°=ô =~¡°‹¬…Õ 䛽_"³á„¬ô# Q® =~¡°‹¬#°
„¬\ìxß `ùyOKÇQê =ü_È= „¬@O U~¡æ_È°`Ç°Ok. `ùyOKǰ䛽O@¶ ~Œ"Œe. =ü_È= x°=ô =~¡°‹¬…Õ
Hê=ô# =ü_È= J_È°Û=~¡°‹¬…Õ z=i „¬@O þ =‹¬°ëOk. 䛽_"³á„¬ô# Q® =$`Œë#° `ùyOKǰ䛽O@¶ ‡éQê
19. (b) ï~O_È= J_È°Û=~¡°‹¬…Õ ̄á J~¡œ=$`ÇëO 180° ‹¬=¼k‰×…Õ OOO ↓
ãu„²æ ãH÷Ok J~¡Ö=$`ÇëŌá =ôOKÇ|_#k. J^Íq^ÎOQê OO ↓
O
→ J=ô`Ç°Ok.
23. (c) ㄬu J_È°Û=~¡°‹¬…Õx „¬\쁰 "³ò^Î@ 180° ‹¬=¼k‰×…Õ
20. (a) =$`ÇëO, ãuƒ’°[O =°i†Çò ‹¬=¶O`Ç~¡ KÇ`Ç°~¡°Ä*쁰 `Ç~¡°"Œ`Ç J„¬‹¬=¼k‰×…Õ u„¬æ|_#q.
J_ÈÛOQê, x°=ôQê =°i†Çò "Œ°Qê (̄á 䛽_ #°O_ M #° 180° ‹¬=¼k‰×…Õ u„¬æQê =‹¬°ëOk.
M
ãH÷Ok Z_È=°"³„á ô¬ 䛽) À+_£ K͆°Ç |_¨Û~ò. Hê=ô# q°Ì‹á ž# 24. (d) ㄬu J_È°Û =~¡°‹¬…Õ "³ò^Î\ ÷ =°i†Çò ï~O_È= „¬\ì#°
1234
1234
„¬@O1234. H›e„²`Í =ü_È= „¬@O =‹¬°ëOk.

Y
21. (b) ㄬu J_È°Û=~¡°‹¬…Õx „¬\ìxß 90° ‹¬=¼k‰×…Õ ãu„¬æQê 25. (a) "³ò^Î\ ÷ J_È°=Û ~¡°‹¬…Õ ãuƒ°’ *쁰 JO`Ç~O¡ …Õ 2 =°i†Çò
3 ª~¡°Á =KŒó~ò. Hê=ô# ~ï O_È= J_È°= Û ~¡°‹¬…Õ "³ò^Î\ ÷

M
`Ç~¡°"Œu „¬@O =‹¬°ëOk.
„¬@O T.

E
123456789012345678901234567890121234567890123456789012345678901212
12345678901234567890123456789012123456789012345678901234567890121234567890123456789012345678901212345678901234567890123456789012123
1234567890123456789012345678901212345678901234567890123456789012123456789012345678901234567890
12345678901234567890123456789012123456789012345678901234567890121234567890123456789012345678901212345678901234567890123
1234567890123456789012345678901212345678901234567890123456789012123456789012345678901234567890121234567890123
1234567890123456789012345678901212345678901234567890123456789012123456789012345678901234567890121234
123456789012345678901234567890121234567890123456789012345678901212
12345678901234567890123456789012123456789012345678901234567890121234567890123456789012345678901212345678901234567890123456789012123
1234567890123456789012345678901212345678901234567890123456789012123456789012345678901234567890
12345678901234567890123456789012123456789012345678901234567890121234567890123456789012345678901212345678901234567890123
1234567890123456789012345678901212345678901234567890123456789012123456789012345678901234567890121234567890123
1234567890123456789012345678901212345678901234567890123456789012123456789012345678901234567890121234
123456789012345678901234567890121234567890123456789012345678901212
12345678901234567890123456789012123456789012345678901234567890121234567890123456789012345678901212345678901234567890123456789012123
1234567890123456789012345678901212345678901234567890123456789012123456789012345678901234567890
12345678901234567890123456789012123456789012345678901234567890121234567890123456789012345678901212345678901234567890123
1234567890123456789012345678901212345678901234567890123456789012123456789012345678901234567890121234567890123
1234567890123456789012345678901212345678901234567890123456789012123456789012345678901234567890121234
PRACTICE TEST - 2
12345678901234567890123456789012123456789012345678901234567890121234567890123456789012345678901212345678901234567890123456789012123
1234567890123456789012345678901212345678901234567890123456789012123456789012345678901234567890

D
9
123456789012345678901234567890121234567890123456789012345678901212
12345678901234567890123456789012123456789012345678901234567890121234567890123456789012345678901212345678901234567890123
1234567890123456789012345678901212345678901234567890123456789012123456789012345678901234567890121234567890123
1234567890123456789012345678901212345678901234567890123456789012123456789012345678901234567890121234
12345678901234567890123456789012123456789012345678901234567890121234567890123456789012345678901212345678901234567890123456789012123
1234567890123456789012345678901212345678901234567890123456789012123456789012345678901234567890
12345678901234567890123456789012123456789012345678901234567890121234567890123456789012345678901212345678901234567890123
1234567890123456789012345678901212345678901234567890123456789012123456789012345678901234567890121234567890123
1234567890123456789012345678901212345678901234567890123456789012123456789012345678901234567890121234
12345678901234567890123456789012123456789012345678901234567890121234567890123456789012345678901212345678901234567890123456789012123
1234567890123456789012345678901212345678901234567890123456789012123456789012345678901234567890
123456789012345678901234567890121234567890123456789012345678901212

59
12345678901234567890123456789012123456789012345678901234567890121234567890123456789012345678901212345678901234567890123
1234567890123456789012345678901212345678901234567890123456789012123456789012345678901234567890121234567890123
1234567890123456789012345678901212345678901234567890123456789012123456789012345678901234567890121234
12345678901234567890123456789012123456789012345678901234567890121234567890123456789012345678901212345678901234567890123456789012123
1234567890123456789012345678901212345678901234567890123456789012123456789012345678901234567890
12345678901234567890123456789012123456789012345678901234567890121234567890123456789012345678901212345678901234567890123456789012123
1234567890123456789012345678901212345678901234567890123456789012123456789012345678901234567890
12345678901234567890123456789012123456789012345678901234567890121234567890123456789012345678901212345678901234567890123
1234567890123456789012345678901212345678901234567890123456789012123456789012345678901234567890121234567890123
1234567890123456789012345678901212345678901234567890123456789012123456789012345678901234567890121234

CA
D ãH÷Ok W=Þ|_# ‹¬=°‹¬¼ zã`ÇO…Õ '?Ñ Q®°~¡°ë ªÖ#O…Õ =KÇ°ó PHê~Œxß ‹¬=¶^¥# „¬\ì#°O_ Q®°iëOKÇO_ ?

95
‹¬¶KÇ# (1-20) :

50
‹¬=°‹¬¼ zã`ÇO ‹¬=¶^¥# zã`Œ°
98
A
:8
1.
E
h
.P
IC
AD
AB
O

(a) (b) (c) (d)


ER
CH
YD

2.
,H
T
AM
H
M
AM

(a) (b) (c) (d)


IG
KH
R

3.

(a) (b) (c) (d)

4.

(a) (b) (c) (d)

REASONING ABILITY (NON-VERBAL) ○ ○ ○ ○ ○ ○


257
○ ○ ○ ○ ○ ○ ○ ○ ○ ○ ○ ○ ○ ○ ○ ○ ○ ○ ○
Study Material
5.

(a) (b) (c) (d)

6.

(a) (b) (c) (d)

Y
M
E
7.

D
9
59
CA
95
(a) (b) (c) (d)

50
98
A
:8
8.
E
h
.P
IC
AD
AB
O

(a) (b) (c) (d)


ER
CH

9.
YD
,H
T
AM
H

(a) (b) (c) (d)


M
AM
IG

10.
KH
R

(a) (b) (c) (d)

11.

(a) (b) (c) (d)

REASONING ABILITY (NON-VERBAL) ○ ○ ○ ○ ○ ○


258
○ ○ ○ ○ ○ ○ ○ ○ ○ ○ ○ ○ ○ ○ ○ ○ ○ ○ ○
Study Material
12.

(a) (b) (c) (d)

13.

Y
(a) (b) (c) (d)

M
E
14.

D
9
59
CA
95
(a) (b) (c) (d)

50
98
A
:8
15.
E
h
.P
IC
AD

(a) (b) (c) (d)


AB
O

16.
ER
CH
YD
,H
T
AM

(a) (b) (c) (d)


H
M
AM
IG

17.
KH
R

(a) (b) (c) (d)

18.

(a) (b) (c) (d)

REASONING ABILITY (NON-VERBAL) ○ ○ ○ ○ ○ ○


259
○ ○ ○ ○ ○ ○ ○ ○ ○ ○ ○ ○ ○ ○ ○ ○ ○ ○ ○
Study Material
1234567890123456789012
12345678901234567890123456789012123456789012345678901234567890121234567890123456789012345678901212345678901234567890123
12345678901234567890123
12345678901234567890123456789012123456789012345678901234567890121234567890123456789012345678901212345678901234567890123
1234567890123456789012 12345678901234567890123
12345678901234567890123456789012123456789012345678901234567890121234567890123456789012345678901212345678901234567890123
1234567890123456789012
1234567890123456789012
q=~¡}ì`ÇàH› ["Œ|°°
12345678901234567890123
12345678901234567890123456789012123456789012345678901234567890121234567890123456789012345678901212345678901234567890123
PRACTICE TEST - 2 :
12345678901234567890123
12345678901234567890123456789012123456789012345678901234567890121234567890123456789012345678901212345678901234567890123
1234567890123456789012
12345678901234567890123456789012123456789012345678901234567890121234567890123456789012345678901212345678901234567890123
1234567890123456789012
12345678901234567890123
12345678901234567890123
12345678901234567890123456789012123456789012345678901234567890121234567890123456789012345678901212345678901234567890123
1234567890123456789012 12345678901234567890123

1. (b) ㄬu J_È°Û =~¡°‹¬…Õx „¬\ìxß 90° ‹¬=¼k‰×…Õ u„²æ#@°Á J^Íq^ÎOQê ï~O_È= J_È°Û=~¡°‹¬Qê ö~MìYO_ÈO Z^ΰ\÷
WKŒó~¡°. Hê=ô# #° ‹¬=¼k‰×…Õ u„¬æQê U~¡æ_# k‰×䛽 =¶i ï~\÷“Ōá =°i†Çò =$`ÇëO ̄áH÷, 䛽_"³á„¬ô䛽
„¬@O . ï~O_È°Qê =¶i#„¬ô_È° U~¡æ_È° „¬@O A.
2. (c) ㄬu J_È°Û=~¡°‹¬…Õ XöH~¡H›„¬ô 2, 3, 4 KÇ`Ç°~¡ãª° 10. (d) , , „¬\쁰 "³ò^Î\ ÷ J_È°Û=~¡°‹¬…Õ z#ßqQê#¶,
=ô<Œß~ò. Hê=ô# ï~O_È= J_È°Û =~¡°‹¬…Õ q°Ì‹á ž# ï~O_È= J_È°Û=~¡°‹¬…Õ =°^Î¼=° „¬i=¶}O…Õ#¶,
„¬@O w`ǁ`Ë À+_£ K͆ǰ|_# =ü_È° KÇ`Ç°~¡ãª°. =ü_È = J_È ° Û = ~¡ ° ‹¬ … Õ Ì„^Î í „¬ i =¶}O…Õ#¶
3. (d) 3 ~¡Hê =òM쁰, 3 ~¡Hê KÍ`Ç°°, 3 ~¡Hê Hêˆ×ä =KŒó~ò.
ㄬu =~¡°‹¬…Õ =ô<Œß~ò. 3= J_È°Û =~¡°‹¬…Õ q°Ì‹á ž# 11. (b) J_È°Û =~¡°‹¬…Õx „¬\ì JO`Ç~¡O…Õ , ,

Y
„¬@O ãuƒ’°*ìHê~¡ =òYO, Q®°~¡°ë KÍ`Ç°° =°i†Çò Q®°~¡°ë° U^Ë XH› ªÖ#O…Õ =KŒó~ò. Jxß J_È°Û

M
Q®°~¡°ë Hêˆ×ä. =~¡°‹¬…Õ ƒì‚¬ì¼ „¬\쁰 =¶ã`ÇO J"Í =KŒó~ò.

E
4. (a) "³ò^Î\ ÷ J_È°Û=~¡°‹¬…Õx „¬\ì…Õ ã„¬u „¬@O…Õ#¶ 12. (b) ㄬ u J_È ° Û = ~¡ ° ‹¬ … Õ "³ ò ^Î \ ÷ „¬ @ O…Õx <Œ°Q® °
ï~O_È° =$`Œë° XH›^¥xH÷ =°~ùH›\ ÷ JO`Ç~¡O…Õ =ôO_ ƒìQê…Õ XH›^¥xx À+_£ K͆ǰQê ï~O_È= „¬@O

D
9
59
XH› =$`Ç ë O =°~ùH› =$`Ç ë O †³ ò H› ø Ì„á ƒ  ì Qêxß =°i†Çò ï~O_È° ƒìQê#° À+_£ K͆ǰQê =ü_È= „¬@O
CA
95
`Œä›½`Ç°#ß@°Á =ôOk. ï~O_È= J_È°Û=~¡°‹¬…Õ =$`Œë° U~¡æ_¨Û~ò. (À+_OQ· k‰×#° 䛀_¨ Q®=°xOKŒe).

50
JO`Ç~¡OQê ‹¬æiÅOKÇ° HË䛽O_¨ =ô<Œß~ò. =ü_È= 13. (c) ㄬu J_È°Û=~¡°‹¬…Õ "³ò^Î\ ÷ „¬\ìxß 90° ‹¬=¼k‰×…Õ
98
A
J_È°Û=~¡°‹¬…Õ =$`Œë° XH›^¥xx =°~ùH›\ ÷ ãH÷Ok"³„á ô¬ u„¬æQê ï~O_È= „¬@O =°i†Çò ï~O_È= „¬\ìxß 90°
:8
‹¬æiÅOKǰ䛽<Œß~ò. ‹¬=¼k‰×…Õ u„¬æQê =ü_È= „¬@O U~¡æ_È`Œ~ò.
E
h
.P

5. (c) , =°i†Çò Q®°~¡°ë° qq^Î (z#ß, =°^Î¼=°, 14. (a) ㄬu J_È°Û=~¡°‹¬…Õ "³ò^Î\ ÷ „¬\ìxß 90° ‹¬=¼k‰×…Õ
IC
AD

̄^Îí) „¬i=¶}ì…Õ =ô<Œß~ò. q°Ì‹á ž# Q®°~¡°ë u„¬æQê ï~O_È= „¬@O =°i†Çò ï~O_È= „¬\ìxß 90°
AB
O

†³òH›ø =°^Î¼=° „¬i=¶}O. ‹¬=¼k‰×…Õ u„¬æQê =ü_È= „¬@O U~¡æ_È`Œ~ò.


ER
CH

6. (b) ㄬ u J_È ° Û = ~¡ ° ‹¬ … Õ "³ ò ^Î \ ÷ „¬ @ O…Õx <Œ°Q® ° 15. (c) ̄á J_È°Û=~¡°‹¬…Õx „¬\쁅ÕQ® x°=ô KÇ°H›ø
YD

ƒìQê…Õ XH› ƒìQêxß À+_£ K͆ǰQê ï~O_È= „¬@O w`ǁ#° `³Á~¡OQ®°`Ë#¶, w`ǁ#° KÇ°H›ø`Ë#¶,
,H

=°i†Çò ï~O_È° ƒìQê#° À+_£ K͆ǰQê =ü_È= „¬@O #°„¬ô~¡OQ®°#° w`ǁ`Ë#¶, `³°„¬ô~¡OQ®°#° #°„¬ô
T
AM

U~¡æ_¨Û~ò. ~¡OQ®°`Ë#¶ =¶~¡óQê ãH÷Ok J_È°Û=~¡°‹¬…Õx „¬\쁰


H

7. (d) x°=ô =~¡°‹¬hß , =°i†Çò „¬\ì<Í U~¡æ_È`Œ~ò.


M
AM

H›ey XöH ~¡H›"³°Ø# À+_OQ· H›ey †Çò<Œß~ò. =ü_È= 16. (a) ㄬu J_È°Û=~¡°‹¬…Õ "³ò^Î\ ÷ =°i†Çò ï~O_È= „¬\ì
IG
KH

x°=ô =~¡°‹¬…Õx z=i„¬@O . H›~òH› =ü_È= „¬@O.


17. (a) ㄬu J_È°Û=~¡°‹¬…Õ "³ò^Î\ ÷ „¬@O#° 90° ‹¬=¼k‰×…Õ
R

8. (b) "³ò^Î\ ÷ x°=ô =~¡°‹¬…Õx KÇ`Ç°~¡ãª°, { Q®°~¡°ë°


XH›\ ÷ ̄OKÇ|_ 4 #°O_ 5䛽 =KŒó~ò. J^Íq^ÎOQê u„¬æQê ï~O_È= „¬@O U~¡æ_#k, =°i†Çò ï~O_È=
ï~O_È= x°=ô =~¡°‹¬…Õ 䛀_¨ [iy# 5 ãuƒ’°*쁰, „¬@O#° 180° u„¬æQê =ü_È= „¬@O U~¡æ_#k.
5{ Q®°~¡°ë° =KÇ°ó#°. Hê=ô# #° 180° u„¬æQê U~¡æ_È°`Ç°Ok.
9. (a) "³ò^Î\ ÷ J_È°Û=~¡‹¬…Õ ö~MìYO_ÈO (z#ßw`Ç) Z^ΰ\÷ 18. (c) "³ ò ^Î \ ÷ J_È ° Û = ~¡ ° ‹¬ … Õx ㄬ u „¬ @ O, =ü_È =
k‰×䛽 =¶i ï~\÷“Ōá#k =°i†Çò ãuƒ’°[O ̄áH÷, =~¡°‹¬…Õx ㄬu „¬@O XH›^¥xH÷ =°~ùH›\ ÷ h\÷
䛽_"³á„¬ô䛽 ï~O_È°Qê =¶i#k. ㄬuaOƒì°.

REASONING ABILITY (NON-VERBAL) ○ ○ ○ ○ ○ ○


260
○ ○ ○ ○ ○ ○ ○ ○ ○ ○ ○ ○ ○ ○ ○ ○ ○ ○ ○
Study Material
n~¡„É °¬¦ #O, ‹¬=°„¦°¬ #O =°i†Çò ‡zH›°
11 (Cuboid, Cube & Dice)

=òMì¼O‰§°...
G ‡Ú_È=ô, "³_ȁ°æ =°i†Çò Z`ǰ끰 ‹¬=¶#OQê Q® XH› ãuq°f†Ç° PHê~Œxß …è^¥ =‹¬°ë=ô#° '‹¬=°„¦¬°#OÑ JO\ì~¡°.
G ‡Ú_È=ô, "³_ȁ°æ =°i†Çò Z`ǰ끰 J‹¬=¶#OQê Q® ãuq°f†Ç° PHê~Œxß …è^¥ =‹¬°ë=ô#° 'n~¡É„¦¬°#OÑ JO\ì~¡°.
G XH› n~¡É„¦¬°#O …è^¥ ‹¬=°„¦¬°#O (i)6 L„¬i`Džì#°,(ii) 8 j~ŒÂ#°, (iii) 12 JOKÇ°#° H›ey =ôO\ì~ò.

Y
M
E
D
9
59
CA
95
(a) (b) (c)

50
„¬@O (a) ‹¬=°„¦¬°#O †³òH›ø 6 L„¬i`Džì#° KǶ„²‹¬°ëOk.
98
A
„¬@O (b) ‹¬=°„¦¬°#O †³òH›ø 8 j~ŒÂ#° KǶ„²‹¬°ëOk.
:8
„¬@O (c) ‹¬=°„¦¬°#O †³òH›ø 12 JOKÇ°#° KǶ„²‹¬°ëOk.
E
h
.P

G XH› ‡zH› „¦¬°<ŒHê~¡O…Õ =ôO@°Ok. ‡zH› †³òH›ø =òMì g°^Î 1 #°O_ 6 =~¡ä›½ JOïH° ã"Œ†Ç°|_ =ôO\ì~ò. XHùøH›ø
IC
AD

=òYO g°^Î XH› JOïH =¶ã`Ç"Í° ã"Œ†Ç°|_ LO@°Ok.


G XHùøH›øªi „¦¬°#=ò =òMìä›½ ~¡OQ®°° "͆ǰ|_ 1 #°O_ 6 #Á KÇ°H›ø° =ôO\ì~ò. Hùxߪ~¡°Á "Íö~Þ~¡° Q®°~¡°ë#° 䛀_¨
AB
O

„¦¬°#O †³òH›ø =òMìÌ„á =òãkªë~¡°. Hùxߪ~¡°Á POQ®Á JH›Æ~Œ#° 䛀_¨ „¦¬°#O =òMìÌ„á ã"Œªë~¡°.
ER
CH

G Hùxß ‹¬O^Î~ŒÄ…Õ 6 ~¡Hê ~¡OQ®°° „¦¬°#O †³òH›ø 6 =òMìÌ„á "ͪë~¡°.


YD

qq^Î ~¡Hê ‡zH›° :


,H
T
AM

C F Yellow
4 # Green
H

®
1 φ
M

$ B E
e

6
Blu

Blu

¬
5 2 ¥ A D Red Pink
AM

*
IG

¯ `
3
KH

(i) (ii) (iii) (iv) (v)


R

G „¦¬°#O …è^¥ ‡zH›…Õx XH› ƒ’°*ìxH÷ ZÁ„¬C_ȶ 4 ƒ’°*쁰 ㄬH›øQê =ôO\ì~ò.


ㄬH›ø# W=Þ|_# ‡zH›…Õ ƒ’°[O …è^¥ =òYO 1H÷ 6 `DŽ¬æ q°ye#=hß P‹¬#ß …è^¥ ㄬH›ø =òMì…è.
1H÷ Z^ΰ~¡° ƒ’°[O 6.
2H÷ Z^ΰ~¡° ƒ’°[O 5
3H÷ Z^ΰ~¡° ƒ’°[O 4.
G ï~O_È° Z^ΰï~^ΰ~¡° =òM쁰 ㄬH›øㄬH›øQê =ôO_ȅè=ô.
‡zH›ä›½ ‹¬O|OkOz# x†Ç°=¶° :
; x†Ç°=°O 1 : XH› ‡zH›#° ï~O_È° „¬~Œ¼†Ç°=ò° ^ùiÁOz# `Ç~ŒÞ`Ç aOz# „¬\ì…Õ L=°à_ ‹¬OY¼ä›½ ㄬH›ø# Q®
‹¬OY¼#° ~Œ†Ç°Qê XH› ‹¬OY¼ q°ye‡é=ô#°. J\÷“ q°ye‡é~ò# ‹¬OY¼ L=°à_ ‹¬OY¼ä›½ Z^ΰ~¡°Qê =ôO_È°#°.

REASONING ABILITY (NON-VERBAL) ○ ○ ○ ○ ○ ○


261
○ ○ ○ ○ ○ ○ ○ ○ ○ ○ ○ ○ ○ ○ ○ ○ ○ ○ ○
Study Material
L^¥‚¬ì~¡} :

W=Þ|_# „¬\ì…Õ 1 L=°à_ ‹¬OY¼.


1H÷ ㄬH›ø# Q® ‹¬OY¼° 2, 3, 4, 6.
q°ye# ‹¬OY¼ 5
Hê=ô# 1H÷ Z^ΰ~¡°Qê =ôO_È° ‹¬OY¼ 5.
; x†Ç°=°O 2 : XöH ‡zH›#° ï~O_È°ª~¡°Á ^ùiÁOKÇQê aOz# „¬\ì…Õ L=°à_ ‹¬OY¼#° f‹²"͆ǰQê q°ye# ‹¬OY¼°

XH›^¥xHùH›\ ÷ Z^ΰ~¡°Qê =ôO\ì~ò.

Y
L^¥‚¬ì~¡} :

M
W=Þ|_# „¬\ì…Õ 2 =°i†Çò 4° L=°à_ ‹¬OY¼°. q°ye# ‹¬OY¼° 6 =°i†Çò 3.
∴ 6 =°i†Çò 3° Z^ΰï~^ΰ~¡° ‹¬OY¼°.

E
; x†Ç°=°O 3 : ‡zH›° †³òH›ø ï~O_È° "Íö~Þ~¡° ªÖ<Œ…Õ L=°à_Qê =ôO_Í =òY=ò †³òH›ø ªÖ#=ò XH›>è J~ò`Í, q°Q®`Œ

D
9
59
=òMìä›½ Ja=òYOQê =ô#ß =òM쁰 J^Í ªÖ#O…Õ =ôO\ì~ò.
CA
95
50
L^¥‚¬ì~¡} :

98
A
WH›ø_È ï~O_È° ªÖ<Œ…Õ#° 3 JOïH L=°à_Qê =ô#ßk. Hê=ô# 6䛽 Ja=òYOQê 5, 2䛽 Ja=òYOQê 4 =ôO\ì~ò.
:8
E

; x†Ç°=°O 4 : ï~O_È° Z^ΰï~^ΰ~¡° =òM쁰 ㄬH›øㄬH›øQê =ôO_ȅè=ô.


h
.P
IC
AD

L^¥‚¬ì~¡} :
AB
O
ER
CH

̄á# W=Þ|_# „¬\ì…Õ L=°à_ ‹¬OY¼ 3.


YD

x†Ç°=°O 1 #°O_, 3䛽 Z^ΰ~¡° ‹¬OY¼ 6.


„¬\ì #°O_, 3䛽 ㄬH›ø#Q® ‹¬OY¼° 1, 2, 4, 5.
,H
T

q°ye#k 6. J#Qê 6 J#°#k 3䛽 Z^ΰ~¡°Qê<Í =ôO@°Ok `DŽ¬æ ㄬH›ø# =¶ã`ÇO =ôO_È^ΰ.
AM
H

x†Ç°=°O 5 : ‡zH› †³òH›ø ï~O_È° "Íö~Þ~¡° ªÖ<Œ…Õ, L=°à_Qê =ôO_Í =òY=ò †³òH›ø ªÖ#=ò XH›>è HêH›‡é`Í, P
M

;
AM

L=°à_Qê =ôO_Í =òMìxH÷ Ja=òYOQê =ôO_Í =òY=ò zã`Ç=ò…Õ KǶ„²# U JOïH Hê^ΰ. JO`ÍHêH› q°Q®`Œ =òMìä›½
IG

Z^ΰï~^ΰ~¡°Qê =ôO_Íq 䛀_¨ XH›>è Hê=ô.


KH
R

1 4
L^¥‚¬ì~¡} : 3 1
5 2

̄á# W=Þ|_# „¬\ì…Õ L=°à_Qê =ô#ß ‹¬OY¼ 1 †³òH›ø ªÖ#O. L=°à_Qê …è^ΰ, 6 †³òH›ø ªÖ#O W=Þ|_ȅè^ΰ.
Hê=ô# 1 JOïH䛽 Z^ΰ~¡°Qê Q® =òYO 6. JO`ÍQêH›, 3 JOïH䛽 Z^ΰ~¡°Qê 2 JOïH =°i†Çò 5 JOïH䛽 Z^ΰ~¡°Qê 4
JOïH =ôO\ì~ò.
G ̄á x†Ç°=¶hß J~¡ÖO K͋¬°HËQ®ey`Í ‡zH›ä›½ ‹¬O|OkOz# Z…ìO\÷ ㄬ‰×߁<³á<Œ ‹¬°ƒ’OQê ªkOKÇ=KÇ°ó#°.

REASONING ABILITY (NON-VERBAL) ○ ○ ○ ○ ○ ○


262
○ ○ ○ ○ ○ ○ ○ ○ ○ ○ ○ ○ ○ ○ ○ ○ ○ ○ ○
Study Material
=¶ki ㄬ‰×߁°
; W=Þ|_# „¬@O…Õx „¦¬°<Œ#° …ÿH÷øOKÇ°@ : 3 „¦¬°<Œ° Q® x°=ô =~¡°‹¬ ‹¬OY¼ 5.

2 „¦¬°<Œ° Q® x°=ô =~¡°‹¬ ‹¬OY¼ 3.

1 „¦¬°#O Q® x°=ô =~¡°‹¬ ‹¬OY¼ 1.


1. ㄬH›ø# KǶ„¬|_# „¬@O…Õx "³ò`ÇëO
∴ "³ò`ÇëO „¦¬°<Œ ‹¬OY¼ « (5 þ 9) { (4 þ 7) {

(3 þ 5) { (2 þ 3) { (1 þ 1)
„¦¬°<Œ ‹¬OY¼#° …ÿH÷øOKÇO_.
« 45 { 28 { 15 { 6 { 1
ª^Î# : 3 „¦¬°<Œ° Q® x°=ô =~¡°‹¬ 1.
« 95.
2 „¦¬°<Œ° Q® x°=ô =~¡°‹¬° 2.

Y
1 „¦¬°#O Q® x°=ô =~¡°‹¬° 3.

M
∴ "³ò`ÇëO „¦¬°<Œ ‹¬OY¼
« (3 þ 1) { (2 þ 2) { (1 þ 3) 3. ㄬH›ø# KǶ„¬|_# zã`ÇO…Õ Zxß „¦¬°<Œ°

E
« 3 { 4 { 3

D
=°# H›O\÷H÷ H›x„²OKÇ_ÈO …è^ΰ?

9
« 10.

59
ª^Î# : "³ò`ÇëO „¦¬°<Œ ‹¬OY¼

CA
95
« (3 þ 1) { (4 þ 2) { (4 þ 1)
2. ㄬH›ø# KǶ„¬|_# „¬@O…Õx "³ò`ÇëO

50
« 3 { 8 { 4 « 15
98
A
„¦¬°<Œ ‹¬OY¼#° …ÿH÷øOKÇO_. H›O\÷H÷ H›x„²‹¬°ë#ßq « 10
:8
E

ª^Î# : 5 „¦¬°<Œ° Q® x°=ô =~¡°‹¬ ‹¬OY¼ 9. ∴ H›O\÷H÷ H›x„²OKÇxq « 15 - 10 « 5.


h
.P
IC

4 „¦¬°<Œ° Q® x°=ô =~¡°‹¬ ‹¬OY¼ 7.


AD

123456789012345678901234567890121234567890123456789012345678901212
12345678901234567890123456789012123456789012345678901234567890121234567890123456789012345678901212345678901234567890123456789012123
1234567890123456789012345678901212345678901234567890123456789012123456789012345678901234567890
12345678901234567890123456789012123456789012345678901234567890121234567890123456789012345678901212345678901234567890123
1234567890123456789012345678901212345678901234567890123456789012123456789012345678901234567890121234567890123
1234567890123456789012345678901212345678901234567890123456789012123456789012345678901234567890121234
123456789012345678901234567890121234567890123456789012345678901212
12345678901234567890123456789012123456789012345678901234567890121234567890123456789012345678901212345678901234567890123456789012123
1234567890123456789012345678901212345678901234567890123456789012123456789012345678901234567890
12345678901234567890123456789012123456789012345678901234567890121234567890123456789012345678901212345678901234567890123
1234567890123456789012345678901212345678901234567890123456789012123456789012345678901234567890121234567890123
1234567890123456789012345678901212345678901234567890123456789012123456789012345678901234567890121234
AB
O

12345678901234567890123456789012123456789012345678901234567890121234567890123456789012345678901212345678901234567890123456789012123
1234567890123456789012345678901212345678901234567890123456789012123456789012345678901234567890
123456789012345678901234567890121234567890123456789012345678901212
12345678901234567890123456789012123456789012345678901234567890121234567890123456789012345678901212345678901234567890123
1234567890123456789012345678901212345678901234567890123456789012123456789012345678901234567890121234567890123
1234567890123456789012345678901212345678901234567890123456789012123456789012345678901234567890121234
PRACTICE TEST - 1
12345678901234567890123456789012123456789012345678901234567890121234567890123456789012345678901212345678901234567890123456789012123
1234567890123456789012345678901212345678901234567890123456789012123456789012345678901234567890
12345678901234567890123456789012123456789012345678901234567890121234567890123456789012345678901212345678901234567890123
1234567890123456789012345678901212345678901234567890123456789012123456789012345678901234567890121234567890123
1234567890123456789012345678901212345678901234567890123456789012123456789012345678901234567890121234
123456789012345678901234567890121234567890123456789012345678901212
12345678901234567890123456789012123456789012345678901234567890121234567890123456789012345678901212345678901234567890123456789012123
1234567890123456789012345678901212345678901234567890123456789012123456789012345678901234567890
12345678901234567890123456789012123456789012345678901234567890121234567890123456789012345678901212345678901234567890123
1234567890123456789012345678901212345678901234567890123456789012123456789012345678901234567890121234567890123
1234567890123456789012345678901212345678901234567890123456789012123456789012345678901234567890121234
12345678901234567890123456789012123456789012345678901234567890121234567890123456789012345678901212345678901234567890123456789012123
1234567890123456789012345678901212345678901234567890123456789012123456789012345678901234567890
123456789012345678901234567890121234567890123456789012345678901212
12345678901234567890123456789012123456789012345678901234567890121234567890123456789012345678901212345678901234567890123
1234567890123456789012345678901212345678901234567890123456789012123456789012345678901234567890121234567890123
1234567890123456789012345678901212345678901234567890123456789012123456789012345678901234567890121234
12345678901234567890123456789012123456789012345678901234567890121234567890123456789012345678901212345678901234567890123456789012123
1234567890123456789012345678901212345678901234567890123456789012123456789012345678901234567890
ER
CH

; (1-8 ㄬ‰×߁°) XH› „¦¬°#O †³òH›ø Z^ΰï~^ΰ~¡° `Dž쁰 6. ï~O_È° `Dž쁰 =¶ã`Ç"Í° ~¡OQ®° "͆ǰ|_# „¦¬°<Œ° Zxß?
YD

Z~¡°„¬ô, hO =°i†Çò #°„¬ô ~¡OQ®°`Ë Ì„~òO\˜ 1) 36 2) 48 3) 24 4) 16


,H

"͆ǰ|_#k. P „¦¬°<Œxß 216 ‹¬=¶# „¦¬°<Œ°Qê 7. Zxß „¦°¬ <Œ †³òH›ø =ü_È° `Dž쁰 ̄~òO\˜ K͆°Ç |_¨Û~ò?
T
AM

qƒl
’ OKŒ~¡°. XHùøH›ø z#ß „¦°¬ #O ƒ°’ [O ‡Ú_È=ô 1 ̋O.g°. 1) 4 2) 6 3) 8 4) 10
H

8. Zxß „¦¬°<Œ †³òH›ø =ü_È° `Dž쁰 "Íö~Þ~¡° ~¡OQ®°#°


M

D ^Î`ŒëO‰×O P^¥~¡OQê ãH÷Ok ㄬ‰×߁䛽 ‹¬=¶^¥#O


AM

H›ey=ô<Œß~ò.
IG

~Œ†Ç°O_.
1) 10 2) 8 3) 6 4) 12
KH

1. Zxß „¦°¬ <Œ° ~


ï O_È° "³„
á ô¬  #°„¬ô ~¡OQ®°#° H›ey=ô<Œß~ò?
; (9 - 13 ㄬ‰×߁°) 10 ̋O.g°. ƒ’°[O H›ey# XH› Z~¡°„¬ô
R

1) 0 2) 4 3) 6 4) 8
2. XH›"³á„¬ô Z~¡°„¬ô =°i†Çò =°~ùH›"³á„¬ô #°„¬ô ~¡OQ®°°
~¡OQ®° „¦¬°#O#䛽 2 ̋O.g°. P䛽„¬KÇóx ~¡OQ®°`Ë Jxß
ㄬH›ø`ǁ=ò° ~¡OQ®° "͆ǰ|_#k. P „¦¬°#=ò#° 125
=¶ã`Ç"Í° H›ey†Çò#ß „¦¬°<Œ ‹¬OY¼
‹¬=¶# z#ß „¦¬°#=ò°Qê H›uëiOz<Œ~¡°. ̄á# W=Þ|_#
1) 12 2) 16 3) 24 4) 28
^Î`ŒëO‰×O P^¥~¡OQê D ãH÷Ok ㄬ‰×߁䛽 ‹¬=¶^¥<Œ°
3. XöH XH› `ǁO ~¡OQ®° "͆ǰ|_, Jk 䛀_¨ Z~¡°„¬ô~¡OQ®° ~Œ|@“O_.
J~ò#@°=O\÷ „¦¬°<Œ° Zxß? 9. XH›"³á„¬ô Z~¡°„¬ô~¡OQ®° ^¥x ㄬH›ø# P䛽„¬KÇó~¡OQ®° Q®
1) 32 2) 84 3) 36 4) 81 „¦¬°#=ò ‹¬OY¼
4. Z…ìO\÷ ~¡OQ®¶ "͆ǰ|_Èx „¦¬°<Œ ‹¬OY¼ 1) 6 2) 8 3) 16 4) 0
1) 48 2) 64 3) 72 4) 81 10. =ü_È° =òMìä›½ P䛽„¬KÇó~¡OQ®° "͆ǰ|_# „¦¬°#=ò
5. XH›ø `ǁO =¶ã`Ç"Í° ̄~òO\˜ "͆ǰ|_# „¦¬°<Œ ‹¬OY¼ ‹¬OY¼
1) 144 2) 88 3) 96 4) 102 1) 16 2) 6 3) 8 4) 0

REASONING ABILITY (NON-VERBAL) ○ ○ ○ ○ ○ ○


263
○ ○ ○ ○ ○ ○ ○ ○ ○ ○ ○ ○ ○ ○ ○ ○ ○ ○ ○
Study Material
11. H›h‹¬O XH› =òYO ~¡OQ®° "͆ǰ|_# „¦¬°#=ò ‹¬OY¼ 17. KǶ„¬|_# zã`ÇO…Õ U~¡æ_# "³ò`ÇëO „¦¬°<Œ ‹¬OY¼
1) 102 2) 98 3) 106 4) 96 1) 27 2) 32 3) 48 4) 24
12. H›h‹¬O ï~O_È° "³á„¬ô P䛽„¬KÇó~¡OQ®° Q® „¦¬°#=ò ‹¬OY¼ 18. öH=O XH›"³á„¬ô =¶ã`Ç"Í° he~¡OQ®°
1) 44 2) 36 3) 48 4) 27
H›e¾†Çò#ß „¦¬°<Œ ‹¬OY¼
13. U "³á„¬ô 䛀_¨ ~¡OQ®° "͆ǰ|_Èx „¦¬°#=ò ‹¬OY¼ 1) 8
1) 64 2) 36 3) 16 4) 27
2) 4
; (14-18 ㄬ‰ß× °) XH› „¦°¬ #=ò#䛽 he~¡OQ®°…Õ 4 "³„
á ô¬ ° 3) 6
ㄬH›øㄬH›ø# =°i†Çò P䛽„¬KÇó ~¡OQ®°`Ë q°ye# ï~O_È° 4) 12
Z^ΰ~¡° =òMìä›½ ~¡OQ®° "͆°Ç |_#k. P „¦°¬ <Œxß <Œ°Q®° 19. XH› ‹¬ = °„¦ ¬ ° #O †³ ò H› ø Jxß "³ á „ ¬ ô …ì „¬ ‹ ¬ ° „¬ ô ~¡ O Q® °
‹¬=¶#"³°#Ø Hù`ǁ`Ë x°=ôQê ̄á #°O_ ãH÷OkH÷ =°i†Çò "͆ǰ|_#k. P ‹¬=°„¦¬°<Œxß ‹¬=¶# „¦¬°#„¬i=¶}O Q®
ï ~ O_È ° ‹¬ = ¶#"³ ° Ø # Hù`Ç  `Ë J_È Û O Qê ㄬ H › ø ä› ½ 27 z#ß ‹¬=°„¦¬°<Œ°Qê H›uëiOz# XH›"³á„¬ô# =¶ã`Ç"Í°

Y
H›uëiOKÇ_ÈO [iy#k. „¬@O…Õ KǶ„¬|_# KÇ°H›ø …ÿá<£° „¬‹¬°„¬ô ~¡OQ®° H›ey# „¦¬°<Œ ‹¬OY¼ (R.R.B. - 2004)

M
H›uëiO„¬ô#° ‹¬¶zªë~ò. „¬\ìxß ƒìQê J^Î¼†Ç°#O K͋² 1) 16 2) 6 3) 12 4) 8
D ãH÷Ok ㄬ‰×߁䛽 ‹¬=¶^¥<Œ° ~Œ†Ç°O_.
20. XH› „¦¬°#O †³òH›ø Jxß "³á„¬ô…ì XöH ~¡OQ®°`Ë Ì„~òO\˜

E
14. U "³á„¬ô 䛀_¨ ~¡OQ®° "͆ǰ|_Èx „¦¬°<Œ ‹¬OY¼
"ͪ~¡°. P „¦¬°#O †³òH›ø ƒ’°[O…Õ = =O`Ç° ƒ’°[O ¼

D
9
1) 1 2) 2 3) 3 4) 4

59
‡Ú_È = ô H› e y†Ç ò O_È ° #@°Á P „¦ ¬ ° #O "³ ò `Œë x ß
15. öH=O XH›"³á„¬ô䛽 P䛽„¬KÇó~¡OQ®° "͆ǰ|_# „¦¬°#=ò
CA
95
‹¬=°„¦°¬ <Œ°Qê H›ui
ë Oz# U~¡æ_È° z#ß „¦°¬ <Œ…Õ XH›"„
á³ ô¬
‹¬OY¼

50
=¶ã`Ç"Í° "͆ǰ|_# ~¡OQ®°#° H›ey†ÇòO_È° „¦¬°<Œ ‹¬OY¼
1) 3 2) 4 3) 2 4) 6
98
A
(M.B.A. - 2006)
16. =ü_È°"³á„¬ô…ì ~¡OQ®° H›ey†Çò#ß „¦¬°<Œ ‹¬OY¼
:8
1) 48 2) 12 3) 6 4) 24
1) 12 2) 10 3) 8 4) 16
E
h

1234567890123456789012
12345678901234567890123456789012123456789012345678901234567890121234567890123456789012345678901212345678901234567890123
.P

1234567890123456789012 12345678901234567890123
12345678901234567890123456789012123456789012345678901234567890121234567890123456789012345678901212345678901234567890123
12345678901234567890123
q=~¡}ì`ÇàH› ["Œ|°°
IC

1234567890123456789012
12345678901234567890123456789012123456789012345678901234567890121234567890123456789012345678901212345678901234567890123
1234567890123456789012 12345678901234567890123
12345678901234567890123
12345678901234567890123456789012123456789012345678901234567890121234567890123456789012345678901212345678901234567890123
1234567890123456789012 PRACTICE TEST - 1 : 12345678901234567890123
12345678901234567890123456789012123456789012345678901234567890121234567890123456789012345678901212345678901234567890123
AD

1234567890123456789012
12345678901234567890123456789012123456789012345678901234567890121234567890123456789012345678901212345678901234567890123
1234567890123456789012 12345678901234567890123
12345678901234567890123
12345678901234567890123456789012123456789012345678901234567890121234567890123456789012345678901212345678901234567890123
AB
O

;˜ (1 #°O_ 8 ㄬ‰×߁䛽 q=~¡}°) (1)


ER

3. XöH XH› `ǁO H


ö =O Z~¡°„¬ô~¡OQ®° H›ey†Çò#ß „¦°
¬ <Œ
CH

S3 = 216 ‹¬OY¼
YD

2
⇒ 4          « 2(ƒ’°[O-2)
,H

2

T

̄^Î턦¬°#O ƒ’°[O ‡Ú_È=ô « 6 ̋O.g°. « 2(6-2)


AM

2
« 2(4)
H
M

« 2(16)
AM

Blue
IG

« 32
KH

(2)
Black
4. Z…ìO\ ÷ ~¡OQ®¶ "͆ǰ|_Èx „¦¬°<Œ ‹¬OY¼

3
R

« (ƒ’°[O-2)

3
« (6-2)

3
Red « 4

1. (1) #°„¬ô ~¡OQ®° Z^ΰï~^ΰ~¡° `Džìä›½ "͆ǰ|_È@O =#


« 64.

ï~O_È° "³á„¬ô #°„¬ô~¡OQ®° H›ey# „¦¬°<Œ° =ôO_È=ô.


5. (3) XH›ø `ǁO =¶ã`Ç"°
Í Ì„~òO\˜ "͆°
Ç |_# „¦°
¬ <Œ ‹¬OY¼

2
∴ ‹¬=¶^¥#O '0Ñ.
« 6(ƒ’°[O-2)

2
2. (2) „¬@O…Õ KÇ°H›ø`Ë KǶ„¬|_# „¦¬°<Œ° ̄á`ǁO…Õ '8Ñ
« 6(6-2)

2
« 6þ4
=°i†Çò ãH÷Ok `ǁO…Õ '8Ñ, "³ò`ÇO
ë 16 H
ö =O XH›"„
á³ ô
¬

« 6þ16
Z~¡°„¬ô =°i†Çò =°~ùH›"³á„¬ô #°„¬ô~¡OQ®°#° H›ey

« 96.
=ôO\ì~ò.

REASONING ABILITY (NON-VERBAL) ○ ○ ○ ○ ○ ○


264
○ ○ ○ ○ ○ ○ ○ ○ ○ ○ ○ ○ ○ ○ ○ ○ ○ ○ ○
Study Material
6. (2) ~
ï O_È° `Dž쁰 =¶ã`Ç"°
Í ~¡OQ®°"͆°
Ç |_# „¦°
¬ <Œ ‹¬OY¼ 13. (4) ̄^Îí „¦¬°#=ò…Õ U `ǁO"³á„¬ô …è䛽O_¨ =°^Î¼…Õ =ô#ß

« 12 (ƒ’°[O-2)
„¦¬°#=òä›½ Z…ìO\ ÷ ~¡OQ®°"͆ǰ|_È^ΰ.

« 12(6-2)
̄^Î턦¬°#O †³òH›ø Jxß `Dž쁰 H›ey†Çò#ß „¦¬°#=ò

« 12þ4
‹¬OY¼ « 98

« 48.

∴ =°^Î¼…Õx „¦¬°#=ò ‹¬OY¼ « 125 - 98 « 27

7. (3) =ü…ÕÁ =ô#ß „¦¬°<ŒöH =ü_È° `Dž쁰 ̄~òO\˜

K͆ǰ|_È`Œ~ò. ;˜ (14 #°O_ 18 ㄬ‰×߁䛽 q=~¡}°)


∴ 8 „¦¬°<Œ †³òH›ø =ü_È° `Dž쁰 ̄~òO\˜ Wzó# „¦°
¬ #=ò#° "³ò`ÇO
ë 3 ‹¬=¶# ƒìQꁰQê H›ui
ë OKÇ

K͆ǰ|_¨Û~ò.
=KÇ°ó#°. Jq ̄჏ìQ®O, =°^Î¼ƒìQ®O =°i†Çò ãH÷Ok ƒìQ®O.

8. (2) „¦°
¬ #O †³òH›ø =ü…Õ =ô#ß „¦°
¬ <Œ° =¶ã`Ç"°
Í =ü_È°

"Íö~Þ~¡° ~¡OQ®°#° H›ey =ôO\ì~ò.


1. Ì„á ƒìQ®O P䛽„¬KÇóx ~¡OQ®°

Y
he ~¡OQ®°
XH› „¦¬°<ŒxH÷ 8 =ü° =ôO\ì~ò.

M
=ü_È° "Íö~Þ~¡° ~¡OQ®°#° H›ey†Çò#ß `Džì#°

H›ey# „¦¬°<Œ ‹¬OY¼ « 8.

ãH÷Ok ƒìQ®O ~¡OQ®° "͆ǰ…è^ΰ

E
;˜ (9 #°O_ 13 ㄬ‰×߁䛽 q=~¡}°) P䛽„¬KÇó
2. =°^Î¼ ƒìQ®O ̄á# ~¡OQ®°"͆ǰx ƒìQ®O

D
9
59
he ~¡OQ®° (ㄬH›øä›½)
CA
95
50
Z~¡°„¬ô
ãH÷Ok ƒìQ®O ~¡OQ®° "͆ǰ…è^ΰ
98
A
:8
3. ãH÷Ok ƒìQ®O ̄á# ~¡OQ®°"͆ǰx ƒìQ®O
E
h
.P

he ~¡OQ®° (ㄬH›øä›½)
IC
AD

(4)
P䛽„¬KÇóx ~¡OQ®° (ãH÷Ok ƒìQ®O)
9. XH›"³á„¬ô Z~¡°„¬ô~¡OQ®°, ^¥x ㄬH›ø# P䛽„¬KÇó ~¡OQ®°
AB
O

"͆ǰ|_# „¦¬°<Œ° …è=ô. J#Qê ‹¬=¶^¥#O '0Ñ.


ER
CH

10. (3) ̄^Î í „¦ ¬ ° #O †³ ò H› ø Jxß =ü Q®  „¦ ¬ ° <Œä› ½ 14. (1) =°^Î¼ƒìQ®O…Õx „¦¬°#=ò…Õx =°^Î¼„¦¬°#=ò#䛽
YD

=ü_È°"³á„¬ô…ì P䛽„¬KÇó ~¡OQ®° "͆ǰ|_ =ôO@°Ok. =¶ã`Ç"Í° U ~¡OQ®¶ =ôO_È^ΰ.


,H

„¦¬°#O †³òH›ø =ü° '8Ñ. Hê=ô# =ü°


15. (3) „¬\ì…Õ Ì„áƒì
 Q®O =°i†Çò ãH÷Ok ƒìQꁅÕx =°^¼
Î …Õ
T
AM

H›ey†ÇòO_È° „¦¬°<Œ ‹¬OY¼ « 8.


Q® „¦°
¬ #=ò =¶ã`Ç"°
Í H
ö =O P䛽„¬Kó
Ç ~¡OQ®° "͆°
Ç |_


H

=ü_È° =òMìä›½ P䛽„¬Kó


Ç ~¡OQ®° "͆°
Ç |_# „¦°
¬ <Œ
M

=ôO@°Ok. J#Qê öH=O XH›"³á„¬ô䛽 P䛽„¬KÇó ~¡OQ®°

‹¬OY¼ « 8.
AM
IG

"͆ǰ|_# „¦¬°#=ò ‹¬OY¼ « 2.

11. (2) ̄^Îí „¦¬°#O †³òH›ø Jxß `Džì"³á„¬ô Q® z#ß

(3)
KH

16. „¦¬°#=ò †³òH›ø =ü° =ô#ß „¦¬°<ŒxßO\ ÷H©

„¦¬°<ŒxßO\ ÷H© H›h‹¬O XH›"³á„¬ô ~¡OQ®°"͆ǰ|_


R

=ü_È°"³„
á ô
¬ …ì ~¡OQ®° =ôO@°Ok. ㄬ‰ß
× …Õ Wzó# „¬@O#°

=ôO@°Ok.

Q®=°xOKÇQê P „¦¬°#O †³òH›ø 8 =ü…Õ 8


Jxß `Dž쁰 H›ey†Çò#ß „¦¬°<Œ ‹¬OY¼ « 98

∴ H›h‹¬O XH›"„
á³ ô
¬ ~¡OQ®°"͆°
Ç |_# „¦°
¬ <Œ ‹¬OY¼ «
„¦¬°<Œ°<Œß~ò.

98.
17. (1) =ü_È° ƒìQê #°O_ "³ò`ÇëO „¦¬°#=ò ‹¬OY¼ «

12. (1) ̄^Îí „¦°


¬ #O †³òH›ø JOKÇ° =^Îí Q® z#ß „¦°
¬ <ŒxßO\ H
÷ © 27.

ï ~ O_È ° "³ á „ ¬ ô …ì Pä› ½ „¬ K Ç ó ~¡ O Q® ° =ôO@°Ok. H› h ‹¬ O 18. (2) Ì„á „¬\쁅Õx =°^Î¼ƒìQ®O#° Q®=°xOz#, =°^Î¼

~
ï O_È°"³„
á ô
¬ …ì P䛽„¬Kó
Ç ~¡OQ®° =ôO_¨e Hê|\ “÷ ̄^΄
í °
¬¦ #O
ƒìQ®O…Õx 4 =òMì…Õ Q® =°^Î¼ „¦¬°#=ò° öH=O

†³òH›ø =ü…ì Q® „¦¬°<Œ#° 䛀_¨ …ÿH÷øOKŒe.


XH›"³á„¬ô =¶ã`Ç"Í° he~¡OQ®°#° H›ey=ô<Œß~ò.

„¬@O #°O_ J…ìO\ ÷q "³ò`ÇëO 44 =ô<Œß~ò.


∴ XH›"³á„¬ô䛽 he~¡OQ®°`Ë "͆ǰ|_# „¦¬°#=ò ‹¬OY¼

∴ H›h‹¬O ï~O_È° "³á„¬ô P䛽„¬KÇó~¡OQ®° Q® „¦¬°#=ò


« 4.

‹¬OY¼ « 44.

REASONING ABILITY (NON-VERBAL) ○ ○ ○ ○ ○ ○


265
○ ○ ○ ○ ○ ○ ○ ○ ○ ○ ○ ○ ○ ○ ○ ○ ○ ○ ○
Study Material
19. (2) ㄬH›ø# KǶ„¬|_# „¬@O…Õ l Q®°~¡°ë H›ey# „¦¬°<Œä›½ 20. (4) „¦ ¬ ° #O ƒ ’ ° [O ‡Ú_È = ô 1

¼
=¶ã`Ç"Í° XH›"³á„¬ô „¬‹¬°„¬ô~¡OQ®° =ôO@°Ok. J…ìO\ ÷q †Çüx\˜ J#°Hùx# z#ß

XHùøH›ø =òMìxH÷ XH›ø\ ÷ =¶ã`Ç"Í° =ôO@°Ok. "³ò`ÇëO „¦¬°#O ƒ’°[O ‡Ú_È=ô

†Çüx\˜ J=ô`Ç°Ok.
6 „¦¬°<Œ° J…ìO\ ÷q =ôO\ì~ò.

∴ "³ò`ÇëO U~¡æ_È° „¦¬°<Œ

‹¬OY¼ « 4 þ 16 « 64.

XH› =òYO"³ á „ ¬ ô =°^ Î ¼ …Õ

=ô#ß 4 „¦¬°<Œ° =¶ã`Ç"Í° XH›"³á„¬ô# ~¡OQ®° H›ey

=ôO\ì~ò. „¬@O…Õ "Œ\ ÷x KÇ°H›ø ( l) Q®°~¡°ë`Ë

KǶ„¬_ÈO [iy#k.

∴ ∴

Y
XH›"³á„¬ô# =¶ã`Ç"Í° „¬‹¬°„¬ô~¡OQ®° H›ey# „¦¬°<Œ 6 =òMì „¬~O
¡ Qê "³ò`ÇO
ë XH›"„
á³ ô
¬ # =¶ã`Ç"°
Í ~¡OQ®°

‹¬OY¼ « 6. "͆ǰ|_# z#ß „¦¬°<Œ ‹¬OY¼ « 6 þ 4 « 24.

M
123456789012345678901234567890121234567890123456789012345678901212
12345678901234567890123456789012123456789012345678901234567890121234567890123456789012345678901212345678901234567890123
1234567890123456789012345678901212345678901234567890123456789012123456789012345678901234567890121234567890123
1234567890123456789012345678901212345678901234567890123456789012123456789012345678901234567890121234
12345678901234567890123456789012123456789012345678901234567890121234567890123456789012345678901212345678901234567890123456789012123
1234567890123456789012345678901212345678901234567890123456789012123456789012345678901234567890
12345678901234567890123456789012123456789012345678901234567890121234567890123456789012345678901212345678901234567890123
1234567890123456789012345678901212345678901234567890123456789012123456789012345678901234567890121234567890123
1234567890123456789012345678901212345678901234567890123456789012123456789012345678901234567890121234
123456789012345678901234567890121234567890123456789012345678901212
12345678901234567890123456789012123456789012345678901234567890121234567890123456789012345678901212345678901234567890123456789012123
1234567890123456789012345678901212345678901234567890123456789012123456789012345678901234567890
12345678901234567890123456789012123456789012345678901234567890121234567890123456789012345678901212345678901234567890123
1234567890123456789012345678901212345678901234567890123456789012123456789012345678901234567890121234567890123
1234567890123456789012345678901212345678901234567890123456789012123456789012345678901234567890121234
12345678901234567890123456789012123456789012345678901234567890121234567890123456789012345678901212345678901234567890123456789012123
1234567890123456789012345678901212345678901234567890123456789012123456789012345678901234567890
123456789012345678901234567890121234567890123456789012345678901212
PRACTICE TEST - 2

E
12345678901234567890123456789012123456789012345678901234567890121234567890123456789012345678901212345678901234567890123
1234567890123456789012345678901212345678901234567890123456789012123456789012345678901234567890121234567890123
1234567890123456789012345678901212345678901234567890123456789012123456789012345678901234567890121234
12345678901234567890123456789012123456789012345678901234567890121234567890123456789012345678901212345678901234567890123456789012123
1234567890123456789012345678901212345678901234567890123456789012123456789012345678901234567890
123456789012345678901234567890121234567890123456789012345678901212
12345678901234567890123456789012123456789012345678901234567890121234567890123456789012345678901212345678901234567890123456789012123
1234567890123456789012345678901212345678901234567890123456789012123456789012345678901234567890
12345678901234567890123456789012123456789012345678901234567890121234567890123456789012345678901212345678901234567890123
1234567890123456789012345678901212345678901234567890123456789012123456789012345678901234567890121234567890123
1234567890123456789012345678901212345678901234567890123456789012123456789012345678901234567890121234
12345678901234567890123456789012123456789012345678901234567890121234567890123456789012345678901212345678901234567890123456789012123
1234567890123456789012345678901212345678901234567890123456789012123456789012345678901234567890
12345678901234567890123456789012123456789012345678901234567890121234567890123456789012345678901212345678901234567890123
1234567890123456789012345678901212345678901234567890123456789012123456789012345678901234567890121234567890123
1234567890123456789012345678901212345678901234567890123456789012123456789012345678901234567890121234
123456789012345678901234567890121234567890123456789012345678901212
12345678901234567890123456789012123456789012345678901234567890121234567890123456789012345678901212345678901234567890123456789012123
1234567890123456789012345678901212345678901234567890123456789012123456789012345678901234567890
1234567890123456789012345678901212345678901234567890123456789012123456789012345678901234567890121234567890123
1234567890123456789012345678901212345678901234567890123456789012123456789012345678901234567890121234

D
9
59
1. D ãH÷O^Î W=Þ|_# „¬\ì…Õ ? Q®°~¡°ë =ô#ßKË@ =KÇ°ó 4. D ãH÷Ok „¬\ì #°O_ 3䛽 Z^ΰ~¡°Qê =KÇ°ó JOïH
CA
95
JH›Æ~Œ° Uq?

50
98
A
:8
(i) (ii) (iii) (iv)
E
h

(i) (ii) (iii) 1) 6 2) 1 3) 4 4) 5


.P

R, S P, K Q, R K, P 5. XH› ‡zH›#° <Œ°Q®°ª~¡°Á q‹²i, P <Œ°Q®° ƒ’Oy=°°


IC

1) 2) 3) 4)
AD

2. D ãH÷O^Î W=Þ|_# „¬\ì#° J#°‹¬iOz P ‡zH›#° ãH÷O^Î KǶ„²# q^ÎOQê =ôO>è, 2䛽 Ja=òYOQê =ôO_Í
`³iz#@Á~ò# U~¡æ_È° „¬@O. JOïH Uk?
AB
O

(Dy. Jailors - 2012)


ER
CH

   
YD

       
,H
T
AM

1) 1 2) 3 3) 5 4) 4
H

1) 2)
6. XH› ‡zH› †³òH›ø ï~O_È° ƒ’Oy=°° ãH÷O^Î KǶ„¬|_¨Û~ò.
M

P ‡zH› †³òH›ø ̄á =òYŌá =ô#߄¬ô_È° J_È°Q®°# =ôO_È°


AM
IG

JOïH (Dy. Jailors - 2012)


KH
R

1) 3

3) 4)

2) 6

3) 4

3. D ãH÷Ok „¬\ì #°O_ Yellow ~¡OQ®°ä›½ Z^ΰ~¡°Qê =KÇ°ó 4) 2

~¡OQ®° 7. ㄬH›ø# KǶ„¬|_# „¬@O…Õx "³ò`ÇëO „¦¬°<Œ ‹¬OY¼


1) 110

2) 100

1) Purple 2) Blue 3) 120

3) Orange 4) Red 4) 130

REASONING ABILITY (NON-VERBAL) ○ ○ ○ ○ ○ ○


266
○ ○ ○ ○ ○ ○ ○ ○ ○ ○ ○ ○ ○ ○ ○ ○ ○ ○ ○
Study Material
; (8-12)XH› „¦¬°#=ò †³òH›ø ƒ’°[O 8 ̋O.g°. P 11. U^Ë XH› "³á„¬ô =¶ã`Ç"Í° ̄~òO\˜ K͆ǰ|_# „¦¬°#=ò
„¦ ¬ ° #=ò#ä› ½ Z~¡ ° „¬ ô , hO, #°„¬ ô ~¡ O Q® ° #° ‹¬OY¼
Z^ΰï~^ΰ~¡°Qê =ô#ß =òMìä›½ ̄~òO@° "͋² 2 ̋O.g°. 1) 28 2) 8 3) 16 4) 24

ƒ’°[O Q® „¦¬°#=ò°Qê H›uëiOKÇ_ÈO [iy#k. (8 #°O_ 12. U =òY=ò#䛽 ~¡OQ®° …è䛽O_¨ =ô#ß „¦¬°<Œ ‹¬OY¼
12 ㄬ‰×߁°) (Communication S.I. - 2013) 1) 0 2) 4 3) 8 4) 12

13. ㄬH›ø# KǶ„¬|_# „¬@O…Õx "³ò`ÇëO


„¦¬°<Œ ‹¬OY¼
(SSC level Data entr y

operator & LDC Exam 2012)

1) 256 2) 196 3) 144 4) 160

Y
14. ㄬH›ø# KǶ„¬|_# „¬@O…Õx "³ò`ÇëO
„¦¬°<Œ ‹¬OY¼ (SSC level Data en

M
8. =ü_È ° =òY=ò° "Í ö ~ Þ~¡ ° ~¡ O Q® ° `Ë Ì„~òO\˜
try operator & LDC Exam 2012)
K͆ǰ|_# „¦¬°<Œ ‹¬OY¼

E
1) 10 2) 12 3) 14 4) 16

D
1) 6 2) 8 3) 10 4) 12

15. ㄬH›ø# KǶ„¬|_# „¬@O…Õx "³ò`ÇëO „¦¬°<Œ ‹¬OY¼

9
9. =ü_È° =òY=ò° ̄~òO\˜ K͆ǰ|_# „¦¬°#=ò ‹¬OY¼

59
CA (SSC Graduate Level Tier -I 2013)

95
1) 12 2) 16 3) 8 4) 20

1) 24

10. ~
ï O_È° =òY=ò° =¶ã`Ç"°Í ̄~òO\˜ K͆°Ç |_# „¦°¬ #=ò

50
2) 16

‹¬OY¼
98
A
3) 18
:8
1) 16 2) 20 3) 18 4) 24
4) 20
E
h
.P

1234567890123456789012
12345678901234567890123456789012123456789012345678901234567890121234567890123456789012345678901212345678901234567890123
12345678901234567890123
IC

q=~¡}ì`ÇàH› ["Œ|°°
12345678901234567890123456789012123456789012345678901234567890121234567890123456789012345678901212345678901234567890123
1234567890123456789012
1234567890123456789012 12345678901234567890123
1234567890123456789012 12345678901234567890123
12345678901234567890123456789012123456789012345678901234567890121234567890123456789012345678901212345678901234567890123
12345678901234567890123
12345678901234567890123456789012123456789012345678901234567890121234567890123456789012345678901212345678901234567890123
AD

1234567890123456789012
1234567890123456789012
PRACTICE TEST - 2 :
12345678901234567890123456789012123456789012345678901234567890121234567890123456789012345678901212345678901234567890123
1234567890123456789012 12345678901234567890123
12345678901234567890123
12345678901234567890123456789012123456789012345678901234567890121234567890123456789012345678901212345678901234567890123
12345678901234567890123
12345678901234567890123456789012123456789012345678901234567890121234567890123456789012345678901212345678901234567890123
AB
O
ER

(4)
CH

1. W=Þ|_# „¬\쁅Õ
3. (1)
YD

"³ò^Î\ ÷ „¬@O #°O_ → =°i†Çò

Yellow – Blue – Orange


,H

Yellow – Pink – Red


T


AM

ï~O_È= „¬@O #°O_



H

Purple
M

Blue, Orange, Red Yellow


AM

Hê=ô# =ü_È= „¬@O…Õ {5 K͋²# J#Qê …Õ Un 䛽


IG

Z^Î ° ~¡ ° Qê =ôO_È = ô. Hê=ô# Wzó# P„¬  <£  #°O_


KH

Yellow Purple.
=KÇ°ó#°.

䛽 Z^ΰ~¡°Qê =ôO_È°#k


R

2. (3) W=Þ|_# „¬\ì #°O_

 
2䛽 ㄬH›ø# Q® JOïH° 1, 3, 4, 6.
(3)  
4.
 
2 - 6 - 4

3䛽 ㄬH›ø# 1, 2, 5, 6 =ô<Œß~ò.

2 - 1 - 3

Hê=ô# 3䛽 Z^ΰ~¡°Qê =ôO_È° JOïH 4.


5. (3) 2䛽 ㄬH›ø# Q® JOïH° 1, 3, 4, 6

° Hê=ô# 2 䛽 Ja=òYOQê =ôO_Í JOïH 5.

nx #°O_ ‹¬æ+¬“OQê, 5䛽 Z^ΰ~¡°Qê 2 =ôO@°Ok. 6. (2) ‡zH› x†Ç°=¶ #°O_, ‡zH› †³òH›ø ï~O_È°

1䛽 Z^ΰ~¡°Qê 6 =ôO@°Ok "Í~


ö Þ~¡° ªÖ<Œ…Õ L=°à_Qê =ôO_Í =òYO †³òH›ø ªÖ#O

XH›>è J~ò`Í, q°Q®`Œ =òMìä›½ Ja=òYOQê =ôO_Í


=°i†Çò

=òM쁰 J^Í ªÖ<Œ…Õ =ôO\ì~ò.


3䛽 Z^ΰ~¡°Qê 4 =ôO@°Ok.

Hê=ô# 1䛽 Ja=òYOQê 6 =ôO@°Ok.

REASONING ABILITY (NON-VERBAL) ○ ○ ○ ○ ○ ○


267
○ ○ ○ ○ ○ ○ ○ ○ ○ ○ ○ ○ ○ ○ ○ ○ ○ ○ ○
Study Material
7. (1) "³ò`ÇëO „¦¬°<Œ ‹¬OY¼ « (2þ25) { (4þ5) { (4þ6) 9. (3) =ü_È° =òY=ò° ̄~òO\˜ K͆ǰ|_# „¦¬°#=ò

{ (4þ2) { (3þ2) { (1þ2)


‹¬OY¼ « 4 { 4 « 8.

« 50{20{24{8{6{2
10. (4) ï ~ O_È ° =òY=ò° =¶ã`Ç " Í ° ̄~òO\˜ KÍ † Ç ° |_ #

« 110.
„¦¬°<Œ ‹¬OY¼ « 8 { 8 { 4 { 4 « 24.

; ƒ’°[O ‡Ú_È=ô 8 ̋O.g°. Q® „¦¬°<Œxß 2 ̋O.g°. ƒ’°[O


11. (4) U^Ë XH› "³á„¬ô =¶ã`Ç"Í° ̄~òO\˜ K͆ǰ|_# „¦¬°<Œ

=ôO_È°#@°Á qƒ’lOKŒ~¡°.
‹¬OY¼ « 6 þ 4 « 24.

"³ò`ÇëO „¦¬°<Œ ‹¬OY¼ « 16 þ 4 « 64.

12. (3) U =òMìxH÷ ~¡OQ®°° …è䛽O_¨ =ô#ß „¦¬°<Œ°

« =°^Î¼…Õx „¦¬°<Œ°

« 4 { 4

« 8 „¦¬°<Œ°.

Y
13. (3) "³ò`ÇëO „¦¬°<Œ ‹¬OY¼ « 6 þ 24 « 144

M
14. (2) "³ò`ÇëO „¦¬°<Œ ‹¬OY¼ « 12.

15. (4) "³ò`ÇëO „¦¬°<Œ ‹¬OY¼

E
« (4þ1) { (3 þ2) { (2þ3) { (1 þ 4)

(2)

D
9
8. =ü_È° =òY=ò° "Íö~Þ~¡° ~¡OQ®°`Ë Ì„~òO\˜

59
« 4 { 6 { 6 { 4

K͆ǰ|_# „¦¬°#=ò ‹¬OY¼ « ̄^Îí „¦¬°#O †³òH›ø

CA
95
« 20.

j~ŒÂ =^Îí =ôO_È° „¦¬°<Œ ‹¬OY¼ « 8.

123456789012345678901234567890121234567890123456789012345678901212

50
12345678901234567890123456789012123456789012345678901234567890121234567890123456789012345678901212345678901234567890123456789012123
123456789012345678901234567890121234567890123456789012345678901212345678901234567890123
123456789012345678901234567890121234567890123456789012345678901212
12345678901234567890123456789012123456789012345678901234567890121234567890123456789012345678901212345678901234567890123
1234567890123456789012345678901212345678901234567890123456789012123456789012345678901234567890121234567890123
1234567890123456789012345678901212345678901234567890123456789012123456789012345678901234567890121234
12345678901234567890123456789012123456789012345678901234567890121234567890123456789012345678901212345678901234567890123456789012123
123456789012345678901234567890121234567890123456789012345678901212345678901234567890123
12345678901234567890123456789012123456789012345678901234567890121234567890123456789012345678901212345678901234567890123
1234567890123456789012345678901212345678901234567890123456789012123456789012345678901234567890121234567890123
1234567890123456789012345678901212345678901234567890123456789012123456789012345678901234567890121234
123456789012345678901234567890121234567890123456789012345678901212
12345678901234567890123456789012123456789012345678901234567890121234567890123456789012345678901212345678901234567890123456789012123
123456789012345678901234567890121234567890123456789012345678901212345678901234567890123
12345678901234567890123456789012123456789012345678901234567890121234567890123456789012345678901212345678901234567890123
1234567890123456789012345678901212345678901234567890123456789012123456789012345678901234567890121234567890123
1234567890123456789012345678901212345678901234567890123456789012123456789012345678901234567890121234
PREVIOUS BITS
12345678901234567890123456789012123456789012345678901234567890121234567890123456789012345678901212345678901234567890123456789012123
123456789012345678901234567890121234567890123456789012345678901212345678901234567890123
123456789012345678901234567890121234567890123456789012345678901212
98
A
12345678901234567890123456789012123456789012345678901234567890121234567890123456789012345678901212345678901234567890123
1234567890123456789012345678901212345678901234567890123456789012123456789012345678901234567890121234567890123
1234567890123456789012345678901212345678901234567890123456789012123456789012345678901234567890121234
12345678901234567890123456789012123456789012345678901234567890121234567890123456789012345678901212345678901234567890123456789012123
123456789012345678901234567890121234567890123456789012345678901212345678901234567890123
12345678901234567890123456789012123456789012345678901234567890121234567890123456789012345678901212345678901234567890123
1234567890123456789012345678901212345678901234567890123456789012123456789012345678901234567890121234567890123
1234567890123456789012345678901212345678901234567890123456789012123456789012345678901234567890121234
123456789012345678901234567890121234567890123456789012345678901212
12345678901234567890123456789012123456789012345678901234567890121234567890123456789012345678901212345678901234567890123456789012123
123456789012345678901234567890121234567890123456789012345678901212345678901234567890123
12345678901234567890123456789012123456789012345678901234567890121234567890123456789012345678901212345678901234567890123
1234567890123456789012345678901212345678901234567890123456789012123456789012345678901234567890121234567890123
1234567890123456789012345678901212345678901234567890123456789012123456789012345678901234567890121234
12345678901234567890123456789012123456789012345678901234567890121234567890123456789012345678901212345678901234567890123456789012123
123456789012345678901234567890121234567890123456789012345678901212345678901234567890123
12345678901234567890123456789012123456789012345678901234567890121234567890123456789012345678901212345678901234567890123456789012123
123456789012345678901234567890121234567890123456789012345678901212345678901234567890123
:8
E

1. ï~O_È° ‡zH›° H÷Ok q^ÎOQê L<Œß~ò. WO^ΰ…Õ 5 xö~í‰×Hꁰ : ãH÷Ok ‹¬=¶KŒ~Œxß KÇkq, 4 #°O_ 8 =~¡ä›½ Q®
h
.P

aO^ΰ=ô° Q® =òMìxH÷ Z^ΰ~¡°Qê L#ß =òYŌá ㄬ‰ß× ä›½ ‹¬=¶^¥#=ò ~Œ†Ç°O_. (A.P.S.I.Mains-2017)
IC
AD

Q® aO^ΰ=ô ‹¬OY¼ (T.S. S.I.Mains -2016) XH› „¦¬°#=ò †³òH›ø ï~O_È° P‹¬#ß L„¬i`Džìä›½ Zã~¡x
AB
O

~¡OQ®° "͆ǰ|_#k. Zã~¡x L„¬i`Džìä›½ Z^ΰ~¡°Qê L#ß


ER
CH

L„¬i`Džìä›½ #Á~¡OQ®° "͆ǰ|_#k. q°ye#

L„¬i`Džìä›½ P䛽„¬KÇó~¡OQ®° "͆ǰ|_#k. J„¬C_È° P


YD

1) 3 2) 1 3) 2 4) 4
„¦¬°<Œxß ‹¬=¶# „¬i=¶}O Q® 64 z#ß „¦¬°<Œ°Qê
,H

2. ãH÷Ok „¬@=ò…Õx "³ò`ÇëO †Çüx\˜ „¦¬°#=ò ‹¬OY¼ Hˋ² qƒ’lOKŒ~¡°.


T
AM

(A.P. S.I.Mains -2016) 4. XH› L„¬i`ǁO =¶ã`Ç"°Í ~¡OQ®° "͆°Ç |_# z#ß „¦°¬ #=ò
H

‹¬OY¼.
M
AM
IG

1) 8 2) 24 3) 16 4) 32

5. P䛽„¬KÇó ~¡OQ®° "͆ǰ|_# L„¬i`Džìxß H›h‹¬O XH›ø>ÿØ<Œ


KH

H›ey# z#ß „¦¬°#=ò ‹¬OY¼


R

1) 8 2) 56 3) 24 4) 32

1) 8 2) 9 3) 12 4) 15
6. =ü_È ° ~¡ O Q® ° #° H› e y# L„¬ i `Œ° Q®  z#ß
3. ãH÷Ok „¬\ì…Õ XöH‡zH› †³òH›ø =ü_È° ƒ’Oy=°° I, „¦¬°#=ò ‹¬OY¼
II, III W=Þ|_#q. JOïH 1 Q® =òMìxH÷ Z^ΰ~¡°Qê 1) 8 2) 16 3) 24 4) 4

L#ß =òY=ò̄á LO_Í JOïH Uk? 7. XH› =òY=ò̄á P䛽„¬KÇó~¡OQ®°, ^¥xH÷ P‹¬#ß=òQê #°#ß
(A.P. S.I.Mains -2016)
XH› =òY=ò̄á #°„¬ô …è^¥ Z~¡°„¬ô ~¡OQ®° H›ey, ï~O_È°
L„¬ i `Ç  =ò° =¶ã`Ç " Í ° ~¡ O Q® ° "Í † Ç ° |_ # z#ß
„¦¬°#=ò ‹¬OY¼.
1) 28 2) 24 3) 16 4) 8

8. U L„¬i`ǁŌá<Œ ~¡OQ®°"͆ǰx z#ß „¦¬°#=ò ‹¬OY¼


1) 0 2) 4 3) 8 4) 16
1) 6 2) 5 3) 4 4) 3

REASONING ABILITY (NON-VERBAL) ○ ○ ○ ○ ○ ○


268
○ ○ ○ ○ ○ ○ ○ ○ ○ ○ ○ ○ ○ ○ ○ ○ ○ ○ ○
Study Material
; (9 - 10 ㄬ‰×߁°) XöH ‡zH›ä›½ K³Ok# =ü_È°

ƒ’Oy=°#° KǶ„¬ô „¬\쁰 D ãH÷O^Î KǶ„¬|_#q.

(A) (B) (C) (D)


1) D 2) C 3) B 4) A
15. D ãH÷ O ^Î W=Þ|_ # Xö H ‡zH› ä › ½ K³ O k# qq^ Î
9. 'OÑ Q®°~¡°ë䛽 Ja=òYOQê =ôO_È° Q®°~¡°ë ƒ’Oy=° #°O_ 4䛽 Z^ΰ~¡°Qê =ôO_Í ‹¬OY¼ Uk?
1) → 2) þ 3) 4) u (S.I. - 2008)

10. KÇ°H›ø ( .) Q®°~¡°ë䛽 Ja=òYOQê =ôO_È° Q®°~¡°ë.


1) þ → 2) O 3) 4)

; (11 - 12 ㄬ‰×߁°) XöH ‡zH›ä›½ K³Ok# <Œ°Q®° 1) 2 2) 3 3) 5 4) 6

Y
ƒ’Oy=°#° KǶ„¬ô „¬\쁰 D ãH÷O^Î KǶ„¬|_#q. 16. D ãH÷O^Î KǶ„¬|_# „¬\ì #°O_ 3䛽 Ja=òYOQê

M
=ôO_È° JOïH Uk? (Jxß „¬\쁰 XöH ‡zH›ä›½ K³Ok#
qq^Î ƒ’Oy=°°)

E
(S.I. - 2012)

D
   

9
(i) (ii) (iii) (iv)

59
11. Ì„á „¬\ì #°O_ 'OÑ Q®°~¡°ë J_È°Q®° ƒìQ®O…Õ =ô#߄¬C_È°        

CA
95
‡zH› Ì„á ƒìQ®O…Õ =ôO_È° Q®°~¡°ë
T

50
1) 5 2) 2 3) 4 4) 1

17. XH› „¦¬°#=ò#䛽 Jxß "³á„¬ô…ì „¬‹¬°„¬ô~¡OQ®° "͋², ^¥xx


1) 2) 3) 4)

98
A
12. Ì„á „¬\ì #°O_ '«Ñ Q®°~¡°ë䛽 Ja=òYOQê =ôO_È° Q®°~¡°ë
27 z#ß „¦¬°#=ò°Qê Hˋ²#„¬ô_È°, XH› =òY=ò#䛽
:8
1) 2)
T O 3) 4)
=¶ã`Ç"Í° ~¡OQ®° H›ey†ÇòO_È° „¦¬°#=ò ‹¬OY¼
E
h

13. XH› ‡zH› †³òH›ø ï~O_È° ‹²Ö`Ç°#° D ãH÷Ok „¬\쁅Õ


.P

(Constables - 2012)
IC

KǶ„¬_ÈO [iy#k. J_È°Q®° ƒìQ®O…Õ 4 =ô#߄¬ô_È° ̄á


AD

1) 12 2) 3 3) 9 4) 6

ƒìQ®O…Õ =ôO_È° ‹¬OY¼ (S.I. - 2008)


18. Xö H ‡zH› †³ ò H› ø =ü_È ° ƒ ’ O y=°° D ãH÷ O ^Î
AB
O

1) 5
KǶ„¬|_¨Û~ò. J~ò# 2 KÇ°H›øä›½ Ja=òYOQê =ôO_È°
ER
CH

1
2) 3
1
2 6 =òYŌá =ôO_È° KÇ°H›ø ‹¬OY¼ (S.I. - 2012)
YD

3) 1 5
3
,H

4) 2 (i) (ii)
T

14. XH› „¦ ¬ ° <ŒHê~¡ „ ¬ ô k"³ ° à †³ ò H› ø Ja = òY=ò


AM
H

‡~¡ÅÞ=òÌ„á L#ß KÇ°H›ø "³ò`ÇëO ‹¬OY¼ Z„¬C_ȶ 7 (i) (ii) (iii)


M

J†Í°¼@°Á#ß D ãH÷Ok "Œ\÷…Õ ‹¬i†³Ø°#k. (S.I. - 2008)


AM
IG

1) 4 2) 5 3) 3 4) 6

1234567890123456789012
12345678901234567890123456789012123456789012345678901234567890121234567890123456789012345678901212345678901234567890123
12345678901234567890123
KH

q=~¡}ì`ÇàH› ["Œ|°°
12345678901234567890123456789012123456789012345678901234567890121234567890123456789012345678901212345678901234567890123
1234567890123456789012 12345678901234567890123
1234567890123456789012
12345678901234567890123456789012123456789012345678901234567890121234567890123456789012345678901212345678901234567890123
1234567890123456789012 12345678901234567890123
12345678901234567890123
12345678901234567890123456789012123456789012345678901234567890121234567890123456789012345678901212345678901234567890123
1234567890123456789012 PREVIOUS BITS : 12345678901234567890123
12345678901234567890123456789012123456789012345678901234567890121234567890123456789012345678901212345678901234567890123
R

1234567890123456789012 12345678901234567890123
12345678901234567890123456789012123456789012345678901234567890121234567890123456789012345678901212345678901234567890123
1234567890123456789012 12345678901234567890123
12345678901234567890123456789012123456789012345678901234567890121234567890123456789012345678901212345678901234567890123

1. (4) W=Þ|_# 1, 2 KÇ°H›ø° Common Qê L<Œß~ò. « (4 þ 1) { (4 þ 2) { (1 þ 3)

Hê=ô# g\ ÷ ㄬHê~¡O KǶÀ‹ë '5Ñ H÷ Z^ΰ~¡°Qê 4 KÇ°H›ø° « 4 { 8 { 3 « 15

LO\ì~ò Jx `³°‹¬°ëOk.
3. (4) I, II = ‡zH›#° „¬ijeÀ‹ë 2, 4° L=°à_ ‹¬OY¼°.

2. (4) 4 x°=ô =~¡ ° ‹¬  …Õx ㄬ u ^¥x…Õ#° 1 „¦ ¬ ° #=ò Hê=ô# 3H÷ Z^ΰ\ ÷ ƒ’°[Ōá LO>è ‹¬OY¼ '6Ñ. Hê=ô#

H›e¾ L#ßk. J^Íq^ÎOQê 4 x°=ô =~¡°‹¬…Õ nxß|\ ÷“ III ‡zH›#° „¬ijeOz# 1H÷ ㄬH›ø<Í LO_È°

ㄬu^¥x…Õ#° '2Ñ „¦¬°#=ò° H›e¾ =°i†Çò 1 ‹¬OY¼ 2, ãH÷O^Î LO_È° ‹¬OY¼ '4Ñ.

 
=~¡°‹¬…Õ ㄬu^¥x…Õ#° 3 „¦¬°#=ò° H›e¾ L<Œß~ò.
(4 - 8) z#ß ƒ’°[O †³òH›ø „¦¬°#„¬i=¶}O «
 «

"³ò`Çë=ò „¦¬°#=ò ‹¬OY¼
« 4 ̋O.g°.

REASONING ABILITY (NON-VERBAL) ○ ○ ○ ○ ○ ○


269
○ ○ ○ ○ ○ ○ ○ ○ ○ ○ ○ ○ ○ ○ ○ ○ ○ ○ ○
Study Material
4. (2) XH› L„¬ i `Ç  O =¶ã`Ç " Í ° ~¡ O Q® ° "Í † Ç ° |_ # z#ß 13. (3) „¬@O I =°i†Çò II #° „¬ijeOz# 1䛽 ㄬH›ø#

„¦¬°#=ò ‹¬OY¼ « 4 þ 6 (ƒìQꁰ) « 24


Q® JOïH° 2, 3, 5, 6. q°ye#k 4.

5. (4) Hê=‹²# „¦¬°#=ò ‹¬OY¼ « 4 þ 8 « 32


Hê=ô# 1 Z^ΰ~¡°Qê 4 =ôO@°Ok.

6. (4) =ü_È° ~¡OQ®°° H›e¾#q ï~O_È° =~¡°‹¬…Õ LO_È°#°.

J#Qê J_È°Q®°ƒìQ®O…Õ 4 =ô#߄¬C_È° Ì„á ƒìQ®O…Õ 1


Hê=ô# Hê=‹²# „¦¬°#=ò ‹¬OY¼ « 4 þ 2 « 8

=ôO@°Ok.
7. (3) J@°=O\ ÷ Hê=‹²# „¦¬°#=ò° '4Ñ LO\ì~ò.

14. (2) „¬@O (C) #° „¬ijeOz#


Hê=ô# Hê=‹²# „¦¬°#=ò ‹¬OY¼ « 4 þ 6 « 16

8. (3) ㄬ u L„¬ i `Ç  Ōá U^Ë XH› " ³ á „ ¬ ô U^Ë XH› ~¡ O Q® °


5 { 2 « 7

„¬î‹² LO@°Ok. Hê=ô# 4 { 3 « 7

Hê=‹²# „¦¬°#=ò ‹¬OY¼ « 8 1 { 6 « 7 J†Í°¼ J=Hê‰×O =ô#ßk.


3

Y
« (4 - 2)
15. (1) I =°i†Çò II ƒ’Oy=° #°O_ 4䛽 ㄬH›ø# Q® JOïH°
3

M
« 2
1, 3, 5, 6. q°ye#k 2.

« 8

E
4䛽 Z^ΰ~¡°Qê =ôO_È° JOïH 2.

9. (4) „¬@O (X) =°i†Çò (Z)  #°O_, 'O' Q®°~¡°ë䛽 ㄬH›ø#


16. (3) ƒ’Oy=° =°i†Çò  #°O_ 3䛽 ㄬH›ø#Q®

D
III IV

9
., u.

59
Q® Q®°~¡°ë° , →, =°i†Çò þ. q°ye#k
JOïH° 1, 2, 5, 6. q°ye#k 4.

CA
95
Hê=ô# 'O' Q®°~¡°ë䛽 Ja=òYOQê Q® Q®°~¡°ë u. ∴ 3䛽 Ja=òYOQê =ôO_È° JOïH 4.

50
10. (1) „¬@O (Y) =°i†Çò (Z)  #°O_, þ Q®°~¡°ë䛽 ㄬH›ø# 17. (4) „¬ \ ìxß Q® = °xOz#, Jxß =òMìÌ„á =°^ Î ¼ …ÕQ® 

98
A
u, →. ..
:8
Q® Q®°~¡°ë° , O, =°i†Çò q°yek XH›ø „¦¬°#O =¶ã`Ç"Í° XH›"³á„¬ô ~¡OQ®°#° H›ey
E

..
h

Hê=ô# þ Q®°~¡°ë䛽 Z^ΰ~¡°Qê Q® Q®°~¡°ë †ÇòO@°Ok. Hê=ô# XH› =òY=ò#䛽 =¶ã`Ç"Í° ~¡OQ®°
.P
IC

J#Qê . Q®°~¡°ë䛽 Ja=òYOQê =ôO_È° Q®°~¡°ë þ. H›ey†ÇòO_È° „¦¬°#=ò ‹¬OY¼ « 6.


AD

11. (2) „¬@O (i) =°i†Çò „¬@O (ii)  #°O_ 'O' Q®°~¡°ë䛽
AB
O

ㄬH›ø# Q® Q®°~¡°ë° ,


T , , «. q°ye#k
ER
CH

.
YD

∴ 'O' Q®°~¡°ë J_È°Q®°ƒìQ®O…Õ =ô#ß Ì„áƒìQ®O…Õ =ôO_È°


,H

Q®°~¡°ë .
T
AM

12. (3) „¬@O (ii) =°i†Çò „¬@O (iii) #° Q®=°xOz# '«Ñ
(2)
H

18. (ii) =°i†Ç ò (iii) ‹² Ö ` Ç ° #° Q® = °xOz#, ï ~ O_È °


M

Q®°~¡°ë䛽 ㄬH›ø# Q® Q®°~¡°ë° , , , °.

T
AM

KÇ°H›øä›½ ㄬH›ø# Q® KÇ°H›ø ‹¬OY¼ 1, 3, 4, 6.


IG

q°ye#k .

T q°ye#k 5.
KH

Hê=ô# '«Ñ Q®°~¡°ë䛽 Z^ΰ~¡°Qê Q® Q®°~¡°ë .


R

REASONING ABILITY (NON-VERBAL) ○ ○ ○ ○ ○ ○


270
○ ○ ○ ○ ○ ○ ○ ○ ○ ○ ○ ○ ○ ○ ○ ○ ○ ○ ○
Study Material
2016 - KŒ„¬“~Ÿ "ŒsQê =zó# ㄬ‰×߁°
‹¬OMì¼ ã‰õ}÷ (Number Series) 11.       
       
1. 1, 3, 4, 8, 15, 27, ---- J#°ãH›=°O…Õx `Ç~¡°"Œ`Ç
(TS (SI Prelims) - 2016)
‹¬OY¼ (TS (SI Prelims) - 2016)
1) 16 2) 8 3) 10 4) 2
1) 37 2) 44 3) 50 4) 55

2. 5, 11, 28, 53, -----, 175, 296.
(TS (SI Prelims) - 2016) 12.   (TS (SI Prelims) - 2016)

Y
1) 89 2) 123 3) 126 4) 152 
3. 7, 6, 11, 13, 15, 20, 19, 27, 23,----

M
1) 26 2) 28 3) 30 4) 32

(TS (SI Prelims) - 2016)




E
1) 27 2) 34 3) 36 4) 37

4. 4, 10, 18, 28, 40, ----. 70, 88. 13.    (TS (SI Prelims) - 2016)

D
9
59
(TS (SI Prelims) - 2016)

CA
95
1) 36 2) 44 3) 54 4) 64
1) 2 2) 3/2 3) 1/2 4) 1

5. 21, ----, 55, 78, 105, 136

50
98 a#ß „¬sH›Æ
A
(TS (SI Prelims) - 2016)
:8
1) 36 2) 38 3) 42 4) 45
(Odd man Out I Classification)
E

6. 3, 7, 13, 21, 31, ----, 57, 73.


h
.P

‹¬¶KÇ# : 1-6 =~¡ä›½ ㄬ‰×߁…Õ ‹¬i‡éx ‹¬OY¼x Q®°iëOKÇ°=ò.


IC

(TS (SI Prelims) - 2016)


AD

1) 39 2) 43 3) 49 4) 53 14. 1) 282 2) 364 3) 4168 4) 6248


AB
O

7. 24, 60, 120, 210, ----, 504, 720. (TS (SI Prelims) - 2016)

15.
ER
CH

(TS (SI Prelims) - 2016) 1) 1285 2) 2213 3) 3074 4) 4172

(TS (SI Prelims) - 2016)


YD

1) 226 2) 306 3) 336 4) 420

8. 120, 99, 80, 63, 48, ? J<Í J#°ãH›=°O…Õ …Õ„²Oz# 16. 1) 1339 2) 2665 3) 3991 4) 4223
,H

‹¬OY¼#° H›#°Qù#°=ò (TS (SI Prelims) - 2016) (TS (SI Prelims) - 2016)
T
AM

1) 35 2) 38 3) 39 4) 40
17. 1) 11 2) 246 3) 327 4) 464
H
M

9. 589654237, 89654237, 8965423, 965423, ? J<Í (TS (SI Prelims) - 2016)


AM

18.
IG

J#°ãH›=°O…Õx `Ç~¡°"Œ`Ç „¬^ÎO H›#°HËøO_. 1) 5 2) 12 3) 25 4) 36


KH

(TS (SI Prelims) - 2016)


(TS (SI Prelims) - 2016)
19.
R

1) 81 2) 441 3) 529 4) 613


1) 96542 2) 65423 3) 96543 4) 65234

(TS (SI Prelims) - 2016)

q°‹²žOQ· #O|~Ÿ (Missing Number) k‰§`ÇàH› „¬sH›Æ


ãH÷Ok Wzó# 1- 5 ㄬ‰×߁ „¬@=ò…Õ ‹¬OY¼° XH› (DIRECTION SENSE TEST)
ãH›=°„¬^u
œÎ …Õ L<Œß~ò? ̄\÷#
“ KË@ =KÍó ‹¬OY¼ H›#°HËøO_. 20. XH› =¼H÷ë D‰§#¼ k‰×#° 40 g°. ^ζ~¡=ò #_z, `Ç#
10.        Z_È=° "³á„¬ô 90° uiy 40 g°. ^ζ~¡=ò #_È°ªë_È°.
        J„¬C_È° ^ÎH÷Æ} k‰×#°   g°. ^ζ~¡=ò #_È°ªë_È°.
#_ÈH› ㇐~¡OaOz# aO^ΰ=ô. #_ÈH› „¬îiëK͋²# aO^ΰ=ô
(TS (SI Prelims) - 2016)
=°^Î¼ ^ζ~¡=ò (g°.…Õ) (TS (SI Prelims) - 2016)
1) 9 2) 8 3) 7 4) 6
1) 0 2) 60 3)   4)  
REASONING ABILITY (NON-VERBAL) ○ ○ ○ ○ ○ ○
271
○ ○ ○ ○ ○ ○ ○ ○ ○ ○ ○ ○ ○ ○ ○ ○ ○ ○ ○
Study Material
21. XH› =¼H÷ë ãH÷O^Î Wzó# k䛽ø…Õ #_È°ªë_È°. (1) `Ƕ~¡°æ 2. 5 Q®°~¡° q^¥¼~¡°Ö° =ü_È° ƒì+¬…Õ Un 䛀_¨

k‰×…Õ 60g°   (2) `Ç~¡°"Œ`Ç ^ÎH÷Æ} k‰×…Õ 80 g°  (3) KÇ^Î"Œ#°HË=_ÈO …è^ΰ.

PÌ„á „¬_È=°~¡ k‰×…Õ 120 g°  (4) `Ç^ΰ„¬i L`Çë~¡ k‰×…Õ 3. 4 Q®°~¡° q^¥¼~¡°Ö° =ü_È° ƒì+¬°

30 g°. =°i†Çò (5) J#O`Ç~¡O `Ƕ~¡°æ k‰×…Õ 60 g°. KÇ^ΰ=ô`Œ=°#°ä›½O@°<Œß~¡°.

#_È°ªë_È°. J`Ç#° #_ÈH› ㇐~¡OaOz# aO^ΰ=ô,#_ÈH› 4. 7 Q®°~¡° q^¥¼~¡°Ö° ã̄¦Oz =¶ã`Ç"Í°

„¬îië K͋²# aO^ΰ=ô =°^Î¼ ^ζ~¡=ò (g°.…Õ) KÇ^Î"Œ#°ä›½O@°<Œß~¡°.

(TS (SI Prelims) - 2016) 1) 1 2) 2 3) 3 4) 4

1) 30 2) 50 3) 60 4) 120 26. ãH÷Ok "Œ\÷x [`Ç K͆ǰO_. (TS (SI Prelims) - 2016)
22. XH› =¼H÷ë `Ƕ~¡°æ k‰×…Õ 250g°. #_È°ªë~¡°. J„¬C_È° *ìa`Œ - 1 *ìa`Œ - 2
L`Çë~¡ k=…Õ 50g°. #_È°ªë_È°. J„¬C_È° „¬_È=°~¡ k‰×…Õ (ƒì+¬ ZxßH›°) (q^¥¼~¡°œ ‹¬OY¼)
300 g°. #_È°ªë_°È . J`Ç#° #_ÈH› ㇐~¡OaOz# aO^ΰ=ô. Z) ã̄¦Oz =°i†Çò ‹¬O‹¬ø $`ÇO 1) 2

Y
#_ÈH› „¬îiëK͋²# aO^ΰ=ô =°^Î¼ ^ζ~¡=ò (g°.…Õ) =¶ã`Ç"Í°

M
(TS (SI Prelims) - 2016)
a) ã̄¦Oz, ‹¬O‹¬ø $`ÇO =°i†Çò 2) 1

1) 30 2)  3)     4) `³°Q®° Jhß

E
23. `Ƕ~¡°æ䛽 Ja=òYOQê L#ß =¼H÷ë `Ƕ~¡°æ k䛽øQê 30 ‹²) ã̄¦Oz =°i†Çò `³°Q®° =¶ã`Ç"Í° 3) 4

D
9
g°@~¡°Á #_KŒH› ^ÎH÷Æ}O "³á„¬ô䛽 40 g°.° #_z P̄á

59
_) ‹¬O‹¬ø $`ÇO =°i†Çò `³°Q®° 4) 3

„¬_È=°~¡ "³á„¬ô 60 g°.° #_KŒ_È°. "³ò^Î\ ÷ aO^ΰ=ô


CA
95
=¶ã`Ç"Í°

#°O_ J`Ç_³O`Ç ^ζ~¡O…Õ L<Œß_È° (g°.…Õ)

50
Z a ‹² _

(TS (SI Prelims) - 2016)

98
A
1) 3 4 1 2

1) 50 2) 60 3) 70 4) 130
:8
2) 3 1 2 4
E

‹²\÷“OQ· Jö~O*˜"³°O\˜ >ÿ‹¹“


3) 4 3 2 1
h
.P
IC

4) 2 1 3 4
AD

(Sitting Arrangement Test) 27. ãH÷Ok ㄬ=KÇ<Œ#° KÇ^=


Î O_. (TS (SI Prelims) - 2016)
AB
O

1. 30 =°Ok q^¥¼~¡°Ö° 3 ƒì+¬…Õ H›h‹¬O


24. S^Î ° Q® ° ~¡ ° J=¶à~ò° ‡¦ é \Õ Hˋ¬ O XH› |Á Ì „á
ER

XH›ø^¥xïHá<Œ KÇ^Î"Œ#°ä›½O@°<Œß~¡°.
䛀~¡°ó<Œß~¡°. ~Œ}÷H÷ Z_È=° ㄬH›ø#, aO^ΰH÷ 䛽_ ㄬH›ø#
CH

2. 20 Q®°~¡° q^¥¼~¡°Ö° =ü_È° ƒì+¬…Õ XH›ø^¥xx


‹Ô=° LOk. ~Œ}÷H÷ 䛽_ ㄬH›ø# "Í°s LOk. ~Œ}÷H÷ "Í°sH÷
YD

=¶ã`Ç"Í° KÇ^Î"Œ#°ä›½O@°<Œß~¡°.
=°^Î¼# s\ì LOk. ‡¦é\Õ =°^Î¼…Õ L#ßk Z=~¡°?
,H

3. W^Î í ~ ¡ ° q^¥¼~¡ ° Ö  ° ‹¬ O ‹¬ ø $`Ç O =°i†Ç ò `³  °Q® °


T

(TS (SI Prelims) - 2016)


AM

ï~O_O\ ÷x =¶ã`Ç"Í° KÇ^Î"Œ#°ä›½O@°<Œß~¡°.


H

1) ~Œ}÷ 2) "Í°s 3) s\ì 4) aO^ΰ


‹¬i†³Ø°# ‹¬=¶^¥#=ò
M

"³<£ zã`Œ° (VENN DIAGRAMS)


AM
IG

1) 1 ‹¬ï~á#^Í Hêx 2 =°i†Çò 3 ‹¬iHê=ô


KH

ãH÷Ok ^Î`ŒëO‰×=ò KÇkq 1, 2, 3 ㄬ‰×߁䛽 ["Œa=ÞO_: 2) 1, 2 =°i†Çò 3 ‹¬ï~á#^Í


R

3) 1 =°i†Çò 2 ‹¬ï~á#"Í Hêx 3 ‹¬iHê^ΰ


"³ò`ÇëO 35 =°Ok q^¥¼~¡°Ö…Õ 16 =°Ok ã̄¦Oz, 16

4) 1 =°i†Çò 3 ‹¬ï~á#q Hêx 2 ‹¬iHê^ΰ.


=°Ok ‹¬O‹¬ø $`Ç=ò =°i†Çò 11 =°Ok `³°Q®°

KÇ^ΰ=ô`Œ=°#°ä›½O@°<Œß~¡°. 5 Q®°~¡° ã̄¦Oz =°i†Çò


["Œ|°°
`³°Q®° ï~O_ȶ KÇ^Î"Œh, gi…Õ =òQ®°¾~¡° ‹¬O‹¬ø $`ÇO

䛀_¨ KÇ^Î"Œh J#°ä›½O@°<Œß~¡°. 5 Q®°~¡° `³°Q®° 1) 3 2) 3 3) 2 4) 3 5) 1


=¶ã`Ç"Í° KÇ"Œx ƒìqÀ‹ë, 8 =°Ok ‹¬O‹¬ø $`ÇO =¶ã`Ç"Í° 6) 2 7) 3 8) 1 9) 1 10) 1
KÇ^Î"Œx J#°ä›½O@°<Œß~¡°.
11) 3 12) 1 13) 3 14) 1 15) 2
25. ãH÷Ok Wzó# ㄬ=KÇ<Œ…Õ Uk ‹¬iHê^ΰ? 16) 4 17) 2 18) 3 19) 4 20) 1
(TS (SI Prelims) - 2016)
21) 2 22) 4 23) 1 24) 1 25) 3
1. 10 =°Ok q^¥¼~¡°Ö° 3 ƒì+¬…Õ H›h‹¬O ï~O_O\ ÷h
26) 1 27) 3
KÇ^Î"Œ#°ä›½O@°<Œß~¡°.

REASONING ABILITY (NON-VERBAL) ○ ○ ○ ○ ○ ○


272
○ ○ ○ ○ ○ ○ ○ ○ ○ ○ ○ ○ ○ ○ ○ ○ ○ ○ ○
Study Material
~ŒRO…Õ J`Ç°¼#ß`Ç ‡¦¼H›b“KÍ
~¡¶‡ÚOkOKÇ|_#
''ï~á\˜KŒ~ò‹¹ ‹¬“_™"³°\©i†Ç°…˜ÑÑ

Hê=ež#"Œ~¡° ‹¬OㄬOkOKÇQ®~¡°.
RIGHT CHOICE ACADEMY
Khammam, Hyderabad.

Ph : 89850 95599

También podría gustarte